You are on page 1of 1485

MCQ MAY 2005, ‘VERSION LINSEY’, 05B1

1. 4-6 week old baby comes for a routine check up. Scrotal swelling is discovered. Swelling is soft,
transluminate. Testes are normal size after palpation. What is the management?

review at 1-2 years

elective surgery – refer

urgent surgical appointment

ultrasound

reassure parent

2. Which of the following is not associated with vomiting in infants

a) gastroesophageal reflux

a) pyloric stenosis

a) congenital adrenal hyperplasia

a) intersusseption

a) galactosaemia

3. A Pregnant lady - 26 weeks, has a dilated cervix 2-3cm and is experiencing regular contractions.
Which of the following will you do except

a) cervical suture

a) give tocolytic drug

a) give celestone

a) give salbutimol
4. Female with acne and hirsutism. What is the treatment (recall question!)

a) anti androgens

a) isotretinion

a) tetracycline

a) OCP

5. Picture of scalp with a round crusty hairless lesion with some bloody patches. What is the
diagnosis

a) psorisis

a) tinea

a) eczema

a) alopecia areata

a)

6. New born baby with 1 week history of yellow discharge from umbilical stump. A red fleshy lump
can be seen on examination. What is the treatment?

a) reassure parent

a) povidone iodine (Betadine)

a) silver nitrate

a) ultrasound

a) surgical removal

7. Patient with black pigmented skin lesion. What would be the most likely association to suggest
malignancy
a) ulceration

a) greater than 5cm

a) located on upper back

a) 2 or more of the same lesion else where on the body

a) hair seen growing out of it

8. Picture of groin with lump shown. Patient came with complaint of swelling in groin for last 1-2
years. It is growing slowly and is soft and lateral to pubic tubercle. What is the diagnosis

a) female hernia

a) lymph nodes

a) lipoma

a) inguinal hernia

a) pre-vascular hernia

9. Antenatal checkup chart given. The last checkup was at 28 with normal fundal height for dates.
At 32 weeks the fundal height is still the same as 28 weeks. Fetal heart rate is normal. What should
you do?

a) Instruct patient to come back in one week

a) Perform a CTG

a) Check to see if patient is developing preeclampsia

a)

10. How will you detect increase ICP


a) rise in BP

a) papilloedema

a) papillary reflex

a) level of consciousness

11. A boxer comes to hospital after a fight in which he was knocked out. His right pupil is dilated
but otherwise he feels ok and everything else is normal. What would you do?

a) give manitol IV

a) CT head

a)

12. A 26 week pregnant lady has developed facial palsy. What do you advise?

a) surgical decompression

a) Prednisolone 75mg for 2 weeks

a) Hydrochlorothiazide

a) Advise it will resolve in 6-8 weeks

a)

13 How do you calculate the diameter of the endotracheal tube for paediatric intubation?

a) (age + 4) /4

a) (weight+4) /4

a) (weight + 4) /2

a) (height + 4)/4
a) (height + 4) /2

14. A 14 year old pregnant girl comes to you to request a termination. Her boyfriend who is 21 left
her when she discovered she was pregnant. She is living away from home. What do you do?

a) ring parents for consent

a) ring child care protection agency

a) inform police because of difference in age of male

a) take her informed consent and terminate pregnancy

15. A photo of skin tags on eyelids of a small child. What is the diagnosis?

a) Molluscum contagiousum

a) Warts

a) Skin tags

a)

16. A 15 year old with a perforated appendix comes to hospital with his 18 year old sister. Parents
at home. What do you do to obtain consent?

a) obtain from sister

a) ring parents

a) get informed consent from 2 doctors

a) take advice from witness/another doctor

17. Breast enlargement in an 18 month old girl. The most likely cause? (recall qu)
a) idiopathic premature theliarche

a) precoscious puberty

a) adrenal hyperplasia

a)

18. Staghorn calculi (recall qu) are found in all of the following except:

a) UTI

a) Immobilization

a) Hyperuricaemia

a) Hyperparathyroidism

19. What is a characteristic of fibrosing alveolitis

a) reduced FEV

a) reduced FEV/VC

a) increased CO2 concentration

20. Tension pneumothorax (recall qu). What is the first thing you do?

a) endotracheal intubation

a) insert large bore needle into 2nd intercostal space

a) insert tube into 4th intercostal space


a) tracheostomy

a)

21. Photo of buttock (recall qu) showing a large red area with elevated margins. Spread to both
buttocks. What is the treatment?

a) tar

a) corticosteroids

a) UV

a) Imidazol

a) Emoliant

22. Irish girl living in Sydney for 10 months. Has had diarrhoea and weight loss for 10 months.
Blood test shows low folate and IGA, iron and B12 normal, enomisial and anti gladyn antibodies
negative. What do you do?

a) colonoscopy biopsy

a) small bowel biopsy

a) it’s not caeliac diseas because of negative findings

a)

23. 3 year old with 10 brown spots on body (recall qu) He could have all the following associated
except

a) hypothyroidisn

a) mental retardation

a) deafness
a) hypertension

24. An Aboriginal woman 57 years old. Lives in a remote community. Has never been to a doctor
before. She has swelling in the legs plus difficultly breathing. Bilateral basal creps. Non smoker.
Drinks a lot of alcohol. You do all of the following except

a) chest Xray

a) ECG

a) Echocardiogram

a) CT pulmonary angiogram

25. An Aboriginal woman 57 years old. Lives in a remote community. Has never been to a doctor
before. She has swelling in the legs plus difficultly breathing. Bilateral basal creps. Non smoker.
Drinks a lot of alcohol. What is the diagnosis?

a) heart failure

a) alcoholic cardiomyopathy

26. A 50 year old woman who had a CT scan abdomen. CT shows 1.5cm mass on the R) adrenal
gland. Non symptomatic, no electrolyte imbalance. No hormonal imbalance. What do you do
next?

a) do MRI

a) PET scan

a) Biopsy

a) Repeat CT in 6 months

a) Ultrasound
27. A 19 year old woman with pelvic pain. Ultrasound shows a 4x6cm cyst on her ovary. What do
you do?

a) reassure

a) repeat ultrasound in 6 weeks

a) laparotomy

a) laparoscopy

28. Which one of the following is the least effective contraceptive?

a) condom

a) diaphram & spermicidal cream

a) progesterone only pill

a) IUD

29. What is the most common way of transmitting Hepatitis B in South East Asia? (recall qu)

a) perinatal

a) sexual contact

a) blood transfusion

30. In which of the following diseases does a person have a distorted perception of their body
except

a) amputation

a) acromegally
a) anorexia nervosa

a) hyperchondria

31. A man attends hospital after being pinned to wall by forklift at work. He has epigastric
discomfort and mild pain. He is haemodynamically stable. What is the indication for urgent
laparascopic exploration

a) liver heamatoma

a) splenic heamatoma

a) heamorrhagic fluid in peritoneum

a) retroperitonial gas

32. Multiple red inflamed lesions with discharege in auxilla of left groin. What is the diagnosis

a) pyelenodeal sinus

a) infected epidermoid cyst

a) hydroadenitis

33. A 42 weeks old baby weighs 2.5kg. What is the least likely association?

a) mother with diabetes

a) chance of hypoglyceamia is low

a) length, head circumference and weight not same percentile


34. A lady with a hisotry of psychosis says she needs to contact you after hours. What should you
do?

a) give your home number

a) give the number of emergency telephone counseling

a) give number of nearest psychiatric hospital

a) give number of general hospital

25. A 17 year old girl who had a seizure with loss of consciousness for 1 minute. She says that for 1
year she has been experiencing ‘jerks’ in her hands. What is the treatment?

a) phenitonin

a) carbamazepine

a) gabapentin

a) itho suxximide

a) sodium valporate

36. A 19 year old woman who has been to you before for sleeping pills. Has a past history of
occasional drug use. Mother who is also your patient has called to tell you the daughter is skipping
school. Mother wants to know what the daughter came to see you about. What should you do?

a) tell her what the daughter said

a) tell her that her daughter is depressed

a) tell her that her daughter is a drug addict

a) interview the daughter

a) interview the mother and the daughter


37. All are typical findings in acute asthma in a young adult except:

a) pneumothorax

a) right heart failure

a) atelextasis

38. An x-ray of 67 year old woman with abdominal pain in left IF and some loose occasional
diarrhoea. On x-ray there are some stones. What is most likely?

a) renal stones

a) gall stone ileous

a) appendicular obstruction by faecolith

a) previous Hx of TB

39. A couple who wanted advice regarding contraception using rhythm method. Her period in one
year varied between 26 and 29 days. What stage do you advise abstinence? (recall qu)

a) 8-19 days

a) 10-19 days

a) 12-19 days

a) 6-16 days

a) 4-21 days

40. Without a previous history of any operation what is the most common cause of small bowel
obstruction?
a) hiatus hernia

a) groin hernia

a) sigmoid volvulos

a) gall stone illeus

41. A baby born at term. Normal appearance. 6 hours after birth develops tachypnea and cyanosis
and was corrected with 45% O2. What is the diagnosis?

a) transient tachypnia of infant

a) merconium aspiration

a) hyaline membrane disease

42. A 65 year old lady with rheumatoid arthritis has developed a swollen and painful knee and
fever. Pulse 120. She also has stiffness of hands in the morning. What do you do?

a) take blood and cynovial fluid for tests

a) take fluid from knee, test for gout and give allopurinol

a) NSAIDs

a) paracetamol

43. CT scan at L1 level. Patient has had epigastirc pain 2 months ago. What is the diagnosis

a) pancreatic pseudocyst

a) hyadatic cyst

a) splenic cyst

a) gastric leiomyome
a) gastric obstruction

44. Pregnant lady previously has caesarian and wants to have a vaginal deliver. What will you say
about risk of rupture and wound dehiscence?

a) 0.5% risk of rupture

a) increased risk if continues after 41 weeks

a) increased risk if labour induced

a) increased if difficult or laborious labour

45. Male with heart failure and AF. What is the risk of him having a stroke? On echocardiogram
you see some shadow in atrium

a) 0.5% /year increased risk of stroke

a) 25% increased risk of stroke

a) treat with warfrin

a) treat with aspirin

46. A 60 year old Pacific Islander who is a heavy smoker. Has bloating and oedema of ankles. What
is the likely diagnosis?

a) lung cancer with vena cava superior obstruction

a) ischaemic heart disease

a) pulmonary emboli with hepatic metastisis

47. Coagulation deficit in obstetrics in all except:


a) prolonged dead fetus in utero

a) protein C deficit

a) amniotic fluid embolism

a) placenta previa

48. A menopausal lady comes for HRT to treat hot flushes. No other medical history. You do all of
the following except:

a) FSH

a) Oestradiol

a) Mammogram if not done in last 2 years

a) Lipid profile

49. Patient with right sides sciatica and lower limb pain. He was hospitalized and given narcotic
analgesia and bed rest. After 8 hours he cannot pass urine. What is the reason?

a) narcotic analgesic side effect

a) prostetic bladder outflow obstruction

a) central cord disc prolapse

50. A 12 year old girl with anorexia. Has lost 5kg over last few months. She has abdominal pain and
anal pruritis. The diagnosis is:
a) giardia

a) Chrohns disease

a) Amoebiec parasite

51. An 87 year old man that you are going to do a colonic operation on. His daughter has had a 3
hour meeting with the anaesthetist and she is very anxious and agitated about the operation. Now
she wants to see you to discuss her father’s surgery. What do you do?

a) speak to her

a) choose not to see her

a) see her only if she has power of attorney

a) see her only with her father present

52. Lady on OCP has missed one pill. Had mid cycle intercourse and took morning after pill post
intercourse and continues with daily of usual OCP. She is now experiencing spotting after 3 days.
What should you do?

a) reassure and tell to continue taking OCP

a) pregnancy test HCG

a) tell her to stop, have a normal periods and then continue taking OCP

53. What is true except:

a) gestational sac can be seen after 51/2 weeks of last menstruation

a) at 7 weeks head, trunk and limbs can be identified


a) at 8 weeks fetal heart can be seen on ultrasound

a) fetal bradycardia indicates poor prognosis

a) Ultrasound at 6-11 weeks can be correct plus minus three days

54. Inappropriate use of diuretics frequent in the following:

a) football players

a) bulimia

a) anorexia

a) obese people

55. Shown a fluid balalnce chart for a 5 days post op for rectal cancer man. He has received 2.5L IV
fluids fluids. His output was colostomy 1500ml and urine 2.3L. Which of the following may be the
cause?

a) diabetes insipid

a) osmotic diuresis since operation

a) wrong figures in fluid chart

a) due to resolve of the ileus

a)

56. A 41 weeks gestation baby. Normal delivery of 6 hours duration. Baby weighed 3kg. While
delivering placenta the woman became suddenly shocked. What is the cause

a) pulmonary embolism

a) uterine inversion

a) amniotic fluid embolism


a) uterine rupture

57. Which CTG reading shows fetal hypoxia except (recall)

a) baseline 180 and variable moderate deceleration

a) baseline 110 and increased by 20 beats twice in 20 minutes

a)

58. In psychiatric evaluation which shows the least lack of insight and proper judgment?

a) altruism

a) denial

a) projection

a) splitting

59. Lady with nausea and sudden onset of abdominal pain. LMP 3 weeks ago. Tenderness in RIF.
What is the diagnosis? BP systolic 100 and Pulse 100

a) perforated appendix

a) hemorrhaging ovarian cyst

a) torsion of ovarian cyst

a) ruptured ectopic pregnancy

60. A 21 month old baby with occasional diarrhoea that contains undigested vegetables. Normal
development otherwise. What is diagnosis?
a) toddlers diarrhoea

a) giardia

a) galactosaemia

a) lactose/inulose deficiency

61. ADHD in children. Which of the following is most common:

a) develops frequently in 5-7 year olds

a) reading difficulties

62. Least likely to be primary hypothyroidism

a) 6 year old child with less bone age

a) 35 year old lady with menorrhagia

a) 70 year old lady with goiter

a) 10 year old boy with mental retardation

a) lady with anaemia that didn’t respond to folic acid, iron and B12

63. Elderly lady with large goiter. For which of the following would you advise her to have it
removed.

a) hoarse voice at end of day

a) difficultly breathing when lying down


64. 60 year old man attends with his wife. He has lost 10kg in last year. Light smoker for last 30
years. Lately he has lost interest in activities of daily living. On examination all is normal. What do
you think is the problem?

a) chronic fatigue syndrome

a) occult malignancy

65. Corticosteroids and psychosis

a) if no previous psychosis before commencement of Tx there is no increased risk of developing


psychosis

a) prior psychosis menas a high risk

a) low dose treatment can cause depression

a) delirium can occur after abrupt withdrawal

66. Naltrexone given for opiod poisoning is:

a) short acting

a) has good absorption orally

a) only efficient 3 days after heroine intoxication

67. 40 nulliperous woman with post coital bleeding. All of the following are possible except:

a) enometrial carcinoma

a) cervical polyp

a) submucous fibroid

a) intramural fibroid
68.CVS and amniocentesis except:

a) CVS done earlier than amniocentisis

a) Both good for detecting chromosomal abnormalities

a) Both good for spinal bifida

a) Risk of abortion higher in CVS

69. O negative Rh negative woman delivers an A negative Rh negative child who is jaundiced at 24
hours. What is the most likely cause of the jaundice?

a) ABO incompatibility

a) Breast milk jaundice

a) Rh incompatiblity

70. A lady who is 26 weeks pregnant. Fundal height normal at 26 weeks with Coombes test mild
positive. At 28 weeks fundal height is 38cm. What is the likely cause?

a) polyhydramnios

a) hydrops fetalis

a) diabetes

a) UTI

71. Mother Rh negative, baby Rh negative, Coombes test negative, Kleinhower test ok. What do
you do?

a) still give anti D at 72 hours


72. A 40 year old nurse threatening suicide. Has had previous episodes but no associated risk of
crisis. States she has a syringe in her bag and will kill herself. What is the diagnosis?

a) depression

a) bipolar disorder

a) borderline personality disorder

a) schizophrenia

73. Venous ulcer on leg. Treat with all of the following except:

a) topical antibiotics

a) enzymes for debridement

a) compression stocking

a) bed rest

74. An olive shaped abdominal mass in a 6 week old infant. Has non projectile, non bile stained
vomiting. What is the diagnosis

a) malrotation

a) volvolus

a) pyloric stenosis

75. All of the following drugs cause gynocomastia except:

a) digoxin
a) simetidine

a) labetolol

a) alphmethyldopa

76. All of the following drugs cause gynocomastia except

a) digoxin

a) simetidine

a) fruesemide

a) alphmethyldopa

77. Endometrial cancer. All of the following can cause except

a) obesity

a) raloxifin

a) oestragen therapy

a)

78. Which of the following can cause osteoporosis except:

a) early menopause

a) smoking

a) chemotherapy in a 60 year old lady for cancer


79. A 35 year old woman with menorrhagia. Bleeding for 8 days and having to change tampon
every 2 hours. Mother had colonic cancer. Hb was 115. Is being treated with iron tablets. Should
you

a) do colonoscopy

a) hysteroscopy and D&C

a) give norethisterone

80. Treatment for cluster headache

a) methyseigite

a) sumotryptine

a) ergotimine

81. Lady with headache for 3 days. Located in temporal and frontal region. Otherwise well. Has
been off work for 3 days because of pain. What do you do?

a) CT

a) Give ergotimine

a) Reassure

82. A 40 year old man who is anxious about traveling to other places for business. Uncomfortable
outside his home. What is the likely diagnosis (recall qu)

a) social phobia

a) panic disorder with agoraphobia


83. 5 year old child with contaminated abrasion. Had vaccinations at 2 months and 4 months, but
missed the 6 month dose. What do you do?

a) give tetanus plus penicillin

a) give tetanus plus immunoglobulin plus penicillin

a) DTPa and immunoglobulin

a) DTPa and booster after 2 months

84. Radio opaque stone in renal pelvis, 2cm in diameter. What is the treatment?

a) ECLT

a) Advise the stone will pass

a) The stone being there will cause cancer

a) He has a urate calculi

85. Daughter brings in father who is confused. CT scan is given. What does it show (recall qu)

a) cerebral hemorrhage

a) cerebral infarct

a) cerebral tumor

86. For the past few days woman who has weakness in right hand. She can’t use it properly. Look
at CT scan. What is the diagnosis?
a) cerebral hemorrhage

a) cerebral infarct

a) cerebral tumor

87. Haemolytic anaemia (recall qu) what do you find?

a) increased lymphoblasts

a) target cells

a) reduced reticulocytes

88. Maculopapular rash in a 35 year old lady. 1 week prior had malaise, headache and fever. Do all
of the following screens except

a) HIV

a) EBV

a) Rubella

a) Varicella

89. Tonsil ulceration with exudates. Malaise, fever. Monospot test negative

a) measles

a) EBV

a) Rubella

90. Av 25 year old student with fever, malaise and headache for 2-3 weeks. Exudative tonsils.
Perianal soft lesions. Diagnosis?
a) Yaws

a) EBV

a) Secondary syphilis

a)

91. A 1 week old baby with neck stiffness, vomiting and fever. Lumbar puncture shows raised
protein. What do you give?

a) acyclovir

a) ceftriexone

a) vancomycin

92. A 2-3 week old baby with tachypnea and distress. Chest x-ray shows shadow on lower left

a) pneumonia

a) congenital diphramatic hernia

a)

93. Child with expiratory wheezing after exercise. Presents in Autumn. Bilateral expiratory
wheezes on auscultation. What is the diagnosis?

a) asthma

94. Child was at birthday party. Came to hospital with sudden onset of wheeze on one side. What
do you do?
a) inspiratory and expiratory x-ray

a) bronchoscopy

a) allergy test

a)

95. Which of the following has the most severe outcome for a child (recall)

a) with a bruise on the cheek

a) bluish discoloration on buttocks

a) diamond shaped lesion on scalp

a)

96. Patient delirious due to multiple factors - hepatic failure and alcohol withdrawal. What is the
least likely associated issue?

a) asterixes

a) unable to give consent

a) sundowning effect

a)

97. LFT elevated, billirubin elevated, ALP and gama GT high. ALT 600 and AST 500. Igg for Hepatitis
A and he drinks 60g alcohol daily. IGM core antigen positive. Diagnosis?
a) Hepatitis B

a) Cirrhosis for Hepatitis B

a) Acutem alcohol hepatitis

a) Acute Hepatitis A

98. Which of the following is true regarding haemocromatosis?

a) you don’t do venesection if the pacient is asymptomatic

a) 1: 50.000 Australians have the disease

a) The gene implicated is C28CY

99. A 4 yo girl wakes up every night complaining of cramps( not related to daily sports and
activities) and wants her parents to massage her legs, then falls asleep. Wof is not true?

a) she has abnormal calcium and electrolites

b) her parents used to do the same as kids

c) reasurre that she only wants attention

100. A 3 yo girl gets apnoeic and cyanosed and falls to the ground when upset or the parents are
not doing what she wants. What do you tell the parents?

a) reassure that it is a benign condition

b)

101. Lady with BMI 28 she drinks heavily and also has high cholesterol (some other asso features).
Mx-
a) Advice to loose weight.

a) Tell her to maintain a well balanced protein diet.

a) Tell her to stop drinking.

102.Pt with tremor, heartrate…, TSHlow(.2),T3 and T4 high. Mx

a) Necarmazole

a) b) Radioactive iodine.

a) c)Suppress with thyroxine

103. Pt had fever and neck pain for 2 days, came with slightly enlarged thyroid gland, tender. Mx

a) Aspirin and atenolol

104. School teacher anxious and at class keen to do everything perfectly. Goes over things of the
day in his mind at the end of the day Tries to stop doing it but can’t. Mx

a)SSRI

b)Antidepressants

105. All of the following will give increased pulmonary vascular evidence except

a)ASD

b)VSD

c)Anomalous pulmonary vein

d)Overriding of aorta.

106.Child 10 yrs cough for 2 weeks, fever. Bilateral hilar diffuse shadows. Likely organism

a) Bordetella
a) Mycoplasma

107.Woman after 3 days of laparotomic operation during shower became cyanosed, tachypnoec,
breathless. Dx

a) Myocardial Infarction.

b)Pulmonary embolism

c)Vasodilatation due to hot shower

108.Daughter lost father 6 months ago. Can’t get over it.Lost weight 6-10 kg. What will you
enquire about

a)Eating pattern

b)Suicidal ideas

c)Menstruation regularity

109.In senile dementia what pt most likely to forget last

a)Recent events

b)Vocubulary

c)People’s faces

d)Where they are (get lost)

110.Pt with aortic aneurysm repair after 3 days developed distention of abdomen(other features).
Cause

a)Hypokaelimia

b)Dissecting aortic aneurysm

111.Haemochromatosis indication for venesection except

a)Asymptomatic patient
b)Genetic pattern given

112.Iron deficiency anaemia, which is true

a)Ferritin increased

b)Premature babies more prone

113. Haemolytic anaemia what does not occur

a)Increased urine urobilinogen

b)Increased unconjugated bilirubin

c)Reticulocytosis

114.1 hr after difficult urethral instrumentation under GA pt developed sweating, pale, breathless.
Cause

a)Concealed hge

b)PE

c)Gram –ve bacteraemia

115.Before cardioversion to see progress what do you do

a)Adrenaline IV every 10 min

b)Put ECG monitor


116. Regarding IUCD use what is true except

a)Excellent form of contraception

b)Risk of ectopic pregnancy

117.Which of the following is true about depression presentation in a GP practice

a) It affects all age groups

a) Both sexes are affected equally

a) It is the commonest psychiatric disorder to present to a GP

118. X-ray of suprachondylar fracture(anthology book), pt’s limb is pulseless. First step of
management

119.Mammogram and USG of breast given what is it (Anthology book pg 104)

a) breast cyst

a)

120.Pt came from warzone area,what he might suffer from most likely

121.Man has fear of flying. Avoids flying and staying away from home. Dx

122.Most likely cause of wasting of small muscles of hand is due to which nerve lesion

a) ulnar

a) median

a) digital

a) radial
123.Pt can do planter flexion and inversion but dorsiflexion and eversion weakly. Knee jerk and
ankle jerk okay. Where is the lesion.

a)Common peroneal nerve

b) L4

c)L5

d)Sciatic nerve

124.Seamstress came with needle prick on finger. Can’t extend it. Dx

125.Child came acyanotic, when feeding cries. Systolic murmer at left sternal border, loud S2. apex
beat not shifted but tapping in 5th intercostals space. Dx

a)ASD

b)VSD

c)Disposition of great vessels.

126. Commonest form of epilepsy that occurs between ages 4-10years

a) Petit mal

a) Generalised tonic-clonic

a) Myoclonic

a) Infantile spasms

127.In Polycythaemia rubra vera is associated except

a)Anaemia persistent

b)Normal lifespan of RBC


c)Osmotic fragility

d)Persistent spherocyte

128. Photo of skin lesion (from Anthology of Medical Illnesses pg 396). Dx

a) melanoma

a) psoriasis

a) Bowen’s disease

a) SCC (invasive)

129.Girl wants to have umbilical ring but on examination splenomegaly. What to do.

130.Keratin scaling does not occur in which one

a)SCC

b)BCC

131.What is NOT true about Australian snake bites?

a) most are not poisonous

a) cranial nerves are usually affected

a) you should apply a torniquet immediately

a) observe for symptoms before administering antivenom

132.Lady can’t see outside in bright light. Dx

133. Lady with tremor at rest and when she reaches for phone but not when she looks at her
hand. Treatment

a) propanolol
a) carbidopa and l.dopa

a) benzehexol

134.Child with signs of pneumonia and in X-Ray consolidation of lower lobe. Rx

a)Penicillin

b)Erythromycin

c)Flucloxacillin

d)Roxithromycin

135.Paracetamol overdose.

136.Pt with left sided scrotal hydrocele growing fast, what is likely cause

a) left renal cancer

b) left testicular cancer

137. Unilateral nasal discharge foul smelling and child not allowing you to examine nose. What to
do.

138.Boy with knee pain, worse after exercise. Dx

139.Mother came with son 12yrs old who had limp. Dx

a) slipped femoral epiphysis

a) Perth’s disease

a) Osteomyelitis

a)
140.Child crying and vomiting due to intussucception.

a) surgery

a) air contrast enema

a) N/G tube

141.Common cause of bleeding from nipple in 50 Yr old woman.

142. Cause of toothpaste like discharge from nipple.

143. 80 Kg man came with 15% burn how much IVF needed?

a) 3L Hartmanns + 2L 5% D/W

a) 3L N/S + 2? 5% D/W

a) 3L Hartmanns + 2 L blood

a)

144.Man came with vomiting, eyes sunken, dehydrated what type of fluid does he need?

145. What is true about febrile convulsion.

a) recur in 3% of children

a) the majority will develop epilepsy

a) commonly occur between 6 months and 4 years

a) associated with focal seizures

a) last less than 15 minutes

146.Pt came in labour with PV bleeding about 1000 ml. How to know it was placenta praevia.
147.Infant with urine in bag culture 105. Next what to do?

148.Second commonest cause of death in Australia after cardiovascular disease.

a) motor vehicle accident

a) malignancy

a) respiratory illness

a)

149. Which tumour does not metastasise to brain.

a) breast

a) lung

a) kidney

a) melanoma

a) prostate

150.Lady with DVT and superficial thrombophlebitis taken medication but still it occurs. Reason.

151. An old man who has difficulty reading the telephone book. The Dx is

a) myopia

a) cataract

a) hypermytropia

a) presbyopia

152. A butcher presents with fever, myalgia, jaundice, maculopapular skin rash and
hepatosplenomegaly. What is the most likely Dx
a) leptospirosis

a) brucellosis

a) Q fever

153. WOF is true about iron deficiency anaemia in infancy

a) malabsorption is the commonest cause

a) peak time is around 2 months

a) blood loss is the commonest cause

154. What is the first sign of puberty inm

males?

a) increased penile shaft size

a) increased testicular size

a) increased breast size

a) presence of pubic hair

155. A young adult from Fiji presents with fatigue, ascites, hepatomegaly, increased JVP. What is
the most likely Dx

a) Tuberculosis pericarditis

156. Which of the following is not the first investigation of an infertile couple

a) semen analysis

a) karyotype

a) hormone profile

a) laparoscopy and hystersalpinogram


157. Methotrexate is effective in the treatment of ectopic pregnancy in all except

a) unruptured gestational sac

a) live fetus

a) HCG level low

a) The gestational sac is small on U/S

158. The best method for imaging bile ducts

a) CT

a) U/S

a) ERCP

a) Oral cholecystography

159. Most common manifestation of pyloric stenosis in adults

a) frequent vomiting

a) abdominal distension

a) one or two large vomits per day

160. The commonest form of cancer in non smokers

a) SCC

a) Small cell ca

a) Adenocarcinoma

161. WOF is not true about amniocentisis and chorionic villus sampling

a) both can detect chromosomal abnormalities

a) CVS is done at earlier gestation

a) Amnio is slightly less accurate


162. WOF does not get transmitted to baby during delivery

a) herpes

a) syphilis

a) B. Streptococci

a) CMV

163. WOF is less likely to be a manifestation of first degree hypothyroidism

a) an 8 year old child with retarded growth

a) a 65 year old woman with multi nodule goiter

a) a 35 year old woman with depression

a) a woman with menorrhagia

164. In WOF is anaerobic bacteria growth most likely

a) lung absess

a) brain absess

a) hip prosthesis

a) GI tract

165. WOF is not true about emphysema

a) decreased FEV

a) decreased VC

a) increased TLC

166. Management of a breast cyst

a) fine needle aspiration


a) biopsy

a) mammectomy

167. A lady immediately post partum develops shortness of breath and increased JVP

a) conjestive heart failure

a) PE

168. Drug of choice for myoclonic seizures

a) carbamazepine

a) sodium valproate

a) phenytoin

a) phenobarbitone

169. In a patient with a loss of dorsiflexion and eversion but has intact inversion, plantar flexion,
ankle and knew reflex. WOF nerves is injured

a) L4

a) L5

a) S1

a) S 2 or 3

170. WOF is contraindicated in a child with ALL in remission

a) MMR vaccination

a) DPT vacc

a) Polio vacc

171. An elderly man 10 days post op from a hemicolectomy collapses following a hot bath. Reason
for collapse
a) vasodilation due to hot bath

a) bleeding

a) anastomosis leak

172. In a patient who is agitated and depressed, with suicidal threats, appropriate management is

a) haloperidol

a) contact the police

a) try to convince him to take medication

a) involuntary admission

173. A pregnant lady delivers at 34 weeks gestation. The neonate develops respiratory distress.
What is the cause?

a) hyaline membrane disease

a) transient tachypneoea of the newborn

a) congenital pneumonia

174. A lady with avenous ulcer. WOF is not the appropriate management

a) elevation of the leg

a) sustemic antibiotics

a) local antibiotics

a) debridement

175. A group of people at a banquet present with GI symptoms within 6 hours. Some recover
spontaneously, others develop diarrhoea and are admitted to local hospital. WOF is the most likely
causative agent

a) salmonella

a) staph aureas
a) clotridium

a) E. Coli

176. WOF is not found in pulmonary embolis

a) synope

a) pleuritic pain

a) SOB

a) Bronchial breath sounds

177. In subacute bacterial endocarditis, prophylaxix antibiotics is appropriate in all except

a) appendisectomy

a) cystoscopy

a) reduction of simple #

a) dental filling

178. WOF is not a side effect of Depoprovera

a) mild hypertension

a) prolonged ammenorrheoa

a) thromboembolism

a) hypelipidemia

179. WOF won’t decrease potassium level in a patient with hyperkalemia

a) glucose + insulin

a) calcium

a) resonium

a) sodium bicarbonate
180. In a ‘diabetic foot’ WOF isn’t true

a) unlikely in the presence of intact pulses

a) neuropathy plays a significant role

a) major presenting problem in hospital

a) 3 year survival rate after amputation is around 50%

181. A young man is brought to hospital by his girlfriend. He is angry and repeatedly shouting ‘I
love you, drive to Timbuctu’. Most likely DX

a) mania

a) thought disorder

a) faetitious disorder

a) drug abuse

182. Evaluation of an acute head injury

a) change in level of consciousness

a) pupil reaction change

a) BP

a) Pulse rate

183. Varicocele is associated with

a) left testicular cancer

a) left renal cancer

184. Tension pneumothorax management

a) wide bore needle 2nd intercostal space


a) chest tube

185 Diagnosis of gout arthritis is best made by

a) serum uric acid

b) x-ray

c) joint aspiration

SOME IMPORTANT POINTS have 2 know

GASTROENTEROLOGY

Vitamin A Night blindness, conjunctival dryness, corneal keratinization

Vitamin D Ricketts - kids, long bone bowing; Osteomalacia-adults, demineralization

Vitamin K Clotting deficiency with prolonged PT

Thiamine (B1) Beriberi - peripheral neuropathy, Cardiomyopathy - dry or wet (high output
failure)

Wernicke-Korsakoff - Alcoholics, confabulation, nystagmus, confusion

Niacin Pellagra - Diarrhea, dermatitis, dementia, death

Pyroxidone (B6) Rare, neuropathy, Cheilosis (swollen cracked bright red lips)
Cobalamin (B12) Macrocytosis, Pernicious Anemia- megaloblastic, neuro chg. ataxia, Schilling
test , more in Strict Vegitarian.

Folate Macrocytosis, megaloblastic anemia w/o neuro chgs., common in alcoholics

Vitamin C Scurvy, bleeding gums, Connective Tissue problems, Can manifest 1 yr post defic.

Failure to Thrive Org.= decr. wt gain w/ other disease; Nonorg.=growth failure due to neglect
ßstimulation

< 80% wt for Ht

Obesity mild 20-40%, moderate 41-100, severe <101%; age, black women, low income

BMI= body wt (kg)/Ht (mm2) Normal 20-25

Pickwickian Syndrome = obesity, dyspnea, hypovent, CO2 retention, hypoxia

Boerhaave's Syndrome esophageal rupture due to forceful vomit; Gastromediastinal fistula,


dyspnea w/o hemetemesis, + Hammonds sign = pneumomediastinum, L lung effusion

Dysphagia Obstructive - solids 1st; Motor -solids = liquids

Infectious Esophagitis Candida (thrush), HSV, CMV, immunocomp, diabetics, Dysphagia &
odynophagia

Esophageal Atresia Proximal esophagus, blind pouch


Trachesoph Fistula Congenital defect, distal esoph, coughing & cyanosis when feeding, abd
distention

Achalasia Dysphagia for solids & liquids, nocturnal cough, aspiration; Absent peristalsis & tight
LES, "Beak" esoph on x-ray, 20-40 yrs old

Esophageal Cancer squamous 90% Dysphagia solids 1st, Cough & hoarse = laryngeal nerve,
constricting bands = annular lesion, Risk factors= smoking, alcohol, GERD, Barretts Esoph = adeno
CA

Gastritis Antral Creep=fundal tissue replaced by antral mucosa, Not preCA,

Risk Factors = NSAIDS, Alcohol, H.Pylori

PUD Gastric = NSAIDS, eating no help, COPD,blood type A; 25%

Duodenal = H.Pylori, better w/ food, Liver cirrhosis, Blood type O ; 75%

Cullen's Sign Periumbilical cyanosis d/t hemoperitoneum = hemorrhagic pancreatitis, ruptured


ectopic, ruptured spleen

Zollinger-Ellison Synd. Gastrinoma ( incr. gastrin); recurrent ulcers, ½ are malignant

Gastric CA Adeno, H.Pylori gastritis, Virchow's Nodes, Types = ulcerating (shallow edges);
polyploid (intraluminal late mets); superficial (early CA) ; Linitus Plastica (all layers decr. elasticity)
Mets to ovary = Krukenberg Tumor

Diarrhea Osmotic = incr. H2O lumen incr. solutes in bowel; Secretory = electrolytes & H2O
secreted not absorbed; Malabsorption; Exudative secretion of blood plasma & mucus (mucosal
inflammation); decr. transit time (short bowel); incr. transit (bact. Proliferation)
Ischemic Colitis Vascular compromise (atherosclerotic or embolic); abrupt abd pain after eating,
bloody diarrhea, systemic sx.; Barium X-Ray - Thumbprint = pseudo tumor

Irritable Bowel Synd. Dx of exclusion (psych?); Tx: bulk supp, anticholinergics, antidiarrheals, TCA

Colonic Polyps villous>tubular ; sessile>pedunculated for being CA; familial adeno polyps
autodom.

Lactose Intolerance Lactase deficiency, bloating & explosive diarrhea after milk; Ages 10-20

Celiac Sprue Gluten sensitivity (wheat, rye, barley); amenorrhea 1st sx girls, Infants = FTT,
abnormal stool, bloating, Adults = malabsorption, vit deficiency; X-ray - dilated loops of bowel with
thin mucosal folds; most common cause of malbsorption

Tropical Sprue nutritional defic, small bowel mucosal abnormal; Acquired - Caribbean, India, SE
Asia; Megaloblastic Anemia, glossitis, diarrhea, wt loss Tx Folic Acid & Tetracycline

Whipple's Disease Infectious; middle aged men; multi-organ; Thickened mucosal folds, Foamy
macrophage with rod shaped bacilli that stain w/periodic acid (Schiff's Reagent)

Intestinal Lymphangiectasia Children & young adults; cong or acquired telangiectasia of


intramucosal lymphatic

Massive extremity edema w/ diarrhea, N/V; Tx: low fat w/ triglyceride supplement

Toxic Megacolon Dilation > 6cm, Adults preceded by IBD (UC or Crohn's) Kids preceded by
Hirschsprungs Ds; Sx: severely ill, incr. temp, abd pain, rebound, leukocytosis; X-ray - intraluminal
gas along continuous seg of dilated bowel; Tx: NPO, IV fluids & electrolytes, Antibiotics & Steroids,
Rectal tube may alleviate but can cause perforation
Inguinal Hernia Indirect = infants, persistent processus vaginalis, protrudes thru ring, lateral Inf
Epig.

Direct = Adults, medial to ing ring & inf epig artery, Weakness in Hesselbach's

Ulcerative Colitis Colon & Terminal Ileum w/o skip lesions, w/ rectal bleeding, "lead Pipe" on X-
ray (shortened, narrowed, loses haustrations) Tx: Sulfasalazine, steroids, Immunosuppresents;
Complications=perf, hemorrhage, Toxic Megacolon, Colon CA

Diverticular Disease Diverticulosis = false diverticulae, pearl sign on xray

Diverticulitis = infection, acute abd pain usually on left, may form fistulas to bladder, vagina or
skin, CT w/ water soluble contrast during acute attack

Crohn's Disease Granulomatous colitis; No bloody stools, 1st in terminal ileum, transmural, skip
lesions, cobblestoning: Complications=small bowel abscess, obstruction, perianal disease,
malabsorption, toxic megacolo, Colon CA. Surgery is not curative

Colon CA Right Sided=napkin ring, anemia Left Sided=Apple core, pencil stools; Genetic; Annual
rectal>40, Annual Guaiac >50; Flex Sig q3-5y >50; Rectal CA-hematochezia

Volvulus Rotation of Bowel; Newborns & elderly; Double Bubble; Birds Beak on Barium Enema,
Tx: Left Sided = decompression; Rt sided & kids = surgery

Intussusception Telescoping of large bowel into an adjacent section; most common cause of
obstruction in kids under 2; Episodic Abd pain 1-2 min. Reflex = early vomit; Obstructive = omit
late; Currant Jelly Stool; Small Infants = Pallor, sweating, and vomiting. Leukocytosis with
hemoconcentration, BE to reduce x 2 before surgery

Necrotizing Enterocolitis Premature, decr. birth weight, older infants with malnutrition; bilious
vomit, abd distention, bloody stool, lethargy; Thrombocytopenia; Small Bowel Distention;
pneumatosis (air in bowel wall) TX; NG, TPN, IV antibiotics, surgical Rx necrotic part
Cholera Fecal - Oral, Rice Water Stools, Vibro Cholera; Endemic gulfcoast, Asia, Africa, Mid East;
Severe dehydration, Metabolic Acidosis, Tx: Tetracycline or Doxycycline

Shigella Dysentery Small bact dose needed, Blood & mucous, Kids worse than adults, Fluid
Replacement & Ciprofloxacin

Staph Enteritis Onset 3-6 hrs; "Church Picnic epidemic; N/V/D, HA, fever recovery w/in 24 hrs

Salmonella Enteritis Undercooked Poultry; nausea & cramps => watery &/or bloody diarrhea; No
antibiotics prolongs excretion of the organism

Viral Enteritis Norwalk = yr round, Rota (kids) = winter; Coxsackie A1; echo, adeno

Botulism Clostridium Botulinum, neuromuscular; onset 12-36 hrs; N/V/D, cranial nerve palsy,
fixed dilated pupils, resp failure, no fever, Wound induced = neuro w/o gi sx

Infants constipation 1st=> cranial nerve sx => cranial nerve Sx => Neuromuscular

Hemorrhagic Colitis E coli 157, cramps, => watery diarrhea => bloody diarrhea Complications=
Hemolytic-Uremic Syndrome, thrombotic thrombocytopenic pupura

Pseudomembraneous Colitis Antibiotic induced (clindamycin, ampicillin, cephalosporin) C. Diff;


mail -> severe bloody diarrhea; Tx: stop antibiotics, oral metronidazole in severe cases

Acute Pancreatitis Pain radiates to back w/ N/V; Grey Turner's Sign (blue flank) Cullen's Sign
(blue at umbilicus; Amylase & Lipase incr. ;

Ranson's Criteria: 3 or more = incr mortality

Admission: >55, gluc>200, LDH >350, AST >250 WBC > 16000
Within 48 hrs: Hct decr. 10%, BUN incr. >5, Ca< 8, PaO2 < 60, Base Def > 4, Fluid Seq>6L

Chronic Pancreatitis ERCP to Dx; Alcoholics, Malabsorption & diabetes are results

Hepatitis Hep A = Fecal oral, shedding before Sx; IG to travelers & contact with HAV infected

Heb B= Blood & STD; HbsAg early if persists = carrier; HbcIGM then HbcIgG for life

HBIG for needle sticks and infants born to + moms; assoc w/ hepatocell CA

Hep C Most common post transfusion hepatitis

Hep D = Co infection with B; Hep E = fecal oral

Cholelithiasis Female, fertile, fat, forty; Ultrasound, RUQ pain after fatty meals, Calcified = preCA

Cholangitis Charcot's Triad = Biliary Colic, Jaundice, Fever; Leukocytosis, incr. Alk Phos

Hepatocellular CA Mets 2x more than primary CA (breast, lung, colon), Budd Chari-thrombosis
hep V.

Risk Factors = HBV, HCV, Alcoholic cirrhosis, Aflatoxins (fungal metabolites)

Benign Hep Adenomas oral contraceptives

Exocrine Pancreatic CA Ductal CA, Courvoiser's Law=palpable nontender GB in a jaundiced


patient is a head of Pancreas tumor. Tumor Body or Tail=splenic vein obstruction=> splenomegaly,
gastritis, esoph varices

Insulinoma Insulin hypersecretion, hypoglycemic symptoms, Insulin levels still incr. after fasting

Whipple Triad: confirms hypoglycemia as source of Sx 1. Hypoglycemia, 2. Relieved with carb


ingestion 3. Sx occur while fasting
VIPoma Makes Vasoactive peptide (VIP); Sx WDHA (watery diarrhea, hypokalemia, achlohydria;
unexplained secretory diarrhea; laparoscopy for Dx

Glucagonoma tumor of alpha islet cells; 80% women; 80% malignant; Necrolytic Exfoliating
Erythema characteristic exfoliating lesion of the extremities

Pompe's Disease a 1,4 glucosidase deficiency; fatal by age 2

VonGierke's Disease glucose-6-phosphatase def.; big liver & kidneys, growth retardation,
electrolyte prob.

McArdles Disease musclephosphorylase is absent, muscle cramps & incr. myoglobin after
exercise

Jaundice Prehepatic = hemolysis, gilbert's disease, Crigler Najjar; Hepatic = hepatocellular or


cholestatic; Post Hepatic = Biliary obstruction, AST & LDH also incr. ;

incr. unconj prehepatic; incr. conj = intrahepatic cholestasis or post hepatic disease ; incr. all
fractions = hepatocellular, hyperbilirubinuria = conj bili water soluble

ETOH Hepatic Disease Fatty liver => ETOH hepatitis => cirrhosis; AST incr. > ALT incr. ; incr. PT;
decr. II, VII, IX, X Clot

Cirrhosis necrosis and fibrosis, decr. serum albumin, anemia incr. PT, Not curable or reversible

Esophageal Varices veins that expand to circumvent congested hepatic flow; Tx: vasopressin,
balloon tamponade, endoscopic sclerotherapy, transjugular hepatoporto shunt (TIPS
Hepatic Encephalopathy altered consciousness, incr. ammonia incr. glutamine in CSF, EEG
abnormal

Ascites Complication of hepatic disease; Paracentesis to examine ascitic fluid values should
equal serum if incr. albumin = malignant; incr. LDH > 60% of serum = malig or infective;

incr. WBC = infection

GI Bleeding Upper GI = melaena (black tarry) ; Lower GI = hematochezia (bright red) decr. Lig
Trietz

Intestinal Obstruction Vomiting common in small bowel, late in lg bowel, High pitched "tinkling"
BS.

X-ray - prox dist = lg bowel; dilated loops of bowel with air fluid levels = sm bowel

Ileus Paralytic obstruction of bowel due to loss of peristalsis

Pyloric Stenosis Projectile vomit in neonates, visible peristaltic wave, String Sign

Meconium Ileus Abnormal thick Meconium with undigested protein, associated with CF

Hirschsprung's Disease No autonomic nerves in colon, Obstipation, late vomit, Megacolon, Toxic
Enterocolitis if left untreated, BE proximal dilated & distal narrow, Colostomy

If it will help u guys just let me know I'll post rest of the systems

Thxn 2 eaziz

Cheers
MCQ 2001 MAY RECALL QUESTIONS

MEDICINE

1. In which of the following diabetic ketoacidosis most commonly presents with

1. Undiagnosed IDDM

2. Undiagnosed NIDDM

3. Known IDDM when stopped Insulin

4. Known IDDM with foot infection

2. Which is correct for NIDDM

1. Genetic factor is more important in IDDM then NIDDM

2. They will never require insulin

3. 10-20% of IDDM need hypoglycaemic agents

4. Abdominal fat is the risk factor for NIDDM

3. Papillary necrosis Except

1. Analgesic nephropathy

2. Alcohol Nephropathy

3. Medullary sponge kidney

4. Diabetis

4. Picture – girl Acne and hirsutism on the face. Presented for the first time

1. Antibiotics

2. Antibiotics and retinoids

3. steroids cream

4. antiantigen(Danazol)

5. cypropterone acetate

5a. Picture with 3 lesions- with raised border

1. Granuloma annulare
2. Erythema multiforme

3. picture –AMC book – leision on the face

4. Mycrosporum canis

5. What is not true about parkinsons disease

1. can stop while walking involuntary

2. tremor at sleeping

3. rest tremor

4. affect one side more than the other

6. Mycoplasma pneumonia not true

1. severe cough

2. high fever

3. pleuratic paing]

AMC MCQs WebWare.8m.com

211

Recall Paper AMC MCQ EXAM OCT 1999

Surgery

1. A young patient present with a lump in the arm. You are unsure about its

nature. What do you ask to help to establish your diagnosis? Choice what is the

best response?

A. Is it painful?

B. Is there any lump in other side of the body?

C. Is there any change of bowel habit?

D. What do you think cause of the lump?

E. How long it present?


2. What is the common cause of bone tumour in the pelvis of young adult?

A. Metastatic Tumor

B. Osteoma

C. Osteosarcoma

D. Chondroma

E.

3. What is the common cause amaurosis fugax (transient mononuclear vision)?

A. Stenosis of internal carotic artery?

B. Giant cell arteritis

C. Vertebral artery occlusion

D. Embolism to middle cerebral artery.

E.

4. A women developed tinnitus and sensory neural deafness decreased sensation

in trigeminal area. What is the most likely cause?

A. Acoustic neuroma.

B. Brain stem infarction.

C. Vestibular neuritis.

D. Vasculitis.

E. Meningioma.

5. What is true about carcinoma of lip?

A. Usually in upper lip.

B. Usually squamous cell in origin.

C. Metastasis to cervical lymph node.

D. Exclusively occur in men.


AMC MCQs WebWare.8m.com

158

AMC MCQ recall-paper October 2001

Medicine

1.A 18 yrs old lady came with hypertension. She has a history of enuresis up to 13

years. WOF is the most likely diagnosis?

a) Reflux nephropathy

b) Hypertension

c)

2. Erythema nodosum can occur in all of the following condition except

a) streptococcal infection

b) TB

c) Rheumatic fever

d) SLE

e) Leprosy

3.A picture of scalp with hairless round area

a) Tinea capitis

4. A picture of hand with scaly shiny things on it

a) Psoriasis

5.How can we monitor heparin?

a) INR

b) APTT (activated partial thromboplastin time)

c) Fibrinogen

6.A pt with von Willbrand’s disease . What is the most likely she has

a) Prolonged bleeding time

7.A 62 yr old man has a long history of COPD and dyspnoea, blood gas showed
pCO2 68 mm Hg pO2 60 mmHg. He was given28% O2 therapy. After an hour the

blood gas showed 7.37, pO2 80mm of Hg and pco2 40 mm of Hg but the pts

clinical condition didn’t improved. WOF IS NEXT THERAPY

a) Bronchodilator

b) Hydrocortisone

c) Intubations and ventilation

8.ABOUT AMYLOIDOSIS all of them are associated except

a) Leprosy

b) TB

c) Multiple myeloma

AMC MCQs WebWare.8m.com

102

MAY 2006

3. PT comes to u with dec breathing sounds on the right middle lobe and

consildation and dullness in percussion

1 Bronchiactasis

2 PL effusion

3 pulmonary collapse

4 Peumothorax

9. Picture of Psoriasis Anthology book and ask about the RX

1 topical steroid

2 oral steroid

Erythromycin

4intralesional steroid
11. Patient with dilated pupil and history of pain in the eye with nausea and

vomiting

1 apply topical acetazolamid

2 Topical pilpcarpine

3 check the eye to look for foreign body4 apply atropine

12. an old male C/o of short history of malaise anorxia when he was in a short

trip to Balli His wife think he is jundised now he C/o chills and inc bilrubin, inc A

L P sligh inc in AST AND ALT the diagnosis is

a1 cholangitis

2 viral hepatitis

Malaria

16.An old man with chronic renal familiar His MCV count is of (80-90) it falls even

after some time when he is started erythropoiten therapy when he stopped he

became also anaemic what is this due to?

1 Iron def. anaemia

2bone marrow fibrosis

3 folic acid def

4 red cell atypia

24 .You are called to solve a fight between head nurse and a psychiatric patient .

On your arrival the patient tells you that he should be allowed to remove the IV

line as he has private ensurance & you being a doctor would understand this

because you will also have private insurance you wouldn't be like these nurses

who are without private insurance .what is the patient trying to do

1 regression

3 projection

splitting
AMC MCQs WebWare.8m.com

49

AMC MCQ 2005B2

Recall, Adelaide, ‘Version alter, off handwritten original’

1. 4 days after Normal Vaginal Delivery came with 38oC fever,

a. Unsutured vaginal tear.

b. DVT

c. Endometritis

d. Mastitis

e. Breast Engorgement

2. Which one is normal in new born

a. fever

b. HR less than 70

c. Breast enlargement

3. Best to know gestational age

a. US at about 8 wks

b. US at about 18 wks

4. Common side effect of olanzepine

a. Nuetropenia

b. Hypotension

c. Sexual dysfunction

d. Weight gain

e. Parkinsonism

5. Treatment of trigeminal neuralgia


a. Carbamazepine

b. Nortriptyline

6. Not in MS

a. 3rd heart sound

b. calcification

7. 38wks gestation, in labour with some complain (cant remember properly), PV

done, next step.

a. PV

b. CTG

c. CS

8. A pg women of 32 wks of gestation will report to the hospital for all of the

reason except,

a. show

b. rupture membrane

c. profuse bleeding

d. blurring vision with ankle edema

AMC MCQs WebWare.8m.com

AMC MCQs from Web

PSYCHIATRY

1.A lady with a previous divorce now comes to you with a seductive behaviour

a. Narcistic

b. Histrionic

c. Borderline
2.In Australia bush fire are common either accidentally or some people lighting

fire deliberately. Which is true regarding pyromaniacs?

a. Done for notoriety or publicity

b. To hide their acts

c. As they like to play with fire

d. Set fire and get panic attacks

e. For satisfaction

3.Depersonalization may occur in

a. Schizophrenia

b. Ecstatic religious experience

c. Depression

d. Post traumatic disorder

e. All of the above

4.A middle aged lady present to you with nausea dyspepsia abdominal

distension. She had a past history of going to many doctors and being treated for

many disorders. She has been treated by a rheumatologist for aches and pains,

cardiologist for her palpitations and gave her propanolol without improvement, a

neurologist for her epilepsy. On examination you find a tense anxious woman in

spite of her daily dose of benzodiazepine. There are scar from appendectomy and

hysterectomy operations. What is the most appropriate diagnosis?

a. Munchausen syndrome

b. Conversion disorder

c. Hypochondriasis

d. factitious disorder,

e. Neurotic anxiety

5.Many psychopharmacologic substance are lipophilic the advantage is


a. better absorbed with meals

b. promptly binds with free fatty acids

c. more resistant to be metabolized by the liver

d. the pass rapidly to the brain and to the blood

e. a drug which has a long life and needs to be given once a day daily

Guest, Oct 16, 2006

#1

anne123

anne123 Guest

these recalls are dedicated to people who called me s...

July 2005 AMC MCQ Recall Topics

PICTURES

1. Kaposi’s sarcoma x 2 on arm – most likely cause of cough and dyspnoea in the patient?

Pneumocystis carinii pneumonia

2. Radiograph Colles fracture – what is an important step in assessment of this fracture?

Assessment and documentation of median nerve function

3. Radiograph Colles fracture – after reduction and casting, patient complains of
paraesthaesia and swelling of fingers and hand, what is the most appropriate next step in
management?

Split cast along both sides

4. Lower limb cellulitis in a diabetic man, no ulceration. What is the most appropriate antibiotic
combination in this patient?
a)Flucloxacillin and penicillin b)flcuxacillin and gentamycin c) ampicillin, gentamycin ,
metronidazol

5. Ulcerated nodular skin lesion on the dorsum of hand with variegated pigmentation, notably
arcs of melanin surrounding central nodule. What is the most likely diagnosis?

Malignant skin lesion

6. Hand showing a swan neck deformity on index finger. Which of the following is true?

This deformity is more likely to be found in rheumatoid arthritis than osteoarthritis

7. Completely round clearing in the hair of a child. No satellites. What is the most likely
diagnosis?

Tinea capitis

8. Two skin lesions, only one in focus. Erythematous macular lesion, lichenified with silvery
keratin scaling. Clearly defined borders. What is the most likely diagnosis?

Psoriasis

TOPICS

1. Jaundice, elevated ALT,sorethrought ,atypical lymphocytes in young student. What is the


most likely diagnosis?

Viral Hepatitis

2. Young student complains of malaise. On examination no jaundice, cervical lymphadenopathy


and investigations reveal mildly elevated ALT. What is the most likely diagnosis?

Infectious mononucleosis

3. Young medical student with scleral icterus, mildly elevated bilirubin, mostly unconjugated and
no significant history. othrwise fine What is the most likely diagnosis?

Gilbert’s syndrome

4. 65 year old man complains of lethargy. On examination pale and splenomegaly present. FBC
and diff shows anaemia with elevated WCC (predominantly lymphocytic). What is the most likely
diagnosis?

a)CLL b )CML d)lymphoma


5. Elderly lady complains of lethargy. On examination pale and glossitis present. FBC and diff
show anaemia with macrocytosis.and segmented monocytesnucleus and reduced PLT What is the
most likely diagnosis?

Pernicious anaemia

6. Which of the following conditions is not associated with erythema nodosum?

Rheumatic fever

7. Which of the following results would be inconsistent with a diagnosis of neural tube defect?

Decreased AFP

8. What is the most appropriate test to monitor eradication of H.pylori one week after
commencement of triple therapy?

Urea breath test

9. Elderly man complains of non bilious vomiting on a background history of treated duodenal
ulcers. What is the most likely cause of his vomiting?

Sequelae of chronic duodenal ulceration

10. Which of the following is characteristic of carpal tunnel syndrome?

Pain at night along lateral 2.5 fingers

11. Carpal tunnel syndrome in pregnancy, which of the following is true?

Rarely requires surgical intervention

12. Patient with wasting of all small muscles of one hand including thenar ones. Where is the
site of the lesion?

Brachial plexus

13. Patient with symmetrical distal wasting of small hand muscles bilaterally. What is the most
likely diagnosis?
Syringomyelia

14. Which nerve is most likely to be involved in spiral fracture of the humerus?

Radial nerve

15. Which nerve is most likely to be involved in anterior dislocation of the shoulder?

Axillary nerve

16. What is the best choice for pain relief during reduction of a dislocated shoulder?

Fentanyl and midazolam

17. Which of the following regarding narcotic analgesics is true except?

In general, morphine is safer than pethidine

18. Define incidence.

Number of new cases in a period of time

19. After cardiovascular diseases, what is the next most common cause of death in the general
Australian population?

Malignancy

20. In a patient with a molar pregnancy, what is the most important component of her follow up
with regards to possible malignancy?

Serial hCG estimation

21. Which of the following is true regarding managing chest pain?

No thrombolysis with a normal ECG and cardiac enzymes

22. Which of the following drugs is least likely to cause serotonin syndrome in a patient on a
SSRI?

Moclobemide ?? Other options are:

L-tryptophan

Citalopram

Amitryptyline

23. Working as a pathologist, reviewing the results of a woman after an episode of vaginal
bleeding. Findings – endometrial hyperplasia, US shows empty uterus, elevated hCG.
Considering the female patient has been discharged into the community, what is the most
appropriate next step in management?

Recall patient immediately, arrange for laparoscopy

24. What is the least likely diagnosis for right iliac fossa pain in a woman at 17 weeks gestation?

Ectopic pregnancy

25. GCS score estimation – withdraws to pain, incomprehensible speech, eye opening to pain.

26. Find the PA-aO2 given FiO2=0.21 and PO2=68 and PCO2=40 (alveolar arteriolar oxygen
tension difference).

32 from PAO2 = FiO2 x 760-47 – (PaCO2/0.8) and gradient = PAO2 – PaO2

27. Regarding the Mental Health Act, which of the following is true?

May save lives in the mentally ill

28. Regarding splenectomy in patients with spherocytosis, which of the following is false?

Persistence of anaemia

29. Hyperpigmented man with small testes and diabetes. What is the diagnosis?

Haemochromatosis

30. What is the most appropriate test for diagnosing haemochromatosis?

Ferritin (liver biopsy is NOT a given option)

31. Child vomiting, electrolyte derangement high K low Na and low Cl. What is the most likely
diagnosis?

Congenital adrenal hyperplasia

32. Child with vomiting and diarrhoea. Which electrolyte abnormality is most likely to cause
convulsions?
Hypernatraemia

33. Child seizing occasionally. Characteristic folding over whilst watching television. What is the
diagnosis?

Infantile spasm

34. Elderly man with symptoms and signs of small bowel obstruction. What is the most
appropriate IV fluid for replacing losses?

Hartman’s solution

35. Male patient complains of retroorbital headache that awakens him during the night
accompanied by watering nasal mucosa and tearing from one eye. Which of the following
medications will be appropriate for use in this patient?

Methysergide

36. In cardiopulmonary resuscitation, which of the following signs is the best indicator of
successful resuscitation?

Reversal of pupillary changes

37. Which of the following organisms is most likely to cause diarrhoea from a patient with
recent travel history to Indonesia?

ET E. coli

38. A patient with recent travel history to Nairobi complains of cyclical fevers. Which of the
following investigations will be most useful in reaching a diagnosis?

Examination of thick peripheral blood film

39. A child complains of a clean laceration on his scalp. Has not been vaccinated. What is the
most appropriate management?

DTP vaccine

40. A man who is grandiose, arrogant, little empathy for others, considers himself special. What
is the most likely psychological disorder in this man?

Narcissist

41. What is the treatment of choice for migraine?

42. a patient presents with distal weakness and atrophy ob small muscles of both hands/
syringomyelia

43. whixh of the following ia most appropriate for EDC?

a.Sonogrophy at 8 weeks

Recall Questions 18 July 2005

1- Oestrogen and Progesteron, as oral contraceptives where do they function?

a) Endometrium b) Ovaries c) Hypophysis d) Hypothalamus

2- Mental Health Act

a) Same in every state b) Restricts liberty c) Saves lives

3- X- Ray of wrist showing Smith’s fracture, initial management?

a) Plaster elbow and wrist b) Plaster wrist & hand c) ORIF d) Check median nerve

4- Same X- Ray, it was put in plaster, patient feels pins & needles, management?

a) Release distal part of the plaster b) Remove plaster c) Pain relief, send her home

5- Picture of lesions on dorsal hand (blisters), cause of his pneumonia?

a) Strep. Pneumonia b) Listeria c) Mycoplasma d) ?

6- Hypnagogic hallucinations occur in?

a) Schizophrenia b) Delirium c) Normal sleep

7- Patient with vertigo and tinnitus, no deafness, diagnose?

a) Meniere b) Vestibular neurinitis c) Acoustic neurinoma


8- Baby was delivered with forceps, not moving one arm, diagnose?

a) Erb’s palsy

9- Picture of a man arms raised (AMCQ book)

a) Retrosternal pressure b) Cervical rib c) Subclavian steal

10- Young man wakes up with pain behind one eye, pain spreads to same side of his head, lasts
one hour, diagnose?

a) Migraine b) Cluster headache c) ?

11- Which is not characteristic of Rheumatic Fever?

a) Erythema nodosum

12- Malignant cells in ascitis, what other symptom would you expect?

a) Supraclavicular lymph node enlargement

13- Young man had MVA, opens his eyes to stimuli .....

What blood levels would you expect to find?

Answers included five different O2, CO2 and pH levels

14- Lady is HIV positive, which statement is correct?

a) She has a life long disease

15- Which of the medications below would cause a serotonin syndrome when used together
with SSRI(except)?
a) Tryptophan b) Olanzapine c) Meclobomide d)halopridol

16- Man has a recently darkened skin, lethargy. Ferritin, Iron, AST and ALT levels are elevated,
diagnose?(what is the best diagnosis? a)ferritin level b)serum Iron c)TIBC)

a)Haemachromatosis

17- Young man has neck rigidity, headache, fever. CSF results: Protein: N, Glucose: lower end of
normal range, also PMN cells, diagnose?

a) TB meningitis b) Viral meningitis c)bacterial meningitis

18- 24 yr old pregnant, P2 G1, 38 weeks and she is in labour. She is 150cm tall. On exam, her
pelvis looks small but cervix is 4cm dilated, next management?

a) Pelvimetry b) U/S c)Caesarian d) Trial of vaginal delivery

19- Reason for UTI in pregnancy?

a) Dilated ureter b) Dilated ureter&calices c) Immune deficiency

20- Which statement is correct regarding Hyperkalemia?

a) Calcium gluconate given to reverse ECG changes occurred due to Hyperkalemia

21- A few Qs regarding facial oedema, proteinuria, haematuria and differential diagnoses
between Nephrotic synd, AGN, etc.

22- Old lady with unilateral headache, tenderness, ESR elevated, treatment?

a) Aspirin b) NSAID c) Prednisone (15)25mg d) Prednisone 75mg

23- For what age group is mammography most useful?


a) <35 yrs b) 35-50 c) 50-65 d) All ages

24- Boy with a small clean cut to his scalp, he is not immunised, management?

a) Toxoid b) Clean, send home c) Immunglobulin d) Toxoid & Immunglobulin

25- Picture of Dupuytren contracture, location of the lesion?

a) Distal flexors b) Digital extensors c)?

26- Facial nerve damage is most likely to occur in(except)?

a) Parotitis b) Parotid Ca c) Temporal fracture d)?

27- All expected in Post splenectomy except?

a) Normal life span of erythrocytes

b)persistant anaemia c)postive osmolarity fragility test

28- Child sits unsupported, can weight bear with support, how old is he?

a) 7 months b) 5 months c) 9 months d) 11 months

29- Patient with exacerbation of COPD, in respiratory failure. O2 given at 28L/min, ABG: O2: 48
CO2: 58. Next management?

a) Mechanical ventilation b) Stop O2, repeat ABG in 30 min.

30- She had 3 miscarriages from first partner, 2 miscarriages from second partner. Most likely
cause?

a) Chromosomal abnormality b) Cervical incompetence c)?

31- 50 yr old man with Hx of Duodenal Ulcer, lost weight, pale. Most likely cause?
a) Duodenum Ca b) Gastric Ca c)?

32- Picture of scalp, area of hair loss, psoriasis-like appearance, diagnose?

a) Alopecia areata b) Psoriasis c) Trichotillomania d)Tinea Capitis

33- A few Qs re differentiation of hepatic and gallbladder diseases

34- Q re neurosyphilis

a) tabbes dorsalis

35- Picture of large goitre in which condition does the surgery requiered?

a)airway constriction

36- Values of osmolality and sodium in urine and plasma given, differential Dx between SIADH
and D Insipidus

MCQ Exam – 18th July 2005

RECALL QUESTIONS

Pictures –

1 SU Q-19 (AMC) book –

Man presented with a neck swelling and a feeling of dizziness when reaches upwards –
Retrosternal goitre (venons obst from the ext: of goitre)

2 On the dorsum of hand 2cm irregular margin, skin lesion, black brown
– malignant melanoma of skin

3 Two brownish – red large (2cm) size lesion with ceratitis – back of a young man – Kaposi
sarcoma

– suggestive of AIDS (HIV)

4 Hair loss on the scalp of a child

– Taema Capitis

5 Two lesions with scaly lesions over – look like silvery scales

– Psoriasis?

6 Index finger – extruded and flared at distal Interphal joint – what is the lesion? (Trigger
finger)

• Nerves or tendon affected

• Flexion or ext: deformities

• More in RA than osteoarthritis

7 A diabetic man with severe redness of one lower limb with some hair follicles infection –
whole lege below knee to ankle – cellulitis with infection. Not sensitive to any drug –
treatment – many options:

• Flucloxacillin & penicillin

• Gentamycin & ampicillin & metronidazole

• Amoxicillin

• Ampicillin & Gentamycin

8 An old woman fell on her out-stretched hand – X-ray – wrist.

Diagnosis – Colle’s #

9 (Same picture in 2nd paper)

Patient complains of severe pain and swelling of fingers and redness – immediate
management?

• Repeat X-ray

• Refer to the nearest GP


• Review after 24 hrs

• Split both sides of the plaster

• Antibiotics

10 Patient with fever, pleuritic pain – dull or percussion (R) side, bronchial breath sounds on
(R) side. Most consistent with

– Consolidation (R) lung.

11 Picture of a lady with large swelling neck – visible as multiple modules

– Multinodular goitre.

All the pictures probably from ‘the Anthology of Medical Condition’ (AMC publication)

12 Confused patient in ED after the trauma assess the level of consciousness

– Glasgow Coma Scale

13 Patient with wernicks encephalopathy – treatment options

– IV thiamine & glucose infusion

14 In a hypertensive patient – thrombolytic therapy. Investigation most important

PT, APTT. FBC.

INR, Bleeding time

15 Nephrology – 1 question

16 Which condition will lead to chronic hepatitis?

- Hepatitis C

17 Liver disease – ALT/AST/bilirubin elevated. Patient with fever. Dark urine – most
suggestive of?

- Acute Hepatitis B

18 Haemochronatosis – most favourable diagnostic test

Eg. Ferritin, TIBC

Serum transferrin binding

Prothrombin time
19 A man had bike accident – fractured end of tibia, 3 cm over the skin – pierced
(compound #). Most appropriate treatment

– TT & immunoglobulin & Antibiotics

20 2 yr old child fell down – with minor lacerations on scalp – no H/O immunisation –
what will you do?

- Give DPT

21 5 yr old child with severe diarrhoea convulsion – cause?

- Hypernatraemia = Na+ 165

22 6 wk child with forceful vomiting, mild dehydration. Elecholylis Na+ 119, K+ 7.5, Cl+ 110.

Likely diagnosis

– Cong: adrenal hyperplasia

23 7 yr old child with morning headache and vomiting for 3 months – last six weeks he had
ataxia family H/O migraine and school epidemic of chicken pox – possible diagnosis?

- Posterior fossa tumouse

24 2 yr old child with severe cough, recurrent resp: infection. Failure to thrive – what test is
important?

- Sweat Chloride Test

25 6 month old infant brought with funny turns, sudden flexion of upper and lower limbs – for
1 week

– Infantile spasm

26 2 yr old child with wheezing on one side of lung for two days – investigation

- Inspiratory and expiratory lung x-rays

27 A child – rolls from side to side, sits for some time without support, grasp things with palm
– assess the age – development

3, (7), 10 1 yr – 18 months

28 New born baby – first step in resuscitation – on the baby?

- Pharyngeal suction, naloxone injection. Intubation, oxygen mask, nasal suction


29 A child can’t see writing on the board – in the class but can use computer. What is he
suffering from?

- Hypermetropria, Ashigmatism, Myopia. Cataract, Presbyopia

30 21 yr old lady returned from Bali, after holidays, with diarrhoea with no blood – Diagnosis?

- Toxogemic E. Coli

31 A medical intern from other country, colleagues noticed yellowish discolouration of her eyes
– she has no other symptoms – Probable diagnosis

• Early Hepatitis

• Mild jaundice

• Iron deficent: anaemia

• Acute Heaptitis B

32 Patient with H/O snake bite – no bit mark – no symptoms of poisoning

• Send him back home – nothing wrong

• Observation for 24hrs

• Open the area, apply tourniquet in tight, antibiotics

• Inject polyvenom

33 A case of temperal arteritis – Management?

34 Otoselerosis

• Appearnance of tympairic membre

• Narrow eustachian tube

• Deafness

• Merrieres disease

35 A boy with fever, pain and swelling of (R) knee – pain at upper end of tibia – mobility of
the knee joint affected.

? septic arthritis

? osteomychitis

36 Question about reifer’s disease


37 One case of growth & development retardation

- X-ray wrist and other investigation

38 19 yr old boy – height and weight m……. with BP 150/76

• Check BP 24 hours

• Blood test

• Check the BP cuff size

• Antihypertensive treatment

39 Lady with weakness of small muscles and of forearm bth hands. Diagnosis is?

• Vertibrobaislar artery

• Multiple sclerosis

• Syringomyelia

• Bilaterial ulrase nerve palsy

• Meuieves disease

40 Dislocation of shoulder –

- what is the Anaesthetic used to correct this dislocation

41 Spiral # midshaft of humerus which structure affected

- Radial nerve

42 Paralysis of small mucles of hand, wrist flexion affected (except ….., interrossie,
hypothenar)

- ulnar nerve injury above wrist

43 Carpel tunnel syndrome

- survery usually not required

44 For major surgery – antibiotic is given-

• 1 hour before operation ( )

• time of incision

• 12 hours after operation


• 24 hours before operation

45 Undescended testis – bring down this testis and fix

- Orchidopexy

46 Inguinal swelling that disappear when lying down

- direct inguinal hernia

47 Breat engorgement in a male baby (new born – 2 days)

- normal condition

48 Fistula is – correct description?

49 A man with claudication (calf) pain, forced to stop after 100 metres – investigation

• Arteriogrphy

• Doppler method

• X-ray

50 Ca – don’t metastasis to brain

- Ca prostate

51 WOF Ca mostly metasise to bone?

Ca Stomach

Breast

Bladder

Rectum

Oesophagus

52 Patient with mastalgic – failed conservative treatment. WOF to be considered –

Bromocryptive, danasol, oestrogen, combine OCP, progertrone

53 3 month old boy with recurrent sticky eye since birth – 1st time responded to antibriotic
– what is the diagnosis

• Chlamydia infection

• Gonorrhoea infection


• Cong: duct blockage

• Herpes infection

54 Sensory supply to ant: 2/3 of tongue:-

Trigeminal N; IXth CN

Facial N; IIIrd CN

55 A patient with duodenal ulcer after treatment for erradication of H-plori – most accurate
test to prove irradication?

Endoscopy – biopsy

Urease test, serolgy, breath test

56 A newborn developed cyanosis and breathing problem. What is the cause?

• Fallol’s tetrology – VSD

• Pul: hypertension

• Transpositon of GA

57 How will you differentiate – finding in ASCITES with large ovarian tumour

58 Diabetic man with ulcer sole of foot (toe). Associated with

• Venous ulcer

• Arterial supply decreased

• Peripheral neuropathy

59 Patient with tension Pneumothorax – Large bore needle in the 2nd space anteriorly

60 Side effect of depo-provera – Except?

61 Contraceptive effect (action) of microgynon-30

62 One question about CTG

63 40 year old man came with H/O. Mother had colorectal Ca – advise for him – he is scared

? Do colonoscopy for him

64 Patient with serum Na+ and changes with urine osmolality –

SIADH
Diabetis insipedus

Hyperlipridaemia – Adrenalhyperplasia

65 Child with H/O noe day fever and reashes on face – what investigatrion to be done

Throat swab, Urinalysis

Blood culture, Antibiotic treatment ?

66 18 year old female with hypertension and history of enuresis. Her 2 sisters have the same
problem – Diagnosis?

- vesico – ureteric reflux

67 A male with grandiose sense of self-importance and preoccupied with fantasies of power –
not sympathetic to others –

- narcisstic personality

68 A patient with suicidal ideas – WOF will cause his safety least likely?

Unemployment, poor

Plan for suicide

No relationship with others

Possessing a gun

69 Mammography is benefitted mostly by which group?

• 35 - 45 yrs old

• post menopausal over 65 yrs

• 50 – 64 yrs

• unmarried young ladies

70 Bloody discharge from the nipple

• duet ectasia

• Intraductal papilloma

• Breast abscess

• Intraductal carcinoma


71 A lady with H/O 3 abortions at 8 – 10 wks period – no other abnormailities detected –
had abortion now at 10 wks gestation – what is most likely cause –

• retroerted uterus

• fetal infection

• pelvic infection

• foetal abnormalities

• cervical incompetence

72 A women with greenish mucopurulent discharge from vagina – WOF condition except?

• Trichomonase vaginatis

• Cegtomegatovirus

• Ca cervix

• Vag: Bacteriodosis

73 25 year old lady – found to have 3 ovarian cysts – of less than 5cm size –
asymptomatic, not pregnant. Best plan of management –

• Short course contraceptives

• Laprotomy for removal

• US exam after 4 weeks

• Tumour marker

74 Question about placenta previa – associated ?

- H/O bleeding at 26 and 30 (32) weeks

75 Patient in lithotomy position – injection given (pudental block) which muscle involved
(affected)

76 Couple adopted a baby with Down’s Syndrome – chances of recurrence of Down’s
Syndrome in future his mother ?

• 0

• at least 2%

• 10%
• 25%

• 50%

77 Anorexia nervosa

- secondary amenorhoea

78 A child 2 years with multiple new bone ossification in different sites

- non-accidental injury

79 Qs about mental heatlh act

80 Child jittery and fitting at home – management

81 Schzophrenia

82 Polycystic ovarian disease – true

• can be cured by multiple cyst removal by laparotomy

• associated with LA: FSH ratio of >3:1

• treated with chomiphene citrate always

• only occur in obese women

• causes hirsulism due to peripheral andogen production

83 Immature defence mechanism

84 CSF findings – 50-100 lymphocytes, protein 1.1gm, glucose normal

- viral infection

85 Hypertension, thrombosis – treatment?

• Warfarin

• Aspirin

• ACE inhibitors

• Calcium channel blockers

86 Serotonin syndrome – causing drugs – least likely?

• Meclobemide

• L. Trytophan
• Citalopram

• Clonipramine

• Haloperidol

87 Dejar phenomena is a psychiatric finding – in WOF this phenamin is seen other than
psychiatric disorder

- porphyria, tenporal lobe trauma, frontal lobe trauma, alcoholism

88 Patient with hypertension and weakness of (R) arm and hemparassis

- site of lesion in brain

89 45 year old lady with confound, firm, discrete, mass in the breast

- fibroadenomia, fibrocystic mass, Ca breat, breast abscess

90 Vesicular mole

91 Old women – using pessary (baginal) for a long period – what will be the complication

92 30 year old woman – FT, contractions + raptured membrane, Cx 4 cm dialated, well
effaced. Head at high level. Vag: exam reveal short pelvic …….. How will you manage the case?

• Start oxytocin drip

• Observation for – 12 hours

• Caescarian section

93 Which vitamine is deficient in breast milk?

94 About leukaemia in an old aged man

95 Absolute contraindication of OCP?

• C/- liver disease

• Hyperlipidaemia

• Migraine with H/O thromboembolism

• Thrombophlebilis

• Hypertension

96 A women 16 wks gestation with hypertension, slightly creatinime and urinalysis shows
protiens +++ most likely cause
• Pre-eclaiysia

• She can continue preg: without complication

• Pre-existing renal disease

• Essential HT, acute nephritis

97 Women with gestational diabetis – management?

98 Question about dementia

99 Acid-base balance

100 Mesentrie arterial occlusion

101 Ulcerative colitis

PAPER COMPILED BY: AYESHA MALIK

PAPER : JULY 2005C

TOTAL QUESTIONS RECALLED=157/250

PICTURES=8

PEADIATRICS=28

PSYCHAITRY=17

OBS & GYNAE=35

MEDICINE=44

SURGERY=25

NOTE.:THERE R SOME QUESTIONS OF WHOM I DO NOT REMEMBER ALL OPTIONS…SO U CAN


LOOK FOR THEM

PICTURES
1.PIC with plaques on it.

A.PSORIASIS

2.PIC of scalp showing a patch of hair loss.some crusting,redness,broken hair

A.ALOPECIA AERATA

B.TINEA CAPITIS

C.PSORIASIS

3.PIC of a lesion on dorsum of hand

A.KERATOCANTHOMA

B.INFECTED SEBECOUS CYST

C.MALIGNANCY

4.PIC of streptococcal cellulitis.Treatment

A.PENICILLIN

B.FLUCLOXACILLIN AND AMPICILLIN

5.PIC of swan neck deformity.WHAT IS TRUE

A.MORE IN RHEUMATOID ARTHRITIS AS COMP TO DEGENERATIVE OSTEOARTHRITIS

B.IT IS ALMOST ALWAYS ASSOCIATED WITH ULNAR DEVIATION

C.CAUSED BY RUPTURE OF DISTAL EXTENSOR TENDON

D.CAUSED BY RUPTURE OF PROXIMAL FLEXOR TENDON

6.PIC of displaced colles fracture manegement

A.PLASTER IMMOBILIZATION FOR ATLEAST 10 WEEKS

B.CLOSE REDUCTION????
7.PIC of diffuse swelling in thyroid gland MOST LIKELY

A.THYROTOXICOSIS

B.MUTINODULAR GOITRE

8.PIC OF SKIN LESIONS MOST LIKELY ASSOCIATED WITH

A.STREPTOCOCCAL PNEMONIA

B.LEGIONELLA PNEUMONIA

C.MYCOPLSMA PNEUMONIA

D.TUBERCULOSIS

SURGERY

1.Which nerve is likely injured in spiral fracture of humerus

A.ULNAR NERVE

B.RADIAL NERVE

C.MEDIAN NERVE

2.In uncomplicated mucosal hemmorids EXCEPT

A.FRESH BLEEDING

B.MUCOUS DISCHARGE
C.PRURITIS ANI

D.PERIANAL PAIN

3.The most common cause of bloody discharge from nipple is

A.BREAST CA

B.INTRADUCTAL PAPILLOMA

C.TRAUMA

4.WOF is not related to breast cancer

A.CONTRALATERAL BREAT CA

B.FAMILY HISTORY

C.UTERINE CA

D.ARTIFICIAL MENOPAUSE

5.CORRECT about mammography

A.PAINLESS

B.CAN DETECT BETTER THAN SELF EXAMINATION

6.MAMMOGRAPHY is mostly helpful for

A.WOMEN OVER 35 YRS

B.50 –60 YRS

C.LESS THAN 30 YRS

D.WITH FAMILY HISTORY OF BREAST CANCER

7.The most likely for a male who is 22yrs old having an inguinoscrotal swelling which disappears
on lying down
A.VARICOCELE

B.SAPHENA VARIX

C.LYMPHOMA

D.DIRECT INGUINAL HERNIA

8.In management of compound fracture of tibia and fibula MOST CORRECT is

.A.IF NO CONTAMINATION DEBRIDEMENT IS UNNECESSARY

B.EXTERNAL FIXATION

C.ANTI TETANUS PLUS ANTIBIOTICS

D.DEBRIDEMENT AND INTERNAL FIXATION

9.A fistula is

A.OPENING BETWEEN TWO EPITHELIAL SURFACES

10.FASCIAL nerve palsy LEAST likely in

A.ACOUSTIC NEUROMA

B.MASTOIDITIS

C.BASE OF SKULL FRACTURE

D.CHRONIC PAROTITIS

11.Immediate manegement of tension pneumothrorax

A.INTERCOSTAL TUBE DRAINAGE

B.WIDE BORE NEEDLE AT 2ND INTERCOSTAL SPACE ANTERIORLY

C.ENDOTRACHEAL TUBE
12.An old man c./o colicky abd pain which has become generalized now,o/e bowel sounds r
absent,on p/r reddish tinge on fingerMOST LIKELY

A.DIVERTICULOSIS

B.CA RECTUM

C.MESENTERIC ARTERY OCCLUSION

13.A pt mother has colon cancer at 65 yrs,he is 45 yrs old.he thinks he also has colon
cancer.WOF IS TRUE

A.HE WILL HAVE ANNUAL COLONOSCOPIES FROM 50 YRS ONWRDS

B.HE SHOULD HAVE FECAL OCCULT BLOOD TESTING TWO YEARLY

C.BARIUM ENEMA

D.HE SHOLUD HAVE COLONOSCOPY

E.REASSURANCE

14.IN intestinal obstruction what is correct

A.2000 ML OF HARTMAN SOL BEFORE OPERATION

B.2000 ML HARTMAN SOL DURING OPERATION

C.5%DEXTROSE WATER BEFORE OPERATION

15.A ot has undergone craniotomy his electrolytes r

Na 168 , serum osmollality200 mlMOST LIKELY

A.SIADH

B.DIABETES INSIPIDUS

C.WATER INTOXICATION

16.A pt is admitted in hospital has electrolytes


Na 117,Cl 86,K 3.2,serum osm 900

A.DI

B.SODIUM DEPLETION

C.SIADH

17.In intestinal surgery when to give antibiotics

A.ONE DAY BEFORE SURGERY

2.DURING SURGERY

C.WHEN ABDOMEN IS CLOSED

D.2 HRS BEFORE SURGERY

18.Acute pain in abdomen LEAST LIKELY

A.MYOCARDIAL INFARCTION

B.PERFORATED DUODENAL ULCER

C.ACUTE APPENDICITIS

D.

19.Unilateral exopthalamus MOST LIKELY

A.CA MAXILLARY ANTRUM

B.THYROTOXICOSIS

C.INJURY IN EYE????

20.Commonest cause of diarhea in bed ridden patients

A.FECAL IMPACTION

B.CA RECTUM
21.A woman has h/o tingling at night in her right hand o/e flexion is weak there is wasting of
abductor pollicis MOST LIKELY

A.CARPEL TUNNEL SYNDROME

B.ULNAR NERVE WEAKNESS

22.Postope rative pt who was underwent surgery on hip ???/ is now complaining of unilateral
weakness.u have diagnosed a nerve compression.MOST LIKELY CAUSE IS

A.FLEXOR HALLUCIS LONGUS WEAKNESS

B.SOLEUS

C.TIBIALIS POSTERIOR

23.A pt came with trauma o/e iplilateral pupil is dilated,bradycardia,BP I (I am not sure)MOST
LIKELY

A.SUBDURAL HEMATOMA

B.EXTRADURAL HEMATOMA

C.SUBARACHONOID HEMORRAGE

24.AOF is true regarding gall bladder stones EXCEPT

A.THEY R MOSTLY RADIOLUCENT

B.SINGLE STONE IS MOSTLY OF CHOLESTEROL

C.MOSTLY ASYMPTOMATIC

D.PIGMENT STONES R HEMOLYTIC

E.STONE AT CBD CAUSES SEPSIS OF GALL BLADDER

25.PT h/o vomiting ,some wt loss.has past h/o duodenal ulcer.MOST LIKELY

A.ACHALASIA CARDIA
B.CA DUODENUM

MEDICINE

ENT

1.About nasophryngeal carcinoma MOST LIKELY

A.BENIGN BUT LOCALLY DESTRUCTIVE

B.ASSOCIATED WITH EBV ANTIBODIES

2.Otosclerosis MOST LIKELY

A.BLUISH TINGE ON TYMPANIC MEMBRANE

B.SESORINEURAL DEAFNESS

C.NORMAL TYMPANIC MEMBRANE

3.PT with acute onset of vertigo,tinnitus no hearing loss.MOST LIKELY

A.MENEIRE DISEASE

B.VESTIBULAR NEURONITIS

C.ACUTE LABYRYNTITIS

D.ACOUSTIC NEUROMA

4.A child comes 3yr old h/o room strats moving in a circle and then he falls down .MOST
CORRECT STATEMENT IS
A.EEG WILL BE CONFIRMATORY

B.START PHENYTOIN

C.BENIGN POSITIONAL VERTIGO W/C IMPROVES WITH AGE.

LIVER.MOST OF MCQS WERE FROM LIVER

5.Diagnostic test for hemochromatosis

A.SERUM FERRITIN

B.SERUM IRON

C.SERUM TRANFERRIN SATURATION

6.A man has h/o impotence,cirrhosis.His father died b/c of same cirrhosis.RULE OUT

A.HEMOCHROMATOSIS

B C D ????

7.A woman came with c/o pain in epigastrium last evening.today morning she passsed dark
color urine.her lfts r deranged ,alk po4 is markedly increased.MOST LIKELY

A.BILIARY COLIC

B.ACUTE PANCREATITIS

C.CHOLEDOCHOLITHIASIS

8.Student 18 yr old is feeling vague has icteric eyes.o/e bibirubin is increased.rest of the
examination is normal.MOST LIKELY

A.GILBERT SYNDROME

B.HEPATITIS A

C.HEPATITS B
9.A woman had cholecystectomy 3 months ago now she is having right upper quadrant
pain.MOST LIKELY

A.STONE LEFT AT CBD

10.Hepatitis likely to become chronic

A.HEP A

BHEP C

C.HEP B

D.HEP E

MCQS FROM OTHER MISC.TOPICS

11.Hyperkalemia immediate NEXT STEP

A.GIVE SALBUTAMOL

B.CA RESONIUM

C.DIALYSIS

D.CALCIUM GLUCONATE

E.GLUCOSE AND INSULIN

12.ORG involved in food poisoning

A.E COLI

B.STAPH AUREUS

C.SALMONELLA

13.A pt sedative overdose what is typical blood picture

A.PH 7.35 PO2 70 PCO2 80


B.PH 7.1 PO2 70 PCO2 80

14.Which tumor never metastasize to brain.

A.BREAST CA

B.LUNG CA

C.PANCREATIC CA

D.PROSTATE CA

15.A bisexual man has several mouth ulcers ,arthritis,sausage fingers.also has h/o diarhea.MOST
LIKELY

A.BEHCET DISEASE

B.REITER DISEASE

C.RHEUMATOID ARTHRITIS

D.ANKOLYSING ARTHRITIS

E.PSORIATIC ARTHRITIS

16. A man has h/o unilateral headache,mostly occurs in morning.h/o lacrimation.DRUG OF


CHOICE

A.ERGOTAMINE

B.PROPRANOLOL

C.CARBAMAZEPINE

D.ATROPINE

17.Postsplenectomy hereditary spherocytosis.AOF is true EXCEPT

A.OSMOTIC FRAGILITY

B.ANEMIA PERSISTENCE

C.NORMAL LIFE SPA OF RBC


D.PRESENCE OF SPHEROCYTES IN PERIPHERAL BLOOD

18.Alcoholic ketoacidosis EXCEPT

A.NORMAL ANION GAP

19.A pt is brought to ER he is confused,ataxic,nystagmus,diplopia,…has h/o alcohol.MOST


LIKELY

A.ALCOHOLIC HALLUCINOSIS

B.WERNICKE ENCEPHLOPATHY

20.Treatment of wernicke encephlopathy with low plasma sugar

A.THIAMINE

B.THIAMINE WITH GLUCOSE

C.IV 5%GLUCOSE

D.VIT B1

21.Ankolysing spondylitis LEAST ASSOCIATED

A.AORTIC INCOMPETENCE

B.SERO –VE ARTHRITIS

22.A middle aged woman has c/o mouth ulcers,morning stiffness of joints…it gets better with
day….????MOST LIKELY

A.SLE

B.RHEUMATOID ARTHRITIS

23.Alveolar exchange difference


PaO2=?

PaCO2=?

Inspiratory O2=?

A.9

B12…..????

24.B/L weakness of hand muscles

A.MULTIPLE SCLEROSIS

B.ULNAR NERVE LESION

C.SYRINGOMYELIA

25.Weakness of all muscles unilateral in hand

A.SYRINGOMYELIA

B.?????

26.PT with atrial fibrillation what is CORRECT

A.WARFARIN SHOULD BE GIVEN

B.RISK OF STROKE????

27.Diff between hemolytic anemia and anemia due to chronis blood loss

A.DECREASED HAPTOGLOBIN

B.POLYCHROMASIA

28.Question on alpha 1 deficiency

27.Amouroxis fugax CORRECT

A.IPSILATERAL INTERNAL CAROTID ARTERY STENOSIS


29.Opening snap in mitral stenosis denotes

A.DIMINISH WITH ATRIAL FIBRILLATION

B.DUE TO VALVE MOBILITY

30.A child with suspicion of meningitis,LP lymphocytes increased,????MOST LIKELY

A.ECHO VIRUSES

B.BACTERIAL

31.Polycythemia and cor pulmonale MOST LIKELY ASSOCIATED WITH

A.TUBERCULOSIS

B.BRONCHIACTASIS

C.EMPHYSEMA

32.A man with h/o flaccid paralysis of lower limb over a week.o/e reflexes r diminished,decrease
sensation of touch,???? MOST LIKELY

A.ACUTE POLYMYOSITIS

B.ACUTE POLYNEUROPATHY

33.Amylodosis can complicate all EXCEPT

A.TUBERCULOSIS

B.LEPROSY

C.MULTIPLE MYELOMA

D.CHRONIC ACTIVE HEPATITIS.

E.RHEMATOID ARTHRITIS
34.A pt came with fever,o/e lyphadenopathy,splenomegaly,on bllod report

abnormal lfts,atypical lymphocytes r presentMOST LIKELY

A.INFECTIOUS MONONUCLEOSIS

35.A female h/o enuresis,2 sisters also have same problem MOST LIKELY

A.HORSE SHOE KIDNEY

B.HERIDITARY NEPHRITIS

C.VESICOURETERIC REFLUX

36.Heparin is measured by

A.PT

B.APTT

37.BEST method of detecting H PYLORI eradication

A.UREA BREATH TEST


38.In manegement of COPD recent improvement in health is due to

A. HOME OXYGEN THERAPY

39.ROLE OF T LYMPHOCYTES

29.Indicator of good prognosis in asthma

A.INCRESING PULSUS PARADOXUS

B.DECREASING INTENSITY OF WHEEZING

C.INCREASING PaO2 and decreasing PACO2

40.CO2 retention is associated with EXCEPT

A.HYPERTENSION

B.CYANOSIS

C.INCREASE MUSCLE TONE

41.ULCER at third matatarsal in DM MOST LIKELY

A.ISCHEMIC

B.NEUROPATHY

C.INFECTION

42.A girl has c/o colicky abdominal pain associated with diarhea,tenesmus.MOST LIKELY

A.IRRITABLE BOWEL DISEASE

B.CROHN DISEASE
43.What is true regarding myocardial infarction

A.THROMBOLYTICS ARE NOT GIVEN IF ECG IS NORMAL

44.A woman has c/o depression,wt loss,constipation.o/e

serum Ca increased,serum PO4 normal,urine calcium increased,serum urea and creatinine


deranged.MOST LIKELY

A.PARATHYROID ADENOMA

B.RENAL FAILURE

C.METASTATIC DISEASE

PAEDRIATICS

1.A mother came with complain that her child is not feeding properly,is not gaining weight and
is mostly drowsy.O/E child has bilateral cataracts,is unable to gain enough weight.most likely

a.Galactosemia

b.Phenylketonuria

c.G6PDH

d ,e ??

2. A 9 days old infant with h/o forceful vomiting for 2 days.The elctrolytes are:

Na 125mmol/l

K 7.7mmol/l

CL 80mmol/l

HCO2 18mmol/l
MOST LIKELY

A.PYLORIC STENOSIS

B.CONGENITAL ADRENAL HYPERPLASIA

C.DUODENAL ATRESIA

D.GORD

3.A mother came with 6 weeks old infant h/o repeated non forceful vomiting.o/e there is weight
loss most likely

A.PYLORIC STENOSIS

B.CONGENITAL ADRENAL HYPERPLASIA

C.OESOPHAGEAL ATRESIA

D.GORD

4.A mother came with a 6 weeks old infant who has h/o repeated vomiing after meals.o/e there
is no weight loss.rest examination is normal.What is correct.

A.DO CHEST XRAY

B.NO RADIOLOGICAL INVESTIGATION IS NEEDED.

C.BLOOD CULTURE

D.URINE CULTURE

5.Parents have one child with phenylketonuria.What is the chance of their another child of
having phenyl ketonuria

A.1IN 2 REGARDLESS OF SEX

B.1 IN 4 REGARDLESS OF SEX

C.AS IN GEN POPULATION

D.1 IN 2 IF IT’S A GIRL

E.1 IN 2 IF IT’S A BOY.


6.A mother came with 3 yr old child h/o bleeding afterstool on toilet paper.o/e fecal masses r
palpable.there is anal fissure.Next immediate step

A.LAXATIVE HIGH FIBRE DIET

B.REASSURANCE IT IS NORMAL VARIANT

C.APPLICATION OF ANNUSOL CREAM ON FISSURE

D.ABD XRAY

7.Child with type 1 DM mother calls u on phone and says he is not feeling well.blood glucose is
1.1 mmol.What is immediate next step

A.GIVE HIM HIS INSULIN

B.HIGH CHO DIET

C.GIVE HIM GLUCOTHON???RUB IT ON GUMS

D.CALL AMBULANCE

E.GLUCAGON

8.School going child cannot see blackboard .He can easily see computer and uses it daily. Most
likely

A.MYOPIA

B.HYPERMETROPIA

C.AMBYLOPIA

D.STRABISMUS

9.A baby with aphyxia first line treatment

A.O2 BY MASK

B.ASPIRATION OF PHARYNX

C.INTUBATE
D E??

10.A mother with a baby of Down syndrome wants to know foture risks other than congetital
heart diseases EXCEPT

A.OESOPHAGEAL ATRESIA

B.CONDUCTIVE DEAFNESS

C.LEUKEMIAS

D.HYPOTHYROIDISM

E.EPILEPSY

11.A child came o/e periorbital edema, ascites proteinuria +++ all is true EXCEPT

A.MOSTLY HAS GOOD PROGNOSIS

B.TREATMENT OF CHOICE IS HYDROCORTISONE.

C.IF BIOPSY IS DONE IT WILL SHOW MOSTLY GLOMERULOSCLEROSIS.

D.IS ASSOCIATED WITH PNEUMOCOCAL PEROTONITIS.

12.3 YR child can do

A.HOP ON ONE LEG

B.DRESS WITHOUT SUPERVISION.

C.DRAW A RECOGNISABLE MAN

D.CLIMB STAIRS.

13.A child can sit unsupportedly,can stand with support and plays with toys around him and can
roll over by himself.Age

A.10 MONTHS

B.7 MONTHS
C.6 MONTHS

D.11 MONTHS

E .12 MONTHS

14.6 week old infant on routine examination one testis is not palpable.Next step

A.REVIEW IN 6 WEEKS

B.REVIEW IN 6 MONTHS

C.REVIEW IN 2 YRS

D.URGENT SURGICAL REFERENCE FOR ORCHIDOPEXY

15.A child with h/o vomiting mostly in morning,it is associated with headache.o/e there is
ataxia.Most likely

A.MIDDLE CRANIAL FOSSA TOMOUR

B.POST CRANIAL FOSSA TOMOUR.

C.OPTIC GLIOMA

D.NEUROBLASTOMA

16.A 6 months old baby h/o repeated jerks mostly multiple in a day.During jerks arms r flexed
and limbs r drawn up.There is decline in milestones as well.MOST LIKELY

A.PETIT MAL EPILEPSY

B.INFANTILE COLIC

C.INFANTILE SPASM

D.BENIGN SPASMS

17.All is true about Tourette syndrome EXCEPT

A.IN THIS THERE R MULTIPLE VOCAL AND MOTOT TICS.


B.CORPORALIA IS SEEN IN LESS THAN 10%

C.IT IS NOT DISTRESSING FOR PATIENT.

D.TICS OCCUR SEVERAL TIMES IN A DAY

E.ALWAYS START LESS THAN 18 YRS

18.A 3 yr old child has mild dirty wound on his scalp.Treatment of choice

A.TETANUS TOXOID

B.TETANUS IMMUNOGLOBIN

C.DTP

D.PENICILLIN

19.A 7 yr old child fell from tree now he has c/o fever,pain in knee.o/e there is tenderness in
upper part of tibia.MOST LIKELY

A.SEPTIC ARTHRITIS

B.OSTEOMYELITIS

C.FRACTURE TIBIA

D.FRACTURE PATELLA

20.Unilateral undescended testis in a child is associated with

A.INFERTILITY

B.DIRECT INGUINAL HERNIA

C.MALIGNANCY

D.HYDROCELE

E.VARICOCELE
21.10 week old child with persistent unilatetral discharge.treated with antibiotics.but it is still
recurring

A.GONOCOCCUS

B.CHLAMYDIA

C.NASOLACRIMAL DUCT OBSTRUCTION

D.FOREIGN BOBY.

22.What is normal for a infant.

A.FEMALE WITH ENLARGED CLITORIS

B.MALE WITH ENGORGED BREAST

23.An 8 yr old child o/e wt is 48 kg w/c is more than 98 percentile.height is 140 cm which is
more than 90 percentile.otherwise is normal.WOF IS CORRECT.

A.ADVANCED BONE AGE

B.INCREASED BLOOD SUGAR LEVEL

C.PRIMARY HYPERALDOSTERONISM

D.THYROTOXICOSIS

24.A boy came with h/o persistent cough and wheeze from one day.Two of the family members
have also got cough.FIRST INVESTIGATION

A.CHEST X RAY

B.SPUTUM EXAMINATION

C.FULL BLOOD COUNT

D.CHEST XRAY IN FULL INSPIRATION AND EXPIRATION

25.A 8 week old child h/o persistent cough.o/e wheeze and fine crepitations.BEST DIAGNOSTIC
TEST

A.CXR
B.SWEAT CHLORIDE TEST

C.BLOOD CULTURE

FULL BLOOD COUNT

26.Mother of 18 months old child says that the child is not babbling.audiological assesment was
done when he was 10 months old what is next step

A.REASSURANCE

B.REPEAT HEARING TEST

C.ARRANGE AUDIOMETRY

D.

27.A baby born at term collapsed in cot after 10 days o/e central cyanosis no peripheral
pulses,no murmur.most likely

A.FALLOT TETROLOGY

B.PULMONARY HYPERTENSION

C.PDA

D.LEFT HYPOPLASTIC HEART DISEASE

E.TRANSPOSITION OF GREAT VESSELS

28.A new born is born by forecep delivery.he is unable to move his forearm MOST LIKELY

A.ERBS PALSY

B.FRACTURE HUMERUS
PSHCHIATRY

1.Immature defence mech

A.HUMOR

B.ALTRUISM

C.ANTICIPATION

D.PROJECTION

E.REPRESSION

2.Parents have adopted a girl.she has SCH.they want to know the risk of their own daughter of
having SCH.

A less than 2 %

B.NIL

C.40%

D.17%

E.2.6%

3.Female throws bottles on people.How would u differentiate SCH from Delirium

A.DURATION OF SYMTOMS

B.LEVEL OF CONSIOUSNESS

C.INSIDIOUS ONSET

4.SSRI can be caused by all drugs except

A.L-TRYPTOPHAN

B.FLUOXETINE
C.HALOPERIDOL

D.CLONAZEPAM

E.MOCLEBEMIDE

5.A man has h/o grandiose ideas .MOST LIKELY

A.BORDERLINE PERSONALITY

B.NARCISSTIC PERSONALITY

C.HYPOMANIA

D.DRUG EFFECT

6.Prodomal features of SCH

A.IDEAS OF REFERENCE

B.HALLUCINATION

C.LEVEL OF CONSIOUSNESS

7.A good prognostic factor in SCH

A.PRESENCE OF AFFECTIVE DISORDER

2.LACK OF PPT FACTORS

C.

8.MENTAL HEALTH ACT.WHAT IS TRUE

A.MAY BE LIFE SAVING

B.SIMILAR IN EVERY STATE

9.A bank manager was robbed 3 weeks ago.WOF MOST LIKELY


A.IRRTIBILITY AND OUT BURST OF ANGER

B.DEPRESSION

C.PANIC ATTACK

10.Hypnogogic hallucination what is TRUE

A.DELUSION

B.SCH

C.IT CAN HAPPEN IN NORMAL SLEEP

D.ALCOHOLISM

11.In memtal disorders people mostly have violence.Who r affected most by this violence

A.PARENTS

B.SIBLINGS

C.STRANGERS

D.THEMSELVES

12.A patient with suicide risk all is imp EXCEPT

A.ANY RECENT SUICIDAL PLAN

B.POOR FAMILY SUPPORT

C.UNEMPLOYMENT

13.Proverb explanation is done in assessment of

A.DYSTHYMIA

B.SCH

C.DEMENTIA

D.ANXIETY
14.Woman 35 yr old h/o she feels detached from her husband.She has 2 children and is living in
rented home she cannot afford to go for holidays.has h/o menorragia her hb was given???Next
step

A.REFER HER TO PSYCHIATRIST

B.SEND HER TO MARRIAGE COUNCILOR

C.REFER HER TO GYNAECOLOGIST

D.ADVISE HER TO HAVE HOLIDAYS

E.HAVE A INTERVIEW WITH HER HUSBAND

15.A woman comes she is concerned about her husband.he is very detached from her,he is
getting up earlier than his usual time.APPROPRIATE TREATMENT

A.BEZO DIAZEPINES

B.COGNITIVE BEHAVIOR THEARPY

C.REFER TO PSYCHOTHEARAPIST.

16.In panic attack except

A.OCCUR IN BOTH MALES AND FEMALES

B.AVIDANCE OF SPECIFIC FACTORS IS RECOMMENDED DURING TRAETMENT.

17.De ja vu is found in

A.FRONTAL LOBE TUMORS

B.TEMPORAL LOBE TUMORS

C.PARIETAL LOBE TUMORS

GYNAECOLOGY & OBSTETRICS

1.COC work at level of


A.OVARY

B.HYPOTHALAMUS

C.PITUITARY

D.ENDOMETRIUM

E.CERVICAL MUCUS

2.The most appropriate OCP for a 24 yr old on phenytoin is

A.MICROGYNON 30

B.MICROGYNON50

C.DIANE 35

D.CLOMIPHENE

3.Depoprovera has all s/e EXCEPT

A.ALTERED MENSTRUAL CYCLE LEADING TO AMENORHEA

B.DEPRESSION

C.HYPERTENSION

D.WEIGHT GAIN

4.COC decreases risk EXCEPT

A.DECREASE RISK OF COLON CANCER

B.DECREASES RISK OF CEREBROVASCULAR DIDEASES

C.DECREASE RISK OF ENDOMETRIAL CANCER

D.DECREASE RISK OF OSTEOPOROSIS

E.DECREASE RISK OF OVARIAN CYSTS

5.Absolute c/I for OCPS


A.SUPERFICIAL THROMBOPHELIBITIS

B.MIGRAINE WITH AURA

C.VARICOSE VEINS

D.

6.Ovarian tumor o/e there is

A.SHIFTING DULLNESS LATERALLY

B.DULLNESS ANTERIORLY AND RESONANCE LATERALLY

C.RESONANCE ANTERIORLY

7.A woman with mastalgia.Not responding to conventional treatment.Next step

A.BROMOCRIPTINE

B.OCPS

C.DANAZOL

8.21 year old c/o abd pain .lmp was 2 weeks ago.on usg there r 3mm,4mm,2.2cm cystic
structures.What is next step in management

A.REPEAT USG IN 4 WEEKS

B.LAPROSCOPY AND DRAIN CYST

C.OCP

D.IGNORE FINDINGS

9.A pregnant female presents to u at 38 weeks with 1000ml blood loss.WOF is not related to
placental abruption

A.FETAL DEATH

B.TENSE UTERUS
C.BP180/110

D.FETAL HEAD HIGH UP AND FREELY MOBILE

E.NORMAL PULSE RATE

10.Anti D should be given to Rh-ve mother in w/c situation

A.BETWEEN 28 TO 34 WEEKS

B.MUST BE GIVEN WITHIN 72 HRS OF DELIVERY

C.SHOULD BE GIVEN TO ALL MOTHERS.

11.26 yr old primigravida,ht 150cm,at 35 weeks.is in labour.In your assessment pelvis is


reduced,cervix is 4 cm dilated.head is at station 0.What is your mx

A.CONTINUE TRIAL OF LABOUR

B.INDUCE WITH OXYTOCIN

C.XRAY PELVIMETRY

D.CSECT

12.Effect of progesterone in pregnancy

A.DILATATION OF URETERS

B.DILATATION OF CALYCES

C.DILATATION OF URETERS AND CALYCES.

13.28 YR old primigravida at 16 weeks with BP 180/90 PROTEIN IN URINE+++most likely

A.PREECLAMPSIA

B.ESSENTIAL HTN

C.PRE EXISTING RENAL DISEASE.


14.With regard to carpel tunnel in pregnancy WOF is true

A.RARELY REQUIRES SURGERY

B.SEVERE PAIN IN THENAR EMINENCE.

15.About twin delivery ,after the delivery of first u will do all except

A.PALPATE ANOTHER BABY

B.PER VAGINAL EXAMINATION

C.DO A EXTERNAL CEPHALIC VERSION

D.LOOK ANY BLOOD COME OUT

E.IF PAIN DOES NOT START INDUCE HER WITH OXYTOCIN.

16.About oxytocin

A.IT HAS ANTI DIURETIC EFFECT

B.

17.A lady is 8 week pregnant.She has c/o vaginal bleeding o/e os is open o/e uterus is 12weeks
in size.What is true in management

A MEASURE HCG REGULARLY UNTIL IT BECOMES NEGATIVE.

18.A woman at 18 weeks is complaining of abdominal pain LEAST likely

A.ACUTE APPENDICITIS

B.ECTOPIC PREGNANCY

CINEVITABLE ABORTION

19.A woman has come in her second pregnancy.In her previous pregnancy she delivered baby
weighing 4 kg and it was a difficult delivery and baby has a fracture clavicle.Y ou should do all
EXCEPT
A.INDUCTION OF LABOUR AT 37-38 WEEKS

B.DO C SECT AT TERM

C.CARE OF DM IS IMP AFTER 20-30 WEEKS ONLY.

20.A 19 yrs old primigravida at 36 weeks with hyperreflexia,htn,edema might be expected to


have all elevated EXCEPT

A.LIVER ENZYMES

B.URIC ACID

C.CREATININE

D.PLATELETS

E.BUN

21.A postmenopausal woman used to have withdrawal bleeding but not any more.LIKELY CAUSE

A.OVARY

B.UTERUS

C.PITUITARY

D.HYPOTHALAMUS

22.All of folowing could be the cause of greenish yellow vafinal discharge EXCEPT

A.TRICHOMONAS VAGINALIS

B.BACTERIAL VAGINISIS

C.CA CERVIX

D.FOREIGN BODY

E.CHALMYDIAL URETRITIS

23.Primary dyemenorrhea AOF r used in treatment EXCEPT


A.PROSTAGLANDIN INHIBITORS

B.INDOMETHACIN

C.BROMOCRIPTINE.

D.OCP

24.Secondary ovarian tumor r mostly due to

A.CA COLON

B.CA STOMACH

25.A woman has complain of Postmenopausal bleeding.o/e ovarian mass is present.D&C shows
endometrial hyperplasia.MOST LIKELY

A.CYST

B.TERATOMA

C.GRANULOSA CELL TUMOR

26.A pt at 26 weeks has herpes simplex.What is management EXCEPT

A.CSECT AT TERM.

B.GIVE HER ACYCLOVIR

C.PRESENCE OF HS PRECLUDES VAGINAL DELIVERY AT TERM

27.A pregnant female at 26 weeks .her daughter has fever and red cheeks.What is TRUE

A.CHECK MATERNAL RUBELLA ANTIBODY

B.DO HER COUNCILING REGARDING DISEASE.

C.DO BLOOD TESTS ON DAUGHTER.

28 Recurrent abortion.MOST LIKELY


A.ANTICARDIOLIPIN ANTIBODY

29.Spontaneous abortion at 6-10 weeks.MOST LIKELY

A.CHROMOSOMAL ABNORMALITY

30.A woman has h/o recurrent abortion at 6-10 weeks.MOST LIKELY

A.CERVICAL INCOMPETENCE

B.FETAL ANOMALY

C.POOR DIET

31.USG at 6-10 weeks is done fior all EXCEPT

A.INEVITABLE ABORTION

B.CONGENITAL MALFORMATION

32.NTD AOF is correct EXCEPT

A.GIVE FOLIC ACID IN PREGNANCY

B.LOW ALPHA FETO PROTEIN

33.A woman has inevitable abortion,histopathology shows decidualization.NEXT STEP

A.REASSURANCE

B.READMIT FOR HYSTERCROMY

C.DO AGAIN D& C

D.READMIT FOR LAPROSCOPY

34.Anorexia nervosa EXCEPT

A.ANEMIA

B.AMENOHEA
C.LAUNGO HAIR???

35.A young girl came with abdomonal pain o/e there are 2mm,4mm,2.2cm cysts.NEXT STEP

A.REPEAT USG IN 6 WEEKS

B.LAPROSCOPIC ASPIRATION OF CYST

C.LAPROTOMY

D.REASSURANCE

PAPER COMPILED BY: AYESHA MALIK

PAPER : JULY 2005C

TOTAL QUESTIONS RECALLED=157/250

PICTURES=8

PEADIATRICS=28

PSYCHAITRY=17

OBS & GYNAE=35

MEDICINE=44

SURGERY=25

NOTE.:THERE R SOME QUESTIONS OF WHOM I DO NOT REMEMBER ALL OPTIONS…SO U CAN


LOOK FOR THEM

PICTURES

1.PIC with plaques on it.

A.PSORIASIS
2.PIC of scalp showing a patch of hair loss.some crusting,redness,broken hair

A.ALOPECIA AERATA

B.TINEA CAPITIS

C.PSORIASIS

3.PIC of a lesion on dorsum of hand

A.KERATOCANTHOMA

B.INFECTED SEBECOUS CYST

C.MALIGNANCY

4.PIC of streptococcal cellulitis.Treatment

A.PENICILLIN

B.FLUCLOXACILLIN AND AMPICILLIN

5.PIC of swan neck deformity.WHAT IS TRUE

A.MORE IN RHEUMATOID ARTHRITIS AS COMP TO DEGENERATIVE OSTEOARTHRITIS

B.IT IS ALMOST ALWAYS ASSOCIATED WITH ULNAR DEVIATION

C.CAUSED BY RUPTURE OF DISTAL EXTENSOR TENDON

D.CAUSED BY RUPTURE OF PROXIMAL FLEXOR TENDON

6.PIC of displaced colles fracture manegement

A.PLASTER IMMOBILIZATION FOR ATLEAST 10 WEEKS

B.CLOSE REDUCTION????

7.PIC of diffuse swelling in thyroid gland MOST LIKELY


A.THYROTOXICOSIS

B.MUTINODULAR GOITRE

8.PIC OF SKIN LESIONS MOST LIKELY ASSOCIATED WITH

A.STREPTOCOCCAL PNEMONIA

B.LEGIONELLA PNEUMONIA

C.MYCOPLSMA PNEUMONIA

D.TUBERCULOSIS

SURGERY

1.Which nerve is likely injured in spiral fracture of humerus

A.ULNAR NERVE

B.RADIAL NERVE

C.MEDIAN NERVE

2.In uncomplicated mucosal hemmorids EXCEPT

A.FRESH BLEEDING

B.MUCOUS DISCHARGE

C.PRURITIS ANI

D.PERIANAL PAIN
3.The most common cause of bloody discharge from nipple is

A.BREAST CA

B.INTRADUCTAL PAPILLOMA

C.TRAUMA

4.WOF is not related to breast cancer

A.CONTRALATERAL BREAT CA

B.FAMILY HISTORY

C.UTERINE CA

D.ARTIFICIAL MENOPAUSE

5.CORRECT about mammography

A.PAINLESS

B.CAN DETECT BETTER THAN SELF EXAMINATION

6.MAMMOGRAPHY is mostly helpful for

A.WOMEN OVER 35 YRS

B.50 –60 YRS

C.LESS THAN 30 YRS

D.WITH FAMILY HISTORY OF BREAST CANCER

7.The most likely for a male who is 22yrs old having an inguinoscrotal swelling which disappears
on lying down

A.VARICOCELE

B.SAPHENA VARIX
C.LYMPHOMA

D.DIRECT INGUINAL HERNIA

8.In management of compound fracture of tibia and fibula MOST CORRECT is

.A.IF NO CONTAMINATION DEBRIDEMENT IS UNNECESSARY

B.EXTERNAL FIXATION

C.ANTI TETANUS PLUS ANTIBIOTICS

D.DEBRIDEMENT AND INTERNAL FIXATION

9.A fistula is

A.OPENING BETWEEN TWO EPITHELIAL SURFACES

10.FASCIAL nerve palsy LEAST likely in

A.ACOUSTIC NEUROMA

B.MASTOIDITIS

C.BASE OF SKULL FRACTURE

D.CHRONIC PAROTITIS

11.Immediate manegement of tension pneumothrorax

A.INTERCOSTAL TUBE DRAINAGE

B.WIDE BORE NEEDLE AT 2ND INTERCOSTAL SPACE ANTERIORLY

C.ENDOTRACHEAL TUBE

12.An old man c./o colicky abd pain which has become generalized now,o/e bowel sounds r
absent,on p/r reddish tinge on fingerMOST LIKELY

A.DIVERTICULOSIS
B.CA RECTUM

C.MESENTERIC ARTERY OCCLUSION

13.A pt mother has colon cancer at 65 yrs,he is 45 yrs old.he thinks he also has colon
cancer.WOF IS TRUE

A.HE WILL HAVE ANNUAL COLONOSCOPIES FROM 50 YRS ONWRDS

B.HE SHOULD HAVE FECAL OCCULT BLOOD TESTING TWO YEARLY

C.BARIUM ENEMA

D.HE SHOLUD HAVE COLONOSCOPY

E.REASSURANCE

14.IN intestinal obstruction what is correct

A.2000 ML OF HARTMAN SOL BEFORE OPERATION

B.2000 ML HARTMAN SOL DURING OPERATION

C.5%DEXTROSE WATER BEFORE OPERATION

15.A ot has undergone craniotomy his electrolytes r

Na 168 , serum osmollality200 mlMOST LIKELY

A.SIADH

B.DIABETES INSIPIDUS

C.WATER INTOXICATION

16.A pt is admitted in hospital has electrolytes

Na 117,Cl 86,K 3.2,serum osm 900

A.DI

B.SODIUM DEPLETION
C.SIADH

17.In intestinal surgery when to give antibiotics

A.ONE DAY BEFORE SURGERY

2.DURING SURGERY

C.WHEN ABDOMEN IS CLOSED

D.2 HRS BEFORE SURGERY

18.Acute pain in abdomen LEAST LIKELY

A.MYOCARDIAL INFARCTION

B.PERFORATED DUODENAL ULCER

C.ACUTE APPENDICITIS

D.

19.Unilateral exopthalamus MOST LIKELY

A.CA MAXILLARY ANTRUM

B.THYROTOXICOSIS

C.INJURY IN EYE????

20.Commonest cause of diarhea in bed ridden patients

A.FECAL IMPACTION

B.CA RECTUM

21.A woman has h/o tingling at night in her right hand o/e flexion is weak there is wasting of
abductor pollicis MOST LIKELY

A.CARPEL TUNNEL SYNDROME


B.ULNAR NERVE WEAKNESS

22.Postope rative pt who was underwent surgery on hip ???/ is now complaining of unilateral
weakness.u have diagnosed a nerve compression.MOST LIKELY CAUSE IS

A.FLEXOR HALLUCIS LONGUS WEAKNESS

B.SOLEUS

C.TIBIALIS POSTERIOR

23.A pt came with trauma o/e iplilateral pupil is dilated,bradycardia,BP I (I am not sure)MOST
LIKELY

A.SUBDURAL HEMATOMA

B.EXTRADURAL HEMATOMA

C.SUBARACHONOID HEMORRAGE

24.AOF is true regarding gall bladder stones EXCEPT

A.THEY R MOSTLY RADIOLUCENT

B.SINGLE STONE IS MOSTLY OF CHOLESTEROL

C.MOSTLY ASYMPTOMATIC

D.PIGMENT STONES R HEMOLYTIC

E.STONE AT CBD CAUSES SEPSIS OF GALL BLADDER

25.PT h/o vomiting ,some wt loss.has past h/o duodenal ulcer.MOST LIKELY

A.ACHALASIA CARDIA

B.CA DUODENUM
MEDICINE

ENT

1.About nasophryngeal carcinoma MOST LIKELY

A.BENIGN BUT LOCALLY DESTRUCTIVE

B.ASSOCIATED WITH EBV ANTIBODIES

2.Otosclerosis MOST LIKELY

A.BLUISH TINGE ON TYMPANIC MEMBRANE

B.SESORINEURAL DEAFNESS

C.NORMAL TYMPANIC MEMBRANE

3.PT with acute onset of vertigo,tinnitus no hearing loss.MOST LIKELY

A.MENEIRE DISEASE

B.VESTIBULAR NEURONITIS

C.ACUTE LABYRYNTITIS

D.ACOUSTIC NEUROMA

4.A child comes 3yr old h/o room strats moving in a circle and then he falls down .MOST
CORRECT STATEMENT IS

A.EEG WILL BE CONFIRMATORY

B.START PHENYTOIN
C.BENIGN POSITIONAL VERTIGO W/C IMPROVES WITH AGE.

LIVER.MOST OF MCQS WERE FROM LIVER

5.Diagnostic test for hemochromatosis

A.SERUM FERRITIN

B.SERUM IRON

C.SERUM TRANFERRIN SATURATION

6.A man has h/o impotence,cirrhosis.His father died b/c of same cirrhosis.RULE OUT

A.HEMOCHROMATOSIS

B C D ????

7.A woman came with c/o pain in epigastrium last evening.today morning she passsed dark
color urine.her lfts r deranged ,alk po4 is markedly increased.MOST LIKELY

A.BILIARY COLIC

B.ACUTE PANCREATITIS

C.CHOLEDOCHOLITHIASIS

8.Student 18 yr old is feeling vague has icteric eyes.o/e bibirubin is increased.rest of the
examination is normal.MOST LIKELY

A.GILBERT SYNDROME

B.HEPATITIS A

C.HEPATITS B

9.A woman had cholecystectomy 3 months ago now she is having right upper quadrant
pain.MOST LIKELY
A.STONE LEFT AT CBD

10.Hepatitis likely to become chronic

A.HEP A

BHEP C

C.HEP B

D.HEP E

MCQS FROM OTHER MISC.TOPICS

11.Hyperkalemia immediate NEXT STEP

A.GIVE SALBUTAMOL

B.CA RESONIUM

C.DIALYSIS

D.CALCIUM GLUCONATE

E.GLUCOSE AND INSULIN

12.ORG involved in food poisoning

A.E COLI

B.STAPH AUREUS

C.SALMONELLA

13.A pt sedative overdose what is typical blood picture

A.PH 7.35 PO2 70 PCO2 80

B.PH 7.1 PO2 70 PCO2 80

14.Which tumor never metastasize to brain.


A.BREAST CA

B.LUNG CA

C.PANCREATIC CA

D.PROSTATE CA

15.A bisexual man has several mouth ulcers ,arthritis,sausage fingers.also has h/o diarhea.MOST
LIKELY

A.BEHCET DISEASE

B.REITER DISEASE

C.RHEUMATOID ARTHRITIS

D.ANKOLYSING ARTHRITIS

E.PSORIATIC ARTHRITIS

16. A man has h/o unilateral headache,mostly occurs in morning.h/o lacrimation.DRUG OF


CHOICE

A.ERGOTAMINE

B.PROPRANOLOL

C.CARBAMAZEPINE

D.ATROPINE

17.Postsplenectomy hereditary spherocytosis.AOF is true EXCEPT

A.OSMOTIC FRAGILITY

B.ANEMIA PERSISTENCE

C.NORMAL LIFE SPA OF RBC

D.PRESENCE OF SPHEROCYTES IN PERIPHERAL BLOOD

18.Alcoholic ketoacidosis EXCEPT


A.NORMAL ANION GAP

19.A pt is brought to ER he is confused,ataxic,nystagmus,diplopia,…has h/o alcohol.MOST


LIKELY

A.ALCOHOLIC HALLUCINOSIS

B.WERNICKE ENCEPHLOPATHY

20.Treatment of wernicke encephlopathy with low plasma sugar

A.THIAMINE

B.THIAMINE WITH GLUCOSE

C.IV 5%GLUCOSE

D.VIT B1

21.Ankolysing spondylitis LEAST ASSOCIATED

A.AORTIC INCOMPETENCE

B.SERO –VE ARTHRITIS

22.A middle aged woman has c/o mouth ulcers,morning stiffness of joints…it gets better with
day….????MOST LIKELY

A.SLE

B.RHEUMATOID ARTHRITIS

23.Alveolar exchange difference

PaO2=?

PaCO2=?

Inspiratory O2=?
A.9

B12…..????

24.B/L weakness of hand muscles

A.MULTIPLE SCLEROSIS

B.ULNAR NERVE LESION

C.SYRINGOMYELIA

25.Weakness of all muscles unilateral in hand

A.SYRINGOMYELIA

B.?????

26.PT with atrial fibrillation what is CORRECT

A.WARFARIN SHOULD BE GIVEN

B.RISK OF STROKE????

27.Diff between hemolytic anemia and anemia due to chronis blood loss

A.DECREASED HAPTOGLOBIN

B.POLYCHROMASIA

28.Question on alpha 1 deficiency

27.Amouroxis fugax CORRECT

A.IPSILATERAL INTERNAL CAROTID ARTERY STENOSIS

29.Opening snap in mitral stenosis denotes

A.DIMINISH WITH ATRIAL FIBRILLATION


B.DUE TO VALVE MOBILITY

30.A child with suspicion of meningitis,LP lymphocytes increased,????MOST LIKELY

A.ECHO VIRUSES

B.BACTERIAL

31.Polycythemia and cor pulmonale MOST LIKELY ASSOCIATED WITH

A.TUBERCULOSIS

B.BRONCHIACTASIS

C.EMPHYSEMA

32.A man with h/o flaccid paralysis of lower limb over a week.o/e reflexes r diminished,decrease
sensation of touch,???? MOST LIKELY

A.ACUTE POLYMYOSITIS

B.ACUTE POLYNEUROPATHY

33.Amylodosis can complicate all EXCEPT

A.TUBERCULOSIS

B.LEPROSY

C.MULTIPLE MYELOMA

D.CHRONIC ACTIVE HEPATITIS.

E.RHEMATOID ARTHRITIS

34.A pt came with fever,o/e lyphadenopathy,splenomegaly,on bllod report

abnormal lfts,atypical lymphocytes r presentMOST LIKELY

A.INFECTIOUS MONONUCLEOSIS
35.A female h/o enuresis,2 sisters also have same problem MOST LIKELY

A.HORSE SHOE KIDNEY

B.HERIDITARY NEPHRITIS

C.VESICOURETERIC REFLUX

36.Heparin is measured by

A.PT

B.APTT

37.BEST method of detecting H PYLORI eradication

A.UREA BREATH TEST

38.In manegement of COPD recent improvement in health is due to

A. HOME OXYGEN THERAPY


39.ROLE OF T LYMPHOCYTES

29.Indicator of good prognosis in asthma

A.INCRESING PULSUS PARADOXUS

B.DECREASING INTENSITY OF WHEEZING

C.INCREASING PaO2 and decreasing PACO2

40.CO2 retention is associated with EXCEPT

A.HYPERTENSION

B.CYANOSIS

C.INCREASE MUSCLE TONE

41.ULCER at third matatarsal in DM MOST LIKELY

A.ISCHEMIC

B.NEUROPATHY

C.INFECTION

42.A girl has c/o colicky abdominal pain associated with diarhea,tenesmus.MOST LIKELY

A.IRRITABLE BOWEL DISEASE

B.CROHN DISEASE

43.What is true regarding myocardial infarction

A.THROMBOLYTICS ARE NOT GIVEN IF ECG IS NORMAL


44.A woman has c/o depression,wt loss,constipation.o/e

serum Ca increased,serum PO4 normal,urine calcium increased,serum urea and creatinine


deranged.MOST LIKELY

A.PARATHYROID ADENOMA

B.RENAL FAILURE

C.METASTATIC DISEASE

PAEDRIATICS

1.A mother came with complain that her child is not feeding properly,is not gaining weight and
is mostly drowsy.O/E child has bilateral cataracts,is unable to gain enough weight.most likely

a.Galactosemia

b.Phenylketonuria

c.G6PDH

d ,e ??

2. A 9 days old infant with h/o forceful vomiting for 2 days.The elctrolytes are:

Na 125mmol/l

K 7.7mmol/l

CL 80mmol/l

HCO2 18mmol/l

MOST LIKELY

A.PYLORIC STENOSIS

B.CONGENITAL ADRENAL HYPERPLASIA


C.DUODENAL ATRESIA

D.GORD

3.A mother came with 6 weeks old infant h/o repeated non forceful vomiting.o/e there is weight
loss most likely

A.PYLORIC STENOSIS

B.CONGENITAL ADRENAL HYPERPLASIA

C.OESOPHAGEAL ATRESIA

D.GORD

4.A mother came with a 6 weeks old infant who has h/o repeated vomiing after meals.o/e there
is no weight loss.rest examination is normal.What is correct.

A.DO CHEST XRAY

B.NO RADIOLOGICAL INVESTIGATION IS NEEDED.

C.BLOOD CULTURE

D.URINE CULTURE

5.Parents have one child with phenylketonuria.What is the chance of their another child of
having phenyl ketonuria

A.1IN 2 REGARDLESS OF SEX

B.1 IN 4 REGARDLESS OF SEX

C.AS IN GEN POPULATION

D.1 IN 2 IF IT’S A GIRL

E.1 IN 2 IF IT’S A BOY.

6.A mother came with 3 yr old child h/o bleeding afterstool on toilet paper.o/e fecal masses r
palpable.there is anal fissure.Next immediate step
A.LAXATIVE HIGH FIBRE DIET

B.REASSURANCE IT IS NORMAL VARIANT

C.APPLICATION OF ANNUSOL CREAM ON FISSURE

D.ABD XRAY

7.Child with type 1 DM mother calls u on phone and says he is not feeling well.blood glucose is
1.1 mmol.What is immediate next step

A.GIVE HIM HIS INSULIN

B.HIGH CHO DIET

C.GIVE HIM GLUCOTHON???RUB IT ON GUMS

D.CALL AMBULANCE

E.GLUCAGON

8.School going child cannot see blackboard .He can easily see computer and uses it daily. Most
likely

A.MYOPIA

B.HYPERMETROPIA

C.AMBYLOPIA

D.STRABISMUS

9.A baby with aphyxia first line treatment

A.O2 BY MASK

B.ASPIRATION OF PHARYNX

C.INTUBATE

D E??
10.A mother with a baby of Down syndrome wants to know foture risks other than congetital
heart diseases EXCEPT

A.OESOPHAGEAL ATRESIA

B.CONDUCTIVE DEAFNESS

C.LEUKEMIAS

D.HYPOTHYROIDISM

E.EPILEPSY

11.A child came o/e periorbital edema, ascites proteinuria +++ all is true EXCEPT

A.MOSTLY HAS GOOD PROGNOSIS

B.TREATMENT OF CHOICE IS HYDROCORTISONE.

C.IF BIOPSY IS DONE IT WILL SHOW MOSTLY GLOMERULOSCLEROSIS.

D.IS ASSOCIATED WITH PNEUMOCOCAL PEROTONITIS.

12.3 YR child can do

A.HOP ON ONE LEG

B.DRESS WITHOUT SUPERVISION.

C.DRAW A RECOGNISABLE MAN

D.CLIMB STAIRS.

13.A child can sit unsupportedly,can stand with support and plays with toys around him and can
roll over by himself.Age

A.10 MONTHS

B.7 MONTHS

C.6 MONTHS

D.11 MONTHS
E .12 MONTHS

14.6 week old infant on routine examination one testis is not palpable.Next step

A.REVIEW IN 6 WEEKS

B.REVIEW IN 6 MONTHS

C.REVIEW IN 2 YRS

D.URGENT SURGICAL REFERENCE FOR ORCHIDOPEXY

15.A child with h/o vomiting mostly in morning,it is associated with headache.o/e there is
ataxia.Most likely

A.MIDDLE CRANIAL FOSSA TOMOUR

B.POST CRANIAL FOSSA TOMOUR.

C.OPTIC GLIOMA

D.NEUROBLASTOMA

16.A 6 months old baby h/o repeated jerks mostly multiple in a day.During jerks arms r flexed
and limbs r drawn up.There is decline in milestones as well.MOST LIKELY

A.PETIT MAL EPILEPSY

B.INFANTILE COLIC

C.INFANTILE SPASM

D.BENIGN SPASMS

17.All is true about Tourette syndrome EXCEPT

A.IN THIS THERE R MULTIPLE VOCAL AND MOTOT TICS.

B.CORPORALIA IS SEEN IN LESS THAN 10%

C.IT IS NOT DISTRESSING FOR PATIENT.


D.TICS OCCUR SEVERAL TIMES IN A DAY

E.ALWAYS START LESS THAN 18 YRS

18.A 3 yr old child has mild dirty wound on his scalp.Treatment of choice

A.TETANUS TOXOID

B.TETANUS IMMUNOGLOBIN

C.DTP

D.PENICILLIN

19.A 7 yr old child fell from tree now he has c/o fever,pain in knee.o/e there is tenderness in
upper part of tibia.MOST LIKELY

A.SEPTIC ARTHRITIS

B.OSTEOMYELITIS

C.FRACTURE TIBIA

D.FRACTURE PATELLA

20.Unilateral undescended testis in a child is associated with

A.INFERTILITY

B.DIRECT INGUINAL HERNIA

C.MALIGNANCY

D.HYDROCELE

E.VARICOCELE

21.10 week old child with persistent unilatetral discharge.treated with antibiotics.but it is still
recurring

A.GONOCOCCUS
B.CHLAMYDIA

C.NASOLACRIMAL DUCT OBSTRUCTION

D.FOREIGN BOBY.

22.What is normal for a infant.

A.FEMALE WITH ENLARGED CLITORIS

B.MALE WITH ENGORGED BREAST

23.An 8 yr old child o/e wt is 48 kg w/c is more than 98 percentile.height is 140 cm which is
more than 90 percentile.otherwise is normal.WOF IS CORRECT.

A.ADVANCED BONE AGE

B.INCREASED BLOOD SUGAR LEVEL

C.PRIMARY HYPERALDOSTERONISM

D.THYROTOXICOSIS

24.A boy came with h/o persistent cough and wheeze from one day.Two of the family members
have also got cough.FIRST INVESTIGATION

A.CHEST X RAY

B.SPUTUM EXAMINATION

C.FULL BLOOD COUNT

D.CHEST XRAY IN FULL INSPIRATION AND EXPIRATION

25.A 8 week old child h/o persistent cough.o/e wheeze and fine crepitations.BEST DIAGNOSTIC
TEST

A.CXR

B.SWEAT CHLORIDE TEST

C.BLOOD CULTURE

FULL BLOOD COUNT


26.Mother of 18 months old child says that the child is not babbling.audiological assesment was
done when he was 10 months old what is next step

A.REASSURANCE

B.REPEAT HEARING TEST

C.ARRANGE AUDIOMETRY

D.

27.A baby born at term collapsed in cot after 10 days o/e central cyanosis no peripheral
pulses,no murmur.most likely

A.FALLOT TETROLOGY

B.PULMONARY HYPERTENSION

C.PDA

D.LEFT HYPOPLASTIC HEART DISEASE

E.TRANSPOSITION OF GREAT VESSELS

28.A new born is born by forecep delivery.he is unable to move his forearm MOST LIKELY

A.ERBS PALSY

B.FRACTURE HUMERUS

PSHCHIATRY
1.Immature defence mech

A.HUMOR

B.ALTRUISM

C.ANTICIPATION

D.PROJECTION

E.REPRESSION

2.Parents have adopted a girl.she has SCH.they want to know the risk of their own daughter of
having SCH.

A less than 2 %

B.NIL

C.40%

D.17%

E.2.6%

3.Female throws bottles on people.How would u differentiate SCH from Delirium

A.DURATION OF SYMTOMS

B.LEVEL OF CONSIOUSNESS

C.INSIDIOUS ONSET

4.SSRI can be caused by all drugs except

A.L-TRYPTOPHAN

B.FLUOXETINE

C.HALOPERIDOL

D.CLONAZEPAM

E.MOCLEBEMIDE
5.A man has h/o grandiose ideas .MOST LIKELY

A.BORDERLINE PERSONALITY

B.NARCISSTIC PERSONALITY

C.HYPOMANIA

D.DRUG EFFECT

6.Prod

anne123, Oct 17, 2006

#2

anne123

anne123 Guest

dedicated to all members of this forum.....

MCQ 2006 March, Brisbane

1 After a carotid massage the patients heart rate goes down 50%. WOF

a. VT

b. SVT

c. atrial fibrillation

d. Atrial flutter
2 A 52 yr old obese, male with a BMI of 31,complains of a nocturnal cough .It’s a dry cough,
worse after eating. Also has a H/O URTI. The most likely possibility..

a. Postnasal drip

b. Chronic bronchitis

c. Asthma

d. GORD

3. A 25 yr old construction worker suffers from an L5-L4 disc prolapse after lifting a heavy object
at work. The best Investigation

a. CT

b. Myelogram

c. MRI

d. Ultrasound

4. ECG. The diagnosis

a. COPD

b. cardiomyopathy

c. RBBB

5. Which of the following is an indication for throbolysis.

a. RBB

b. LBB

6.A man develops bilateral,small muscle wasting of the hands with loss of pain but no loss of
touch. ( dissociated sensory loss).WOF could be the cause?

a. syringomyalia

b. MND
c. Transverse myelitis

d. Gullien –Barre

7. A 69 yr old patient suffers unilateral osteoarthritis of the hip.On which side should the
supportive walking stick be used.

a. Stick on the contralateral side

b. Stick on the ipsilateral side

8. A 9 month old male child, presents with vomiting and diarrhea. He has also passed blood
stained stools and is suffering from dehydration.(7%). The initial investigation

a. ELISA

b. stool culture

c. Laparotomy

d. Air contrast enema

9. A 31 yr old primipara with 10 weeks amenorrhea , comes with no medical, surgical or


psychiatric problems. Which of the following positive tests will decide a change in her
management.

a. HIV

b. Rubella

c. CMV

d. Syphilis

e. Blood sugar level


10. Cystic Fibrosis is diagnosed in a 5 week old child. What would you discuss in regards to
genetic councelling with the parents about the fate of a second pregnancy.

a. There is a 1;4 chance of tdevelopment of CF.

b. There is a 1;4 chance of child being a carrier

c. There is a 1;2 chance of being normal

d. Prevelence in the community is 1 :2500, Incidence in the gene is 1:25, Every child born to this
couple has a 1:4 chance of being affected

11 A 42 yr old man,has a 65 yr old mother who has recently been diagnosed with colon cancer.
What advice would you give him.

a. Feccal occult blood testing for 2 yrs

b. Colonoscopy after 50yrs

c. Sigmoidoscopy after 50 yrs

d. Sigmoidoscopy and colonoscopy after 45 yrs of age

12. A patient presents 4 weeks post acute pancreatittis, and their Serum Amylase is still raised.
What is the most likely DX

a. Cholilithiasis

b. chronic pancreatitis

c. pancreatic duct obstruction

d. pseudo pancreatic cyst

13. What is the most commonest complication of ERCP

a. haemmorrahage in the duodenum

b. duodenal perforation

c. pancreatitis
14. Routine Ct scan on a 62 yr old male,shows an AAA of 6.3 cms.The next step in MX.

a. Angigram and angioplasty

b. Repeat CT in 6 months

c. US

d. Anti- hypertensives

15. A 24 yr old male on a routine insurance examination shows an increase of urine protein ( 2+)
but no other significant findings.What is the initial management?

a. IVP

b. US renal

c. Repeat testing of an early morning urine sample.

16. Which nerve is responsible for impaired extension of the wrist.

a. Median n

b. Radial n

c. Ulnar n

d. Post interiouse n

e. Ant interiouseis n

17. A 65 yr old male patient with a history of MI , was treated with a stent and is currently taking
CLOPIDOGREL. He suffers a femoral artery aneurysm.What is the initial MX?

a. Stop clopidogrel

b. tight band application

c. Artery repair

18. What is the long term outcome in a patient with an Antisocial personality disorders?
a. 75 % end up in jail

b. 50 % return to normal

c. 25 % die in jail

d. 30 commit suicide

19. The commonest complication of drug toxicity with anti psychotics on day1/2?

a. Akathisia

b. Dyskinesia

c. Dystonia

d. Chorea

20. Breast feeding as a means of contraception includes all the following benefits, except.

a. affective upto 6 months

b. Baby not allowed to go on solid foods

c. Failure rate more then 3 per 100 women years

21. A young male, a machine tool operator, complains of restriction of movement and pain in
the dorsum of the hand .OE there is a tender palpable swelling on the dorsum of the hand. What is
the initial investigation.

a. D diamer

b. Venogram

c. US dopler

d. XRY

22. A 9yrs old boy initatied a fire after lighting a match. He suffers sever burns of the face ,with
blackening ( soot ) on the facial area. What is the initial MX.

a. intubation
23. A young man presents with 15% burns.What is the fluid replacement for the next 24hrs?

24. A 62 yr old man presents with vertigo, horners syndrome ,vomiting and ataxia. What is the
site of lesion?

a. a.Ant cerebral artery

b. b perforating arterys

c. c.Vertebral artery

d. d.MCA

25. A 21 yr old lady previously normal patient complains of Trigeminal neuralgic pain, with
numbness of the left half of upper lip. No previous neurological deficeits prior to onset.What is the
DX?

a. MS

26. A patient presents with a pnemothorax..What is the initial management?

27. A male presents with diplopia and photophobia. Pupils are normal and reflexes are intact.
What could be the cause?

a. DM

28. What is the best investigation for the detection of gallbladder pathology?

a. US

b. ERCP

c. CT scan
29. A child presents with clinical features of URTI. He is allergic to penicillin. What is the
treatment of choice?

a. a.roxythromycin

b. b.cephalexin

c. c.augmentin

30. A 36yr old women comes with the complaint of stress incontinence? What would you advise
her?

a. Pelvic floor exercises

31. A 43yr old female patient presents with a painless 2cm lump in the left upper quadrant of
the left breast. What is the initial investigation.

a. FNAC

b. Local biopsy

c. Mastectomy

32. A young male presents with the complaints of tinnitus, vertigo and deafness. What is the
possible diagnosis.

a. Menniers disease

b. Accoustic neuroma

c. meningioma

33. A 4yr old child presents with history of ASOM. .It was treated with penicillian. The child
returns after 4 days with no improvement.What is the management.

34. 12 hour old newborn baby presents with difficulty in breathing and cynosis. No murmers are
present. Xray shows pulmonary plethora .What is the possible diagnosis?
a. TGA

b. PDA

c. VSD

d. ASD

35. In a patient with a mid radial fracture. Which nerve is damaged?

36. A photo of HSC. What is the diagnosis?

37. There were 2 questions in which the Glasgow coma scale has to be calculated.

38. A question 8 lines long .The diagnosis is infantile spasm.

39. A CT scan of the brain showing a round low density lesion. What is the diagnosis.

40.What is the initial management of Hyperkalemia?

41. In a patient with 15% burn injury what solution would be used?

a. Hartmanns solution

b. Normal saline

c. 4% dextrose

42. A 42 yr old male patient complains of erectile dysfunction. What is the management?( no
drugs were listed)

a. Pressure method

b. Female masterbation
c. Manipulation of the breast

43. A 25 yr old primigravida presents with 12wks amenorrhea.On examination the uterus is 15
weeks in size. Which of the following could be the cause?

a. twin pregnany

b. moler pregnancy

c. Miscalculated LMP

44. A patient presents with pre eclampsia fits. What is the initial management.

a. C section

b. magnesium sulphate

c. induce labour

45. In a patient on HRT? Where do they act?

a. ovary

b. endometorium

c. pituitary

d. hypothalamus

46. A near term pregnant women presents to the emergency department after being physically
abused by her partner.She was struck on the abdomen.What would be the management?

a. after a normal CTG send the patient home

b. keep her for 24hrs observation

c. immediate c/s

47. A question on pagets disease


48. 27y woman presents with a 2cm breast lump noticed while showering. What is the initial
management?

a. FNAC

b. US

c. CT

49. most accurate investigation of bile duct

a. US

b. CT

c. ERCP

50. What is the most common cause of death in a child suffering from burns?

a. sepsis

b. renal failure

c. respiratory failure

51. An obese women with HT and diabetes comes for advice regarding pregnany. On a urine
examination,she tested positive for sugar.What advice would you give her?

a. An OGTT with advice on controlling her diabetes.

b. A change of life style.

c. she should not get pregnant at all.

52. A 42 yr old male with a history of surgery ,5 days post-op suddenly collapses. What could be
the cause?
a. PE

b. atelectasia

53. A 35 yr old male who was involved in a car accident and suffered a fracture of the femur. He
had surgery a week ago and is suddenly becoming disorientated and believes that people are
trying to kill him. What is the cause?

a. fat embolism

54. There were a few questions on asthma therapy. The use of preventers and also how to
manage an acute attack in a child and in an adult.

55. A child presents with a pan systolic murmur and cyanosis in childhood.What could be the
cause?

56. A patient presents in the emergency department with pnemothorax . Where should the
needle be inserted?

a. 3rd mid clavicular

b. 5th axillary

57. A photo showing a peri anal fistula .What symptom would be associated with this lesion?

a. pain on defecation

b. blood on stool

c. discharge

58. A photo of varicocele .What condition is commonly associated with it?

a. renal ca

b. testicular ca
59. A 42 yr old patient undergoes an MRI and by chance an adrenal mass of small dimensions is
found.on further hormonal investigations no abnormality was found. What is the management?

a. Surgery

b. reasuurence

c. after 6 months a repeat CT scan

60. A fat young boy presents for a routine cheak-up. On measuring the BP high readings are
obtained. What is the management?

a. 24Hr BP monitoring

b. check cuff size ,and re cheak the BP with an adult cuff

c. prescribe anti- hypertensive medications

61. A 2yr old child presents with short stature and hyperflexibility and mental retardation.What
could be the cause?

a. thyroid

62. A question on kawasaki disease

63. A 34 yr old male patient presents with history of an injury involving a rusty nail and
gravel.Not previously immunised.What would be the management?

a. TT+immunoglobulin + antibiotics

64. In a pt with a suspected snake bite …what would you not do?

65. In a pregnant diabetic patient, which of the following conditions are seen more commonly in
comparison to a normal pregnancy?
a. a macrosomia

b. b hydramnious

66. A patient wanted to know her chances of having a down syndrome baby. After a maternal
serum test the following results were obtained.

Down 1: 200

spina bifida 1: 400

what will you advise .

a. termination

b. amniocentesis

c. US

67. A 23 yr old primi who is 28wks pregnant comes with the history of contact with a patient
suffering from rubella .What would you advise?

a. us

68. What is the best diadnostic test for haemochromocytosis.( please know all the biochemical
results for diagnosis of this condition,as this question is often repeated in different forms)

a. tranferrin saturation

b. Liver biopsy

69 A chest X ray showing diffused lesions.What is the diagnosis?

a. a broncolitis

b. b pneumonia
70 . A 45 yr old patient presents with 3rd nerve palalysis without any changes in the ocular
reflexes.What is the cause?

a. DM

71. A pregnant women presents with vaginatis .She has a prior history of a miscarriage. How do
you manage the patient?

a. immediate treatment

b. b recheck at 28wks and treatment

72. A photograph of an SCC on the face.

72. A photograph of a HUCHINGSON freakle

73. A photograph of a bcc

74. What is the investigation of choice in a patient with a doubtful submundibular cyst?.

75. A 7y boy presents with a cyst …cant remember the location. What is the management. The
same question appears again but the patient is 17yrs.

76. A question on the physical examination of scabies in a child

O&G

1. A 22 nulliparous woman comes for routine first antenatal visit at 12 weeks amenorrhoea. On
examination, the uterus was found to be 15 weeks gestation. What would be the likely cause?

a. twin pregnancy
b. acute polyhydramnious

c. incorrect dates

d. macrosomia

2. A woman had a quick normal 6-hour labour and delivery to a healthy baby. A couple of days
later she lost 800mls of blood after passing a large clot and is pale and tachypnoea. She was found
to have a boggy uterus and a fundal height 2cm above the umbilicus. What would you do
immediately?

a. dilatation and curettage of retained products

b. give prostaglandin into uterine muscle

c. give i.m syntometrine

d. start her on antibiotics

e. intravaginal prostaglandin

3. A nulliparous woman came for routine antennal at 12 weeks. A week later, her cervical smear
can back as CIN3. What is the next course of investigation?

a. Hysterectomy

b. Colposcopy and biopsy

c. Colposcopy and LLETZ excision

d. Perform abortion

4. A 16-week multipara was found to have bacterial vaginosis at routine care. She had a previous
premature delivery at 34 weeks, she has no symptoms, and every thing is fine. What treatment
will you give?

a. metronidazole

b. wait till 28 weeks

c. tetracycline

d. vaginal pessaries

5. A women presents with foul smelling green vaginal discharge and a smear was taken. The
smear was found to have gram-positive rods and cells that were stuck together as “clueâ€
cells. What do you think she has?

a. trichomoniasis
b. bacterial vaginosis

6. A 37-year-old woman asks for contraception, as she would still like to have children in future.
She has children from previous relationship and is now in a new relationship. She is a smoker.
What method would you recommend?

a. OCP

b. IUCD with nor ethisterone

c. Depo Provera

d. Mini pill

7. A woman with CIN3. What advise you will not give her?

8. A 42-year-old woman with 6 months history of menorrhagia with the bleed lasting 8 days
every month. Management?

a. dilation and curettage

b. hysteroscopy

c. US for endometrial thickness

d. OCP

9. A primigravida woman whose labour was progressing normally was given epidural pain relief,
suddenly the foetal heart rate drops to 100 pm and?2 sustained deceleration. What could be the
cause?

a. maternal hypotension

b. foetal hypoxia

c. foetal cord compression

10. A primigravida in labour with irregular pain and contractions, the cervix was 4 cm dilated on
admission. She was examined four hrs later and her cervix was still 4cm. What will be the next
management?

a. set up an oxytocin drip

b. do caesarean

c. give analgesia
d. rupture the membranes

11. What can be found in association with a choriocarcinoma?

a. adenocarcinoma

b. mucinous cystadenoma

c. a metastatic choriocarcinoma of the ovary

d. leiomyosarcoma

12. Breast-feeding provides good contraceptive. What is not correct?

a. birth in the last 6 months

b. baby should be breast fed with no supplementation

c. night feeding compulsory

d. no menstrual periods after lochia

e. intercourse is not more than once a week

13. What advise would you give about premature ejaculation?

a. pull woman’s nipples

b. grab base of scrotum

c. woman on the top

14. A 34-year-old woman wants to know what is her chance of having a baby with neural tube
defect as her sister has just given birth to a child with the problem. What will you tell her?

a. chorionic villi sampling at 16 weeks

b. she has 50% chance of having child with defect

c. maternal serum test with US

d. no problems
1. What is the finding in ECG for Hyperkalaemia?

a) Peaked P wave and prolonged P R interval

Wide

b) QRS complex

c) Presence of U wave.

d) Bigemmni

e) Inverted T wave.

2. Which of the following is not matched?

a) Promiscuity CA cervix

b) Coeliac disease & lymphoma

C) EBV & Nasopharingial carcinoma

D) Schistosomiasis & bladder cancer

E) Nickel & liver ca

3. Where you can hear mitral stenosis murmur best?

a) 2nd left inter costal space.

b) Left lower sternal border

c) At the apex.

d) Rt. Lower sternal border

e) Mi axillery line.

4. A man has developed headache (cluster headache) what is the acute stage

Management?

a) Ergotamin

b) 100% O2
c) Paracetamol

d) NSAID

e) Relaxation

5. Which one of the following indicates carrier state of Hepatitis B?

a) HbsAG

b) HbsAB

c) HbcAG

d) HBeAB

6. A man has presented with prominent a wave raised JVP and also 4th heart sound what is the
Dx?

a) Myocardial infarction

b) Pulmonary embolism

c) Infective endocarditis

d) Heart failure

7. A 45 yrs. Old man past history of peptic ulcer disease presented with haemetasis on
gastroscopy one shaggy ulcer was found. Histology was negative. What will you do next?

A) Antacid

B) Omeprazol

C) Breath test for H. Pylory.

D) I/V fluid.

e) Antibiotic
8. A 16 yrs. Old boy brought to emergency with rope around his neck. Several pornographic
magazine were found around him at home, What can it be?

a) Suicidal attempt.

b) Autoerotic asphyxia.

c) Frotturism

9. a woman with history of anencephalic baby come for antenatal visit. How much of folic acid
you will advice her to take every day?

a) 4-5mg./day

b) 10mg./day

c) .4 -.5 mg./day

d) Increased intake of green leafy vegetables daily.

e)

10. For antenatal care to prevent neural tube defect when and how long to offer folic acid?

a) 3month before conception until ist. trimester

b) From conception to whole of the pregnancy.

c) From one month before pregnancy to full period of pregnancy

d) 1 month before pregnancy up to ist. trimester.

11. A businessman afraid of traveling of air, he feels nauseated, palpitition, sweating and
dizziness while he leaves his home. He always wanted to be at home. What is the diagnosis?

a) Social phobia

b) Agoraphobia

c) Panic attack with agoraphobia

d) Personality disorder
12. A rescuer while working get burned about 45% of his body from a patrol tank blast. He was
rushed to hospital. He was coughing out carbonaceous material. His vital signs were stable. What
will you do next?

a) Give O2

b) Intubate

c) Iv fluid

d) Do ABG to see wheathere intubation is warranted

e) Ad minister antibiotics

13. A 65 yrs. Old with history of COPD found unconscious by his neighbor. During transportation
to hospital he received 10lit O2 by mask. At the emergency he is unarousable What could be his
ABG ?

a) Pco2 75% Po2 55% P.H. 7.45

b) Pco2 100% Po2 60% P.H. 7.15

c) Pco2 90% Po2 89% P.H. 7.50

d) Pco2 40% Po2 60% P.H. 7.36 ?

14. One child accidentally took 20 iron tab.He is brought in hospital. What could not be his signs
& symptoms?

a) Abdominal pain

b) Signs & symptoms can develop after 24 hours.

c) Cerebellar ataxia

d) Hepatic failure

e) He can develop signs & symptoms within 4 hours.


15. In Lead poisoning you may not find which of the following?

a) Anemia

b) Neuropathy

c) Deafness

d) Abdominal pain

e) Inflammation of liver

16. 8 yrs. old boy took 25 tab. of Imipramin and presented to your sugary after an hour, before
calling to the poisoning center what you will you next?

a) Vomiting with ippicahe

b) Measure imipramine level

c) Activated charcoal

d) I/V fluid

e) Dialysis

17. Which of the drugs cannot be prescribed together?

a) Lithium carbonate & Carbamezum

b) Lithium carbonate & Sertrelline

c) Lithium carbonate & Clonazepine

18. A middle aged man presented with central abdominal pain radiating to the back. During the
attack he becomes hypotensive , after 6 hours he returns to normotensive. This is happening for
last 6 months, what can be the cause?

a) Acute pancreatitis
b) Leaking aneurysm

c) Ureteric calculus

d) Mesenteric ischemia

e) Mesenteric adenitis

19. A patient on a routine examination found to have a 5cm. mass on the upper pole of right
kidney, what you will do next?

a) USG

b) Laparotomy

c) Rescan after 6 weeks time

d) CT guided biopsy

e) Reassure

20. A patient were taking allupurinol, atenolol, diclophenac sodium, frusemide, simvastatin,
rimipril ,slow K+ and also aspirin. Now he presents with oliguria. What could be reason for his
oliguria?

a) K + rimipril + atenolol

b) Aspirin + atenolol + frusemide

c) Slow K + diclophenac sodium + Frusemide

d) Slow K + diclofenac sodium + atenolol

e) Rimipril + frusemide + atenolol


21. You been asked to deliver lecture in a seminar regarding retardation of child development,
you will say……..

a) My child does not roll over from back to front at the age of 4 months of age

b) My child do not smile at 2 months of age

c) My child do not sit at 5 months of age

d) My child does not have dentition at 10 months of age

e) My child does not stand at 11 months of age

22. In a diabetic keto-acidosis patient which of the following causes for hyperkalaemia ?

a) acute renal failure

b) lack of insulin

c) Acidosis

d) infection

e) lack of glucose

23. A 10 months old baby, she was on cow’s milk formula, she is very relactant to take solids.
Her wt. Is on 25th percentile, ht. Is on 50th percentile. What will you advice……….

a) Change to a different cow’s milk formula

b) Offering solids prior to cow’s milk formula

c) Give her nutritional supplement like multivit, polyjoule etc.

d) Give her breast milk

e) Give her only solids


24. A middle aged man has flashes of light and shimmering. It resolves without any significance.
After 4 days symptoms returned and he completely lost his vision in one eye over a period of 6
hours. What could be the DX?

a) Retinal artery thrombosis

b) Macular degeneration

c) Carotid artery disease

d) Retinal & macular detachment

e) Vertibrovasilar artery insufficiency

25. What are the features of vertibrovasilar artery insufficiency?

a) Ipsilateral blindness

b) Amareousis fugues

c) Diplopia, nystagmus, ataxia and visual disturbances

d) Contralateral hemiplegia & horner’s syndrom

26. Which of the following is the feature of Labyrinthitis?

a) Vertigo

b) Hyperemic eardrum

c) Nystagmus

d) Otorrhoea

e) Hypotension
27. A lady had a MVA 6 months before. Now she presents with pain in her Rt. Arm and hand
with disturbed sensation. Her biceps and triceps jerks are normal. Muscle power is 6/6 in biceps
and 6/5 in triceps respectively. Other tests were normal, What could be the cause of her situation?

a) Spinal cord injury

b) Fracture in the cervical vertebra

c) Multiple sclerosis

d) Intervertebral disc protrusion

e) Spinal artery thrombosis

28. A man has undergone treatment for carcinoma of lungs. Now he presents with pain
alongside his left arm and hand, what could be the cause?

a) Paraneoplastic neuropathy

b) Ca involving brachial plexus

c) Multiple sclerosis

d) Disc prolapse

e) Motor neuron disease

OBSTETRICS & GYNECOLOGY

1. About Bartholin cyst which one is correct?(MAY 2001)

A.most commenenst cause is Staphylococcus Aureus

B.maybe due to gonococcus

C.usually treated by antibiotics

D.usually recovers spontaneously


E.treated by marsupialization

2.During uterus contraction,what happens to placenta blood supply?(MAY 2001)

A.it reduces

B.it ceased

C.it increases

D.unchanged

E.

3.A 28 year old lady 16 weeks pregnant,G3P0 with 2 previous second trimester miscarriage
whith no uterus contraction or severe pain,

is concerned about her pregnancy,except from reassurance what is the best preventive
management?(MAY 2001)

A.Sulbutamol

B.observation

C.cervical stitch

D.Amniocentesis in 15-17 weeks ,looking for fetal abnormality

E.complete bed rest between 16 to 28 weeks

4.In a slightly obese 54 year old lady who has been menopaused 3 years ago and still does not
have hotflush what is the main reason for her

not having the symptom? (MAY 2001)

A.she has enough progestrone hormone that prevents hotflush symptoms

B.She converts aldosteronedione to estrone

C.she converts ADHD to estradiol

D.she has endrogens in fat tissues which prevents hot flush symptoms

E.obese women have more FSH that prevents hotflush


5.What is the risk of a 38 yr old lady having a baby with Down syndrome?(MAY 2001)

A.1:50

B.1:100

C.1:200

D.1:500

E.1:1000

6.A39 vweeks pregnant lady(G1P0)with normal antenatal care admits to the ward with
contractions started at 1am with interval of every 4-5 min.

Cervix is 2 cm dilated and effacement is complete.Fetus is in LOA presentation(left occiput


anterior)In which situation Obstetric consultation

opinion is needed?(MAY 2001)

A.5 am,cervix fully dilated ,membrane broken,contractopn 5 min still 2 cm above ischial spines.

B.5 am cervix 3 cm,membrane broken,contractions 5 min

C.5am cervix 3 cm ,membrane not ruptured ,contractions become weak and irregular

D.5 am cervix 3 cm ,membrane not ruptured ,contractions every 5 min

E. 8 am ,cervix 5 cm ,membrane ruptured ,contractions every 3 min

7.In a 32 year old lady with mastalgia ,which did not respond to conservative management,what
will you offer as a next step

in management?(MAY 2001)

A. bromocriptine

B.Medroxy progestrone for 10 days from day 14 to day 24 each cycle

C.clomiphene citrate

D.Danazol

E.combined contraceptive pills


8.Which of the following is not a benefit of taking combined oral contraceptive pills?(MAY 2001)

A.reduction in incidence of menorrhagia

B.reduction in incidence of benign breast disease

C.reduction in incidence of pelvic inflamatory disease

D.reduction in incidence of cervical cancer

E.reduction in incidence of ovarian cancer

9.A lady complains of depression ,breast tenderness and low mood related to her periods.Which
one of the followings is useful to confirm

that she has premenstrual syndrome?(MAY2001)

A.low stradiol level

B.slightly increase in prolactin level

C.chart her symptoms for 3 months

D.increase in progestrone level

E.respond to antidepresant treatment

10.A28 year old girl complains of hirsutism and irregular periods.17-hydroxyprogestrone is


normal and DHAS(dehydroepiandrosterone sulphate)

is slightly increased.She also has increased ratio of LH/FSH.What is your diagnosis?(MAY2001)

A.prolactinoma

B.primary ovarian failure

C.poly cyctic ovary

D.cervical cancer

E.endometriosis
11.A lady is going to do laparascopy because of infertility.During discussion about the risk of the
procedure,what advice will you give to

the patient, except?(MAY 2001)

A.Because you can see the surface and do biopsy, laparoscopy is the efficient diagnostic method
for polycystic ovary disease

B.Because CO2 gas can not be fully removed ,she may experience some shoulder tip pain

C. It may cause rupture of internal organs

D.It is a diagnostic tool for endometriosis

E.Infection is very rare and nearly always the result of unnoticed bowel damage.

12.Which one of the following is not related to hyperprolactinaemia?(MAY 2001)

A.amenorrhea

B.dry vagina

C.galactorrhea

D.osteoporesis

E.increased libido

13.Which one of the following indicates ovarian tumor rather than ascites?(MAY 2001)

A.shifting dullness

B.abdominal distension

C.dull anteriorly and tympany laterally

D.dull laterally and tympany anteriorly

E.tympany all over abdomen

14.Which of the following is not a tumor marker for gynaecological cancer?(MAY 2001)

A.BCA1
B.CA125

C.AFP

D.B-hCG

E.inhibin

15.A 54 year old lady who has 3 children complains of need to void many times during the day
and passing small amount of urine after

micturation .What is her diagnosis?(MAY 2001)

A. stress incontinence

B.urge incontinence

C.overflow incontinence

D.she will pass urine while running or jumping

E.she can lose large amount of urine

16.Which one of the following is the main predisposing factor of asymptomatic urinary tract
infection in a pregnant woman?(MAY 2001)

A.pressure of enlarged uterus on bladder

B.low immune system during pregnancy

C.dilatation of ureter and calyces due to progestrone

D.change of flora in perineal area during pregnancy

E.high level of oestrogen increase urinary tract obstruction which predispose to infection

17.Which of the following is not correct ?(MAY 2001)

A.Full breast feeding is a good method of contraception in the first 6 months after child birth

B.innitial menstruation after delivery is usually anovulatory

C.bromocriptine may provoke ovulation


D.progestrone is better than combined pills for contraception if she is breast fed

E.progestrone increase milk production

18.What is true in bronchial asthma in pregnancy?(MAY 2001)

A.Do not cause more bronchospasm

B.beta blockers should be used for treatment

C.steriods should not be used

D.it can cause premature labour

E.salbutamol should not be used for treatment

19.Which of the following is not true about oral contraceptive pills?(1997)

A.it reduces the incidence of benign breast disease

B.it reduces the incidence of endometrial cancer

C.it reduces the incidence of ovarian cancer

D.it reduces the plasma concentration of lipids

E.it reduces the incidence of iron deficiency anaemia

20.Which of the following is protective factor in breast cancer?(1997)

A.nulliparus

B.early menarche

C.late menopause

D.first child under the age of 34

E.breast feeding

21.What is your plan of management in a 30 weeks old pregnant lady who has a asymptomatic
streptococcus B infection?(1997)
A. treat the patient with penicillin immediately

B.do not treat because 25% of women are carriers

C.treat for doxycycline till the end of pregnancy and during labour

D.It is part of normal flora which does not need treatment

E.treat with IV penicillin during labour

22.A woman that in her previous pregnancy had a child affected by anencephaly is pregnant
again.With respect to management of this pregnancy which

of the following is not correct?(1997)

A.amniocentesis at 18 weeks

B.CVS at 11 weeks

C.ultrasound at 11 weeks to look for malformations

D.ultrasound at 11 weeks looking for nuchal thickness

E.maternal screening at 16 weeks

23.A 36 year old primigravida is worried about Down`s syndrome .Which of the following
investigations is the most specific and at the same time has the

lowest risk for the foetus?(1997)

A.maternal screening at 16 weeks gestation

B.amniocentesis at 18 weeks

C.CVS at 12 weeks

D.ultrasound at 11 weeks looking for nuchal thickening

E.cordocentesis at 24 weeks

24.A 48 year old woman had a CIN lesion removed 10 years ago and has recently had
hysterectomy but has her ovaries left.Which of the following

is not true about her?(May 1997)


A.she will need to do vaginal smear every 2 years

B.she will enter menopause sooner or at normal age

C.she will not need progestrone in her HRT

D. women with this type of operation have less chance of getting ovarian cancer

E. oestrogen therapy will be beneficial for her when she enters menopause

25.A 52 year old woman that has had hysterectomy and post-operative DVT ,comes to see you
complaining of hot flushes.Which of the following

treatment do you recommend?(May 1997)

A.combined oral contraceptive pill

B.oestrogen transdermal patches ,50 microgram

C.low dose oestrogen therapy per os

D.oestrogen and progestrone transdermal patches

E.progestrone therapy only

26.A 26 nulliparus woman who has recently married and wants to have children comes to you
complaining of severe menorrhagia .On

examination,you find that her uterus was completely deformed by uterine fibroids and that her
uterus can be felt just under her umblicus,and

haemoglobin is decreased by 4 gram%.Which of the following would you recommend?(May


1997)

A.total hysterectomy

B.open myomectomy after controlling her anaemia

C.give 3 months therapy with gonadotropin agonists,then do an open myomectomy

D.do a transcervical myomectomy

E.prescribe her the oral contraceptive pill


27.A young woman with infertility is recommended to do laparoscopy.She comes to GP office
looking for information about laparoscopy.

All the following are true except?(May 1997)

A. it is a good way to diagnose endometriosis

B.pain in shoulder is common post operatively

C.you can not find or diagnose polycystic ovary disease

D.there is a risk for damage to aorta or vena cava which can lead to death

E. minimal risk of damage to the internal organs rather than laparotomy

28.A 50 year old woman whose last period was 2 years ago,comes with 3 days painless vaginal
bleeding .What is the most likely diagnosis?

(May 1997)

A.early stage of cervical cancer

B.endometral cancer

C.occasional follicular hypertrophy

D.vaginal atrophy

E.anovulatory cycles

29.Regarding endometrial ablation all is true except?(May 1997)

A.difficult to operate if the uterus is bigger than 10 cm

B.cause amenorrhea in 40-60% of the patients

C.oestrogen without progestrone can be used postop

D.it is safer to use roller ball

E.hospital stay is usually short

30.A 18 year old female who has had multiple partners during the last few years, presents with
profuse vaginal discharge.Which of the
following investigations is not correct?(May 1997)

A.request HIV test

B.take lower endocervical swab for gonorrhea

C.take posterior fornix swab for chlamydia

D.treat sexual partner

E. send cervical smear for cytology

31.A 28 year old (G2,P1) woman who previous had a normal vaginal delivery,is currently 37
weeks pregnant and is admitted for induction of

labour due to pre-eclampsia.The baby is in longitudinal lie,cephalic,not engaged.At this time,you


rupture the membrane and put the patient on

oxytocin infusion.With the firsts uterine contractions the foetus becomes bradychardic,60 beats
per minute.What is the next step of

Mnagement?(May 1997)

A. do caesarean section promptly

B.do abdominal CTG(cardiotocography)

C. do vaginal examination

D.observe mother ,check mother`s vital sign

E.increase oxytocin dose to accelerate delivery

32.After 10 hours of obstructive labour ,a woman delivered her baby by caesarian section.3 days
later she developed fever of 37.9C.

Which of the following is the least likely cause for fever in this lady?(May 1997)

A.urinary tract infection

B.wound haematoma

C.deep vein thrombosis

D.endometrial infection
E.engorgement of the breasts

33.A mother brings her 4 year old girl with intermittent yellow vaginal discharge.In the
edxamination you find that the vulva is structurally

normal except that it is a little red and there is sign of scratching,the girl tends to put her hand
to the area .What is the most likely diagnosis?

(May 1997)

A.foreign body

B.candidial infection

C.gardenella vaginitis

D.syphilis

E.sexual abuse

34.Regarding Chlamydia infection during pregnancy ,all are true except?(May 1997)

A.erythromycin can be given if she is allergic to doxycycline

B.can be treated with doxycycline

C.treat with erythromycin

D.sulphuonmides should not be given

E.cefotaxime is usually added to treatment

35.A 30 weeks pregnant woman presents with proteinuria 8g/24 h.Her blood pressure is
180/125 mmHG and she has no oedema.

All of the following are true in her treatment except?(May 1997)

A.give her betamethasone

B.caesarian section within 48 hours in all cases

C.Mg-sulphate IV and other sedative drugs

D.start Labetolol
E.continous CTG cardiotocography

36.A woman who has had a troublesome delivery of a 4kg baby in the past,is now 33 weeks
pregnant with breech presentation.

You should do all the following except ?(May 1997)

A.do x-ray for pelvimetry

B.caesarian section if the baby is more than 4 kg

C.if the baby is less than 3.8 kg ,you can try a normal delivery if it is in frank or complete breech
presentation

D.do ultrasound to measure fetal size and position

E.do external version

37.A 25 year old married man comes with the complaint of infertility.You advise him to do
semen analysis,the result is as follows:

volume:2 ml, number :950,000 per ml , abnormal morphology:86% , motility:10%.What advice is


best appropriate for him?(May 1997)

A.he has slightly reduced fertility

B.give him testostrone and review him after 3 months

C.intracytoplasmic sperm injection will probably result a successful pregnancy

D.IVF will result in pregnancy in 20%

E.insemination by transvaginal method

38.All of of the follwing can cause congenital deafness EXCEPT?(May 1997)

A.rubella infection during second trimester

B.gentamycin treatment during pregnancy

C.cytomegalovirus infection during pregnancy

D.sulphonamides can cause deafness during third trimester


E.trimethoprim can cause deafness during first trimester

39.A woman in 37th weeks of pregnancy presents with vaginal bleeding.In exam,you notice that
she has lost approximately 900 ml

of blood,her blood pressure is 130/80 mmHg ,uterus is tender and there is no fetal heart
rate.Her haemoglobin is 68.Management

includes all of the following EXCEPT?(May 1997)

A.ultrasound to see if fetus is alive

B.anticoagulation profile test

C.if RH negative,give immunoglobulin to mother

D.blood transfusion and immediate surgery

E. start oxytocin transfusion for induction of labour

OBSTETRICS & GYNAECOLOGY

1.CORRECT ANSWER:D

The bartholin`s glands are mucus secreting glands located bilaterally on the labia majora just
external to hymenal ring.The duct can become obstructed

and lead to enlarged bartholin`s cyst.most of the time it will resolve by itself ,but sometimes it
may get infected ,bartholin abscess,which then need

surgical drainage like any other abscess.Adjunct antibiotic therapy is only recommended when
the drainage is cultured for neisseria gonorrhea which is

in 10 %of the time.Abscess that seems refractory to simple surgical drainage need antibiotic that
cover staphylococcus aureus.(blueprints in obs &gyn
,p98)

2.CORRECT ANSWER:A

3. CORRECT ANSWER:B

It should be noted that about 30-40%of perinatal death occur in pregnancies which terminate
between 20 and 28 weeks ,incompetent cervix being a

major contributor.Occasionally ,the condition results from surgical procedures such as cone
biopsy,cervical amputaion or Manchester repair.Classically

,there is a history of mechanical dilatation of cervix ,followed by one or more spontaneous


abortions after the 14 th week in which pain has not been

a feature. Transvaginal cerclage between the 14th and 20th weeks of gestation is the usual
procedure in case of cervical incompetence.If it fails ,usually

because there is insufficient vaginal cervix for ligation ,transabdominal cerclage can be
done.After 32 weeks,conservative treatment with bed rest ,

sedation ,mild terendelenburg position ,and perhaps progestrone is usual.Chromosome


abnormalities is the main cause of first trimester abortions.

(beischer Obstetrics,p 187-188)

4.CORRECT ANSWER:B

After manopause,a change in the ratio of oestradiol:eek:estrone occurs ,oestrone becoming the
dominant circulating oestrogen.Any circulating oestrogen is

synthetized in the peripheral fat by aromatization of androstendione,derived mainly from the


adrenal glands with some from the ovarian stroma.

(llewellyn-jones ,p 319)

5.CORRECT ANSWER :C

Incidence of down syndrome according to maternal age is 1 in 800 birth between 30-34 year old
and 1 in 260 birth between 35-39
(beischer Obstetrics,p679)

6.CORRECT ANSWER:C

This lady is in her first stage of labour.In general,a cervical dilatation rate of more than 1
cm/hour will result in spontaneous delivery,less than 0.5 cm/hour

usually means Caesarian section unless improved by oxytocin which needs Onstetrition
review.Contractions must continues progressively ,any delay or

irregularity needs further investigation.(beischer Obstetrics,p395-397)

7.CORRECT ANSWER:D

Danazol, a progestogen derived from testosterone, is effective in treatment of endometriosis


and menorrhagia and has been used with considerable

success in patients with breast pain and discomfort due to fibrocystic disease .Progestrone is
also used in treatment of mastalgia ,but its effectiveness

is not confirmed.Bromocriptine has many side effects ,so is used if no other treatment is
useful.(Gynaecology illustrated,David McKay,p133)

8.CORRECT ANSWER:D

The benefits of taking combined oral contraceptive pills are:

excellent contraception, reduced dysmenorrhoea,reduced iron-deficiency anaemia , reduced


menstrual loss , reduced benign breast disease, reduced

functional ovarian cysts , reduced ovarian and endometrial cancer , reduced pelvic inflamatory
disease

The cervical carcinoma incidence is not decreased .It appears to be slightly increased ,but this is
probably due to the increased sexual activity and not to

the ingestion of the contraceptive pill.(Annotated MCQ book,p322)

9.CORRECT ANSWER:C
The diagnosis of PMS is made after evaluating the periodicity of the physical and mood
symptoms,ascertaining that there is a symptom-free period after

menstruation and ensuring that the symptoms can not explained by some other illness.The
suspected diagnosis should be confirmed by asking the woman

to complete a daily record of symptoms over three menstrual cycles.(Llewellyn-jones ,p229)

10.CORRECT ANSWER:C

Polycystic ovarian syndrome is a functional derangement of the hypothalamo-pituitary ovarian


axis associated with anovulation. Clinical features are

oligomenorrhea,hirsutism and obesity.A relatively steady state of gonadotropins and sex


steroids exists.LH:FSH ratio increased ,oestradiol levels aresimilar

to those in the early follicular phase.Adrenal glands produce elevated levels of


DHAS.(Gynaecology,D. McKay,p94-95)

11.CORRECT ANSWER:A

Diagnosis of polycystic ovary is made on clinical grounds ,following by ultrasound to see multiple
ovarian cysts and also measurement of LH:FSH ratio.

Laparoscopy and biopsy is not indicated nor diagnostic.Laparoscopy is indicated in infertility


investigation looking for patency of fallopian tubes.Also, it can

be done in conditions like salpingitis,early tubal pregnancy and investigating vague abdominal
pain.Sterilisation,division of adhesions,oophorectomy,ovarian

cystectomy, salpingectomy and salpingostomy,laparoscopically assisted vaginal


hysterectomy,colposuspension and vaginal vault suspension for vault

prolapse.Complications include:perforation of viscus,hemorrhage,rarely infection because of


bowel damage,and post operation pain in shoulder tip because

of phrenic nerve irritation.complications are more frequent with operative laparoscopy than
with purely diagnostic one.(Gynaecology,D.McKay ,p85)

12.CORRECT ANSWER:E
Most commen cause of hyperprolactinoma is microadenoma of the pituitary gland,other causes
include:hypothyroidism,administration of dopamine depleting

agents.The woman with hyperprolactinoma develops oestrogen deficiency,with menstrual


disturbances(usually amenorrhoea),dry vagina and often reduction

of her libido.If hyper prolactinaemia persists,osteopenia and perhaps osteoporesis will result.In
30%of women inappropriate milk secretion(galactorrhea)

occurs.(Llewellyn-jones,p221-222 )

13.CORRECT ANSWER:C

An experienced examiner will recognise an ovarian tumor by finding midline swelling .A fluid
thrill may be elicited from an ovarian cyst ,and ascites and tumor

may coexist,but as a rulethe distinction should be easily made.With ascites there is resonant
over the top of the swelling and dull over the flanks.

With ovarian cyst percussion note is dull over the top of swelling and resonant in the
flanks.(Gynaecology,D.McKay,p 257)

14.CORRECT ANSWER:E

15.CORRECT ANSWER:B

It is necessary to distinguish between stress incontinence and detrusor instability since their
treatment is different .In stress incontinence,urine appears just

after efforts like coughing,laughing,running.This leakage is attributed to some displacement of


bladder neck so that it cannot respond normally to a sudden

increase in intraabdominal pressure.The cause is likely to be a pelvic floor weakness as a result


of parturition and/or oestrogen deficiency.Urge incontinence

is defined as a desire to void urine before the bladder contains 50 ml of urine and that is
because of detrusor instability .There is usually history of enuresis in

childhood and weak bladder even before pregnancies.In difficult cases,urodynamic assessment
must be done to distinguish these two diagnosis.

(Gynaecology,D.McKay,p317)
16.CORRECT ANSWER:C

Pregnancy is a predisposing factor for urinary tract infection mainly because of the increased
stasis within the urinary tract as a consequence of dilatation

of the ureters and the renal pelves.Significant bacteriuria in pregnancy is important for a
number of reasons.Clinical pyelonephritis will develop in one-third

of these women,obstetric complications is more common in these group like spontaneous


abortion,hypertension,preeclampsiaand perinatal loss.Also

established chronic pyelonephritis may happen in these women that needs investigation in
puerperium.(Obstetrics,Beischer,p373)

17.CORRECT ANSWER:E

For the woman who is breast feeding,the best way of family planning is low dose progestrone
pills(minipill).If she is fully breast feeding,she is unlikely to

ovulate because of the inhibition of ovulation by elevated level of prolactin.this is only reliable
for about 10-12 weeks,and within first 6 months ,there is 5 %

chance of ovulation if she mensturate.Since the combined pill reduces both milk output and
total energy content it is contraindicated in the lactating

women.Bromocriptine is usually prescribed for those who do not want to breast feed,because it
inhibits prolactin secretion so may cause ovulation to

happen.(Obstetrics,Beischer,p563)

18.CORRECT ANSWER:D

There is no uniform pattern of asthma disease in pregnancy but in general there is a tendency to
improve.There is also little evidence of any effect of

asthma on pregnancy unless there are prolonged episodes of hypoxia which can contribute to
spontaneous abortion ,intrauterine growth retardation ,

premature labour and stillbirth.Management of asthma in pregnancy is similar to the non-


pregnant state including,oxygen,nebulized beta

sympathomimetics,oral and inhalational corticosteroids.(Obstetrics,Beischer,341)


19.CORRECT ANSWER:D

The benefits of taking oral contraceptive pills are reducing iron deficiency anaemia ,decreasing
rate of benign breast disease and also ovarian and

endometrial cancers,but progestron increase cholesterol level.(Annotated MCQ )

20.CORRECT ANSWER:E

Among the risk factors for breast cancer are:nulliparous,early menarche,late menopause,first
child after age of 34,obesity,previous breast cancer and

positive family history.

Breast feeding is a protective factor for breast cancer. (blueprints in Obs & Gyn,p239)

21.CORRECT ANSWER:E

This gram positive coccus is present in vagina at some time during pregnancy in approxmately
10-25% of women ,it often can also be isolated from

the rectum in such women.Serotyping suggests a sexual mode of transmission.Premature labour


and premature rupture of membranes are common

in these women and in the baby there are 2 main syndromes:1)early onset septicemia and
pneumonia,often presenting as an idiopathic respiratory

distress in premature babies,and 2)late onset after the first week of life presents with
lethargia,anorexia,jaundice and meningitis.If mother is

symptomatic,she should be treated antepartum,if not,IV penicillin must be given in labour and
the baby should be treated after birth.

(Obstetrics,Beischer,p306-307)

22.CORRECT ANSWER:

Nural tube defect occurs in 2-5 pregnancies per 1000.Prenatal screening method is by
measuring the alphafetoprotein (AFP) level in the blood at the 16th
gestational week .If the serum level is more than 2.5 times the median for the week of
pregnancy ,the test is repeated.If it is raised in the second sample

Ultrasound will be done,to rule out multiple pregnancies which is another cause of high
AFP.Then amniocentesis is made to measure amniotic AFP.

Ultrasound at 11 weeks can also show fetal malformations in most centres.

23. CORRECT ANSWER:D

Incidence of Down syndrome diagnosed by amniocentesis is approximately 1 in 200 at 37 years


and 1 in 70 at 40 years.Because of the increase of

incidence with age ,prenatal screening is helpful especially in older age groups.In more than 90%
of cases there is trisomy 21 and translocation or

mosaicism is found in the remainder.The main element in diagnosis is biochemical screening


between 15 and 17 weeks (lowered serum AFPand

oestriol,raised HCG'),followed by ultrasound which not only excludes errors in dates ,multiple
pregnancies and other anomalies,but also can provide

positive clues,like nuchal thickening which is quite specific.Invasive diagnostic tests like
amniocentesis or chorionic sampling are not usually undertaken

,because of the risk to the pregnancy.(Beischer,p166-167)

24.CORRECT ANSWER:D

Following hysterectomy for CIN,6-monthly smears should be done for the first year
and,thereafter ,every 2 years as abnormal cells may be found

in the upper vagina signifying vaginal CIN.There exists data to the effect that up to 20%of
women aged 40-45 experience ovarian failure within 3 years

of hysterectomy.This can be prevented by hormone replacement therapy.Women who had


hysterectomy in the past will benefit from oestrogen patch

without need for progestrone.The question of bilateral oophorectomy of normal ovaries at the
time of hysterectomy is contentious.It is not usually done

in women under age of 45,but some gynaecologists perform it in older women.The reason is
that if ovaries are left,1 in 1000 women will develop ovarian
carcinoma ,also there is nofunction for ovaries after menopause. (Llewellyn-jones OBS &GYN
,p233)

25.CORRECT ANSWER:B

Hot flushes may begin in the months before the menopause ,but are worse after it,reaching a
peak incidence 1-2 years after the menopause.

Approximately,one third of climacteric women experience no or mild symptoms,one-third have


moderate symptoms but usually do not seek medical

advice and one-third have severe disabling symptoms.The hot flushes may persist for a number
of years after the menopause.Hot flush is caused by

lack of oestrogen in body.History of deep vein thrombosis or pulmonary thrombo-embolism


which occured during pregnancy or when on the oral

contraceptive pills or with anti-thrombin III deficeincy,presence of factor V Leiden or other


thrombophilia defects,is a contraindication to HRT.

A past history of thrombo-embolism without such histories is not a genuine contraindication,but


transdermal ,percutaneous or implants are preferred ,

to minimise oestrogen exposure to the liver.(Llewellyn-jones,p319-320,McKay GYN,p417)

26.CORRECT ANSWER:B

Fibriod is the gynaecological term for a leiomyomaof the uterus.It develops in the myometrium
and is not capsulated .It is the commonest tumor

found in women especially after 35 years of age.Most women,describe a history of incresing


menstrual bleeding and there are some complications

like torsion of its pedicle,cystic degeneration ,irregular bleeding and dysmenorreha and rarely
sarcomatose change.Small asymptomatic fibroids

need not be treated.Hysterectomy is the treatment of choice in older women who are
symptomatic,but in those who wish to be pregnant,myomectomy

can be done,which is through the uterine wall.Medical treatment with GnRH may give reduction
in size in 6 months,but rapid return to normal size

follows cessation of this expensive therapy.There is little place to use GnRH in those who need
myomectomy.(McKay,GYN,p215-217)
27.CORRECT ANSWER:E

Laparoscopy is used for investigation of infertility,to look for tube patancy.It is not adignostic
tool for polycystic ovary disease,which is usually

diagnosed clinically.There is increased risk of internal organ damageLaparoscopy does not


decrease risk of organ damage.

28.CORRECT ANSWER:D

The differential diagnosis of postmenopausal bleeding includes:atrophic vaginitis(60-


80%),exogenous oestrogen/HRT(15-25%),endometrial

carcinoma(10%),uterine and cervical polyps(2-12%),endometrial hyperplasia(5-10%).The older


the patient,the more likely to have endometrial

carcinoma.(blueprint of OBS& GYN,p216)

29.CORRECT ANSWER:C

This technique is used for treatment of menorrhagia .By ablating the basal layer of
endometrium ,the endometrial regeneration is prevented

and menorrhagia is cured.Before the procedure,the cavity of the uterus is inspected with a
hysteroscope.Many gynaecologists prescribe Danazol

or GnRH analogue before the operation to reduce endometrial thickness.So the smaller the
uterus ,the better the outcome.Roller ball is used

commonly in most places,but laser or loop resection can be used,as well.Copmlications


include,perforation of uterus(1%) ,fluid overload

due to absorption of sodium and glycin and amenorrhea in 35-60% of women.The benefits of
endometrial ablation are that it is less invasive and

painfull than hysterectomy ,the woman is in hospital for 1-2 days rather than 7-10 days and it is
less expensive.Oestrogen is never used to treat

menorrhagia.(Llewellyn-jones OBS& GYN ,p226)


30.CORRECT ANSWER:C

Most cervical infections occur in sexually active women,and usually follow a sexually
transmitted organism aquired from asymptomatic male

partner.It is currently thought that chlamydial infection causes 50-65%,gonorrhea 15-30% and in
the remainder the cause is unclear,usually

multibacterial.Laboratory tests should be made,urethral and cervical swabs for gonorrhea and
chlamydia (vaginal swab is not accurate) .

Partners must be treated as well.Cervical smear (pap smear) should be done in any sexually
active woman and should be repeated every

2 years,if the result is normal.(Llewellyn-jones, p 288)

31.CORRECT ANSWER:C

Preeclampsia is an indication for induction of labour,but itshould be noted that there are
contraindications when there is evidence of cephalo-

pelvic disproportion,abnormal,eg breech presentation,unstable lie,fetal distress,placenta previa


or vasa previa,as there is risk of cord prolapse.

Oxytocin should be stopped when there is fetal bradicardia.Prolapse of the umblical cord occurs
with a frequency of approximately 0.1-1 %

its occurance increased when there is a poorly fitting part or no engagement at the time of
induction.So the first step in this woman is to do

vaginal exam to look for cord prolapse as a cause of fetal distress.(OBS,Beischer,p458)

32.CORRECT ANSWER:E

Like any other operation ,caesarian section has some complications including infection
(abdominal wall,uterus,urinary tract,chest),thrombo

embolism,ileus,haemorrhage and wound dehiscence.Breast engorgement commonly occurs on


the third or fourth day.Hormonal activity causes

an increase in blood flow to the breasts and a sudden increase in milk production.Tension in the
breasts increase,with resulting obstruction to
the venous and lymphatic vessels and interference with the flow of milk along the ducts.It can
also occur at any time during lactation.Pyrexia

may signify infection,possibly mastitis.(OBS,Beischer,p492,618)

33.CORRECT ANSWER:A

34.COORECT ANSWER:B

In some centres,up to 20%of women have cervical and/or urethral chlamydial infection.It is 10 -
15 times more common than gonococcus.

The obstetrical significance of this infection is unclear ,but a relationship to premature labour
and premature rupture of membranes is

probable.30-40% of the babies of women with chlamydial infection will become infected.The
infection responds to tetracycline,doxycycline

or erythromycin,however during pregnancy doxycycline is contraindicated.Usually treatment


with penicillin or ceftriaxone or cefotaxime

to cover gonococcal infection is advised.(Beischer OBS,p 306)

35.CORRECT ANSWER:B

This woman has severe pregnancy induced hypertension or pre-eclampsia,that is blood pressure
higher than 170/110mmHg or presense of

marked proteinuria.It affects about 1% of primigravida.Treatment is ,admit her to


hospital,measure blood pressure every 4 hours.Do urine

protein test twice daily.She needs to be treated with IV antihypertensives ,like hydralazine and
MG sulphate to prevent seizure.Caesarian

section can be done if fetus is in distress,which is controlled by cardiotocography and


ultrasound.Before the 32 nd week of pregnancy

the objective is to keep the fetus in utero until the 35th week if it is possible.(OBS and GYN
,Llewellyn-Jones ,p126-127)
36. CORRECT ANSWER:E

The frequency of breech presentation falls as pregnancy advances.At the 30th week of
pregnancy 15%of the fetuses are breech,by 35th

week the proportion falls to 6% and by term to 3%.If the fetus is found to be breech ,external
cephalic version can be done only after

35-37 weeks.The main morbidities with breech presentation are :intracranial


haemorrhage,asphyxia,and fracture of the humerus,femur

or clavicle.Because of the risks mentioned,many of the breech presentations are delivered by


elective caesarian section.The success of

vaginal delivery of breech depends on the size of baby and maternal pelvis,which can be
assessed by pelvimetry and ultrasound.If the fetus

is less than 3800 g and has extended legs with flexed head ,vaginal delivery can be
tried.(OBS&GYN Llewellyn-jones,p 164,170)

37.CORRECT ANSWER:C

Male infertility is the cause of infertility in couples in 30-40 %of cases.Standards for a normal
seminal specimen have been developed by

the WHO .3 samples should be examined before a prognosis is made.In a normal analysis,there
is >2 ml semen,and more than 2 million

sperms per ml ,with >50%motility and>30% normal morphology.Oligospermia is when sperm


count is less than 20 million per ml,severe

oligospermia defined as less than 5 millon sperm per ml,and azoospermia is when there is very
little abnormal sperm.If azoospermia or

severe oligospermia is diagnosed ,blood level of FSH should bve cheked.A raised level (3 times
the normal upper limit) indicates tesicular

failure.If FSH level is normal testicular biopsy is sometimes made.Absolute infertility is dignosed
if azoospermia and high FSH level are

found.Severe infertility is diagnosed if severe oligospermia is found.Relative infertility is


diagnosed if the sperm count is between 5 and 20

millon per ml.Treatment with testostrone or clomiphene is used in nonsevere


oligospermia.Sometimes IVF is another alternative .In severe
oligospermia , the rate of pregnancy in couple is very very low,intra cytoplasmic sperm injection
is a method of treatment if spermatozoa

can be recovered.So,in this man with severe oligospermia,the best method is intracytoplasmic
sperm injection.

(OBS & GYN ,Llewellyn-jones,p251-254)

38.CORRECT ANSWER: E

Since the advent of mass immunization against rubella,this has become uncommon cause of
congenital infection.Fetal infection is un-

common when maternal infection is after 20th week of gestation.Presentation in newborn


include growth retardation,cataract,congeintal

heart disease ,deafness and bone lesions.Cytomegalovirus is the commonest congenital


infection 1%,but usally asymptomatic.With

sever involvement ,the common features are microcephalus,intracranial calcification


,hepatosplenomegaly,and jaundice.The mortality

rate is 20-30%,with the majority of survivors having mental retardation,cerebral palsy ,deafness
and visual impairment.About 10% of

asymptomatic affected newborns become deaf later in life.Some drugs are teratogenic ,if they
used during pregnancy.Aminoglycosides

can cause deafness,and their usage is forbidden during pregnancy.Sulphanomides can cause
kernicterus in baby if they used in third

trimester that high bilirubin level can cause deafness in newborn.Trimethoprim is a relative safe
antibiotic during pregnancy.

(OBS, Biescher,p662,105-110)

39.CORRECT ANSWER:
AMC –MCQS

A man 56 yrs old, had a swollen knee. The aspiration shows cloudy fluid, neutrophils
and no bacteria on microscopy and culture. What is the most likely diagnosis?

a- Septic arthritis

b- Reiter’s

c- Gout

d- Rheumatoid arthritis

e- Osteoarthritis

Osteoarthritis is:

a- ESR is increased or decreased

b- Night stiffness

c- Calcium pyrophosphate crystals are found in synovial fluid

d- Sodium aurothiomalate can be used in treatment

e- Serum calcium is increased

Brain stem aneurysm:

a- Papilloedema

Which of following would least likely be associated with primary hypothyroidism?

a- 72 yrs old woman with a large multinodular goitre

b- 9 yrs old boy, short and retarted

c- 28 yrs old woman with 3 yrs of menorrhagia


d- 16 yrs old girl anovulatory cycles

e- 2 yrs old boy with jaundice and constipation

Photo of penis (held with no gloves!). Little round lesions for six weeks itching. What is
the diagnosis?

a- Fleas

b- Scabies

c- Mosquito bites

d- Syphilis

e- Lympho Granuloma venereum

(J Type)

With regards to colorectal carcinoma which is correct?

a- Over 50% of cancers detected with screening are Dukes A

b- Flexible sigmoidoscopy picks up more than 40%

c- Faucal occult blood testing is good screening for the population

d- 20% will be missed by faucal occult blood because the cancer is not bleeding at the
time

e- Most of the cancers do not arise from villous adenomata

A patient with years of being unwell presents with a creatinine of 1.2mmol/l and high
urea. He is vomiting and is anaemic. All the following are true, except:

a- Minimal change clomerulonephritis

b- Membranous glomerulonephritis

c- Diffuse proliferative glomerulonephritis


d- Focal glomerulonephritis

e- Hypertensive nephropathy

Cerebral infarction:

a- Homeis syndrome with increased sweating

b- Papilloedema with optic neuritis

c- Pain and sensation loss on the same side of the body

d- Quadriplegia with anterior or middle cerebral artery infarct

e- Unilateral blindness with anterior circulation

A 52 yrs old male patient has ischaemic type chest pain for three hours and has a
normal ECG:

a- If the ECG is normal, he does not have a cardiac problem

b- He should have an exercise stress test immediately

c- Thrombolyties are usually not indicated

d- Heparin and IV nitrates are indicated

e- Wait for any treatment until cardiac enzymes are back

(J Type)

Regarding a serum lipid profile and diet, what is true?

a- Red wine improves LDL/HDL ratio

b- Olive oil will increase the LDL/HDL ratio

c- Positive energy balances increase LDL

d- 100-300g of fish per week will have no effect on plasma or membranes

e- An increase in dietary saturated fatty acids will increase VLDL


(J Type)

Regarding the treatment of pager’s disease, the following is/are true:

a- Patient should be given calcitonin

b- Diphosphonates are indicated

c- Cortisol

d- May need chemotherapy

e- 1000mg calcium everyday

Joseph., Oct 20, 2007

#1

Joseph.

Joseph. Guest

Treatment of chronic renal failure:

a- Antacid syrups can helps

b- Oral phosphate binding agents halt progress of bone disease

c- All patients will have symptomatic bone disease

d- Uraemic patients need to be dialysis

e- Hyperkalaemia can only be treated with dialysis

In regards to polycythemia rubra Vera which is correct:

a- Can be caused by obstructive sleep apnoea

b- Erythropoietin levels are high


c- Iron deficiency is common, so iron supplement should be given

d- ESR is increased

e- Clinical cyanosis may be difficult to detect

(J Type)

In a patient with known aortic stenosis god left ventricular function and clinical
presentation with CCF. Which of the following drugs are contraindicated?

a- Quinidine

b- Verapamil

c- Beta blocker

d- Diuretics

e- Digoxin

(J Type)

In which of the following conditions you be likely to see cannon a-waves?

a- Atrial fibrillation

b- Ventricular extra-systoles

c- Complete heart block

d- Incupid incompetence

e- Aortic stenosis

(J Type)

A patient with mitral stenosis due to rheumatic fever is congestive cardiac failure; the
following is/are true:

a- Surgical mitral valve plasty might improve the situation


b- ACE inhibitors may be indicated

c- Right heart failure is rare

d- Angiography should be done before operation

e- Ejection fraction 30. Patient needs treatment

Pulmonary hypertension and cor-pulmonate are frequency secondary to:

a- Emphysema

b- Bronchiectasis

c- Pneumothorax

d- Cadiomyopathy

e- Sarciodosis

A 60 years old woman with long history of smoking presented with dyspnoea. On
examination there were basal crepitations. FEVI and FVC were both severely reduced but
the ratio (not given but calculated) was 80%. Fibro sing alveolitis

b- Interstitial lung disease

c- Bronchogenic carcinoma

d- Decreased lung compliance

e- Asthma

A 30 years old man with visual disturbance can hear properly but has diplopia when
looking down out and laterally

a- Carotid artery aneurysm

b- Cerebelo pontive angle tumor

c- Acoustic neuroma
d- Horner’s syndrome

How do you differentiate between obstructive and restrictive lung disease?

a- Prolonged expiratory flow rate

b- Use of intercostals muscles

c- Movement of the upper part of the thorax

d- Inspiratory crepitations

e- Increased residual volume

Joseph., Oct 20, 2007

#2

Joseph.

Joseph. Guest

Ptosis may be found in each of the following conditions except:

a- Cluster headache (Horton’s syndrome)

b- Following cervical sympathectomy

c- Posterior artery communication aneurysm

d- Myasthenia gravis

e- Bell’s (facial nerve) palsy

The defect in visual fields most commonly associated with a pituitary tumour is:

a- Crossed homonymous henianopia

b- Central scotoma
c- Bitemporal hemianopia

d- Total blindness in one field

e- Peripheral concentric constriction and enlargement of the blind spot

Cardiac diseases known to be associated with sudden death include all of the following
except:

a- Coronary artery disease

b- Prolonged QT syndrome

c- Hypertrophic obstructive cadiomyopathy

d- Mitral stenosis

e- Aortic stenosis

The commonest arrhythmia associated with thyrotoxicosis is:

a- Sinus tachycardia

b- Atrial Fibrillation

c- Atral flutter

d- Atrial ectopics

e- Ventricular ectopics

A 35 years old housewife presents with headache, extreme muscular weakness, polyuria
and polydypsia and two episodes of carpopedal spasm. On examination she has a blood
pressure of 190/115mmHg and proteinuria. Initial screening blood tests show random
glucose 7.2mmol/L, sodium 158mmol/L, potassium 30.mmol/L. The most likely diagnosis
is:

a- Pancreatic insufficiency

b- Diacates insipidus
c- Adrenocortical adenoma

d- Renal failure

e- Hypoparathyroidism

The diagnosis for the patient in Question 40 is likely to be confirmed by measuring:

a- Dexamethasona suppression test

b- Urinary vanilly mandic acid

c- Plasma renia

d- Plasma cortisc

e- Glucose tolerance

The most common cause of secondary hypertension is:

a- Drugs

b- Coarctation of the aorta

c- Primary aldosteronism

d- Renal abnormalities

e- Elevated catecholamines

The most common cause of normocytic anemia is:

a- acute hemorrhage

b- Chronic inflammation

c- Malignancy

d- Hemolysis
e- Liver disease

The most cause of chronic bacterial orchitis is:

a- Tuberculosis

b- Syphilis

c- Leptospirosis

d- Staphylococcus aureus

e- Pseudomonas sp

A 35 years old woman whose pre-pregnancy body mass index was 35.0 kg/m2, is found
to have glucosuria (3+) and a random capillary blood glucose level of 12.0mmol/L in the
second trimester of her pregnancy. An oral glucose tolerance test confirms diabetes
mellitus. Her first pregnancy results in intra-uterine fetal death at 37 weeks gestation. The
most likely treatment will be:

a- A diet in which total energy intake in restricted severely

b- Insulin

c- No long term follow up, since the risk of non insulin dependent diabetes is negligible

d- Appetite suppressants

e- Oral hypoglycemic agents

Joseph., Oct 20, 2007

#3

Joseph.

Joseph. Guest
A 46 years old woman with a body mass index of 38 presents with a 2 weeks history of
sever headache, blurring of vision and nausea which followed a course of tetracyclines for
a lower respiratory infection. The significant abnormality on examinations is papilledema.
The probability diagnosis is:

a- Cerebral glioma

b- Temporal arteritis

c- Gravitational headache

d- Benign intracranial hypertension

e- Cerebral abocess

Diverticular disease of the colon is:

a- Associated with hypertrophy of circular smooth muscle

b- Most prominent in the right half of colon

c- Often complicated by carcinoma

d- Usually congenital in origin

e- Not associated with any of the above

In a cerebrovascular accident, hemorrhage and thrombosis may be clearly distinguished


on which of the following points?

a- Sequence of the clinical features

b- Degree of loss of consciousness

c- Description of onset

d- Presence of headache

e- None of the above

In which of the following is ascites usually present on clinical examination?


a- Left ventricular failure

b- Cirrhosis of the liver

c- Intra-abdominal Hodgkin’s disease

d- Nephrotic syndrome

e- Carcinoma of the uterus

The ‘opening snap’ of mitral stenosis:

a- Denotes valve mobility

b- Disappears if atrial fibrillation occurs

c- Is usually best head at the apex?

d- Replaces the third heart sound

e- Is really a closing snap?

The notifiable communicable disease that is most commonly reported in Australia is:

a- Hepatitis A

b- Salmonella infections

c- Gonorrhoeae

d- Rubella

e- Syphilis

Which of the following organisms is highly resistant to amoxicillin?

a- Haemophilus influenzae

b- Streptococcus pyogenes

c- Escherichia coli
d- Klebsiella

e- Staphylococcus aureus

Joseph., Oct 20, 2007

#4

Joseph.

Joseph. Guest

Which commonly arises in a solar keratoses?

a- Malignant melanoma

b- Squamous carcinoma

c- Basel cell carcinoma

d- Kerato-acanthoma

e- Sarcoma

A 30 years old woman has a history of one week of sweats and fevers. Yesterday she
had right sided pleuritic chest pain. Questioning reveals a long history of recurrent mouth
ulcers episodic joint pains in he hands, photosensitive skin eruptions on the face and
Raynaud’s phenomenon.

A most likely diagnosis:

a- Systemic Lupus Erythematosis (SLE)

b- Atypical pneumonia

c- Dermatomyositis

d- Hepatitis C

e- Rheumatoid arthritis
Where there is immunodeficiency, bacterial infections characteristically involve of the
following except?

a- Skin

b- Respiratory system

c- Sinuses

d- Joints

e- Gastrointestinal system

A 65 yrs old woman presents with recent onset of severe right sided headaches
associated with soreness of the scalp. Three days prior to admission she developed
blurring of vision in the right eye. On examination she has a visual acuity of less than 6/60
in the right eye and early swelling of the right optic disk. Which of the following is the
most appropriate next step?

a- Bed rest and analgesics

b- Pilocarpine 2% eyes drops hourly

c- Prednisolone 60mg daily

d- Cerebral CT scan

e- Urgent craniotomy

All but one of the following are recognised features of hypertrophic

obstructive cadiomyopathy. The exception is:

a- Sudden death

b- A large pressure gradient between sub aortic and lower ventricular chambers in
ventricular systole

c- Sometimes autosomal dominant inheritance


d- Q waves in anterior ECG leads

e- Eosinophilla

A patient presents with worsening early morning headache and early papilledema and is
considered to have raised intracranial pressure. Which

Of the listed features, if presents, would also be characteristic of this diagnosis?

a- Nuchal rigidity

b- Tachycardia

c- Marked loss of visual acuity

d- Third cranial nerve palsy

e- Hemiation of the cerebellar tonsils through the tentorium serebelli

Lower than normal renin levels may occur in association with:

a- Accison’s disease

b- Pregnancy

c- Beta block ace

d- Diuretic therapy

e- Malignant hypertension

Joseph., Oct 20, 2007

#5

Joseph.

Joseph. Guest
A 55 yrs old housewife complains of persistent widespread pains in the bones getting
worse over the course of 6 months. The differential diagnosis includes all except one of
the following. The one exception is:

a- Osteomalacia

b- Multiple myeloma

c- Osteoporosis

d- Hyperparathyroidism

e- Paget’s disease

A moderately obese man, aged 65 years, presents with a history of pain and mild
swelling of the left knee for the past four months. The only relevant past history is of
transient cerebral ischemic attacks being managed by aspirin, 300mg daily. Full blood
count and renal function are normal but the serum urate is 0.51mmol/L. He suffered from
a single episode of podagra four years previously. Which of the following statements is
correct?

a- The current attacks of arthritis is probably osteoarthritis

b- His 24 hour urinary uric excretion is probably elevated

c- Allupurinol should be used to reduce his serum uric acid level to normal

d- A reduction of aspirin dosage to 100mg daily will lead to a fall in the serum urate
level

e- If uric acid reduction were required, it would be better to prescribe sulphinpyrazone


that allupurinol to a patient already receiving aspirin

Characteristically, type A viral hepatitis:

a- Is a DNA virus infection often associated with seafood poisoning

b- Has an incubation period of three months

c- Carries a greater risk of fulminant hepatic failure in young compared with old patients

d- Often presents with headache plus right hypochoncrial pain and tenderness
e- Can progress to chronic active hepatitis if cholestasis is prolonged

Which of the listed drugs has the potential to induce life threatening cardiac
arrhythmias when taken in overdose?

a- thioricazine

b- Haloperidol

c- Lithium

d- Fluphenazine

e- Levodopa

Which of the following adverse effects is best associated with use of flucloxacillin?

a- Phototoxicity

b- Interstitial nephritis

c- Cholestatio hepatitis

d- Agranulocytosis

e- Acute hemolytic anemia

Hirsutism is recognized side-effect of treatment with all excepts one of the following
agents. Which is the exception?

a- Progestogens

b- Psoralens

c- Minoxidil

d- Trimethoprim

e- Diazoxide
During the first 4 hours after an overdose of paracetamol, a patient will develop:

a- Hypoglycemia

b- Hyperventilation

c- Increased serum ALT and AST

d- Pin-point pupils

e- None of the above

The ECG changes in left ventricular hypertrophy typically include all of the following but
one. The exception is:

a- ST elevation in lead

b- Tall R waves in leads V5 and V6

c- Deep S waves in leads V1 and V2

d- Left axis deviation

e- T waves inversion in lead a VL

A 55 years old man with a 30 pack/year smoking history presents with a history of
angina of effort. With respect to potential drug therapy, which one of the following
statements is covered?

a- verapamil and nifedipine would each reduce myocardial rate and contractility

b- Glyceryl trinitrate transdermal patches should be worn for 24 hours and then
replaced

c- The predominant mechanism of action of glyceryl trinitrate is peripheral arterial


vasodilatation and a reduction in cardiac after-load

d- Beta blockers are indicated as prophylactic therapy for exercise induced but not
vasspastic angina

e- Beta blockers and calcium channel blockers could each precipitate asthma
Which of the listed anti arrhythmic drugs is the drug of choice in most cases of
Supraventricular tachycardia (including nodal tachycardia)?

a- Digoxin

b- Quinidine

c- Flecanide

d- Verapamil

e- Propranolol

In a patient with severe asthma, which of the features listed would be the most
ominous?

a- Arterial PO2 50mmHg

b- Arterial PCO2 50mmHg

c- FEV10.8 litres

d- Very loud wheezes

e- Respiratory rate of 20 per minutes

An elderly man has been hospitalized for one week because of severe and worsening
Parkinson’s disease. He is found in bed tachypnoeic, tachycardic and responsive only
to painful stimuli. Pulmonary examination reveals diffuse wheezes and crackles in the
right lung field posteriorly and absent breath sounds in the left lung field. Which of the
following conditions is most likely to be responsible for this patient’s deterioration?

a- Massive pulmonary embolism

b- Pulmonary oedema after acute myocardial infarction

c- Aspiration pneumonia with unilateral bronchial obstruction

d- Spontaneous pneumathorax
e- Tracheosophageal fistula

Joseph., Oct 20, 2007

#6

Joseph.

Joseph. Guest

All except one of the following statements are characteristic of ventricular septal defect
of the heart. The o exception is:

a- It produces a left to RIGHT SUNT

b- It can be diagnosed by cardiac ultrasound

c- It is associated with a continuous murmur throughout systole and diastole

d- It is one of commonest congenital hear lesions

e- It is associated with a step up in oxygen saturation in the right ventricle

All of the following except one typically produce keratin scales.

The one exception is:

a- Squamous cell carcinoma

b- Solar keratoses

c- Psoriasis

d- Pityriasis versicolor

e- Basal cell carcinoma


A 75 yrs old woman, previously in good health, presents with a history of painless
jaundice of 2 weeks duration. There is a distended gall bladder revealed by ultrasound
examination. What is the most likely cause?

a- Chronic Pancreatitis

b- Alcoholic cirrhosis

c- Gall stones

d- Carcinoma of the pancreas

e- Acute Cholecystitis

In diabetic neuropathy you can find each of the following except:

a- Bradycardia

b- Urine retention

c- Impotence

d- Diarrhoea at night

e- Foot ulcer

In suspected acute MI what is the criteria for thrombolytic therapy?

a- T inversion

b- ST depression/elevation???

c- New LBBB

d- New RBBB

e- Q waves

A 55 yrs old woman was found unconscious in her house. She has a dilated pupil. What
is the cause?
a- overdose

b- Brain infarct

c- Subdural hematoma

d- Subarachnoid hemorrhage

e- Coma

Antidote for heparin is:

a- Protamine sulphate

b- Vitamin K

c- Fresh frozen plasma

In heparin induced thrombocytopenia which of the following is typically seen?

a- Petechial purpura on the skin

b- Thrombosis

c- Joint bleeding

d- Hematuria

e- Ecchymosis

All of the following can cause renal papillary necrosis except:

a- Liver disease

b- Medullary sponge disease

c- Sickle cell disease

d- Diabetes mellitus

e- Analgesic nephropathy
Opening snap indicates:

a- Mitral Valve mobility

b- Atrial fibrillation cause disappearance of the opening snap

c- Replaces S3

d- Best heard at 2nd right intercostals space

e- Remains unaltered despite progression of disease

In a man from overseas, his chest x-ray reveals silent tuberculosis. PPD skin test is 10 in.
treatment of choice is:

a- Sputum culture

b- Observation

c- Triple therapy

d- Isoniazid

e- Isoniazide and ethambutal

In infectious mononucleosis:

a- Incubation period is 4-5 days

b- Is seen more in patients with AIDS

c- Incubation period is 14-21 days

In asthma, which is the most important factor in diagnosing its severity?

a- Decreased PEY

b- Increased diumal variability

c- Decreased diumal variability


Joseph., Oct 22, 2007

#7

Joseph.

Joseph. Guest

A man returning from an overseas trip has Diarrhoea and vomiting for 2 weeks,
salmonella was isolated. What is the next step in management?

a- Ampicillin

b- Repeat stool culture

c- Trimethoprim-sulphamethoxazole

d- Only observation

e- Metronidazole

All of the following are side effects of Prednisolone except:

a- Hirsutism

b- Lymphopenia

c- Leucocytosis

d- Osteoporosis

e- Weight gain

Polycythemia rubra vera:

a- Contraction of plasma

b- Expansion of plasma volume


Which of the following is correct regarding the overdose of TCA?

a- In a mixed overdose with TCA’s and benzodiazepines flumazenil is contraindicated

All of the following are features of Parkinson’s disease except:

a- Postural reflexes lost

b- Can begin unilaterally

c- Tremor gets worse during sleep

d- Cogwheel rigidity

e- Able to stop while walking

A patient presents with impaired sensation on the medical aspect of the hand, 4th and
5th fingers of the hand are flexed. Where is the lesion?

a- Ulnar nerve at the elbow

b- Ulnar nerve at the wrist

c- Redial nerve damage at the wrist

d- Medial nerve damage at the wrist

e- Redial nerve damage at the elbow

A patient presents with moderate hemiparesis of legs and arms, no dysphasia, face
appears normal. Where is the lesion most likely to be?

a- Cerebral cortex

b- Internal capsule

All of the following are features of thyrotoxicosis except:

a- Wasting of small muscles of the hand


b- Fine tremor of hands

c- Irritability

d- Weight loss

e- Wide pulse pressure

Which of the following is the most common cause of ketoacidosis?

a- Cessation of insulin in IDDM

b- Undiagnosed IDDM

c- Undiagnosed NIDDM

d- Adrenal failure

e- Infection

In paracetamol poisoning the involvement of which organ is most important?

a- Renal injury

b- Hepatic injury

c- Brain injury

d- Stomach injury

Joseph., Oct 22, 2007

#8

Joseph.

Joseph. Guest

A 70 yrs old woman complains of fatigue Blood picture:


a- Autoimmune haemolytic anemia

b- Pernicious anemia

c- Anemia of chronic disease

d- Anemia due to silent gastrointestinal bleeding

e- Leukaemia (CML)

A patient has massive ascitis and in the fluid malignant cells are found. Which of the
following is a possible finding?

a- Splenomegaly

b- Hepatomegaly

c- Supraclavicular

d- On digital rectal examination, nodules can be palpated

In bronchial asthma, which is the best indicatory of good treatment?

a- FEVI

b- ABG

c- Decreased RV

d- PEFR

Which category of woman will benefit the most from a mammography?

a- Age 35-50 years

b- Age 50-65 years

c- Age>65 years

d- Woman with first degree relative with breast cancer

e- Woman with breast cancer


Regarding unilateral undescended testis, which of the following is most commonly
associated?

a- Malignancy

b- Varicocele

c- Inguinal hernia

d- Hydrocele

e- Torsion of the testis

All of the following would be excepted after splenectomy for spherocytosis, except?

a- Transient Leucocytosis

b- Persistence of anemia

c- Same osmotic fragility

d- Persistence of spherocytosis

e- Normal life span of the erythrocytes

A 50yrs old man came to the emergency room, presenting with 3 days vomiting
followed by periumbilical pain. On examination, his abdomen is distended and bowel
sounds are absent. Abdominal x-ray shows multiple air-fluid levels. Hb 180 g/L, Which of
the following is the best fluid management for this man?

a- 2000 ml of 4% dextrose in 405% NaCL preoperatively

b- 2000 ml of 4% dextrose in 4.5% NaCL during the operation

c- 2000 ml of Hartmann’s solution preoperatively

d- 2000 ml f Hartmann’s solution during the operation

e- 2000 ml of 5% dextrose
All of the following are risk factors for breast cancer, except:

a- Early artificial menopause

b- First pregnancy after 35 yrs of age

c- Early menarche

d- No breast feeding

e- Cancer of the other breast

A diabetic patient with an ulcer at the head of the 2nd metatarsal on the sole of the
foot. Which of the following would be the most likely cause?

a- Microvascular

b- Diabetic neuropathy

c- Infection

d- Varicose veins

e- Hyperglycaemia

Hutchinson’s melanotic freckle. Which of the following is not correct? (Scborrhic


ketatosis)

a- Mostly occurs in the elderly

b- Mainly occurs in covered parts of the body

c- Malignant

d- Irregular

e- Irregular colour (from back to brown)

A female had mastalgia for the last 1 year. Conservative treatment has not worked. How
would you manage this patient?
a- Bromocriptine

b- Danazole

c- Clomiphene

d- OCP

e- NSAID’s

A middle aged female complains of morning Diarrhoea and cramps. On examination


there are no remarkable findings. The most likely diagnosis:

a- Irritable bowel disease

b- Ca colon

c- Crohn’s disease

d- Ulcerative colitis

Joseph., Oct 22, 2007

#9

Joseph.

Joseph. Guest

A diabetic person presents with a red & swollen leg, from the ankle to the knee. What is
most likely responsible for this?

a- Clostridium welchii

b- Group A streptococcus

c- Group B streptococcus

d- Staphylococcus

e- Thrombophlebitis
All are true about gallstones, excepts:

a- Usually asymptomatic

b- Stone in the bile duct presents with clinical sepsis

c- Usually radiolucent

A 35 yrs old woman having severe sudden abdominal pain throughout the night wakes
up and passes dark urine in the bathroom. She immediately goes to se the doctor.
Investigation shows:

- Bilirubin – 5 times normal

- Alkaline phosphates – 3 times normal

- AST & ALT – 4 times normal

- S. amylase – 2 times normal

What is the most likely diagnosis?

a- Acute cholelithiasis

b- Acute Cholecystitis

c- Ca gallbladder

d- Acute choledocholithiasis

All are true in regards to scaphoid fracture, except?

a- There is pain/tenderness in the anatomical snuff box

b- Necrosis of the distal fragment can occur

c- Treat even if initial x-ray are normal

d- X-ray changes may be seen after 2 weeks


Photography 5: X-ray showing colleens fracture. An elderly person sustained a fall on an
outstretched hand. What is true?

a- Median nerve function should be assessed

b- The plaster cast is not suitable to correct the fracture in the elderly

c- Immediate internal fixation

d- Plaster case from below elbow to MCP joint, for at least 10 weeks

The most appropriate OCP for a 24 yrs old woman taking phenytoin is:

a- Microgynon 30

b- Microgynon 50

c- Clomiphene

d- Oestrogen 85 micrograms

Depo provera can cause all the following, except:

a- Mild hypertension

b- Amenorrhoea after the 3rd injection

c- Fertility problems for 6-12 months after stopping the treatment

d- Menstrual irregularities

e- Weight gain

All of the following are true about neural tube defect, except:

a- Decrease the alpha fetoprotein

b- Polyhydraminos

c- Folic acid prophylaxis can decrease the risk of neural tube defects
d- There is a 4% chance of neural tube defect in the 2nd pregnancy

D % C is indicating in which of the following?

a- Postmenopausal bleeding

A 9 day old baby with projectile vomiting and dehydration with k 7.1, Na 125, Cl 80, &
HCO3 20. What is your diagnosis?

a- Pyloric stenosis

b- Galactosemia

c- Sepsis

d- Hypoglycaemia

e- Congenital adrenal hypertrophy

An 8 week old baby presents with unilateral sticky eye. Similar episode before, was
treated with antibiotics for 3 days and resolved. What is the most likely diagnosis?

a- Gonococcal conjunctivitis

b- Allergic conjunctivitis

c- Inadequate antibiotic treatment

d- Decrease IgA A in the tears

e- Blocked tear duct

Joseph., Oct 22, 2007

#10

Joseph.

Joseph. Guest
An 8 year old boy presents with photophobia, irritability & stiffness. Lumbar puncture
findings show 50 neutrophils, 100 lymphocytes, proteins normal, and glucose normal.
Which is the most likely diagnosis?

a- Tuberculosis meningitis

b- H. Influenza

c- Echo virus

d- E. coil

e- HSV encephalitis

A 4 year old child presents with sudden onset of cough, unilateral wheeze and
decreased respiratory movement on one side. There is no family history of atopy. What is
the most appropriate management?

a- Chest x-ray

b- Full blood examination

c- CT of chest

d- Ultrasound

e- Inspiratory & expiratory chest x- ray

A 5 months old baby unwell for a week is noticed by his parents to have episodes of
leaning forward and shaking his arms. His parents are concerned, because he is not
responding as he used to. What is the likely cause?

a- Infantile spasm

b- Febrile seizures

c- Breath holding spells

d- Infantile myoclonic seizures


e- Petit mal epilepsy

A 2 years old child presents with cough for 2 months. On examination you find clubbing,
wheezing and crackles. What is your management?

a- Prescribe antibiotics

b- Perform a chloride sweat test

c- Prescribe oral corticosteroids

A mother notices a lump in the right groin of her 2 years old son, which disappeared
after a few hours. Despite a thorough examination you are unable to discover anything.
The most appropriate management would be?

a- Reassurance &send home

b- Admission into hospital for surgery

c- Request the mother to bring the child to you immediately when the lump reappears
again

d- Review after 1 week

A full term infant girl is transferred to the postnatal ward when aged 1 hour. On arrival
she is notice to have cyanosed hand & feet. On examination you find the responsive and
cries lustily on handling. The respiratory rate is 40/min and the lungs & heart are clinically
normal. Which of the following is correct?

a- The infant has probably had a convulsion

b- The infant’s rectal temperature should be checked

c- The infant has early sings of respiratory distress

d- Oxygen should be given

e- An immediate chest x-ray should be arranged


An 8 years old boy was stung by a bee. He had difficulty breathing and facial oedema.
What is the best treatment?

a- Adrenaline 1:1000IM

b- Adrenaline S.C.

c- Antihistamine iv

d- Hydrocortisone

e- Intravenous fluids

All of the following can cause short stature, except:

a- Psychosocial deprivation

b- Obesity

c- Congenital adrenal hyperplasia

d- Hypothyroidism

e- Turner syndrome

A 2 week old baby gained 200 grams/week since birth. The mother complains that the
child vomits mild soon after feeding, otherwise okey. What will you do?

a- Reduce the time between feeds

b- Shorten the duration of feedings

c- Give Water before feeding with milk

d- Endoscopy

e- Urine microscopy

A 10 year old child, 139 cm tall, is brought to the doctor by his parents. His parents are
concerned and say, “He had been a fatso forever”. What could be associated with this?
a- Height for age is always more

b- Elevated TSH

c- Insulin resistance

Joseph., Oct 22, 2007

#11

Joseph.

Joseph. Guest

70 yrs old man, smoking 90-pack/year now with sign of respiratory alkalosis. PH=7.38 (N
7.35-7.45) PO2=48, PCO2 68. What will confirm that he has compensatory respiratory
alkalosis?

a- Low PO2

b-

c- Low Po2, high PCO2 and normal PH

d- High CO2

e- Heavy smoker

What is immature defence mechanism?

a- Suppression mature

b- Denial neurotic

c- Intro-fection

d- Projection

e- Sublimation
Woman pregnant, with oedema of the ankle and the hand, hypertension, ascites, baby
born with jaundice and bilateral cataract. Previous baby has similar condition:

a- ABO- incompatibility

b- Acute glomerulonephritis

c- Rubella embryopathy

d- Galactosemia

e- Retrolental fibroplasia

Regarding mammography what is most correct:

a- Has higher detection rate of breast lump than self-examination

b- Can differentiate between cyst and solid mass USG

c- Has more benefit in pre-menopausal than post-menopausal woman

Which of the following drugs is least likely teratogenic?

a- Phenytoin

b- Danazol

c- Stilloestrobeol

d-

e- Corticosteroids

What will least likely produce intrauterine deafness?

a- Phenytoin

b- Birth weight less than 1500gms

c- Kernicterus
d- Microcephaly

X-ray wrist with Cole’s fracture. What is correct treatment?

a- Immobilisation for 10 weeks

b- Reductio and immobilisation is correct treatment

c- Check medianus

d- Internal fixation

Regarding cluster headache all of the following is correct except:

a- Every day and/or day and night same time

b- More in man than woman

c- Pain

d- Awake patient 4 – 6 hrs after sleep

e- Vomiting

Woman 35 yrs old has sharp jaw pain, loss of sensation in trigeminal area:

a- Trigeminal neuralgia

b- Multiple sclerosis

Regarding Seminoma what is correct:

a- Occurs in childhood, locally metastasis, radiosensitive

b- Occurs in childhood, distant metastasis, radiosensitive


c- Occurs in young male, locally metastasis, and radioresistant

d- Occurs in young male, distant metastasis, radiosensitive

e- Occurs in old man, distant metastasis, radioresistant

A baby with a 4 months history of unilateral discharge, few polymorphonuclear


cells……………

a- Gonococcal infection

b- Chlamydia infection

c- Blocked tear duct

d- Antibiotics prescribed for short period

All of the following is true in pulmonary embolism except:

a- Increased second heart sound

b- Syncope

c- Dyspnea

d- Pleural rub

e- Bronchial breath sound

Man has polyuria. What is most common finding in diabetes mellitus?

a- Weight loss regardless of increased appetite

b- Dysuria

c-

d-

e- Sugar craving
70 yrs old lady, on Digoxin 0.25, Hydrochlorothiazide. She fell dizzy, now presents with
arrhythmia 220/min pulse 110/min? K 3.1, urea ^ (high)………….

a-

b- Continue Digoxin + hydrochlorothiazide and add Furosemide

c- Stop Digoxin

d- Stop Digoxin + KCI

e- Do Cardioversion

Joseph., Oct 23, 2007

#12

Joseph.

Joseph. Guest

9 yrs old girl, occasionally headache. Vomiting before breakfast. Child is otherwise well:

a- Migraine

b- Hydrocephalus

c- Medulloblastoma

Implantation of fertilized egg will most likely occur at:

a- 8 days

b- 5 days

c- 10 days

d- 20 days

e- 5 weeks
Patient with ileocolostomy, what is lost in highest concentration before and after
operation?

a- Bicarbonate

b- Calcium

c- Kalium

d- Sodium

e- Potassium

3 and ½ years old boy came on teacher and parent request. Teacher complained that
child is aggressive toward other kinds in childcare, poorly concentrate: parents said that
they could not cope with child. When child is left alone in your room he is quietly playing
with toys, without any above-mentioned problem.

What is most likely?

a- Attention Deficit Disorder

b- Opponent defiance

c- Poor parenting

d- Normal variant

9 yrs old girl truanting from the school, playing arcade games. When interviewed she is
tearful, and she has insomnia……….

a- Anorexia nervosa

b- Attention deficit disorder

c- Anxiety separation disorder

d- Conduct disorder
Mother of 15 year old girl phoned you to discuss her daughter behaviour. She refuses to
eat meal with a family, exercise more than before, refuse to take Benzodiazepines
prescribed by her GP. What will be your advice?

a- Tell her that girl has anorexia nervosa and needs hospitalization

b- Familiar meeting as part of initial assessment

c- She has exercised dependency, which has excellent prognosis

Man with Supraventricular tachycardia. Ventricular beat? Atrial beat?

a- Verapamil

b-

c- Cardioversion

What is best day-to-day estimation of Digoxin therapy?

a- Apex beat

b- Redial pulse

Polycythemia rubra Vera. All of there following is true except:

a- Elevated erythropoietin

b- Sleep apnoea can cause it

c- Plasma contracted

d- Clinically hard to recognise

Estimation of suicidal attempt:


a-

b- Privacy Act forbid giving information to family members regarding suicide

c- Removing fire arms from house doesn’t mean much because of


inevitability……

30 yrs old woman with 3 yrs old child, regular cycles she can’t be pregnant although
coitus usually in the middle of the cycle. Most likely cause is:

a- Hyperprolactinemia

b- Inefficient luteal phase

c- Tube disruption – PID

d- Seminal vesicle anomalies

Woman with C I N 3

a- Cervical biopsy preceded by colposcopy

Photo. Picture of small ulcer surrounding by black deposit patient had colostomy:

a- Squamocellular carcinoma

b- Metastatic deposit

c- Pyoderma gangrenosum

d- Diathermic burn

Joseph., Oct 23, 2007

#13
Joseph.

Joseph. Guest

Malignant cells in gross ascites. What is least likely to find?

a- Palpable left supraclavicular node

b- Palpable liver

c- Palpable spleen

d- Perirectal lymph node

Diabetic woman, on therapy with ACE, asking you for advice regarding her possible
pregnancy:

a- Stop ACE change for Aldomet

b- Continue ACE

Child, 4 yrs old, only breast feed, most likely cause of anemia is:

a- Iron deficiency anemia

Pancreatitis can be caused by all of the following except:

a- Mumps

b- Smoking

c- Alcohol

d- Islet cell tumour

e- Gall stone

What matching pair is correct?


a- Anticardiolipin antibodies – intrauterine growth retardation

b- Lupus anticoagulant – pulmonary embolism thrombosis

c- Anti Ro antibody – heart block

d- Platelet count <100.000 – foetal intracerebral Haemorrhage

Newborn, cyanotic, cyanosis completely disappeared after giving him 100% O2.

a- Lung disease

b- Methemoglobinemia

c- Tetralogy Fallot

70 yrs old woman losing weight, pain in hands. She has hypercalcemia in blood. Most
important to distinguish between malignancy or hyperparathyroidism is:

a- Elevated serum Ca

b- Elevated alkaline phosphates

c- Elevated hyper parathyroid hormone

Man with acute respiratory distress involved in car accident, trachea shifted to the right
no breathing on the left:

a- Pneumothorax

b- Haemato pneumothorax

c- Tension pneumothorax

d- Flail chest

Man involved in car accident subcutaneous emphysema on the neck


a- Rupture of diaphragm

b- Rupture of trachea

Newborn, couple hours after delivery, resp. distress. ? Abdomen empty, no breathing
on the left

a- Left pneumothorax

b- Diaphragmal hernia

Man had craniotomy. Lab:…………………………….Urine osmolarity 805

a- Water intoxication

b- S I A D S- may be

c- Diabetes insipidus

Old man with a recurrent hemoptysis and cough…….

a- Recurrent pulmonary emboli

b- Chronic bronchitis

c- Teleangiectasis

d- Bronchiactasis /TB /pneumonia / Neoplasm

Sucking pneumothorax, what is management?

a- Tight air dressing

Man three attack of gout in left knee. Now present with a pain and swollen right knee,
what is Rx?
a- Indomethacin in the first 24 hours

b- Allopurinol in the first 24 hours

c- Hydrocortisone 100mg in the knee

d- Colchicine in the first 24 hours

What will decrease uptake to iodine on scintigraphy?

a- Hashimoto’s thyroiditis

b- Subacute thyroiditis

Joseph., Oct 23, 2007

#14

Joseph.

Joseph. Guest

. Small boy 2yrs, with wheezing, cough, symptoms stared after his older brother
birthday party:

a- Spiral CT of thorax

b- Indirect laryngoscopy

c- Bronchoscopy + direct laryngoscopy

Woman normal pregnancy, normal delivery, stillbom baby. The most likely explanation
is:

a- Unexplained intra-uterine hypoxia

b- Cord accident during delivery


Old lady sudden sharp abdominal pain with abdominal distension, long history of
constipation, no stool or flatus passed today:

a- Superior mesenteric embolus

b- Sigmoid carcinoma

c- Diverticulosis

Prodromal symptom of SCH:

a- Ideas of reference

b- Dysfunctional parenting

c- Unemployment

Diabetic woman depressed. What will be most ominous sign in prognosis?

a- Alcohol abuse

b- Decreased level of HbAlc

A man with a long standing rheumatic arthritis, Hb reduced MCV – normal ferritin
increased 3x……….

a- Anemia of chronic disease

b- Iron deficiency anemia

Woman second pregnancy, first child has spina bifida, and she wants advice regarding
her next pregnancy. All of the following is true except:

a- Preconceptual intake of folate and thiamine will not increase risk

b- Amniocentesis at 11 weeks

c- Ultrasound at 11 weeks to check Nuchal thickness


Young couple, advice regarding sterility. Your advice them IVF all of the following
except:

a- Regular anovulatory cycles

b- < 1.000.000 spermatozoid in ejaculate in 6 times

Young girl monospot test +-ve, clinical picture of infectious monocleosis. What is your
next step?

a-

b- Antibiotics

c- Paracetamol and bed rest

Mother HBsAg +-ve, newborn, what is correct Rx?

a- Normal immunoglobulin

b- Hepatitis B immunoglobulin + hepatitis B vaccine

50yrs old woman with enlarged spleen, moderate generalized lymphadenopathy. Most
likely is:

a- Chronic lymph. Leukaemia

b- Hodgkin disease

Best method to visualize biliary tract is:

a- Ultrasound
b- E R C P

c- X-ray

What is not feature of subdural haematoma?

a- Bradycardia

b- Hypotension

c- Lucid intervals

Characteristic of genitor-urethral tract in pregnancy is:

a- Dilatation of ureter

b- Dilatation of ureter and calyces due to progesterone

c- Reduced incidence of infection

Regarding post operative management of pain:

a- Pethidine is better than morphine

b- Older people require lower dose because they are more sensitive

c- Pethidine should not be repeated more than every 4 hours

d- Pethidine is given by calculating mg/kg of body weight

e- Pethidine has longer duration than morphine

Woman had baby with neural tube defect/spina bifida. She wants advice regarding her
next pregnancy. All is correct except:

a-

b- Preconceptual intake of thiamine is important in prevention of NTD/SB

c- She will have low amniotic alpha – fetoprotein


d- Environmental and social factors are important

Joseph., Oct 23, 2007

#15

Joseph.

Joseph. Guest

Patient with Helicobacter pylorus infection. He had 1 week treatment. What is simple
test to check eradication?

a- Complement test

b- Serologic test

c- Urea breath test

d- Endoscopy

e- Histological test

Patient with Wernicke’s encephalopathy. What is appropriate Rx?

a- 50 ml 50% dextrose

b- Immediate glucagon injection

c- Immediate glucose injection

d- Thiamine injection followed by glucose infusion

Erythema nodosum is associated with all of the following except:

a- Rheumatic fever

b- Tuberculosis
c- B-haemolytic streptococcus infection

d- Streptococcus viridans infection

e- Rheumatic arthritis

Amyloidosis can occur in all of the following except:

a- Chronic active hepatitis

b- Leprosy

c- Multiple myeloma

d- Tuberculosis

e- Renal failure

Short stature in adulthood is associated with all of the following except:

a- Congenital adrenal hyperplasia

b- Turner’s syndrome

c- Premature puberty

d- Obesity

e- Psycho-social deprivation

Uncomplicated haemorrhoids are associated with all of the following except:

a- Discharge with mucus

b- Blood

c- Pain

d- Protruding limp (bulging mass)

e- Pruritus ani
Young woman primigravida, 17 weeks pregnant, lower & sharp abdominal pain. Could
be all of the following except?

a- Red degeneration of fibroid

b- Ectopic pregnancy

c- Ruptured ovarian cyst

d- Acute appendicitis

Man was exposed to bank robbery 3 weeks ago…………..

a- Denial

b- Insight and hallucination

c- Self- defence

d- Irritability, outburst of anger

The risk of colonic cancer will be highly increased with:

a- Diverticulosis

b- Crohn’s disease

c- Post irradiation colitis

d- Familiar Melanosis coli

e- Ulcerative colitis

70 yrs old man present with fatigue. On psychical exam, slight pallor.

Blood: Hb 96, MCV 118, WBC decreased, Platelet low

Peripheral blood picture show few hyper-segmented polymorphonuclear cells.


a- Autoimmune haemolytic anemia

b- Pernicious anemia

c- Anemia of chronic disease

d- Fe deficiency anemia

Joseph., Oct 23, 2007

#16

Joseph.

Joseph. Guest

Regarding Rosacea what is correct:

a- Acneiform pustules

b- Topical corticosteroids will help

c- Diet has role in aetiology

d- Rhinophyma is not associated with the disease

e- Sunlight exposure helps in the treatment

Woman are scheduled for laparoscopy. She is concerned about that so you should tell
her all of the following except:

a- It is life threaten particularly if vena cava is damaged although risk is very small –
may be

b- Small amount of CO2 will be residual after procedure so she can experience
abdominal tenderness and shoulder tip pain

c- Although it is not standard procedure polycystic ovarium sy, can be easily diagnosed
by laparoscopy

d- Rupture of intestine
e-

I thing B, because first part of statement is true, small amount of CO2 will be residual in
abd. After laparoscopy but second part: abd. tenderness and shoulder tip charac, of
ruptured ectopic pregnancy

Regarding oral contraception all of the following is true except:

a- Reduced incidence of Fe anemia

b- Decreased triglycerides

c- Reduced pelvic inflammatory disease

d- Incidence of endometrial carcinoma

A man had injury sharp pain and knee effusion playing soccer a month ago. He pause for
month and start to play again. Now presented with pain in the knee that is often locked,
give away and he can’t extend his leg:

a- Displaced fracture of Patel

b- Rupture of anterior cruciate ligament

c- Rupture of medical collateral ligament

d- Medial meniscus

Woman on hormone replacement therapy have hot flushes, headache, she can’t
sleep. She has been now prescribed Progesterone for the first 12 days (or 15) of each
month. All her symptoms will improve except:

a- Hot flushes

b- Dry vagina

c- Sleep
d- Increased incidence of endometrial carcinoma

e- Osteoporosis

All of the following are features of pulmonary embolism except:

a- Pleuritic rub

b- Dyspnoea

c- Bronchial breathing

Patient punctured his finger in first digital crease level. Patient has on passive
stretching:

a- Midpalmar abscess

b- Tendosynovitis

Patient from car accident came in ER. he has been treated for his injuries but soon later,
he start bleeding from multiple puncture siestas a part of investigation for his bleeding,
you will do all of the following except:

a- Platelet count

b- Fibrinogen

c- Platelet aggregate study

d- Prothrombin time and A P T T?

e- Degradation fibrin

Joseph., Oct 23, 2007

#17

Joseph.
Joseph. Guest

Which of the following statements is/are true concerning metastatic cancer in the liver?

a- Squamous cell cancer in the head and neck region commonly metastatic to liver

b- In Australia gastrointestinal cancer more commonly metastatic to liver than does any
other malignancy.

c- Once metastatic disease has been diagnosed in the liver the condition cannot be
cured by surgical operation.

d- Average life-expectancy for a patient with diagnosed metastases in the liver without
treatment is in the order of 2-3 years.

e- Carcinoma embryonic antigen titre is a useful diagnostic test for all metastatic liver
cancer.

Indicators of a poor prognosis in acute pancreatitis include -

a- Marked initial elevation of serum amylase

b- Arterial pO less than 60 mmHg

c- Serum calcium concentration less than 1.8 mmol/L

d- Haematocrit increase greater than 10%

e- Urea greater than 10mmol/L

Pancreatic cancer is associated with a poor prognosis because -

a- It is a rapidly growing tumour

b- It has usually spread beyond the pancreas at the time of presentation.

c- It seldom arises before the age of 70 years

d- It is associated with anorexia and weight loss tendering the patient unfit for surgery

e- the presence of jaundice indicates that the cancer is incurable


Of the following malignant tumours indicate which two are most likely to respond
favourably to treatment with chemotherapeutic agents.

a- Squamous carcinoma of lung

b- Chondrosarcoma

c- Wilm’s tumour

d- Teratoma of testis

e- Glioblastoma

Which of the following statements is/are correct about cancer?

a- The most common site for gastric cancer is the fundus of the stomach.

b- Previous hepatitis B infection is recognised as a predisposing factor Hepatoma.

c- Cancer of the bile duct is most common in Southeast Asia.

d- There is a causal relationship between Diverticular disease and colon cancer.

e- Pipe smoking is a risk factor in cancer of the floor of the mouth.

aippg.com

Sympathectomy may be expected to improve-

a- Intermittent claudication

b- Rest pain

c- Ischemic ulceration

d- Varicose ulceration

e- Deep vein thrombosis

A 72 years old non-diabetic man presents with a three year history of intermittent
claudication and a four week history of severe rest pain in his left lung. On examination
there are no pulses below the common femoral pulse, the big toe demonstrates
superficial ulceration and gangrene. He should -

a- Have an angiogram to assess suitability for bypass grafting

b- Have an amputation to the toe with primary wound closure

c- Have an amputation of the toe with secondary wound closure

d- Have a lumber sympathectomy

e- Be placed on bed rest and given beta blockers

Which of the following is/are true of varicose veins?

a- They are usually related to saphenofemoral incompetence

b- They can often be effectively treated by Sclerotherapy

c- The long saphenous vein should always be stripped

d- There is often an inherited predisposition

e- They often commence during pregnancy

Which of the following is/are correct for peripheral vascular disease?

a- Patients with claudication usually progress to threatened limb

b- An absent dorsalis pedis pulse is invariably indicative of vascular disease

c- An upper abdominal bruits is usually associated with renal artery stenosis

d- 50% or more of patients presenting with lower limb claudication have ischemic heart
disease

e- Is more common in patients with analgesic nephropathy

Raynaud’s disease is a condition in which -

a- Small arterioles go into spasm when exposed to cold conditions


b- Capillaries tend to dilate in the extremities

c- Young women are affected more commonly

d- Fingers are affected more commonly than toes

e- Sympathectomy may be expected to give permanent relief

Joseph., Oct 24, 2007

#18

Joseph.

Joseph. Guest

Typical features of a large arterio-venous fistula are -

a- Decreased pulse pressure

b- Bradycardia

c- Dilated local veins

d- A bruit over the fistula

e- Left ventricular hypertrophy

A 50 years old male smoker presents with a history of right calf claudication at 100
meters. He has a god right femora pulse but no pulses distally. Which of following would
you advice as initial management?

a- Stop smoking

b- Regular walking

c- Check fasting glucose and lipids

d- Femoro-popliteal bypass

e- Laser angioplasty
Which of the following blood tests is/are indicators of nutritional status?

a- Albumin

b- Transferrin

c- Retinol binding globulin

d- C-reactive protein

e- Pre-albumin

Which of the following is/are true of total parenteral nutrition?

a- It is best delivered via a jugular vein catheter

b- It requires phosphate replacement to avoid hypophosphataemia

c- It is superior to enteral feeding, even if the gastrointestinal tract is available

d- May cause hyperglycaemia

e- Requires biochemical monitoring

Which of the following is/are true of enteral feeding?

a- can be delivered via a Percutaneous gastrostomy tube

b- Can cause aspiration pneumonia

c- May cause osmotic diarrhoea

d- Is cost-effective nutritional support

e- Should never be used post-operatively

http://www.aippg.net/forum/posting.php?mode=reply&t=69347
A 25 year old male is delivered to the A & E department by ambulance after a high
speed motor vehicle accident in which he has sustained blunt trauma to the head, face
and chest. He is unconscious and cyanosed with pulse 120 and BP 90 systolic gurgling
sounds accompany his shallow, laboured and rapid respiratory efforts. There is much
blood around the nose and mouth. Which of the following do you require first to initial
resuscitation?

a- Intercostal tube, underwater seal drainage apparatus, minor surgical instrument pack

b- Large calibre intravenous cannulae, IV fluid administration set and colloid solution

c- Oropharyngeal airway, light, suction, oxygen mask

d- Tracheostomy tube, suction, minor surgical instruments pack

e- Endotracheal tube, laryngoscope, suction, ventilator

Which of the following are the common modes of presentation of blunt abdominal
trauma?

a- A shocked patient with involuntary guarding on abdominal examination

b- The patient presents with hypovolaemic shock and no source of blood loss can be
found in the chest, the extremities or the soft tissue of the trunk.

c- The patient has a rising blood and a falling pulse rate, and signs of altered
consciousness.

d- The patient has board-like rigidity of the abdomen from the time of the accident and
a chest x-ray taken in the upright posture shows free gas under the diaphragm.

e- A shocked patient has fractures of the lower ribs

A 30 year old man arrives at hospital 40 minutes after being involved in a motor vehicle
accident. He has already received 1000 mls of plasma substitute during transport. He is
unconsciousness and responds only to pain. There are no focal neurological signs. There is
stridor, intercostal retraction and no evidence of pneumothorax pulse rate is 140/min.
blood pressure is 70 mmHg. Systolic and the abdomen is rapidly distending. There is a
right femoral shaft fracture and there is ischaemia of the lower right leg. Before
proceeding to operating theatre for laparotomy. This patient needs -
a- CT scan

b- Peritoneal lavage

c- Rapid transfusion

d- Adequate airway

e- Right femoral angiography

Joseph., Oct 24, 2007

#19

Joseph.

Joseph. Guest

A young man presents after a motor vehicle accident with clinical evidence of head
injury, unconsciousness, tension pneumothorax, hypotension and expanding abdomen,
select the three highest priorities from the following options -

a- Blood volume replacement

b- CT scan of the head

c- Chest x-ray

d- Application of MAST suit

e- Decompression of pneumothorax

http://www.aippg.net/forum/posting.php?mode=reply&t=69347

After total thyroidectomy -

a- Bilateral vocal cord paralysis is common


b- Acute haemorrhage in the neck wound is life-threatening because of resulting
hypovolaemia

c- Transient hypocalcaemia may occur

d- Tracheostomy is often required because of trachemalacia

e- Long-term maintenance thyroxine therapy should begin six months after operation

www.aippg.com

Ulcerative colitis e-recurrent episodes of attack jaundice e- high alkaline phosphates-


What is your diagnosis?

a- Cholelithiasis

b- Sclerosing cholangitis

c- Primary biliary cirrhosis

Snake bite no bleeding -

a- Close observation for 24 hours

b- Hospitalization and antivenom

c- Tourniquet apply

d- Cut & free bleeding from puncture site

Rest pain Awake from sleep sweet 377 -

a- Ischaemic pain

b- Claudication distance (for muscle pain)

Sudden serve backache pain -

a- Leaking aneurysm
40 yrs old two days sever pain in his scrotum -

a- Acute Epididymitis

b- Torsion

c- Tumour

d- Hydrocele

A young adolescent present with sudden sever pain -

a- Needs surgery

Most appropriate for shape size & function of the urinary system -

a- Ultrasound

b- IVU

c- Renal scan

After pelvic injury blood in the urethral ureatus -

a- Urethro-graphy

Pneumathorax-

a- Inter costal tube

b- Intubation

c- 20 cc bore needle drainage


Joseph., Oct 24, 2007

#20

Joseph.

Joseph. Guest

Sueking chest injury, surgical emphysema, noneckvein enlarged, heart sound ok,
breathing problem - What is your management?

a- Monogestric intubation

b- Oral intubation

c- Thoracotomy tube drainage

http://www.aippg.net/forum/posting.php?mode=reply&t=69347

Complication of colitis -

a- Malunion

b- Non union

c- Carpal tunnel syndrome

Gout e- knee arthritis confirmatory test -

a- Bifringement crystals

Thyroid tissue present on the aspirated dup cervical lymph node -

a- Lateral aberrant thyroid

b- Thyroid Ca
Familial associated Ca-

a- Medullary Ca

b- Follicular Ca

c- Mucinous Ca

d- Anaplastic Ca

Picture- Red spot on the Rt. side of the face -

a- Port wine lesion

b- Capillary Humour glioma

c- Melanotic freckle

d- Basal cell ca

Picture - Breast lump of a 45 yrs old woman & mammography scan -

a- Ca breast

b- Cyst

c- Fibro adenoma

Picture- Pyloric stenosis Rt. e- vomiting which fluid is used -

a- Urosomal saline

b- Hartmann’s solution

After intusion his urine out put is 50 ml/hr. How much Rt. is used for the next 24 hrs?

a- No rt. supplement
b- 20 mmol Rt

c- 55 mmol Rt.

d- Above 55 mmol Rt.

e- none of the above

Picture of thigh, look like black spot around & above the medial maleolus -

a- venous hypertension

b- Arterial hyper/ischaemia

40 yrs old man e- left textis enlarged on recumbent position -

a- Varicocele

b- Seminoma

c- Teratoma

d- Hydrocele

e- Epididymitis

Joseph., Oct 24, 2007

#21

Joseph.

Joseph. Guest

Picture of the hand bone (scaphoid) montegia -

a- Plaster up to the proximal interphalangeal joint of the thumbs


Patient comes e- renal colic and wants pathedimic, doctor examines him but no pain
associated e- complains he has came e- bloody urinary sample. On examination physically
he is alight. What is your initial management?

a- Fresh urinary sample examination

b- Morphine & buscopan injury and follow up after 24 hrs

c- Immediate hospitalization

www.aippg.com

Young man coming back after Bali visit, he has lost r kg weight & diarrhoea 3 wks visit
otherwise physical examination ok. What is your appropriate management?

a- Stool culture

b- Flexible sigmoidoscopy & rectal biopsy

c- Do colonoscopy

After severe vomiting of complain heart burn distress, surgical emphysema -

a- Rupture oesophagus

b- Rupture trachea

c- Rupture diaphragms

Myonecrosis Rx -

a- Cleaning & excision of the devitalized tissue & apposition the lays appropriately

b- Hyper baric O2

c- Hyper immune globulin


Hashimotor thyroiditis except -

a- Hypothyroidism

b- Hyperthyroidism

c- Auto immune antibody

Appends sectomy after 5 days on the medial end of the opsation side- where is a
fluctuating swelling present, what is your management?

a- take in the operation room

b- Remove the under lying suture

c- Incision & drainage

Paraplegia patient e- over hoe incontinence what is your management?

a- Suprapubic catheterization

b- Persistant penile catheterization

c- Inter mi hewed

A woman comes e- bladder come to the xiphoid process -

a- Acute retention

b- Cl retention

c- Neuro genic bladder

Patient presents e- 1st degree haemorrhoids what is your management?

a- Roughage diet

b- Opsalion

c- Sclerosis throsy
http://www.aippg.net/forum/posting.php?mode=reply&t=69347

Joseph., Oct 24, 2007

#22

R.dass

R.dass Guest

Regarding CRF all of the following can be used to reduced hyperkalemia except -

a- Calcium

b- Glucose and insulin

c- Magnesium

d- Sodium bicarbonate

e- Resonium

A newborn was peripherally cyanosed and crying lustily. The axillary temp was 37.8C.
What would be the next management?

a- Take an x-ray

b- Reassure

c- Oxygen

d- Urine culture

10/52 old child with persistent unilateral eye discharge responding to antibiotics but
recurring -

a- Nasolacrimal duct obstruction


b- Gonococcus infection

c- Chlamydia infection

All of the following are features of hypercalcemia except -

a- Depression

b- Diarrhoea

c- Polyuria

d- Polydipsia

18/12 old girl breast tanner stage 3, no pubic or axillary hair -

a- Idiopathic thelarche

b- Premature puberty

c- Normal variant

d- Turners syndrome

19yr old lady presented with frequent syncopal attack of 6 wks duration. PR 44/min, BP
180/110 and ejection systolic murmur in the aortic region. Probable DX -

a- Cardiac arrhythmia

b- Vasovagal attack

c- Complete heart block

d- Aortic stenosis

2yr old presented with recurrent chest infection and FTT found to have continuous
murmur below left clavicle and upper if sternal border. Probable diagnosis -

a- PDA b- ASD
c- VSD d- AS

19yr old primi is not followed the an clinic for last 8 wks came in her 39/40 POA, found
to have transverse lie, what will you do next?

a- Immediate CS

b- U/scan

c- External cephalic version

d- Vaginal examination

e- Trial of labour

O2 saturation in SVC - 70%, IVC - 80%, RA - 90%. What could be the examination
finding?

a- Fixed split in sternal border

b- Continuous murmur in the upper if sternal

c- PSM in lower sternal border

Vitamin B complex deficiency will cause which of the following eye complication -

a- Optic neuropathy

b- Retinal pigmentation

c- Papilloedema

d- Cataract

R.dass, Oct 24, 2007

#23

R.dass
R.dass Guest

One yr old child, with If testes in the scrotum and Rt/testes couldn’t be found in the
inguinal region. WOF statement is true -

a- More chance to get torsion of If testes

b- If you do orchidopexy at 3 yr child’s spermatogenesis will be normal

c- Chance to get malignancy of his If/testes is more common than other children

Erythema nodosum is presented in -

a- Sarcoidosis

b- Rheumatoid arthritis

c- Leprosy

d- Rheumatic fever

e- Tuberculosis aippg.com

A lady presented with insomnia (mainly early morning), weight loss and loss of appetite.
She doesn’t like to get up from bed feeling not worth. Possible diagnosis -

a- Somatization

b- Generalized anxiety disorder

c- Major depression

d- Hyperthyroidism

e- Obsessive compulsive disorder

Common side effect of atypical anti psychotics -

a- Neutropenia
b- Sexual dysfunction

c- Weight gain

d- Parkinson

A patient presented with hemiplegia and recovered in few hrs. his CT scan was normal.
What will you do next?

a- Treat with anti-thrombolytic agent

b- Treat with anti-coagulant

c- Do nothing

d- Give aspirin

A patient presented with transient amnesia and blurred vision bilaterally. Where is the
possible side of lesion?

a- Middle cerebral artery

b- Internal cerebral artery

c- Vertebral basilar artery

d- Anterior cerebral artery

Which one of the following is cause for post coital bleeding?

a- Fibroid b- Cervical polyp

c- Candidiasis

Cause of acute retention of urine, except -

a- Spinal cord lesion

b- Genital warts
c- Retroverted simplex

d- Herpes simplex

Which condition will not associated with Raynaud’s phenomenon

a- Rheumatoid arthritis

b- Scleroderma

c- Ankylosis spondylitis

d- PAN e- SLE

13yr old girl presented few months after attained menarche, with heavy periods each
lasting about 10 days. All of the following are correct

a- She may need iron supplement

b- Her periods can be get regular after sometimes

c- If she wants to become pregnant she will need Clomiphene

d- Can be treated with NSAID

R.dass, Oct 24, 2007

#24

R.dass

R.dass Guest

26 yr old lady presented with hematemesis about 1L of blood few hrs ago with 2
episodes of melena. After IV cannulation, what is the next step in the management/

a- Immediate surgery

b- Give Omeprazole IV
c- Immediate Gastroscopy

d- Sucralfate

http://www.aippg.net/forum

Newborn male child with normal genitalia -

a- XXY

b- Testosterone ineffective in the end organ

c- Female child with 17 OH deficiency

d- Mother received cyproterone from 8 wks of POA

During transition from fetal circulation to neonatal, except -

a- Increased RV out put

b- Closer of ductus arteriosus

c- Closer of ductus venous

d- Cessation of blood flow through F.ovale

e- Increased blood flow to pulmonary region

70 yr old lady with pneumaturia, what is the underlying cause

a- CA colon

b- Pelvic inflammatory disease

c- Diverticular disease

d- Tuberculous caecum

e- Ulcerative colitis

A lady traveled from UK in a plane 1/52 back, presented with acute chest pain. What
could be the cause (ECG given)?
a- Inferior infarction

b- Ventricular fibrillation

c- Pulmonary embolism

d- Atrial flutter

Deep vein thrombosis can be reliably diagnosed by -

a- Calf Tenderness

b- Hoffman sign

c- Swelling of calf

Regarding mycoplasma pneumonia

a- Caused by mycoplasma hominis

b- Common presentation is pharyngitis

c- Common in immune compromised person

d- Common cause of labour pneumonia

Which of the following statement is incorrect -

a- Strept pneumonia is the commonest organism causing lung abscess

b- Psittacosis infection is commonly presented with splenomegaly

50 yr old lady presented with intermittent sudden severe facial pain on one side
associated with sensory loss on that side. Possible diagnosis -

a- Trigeminal neuralgia

b- Post herpetic neuralgia

c- Multiple sclerosis
d- Migraine

e- Facial nerve palsy

Hypertension is the important risk factor for which of the following

a- Coronary artery disease

b- Renal failure

c- Ischemic stroke

R.dass, Oct 25, 2007

#25

R.dass

R.dass Guest

A diabetic patient takes 20 unit of short acting insulin and 10 unit of intermediate acting
insulin in the afternoon. Wakes up around 4am with sweating. Which of the following
treatment is best for prevention of this attack?

a- Reduce the short acting insulin

b- Increase the short acting insulin

c- Reduce the intermediate acting insulin

d- Increase the intermediate acting insulin

e- Reduce both short and intermediate acting insulin

aippg.com

An old lady presented with extended bladder up to umbilical without pain. Possible Dx

a- Chronic urinary retention

b- Neuropathic bladder
c- Herpes infection around Vulval area

Elderly abuse is common in Australia. Which of the following is true?

a- Elderly female are more commonly affected than elderly male

b- More often by unknown person

c- The abusers are more likely mentally retarded

d- Mentally ill is least abused

e- Physically ill is least likely affected

A lady presented with frequent hot flushing and dry vagina. Which of the following
management will be more efficient and won’t cause spotting?

a- Oestrogen and progesterone both daily

b- Oestrogen daily with progesterone for last 12 days

c- Oestrogen patch

d- Oestrogen pessary

An 18 yr old dressed as a female came to you his boyfriend for problem in sexual
relationship. He always felt uncomfortable with his sex since childhood. He has been
dressing several times in past. What is this phenomenon?

a- Homosexual

b- Transsexualism

c- Transvestism

d- Masochism

e- Gender identify disorder


A lady has dog phobia but her boyfriend have a germen shepherd dog. WOF would most
appropriate initial management?

a- To tell her to ask her boyfriend to have a small dog

b- To tell her to ask her boyfriend to have a cat instead of dog

c- Ask to leave her boyfriend

d- Ask her to read books regarding dog with some shepherd picture

A lady 150cm, in labour, Cx effaced and 4cm dilated and head at 0 station. With intact
membrane, what is the next management?

a- Trial of labour

b- LSCS

c- U/scan detect fetal abnormality

d- X-ray pelvimetry

e- Oxytocin

A patient has a history and suggestive of hemochromatosis. What is the investigation


you will do to screen the relative?

a- Liver biopsy

b- Ferritin level

c- Transferin level

d- CT abdomen

A baby born to HbsAg +ve mother, what is your management?

a- vaccinate the mother

b- Reassure
c- Separate the child from mother

d- Give immunoglobulin to mother

e- Give hyper-immunoglobulin and Hep B vaccine to baby

Postmenopausal woman with decreased libido. All of the following are the cause except

a- Lack of ability of orgasm

b- Loss of pelvic muscle strength

c- Atrophy of vagina

d- Dryness of vagina

R.dass, Oct 25, 2007

#26

R.dass

R.dass Guest

A pregnant prime G1P0 with 31 weeks came with breech presentation what will you do

a- Urgent U/scan

b- Do Vx examination

c- Do x-ray pelvimetry

d- Do cephalic version

e- Review in 2 week time

WOF association is correct -

a- Mother with platelet count 100 thousand (150-250 thousand) - neonatal


intracerebral haemorrhage
b- Mother with Antiphospholipid ab - IUGR

c- Mother with anti RO ab - neonatal heart block

d- Mother with proteinuria of 4g/I in the 1st trimester - material renal impairment in
3rd trimester

e- Mother with lupus antibody - she has more risk of pul thromboembolism

aippg.com

Regarding carpal tunnel syndrome -

a- Most common presentation is regular awakening in the night with pain

b- Tingling pain along the course of median nerve

c- Paresthesia over little finger

A patient with chronic ulcer on the lower limb, and pain become worse when elevated.
The likely cause is

a- Perforator incompetence

b- Ischemic ulcer

c- Venous ulcer

d- Streptococcal infection

e- Diabetic ulcer

Ascities with malignant cells what is true -

a- Palpable nodules on rectal examination

b- Supraclavicular LN

c- Liver enlargement

d- Splenomegaly

e- Generalized lymphadenopathy
Regarding CTG patterns which is correct -

a- Small (<10bsm) deceleration later and almost at the end of uterine contraction, but it
takes time to reach the base line - foetal hypoxia

b- Loss of beat to beat variation over a period of 5min - need urgent delivery

A patient developed motor weakness of R/face, arm and leg, sensation intact. The site
of lesion -

a- L/middle cerebral artery

b- L/anterior cerebral artery

c- L/internal capsule

d- Vertebro - bascilar artery

In colorectal ca, what percentage of stage duke A is diagnosed by faecal occult blood -

a- Less than 5% b- 15%

c- 25% d- 50

e- More than 75%

Common cause of bloody discharge from nipple -

a- Duct ectasis

b- Intra ductal papiloma

c- Pagget disease

d- Ca breast

Point prevalence in schizophrenia means that -


a- Cases in one year

b- Current cases at that time

c- The total number of cases

13yr old child with 9yr old height, WOF indicate good prognosis for his final height?

a- His bone age is 16yr

b- His bone age is 13yr

c- His bone age is 9yr

R.dass, Oct 25, 2007

#27

R.dass

R.dass Guest

After recovery from fever a child developed a neck swelling which is translucent and is
located in the poster triangle of the neck. What is the possible diagnosis?

a- Cystic hygroma

b- Tuberculous adenitis

c- Branchial cyst

d- Bacterial adenitis

e- Metastatic LN

A patient was treated with anti-H.pyloric regimen for recurrent peptic ulcer. WOF is
safest and least inconvenient Ix to evaluate the effectiveness of treatment

a- Serology
b- Biopsy

c- Serum Gastrin level

d- Ba meal x-ray

e- Urea breath test aippg.com

A patient presented with sweating and agitation, probably opioid toxicity. What is your
treatment?

a- Naltreaxone

b- Naloxone

c- Diazepam

d- Diuretic

e- Anticholinergic

Postoperative patient, his pt chart shows 2 L i. V infusion, 2L loss from nasogastric tube,
urine out put 700 ml (N = 1500ml or > 50ml/h, oliguria < 400ml/h). His potassium level is
3.4 (3.5 - 5mmol/l). What will you order for next 24 hrs?

a- 3L normal saline + 1L 5% dextrose + 100mmol KCL

b- 2L Hartman + 1L normal saline + 100mmol KCL

c- 4L normal saline + 50mmol/l

d- 3L Hartman + 1L 5% dextrose

Which of the following suggestive pulmonary embolism?

a- CXR- normal, ventilation scan - normal, perfusion scan - reduced

b- CXR- normal, ventilation scan - normal, perfusion scan - normal

c- CXR- abnormal, ventilation scan - reduced, perfusion scan - reduced


d- CXR- normal, ventilation scan - reduced, perfusion scan - reduced

Which fining is more common in pleural effusion due to TB chest?

a- Low or absent glucose

b- Mononuclear cells

c- Blood stained effusion

d- Protein < than 3g/dl

A patient was prescribed sertraline for depression he was scared that he might develop
dependency and impotence. What should you advise?

a- Yes he is correct

b- He may develop impotence but it is unlikely to cause dependency

c- Both these side effect are not related to sertraline

A man underwent anterior resection for ca rectum, on the 5th day of operation there
was a copious serosanguineous fluid coming through surgical wound. What is the
expected complication?

a- Failure in anastomosis

b- Concealed haematoma

c- Wound infection

d- Wound dehiscence

What are the complication excepted inmonochorionic di-amniotic twins than


dichorionic di-amniotic twins, except -

a- Acute poly-hydramniosis

b- Cerebral palsy in second twin


c- Feto-fetal blood transfusion

It is important to assess mental state of adolescence, all of the following finding are
normal except -

a- Fighting with their siblings

b- Disregarding their physical well being

c- Most of the time accompanying with peers

d- Conflict with their parents regarding the religion and cultural believes

A patient who had a reflux esophagitis came with progressive dyspepsia of 2-mo
duration. He is otherwise normal. Possible cause -

a- Oesophageal CA

b- Oesophageal stricture

c- Plummer Wilson disease

d- Psychogenic

A child has abnormal blinking movement of eyes, which was more on watching TV and
when he was tired and stressed. They are present even during examination -

a- Habitual tic

b- Partial complex seizure

c- Myoclonic epilepsy

Picture of U/scan and mammography of breast lump given lump seems to be cystic in
u/scan and homogenous round mass in the mammography, what is the next
management?

a- excision biopsy
b- Incision biopsy

c- Fine needle aspiration

d- Mastectomy

e- Radiotherapy

R.dass, Oct 25, 2007

#28

R.dass

R.dass Guest

35 yr old lady came to you with a h/o mild menorrhagia and anxiety, recently she feels
her husband is detected from her and she is closed to her husband as she used to be, she
has two children living in rented house and bought all household things in higher
purchase, they can’t afford to go out together for dinner. Examination was normal.
What is the next step of management?

a- Refer her to a psychiatrist aippg.com

b- Send her for marriage counselling

c- Interview her husband

d- Advise them to take a holiday

e- Refer her to the gynecologist

Which of the following is correct regarding ca nasopharynx?

a- Arises from lymphatics

b- Not sensitive to radiotherapy

c- Respond well to chemotherapy

d- Associated with EB virus infection


e- Locally destructive lesion

All of the following can cause serotonin syndrome except -

a- Haloperidol

b- L - tryptophan

c- Moclobemide

d- Citalopram

e- Imipramine

Nullipara came with menorrhagia, what is the cause except?

a- Adenomyosis

b- Ux myoma

c- Sub mucus myoma

d- Inadequate luteal phase of progesterone

e- Subserosa

A lady held up during an armed bank robbery 3 wks back, she was extremely distressed
and came to you, what would be her likely reaction?

a- Irritability and angry

b- Hypersomnia

c- Amnesia

Regarding Parkinson disease, which is true?

a- Ataxia

b- Hyper reflexia
c- Impaired postural reflex

d- Inability to look downward

50Yr old woman complain her husband will die of ca though he is perfectly healthy, she
also belief that she is the member of secret service. On examination she appeared to be
normal and her premorbid personality was normal, when you asked about her belief she
became withdrawn and gave wake answers what is the diagnosis/

a- Delusional disorder

b- Paronia

c- Schizophrenia

d- Organic mental disorder

e- Dementia

In Australia the most common cause of jaundice in pregnancy in 3rd trimester is -

a- Hepatitis A

b- Hepatitis B

c- Hepatitis C

d- Cholestasis

d- Choledolithiasis

Which of the drug will reverse the effect of heparin?

a- Vitamin K

b- Protamine sulphate

c- APPT
28 yr old female came with a soft swelling in upper 1/3 of anterior triangle, partially
covered by anterior border of sternomastoid muscle. What is the diagnosis?

a- Branchial cyst

b- Thyroglossal cyst

c- Tuberculous abscess

d- Lymph node

R.dass, Oct 25, 2007

#29

R.dass

R.dass Guest

A lady came with POA of 41 wks, which of the following will be excepted -

a- IUGR

b- CPD

c- Meconium aspiration

d- Fetal death rate will increase by 0.12% if pregnancy continues for another wk

e- If fetal well being is good, con allow the pregnancy to continue for next 3-7 days

Pregnant female in her 24/40 POA complaining of calf pain, Doppler done and DVT
confirmed. Before commencing anti coagulant therapy, what are the lx can help to
exclude congenital defect, except?

a- INR

b- Anti thrombin 111

c- Protein S
d- Factor V modulline

aippg.com

Intermittent - claudication, which one of the artery is more commonly affected -

a-lliac artery

b- Superficial femoral artery

c- Profundus artery

A female patient with a h/o self harm, which is the important bad prognostic finding?

a- Past h/o similar attempt

b- F/H of schizophrenia

c- Poverty

d- Depression

e- Alcoholism

Psychodynamic psychotherapy is useful in -

a- Schizophrenia

b- Specific phobia

c- OCD

Primary ovarian failure, what is the important lx?

a- Reduced progesterone

b- Increased LH

c- Reduced oestrogen

d- Increased FSH
Regarding Mullerian agenesis, except -

a- Normal breast development

b- Short Vx

c- X linked recessive

All can be beneficial in treatment of chronic venous ulcer except -

a- Elevation of limb

b- Compression stocking

c- Topical antibiotics

d- Ligation of superficial veins

e- Stripping of veins

Regarding ca Cx, which will improve the early detection of CIN lesion?

a- Make available of Pap smear to more people

b- Improving screening method to detect CIN early

X-ray abdomen showing multiple fluid levels, most probable diagnosis?

a- Early small bowel obstruction

b- Late small bowel obstruction

c- Large intestine obstruction

d- Rectal ca

Child came with earache, on examination red tympanic membrane, which is bilging.
WOF is true -
a- Local antibiotic

b- Cleaning the ear

c- With the rupture of membrane pain will be immediately relieved

R.dass, Oct 25, 2007

#30

R.dass

R.dass Guest

Football player while turning got knee pain and swelling in ½ hr, possible Dx -

a- Rupture of cruciate number

b- Rupture of cruciate ligament

c- Medial ligament

d- Patellar tendon

Dorsal aspect of the hand ulcer and nodule, red and black in colour -

a- Basal cell ca

b- Squamous cell ca

c- Angioma

d- Melanoma

7 mo old child presented with funny flexion movement and during that time he is not
responding, as he is used to. What is the likely cause?

a- Febrile seizure

b- Infantile spasm
c- Breath holding attack

d- Infantile myoclonic seizure

e- Petit mal epilepsy

Thalassemia WOF is incorrect -

a- Detective globulin chin production

b- If both parents are T. minor, 25% of their children have T. major

c- If lady with T. minor become pregnant she needs frequent blood transfusion

8 yr old child presented with fever and jaundice and found to have 4cm splenomegaly.
She had a p/h of neonatal jaundice needed phototherapy for several days. After that
frequent h/o upper Rt/abd pain and jaundice. Possible diagnosis -

a- Pyruvate kinase deficiency

b- G6PD deficiency

c- Spherocytosis

d- Viral hepatitis

18 mo old child brought by grandmother who says that baby babbles but not hearing
well. The hearing test done at 10 mo of age was normal. What is the next step?

a- Reassure that this is normal

b- Repeat the hearing test now

c- Audiological assessment aippg.com

Elderly female came with confusion; she is on digoxin 0.125 mg and Diltiazem. ECG was
given - ? A. fibrillation or couple beats (digoxin toxicity). What is the next management?

a- Increase the digoxin dose to 0.25mg


b- Increase the dose of Diltiazem

c- Stop all treatment and review in 12 hrs time

Regarding MVP which is true -

a- Prone to endocarditis

b- Diastolic murmur

2CT scan of a pt one with contrast and other without contrast was given. A lady
presented with sever headache on vertex and vomiting. Possible cause -

a- Closed head injury

b- SAH

In the limbic system area of affect -

a- Cingulate gyrus

b- Amygdala

c- Thalamus

d- Carpus callosum

3 yr old girl presented with periorbital oedema, proteinuria 3+, all are true except -

a- 90% of cases will improve with Prednisolone

b- Most of the cases are doe to focal glomerulonephritis

c- Penicillin prophylaxis should be given

d- Prognosis is good
Accessory nerve supply to which muscle -

a- Pterygoid muscle

b- Temporalis

c- Masseter

d- Sternomastoid

Which hematogenous organism is commonly causing septic arthritis?

a- H. influenza

b- Strept. Pneumonia

c- Staph aureus

d- Strept. Pyogen

2 yr old with intermittent bile stain vomiting in between normal, otherwise clinically
normal, possible diagnosis -

a- Volvulus

b- Pyloric stenosis

c- Duodenal atresia

d- GOR

e- UTI

ECG with short history -

a- Wolf Parkinson syndrome

ECG with short history -

a- Inf MI
b- Anterio-lateral MI

c- Anterio-septal MI

ECG with short history -

a- LVH + RBBB

b- RVH + LBBB

ECG short history except -

a- Atrial block with variable block

R.dass, Oct 25, 2007

#31

R.dass

R.dass Guest

A patient with years of being unwell presents with a creatinine of 1.2mmol/l and high
urea. He is vomiting and is anaemic. All the following are true, except:

a- Minimal change clomerulonephritis

b- Membranous glomerulonephritis

c- Diffuse proliferative glomerulonephritis

d- Focal glomerulonephritis

e- Hypertensive nephropathy

aippg.com
Cerebral infarction:

a- Homeis syndrome with increased sweating

b- Papilloedema with optic neuritis

c- Pain and sensation loss on the same side of the body

d- Quadriplegia with anterior or middle cerebral artery infarct

e- Unilateral blindness with anterior circulation

A 52 yrs old male patient has ischaemic type chest pain for three hours and has a
normal ECG:

a- If the ECG is normal, he does not have a cardiac problem

b- He should have an exercise stress test immediately

c- Thrombolyties are usually not indicated

d- Heparin and IV nitrates are indicated

e- Wait for any treatment until cardiac enzymes are back

Regarding a serum lipid profile and diet, what is true?

a- Red wine improves LDL/HDL ratio

b- Olive oil will increase the LDL/HDL ratio

c- Positive energy balances increase LDL

d- 100-300g of fish per week will have no effect on plasma or membranes

e- An increase in dietary saturated fatty acids will increase VLDL

Regarding the treatment of pager’s disease, the following is/are true:

a- Patient should be given calcitonin


b- Diphosphonates are indicated

c- Cortisol

d- May need chemotherapy

e- 1000mg calcium everyday

Treatment of chronic renal failure:

a- Antacid syrups can helps

b- Oral phosphate binding agents halt progress of bone disease

c- All patients will have symptomatic bone disease

d- Uraemic patients need to be dialysis

e- Hyperkalaemia can only be treated with dialysis

In regards to polycythemia rubra Vera which is correct:

a- Can be caused by obstructive sleep apnoea

b- Erythropoietin levels are high

c- Iron deficiency is common, so iron supplement should be given

d- ESR is increased

e- Clinical cyanosis may be difficult to detect

In a patient with known aortic stenosis god left ventricular function and clinical
presentation with CCF. Which of the following drugs are contraindicated?

a- Quinidine

b- Verapamil

c- Beta blocker
d- Diuretics

e- Digoxin

In which of the following conditions you be likely to see cannon a-waves?

a- Atrial fibrillation

b- Ventricular extra-systoles

c- Complete heart block

d- Incupid incompetence

e- Aortic stenosis

A patient with mitral stenosis due to rheumatic fever is congestive cardiac failure; the
following is/are true:

a- Surgical mitral valve plasty might improve the situation

b- ACE inhibitors may be indicated

c- Right heart failure is rare

d- Angiography should be done before operation

e- Ejection fraction 30. Patient needs treatment

Pulmonary hypertension and cor-pulmonate are frequency secondary to:

a- Emphysema

b- Bronchiectasis

c- Pneumothorax

d- Cadiomyopathy
e- Sarciodosis

R.dass, Oct 25, 2007

#32

R.dass

R.dass Guest

Ptosis may be found in each of the following conditions except:

a- Cluster headache (Horton’s syndrome)

b- Following cervical sympathectomy

c- Posterior artery communication aneurysm

d- Myasthenia gravis

e- Bell’s (facial nerve) palsy

The defect in visual fields most commonly associated with a pituitary tumour is:

a- Crossed homonymous henianopia

b- Central scotoma

c- Bitemporal hemianopia

d- Total blindness in one field

e- Peripheral concentric constriction and enlargement of the blind spot

Cardiac diseases known to be associated with sudden death include all of the following
except:

a- Coronary artery disease aippg.com

b- Prolonged QT syndrome
c- Hypertrophic obstructive cadiomyopathy

d- Mitral stenosis

e- Aortic stenosis

The commonest arrhythmia associated with thyrotoxicosis is:

a- Sinus tachycardia

b- Atrial Fibrillation

c- Atral flutter

d- Atrial ectopics

e- Ventricular ectopics

A 35 years old housewife presents with headache, extreme muscular weakness, polyuria
and polydypsia and two episodes of carpopedal spasm. On examination she has a blood
pressure of 190/115mmHg and proteinuria. Initial screening blood tests show random
glucose 7.2mmol/L, sodium 158mmol/L, potassium 30.mmol/L. The most likely diagnosis
is:

a- Pancreatic insufficiency

b- Diacates insipidus

c- Adrenocortical adenoma

d- Renal failure

e- Hypoparathyroidism

The diagnosis for the patient in Question 40 is likely to be confirmed by measuring:

a- Dexamethasona suppression test

b- Urinary vanilly mandic acid


c- Plasma renia

d- Plasma cortisc

e- Glucose tolerance

The most common cause of secondary hypertension is:

a- Drugs

b- Coarctation of the aorta

c- Primary aldosteronism

d- Renal abnormalities

e- Elevated catecholamines

The most common cause of normocytic anemia is:

a- acute hemorrhage

b- Chronic inflammation

c- Malignancy

d- Hemolysis

e- Liver disease

The most cause of chronic bacterial orchitis is:

a- Tuberculosis

b- Syphilis

c- Leptospirosis

d- Staphylococcus aureus

e- Pseudomonas sp
A 35 years old woman whose pre-pregnancy body mass index was 35.0 kg/m2, is found
to have glucosuria (3+) and a random capillary blood glucose level of 12.0mmol/L in the
second trimester of her pregnancy. An oral glucose tolerance test confirms diabetes
mellitus. Her first pregnancy results in intra-uterine fetal death at 37 weeks gestation. The
most likely treatment will be:

a- A diet in which total energy intake in restricted severely

b- Insulin

c- No long term follow up, since the risk of non insulin dependent diabetes is negligible

d- Appetite suppressants

e- Oral hypoglycemic agents

A 46 years old woman with a body mass index of 38 presents with a 2 weeks history of
sever headache, blurring of vision and nausea which followed a course of tetracyclines for
a lower respiratory infection. The significant abnormality on examinations is papilledema.
The probability diagnosis is:

a- Cerebral glioma

b- Temporal arteritis

c- Gravitational headache

d- Benign intracranial hypertension

e- Cerebral abocess

Diverticular disease of the colon is:

a- Associated with hypertrophy of circular smooth muscle

b- Most prominent in the right half of colon

c- Often complicated by carcinoma

d- Usually congenital in origin


e- Not associated with any of the above

In a cerebrovascular accident, hemorrhage and thrombosis may be clearly distinguished


on which of the following points?

a- Sequence of the clinical features

b- Degree of loss of consciousness

c- Description of onset

d- Presence of headache

e- None of the above

In which of the following is ascites usually present on clinical examination?

a- Left ventricular failure

b- Cirrhosis of the liver

c- Intra-abdominal Hodgkin’s disease

d- Nephrotic syndrome

e- Carcinoma of the uterus

The ‘opening snap’ of mitral stenosis:

a- Denotes valve mobility

b- Disappears if atrial fibrillation occurs

c- Is usually best head at the apex?

d- Replaces the third heart sound

e- Is really a closing snap?

R.dass, Oct 25, 2007


#33

R.dass

R.dass Guest

The notifiable communicable disease that is most commonly reported in Australia is:

a- Hepatitis A

b- Salmonella infections

c- Gonorrhoeae

d- Rubella

e- Syphilis

Which of the following organisms is highly resistant to amoxicillin?

a- Haemophilus influenzae

b- Streptococcus pyogenes

c- Escherichia coli

d- Klebsiella

e- Staphylococcus aureus

Which commonly arises in a solar keratoses?

a- Malignant melanoma

b- Squamous carcinoma

c- Basel cell carcinoma

d- Kerato-acanthoma aippg.com

e- Sarcoma
A 30 years old woman has a history of one week of sweats and fevers. Yesterday she
had right sided pleuritic chest pain. Questioning reveals a long history of recurrent mouth
ulcers episodic joint pains in he hands, photosensitive skin eruptions on the face and
Raynaud’s phenomenon.

A most likely diagnosis:

a- Systemic Lupus Erythematosis (SLE)

b- Atypical pneumonia

c- Dermatomyositis

d- Hepatitis C

e- Rheumatoid arthritis

Where there is immunodeficiency, bacterial infections characteristically involve of the


following except?

a- Skin

b- Respiratory system

c- Sinuses

d- Joints

e- Gastrointestinal system

A 65 yrs old woman presents with recent onset of severe right sided headaches
associated with soreness of the scalp. Three days prior to admission she developed
blurring of vision in the right eye. On examination she has a visual acuity of less than 6/60
in the right eye and early swelling of the right optic disk. Which of the following is the
most appropriate next step?

a- Bed rest and analgesics

b- Pilocarpine 2% eyes drops hourly


c- Prednisolone 60mg daily

d- Cerebral CT scan

e- Urgent craniotomy

All but one of the following are recognised features of hypertrophic

obstructive cadiomyopathy. The exception is:

a- Sudden death

b- A large pressure gradient between sub aortic and lower ventricular chambers in
ventricular systole

c- Sometimes autosomal dominant inheritance

d- Q waves in anterior ECG leads

e- Eosinophilla

A patient presents with worsening early morning headache and early papilledema and is
considered to have raised intracranial pressure. Which

Of the listed features, if presents, would also be characteristic of this diagnosis?

a- Nuchal rigidity

b- Tachycardia

c- Marked loss of visual acuity

d- Third cranial nerve palsy

e- Hemiation of the cerebellar tonsils through the tentorium serebelli

Lower than normal renin levels may occur in association with:

a- Accison’s disease
b- Pregnancy

c- Beta block ace

d- Diuretic therapy

e- Malignant hypertension

A 55 yrs old housewife complains of persistent widespread pains in the bones getting
worse over the course of 6 months. The differential diagnosis includes all except one of
the following. The one exception is:

a- Osteomalacia

b- Multiple myeloma

c- Osteoporosis

d- Hyperparathyroidism

e- Paget’s disease

A moderately obese man, aged 65 years, presents with a history of pain and mild
swelling of the left knee for the past four months. The only relevant past history is of
transient cerebral ischemic attacks being managed by aspirin, 300mg daily. Full blood
count and renal function are normal but the serum urate is 0.51mmol/L. He suffered from
a single episode of podagra four years previously. Which of the following statements is
correct?

a- The current attacks of arthritis is probably osteoarthritis

b- His 24 hour urinary uric excretion is probably elevated

c- Allupurinol should be used to reduce his serum uric acid level to normal

d- A reduction of aspirin dosage to 100mg daily will lead to a fall in the serum urate
level

e- If uric acid reduction were required, it would be better to prescribe sulphinpyrazone


that allupurinol to a patient already receiving aspirin
R.dass, Oct 25, 2007

#34

R.dass

R.dass Guest

Characteristically, type A viral hepatitis: aippg.com

a- Is a DNA virus infection often associated with seafood poisoning

b- Has an incubation period of three months

c- Carries a greater risk of fulminant hepatic failure in young compared with old patients

d- Often presents with headache plus right hypochoncrial pain and tenderness

e- Can progress to chronic active hepatitis if cholestasis is prolonged

Which of the listed drugs has the potential to induce life threatening cardiac
arrhythmias when taken in overdose?

a- thioricazine

b- Haloperidol

c- Lithium

d- Fluphenazine

e- Levodopa

Which of the following adverse effects is best associated with use of flucloxacillin?

a- Phototoxicity

b- Interstitial nephritis

c- Cholestatio hepatitis

d- Agranulocytosis
e- Acute hemolytic anemia

Hirsutism is recognized side-effect of treatment with all excepts one of the following
agents. Which is the exception?

a- Progestogens

b- Psoralens

c- Minoxidil

d- Trimethoprim

e- Diazoxide

During the first 4 hours after an overdose of paracetamol, a patient will develop:

a- Hypoglycemia

b- Hyperventilation

c- Increased serum ALT and AST

d- Pin-point pupils

e- None of the above

The ECG changes in left ventricular hypertrophy typically include all of the following but
one. The exception is:

a- ST elevation in lead

b- Tall R waves in leads V5 and V6

c- Deep S waves in leads V1 and V2

d- Left axis deviation

e- T waves inversion in lead a VL


A 55 years old man with a 30 pack/year smoking history presents with a history of
angina of effort. With respect to potential drug therapy, which one of the following
statements is covered?

a- verapamil and nifedipine would each reduce myocardial rate and contractility

b- Glyceryl trinitrate transdermal patches should be worn for 24 hours and then
replaced

c- The predominant mechanism of action of glyceryl trinitrate is peripheral arterial


vasodilatation and a reduction in cardiac after-load

d- Beta blockers are indicated as prophylactic therapy for exercise induced but not
vasspastic angina

e- Beta blockers and calcium channel blockers could each precipitate asthma

Which of the listed anti arrhythmic drugs is the drug of choice in most cases of
Supraventricular tachycardia (including nodal tachycardia)?

a- Digoxin

b- Quinidine

c- Flecanide

d- Verapamil

e- Propranolol

In a patient with severe asthma, which of the features listed would be the most
ominous?

a- Arterial PO2 50mmHg

b- Arterial PCO2 50mmHg

c- FEV10.8 litres

d- Very loud wheezes

e- Respiratory rate of 20 per minutes


An elderly man has been hospitalized for one week because of severe and worsening
Parkinson’s disease. He is found in bed tachypnoeic, tachycardic and responsive only
to painful stimuli. Pulmonary examination reveals diffuse wheezes and crackles in the
right lung field posteriorly and absent breath sounds in the left lung field. Which of the
following conditions is most likely to be responsible for this patient’s deterioration?

a- Massive pulmonary embolism

b- Pulmonary oedema after acute myocardial infarction

c- Aspiration pneumonia with unilateral bronchial obstruction

d- Spontaneous pneumathorax

e- Tracheosophageal fistula

R.dass, Oct 25, 2007

#35

R.dass

R.dass Guest

All except one of the following statements are characteristic of ventricular septal defect
of the heart. The o exception is:

a- It produces a left to RIGHT SUNT

b- It can be diagnosed by cardiac ultrasound

c- It is associated with a continuous murmur throughout systole and diastole

d- It is one of commonest congenital hear lesions

e- It is associated with a step up in oxygen saturation in the right ventricle

All of the following except one typically produce keratin scales. aippg.com
The one exception is:

a- Squamous cell carcinoma

b- Solar keratoses

c- Psoriasis

d- Pityriasis versicolor

e- Basal cell carcinoma

A 75 yrs old woman, previously in good health, presents with a history of painless
jaundice of 2 weeks duration. There is a distended gall bladder revealed by ultrasound
examination. What is the most likely cause?

a- Chronic Pancreatitis

b- Alcoholic cirrhosis

c- Gall stones

d- Carcinoma of the pancreas

e- Acute Cholecystitis

In diabetic neuropathy you can find each of the following except:

a- Bradycardia

b- Urine retention

c- Impotence

d- Diarrhoea at night

e- Foot ulcer

In suspected acute MI what is the criteria for thrombolytic therapy?

a- T inversion
b- ST depression/elevation???

c- New LBBB

d- New RBBB

e- Q waves

A 55 yrs old woman was found unconscious in her house. She has a dilated pupil. What
is the cause?

a- overdose

b- Brain infarct

c- Subdural hematoma

d- Subarachnoid hemorrhage

e- Coma

Antidote for heparin is:

a- Protamine sulphate

b- Vitamin K

c- Fresh frozen plasma

In heparin induced thrombocytopenia which of the following is typically seen?

a- Petechial purpura on the skin

b- Thrombosis

c- Joint bleeding

d- Hematuria

e- Ecchymosis
All of the following can cause renal papillary necrosis except:

a- Liver disease

b- Medullary sponge disease

c- Sickle cell disease

d- Diabetes mellitus

e- Analgesic nephropathy

Opening snap indicates:

a- Mitral Valve mobility

b- Atrial fibrillation cause disappearance of the opening snap

c- Replaces S3

d- Best heard at 2nd right intercostals space

e- Remains unaltered despite progression of disease

In a man from overseas, his chest x-ray reveals silent tuberculosis. PPD skin test is 10 in.
treatment of choice is:

a- Sputum culture

b- Observation

c- Triple therapy

d- Isoniazid

e- Isoniazide and ethambutal

R.dass, Oct 25, 2007

#36
R.dass

R.dass Guest

In infectious mononucleosis:

a- Incubation period is 4-5 days

b- Is seen more in patients with AIDS

c- Incubation period is 14-21 days

In asthma, which is the most important factor in diagnosing its severity?

a- Decreased PEY

b- Increased diumal variability

c- Decreased diumal variability aippg.com

A man returning from an overseas trip has Diarrhoea and vomiting for 2 weeks,
salmonella was isolated. What is the next step in management?

a- Ampicillin

b- Repeat stool culture

c- Trimethoprim-sulphamethoxazole

d- Only observation

e- Metronidazole

All of the following are side effects of Prednisolone except:

a- Hirsutism

b- Lymphopenia
c- Leucocytosis

d- Osteoporosis

e- Weight gain

Polycythemia rubra vera:

a- Contraction of plasma

b- Expansion of plasma volume

Which of the following is correct regarding the overdose of TCA?

a- In a mixed overdose with TCA’s and benzodiazepines flumazenil is contraindicated

All of the following are features of Parkinson’s disease except:

a- Postural reflexes lost

b- Can begin unilaterally

c- Tremor gets worse during sleep

d- Cogwheel rigidity

e- Able to stop while walking

A patient presents with impaired sensation on the medical aspect of the hand, 4th and
5th fingers of the hand are flexed. Where is the lesion?

a- Ulnar nerve at the elbow

b- Ulnar nerve at the wrist

c- Redial nerve damage at the wrist

d- Medial nerve damage at the wrist

e- Redial nerve damage at the elbow


A patient presents with moderate hemiparesis of legs and arms, no dysphasia, face
appears normal. Where is the lesion most likely to be?

a- Cerebral cortex

b- Internal capsule

All of the following are features of thyrotoxicosis except:

a- Wasting of small muscles of the hand

b- Fine tremor of hands

c- Irritability

d- Weight loss

e- Wide pulse pressure

Which of the following is the most common cause of ketoacidosis?

a- Cessation of insulin in IDDM

b- Undiagnosed IDDM

c- Undiagnosed NIDDM

d- Adrenal failure

e- Infection

In paracetamol poisoning the involvement of which organ is most important?

a- Renal injury

b- Hepatic injury

c- Brain injury
d- Stomach injury

R.dass, Oct 25, 2007

#37

R.dass

R.dass Guest

Which of the following is the most common pathology involved in endocarditis?

a- Rheumatic fever

b- Staphylococcus

aippg.com

A 60yrs old male patient has difficulty climbing upstairs. O/E he had wasting of right
biceps, diminished biceps jerk, lower limb spasticity, lower limbs reflexes exaggerated and
Babinski’s sign present, hip flexion weak. What is the most probable diagnosis?

a- Cervical disc lesion

b- Lumbar canal stenosis

c- Motor neuron disease

d- Multiple sclerosis

e- WPW syndrome

A man on anti-hypertension therapy goes for surgery. Discontinuation of which of the


following drugs causes serve rebound hypertension during operation?

a- Enalapril

b- Clonidine

c- Methyldopa
d- Hydralazine

e- Verapamil

After splenectomy for spherocytosis, each of the following is correct except:

a- Erythrocyte osmotic fragility will not change

b- Erythrocyte life span will be normal

c- Anemia will be found

Normocytic, normochromic anemia can be found in which of the following diseases?

a- Sickle cell anemia

b- Vitamin B12 deficiency

c- Renal failure

d- Liver failure

e- Hemolysis

A woman complaining of episodes of intense pain over the lower jaw and zygomatic
bone lasting for about 2-5 min which has been going for 2 weeks.

There are no trigger zones and on examination there is sensory loss on the same side.
Which of the following is the most likely diagnosis?

a- Multiple sclerosis

b- Trigeminal neuralgia

c- Migraine

d- Facial palsy

e- Herpetic neuralgia
What is the best way to diagnose giardiasis?

a- Duodenal aspirate showing trophozoites

b- Fecal microscopy showing ova

c- Fecal culture

d- Serum immunology

e- Villous atrophy

A man with multiple episodes of sore throat developed periorbital edema, BP of


160/120, oliguria, which of the following is correct?

a- Giving water corrects oliguria

b- Hematuria is a prognostic indicator

Which of the following is correctly matched?

a- Decreased transferrin: in iron deficiency anemia

b- Increased transferrin: in hemosiderosis

c- Normal transferrin: in Hemachromatosis

Which of the following is most implicated in causing pseudo membranous colitis?

a- Metronidazole

b- Ampicillin

c- Vancomycin

d- Sulphonamides

e- Tetracyclines
R.dass, Oct 25, 2007

#38

R.dass

R.dass Guest

A result of 120mmol/l sodium is consistent with:

a- Decreased sodium osmolality

b- Decreased total extracellular body Na

c- Water intoxication

d- Inappropriate ADH secretion

e- Diabetes insipidus

Which of the following is correct regarding myasthenia gravis?

a- Some patients do not have anti-cholinergic antibodies

b- Anti cholinergic drugs are used for treatment

c- Thymectomy is only indicated for thymoma

d- Reduced tendon reflexes are characteristic

e- Repeated stimulation of peripheral nerve always shows pathological fatigue

Eradication of helicobacter pylori is important because: aippg.com

a- It influences relapse

b- It reduces complication (incidence of gastric lymphoma)

c- It heals quickly

d- It decreases local gastritis


e- It decreases the cimetidine dosage

Which of the following anti-hypertensive drug causes reflex tacycardia?

a- Methyldopa

b- Hydralazine

c- Prazosin

d- Verapamil

e- Diltiazem

Which of the following is true regarding NIDDM?

a- The genetic factors are more important that in IDDM

b- Abdominal fat is a risk factor

c- Oral hypoglycemic agents can cause insulin resistance

d- Patients with NIDDM will never?? Insulin

e- Patients with gestational diabetes don’t go into NIDDM

Regarding significant aortic stenosis, which of the following is true?

a- If the gradient is>50mmHg, patients needs surgery

b- Poor LV function is a contraindication for surgery

c- Can cause death

d- Chest pain indicates angina pectoris

e- Palpable thrill

A child is presents with long history of unexplained productive cough and wheeze. He is
failing to thrive (signs of malobsorbtion). What is your initial investigation?
a- Stool exam

b- Chest x-ray

c- Sweat test

d- Blood exam

What is true about febrile convulsion?

a- It occurs between 6-8 years of age

b- Prognosis is not good

c- Risk of developing epilepsy is 10%

d- Lasts about 15 min.

Which of following conditions usually resolves spontaneously?

a- Congenital heart block

b- Patient ductus in 15 yrs boy

c- Atrial septal defect

d- Small ventricular septal defect

In a child with Kernicterus which abnormality is found in the blood?

a- High Unconjugated bilirubin concentration

A child is who was well before developed malaise and now he has fever 40°c. 3 days
ago he falls from tree. Mother noticed that he limped. His thigh is swollen there is an area
of pain and tenderness. What is the likely cause?

a- Fracture
b- Thrombophlebitis

c- Ca of bone

d- Osteomyelitis

A child is presents with sudden onset of high fever. He is unable to drink or swallow. He
is stridor. Sever epiglottitis has been diagnosed. What is your management?

a- Antibiotics

b- Intubations may be required

c- Throat should be examined

A 76 yrs old man is investigated for a complaint of weakness. The hematological report
shows:

Hb 5.6g/100ml, MCV 72u , MCHC 23%, WBC 12*10 , Platelets 382*10 ,

ESR 46mm/1 hour.

The film shows a marked hypochromia, a moderate, marked polkilocytosis, slight


Polychromasia, mature neutrophils, reticulocytes 3%

The most likely diagnosis is:

a- Megaloblastic anemia

b- Iron deficiency anemia caused by chronic blood loss

c- Chronic renal failure

d- Anemia due to folate deficiency

e- Thalassaemia minor
The mortality from the initial blood of a ruptured intracranial aneurysm is about:

a- 3-5%

b- 5-10%

c- 20-35%

d- 70-90%

e- 95%

R.dass, Oct 25, 2007

#39

R.dass

R.dass Guest

A 27 yrs old woman in her tenth week of pregnancy was found on a mid-steam urine
culture to have Ecoil 10 colonles/ml. She was asymptomatic. Which of the following
should be considered next?

a- Intravenous pyelogram

b- Repeat of urine culture and antibiotic therapy if the same organisms are present

c- No further investigation

d- Investigation after delivery

e- Immediate antibiotic therapy aippg.com

Creatinine clearance:

a- Is an accurate measure of renal tubular function

b- Requires measurement of only the urine creatinine concentration

c- Cannot be measured in the nephrotic syndrome


d- Approximatles to the glomerular filtration rate

e- Measures the total body pool of creatinine

A 27 yrs old woman presents with increased frequency of nucturition including nocturia
over the recent three weeks. Over this time she has been thirsty and lethargic. Which one
of the metabolic derangements is least likely to present in this way?

a- Hypokalemia

b- Diabetes mellitus

c- Antidluretic hormone deficiency

d- Hypercalcemia

e- Dilantin toxicity

Which of the following rules out radical mastectomy for breast cancer?

a- Satellite nodules over breast skin

b- Parastemal tumor nodules

c- Proven supraclavicular metastasis

d- Oedema of the arm

e- All of the above

Polymyalgia rheumatics is usually

a- Seen in teenagers

b- Associated with a high erythrocyte sedimentation rate

c- Treated initially with penlcillamine

d- Associated with night blindness

e- Is associated with a high white cell count


The luteal phase of the following menstrual cycle is associated with:

a- High luteinizing hormone (LH) levels

b- High progresterone levels

c- High prolactin levels

d- Low basal body temperature

e- Proliferative changes in the endometrium

A 48 yrs old woman has lost all desire for sexual intercourse since hysterectomy 9
months previously. The sexual problem is most likely to reflect:

a- Removal of the cervix

b- The onset of menopause

c- The expectation of sexual alteration by operation

d- A loss of desire due to ageing

e- Loss of any further chance of pregnancy

due to: (Obstruct Bowel)

a- Meckel’s diverticulum

b- Benign tumour

c- Carcinoma

A 38 yrs old woman came with repeated H/O Diarrhoea, since 3-4 months. She had lost
4 kg. in weight and used to get lower abdominal pains at irregular intervals. O/E a ill
defined mass was found in the Rt sliac fosses. What is the most probable diagnosis?

a- Ulcerative colitis
b- Chron’s disease

c- Cacinoma of sigmoid colon

d- Appendicular absuss

e- Effusion

A 38 yrs old man came to you saying that his father and brother both had died from.
(Carcinoma of the bowel) He wanted the same disease to the excluded in him which
investigation would you advice:

a- Occult blood in stools

b- Sigmoidoscopy

c- Sigmoidoscopy +3 stool specirmes for occult bid

d- Colonoscopy

Which are the whom you will part for M.T. emergency surgery in intestinal obstruction?
(ABD – Emergency)

a- strangulation

b- Severely dehydrated RT

c- Pulse>100

d- Inguinal hernia in the newborn

Which of the following conditions will you be able to flicit the history of passing bid per
sectum after the act of bifurcation? (bleeding/Rectum)

a- Carcinoma colon

b- Fissure in ano

c- Internal hemorrhoids

d- External hemorrhoids
In epigastric hernia, (of linea alba) which is true:

a- Situated below the umbilicus

b- Contains intestine

c- Is vary large

d- Contains extra-peritoneal fat

e- Required treatment

R.dass, Oct 25, 2007

#40

R.dass

R.dass Guest

Patient with chronic WPW syndrome presents with AF.

WOF are management:

a- Digoxin

b- Verapamil

c- Cardioversion

d- Sotolol

e- Disepynamide

Metabolism of Digoxin is decreased by:

a- Quinidine

b- Warfarin
c- Verapamil

d- Amiodarone

e- Ametidine

Digoxin intoxication presents with W.O.F. except:

a- Hypokalemia

b- Patient has yellowish vision

c- A.F. can occur

Patient nerve is affected. W.O.F. may be the symptoms:

a- Absence of knee reflex

b- Ankle jerk is affected

c- Sensory loss of the medium aspect below knee

d- Difficulty in dorsiflaxion

e- Sensory loss in the sole aippg.com

Peracetamol poisoning- In the management. W.O.F. is appropriate to prevent:

a- IV acetyl cysteine

b- Oral acetyl cysteine

c- Gestric lavage with charcoal

d- Ipecac syrup

e- Peritoneal dialysis

Patient with severe hypertrophic cardio-myopathy. W.O.F. drugs are suitable for this
patient?
a- Digoxin

b- Amiodarone

c- Verapamil

d- Furasemide

Patient had acute Pancreatitis 4 days later he presents with pseudo cyst. What is your
management?

a- Pseudocyst has connection with pancreatic ducts

b- Cyst has epithelial wall

c- Cyst resolve spontaneously leave it

Blood picture of anemia in chronic disease:

a- Serum Fe increases

b- Macrocytic anemia

c- Microcytic anemia

d- Normocytic anemia

Guillian-barre syndrome after viral infection:

a- Increasing protein with normal cell in CSF

b- Antibodies against myelin shed

c- Nerve conduction can be used

d- Low glucose in CSF

Osteosclerosis:

a- Normal tympanic membrane


b- Bulging blunt tympanic membrane

Congenital spherocytosis except:

a- Transient Leukocytosis

b- Decreasing erythrocyte life span

c- Thrombocytosis

d- Splenectomy

TLA unilateral eyesight lost

a- Atheroma of internal carotid

b- Vertebral basilar

c- Posterior communicating cerebral

Investigation of hypoadrenalism:

a- Low renin

b- Genitalia pigmentation

c- Autoimmune in

d- ACTH stimulation test

R.dass, Oct 25, 2007

#41

R.dass

R.dass Guest
In diabetic neuropathy you can find each of the following except?

a- Bradycardia

b- Urine retention

c- Impotence

d- Diarrhoea at night

e- Foot ulcer

A young man presents with pleuritic chest pain and cough. On examination there is
dullness on percussion and bronchial breathing over the right lower zone, posteriorly.
Which of the following is the most likely diagnosis?

a- Right lower lobe consolidation

b- Right lower lobe collapse

c- Asthma

d- TB cavity

A 37 yrs old woman has unilateral exophthalmos. Which of the following is the most
likely cause?

a- Glaucoma aippg.com

b- Nasopharyngeal tumour invading the orbit

c- A neoplastic lesion in the lamina cribrosa

d- Thyrotoxicosis

e- Infection of the frontal sinus

f- Maxillary arterial truma


A middle aged man is brought to your surgery with a history of being bitten by a snake.
On examination there are scratch marks on the lower leg and on other symptoms. What is
you management?

a- Reassure and send him home

b- Excise wound

c- Administer antivenin immediately

d- Apply Tourniquet

e- Observation and admission, given antivenom if necessary

In a patient presenting with lacunar infarction, which of the following are true?

a- Sensory loss only

b- Dysphasia

c- Due to an aneurysm

d- Haemorrhage from the middle cerebral artery

e- ? Thrombo embolism may cause

Helicobacteopylori causes all of the following excepts?

a- Duodenal ulcer

b- Gastritis

c- Gastro-oesophageal reflux

d- Gastric carcinoma

e- Gastric lymphoma

A 28 years old male presents with distal weakness and atrophy of the small muscles of
both hands (interossei, lumbricals, thenar and hypothenar). What is the most likely
diagnosis?
a- Multiple sclerosis

b- Bilateral median nerve palsy

c- Syringomyelia

d- Bilateral ulnar nerve palsy

e- Brainstem infarction

The single most reliable test for hemochromatosis is?

a- RE cell mass

b- Serum iron

c- Serum Ferritin

d- Serum tranferrin

e- Transferrin saturation

A female age 23 yrs, presents with proteinuria. There is a previous history of enuresis till
13 years of age. Her sister also had enuresis as a child. The history is suggestive of which
of the following?

a- Chromic glomerulonephritis

b- Reflux nephropathy

c- Congenital nephritis

d- Polycystic kidney disease

e- Analgesic nephropathy

In which of the following, is administration of immunologic as prophylaxis not useful?

a- Hepatitis A

b- Hepatitis B
c- Rubella

d- Mumps

e- Varicella

A 68 years old female presents with a 2 week history of unilateral headache and an ESR
8omm/hr. The most appropriate treatment is:

a- NSAID’s

b- Aspirin

c- Carbamazepine

d- Prednisolone

e- Ergotamine

Which of the following is least likely to facial nerve palsy?

a- Skull fracture

b- Mastoiditis

c- Chronic parotitis

d- Parotid tumour

e- Acoustic neuroma

R.dass, Oct 25, 2007

#42

R.dass

R.dass Guest

All of the following drugs are nephrotoxic except:


a- Cyclosporine

b- Gentomicin

c- Digoxin

d- Phenacitine

e- Gold

Common clinical findings in gross sever ascites. Except

a- Shifting dullness

b- Bilateral dullness in the base of the lungs

c- Palpable hepatomegaly

d- Eversion of the umbilicus

In a patient with Emphysema all of the following clinical findings are present, except;

a- Increased TLC

b- Clubbing

c- Hyperinflation

A 50 yrs old woman is suffering from spastic paraplegia with pain at the T8 level. What
is the most likely cause?

a- Syringomyelia

b- Caudal equine claudication

c- Spinal cord compression

d- Multiple sclerosis

e- Guillain-barre syndrome
Patient had two attacks of left side transient blindness during last month. He has
weakness of the right hand. When he speaks, he uses wrong words. What is the most
likely cause?

a- Posterior cerebral artery thrombosis

b- Vertebral artery thrombosis

c- Right carotid thrombosis

d- Left middle cerebral artery thrombosis

e- Right middle cerebral artery thrombosis

An old lady presented with the extended bladder up to Xiphoid process. What is the
most likely diagnosis?

a- Acute urinary retention

b- Chronic urinary retention

c- Neuropathic bladder

All of the following are common clinical features of duodenal ulcer except

a- Loss of appetite

b- Hunger pain

c- Periodicity

d- Nocturnal pain

e- Pain immediately before meal

Hypoglycaemia can be found in all of the following conditions except

a- hyperprolactineamia

b- Anorexia nervosa
c- Primary hyper aldostronizm

d- Addisons disease

A lady is on phenytoin. Her blood level of phenytoin is less than half of the therapeutic
dose. Which of the following is correct?

a- Double the dose

b- Phenytoin interferes with warferine action

Mycoplasm pneumonia is characterized by all of the following features except

a- ESR may be very high

b- WBC normal

c- Lower lob pneumonia

d- Cough is unusual

e- Tetracycline is the treatment

What is the common clinical feature of the acute pulmonary embolism?

a- tachypnia

b- Plural pain

c- Haemoptesis

d- Cyanosis

e- Hypertension

A 7 month’s pregnant lady wide second hart sound, which does not change with
respiration. Chest x-ray shows pulmonary plethora. What is the most likely diagnosis?

a- ASD
b- VSD

c- Mitral regurgitation

A patient is suffering from infective endocarditis, develops a new diastolic murmur in


the lower sternal border. What is the most likely cause?

a- Rapture of chordae tendenia

b- Aortic stenosis

c- Pulmonary regurgitation

d- Tricuspid stenosis

e- Mitral stenosis

R.dass, Oct 25, 2007

#43

R.dass

R.dass Guest

A patient who suffers from chronic arthritis looks pale. He takes NSAID, Hb low, Fe low,
Ferities high, MCV normal. What is the most likely cause?

a- Iron deficiency anemia

b- Anemia of chronic disease

Patient with more than 12 cm splenomegaly, blood film shows tear drops polkilocytosis.
What is the most likely cause?

a- Myelofibrosis

b- CLL
c- CML

d- Multiple myeloma

A 27 yrs old male diagnosed with anaemia. Blood exam shows microcytic hypochromic
anaemia. What is your initial investigation?

a- Stool exam

b- Colonoscopy

c- Segmoidoscopy

d- HB electrophoreses

A patient presented with anaemia. On investigation it was found that Hb-low, A2-high.
What is the most likely cause?

a- Thalassaemia minor

b- Hereditary spherocytosis

What is the common cause of death in a patient with Thalassaemia?

a- Heart failure

b- Renal failure

c- Infection and septicaemia

d- Anaemia

A young lady is suffering from minorragia. She has bruises on her limbs. Her brother and
family have bleeding disorder:

a- Haemophilia A

b- Von Willebrand’s disease


c- Congenital Hypofebrinogenemia

d- Haemophilia B

A woman who have history of Von Willebrand’s disease, has cholecystectomy. What
is your initial management?

a- Cryoprecipitate

b- Vitamin K

c- Blood transfusion

d- Factor VIII concentrates

Which of the following is least likely to cause transmission of HIV and Hip B virus?

a- Albumin

b- Factor VIII

c- Fresh frozen plasma

d- Platelet concentrates

e- Cryoprecipitate

A young man suffers from fever, mild generalized lymphadenopathy, macula-popular


rash on the upper part of the body and palm and soles. He also had erosion on pharynx.
Gray tonsilar exudate, perianal lesion. What is your diagnosis?

a- Yaws

b- Secondary syphilis

c- Infectious mononucleosis

Granuloma annularis:
a- Pruritis

b- Ulceration

c- Griseofulvine should be given

d- Premalignant

e- Raised irregular border

R.dass, Oct 25, 2007

#44

R.dass

R.dass Guest

Australian woman live longer. What is the cause?

a- genetic determination

b- Visit doctor frequently

Which of the following findings suggests that the patient has gout?

a- High serum uric acid

b- Birefringence crystals in Synovial fluid

c- High level of uric acid in urine

Rheumatic fever’s clinical feature:

a- Involves only large joints

b- Usually affects small joints

c- Does not cause permanent deformity


Hypertension is the most preventable factor in which if the following conditions:

a- Stroke

b- Coronary artery disease

c- Renal disease

A patient came to the emergency department claiming that he has got snakebite.
Clinical exams revealed that there was no sign of bite, but there was an area of erosion on
the left foot. What is your management?

a- Administration of anti-venom

b- Observation

c- Apply tourniquet

d- Sent him home

A patient was diagnosed with trigeminal neuralgia. Which of the following clinical
finding is true?

a- Patient has sensory loss

b-

c-

d- Jaw is deviated to the affected side

e- Jaw is deviated to the unaffected side, when asked to open it

Regarding emphysema, all of the following clinical findings are true except:

a- Clubbing

b- CO
c- Fixed expiratory capacity

d- Decreased ventilator capacity

An alcoholic patient is complaining from oily stool and weight loss. What is your next?

a- Stoll fat estimation

b- Pancreatic function test

c- ERCP/CT

d- Sweet teat

e- X-ray of abdomen

Hypertensive patient on ACEI and diuretics. Which of the following changes will occur in
the blood?

a- Decreased K

b- Decreased Na

c- Reduced bicarbonate

d- Hyperlipidaemia

e- Reduced GFR

Treatment of hyperphosphataemia in chronic renal failure

a- Aluminium hydroxide

b- Calcium carbonate

c- CaCO3 with phosphate binder

d- Mg hydroxide

R.dass, Oct 25, 2007


#45

R.dass

R.dass Guest

Regarding treatment of asthma in adults. All are true except:

a- Hydrocortisone tablet before each attack

b- Aminophylline injection

c-

d- Patient can decide to change the treatment

e- Steroid inhaler

Which type of bon loss is common in chronic renal failure?

a- Ostoid fibrosis with osteomalisia

b- Osteoporosis

46. A woman presented with bald patches on the scalp. The area is shiny and oily.

Which of the following investigation would you implement to confirm the diagnosis?

a-

b-

c- Tell her that it is not causing total hair loss

d- Potassium hydroxide (KOH) testing of the sample aippg.com

Cause of cellulites:

a- Group B streptococcus
b- Group A streptococcus

c- Staphylococcus

A patient came with red eye. No proptosis or chemosis, upper left eye lid oedematous.

What is your management?

a- Urgent CT of the eye

b- I/V flucloxacillin

c- Local chloramphenicol

Thrombolytic therapy is indicated in which of the following cases:

a- Q= wave

b- ST depression

c- New RBBB

d- LBBB

e- New LBBB

Pt. With chest pain, which is true:

a- Thrombolytic therapy is not indicated if ECG is normal

b- Heparin I/V

c- Immediate ETT

Pt. was antihypertensives, Stopping of which of the drugs may produce intraoperative
hypertensive crisis:

a- Clonidine

b- Methyldopa
c- Enalepril

d- Propranolol

Regarding infective endocarditis which is true:

a- Vasculitis can be associated

b- Staph. Aureas is the most common cause

c- More common in the atrial wall

d- More common in the ventricular wall

Infectious mononucleosis:

a- Resembles acute HIV infection

b- Liver function is usually impaired

c- Incubation period is -5 days

d- Diagnosed by antibody

Hep- CV infection in Australia most commonly occurs in-

a- I/V drug abuse

b- Homosexuals

A 60 yrs old man alcoholic for 30 yrs. Now lethargic, weak, platelet count low, ESR high,
which of the following is true?

a- Multiple myeloma

b- Aleukaemic leukaemia

c- Primary thrombocytopenia
R.dass, Oct 25, 2007

#46

R.dass

R.dass Guest

Regarding Parkinsons disease, which are true except:

a- Involuntary Stopping while walking

b- Tremor during sleep

c- Postural hypotonia

Pt. with hemiplegia right side, with sensory loss in face and facial palsy atrial WOF is
true:

a- Hypertensive hemorrhage in the internal capsule

b- Middle cerebral artery hemorrhage

c- Left middle cerebral artery

d- ICA occlusion

e- TCA occlusion

Female Pt. 32 yrs old with H/O server sudden attacks of pain, well in-between, one
sided sensory loss in the face. The possible diagnosis is:

a- Multiple sclerosis

b- Trigeminal neuralgia

c- Post herpetic neuralgia


Pt. with wasting of biceps, lost biceps reflex, increased lower limb reflexes. The most
likely cause is:

a- Cervical cord compression

b- Motor neurone disease

c- Lumbar canal stenosis

Pt. with ptosis, dilated pupil, loss of corneal reflex and all movement of eye. The most
possible lesion is:

a- Internal carotid artery aneurism in the cavernous sinus

b- Brain stem inearction

Sensory loss of the medial aspect of the hand, inability to flexion and abduction of the
little and ring finger. The lesion is:

a- Ulner N. lesion in the wrist

b- U.N. lesion in the elbow

c- Median N. lesion at the wrist

Compartment syndrome in the leg. Which is true?

a- Weakness in the dorsiflaxion of the toe

b- Weakness in the plantar flexion

c- Weakness in eversion

d- Weakness in inversion

e- Weakness in flexion of the knee


You were called to see an old lady who was found unconscious in her bedroom, pupil
dilated, hypertensive, QT interval prolonged, ventricular tachycardia and atrial fibrillation.
The possible cause is:

a- Narcotic overdose

b- Subarachnoid haemorrhage

c- Dissecting aneurism

Thrombolytic therapy is indicated in which of the following conditions

a- Q wave in ECG

b- Development of recent RBBB

c- Development of LBBB

d- ST depression

WOF is true for polycythemia rubravera

a- Erythropoietin is decreased

b- It is difficult to diagnose cyanosis

c- Plasma volume is contracted

d- ESR is raised

e- Splenomegaly aippg.com

Regarding papillary necrosis of the kidney, all of the following are true except:

a- Alcohol

b- Medullary sponge kidney

c- Analgesic nephropathy

d- Diabetic nephropathy
e- Sickle cell nephropathy

R.dass, Oct 25, 2007

#47

R.dass

R.dass Guest

A pt. with hyperkalemia and increased S. creatinine WOF is the best method to diagnose
whether his renal failure is due to acute or chronic renal failure:

a- Creatinine clearance test

b- Serum urea level

c- Ultrasonography

d- IVP

Pt. developed puffiness in the face following upper respiratory tract infection. His BP is
180/120mmHg. WOF things can happen: aippg.com

a- If haematuria is present it is dangerous

b- There is immediate danger of development of LVF

Blood Na level is 120 meq/L. What does it mean?

a- Extracellular fluid osmolarity is decreased

b- Total body Na is decreased

c- SIADH

d- Decreased ADH secretion

e- Renal exchange of Na is decreased


Pt. was hypertensive, developed heart failure and has gout and was having Allopurinol
ACE inhibitor, Hydrochlorothiazide and indomethacin. What is the cause?

a- Deplesion of Na in the extracellular fluid

b- Indomethacin toxicity

c- ACE inhibitor toxicity

Hypercalcaemia may occur in all the causes except:

a- Pancreatitis

b- Hyperparathyroidism

c- Renal tubular acidosis

Diabetic autonomic neuropathy- all are true except

a- Bradycardia

b- Nocturnal diarrhoea

c- Urinary retention

d- Impotency

e- Pupil size

Regarding Diabetes mellitus- NIDDM is more related to the following than IDDM

a- Abdominal fat is a risk factor

b- IDDM is more genetically related than NIDDM

c- Oral hypoglycaemic agents reduce insulin resistance

Diabetic ketoacidosis most commonly occurs in


a- Undiagnosed IDDM

b- Known IDDM pt. forgot to take insulin

c- Diabetic pt. with infection

d- Known IDDM c infection

Regarding positive balance diet, WOF is true

a- Olive oil increases LDL

b- Olive oil reduces LDL

c- Taking fish 300gm per day does not decrease LDL

d- Alcohol does not change triglyceride level

An 18 yrs old girl with history of jaundice came with palmer fever and palmer erythema.
Which of the following is true for liver biopsy findings?

a- Periportal lymphocytes and bridging necrosis

b- Fatty change in the liver

A picture of a 23 yrs old girl with infected cystic acne with hirsuitism. Which of the
following is the most appropriate initial treatment?

a- Antiandrogen

b- Isotretinoin for 20wks

c- Isotretinoin with tetracycline

d- Broad spectrum antibiotic

R.dass, Oct 25, 2007

#48
R.dass

R.dass Guest

In a pt. with aortic cardiac catheterization shows pressure gradient of 55mmHg. Which
of the following will help you to take decision for operation?

a- Decreased Lt. ventricular ejection fraction

b- Increased LV end diastolic volume

Mitral Stenosis:

a- Loud SI

b- M.D. M

c- Tapping apex beat

d- Opening snap

e- Graham’s Steel murmur aippg.com

A pregnant lady presents with mild dyspnoea. O/E splitting of the 2nd Ht. Sound and no
change with respiration. What is the possible cause?

a- ASD

b- VSD

Regarding keratoacanthoma which is true

a- rapidly growing and spontaneous resolution

b- 2-3 week to grow

c- 2-3 months to grow


Regarding stent in coronary angioplasty which is true

a- Reduce complication and increase stenosis

b- Reduce complication and reduce stenosis

c- Reduce immediate stenosis least no change in complication

A lady came with dyspnoea. Blood gas analysis shows- HCO3-23 Pa CO2 29, Pa O2 60,
PH 7.4. Which of the following is true for her condition?

a- Respiratory alkalosis with reduced gas exchange

b- Respiratory alkalosis with normal gas exchange

A 65 yrs old man with profuse bleeding per rectum. The cause is-

a- Diverticulosis

b- Ca. rectum

c- Ischemic colitis

A pt. came from Indonesia. Has diarrhoea treated with Metronidazol for 3 days was not
responding. Stool culture 4 days before shows salmonella typhae. Regarding further
management WOF is true-

a- Amoxycillin

b- No Rx repeat culture after 10 days

c- Add Trimethoprim + Sulfonamide

d- Give Metronidazol

Most common cause of pseudomembranous colitis is-

a- Ampicillin
b- Vancomycin

c- Metronidazol

A man suffering from diarrhoea for 3 months. WOF is not true

a- Camylobactor jejuni

b- Giardia

Regarding Giardia infection, diagnosis is confirmed by which of the following

a- Tropozoide finding from duodenum aspiration

b- Cyst in the stool

c- Histopathology of duodenum shows Giardiasis

A young male pt. came with fever and cough. On x-ray it was diagnosed as an inactive
T.B. M.T. was possible. What is your next step of your management?

a- Triple drug Rx

b- Consecutive 6 sputum culture and wait for the result

c- INH prophylaxis for 12 months

d- Repeat X-ray after 3 months

Anaerobe non-spore bearing bacteria could be isolated from which of the following

a- Brain abscess

b- Hip prosthesis

Pt. is in shock. If you ask for blood bank which of the following would be most
appropriate blood group to be transfused?
a- O Negative

b- Same group uncrossed blood

All of the following are characteristic features of Mycoplasma Pneumonae except

a- Pleuritic chest pain

b- Severe cough

c- Fever

d- Lober pneumonia

R.dass, Oct 25, 2007

#49

R.dass

R.dass Guest

Regarding Lt. Renal artery stenosis all of the following are true except

a- Rx with ACE inhibitor will deteriorate the function of the same kidney

b- Hyperkalemia

(Test for Giardia

1. Stool examination - 3 separate Specimen - Cyst or Tropozoid

2. Duodenal or Jejunal fluid piration for Tropozoid

3. Biopsy for Giardia on the surface of epithelium

Rifampicin should not be given to any it vasculitis occur -


Complication of PTCA (Per cutaneous core angipled)

Occlusion of the vessel by thrombosis -

Loose flap of intima

Restenosis is due to combination of elastic recoil + smooth muscle proliferation which


tends to occur in months

- Recurrent angina - 6 months 32%)

Patient with tonsilar membrane (or sore throat) generalized lymphadenopathy, liver
and spleen enlarged, (Dg: infective Mononucleosis) how you will confirm Dg:

a- Atypical lymphocytes

b- AB+ CHV?

Hutchison melanotic freckle is characterized with all of the following except:

a- Irregular edge

b- Irregular pigmentation

c- Become malignant

d- Areas under the cloth

e- Associated with old age

Uncomplicated haemorrhoids except

a- Pruritus ani

b- Discharge mucus

c- Bulging mass

d- Perianal pain

e- Blood in stool
Fistula is:

a- Abnormal tract between two epithelial surfaces

Woman 17 week pregnant, least likely

a- Ectopic pregnancy

Proverb explanation, assessment of concrete thinking?

a- Dysthymia

b- SCH

Deja - vu:

a- Temporal lobe

b- Parietal lobe

Grandiose character doesn’t care about others, feels unique:

a- Narcissistic personality disorder

b- Delusional disorder

c- Histrionic

d- Borderline disorder

e- Schizoid personality

A newborn baby during the first 24 hours can’t move his midarm forceps delivery
was used. Mostly injured is:

a- Branchial plexus
b- Fractured humerus

Patient in ER with overdoses of unknown substance, miosis resp. O2

a- Naloxone

b- Acetylcholine

c- Prostigmine

d- Atropine

Wheat divided by 5 equal good, good is left and right?

a- Clang mixing

b- Association disorder

c- Delusion included

Dilatation and curettage most appropriate:

a- After labour>cervical incompetence

b- Primigravida (delivery) with some vaginal bleeding

c- Post coital bleeding

d- Diagnosis of inappropriate? (Irregular) bleeding*

aippg.com

All of the following are complications of continuous progesterone, Depot-Provera


injection except

a- Amenorrhea 6 - 12 months

b- Amenorrhea after 3rd injection


c- Irregular bleeding

d- Weight gain

e- Hypertension*

Oxytocin, what is correct

a- Is a steroide

b- Secreted by anterior pituitary

c- Action most on involuntary muscles

d- Has antidiuretic effect

e- Formed by oxitocinease?

Greenish, paste discharge from the nipple. Nipple is retracted:

a- Intraductal papiloma

b- Breast Ca

c- Hyperprolactinemia

d- Duct ectasia

e- Breast abscess

Oral contraceptive pills, ethynyl estadiol (COCP), Microgynon, work at the level of:

a- Ovarium

b- Hypothalamus

c- Anterior pituitary

d- Endometrium

e- Cervical mucus
Patient lost 3kg in 1 month, tired, vomiting dark blood. For the last 6 years he was
treated for duodenal ulcer by antacids. Now taking only aspirin for pain:

a- Complication of the Rx of peptic ulcer

b- He needs reassurance

c- Duodenal Cancer

d- Gastric cancer

e- Cancer of antrum

Mother with a baby has oedema on the ankles and hands, ascites, protein+++ jaundice
and bilateral cataract as well. Previous baby with similar condition:

a- ABO incompatibility

b- Retrolental fibroplasia

c- Rubella embryopathy

d- Galactosemia

Bilateral loin pain, signs of infection and fever, hypertensio, hematuria:

a- Papillary necrosis

b- Post streptococcal glomerulonephritis*

c- Horseshoe kidney

Proteinuria occurs in all of the following except:

a- Congenital nephritis

b- Rapid progressive glomerulonephritis

c- U.T. infection
d- Rheumatoid arthritis

e- Horseshoe kidney

In fat embolism Petechiae will be present:

a- Upper part of the body

b- Subungual region

c- Hands

d- All over the body

(2 or 3 lines written) patient with episcleritis, scleritis, not responding to sulphasalazine.


All is true except:

a- Rheumatic arthritis

35 yrs old pt. with compound fractured tibia protruding 2 cm outside, skin laceratio.
Most important first step in Rx is:

a- Debridement of the skin

b- Tetanus and antibiotic prophylaxis

c- Internal fixation by nails and screws

A diabetic patient has fever, pain and tenderness after superficial laceration of the
ankle, red swollen leg up to knee. What is correct?

a- It is streptococcal infection

b- It is risk for vein thrombosis so pts. Needs Doppler for investigations


R.dass, Oct 26, 2007

#51

R.dass

R.dass Guest

Anti D immunization:

a- if given in second trimester reduce risk of autosensitization

b- Is an active immunization

c- Reduce auto hemolysis

d- useful if given in first 24 hours

e- It is not important in B POSITIVE baby

A female patient (older age) Hb 60 (Normal 115-160) present with fatigue,

MCV 118 (N76-100)

WBC 2.3 (N4.3-11)

Platelets 90 (N 150-400.000)

a- haemolytic anaemia

b- Pernicious anaemia

c- Acute lymphoblastic leukaemia

Hypercalcaemia, Ca is 4 x normal, K5.5 (N 3.5-5.5 mmol/L), Ca in urine (was in normal


range), PO4 normal. What is least likely?

a- hypocalciuria hypercalcemia

b- Hyperparathyroidism

c- Paget’s disease
d- Bone tumour, metastatic bone disease, and surge. osteolysis?

Chest X-ray right lower lobe consolidated with effusion, increased vascularity

(Or patchy affection of lung)?

a- Streptococcus pneumonia

b- Mycoplasma pneumonia

c- Pneumococcal pneumonia

d- TBC pneumonia

Patient with acute onset hand weakness………

a- Multiple sclerosis

b- Syringomyelia

Patient taking heparin, you will follow up levels by:

a- Fibrinogen

b- Reduced INR

c- Bleeding time

d- APTT *

A female patient with Von Willebrand’s disease, after operation (+- Bleeding time?)
because:

a- Reduced fibrinogen

b- APTT

c- Bleeding time increased

d- Reduced factor VIII


Pseudo dementia simulating psychiatric disorder most commonly in:

a- Depression

b- Schizophrenia

c- Alcoholism

d- Drug abuse

Hypnagogic hallucination:

a- Delusion

b- Schizophrenia

c- Normal personal variably

Risk of breast cancer is Increased in all of the following except:

a- Pregnancy after 35 yrs old

b- Mother, sister with history of breast cancer

c- Endometrial carcinoma

d- Artificial menopause

e- OCP

Other breast cancer

Syringomyelia:

Characteristic- Dissociated artery loss

Pain temperature

But preservation of dorsal column


BT - Mainly Platelet + other factor - 1-6 min

CT - Coagulation factor - 6-10 min

PT - Prothrombin + other factor - 12 sec.

A man with an inguinoscrotal swelling which decrease when lying down

a- Torsion of testis

b- Epididymis

c- Spermatocele

d- Varicocele

e- Direct inguinal hernia

Pt. old age, bed ridden, constipation, bouts of diarrhoea:

a- Faecal impaction

b- Colonic carcinoma

Acidosis with normal anion gap:

a- Tissue ischaemia

b- Alcoholism

c- Hyperparathyroidism

d- Renal failure

e- Hypoaldosteronism
Baby with diarrhoea, can’t thrive since birth, weight gain rate not adequate:

a-

b-

c- Galactosemia

d- Cystic fibrosis

Gross ascites, cytology proved malignancy. Most likely you except:

a- Supraclavicular lymph node

b- Rectal lymph node

c- Hepatomegaly

d- Splenomegaly

R.dass, Oct 26, 2007

#52

R.dass

R.dass Guest

Pt. 8 week of gestation, you will do ultrasound for all of the following except:

a- Detect site of the foetus in uterus

b- To detect congenital malformation

c- Blood clot to give level of prognosis for pregnancy to continue

d- To confirm FHS

e- Hydatiform mole
Patient with increased pulse (not detectable?) BP reduced sudden severe pain shock,
dark blood, abdominal distension + abd. Tenderness, absent intestinal sounds:

a- Acute hepatitis

b- Cholecystitis

c- Acute pancreatitis aippg.com

d- Ischaemic intestinal pain

e- Perforated peptic ulcer

All of the following help or proved survival after myocardial infarction except:

a- Streptokinase

b- B- blockers

c- Aspirin

d- ACE

e- Calcium channel blockers

Patient, 50 years old lady, has attack of blurred vision for 3 minutes with ipsilateral body
weakness:

a- Optic chaisma

b-

c- Carotid artery stenosis

d- Vertebrobasillar insufficiency

Baby 4 weeks old with 1 week history of vomiting (food related?) otherwise baby is well,
weight gain is 200 grams/week:

a- Decrease mother diet


b- Reduce quantity of breast feeding

c- No investigation needed

d- Pyloric stenosis

Severe hypokalemia

a- Chronic renal failure

b- Vomiting pyloric stenosis

c- Chronic use of purgatives

d- Primary hyperaldosteronism

Female patient (adult age) with 6 months history of diarrhoea with mucus, 4-5 loose
stool at motion:

a- Diverticulosis

b- Repetitious use of laxatives

c- Ulcerative colitis

d- Irritable bowel syndrome

e- Pseudomembranous colitis

Patient with headache, vomiting, papilledema, slowly progressive confusion are early
manifestation of:

a- Frontal lobe meningioma

b- Tumour in the ventricle

c- Subarachnoid haemorrhage

d- Subdural haematoma

e- Temporal /patient
What will become chronic?

a- Hepatitis A

b- Hepatitis B

c- Hepatitis C

d- Listeria monocytogenes

Bilirubin is 5 x normal, ALK 3 x normal, pain in epigastrium, transaminases

Increased, amylases 2 x normal:

a- Liver cholecystitis

b- Acute hepatitis

c- Pancreatitis

d- Cancer ampulla of Vater

Biles duct stones except:

a- Always show gall bladder sepsis

b- Can be asymptomatic

c- Solitary large stone usually cholesterol stone

d- Pigmented stone are haemolytic

e- Mostly radiolucent, could be multiple

A case of myocardial infarction, after improvement of acute attack long-term


prophylaxis:

a- Clonidine

b- Aspirin
c- Warferin

A primigravida 165 cm tell, station 0, and cervix >4 cm, you think palpable pelvic
contraction on vaginal examination: Next step in management:

A- Trial of vaginal labour

b- Pelvic radiometry

c- Ultrasound

d- Caesarean section

R.dass, Oct 26, 2007

#53

R.dass

R.dass Guest

Mamography what is correct:

a- Higher rate of lump detection than self examination

b- Easily differentiate between cyst and solid lump

c- More beneficial in premenopausal than postmenopausal women

d- Confirmation of mass after palpation

aippg.com

A patient with sudden lower abdominal pain, previously well, last menstrual period 2
weeks ago. US of ovarium show 3 cyst: 0.5 cm, 0.7 cm and 2.2 cm. What is your
management?

a- Wait four weeks and repeat medical checks

b- Aspiration of the 2.2 cysts under ultrasound


c- Laparotomy view of ovarian, aspiration of the 2.2 cm cyst for cytology

d- Excision of the ovarium, hysterectomy

e- Reassurance

Panic disorder. All is true except:

a- At least 20% will get panic disorder in the life

b- Same male: female ratio with absence of agorrhaphobia

c- Increased with situation

d- Avoid precipitating factors

Bell’s palsy. All of the following is true except:

a- 50% patient will remain with residual lesion

b- Sudden pain

c- Complete recovery in 9 - 12 weeks

d- Associated with recent virus infection

e- Facial n. palsy

Carpal tunnel syndrome in pregnancy:

a- Improved with progression of pregnancy

b- Pain could be reduced in midway extension of the palm

c- Pain over the thenar muscle

d- Rarely needs surgical interview

e- Needs decompression
Primigravida 28 yrs old, high blood pressure, uric acid elevated, albumine in urine:

a- She has preeclampsia

b- Associated pregnancy as well

c- Treatment with antihypertensives

Tourette syndrome all is correct except:

a- Motor and vocal tics

b- Two or more bout per day

c- The person not distressed with the vocal tics

d- Coprolalia

e- Onset always < 18 years of age

Complication of diabetes mellitus except:

a- Postural hypotension

b- Bradycardia

c- Cataract

How to differentiate between ovarian cyst and ascites:

a- Central dullness lateral tympany

b- Lateral dullness central tympany

c- Shifting dullness

All of the following is true about diffuse fibrous alveolitis except:

a- hereditary, may be present in families


b- Cyanosis or dyspnoea

c- Collapse can be present

d- Reduced Alpha antitrypsin? In other parts of the body

A baby bubbles, smiles, roll over sits with hand s forward, stand supported, leans
forward to reach rattle what is age:

a- 3 months

b- 7 months

c- 10 months

d- 12 months

The risk of colonic cancer is highest with:

a- Ulcerative colitis

b- Post irradiation colitis

c- Chron’s disease

d- Familiar Melanosis colli

You are senior doctor in the hospital asked by hospital director to try a new
antipsychotic drug on patient and you are asked to get consent from the patient:

a- You are not allowed to take consent as a psychiatric patient is considered involuntary
patient and you can not try new drug in his treatment

b- Try the drug because it is for his benefit anyway as involuntary patient he doesn’t
know what is benefit for him

c- Tell director that it is not your job/or try other psychiatric patient who can write
consent

d- Try the drug without taking consent


e- Ask relatives and friends for their opinion, they should be present at a time of trial

Patient is HIV positive. What is correct?

a- HIV+ -ve means he has AIDS

b- He can transmit virus through his saliva

c- After 2 years he will not be infective

d- He has infection and long life infectivity

What is most common cause of preventable blindness in the world?

a- Cataract

b- Trachoma

c- Glaucoma

What is incidence of Down syndrome in 38 years old woman?

a- 1:200

b- 1:50

c- 1:500

d- 1:100

R.dass, Oct 26, 2007

#54

R.dass

R.dass Guest
Which of the following has least likely malignancy potential?

a- Gardner’s Syn

b- Peutz-Jagher Syn

c- Tubular adenoma

d- Villous adenoma

e- Ulcerative colitis

Aims of operation for morbid obesity. All is correct except:

a- Reduced caloric intake

b- Increased satiety period

c- Reduced carbohydrate consumption

d- Reduced energy absorption

e- Increased metabolic rate

Immunoglobulin prophylaxis should be given after contract with certain diseases. Which
disease is least likely important?

a- Rubella (German Measles)

b- Mumps

c- Cytomegalovirus

d- Toxoplasmosis

e- Varicella

Patient with spherocytosis after splenectomy what is least likely to be found?

a- Same RBC fragility

b- Persistence of Spherocytes
c- Persistence of anaemia

d- Retaining normal life span of red blood cells

e- Leukocytosis

A female patient UTI, pain, fever urine culture Escherichia colli after course of
antibiotics no E. colli on urine exam. It was first attack; she had no previous attack before:

a- Come again for follow up

b- Should take antibiotics as proxy axis

c- No further investigation or therapy needed

d- Relapse is common

e- Investigate to establish cause and underlying disorder

55 yrs old female with palpable breast lump most common is:

a- Breast carcinoma

b- Fibroadenoma

c- Lipoma

d- Fibrocystic disease

All of the following cancers usually metastasis to brain except:

a- Prostrate carcinoma

b- Breast cancer

c- Bone cancer

d- Liver cancer

e- Lung cancer
Uveitis is most common:

a- Reiter’s disease

b- Behcet’s disease

c-

d- Ankylosing spondylitis

Mantoux test is negative in:

a- Miliary Tuberculosis (immunocompromised patients)

A female patient had high level of prolactin, ST scan show adenoma 1.8 cm in the sella
turcica that has invaded optic chaisma for 3mm. What is Rx?

a- Transphenoidal surgery

b- Bromocriptine

c- Transnasal surgery

d- Radiotherapy

e- Measure prolactin level later and follow up

Woman with mastalgia, conservative treatment failed, what is next step:

a- Clomiphene

b- Radiotherapy

c- Surgery

d- Bromocriptine

e- Danazole
Neural tube defect all is true except:

a- Environmental and social factors are important

b- Increased risk in next protein

c- Low alpha pheto protein

d- Thiamine intake will reduce risk for NTD

Boy with height 90th percentile and weight 98 percentile for his age

a-

b-

c- Advanced bone age *

PHOT: dorsum of the hand, reddish and black part on the edge?

a- Keratoacanthoma

d- Inflammation of solar keratosis

e- Malignant skin tumour

R.dass, Oct 26, 2007

#55

R.dass

R.dass Guest

3 days old baby, collapsed in the cot, had high respiratory rate, tacycardia, breathless,
central cyanosis no murmurs, peripheral pulse impalpable

a- Fallot Tetralogy

b- Left hypoplastic heart *


c- Patent ductus arteriosus

d- Severe Ventricular Septal Defect

e- Transposition of great vessels

All is consistent with duodenal ulcer except:

a- Hunger pain

b- Loss of appetite aippg.com

c- Night pain

Helicobacter pylori eradication is best diagnosed by:

a- Urea breath test

b- Histology

c- Serology

Older patient, A.F. hypertensio, ECG left atrial? Coronary ECHO

a- Diuretics

b- Warferin

c- Sterosis

d- Aspirin

Infant with contact dermatitis, all are true except:

a- Improve when decrease contact with predisposing factors

b- Systemic steroids

c- Doesn’t respond to topical steroids


d- Oral antibiotic for infary infuse

e- Wash a water pat dry (no seap)

Wernicke’s encephalopathy what should be given immediately?

a- 50ml 50% dextrose

b- Immediate glucagon injection

c- Immediate glucose injection

d- Thiamine followed by glucose infusion

Patient with cardiac arrest success of resuscitation is best monitored by:

a- Colour of the skin

b- Radial artery pulse

c- Pupil dilatation and reaction

Patient with ptosis and miosis. Horner’s Syndrome (sympathetic chain pathway),
lesion can be in all of the following except:

a- Parotid enlargement

b- Common carotid artery

c- III nerve (Oculomotor)

d- Glaucoma

45 years old man with episodes of headache, awake him up early in the morning, he has
periorbital injection, lacrimation (Dg. Cluster headache). What is Rx?

a- Carbamazepine

b- Methysergide
c- Diuretics

d- Amitriptyline

55 years old female patient felt on outstretched hand? Colle’s?

a- Crepe bandage

b- Internal fixation, plaster immobilisation at least 10 weeks

c- External fixation would be enough to return hand to it’s normal function + plastic
immobilisation for 8 weeks

d- Plaster probably no need, only follow up and check n. medianus

e- Plaster cast and reductio are incorrect treatment

Photo: Scalp of the boy, area of reddish and born spots? Only a few pieces of hair:

a- Tinea capitis

b- Trichotillomania

c- Alopecia areata

d- Psoriasis

e- Eczema

Regarding nasopharyngeal carcinoma:

a- Benign by compression, locally destructive*

b- Spread slowly

c- Nasal lymph Origin

d- Trans frontal surgery

e- Radioresistant
50 years old female had amenorrhea for 11 months, now has vaginal bleeding most
common:

a- Atrophic vaginitis

b- Endometrial carcinoma

c- INF.

Female patient with history of psychosocial stress now she has paralysis of one limb
(Psych. May 2001 Rx of conversion):

a- Referral to psychotherapist

b- Wheel chair & social work introduced to seminar about psychosocial stress

c- Admission to hospital

Infant with acute bilateral flaccid paralysis, no sensation, no ankle reflex, received all
immunization regularly according to schedule:

a- Atypical infection with wild type poliovirus

b- Guillain Bare sy

c- Postimunisation polio virus infection

R.dass, Oct 26, 2007

#56

R.dass

R.dass Guest

Anterior 2/3 sensation of tongue is affected. It is lesion of:

a- trigeminal nerve
b- Facial nerve

c- Hypoglossal nerve

d- Vagus

e- Glossopharyngeal

A female pregnant, at 12 weeks ,she had abortion some placental tissue passed, size of
the uterus 15 weeks, evidence of Hydrops, what is next appropriate investigation?

a- Follow up for the maternal B-hCG level

b- Examination of the dead foetus

c- D & C and investigate mole

d- Tell her the incidence of Hydrops in next pregnancy

aippg.com

A patient with COAD has resting dyspnoea, cor pulmonale hypoxemia and polycythemia.
Which of the following measures would have resulted in decreased number of
hospitalization?

a- Long time inhalation cortisone

b- Phlebotomies until Haematocrit is reduced to 50%

c- O 2 inhalation at home

d- Long time prophylactic antibiotics

Regarding Emphysema what is correct:

a- Is limited only to bronchioles

b- Reduced FEVI/Vital capacity

c- Will not progress to worse if smoking

d- One of his parents died of emphysema


e- Doesn’t occur in old age

Erythema nodosum is found in all of the following except:

a- Rheumatic fever

b- Rheumatic viridans

c- Streptococcus viridans

d- Haemolytic arthritis

e- TB

Amyloidosis can occur in all of the following except:

a- Leprosy

b- Chronic active hepatitis

c- Renal failure

d- TBC

e- Multiple Myeloma

Short statue in adulthood most common is associated with all of the following except:

a- Congenital adrenal hypoplasia

b- Tuner’s syndrome

c- Premature puberty

d- Obesity

e- Psychosocial deprivation

A diabetic patient has ulcer on the sole at the head of the II metatarsal bone most likely:
a- Neuropathy

b- Injection

c- Psoriasis

Diabetic neuropathy is associated with all of the following except:

a- Impotence

b- Bradycardia

c- Urinary incontinence

d- Hypotension

Photo: head and neck (woman) extended neck show mass (at site of thyroid gl.
apparently), larger on one side than on other

a- Tumour

b- Multiple nodular goitre

Patient with dullness on the base, right side of the chest. Bronchial breathing above the
dullness:

a- Right pleural effusion

b- Right lower lobe collapse

c- Emphysema

The question mentioned that pt. has bilateral patchy infiltrates on CXR:

a- Pneumococcal pneumonia

b- Staphylococcal pneumonia

c- Streptococcus pneumonia
d- Mycoplasma pneumonia

R.dass, Oct 26, 2007

#57

Joseph.

Joseph. Guest

Which is the commonest cause of neonatal meningitis?

a- E coli

b- Haemophilus influenza

c- Streptococcal pneumonia

d- Meningococcal

e- Viral

Which is the most likely cause of iron deficiency anaemia in Australia infants?

a- Thalassemia b- Dietary deficiency

c- Chronic blood loss d- Meckel’s diverticulum

e- Parasites

A 14 yr old girl presented with pain on the right inner aspect of her right knee, limp,
limited abduction at the hip. Which is the most cause?

a- Slipped femoral epiphysis

b- Perthes disease

c- Osteochondritis of the tibial tuberosity (Osgood Schlater disease)

d- Undiagnosed congenital hip dislocation


e- Osteochondritis dessicans

In petit mal epilepsy, all are true except -

a- Age 5-10 yrs

b- Normal EEG excludes the diagnosis aippg.com

c- Familial incidence

d- Precipitated by photo stimulus

e- Brief episodes of impaired consciousness without postictal symptoms

Hypospadias -

a- Greater incidence in the first borne male child

b- Always associated with meatal block

c- Incidence of 1 : 350 male birth

d- Associated with urinary tract malformation in 30% of cases

e- Associated with abnormal testosterones secretion

Syrup of Ipecac is indicated in all of the following except -

a- Tricyclic antidepressants poisoning

b- Caustic soda poisoning

c- Aspirin poisoning after 4 hours

d- Unknown pills poisoning

e- When using charcoal in the treatment

A pregnant woman in her 35 wks of pregnancy had premature rupture of membrane


and developed streptococcal infection. Which of the following is true?
a- A transplacental transmission infection

b- Prevented by rupture of the membranes

c- Systemic infection which lead to increased mortality rate

d- Not affected by penicillin

A 4 yr old child with left hip pain. The most likely cause is -

a- Septic arthritis b- Osteomyelitis

c- Perthes disease d- Transient synovitis

Reye’s syndrome. All are true except -

a- A likely viral etiology

b- Deep jaundice

c- Metabolic acidosis

d- Occurs in children

e- Hypoglycaemia is not frequent

An infant with patent ductus arteriosus developed congestive heart failure. You except
to find all of the following except -

a- Hepatomegaly b- Collapsing pulse

c- tense anterior fontanelle

d- Congested neck veins

e- Diffuse expiratory wheeze

Joseph., Oct 26, 2007

#58
Joseph.

Joseph. Guest

A mother presents with her 6 ½ month old child whom you have given a DPT two
weeks ago. She complains that he is fussy at daytime especially when she leaves him alone
and he awakens at night. Examination is normal. What’s your action?

a- Urine examination & blood culture

b- Reassure her that this is a normal behaviour after vaccination

c- Reassure her that this is a normal behaviour for his age and will settle with time

d- Prescribe a full course of antibiotics

e- Investigate for peptic oesophagitis

A 6 yr old girl on examination looked pale, had bruises and a palpable spleen, full blood
examination revealed:

HB - 5.5 gm/L

WBC - 2600 (Decreased polymorphs, increased lymphocytes - 80%)

Platelets - 2500

This is consistent with -

a- Acute leukemia

b- Idiopathic thrombocytopenic purpura aippg.com

c- Aplastic anaemia

d- Iron deficiency anaemia

e- Congenital spherocytosis

It is more likely to prevent child abuse by counseling of -

a- A mother with post partum psychosis


b- Parents with a history of child abuse in childhood

c- Parents of a child with congenital anomaly

d- Parents who have abused a previous child

e- A father unemployed since the birth of his 6/12 child

Which of the following is least likely to help the immunological system of a breast fed
baby?

a- IgA b- IgG

c- macrophages d- T lymphocytes

e- Lactoferrin

Regarding Neuroblastoma, all are true except -

a- The younger the child, the worse the prognosis

b- Increased urinary vannilyl mandelic acid

c- Most commonly abdominal

d- If it occurs in the chest, it is most commonly in the post mediastinum

e- Metastasizes early

In a child who developed acute parotitis -

a- Meningo encephalitis may occur before, during or after parotitis

b- Parotitis are the salivary glands that are infected

c- Incubation period is 7-10 days

d- Sterility is common

All are true regarding Down syndrome except -


a- Incidence in women above 40 is 2%

b- Trisomy 21 is the most common

c- The risk is markedly increased if a parent has a translocation of chromosome 21

d- Diagnosed by elevated alpha feto protein

e- Causes mental retardation

In Coeliac disease, what is least likely?

a- Giardia infection can mimic coeliac disease

b- Many fat globules in the stool

c- Anaemia

d- Child’s growth retarded

e- Diarrhoea

A 6 month old child is brought to you because his mother noticed a swelling in the groin
yesterday. Today the swelling is gone and the only thing found on exam is a thickening of
the spermatic cord. What’s your management?

a- Early herniotomy

b- Review after one week

c- Reassure his mother that there is not any problem

d- Plain x-ray

e- Orchidopexy

Joseph., Oct 26, 2007

#59

Joseph.
Joseph. Guest

A child with insulin dependent diabetes mellitus. Which is the true statement?

a- Present within one month of duration

b- 50% have significant family history

c- More common in children of obese parents

d- 10% children can be treated with oral hypoglycemic agents

Two months old child’s mother noticed a firm lump in her baby in the left side of
upper abdomen while bathing. Child has bilateral periorbital Ecchymosis. What’s the
most likely cause?

a- Wilm’s tumour

b- Neuroblastoma

c- Splenomegaly

d- Polycystic kidney

10-day-old male infant presented with vomiting. On examination he was found to be ill
looking and dehydrated. Investigation revealed serum Na 119 meq/L, K - 7.9 meq/L, CI - 80
meq/L. What is the probable diagnosis?

a- Pyloric stenosis

b- Congenital adrenal hyperplasia

c- Gastroenteritis

d- Dehydration

e- Gastro oesophageal reflux

Aippg.com
Mother of a 6-year-old child who had contact history of varicella zoster in the school
asks about the incubation period. What is the incubation period for varicella zoster?

a- 5 - 10 days

b- 10 - 15 days

c- 15 - 20 days

d- 1 month

e- 2 months

25 yrs old mother delivered a baby with cleft lip asked about the chance of getting cleft
lip for the second child -

a- 4% chance

b- Risk is the same as general population

16-month-old child had fever for 2 days and then fever subsided and developed rashes
over the trunk. What’s the most likely cause?

a- Roseola infantum

2yrs old child with one month history of foul smelling serosanguinous unilateral nasal
discharge, most probable cause is -

a- Foreign body

b- Nasal polyps

c- Sinusitis

Child presented with fever and mild neck stiffness. CSF examination showed that
glucose - normal, protein - 1.1 mg, polymorphs - 50, lymphocytes - 100. No organisms
seen in the CSF. Most probable cause is -
a- Herpes

b- Echo virus

c- Tuberculosis

d- Haemophilus influenza

e- E coli

A child who is able to babble and sites without support with an arched back reaches
objects with palmer grasp. Which of the following is consistent with child’s
developmental age?

a- 3 months

b- 6 weeks

c- 7 months

d- 9 months

e- 11 months

18 months old child brought by her mother who says that the child is not babbling. The
hearing test done at 10 month was normal. Now, he appears to be responding to sound.
What is the next step in the management?

a- Reassure that this is normal

b- Audiological assessment

c- Repeat the hearing test now

Joseph., Oct 26, 2007

#60

Joseph.

Joseph. Guest
12 yrs old child presented with abdominal pain, urinary symptoms. Mid stream urine
specimen revealed 100,000 organisms. Neutrophils were less than 10. No proteinuria. No
haematuria. You will do all of the following investigations except -

a- Immediate micturating cysto urethrogram for reflux

b- Treat with Cotrimoxazole

c- Repeat urine culture

Child with funny fanks. The child has occasional episodes of flexion of trunk with spastic
limbs. Child also had upper respiratory tract infection at that time. During that time the
child does not appear to respond to calling -

a- Infantile spasm

b- Infantile myoclonic epilepsy

c- Breath holding attack

d- Petit mal epilepsy

e- Febrile convulsion

Which of the following is a normal phenomenon?

a- Fusion of labia in a female child at birth

b- Engorged breast at birth in a male child aippg.com

Regarding taking a consent in a 15 year old girl awaiting for a surgery, all of the
following statements are true except -

a- Her mother can give formal consent for the surgery over the phone

b- In an emergency life saving situation consent is not necessary

c- Grandmother who is not living with the patient now can give consent
d- If the girl is working and independent of her parents, the patient herself can give
consent

Anterior uveitis commonly found in which of the following condition?

a- Juvenile chronic arthritis

b- Henoch Schonlein purpura

Regarding anti Rh Immunoglobulin given to mother, how does it act?

a- Immunoglobulin coats fetal RBC and prevents immune reaction in the mother

b- Immunoglobulin blinds and destructs the fetal RBC

Child who is 3 months old with a strawberry naevus in the back. What is the
management?

a- Excision

b- CO2 laser therapy

c- Observation

d- Radiotherapy

4 yrs old boy with bronchial asthma. In the emergency department which is the best
investigation to assess the severity?

a- Peak flow rate

b- Intercostals recession

c- Bilateral rhonchi

A child comes with diplopia of recent onset. He has evidence of 3rd nerve palsy, what is
the probable?
a- Oligodendroglioma

b- Medulloblastoma

c- Cerebellar tumour

A child comes with subdural haematoma. Skeletal survey revealed multiple healing
fractures in different ages -

a- Non accidental injury

b- Osteogenesis imperfecta

c- Skull fracture

Joseph., Oct 26, 2007

#61

Guest

Guest Guest

Joseph.

Guest

Posted: Thu Oct 25, 2007 3:39 pm Post subject:


--------------------------------------------------------------------------------

In paracetamol poisoning, the cause of morbidity is-

a- Hepatic injury

b- Metabolic acidosis

c- Metabolic alkalosis

d- Hypoglycemia (cause of mortality)

In tricyclic anti depressant poisoning WOF are true except:

a- HCO3 to prevent arrhythmia

b- Flumazenil should not be given if is due to TCA + Diazepam

c- Bile acid

d- Magnesium

e- Diazepam to

Regarding bronchial asthma all are true except:

a- Total lung volume is decreased aippg.com

b- Increase in residual volume

c- Decreased tidal volume

d- Decreased vital capacity

WOF are true regarding Asthma except:

a- It increases Br. Sensitivity

b- It does not occur after 50 yrs of age


c- Inspiratory crackles

A pt. with one sided dullness with shifting of trachea on the same side, Diminished
breath sound and respiratory movement. Which of the following is true?

a- Pleural effusion with collapse

A 23 yrs old lady with distended abdomen, glositis, muscle wasting, anaemia,
macrocytosis, diarrhoea no history of surgery. WOF is true:

a- Gluten sensitive enterocolitis

b- Duodenal biopsy is reqd. for confirm the diagnosis

c- Crohn’s disease

d- Ulcerative colitis

e- Pernicious anaemia

An abattoir worker came with H/O fever, myalgia and dark urine. The most likely cause
is

a- Leptospirosis

b- Brucellosis

Regarding eradication of H. Pylori which is true

a- it helps in healing of the ulcer

b- Prevents relapse rate

c- Need for long time therapy

Which of the following is not true regarding transferrin?


a- Increased in iron deficiency anaemia

b- Increase in chronic infection

c- Decrease in nephritic syndrome

d- Increased in pregnancy

Pt. with severe chest pain, absent carotid pulse, pain radiating to the back. What would
you find in chest X-ray?

a- Widening of the mediastinum

In thyrotoxicosis all of the following are present except

a- Limb girdle muscle weakness

b- Small muscle wasting

c- Atrial fibrillation

Regarding Myasthenia gravis which is true

a- Some people does not have antibody

b- Thymectomy is only done when thymoma is present

c- Reflexes are diminished after repeated stimulation

d- EMG always slow

e- Electomyograph

Guest, Oct 27, 2007

#62

Guest

Guest Guest
Author Message

Joseph.

Guest

Posted: Fri Oct 26, 2007 2:41 pm Post subject:

--------------------------------------------------------------------------------

A six week old infant vomiting intermittently since birth, she looks thin not gaining
weight. There is no abnormality on physical examination. What is your diagnosis?

a- Cystic fibrosis

b- Phenyl ketonuria

c- Pyloric stenosis

d- Subdural haematoma

e- Regurgitation

In cerebral palsy al of the following are true except -

a- 90% progress to serve mental retardation

b- Most have IQ of 70
c- Mostly associated with Down’s syndrome

d- 70% will have moderate handicap

e- 10% will improve

A 2 yrs old boy has a continuous murmur on the left sternal while sitting. One year later
comes to you because of fever. The murmur disappears while lying down. The cause of
the murmur is -

a- The murmur was innocent

b- The child had developed right heart failure

c- Closure of PDA

d- The child has developed SBE

e- Venous hum

Which one of the following CHD close spontaneously?

a- Small VSD in 12 month old baby

b- ASD in 3 year old aippg.com

c- PDA in a 3 month old

d- Large VSD in 1 year old

e- PDA in a 1 year old

The first step in the management of a new born asphyxiated infant -

a- Aspiration of pharynx

b- Intubation

c- Oxygen mask

d- CPR
e- Chest x-ray

A new born presents with jaundice at 18 hours of age. What is the likely cause?

a- Physiological jaundice

b- Excessive hemolysis

c- Liver enzyme deficiency

d- Thalassemia

e- ISO immunization

The first sign of puberty in boys is -

a- Increase in testicular size before penile enlargement

b- Increase in penile length before grown spurt

c- Growth spurt

d- Axillary hair growth before pubic hair

e- Pubic hair before growth spurt

Mother of a 6 week old child noticed a lump in the groin while crying. On examination
the child is normal. Tests are palpable both in normal position. What is the next step in
management?

a- Reassure the mother

b- Review after one week

c- Arrange for surgery consult

d- Give antibiotics

e- Aspirate
Back to top

Joseph.

Guest

Posted: Fri Oct 26, 2007 2:42 pm Post subject:

--------------------------------------------------------------------------------

One of the following is true regarding the treatment of unconjugated


hyperbilirubinemia -

a- Phototherapy

b- Exchange transfusion

c- Phenobarbitone

d- UV light
e- Antibiotics

aippg.com

A 10 yrs old child repeatedly suffers with right side headache and left side hemiparesis
and recovers completely. Which of the following is your diagnosis?

a- Migraine b- Hysteria

c- Vertebra basilar insufficiency

d- Carotid artery thrombosis

e- Cerebral tumour

A 3 yrs old with a 6 week history of unilateral purulent nasal discharge and occasional
nose bleeding. What would be the diagnosis?

a- Foreign body b- Nasal diphtheria

c- Polyp d- Sinusitis

e- Bilateral adenoids

A 6 week old infant presents with unilateral eye discharge since birth. Pathology shows
polymorph-nuclear cells -

a- Gonococal infection

b- Chlamydia infection

c- Closed nasolacrimal duct

d- Reiter’s syndrome

e- Viral infection

Which one of the following is not useful in the treatment of acute asthma in children?
a- Sodium chromoglycate

b- Steroids

c- Salbutamol

d- Aminophylline

e- Adrenalin

What is the advantage of live measles vaccine?

a- Need refrigeration

b- It can produce atypical features of disease in the future

c- Can be given immunocompromised patient

d- Can be given orally

e- Higher chance to cause allergic reaction

A 7 yrs old child falls from a tree. After 7 days complains of fever 38 degree centigrade
with pain and tenderness in the right leg. What is your diagnosis?

a- Acute leukemia

b- Stills disease

c- Osteomyelius

d- Osteogenic sarcoma

e- Perthes disease

One month child has a capillary nausea (port-wine stain) on face. Which of the following
is correct?

a- It will grow and get worse

b- It will stay forever


c- Cryotherapy is effective treatment

d- They are usually associated with A.V. malformation in brain

e- By one year it will get better

Cause of iron deficiency in infancy. All are true except -

a- Thalassemia

b- Coeliec disease

c- Malabsorption

d- Unfortified cow milk

e- Hook worm

The following are true regarding coelial disease in childhood except -

a- Fat gloodles in stool

b- Flat villa

c- Associated with iron deficiency anaemia

d- Bloody diarrhoea

e- Malabsorption

An 8 yrs old child while watching TV at night suddenly develops deep and rapid
breathing. On examination no abnormal physical signs were noticed. What is your
diagnosis?

a- Ketoacidosis

b- Petit mal epilepsy

c- Panic attack

d- Infantile spasm
e- Pneumonia

Back to top

Joseph.

Guest

Posted: Fri Oct 26, 2007 2:43 pm Post subject:

--------------------------------------------------------------------------------

Management of a schizophrenic patient includes also the family members. The most
popular family approach is -

a- health education about schizophrenia to the family member

b- Job training for the family members

c- Cooking classing for the right technique for family

d- Group therapy
A patient with HIV comes and tells you he wants to die -

a- Refer him to a euthanasia group of social worker

b- Explain him about a doctor’s principles and ethics

c- Tell him about getting overdose of drugs by your medical knowledge

d- Pretend you don’t understand him and ask him what he means?

A truck driver complains that he cannot sleep, he drinks 6 pints of bear/day 5 cups of
strong coffee to help him stay awake on the wheel what’s your management?

a- Tell him to go straight to sick day and lie down while you run some investigation

b- Tell him to stop drinking and use decaffatated coffee

c- Send him to psychiatric hospital aippg.com

An eight year old boy suddenly developed enuresis after being disappear for a long
time. It starts shortly after his father starts to drink, loose a job and quarrel with his
mother. This ay be due to -

a- Family conflict

b- Depression

c- School phobia

d- Antisocial behaviour

e- Childhood delinquency

A 4yr old child has knock knee. On examination, medial maleolus distance of 3 inches
apart. The child is able to just touch the knee. What action would you take?

a- Investigate for rickets

b- Refer for surgery


c- X-ray

d- Reassurance that it will improve with growth

The commonest cause of unproductive cough in children in Australia is -

a- Asthma

b- Whooping cough

c- Pneumonia

3. A 6 yr old child with increased thirst, drinks 2 glasses of milk extra, have to pass urine
after going to bed. What is the first step?

a- Do GTT when convenient

b- Do blood glucose immediately

A child with history of infectious mononucleosis 6 months ago now comes with lethargy
and tiredness, poor school attendance, sore throat. What’s the management?

a- Blood exam and urine microscopy

b- Ignore (psychotic)

c- Recurrence of infectious mononucleosis

A 3 yr old had febrile convulsions 3 months ago now has fever and cough. What would
you do?

a- Give Phenobarbitone

b- Give cold water bath

c- Give aspirin

d- Observe because the chances of getting is 1 in 4

e- Paracetamol
Back to top

Joseph.

Guest

Posted: Fri Oct 26, 2007 2:43 pm Post subject:

--------------------------------------------------------------------------------

A 6yr old born locally comes with cough and weight loss. On exam there is
consolidation, Hilar nodes are enlarged. What is the next action?

a- Do purified protein derivative testing

b- Treat with ampicillin and repeat x-ray aippg.com

On cerebral palsy -

a- More common type in Australia is choreathetotic


b- Have feeding problem

c- Spastic type can be effectively treated with diazepam

d- Mental retardation is not seen

e- Associated with hypoxia in the perinatal period

WOF is true about sudden infant death syndrome?

a- More common in high socioeconomic group

b- Occurs in 6-9 months

c- Common in spring and summer

d- More common than accidents as a cause of death

A baby has swollen joints and cries whenever the napkin is changed, irritable,
echymoses and hyperplastic gums. What is the diagnosis?

a- Scurvy b- Rickets

c- Osteomyelitis

A child with weight gain and high proteinuria. What else would you find in the
investigations?

a- Decreased serum albumin

b- Increased serum globulins

c- Ascites

A mother comes with a 4 week old baby who had vomiting since birth. He is artificially
fed. Baby is feeding well and gaining weight but maternal grandma says that baby is not
well. On exam no abnormality detected. What is your advice?

a- Tell not to hear anybody’s advice


A mother with an 18 month old child complaining that the child is not hearing well. On
exam he turns his head tot the rattle’s noise. What will you do?

a- Do audiometry

b- Review in 6 months time

c- Find out the milestones

d- Reassure

An infant with congestive cardiac failure. How do you diagnose on expectation?

a- Respiratory rate more than 40

b- Pulse more than 120

c- Oedema

d- Enlarged liver

e- JVP

A 6 month old baby with convergent squint. What’s the management?

a- Correct immediately

b- Review in a month

c- Reassure the mother

An 8 yr old boy presents with frequent bouts of asthma interfering with school and
sleep despite full doses of oral Salbutamol and theophyline. The next treatment to try is -

a- Aminophylline

b- Na chromoglycate

c- Beclomethasone dipropionate aerosol


d- Oral Prednisolone

e- Amoxycillin

Back to top

Joseph.

Guest

Posted: Fri Oct 26, 2007 2:44 pm Post subject:

--------------------------------------------------------------------------------

A 3 yr old child is well apart from a cold of few hours duration when put to bed at 6 pm.
Her mother finds that she can not wake her at 7 am and notices that she is feverish and
had a purpuric rash over her baby. The next step in management is -

a- lumbar puncture and blood culture and wait for the result

b- Lumbar puncture, blood culture and treat with penicillin


c- Lumbar puncture, blood culture and treat with cephathrexine

d- Observe

e- Give her paracetamol and wait for more development

A distressed mother brings her infant to casualty with bruising around the mouth and a
broken arm. It’s a good practice to -

a- Avoid asking how the injuries occurred

b- Seek a care and protection order immediately

c- Arrange outpatient psychiatric review after setting the fracture in casualty

d- Admit to hospital ward

e- Examine CFS for evidence of intracranial bleeding

A male infant a birth weight of 2.4 kg. And product of a normal labour becomes
jaundiced at the second day. WOF would be the most likely cause of the jaundice?

a- Physiologic deficiency of glucoronyl transferase

b- Physiologic rapid destruction of excess red blood cells

c- Hereditary deficiency of liver enzymes

d- Iso immunization

e- Atresia of the bile ducts

A 4 yr old baby presents with a firm mobile non tender swelling of 3 cm diameter high in
the left anterior triangle of the neck. Three days earlier, he had an attack of acute
tonsillitis. WOF is most likely to cause of the swelling?

a- Branchial cyst

b- Tuberculosis abscess

c- Thyroglossal cyst
d- Cervical lymphadenitis

e- Dermoid cyst

Jaundice first noted at 16 hrs in a healthy newborn is most likely due to -

a- Physiological deficiency of glucoronyl transferase

b- Physiological rapid destruction of excess RBC

c- Hereditary deficiency of liver enzymes

d- Atresia of the bile ducts

e- iso- immunization

The earliest feature of intussusception is usually -

a- Abdominal distention

b- Bouts of screaming followed by periods of pallor and lethargy

c- An abdominal mass noted by mother

d- Blood and mucus in stools

e- Repeated vomiting aippg.com

WOF features is not consistent with the diagnosis of febrile convulsion?

a- Generalized grand mal seizure

b- Duration of fit 10 min or less

c- Fever not due to meningeal or cerebral infection

d- The convulsion occurs in an infant aged 3 months

e- No localizing neurological sequelae after the convulsion


Back to top

Joseph.

Guest

Posted: Fri Oct 26, 2007 2:45 pm Post subject:

--------------------------------------------------------------------------------

After his baby sister was born, a 6 yr old boy began sucking his thumb and wetting his
bed, behaviour he had grown out f long before. His is an instance of -

a- Reaction formation

b- Regression

c- Acting out

d- Jealously

e- Compensation
A 4 yr old boy presents with a history of sudden onset of difficulty in breathing for 2
hours. On exam he has a fever of 39 C is anxious and refuses to lie down on the
examination table. His respirations are noisy and the child is unable to swallow his saliva.
The most likely diagnosis is -

a- Acute tonsillitis

b- Aspirated foreign body

c- Acute epiglottitis

d- Diphtheria

e- mumps aippg.com

The commonest cause of Anorectal bleeding in children is -

a- Ulcerative colitis

b- Rectal polyp

c- Intussusception

d- Fissure in ano

e- Peptic ulcer

Speech learning disabilities in school children may be manifested by -

a- Writing reversals

b- Aggressive r depressed behaviour

c- Educational retardation

d- Abdominal pain

e- All of the above

Projectile vomiting in a 2-3 week old baby is caused by -


a- Duodenal atresia

b- Duodenal stenosis

c- Gastroenteritis

d- Pyloric stenosis

e- None of the above

A child with recurrent swelling in the groin on exam, no mass palpable but slight
thickness of spermatic cord -

a- Arrange for herniotomy

b- Exam for lymph node

c- Review after 1 week

d- Assure parents (do nothing)

A mother says that her 3 ½ yr old girl can only say 2-3 words and are not well
articulated. But she communicates by gestures using her hands. WOF is the cause?

a- Autism b- Deafness

c- Dysarthria d- Mental retardation

e- Manipulative behaviour

WOF is true about Perthes disease?

a- Recurrent limp in preschool child

b- Painless limp and x-ray reveals of definitive changes

Organ least likely affected by intrauterine growth retardation is -

a- Brain b- Kidney
c- Heart d- Lungs

A 9 month old infant with mild dehydration is managed by -

a- Oral rehydration and assess as necessary

b- Put into isolation ward and IVP

An 8 yr old girl presents with abdominal pain and bilateral ankle edema and swelling of
both hands. O/E, she has urticarial rash on buttocks and legs. WOF investigations would
you do to establish the diagnosis?

a- Micro urine examination

b- Immunoglobin level in serum

c- Chest x-ray

d- Antinuclear antibody

e- Antirheumatic antibody

Back to top

Joseph.

Guest
Posted: Fri Oct 26, 2007 2:46 pm Post subject:

--------------------------------------------------------------------------------

What is true in cow’s milk compared to mother’s milk?

a- High levels of phosphate

b- High levels of lactose

c- High levels of vitamins

d- Low in sodium

e- Low in calcium

A 2 yr old child is having a convulsion. Mother rings you as the convulsion is coming to
an end. She is asking you what to do. What would you advice?

a- Put the child in to a cold bath

b- Turn the child to a side aippg.com

c- Give aspirin 500mg

d- Give paracetamol 500mg

e- To put a spoon with a cloth round it into the mouth

A 5 ½ yr old child brought suffering for the past 6 months with history of recurrent
upper respiratory tract infection. What is the cause of this problem?

a- Asthma
b- He is picking up infection in the first yrs at school

c- Chronic tonsilar and adenoid enlargement

Regarding pertussis, which of the following statements are true?

a- Incubation period is 7-10 days

b- Triple vaccine cannot be given with another live virus vaccine

c- Pertussis vaccine should not be given to a child with neonatal convulsions

d- Erythromycin reduces the severity of signs and symptoms

The commonest cause of bleeding per rectum in an infant is due to -

a- anal fistula

b- Anal fissure

c- Meckel’s Diverticulosis

d- Intussusceptions

In a male child with Perthes disease, which of the following is/are right?

a- 10-14 yrs of age

b- Presents with a limp but rarely with pain

c- There is limitation of internal rotation and abduction

d- Late shortening possible

e- There is muscle wasting

Which of the following cause damage to the fetus?

a- Toxoplasmosis
b- Treponema pallidum

c- Measles

d- HIV infection

e- Cytomegalovirus

The commonest cause of meningitis in neonatal period is -

a- E.coli b- Strepticocci

c- Meningococcal pneumoniae

d- Mycoplasma e- Haemophylus influenza

Gastroenteritis is less common among breast fed baby than bottle fed baby. All of the
following constituent of milk confer immunity against gastroenteritis except -

a- IgA b- IgG

c- Macrophage d- Lactopherin

e- T lymphocytes

All are autosomal recessive except -

a- Neurofibromatosis

b- Phenylketonuria

c- Galactossemia

d- Cystic fibrosis

e- Sickle cell anemia


Back to top

R.dass

Guest

Posted: Fri Oct 26, 2007 2:48 pm Post subject:

--------------------------------------------------------------------------------

A 6 week old child presents with inclusion conjunctivitis since birth develops cough and
respiratory distress. CXR shows bilateral pulmonary infiltrates. Which of the following is
true regarding the child’s condition?

a- Chlamydia trachomatis can cause pneumonitis

b- Streptococcus pneumonia can cause pneumonitis

What is the most common complication of Mumps?

a- Meningoencephalitis

b- Encephalomyelitis

c- Orchitis
d- Myocarditis

e- Blocked parotid duct

A 4 yr old boy presents with a history of sudden onset of fever and difficulty in
breathing for 2 hrs, on examination he has a fever 39 C. is anxious and refuses to lie down
on the examining table. His respirations are noisty and the child is unable to swallow his
saliva. The most likely diagnosis is -

a- Acute tonsillitis

b- Aspirated foreign body

c- Acute epiglottitis

d- Diphtheria

e- Mumps aippg.com

The commonest cause of iron deficiency anemia in infant under 9 months is -

a- Bleeding from Meckel’s diverticulum

b- A diet mainly of milk

c- Folate deficiency

d- Recurrent bleeding from the nose

e- Hemolytic anaemia

A 4 yr old child is brought to you by the kindergarten teacher. She states that the child
was not well that morning, has vomited once and complained of central abdominal pain.
The child’s temperature is 39 C. Your clinical exam is most likely to reveal -

a- numerous pus cells on microscopy of urine

b- Inflamed ear drums

c- Neck stiffness
d- Inflamed tonsil

e- Tenderness localized to the right iliac fossa

An 8 yr old girl presents with abdominal protusion, anaemia and tenderness. On exam
there is a big irregular and mobile mass crossing the middle. What’s the diagnosis?

a- Polycystic kidney

b- Ovarian cyst

c- Neuroblastoma

d- Thalassemia major

e- Nephroblastoma

In bronchiolitis, which is true?

a- Peak incidence is between the age group of one to two yrs

b- Most commonly caused by Haemophilus influenza type B

c- in later age will progress to bronchial asthma

d- Most commonly happens at lat spring time

e- Para influenza is the most common organism

A 4 week old baby girl was brought to casualty with history of vomiting for two days.
She wet three nappies in the past 24 hrs. the last one an hour ago. Her body weight is 3.7
kg. Lab findings are serum Na 128 meq/L (normal 3.9-5.6 meq/L), CI 90 meq/L, HCO3 26,
pH 7.54, base excess+2. WOF fluids replacement will you give for the first 48 hrs?

a- 370 ml of N/2 saline + ammonium Chloride

b- 370 ml of N/2 saline + potassium Chloride

c- 760 ml of N/2 saline + ammonium Chloride

d- 760 ml of N/2 saline + potassium Chloride


e- 1100 ml of N/2 saline + potassium chloride

In intussusception all are true except -

a- More than 90% needs surgery

b- There is more than one causal relation with Henoch Schonlein purpura

c- There is more than one causal relation with adenovirus infection

d- There is more than one causal relation with blood in stool

e- Common under 2 yrs old

A child with cyanotic congenital heart disease presents with vomiting weakness of right
arm and leg. What’s your diagnosis?

a- Brain abscess b- Thrombosis (cerebral)

c- Embolism d- Rupture berry aneurysm

e- Meningitis

Back to top

R.dass

Guest
Posted: Fri Oct 26, 2007 2:49 pm Post subject:

--------------------------------------------------------------------------------

Foreign body inhalation in children, all are true except -

a- It can manifest with wheezing

b- It can presents with repeated cough

c- Peanut particularly is dangerous

d- Left lower lobe is commonly involved

e- Many cause bronchiactasis

A child presents with pain in the tibial tuberosity especially after strenuous exercise and
there is an area of hotness over the tibial tuberosity. What is your diagnosis?

a- Osteochondritis of tibial tuberosity

b- Perthes disease

c- Osteomyelitis

d- Slipped upper femoral epiphysis

e- Osteocarcoma Aippg.com

Regarding appendicitis in children, all of the followings are correct except -

a- Manifestation depends on the relationship between caecuma and appendix

b- Obturator and hip extension signs are seldom + in pelvic appendicitis


c- Abdominal tenderness is a very reliable sign

d- Anorexia and fever are common manifestation

e- Repeated vomiting is an early sign

A child who is in good health brought by his parents and they stated that the child has
an attack of complete loss of movement especially when he watches TV at night. The
attack lasts seconds and during this attack, his eyes are open and he can hear voices.
There is history of similar attack in one of his distant relatives. What’s your Diagnosis?

a- Sleep paralysis

b- Grand mal epilepsy

c- Myoclonic epilepsy

d- Temporal lobe epilepsy

e- Familial periodic paralysis

Gastroenteritis in children all are correct except -

a- Breast feeding should be discontinued

A child with acute lymphoblastic leukemia has been in hematological remission for 18
months. Vomiting has than occurred for about 10 days. The most likely cause is -

a- Recurrent infection

b- Drug toxicity

c- Central nervous system

d- Psychogenic

e- Gastroenteritis
An 8 yr old child presents with a temperature of 39 C. Physical examination reveals a
membranous tonsilar exudate, generalized lymphadenopathy, hepatosplenomegaly and a
faint macular rash. Which of the following is the most likely to be found?

a- Raised ASO titre

b- Atypical lymphocytes in peripheral blood film

c- Bone marrow shows lymphatic leukemia

d- Cytomegalic inclusion body in cells from urine

e- Positive Schick test

The commonest cause of bleeding per rectum in an infant is -

a- Meckel’s diverticulum

b- Fissure in ano

c- Hemorrhoids

d- Intussusception

e- rectal polyps

A 6 week old child presented with a history of persistent conjunctivitis since birth and
chest infection. On examination, there are rales heard bilaterally over the chest. Chest x-
ray revealed bilateral infiltrates of both lungs. Which is the most likely cause?

a- Chlamydia pneumonitis

b- Pneumocystis carinii pneumonia

c- Acute bronchiolitis of infancy (respiratory syncitial virus)

d- Streptococcal pneumonia

A 6 month old child presented with a history of recurrent chest infections (cough &
bronchitis) episodic gastroenteritis. Laboratory investigation revealed:
Decreased serum sodium - 120 mmol/l

Decreased serum potassium - 2.5 mmol/l

Decreased serum chloride - 64 mmol/l

pH - 7.4

Which of the following most likely diagnosis?

a- Salt losing nephritis (chronic renal disease)

b- Cystic fibrosis

c- Cow’s milk allergy

d- Congenital adrenal hyperplasia

Back to top

R.dass

Guest

Posted: Fri Oct 26, 2007 2:54 pm Post subject:


--------------------------------------------------------------------------------

28 yrs old mother give birth to a baby in countryside hospital. When you see the baby it
had all the characteristic features of Down’s syndrome. What is your next step?

a- Say that you are not sure of the diagnosis and the diagnosis is to be confirmed

b- Baby has Down’s syndrome and council the parents

c- Baby is probably having Down’s syndrome and baby should be checked for
chromosomal studies

d- Baby is probably having Down’s syndrome and baby and parents should be
checked for chromosomal studies

2-yrs-old baby has enlargement - Tanner stage 3. No axillary hair or pubic hair
development. Bone age corresponds to the chronological age. What is the probable
diagnosis?

a- Ovarian tumour

b- Precocious puberty

c- Congenital adrenal hyperplasia

d- Premature thelarche

28-yrs-old boy had a fall from tree 2 days ago. He is limping with tenderness and mild
swelling over upper end of the tibia and also has fever. What is the probable cause?

a- Septic arthritis

b- Irritable hip

c- Osteomyelitis

d- Osgood schelters disease


1-month-old baby has vomiting frequently during feeding. But he did not have any
weight loss. What will you do next?

a- Decrease the duration of feeding on each occasion

Breast feed baby can develop which of the following vitamin deficiency -

a- Vitamin E b- Vitamin C

c- Vitamin K d- Vitamin D

e- None of the above

Back to top

R.dass

Guest

Posted: Fri Oct 26, 2007 2:54 pm Post subject:


--------------------------------------------------------------------------------

8 months old infant was brought to emergency department with few hours’ history
of intermittent crying. He looks pale white crying. He also started vomiting. No
abnormality is found on abdominal exams. What is true?

a- gastro -enteritis

b- Intussusceptions should be considered

c- UTI

11yrs old child presents with short history of pain in the scrotum. On examination left
hemi-scrotum is hot, tender, and swollen. What is your management?

a- Arrange for surgical exploration

b- Observation

c- Blood culture

A six weeks old infant has 2 wks history of vomiting. He lost weight and has a palpable
pyloric tumour. What is most likely cause?

a- Intussusception

b- Gastro - enteritis

c- Urinary tract infection

d- Pyloric stenosis

e- Gastro - esophageal reflux

Cellulitis 3 cm around the eye -

a- CT scan of the head

aippg.com
20% second degree burn all are true except -

a- IV fluid

b- Blood transfusion may be needed.

c- Skin grafting

Child speaks 2-3 phrases and follows simple direction. Age of the child?

a- 3 yrs b- 2 yrs

c- 4 yrs d- 16 months

e- 1 yr

4 wks infant has persistent unilateral discharge. Microscopic exam shows


polymorphnuclear granulocytes. What is the most likely cause?

a- Congenital stenosis of lacrimal gland’s duct

b- Chlamydia trachomatis

c- Gonococcal infection

5-yrs-old child presented with vomiting in the morning and headache. Most likely cause
-

a- Supratentorial tumours

b- Brain stern glioma

c- Medulloblastoma

d- Migraine

3-yrs-old presented with unilateral nasal discharge, he is uncooperative. What is your


management?
a- Treatment with antibiotic

b- X-ray of the nose

c- General anaesthesia and x-ray

Which of the following is the most frequent cause of iron-deficiency anaemia in infancy?

a- Breast feeding

b- Milk formula

c- Unmodified cow’s milk introduced before 1 yrs of age

Back to top

R.dass

Guest

Posted: Fri Oct 26, 2007 2:56 pm Post subject:


--------------------------------------------------------------------------------

A 4 yrs old girl is found on medical examination to have murmur, which is not changing
with position. What is your management?

a- Observation

b- Reassuring the parents

c- It should be explained that she needs further investigation

Which one of the following statement concerning cerebral palsy is not correct?

a- Feeding difficulties are common

b- Cerebral pathology is permanent and non progressive

c- Disorder of movement and posture

d- Spastic cerebral palsy is the most common type

e- 90% have mental retardation

A child is presented with long history of unexplained productive cough and wheeze. He
is failing to thrive (signs of malobsorbtion). What is your initial investigation?

a- Stool exam

b- Chest x-ray

c- Sweat test

d- Blood exam

What is true about febrile convulsion?

a- It occurs between 6-8 yrs of age

b- Prognosis is not good aippg.com


c- Risk of developing epilepsy is 10%

d- Lasts about 15 min

Which of the following conditions usually resolves spontaneously?

a- Congenital heart block

b- Patent ductus in 15 yrs boy

c- Atrial septal defect

d- Small ventricular septal defect

In a child with Kernicterus which abnormality is found in the blood?

a- High unconjugated bilirubin concentration

A child who was well before developed malaise and now he has fever 40 degree C. 3
days ago he falls from tree. Mother noticed that he limped. His thigh is swollen there is an
area of pain and tenderness. What is the most likely cause?

a- Fracture

b- Thrombophlebitis

c- Ca of bone

d- Osteomyelitis

A child is presented with sudden onset of high fever. He is unable to drink or swallow.
He has stridor. Sever epiglottitis has been diagnosed. What is your management?

a- Antibiotics

b- Intubation may be required

c- Throat should be examined


Bronchopulmonary dysplasia, what is true -

a- Treatment cause anaemia

A mother is concerned that the child does not seem to hear. Clinical exam revealed no
abnormality. Child looks well. What is your management?

a- Reassurance of the parents

b- Investigation in 4 yr old

c- Tempanometry

d- Audiometry

Atopic eczema. What is not true?

a- Application of cream

b- Keep the bed room cool

c- Apply soap and water

Which of the following conditions may have a fatal out come?

a- Angry looking napkin rash

b- Blue spot on the batacks

c- Red spots on cheeks

d- Diamond shaped area of hair loss on the scalp

A 16 yrs old boy come to your clinic complaining of pain in the hip and knee. He is
limping. What is your working diagnosis?

a- Transient synovitis
b- Perthes disease

c- Slipped upper femoral epiphysis

Complication of meningitis -

a- Cephalohaematoma

Back to top

R.dass

Guest

Posted: Sat Oct 27, 2007 11:54 am Post subject:

--------------------------------------------------------------------------------
One day old baby born after difficult forceps delivery was noticed that she was unable
to move her left arm. O/E the arm was internally rotated, The wrist extended and Moro
reflux was -ve. What is the most likely diagnosis?

a- Fracture of the clavicle

b- Lesion of the branchial plexus

c- Fracture of the humerus

d- Erb’s palsy

A 4 yrs old boy suffers from constipation since infancy. He passes a large bowel motion
every 3 to 4 days. On P/R examination, the ampulla was large & hard faeces were felt.
What is the most likely cause?

a- Hirschsbrung’s disease

b- Hypothyroidism

c- Functional constipation

A 1 yr old boy had an upper resp. tract inf. There was a history of similar complain in his
family members. Late in the evening he developed a fever with a cough and became
moderately ill. O/E he was febrile and had generalized wheezing without basal crackles.
What is the most likely cause?

a- Bronchiolitis

b- Croup

c- Epiglottitis

d- Asthma

e- Pneumonia

A 6 yrs old boy during routine school medical examination was found to have a high
pansystolic murmur loudest at the left sternal border. He was not cyanosed. CXR showed
mild cardiomegaly and pulmonary plethora. What will be your advice to his mother?

a- That he has atrial septal defect

b- That he needs further investigations

c- That the murmur most probably will disappear when he grow older

d- That he has patent ductus arteriosus

e- That he has patent pulmonary stenosis

A 2 months old child has jaundice. He has been vomiting since the age of 2 weeks. The
mother said that her first child had the same problems and later he developed cataract.
What’s the most likely cause?

a- haemolytic disease

b- Physiological jaundice

c- Hep. B infection

d- Galactosemia

e- ABO incompatibility

A 1 yrs old child returned from overseas travel. He developed severe abdominal pain &
was crying & flexing his knees to his abdomen. On his way to the hospital he had a bowel
motion with bloody stool. On P/R examination there was some blood on the tip of your
finger. What’s your initial investigation?

a- Plain abdomen examination

b- Stool microscopy & culture

c- Air contrast enema

What is correct of the following matched modes of inheritance?


a- Haemophilia (factor VIII deficiency) by x-linked Recessive mode

b- Spherocytosis by autosomal dominant

c- Cystic fibrosis autosomal dominant

d- G6PD by autosomal dominant

A child has Hb A2. Which of the following is the most likely cause?

a- Spherocytosis

b- Sickle cell anaemia

c- Thalassemia B minor

Which of the following will be the best sign of cardiac failure in a 6 months old child? R
1988.

a- Increased

b- Cyanosis

c- Dysponea

d- Ankle oedema

e- Hepatomegaly

Which should be taken seriously when examining a child?

a- Developmental delay

b- Diamond shaped crusty rash on head

c- Area of baldness on the occipital region

d- bruise on the left cheek

e- Bluish spot on the buttocks


Back to top

R.dass

Guest

Posted: Sat Oct 27, 2007 11:55 am Post subject:

--------------------------------------------------------------------------------

What is true about jaundice?

a- Physiological jaundice appears during the first 24 hours

b- First noticed on the sclera

c- All jaundices children will be treated by exchange transfusion

d- All jaundices children will be treated by phototherapy

e- Most children don’t need phototherapy


An 8 yrs old child presented with multiple bruises and limbs. He also had fine Petechial
rash on his legs. O/E he is afebrile & otherwise well. What is the most likely cause?

a- Has to be reported to child health unit

b- He may develop arthralgia & macrohaematuria in the future

c- He has Henoch-Schonlein purpura

d- Do a whole body bone scan

Which is the most common cause of blood discharge on nappies?

a- Cystic fibrosis

b- Anal fissure

c- Anal polyps

d- Rectal prolapse

e- Haemorrhoids

All of the following can be diagnosed by fresh stool examination except -

a- Ascariasis b- Giardia lambia

c- Hookworm d- Taenia saginata

e- Enteroblasis

10 weeks old baby was brought by his mother because he still did not develop any head
control and his mother said that he has no social mile. On history taking, the mother tells
you that he was born at 31 weeks of gestation. His birth weight was 1.8 kg. He was
discharged from hospital 6 weeks later. When you examined him, you find that he is
otherwise normal. What will you tell the mother?

a- He is developmentally delayed

b- He is normal
The most common cause of iron deficiency in Australia is -

a- Bleeding from Meckel’s Diverticulum

b- Folate deficiency

c- Recurrent bleeding

d- Fibrocystic disease of the pancreas

e- Dietary deficiency

An eight yrs old boy presents with an acute attack of wheezing. What’s is the most
likely cause?

a- Inhaled FB b- Asthma

c- Cystic fibrosis d- Bronchoectasis

e- Anxiety

Atopic Eczema in a child all are true except -

a- It is very itchy

b- It is more common in flexures

c- It is a contraindication to triple antigen immunization

d- There is F/H of allergenic & hay fever

Obesity in children is usually associated with -

a- Increased height b- Gushing syndrome

c- D.M. d- Hypothyroidism

e- Sleep apnoea
A 2 yrs old child wakes up at night with severe scrotal pain. What is your initial action?

a- Call a surgeon

b- Send for isolate scan a matter of surgery

c- U/S as matter of urgency

d- Reassure him

Back to top

R.dass

Guest

Posted: Sat Oct 27, 2007 11:55 am Post subject:

--------------------------------------------------------------------------------

Regarding the development of 5 ½ yrs old child, all are true except -
a- can name 5 colors

b- Can draw a picture of a parent

c- Can write his name & Address correctly

d- Can use scissors with good control

e- Can draw a diamond shape correctly

A 13 or 16 yrs old girl was watching T.V. and suddenly became breathless & sweaty.
What is your diagnosis?

a- Salicylate poisoning

b- Hysterical

c- Some physical complain

d- Anxiety attack

e- Asthma

Cystic fibrosis can cause all the following except -

a- Iron deficiency

b- Diarrhoea

c- Intermittent constipation

d- Chest infection

A child was doing quite well at school for his first year then his performance started to
deteriorate over that last two years. What is the most likely cause?

a- Strabismus b- Dyslexia

c- High tone deafness d- Epilepsy


A father brought his son to consult you about his son’s left mump orchitis. He did
not have mumps in his childhood. Which of the following is true?

a- The father is likely to be immune because he have had mumps whilst young without
noticing

b- The father should be vaccinated

c- The son’s right testis will be involved

d- He should be given corticosteroids to minimize the risk of infertility

e- The father should be given immunoglobulin

A 1 yrs old child is vomiting and has been losing weight for 2 months. Which of the
following is true?

a- He should be given a trial of gluten free diet

b- A flat Jejunal mucosa means a life long gluten free diet

c- Cystic fibrosis is the cause

Regarding cerebral palsy, which of the following is true -

a- I is a non progressive condition

b- Athetold movements are usually present

c- All will develop mental retardation

d- All will become spastic

e- ? Spastic

Regarding otitis media, which of the following is true?

a- Consult a specialist if there is any hearing problem 2 months after a course of


treatment

b- Staph/ strep/ pseudomonas are the cause


c- Usually associated with inflammation of Eustachian tube

d- Usually associated with Perforation of Tympanic membrane

Regarding mumps meningitis -

a- Rare

b- Most benign meningitis

c- Causes sterility

All of the following may be treated by inducing vomiting with syrup of ipecac except -

a- Salicylate poisoning

b- Accidental drug ingestion

c- Caustic soda ingestion

Back to top

R.dass

Guest
Posted: Sat Oct 27, 2007 11:56 am Post subject:

--------------------------------------------------------------------------------

A child was stung with a bee. He collapsed and became hypotensive. What will be your
initial action?

a- I/V antihistamine

b- I/V hydrocortisone

c- I/V adrenaline + hydrocortisone

A child was swallowed caustic sod 4 hrs ago, what will be your initial action?

a- Ipecac to induce vomiting

b- Gastric lavage

c- Charcoal

d- Dilute withy milk

All of the following are risk factors for congenital deafness, except -

a- Family history of deafness

b- Congenital rubella

c- Malformations of the middle abd inner ear

d- Mother treatment with phenytoin

e- External canal atresia


Common cause of iron deficiency in infants in Australia -

a- Meckel’s diverticulum

b- worm infestation

c- Chronic blood loss

d- Milk formula diet]

WOF is helpful sign of congestive heart failure?

a- Enlarged liver

b- Increased jugular venous pressure

c- Splenomegaly

d- Oedema

e- Dyspnea

WOF will suggest developmental delay?

a- Smile for 1st three months

b- Walks unsupported at 8 months

c- Sits unsupported at 8 months

d- Speaks 2-3 words at 18 months

e- Bedwetting at night at 1st five years

WOF is true about mumps meningitis?

a- One of the most serious other than viral

b- Usually occurs after parotid enlargement

c- One of the mildest forms of meningitis


d- Always associated with convulsion

Commonest cause of blood stained stool in a 8 yr old child -

a- Anal fissure

b- Hemorrhoids

c- Meckel’s diverticulum

d- Crohn’s disease

e- Diarrhoea

Commonest cause of death in a healthy 3 yr old in Australia?

a- Sudden infant death syndrome

b- Leukemia

c- Poisoning

d- Drowning

e- Respiratory infection

Guest, Oct 27, 2007

#63

Guest

Guest Guest

Posted: Sat Oct 27, 2007 12:28 pm Post subject:

--------------------------------------------------------------------------------
Joseph.

Guest

Posted: Thu Oct 25, 2007 3:39 pm Post subject:

--------------------------------------------------------------------------------

In paracetamol poisoning, the cause of morbidity is-

a- Hepatic injury

b- Metabolic acidosis

c- Metabolic alkalosis

d- Hypoglycemia (cause of mortality)

In tricyclic anti depressant poisoning WOF are true except:

a- HCO3 to prevent arrhythmia

b- Flumazenil should not be given if is due to TCA + Diazepam

c- Bile acid

d- Magnesium
e- Diazepam to

Regarding bronchial asthma all are true except:

a- Total lung volume is decreased aippg.com

b- Increase in residual volume

c- Decreased tidal volume

d- Decreased vital capacity

WOF are true regarding Asthma except:

a- It increases Br. Sensitivity

b- It does not occur after 50 yrs of age

c- Inspiratory crackles

A pt. with one sided dullness with shifting of trachea on the same side, Diminished
breath sound and respiratory movement. Which of the following is true?

a- Pleural effusion with collapse

A 23 yrs old lady with distended abdomen, glositis, muscle wasting, anaemia,
macrocytosis, diarrhoea no history of surgery. WOF is true:

a- Gluten sensitive enterocolitis

b- Duodenal biopsy is reqd. for confirm the diagnosis

c- Crohn’s disease

d- Ulcerative colitis

e- Pernicious anaemia
An abattoir worker came with H/O fever, myalgia and dark urine. The most likely cause
is

a- Leptospirosis

b- Brucellosis

Regarding eradication of H. Pylori which is true

a- it helps in healing of the ulcer

b- Prevents relapse rate

c- Need for long time therapy

Which of the following is not true regarding transferrin?

a- Increased in iron deficiency anaemia

b- Increase in chronic infection

c- Decrease in nephritic syndrome

d- Increased in pregnancy

Pt. with severe chest pain, absent carotid pulse, pain radiating to the back. What would
you find in chest X-ray?

a- Widening of the mediastinum

In thyrotoxicosis all of the following are present except

a- Limb girdle muscle weakness

b- Small muscle wasting

c- Atrial fibrillation
Regarding Myasthenia gravis which is true

a- Some people does not have antibody

b- Thymectomy is only done when thymoma is present

c- Reflexes are diminished after repeated stimulation

d- EMG always slow

e- Electomyograph

Guest, Oct 27, 2007

#64

R.dass

R.dass Guest

Most common presentation in carcinoma of rectum -

a- Bleeding per rectum

b- Intestinal obstruction

c- Anaemia

d- Constipation

e- Diarrhoea

In an unconscious patient all of the following are true except -

a- On recovery, there is no complications

b- may relieve airway obstruction on extension of neck

c- Should not be transported on left lateral position

d- if airway is opened, he might region consciousness


e- May be transported supine

Paget’s disease of the least all are true except -

a- Associated with Intraductal carcinoma

b- Carcinomatous cells encroaching epidermis

c- Always diagnosed by careful physical examination

d- Can be diagnosed

How will you diagnose strangulated bowel in an intestinal obstruction?

a- Vomiting

b- Diffuse abdominal pain

c- Absolute constipation

d- Multiple fluid levels

e- Pain becoming constant

Serum Gastrin in increased -

a- Post surgery

b- Pernicious anaemia

c- Omeprazol therapy

d- Chronic renal failure

e- Zollinger Ellison syndrome

Which of the following can be applied correctly regarding Echinococcus granulose and
hydatid cyst? aippg.com

a- Calcification of hydatid cyst implies inactive of parasite


b- the disease may develop after eating infected sheep meat

c- Pulmonary cyst respond to an 8 week cover Metronidazole

d- If the cyst is multiloculated, ultrasound can be visually diagnostic

e- Surgery involves removing germinal epithelium, laminated layer, and cystic fluid

classical haemophilia -

a- 1:10,000

b- Muscle haematoma is most common orthopaedic problem

c-

d- before surgery factor VIII should be given

e- Cryoprecipitate will not transmit Hepatitis B

X-ray of abdomen showing megacolon with fluid levels. What is the most likely
diagnosis?

a- carcinoma of sigmoid colon

b- Carcinoma of rectum

c- Ulcerative colitis

d- Foetal impaction

e- Retroperitoneal inflammation

Barret’s oesophagus -

a- Congenital

b- becomes malignant

c- Cells are columnar

d- Due to recurrent reflux


e- irreversible

A 65 yrs old man collapses in the toilet one night, he rouses himself and returns . Which
of the following might reasonably explain his syncope?

a- Gastrointestinal haemorrhage

b- Bladder neck outflow obstruction

c- Myocardial infarction

d- transient ischaemic attack

e- Aortic stenosis

R.dass, Oct 27, 2007

#65

R.dass

R.dass Guest

Testicular carcinoma -

a- Causes haematuria

b- Metastasize to inguinal nodes

c- Hydrocele frequency present

d- Can be cured when metastasized

e- Presents with pain

Photograph 11 yrs old man with cystic translucent lesion on the inside of lower lip with
history of trauma of 2 weeks duration -

a- Basal cell carcinoma


b- Squamous cell carcinoma

c- Syphilitic lesion

d- Mucoid cyst

Photograph of a big mass at the pre auricular area extending down the margin of the
mandibles view behind of a 65 year old female with swelling in the parotid area 10 Yrs
duration. Most likely diagnosis -

a- Adeno carcinoma of parotid gland

b- Wartheius tumor

c- Stone in Submandibular parotid duct

d- Pleomorphic adenoma

e- Submandibular tumor

Photograph of ultrasound with more than 3 large stones in gall bladder. What is your
management?

a- Cholecystectomy

b- Endoscopic retrograde choledocho pancreatography

c- Chenodoxycholic acid

d- Lithotripsy chenodeoxycholic acid

e- Papillectomy through ERCP

Photograph of front right chest with mastectomy scar, indented skin lesions, radiation
marking. History of a 65 yrs old female who underwent modified radical mastectomy for
cruiser of the breast 2 years ago. Now presents with the above picture. What is the
management?

a- Tamoxifen
b- Testosterone

c- Radiotherapist

d- Removal of pectoralis major

e- Skin grafting

Lateral view of abdomen of an adult showing tortuous venous vessels running from
groin to upper chest region. What is the most likely cause?

a- Venacaval obstruction

b- Axillofemoral artery anastomosis

c- Aorto caval fistula

d- Portal hypertension

e- External iliac artery thrombosis

Two photograph of elderly man who presented with a mass at the anterior part of the
neck. One picture showed he is standing with his hands on the sides. The second picture
showed with both hands raised above the shoulder and redness of head and neck. There
is a symptom of getting dizzy upon raising the hands above the shoulder. What is the most
likely cause?

a- Venous obstruction

b- Vascular steel syndrome

c- Tumor extending to retrosternal area

d- Medullary carcinoma of thyroid with local endocrine effect

e- Sympathetic ganglion pressing on first rib

Photograph of an index finger with a swelling just proximal to nail bed margin 6 months
ago. The most likely diagnosis is -
a- Rheumatoid arthritis with swan neck deformity

b- Gouty tophi

c- Chronic paronychia

d- Osteoarthritis

e- Synovial cyst/cystic lesion

Photograph of skin of the back with multiple brownish dark warty, pedunculated
lesions. The most likely diagnosis is -

a- Seborrhoeic keratosis

b- Solar keratosis

c- Pigmented basal cell carcinoma

d- Hutchinson’s melanotic freckle

e- Squamous basal carcinoma

Photograph of left hand showing the medial aspect of a semiflexed 4th & 5th finger with
atrophy of hypothenar eminence -

a- Dupuytren’s contracture

b- Ulnar nerve paralysis

c- Volkman’s contracture

d- Radial nerve paralysis

e- Congenital malformation

R.dass, Oct 27, 2007

#66

R.dass
R.dass Guest

The Dermoid cyst consists of -

a- Ectoderm

b- Endoderm

c- Three layers

d- Skin sebaceous gland

e- Ectoderm and endoderm

Which of the following is most likely to be damaged during internal carotid


endarterectomy?

a- Hypoglossal nerve

b- Facial

c- vagus

d- trigeminal

e- Superior laryngeal

After removing submandibulars lymph node, which nerve is most likely to be damaged -

a- Motor part of trigeminal nerve

b- Motor part of facial nerve

c- Lingual nerve

d- Hypoglossa

e- Glossopharyngeal nerve
A woman presented with sudden onset of abdominal pain, vomiting for 8 hours. On
examination irregular pulse, abdominal tenderness, absent bowel sounds, bloody
discharge from rectum -

a- Ulcerative colitis

b- Colon cancer

c- Diverticulitis

d- Mesenteric artery thrombosis

e- Chrohn’s disease aippg.com

A woman on warferin for a long period of time (because of DVT) is going to have surgery
(Cholecystectomy)

a- Stop taking warferin

b- Stop taking warferin, replace with heparin and continue for 1 week after operation

c- take aspirin

d- Continue warferin

e- Stop warferin 2 weeks prior to surgery and continue afterwards

A 60 yrs old man has an ulcer on the sole of his feet. He finds difficulty in walking
particularly in dark places. Next step in management -

a- B 12

b- Venography

c- Arteriogram

d- Blood glucose

e- Doppler

The blood flow in vertebral artery is reversed. Your diagnosis -


a- Block of proximal part of subclavian artery

b- Block of distal part of subclavian artery

c- Block of internal carotid artery

d- Block of vertebral artery

e- Block entire carotid artery

Renal cell carcinoma - route of spread:

a- through renal artery

b- Through renal vein

c- Lymphatic spread

d- Local spread

e- Transcoelmic

Caustive organism of cellulitis -

a- Strep group A

b- Strep group B

c- Haemolytic gr

d- Straphyloureus

e- Clostridium welchii

Daily requirement of potassium minerals -

a- 40-60 mmol

b- 150-Mmol

c- 20-Mmol
d- 10-mmol

e- 200-Mmol

R.dass, Oct 27, 2007

#67

Joseph.

Joseph. Guest

All of the following might be associated with appendicitis except -

a- Thread worm

b- Bacteroides

c- Streptococcus fecolis

d- E coli

e- Diarrhoea

53 yrs old man with right sided blindness-

a- Left sided hemiparesis

b- Middle cerestral artery occlusion

c- Internal carotid artery occlusion

d- External carotid occlusion

e- Vertebral artery occlusion

Anal fissure -

a- Either side of the posterior midline


b- Skin tag should be there

c- Kind of ulcer

d- Always leads to a fistula

e- Sinus

Otosclerosis -

a- Normal tympanic membrane

b- Dysfunction of Eustachian tube

c- Ear discharge

d- Is common finding in minor degree n general population

e- Calcification of the bones of the inner ear

Profuse vomiting is characteristic of -

a- Proximal small bowel obstruction

b- Distal small bowel obstruction

c- Distal Large bowel obstruction

d- Proximal large bowel obstruction

e- Pyloric stenosis aippg.com

Male with blood pressure 90/60, pulse 120/min, pain radiating to back. What is the
diagnosis?

a- Acute pancreatitis

b- Leaking aneurysm

c- Perforation of Duodenum

d- Acute cholecystitis
e- Rupture of peptic ulcer

X-ray of the wrist fracture. What is true?

a- A vascular necrosis of distal portion of scaphoid

b- Plaster below elbow for 6/52

c- Plaster with addition of thumb for 6/52

d- Plaster above elbow to distal IPJ

e- Internal fixation

f- Crepe bandage

g- Cast applied to include the wrist and to include DIJ of the thumb

X-ray of Collis fracture of an old woman -

a- Internal fixation and plaster cast for 10 weeks

b- External fixation

c- Fixation immobilisation

d- Check for median nerve damage and review later

e- Plaster cast for 10 weeks

f- Tight crepe bandage

g- Closed reduction and plaster cast is adequate

Picture of an enlarged, red and painful testes f a 10 yrs old bo. What is your
management?

a- Reassure

b- Surgical exploration of the scrotum

c- Urine analysis
d- Give analgesic and send him home

e- Send him to out-patient urology

Joseph., Oct 27, 2007

#68

Joseph.

Joseph. Guest

Swan neck. What is true?

a- usually associated with ulnar deviation

b- Central ulceration of the tendon of the intermediate interphalangeal joint

c- Subluxation of the wrist joint

d- Rupture of the extensor tendons of the hands

e- Distal pnalangeal joints affected

Picture -

Nodule on the palm of the hand the Dx is -

a- Pyogenic granuloma

b- Squamous cell carcinoma

c- Basal cell carcinoma

d- Gangleon

e- Hemangioma

Lump in the breast of a 45 yrs old female, mammogram showed a large white round
lesion. Ultrasound showed a large cystic lesion, what is your diagnosis -
a- Lymphoma

b- Fibrocystic disease

c- Cancer of the breast

d- Fat necrosis

e- Fibroadenoma

Management of a woman with a lump in the breast -

a- Needle aspiration

b- FNA biopsy

c- Excision

d- Radiotherapy

e- Radical mastectomy aippg.com

A girl with partial thickness burn -

a- No pain

b- Liquid rarely is third degree except when there is long contact

c- Scarring is common

d- May need IV fluid replacement

e- May need blood transfusion

Pseudo cyst of pancreas -

a- Self limiting

b- Always follows chronic pancreatitis

c- Covers with endothelium


d- Always needs surgical treatment

e- Common in alcoholic pancreatitis

A man with chronic renal disease suddenly deteriorates in renal function. What is the
best diagnosis to include lower urinary tract obstruction?

a- US

b- IVP

c- Retrograde pyelography

d- X-ray abdomen

e- CT scan

Chronic alcoholic man C/O abdominal pain passes fat with faeces. Next step of
management -

a- Pl x-ray f abdomen

b- Ix exocrine function of pancreas

c- ERCP

d- US

e- Stools examination

Femoral hernia -

a- More common in woman than inguinal hernia

b- Can be easily reduced

c- Needs surgical treatment

d- Never gets strangulated

e- Occurs above the midpoint of the inguinal ligament


X-ray of lung with dark lung fields, fluid level both sides.

A 22 yrs old man with a history of a vehicle accident 2 hours prior to presentation.
Following are the possible findings.

a- Hemopneumo thorax

b- Rupture of diaphyaym

c- Surgical emphysema

d- Lung collapse

e- Pneumothorax

f- Pericardial effusion

Joseph., Oct 27, 2007

#69

Joseph.

Joseph. Guest

A man’s back with large necrotizing cellulitis. What are the predisposing factors of
the disease?

a- Leprosy

b- Diabetes mellitus

c- Lymphoma

d- Gout

e- Melanoma

Management of a man with large necrotizing back -


a- High dose steroids

b- Antibiotics

c- Debridement

d- Cytotoxic drugs

e- Tetanus toxide

Wound healing -

a- Primary intension means leave the wound open and let it heal by epithelalization

b- Secondary healing means approximation the edges of the wound by suturing

c- Surgical technique and suture materials are important

d- Impaired in a jaundiced patient

e- Is poor in an obese patient

aippg.com

Which of the following can true ERCP?

a- Rupture of the gall bladder

b- Stone in the common bile duct

c- Cancer of the gall bladder

d- Recurrent jaundice

e- Septicaemia

Barium meal -

X-ray of large intestine with diverticulitis. Which of the following are associated
complications of diverticulitis?

a- Toxic megadon (colon)

b- Pneumaturia
c- Malena

d- Rectal fistula

e- Perforation

f- Intestinal obstruction

g- Colo-vaginal fistula

In MRI which of the following appear black -

a- Air

b- Fat

c- Metal

d- Moving Blood

e- CSF on TI sequence

A boy presents with severe scrotal pain. Picture showed a red scrotum. (DX probably is
testicular torsion). What is your management?

a- Surgical exploration of the scrotum

b- Reassure parents

c- Urinalysis

d- Give analgesic and send him home

e- Send him to outpatient urology

All of the following are associated with appendicitis except -

a- Threadworm

b- Bacteroides

c- Streptococcus fecalis
d- E.coli

e- Staphylococcus aureus

Supraspinatus Tendinitis -

a- Tenderness of the humeral head

b- Tenderness on the spine of the scapula

c- Pain on initiation of abduction

d- Pain present in a small arc of movement or abduction

e- Inflamed and swollen shoulder

Dequervain tenosynovitis -

a- Extensor pollicis brevis and abductor pollicis longus

b- Pain on movement of the thumb

c- Inflammation and swelling noted

d- Median N involvement

e- Associated with myxedema

Joseph., Oct 27, 2007

#70

Joseph.

Joseph. Guest

Picture of necrotizing cellulitis. The following may contribute to this -

a- DM
b- Leprosy

c- Lymphoma

d- Gout

e- Melanoma

What is the most characteristic feature of fat embolism?

a- Petechia usually occur in the upper body

b- Never involve the CNS

c- Petechia usually involve the lower extremities

d- Droiozinen

e- Dyspnea and tacchypnoea

A 20 yrs old boy with which type of urine sample obviously blood stained. O/E other
thing is normal. What will be the first step in his management?

a- Microscopical examination of fresh sample of urine

b- Arrange for x-ray and IVP

c- Give him buscopan and send him home

d- Give him Pethidine and review one hour later

e- Admit him to the hospital

A 66 yrs old lady after an air trip presents with pain in the calf muscles and swollen on
leg. What is the most likely diagnosis?

a- DVT

b- Rupture of Bayer’s cyst

aippg.com
What is the most common finding in a history of carpal tunnel syndrome?

a- Parasthesia in the distribution of median nerve

b- Consistently awaken her at night

c- Pain of distribution of ulnar nerve

d- Weakness of muscles of the hand

A 45 yrs old male presents with pain and bleeding after defecation. What is the most
likely diagnosis?

a- Internal carcinoma

b- Anal fissure

c- Carcinoma of large gut

d- Granuloma

e- Perianal abscess

f- External haemorrhoids

A 30 yrs old man his father and brother died of cancer of colon. What is the best way to
diagnose early cancer of his colon?

a- PR & sigmoidoscopy

b- Occult blood & sigmoidoscopy

c- Colonoscopy

d- Occult blood in stools

e- Ba enema

In which type of thyroid malignancy family history is important -

a- Papillary
b- Follicular

c- Medullary

d- Anaplastic

e- Lymphosarcoma

What is the most serious complication of an intra-epiphyseal fracture in a long bone in


an adolescent?

a- Malunion

b- Non-union

c- Increased spurt of growth in long bone

d- Premature cessation of growth

e- Stiffness

What is the commonest complication of Colles fracture?

a- Malunion

b- Non-union

c- Reflex sympathetic dystrophy

d- Carpal tunnel syndrome

e- Compartment syndrome

Joseph., Oct 27, 2007

#71

Joseph.

Joseph. Guest
A young man comes with bilateral weakness and wasting of thenar, hypothenar and
interossei muscles of hand. What is the most possible cause?

a- Bilateral ulnar nerve palsies

b- Bilateral median nerve palsies

c- Syringomyelia

d- Multiple sclerosis

e- Brain stern infarction

What is the percentage of detection of Duke stage Al colonic carcinoma after screening
with faecal blood test?

a- <3%

b- 15%

c- 25%

d- 50%

e- 75%

A 70 yrs old woman presented with fever, constipation and left sided lower abdominal
pain of 2 days duration. What is the diagnosis?

a- Diverticulitis

b- Ulcerative colitis

c- Crohn’s disease

d- Meckel’s diverticulitis

e- Cancer of the colon

Which of the following is the most common cause of small bowel obstruction on
Australia?
a- External hernia

b- Adhesions

c- Malignancy aippg.com

Patient with hypertension, diagnosed to have localized renal artery stenosis. What is the
management?

a- Angioplasty

b- Arterial reimplanation

c- Treatment of hypertension

d- Hemodialysis

e- None of the above

In Otosclerosis -

a- Tympanic membrane is green colour

b- Tympanic membrane is retracted

c- Tympanic membrane is red in colour

d- Eustacian tube is abnormal

e- Tympanic membrane is usually normal occasionally pink or orange

Patient having TIA and diagnosed to have internal carotid artery stenosis of >80%.
Which of the following is true?

a- If you don’t surgery for >30 days, monthly is 70%

b- Treat with aspirin

c- Within 30 days he will have a stroke

d- Endarterectomy will decrease the chance of stroke by 2 times


e- No treatment necessary since the patient is likely to have a fatal stroke anyway

What is the best post operative pain management?

a- Pethidine is better than morphine

b- Pethidine is best given by calculating mg/kg body weight

c- Pethidine should not be repeated within 4 hours

d- Pethidine has a longer duration than morphine (duration of action is 2-3 hrs)

e- Morphine and Pethidine o not cause respiratory depression

What is the most accurate method for diagnosis of acute cholystitis?

a- Oral cholecystogram

b- IV cholecystogram

c- Ultrasound

d- CT scan

e- ERCP

Joseph., Oct 27, 2007

#72

Joseph.

Joseph. Guest

All will cause intermittent claudication except -

a- Pain in the thigh, buttock, legs

b- Sudden shooting pain posterior of the tights of the legs


c- Cramps in the calf muscle

d- Sensation of paralysis

Scleritis, LN total bilirubin high, direct bilirubin normal, liver enzymes increased alk
phosphatase normal, OGGT increased atypical lymphocytes -

a- Hepatitis

b- Gilbert’s disease (or other disease of live)

c- Hepatitis C

d- Infectious mononucleosis

e- Cholecystitis

2 yrs old boy with a slight abrasion on the foot, not taken immunisation before -

a- Tetanus immunoglobulin

b- TT + TI

c- DPT

35 yrs old woman nullipara, has fibroid uterus, Hb 60 mg (N 115-160) keen to conceive,
on exam uterus boggy to the level of umbilicus -

a- Therapy of anaemia followed by myomectomy

b- Suppression of the fibroid by GNHR analogue for 3 months

c- Hysterectomy aippg.com

d- Hysterectomy + oophorectomy

Traveler’s diarrhoea, returned from Bali most likely -

a- Giardia lambliasis
b- Cholera

c- E. coli (Entamoeba hystolitica)

d- Salmonella

e- Rotavirus

Intermittent claudication pain in buttock, thigh, legs. What artery is?

a- Aorto-iliac

b- Femoro-popliteal

8 yrs old infant pain in the HIP joint, most commonly -

a- Transient synovitis

b- Perthes disease

c- Septic arthritis

Infectious endocarditis, is most common caused by -

a- Staphylococcus aureus

b- Streptococcus viridans

c- Streptococcus fecalis

Lady with blood discharge from the nipple, not lactating mostly will be -

a- Intraductal Ca

b- Breast abscess

c- Intraductal papiloma
Facial nerve palsy is associated with all f the following except -

a- Parotid carcinoma

b- Parotid

c- Mastoiditis

Joseph., Oct 27, 2007

#73

Joseph.

Joseph. Guest

Sudden onset of severe vertigo, collapse, no signs of deafness (vestibular signs) -

a- Vestibular neuronitis (vomiting + nausea)

b- Meniere disease

c- Occasional positional vertigo

Which of the following cancers mostly give bone metastasis?

a- Breast Ca b- Stomach

c- Prostate

Most common cause of death in Australia after extensive burns is -

a- Irreversible blood shock

b- Nutritional imbalance

c- Spreading sepsis

d- Severe chest bleeding and respiratory failure


e- Severe UTI - renal failure

Otosclerosis is associated with a -

a- Red tympanic membrane

b- Blue sclera

c- Normal tympanic membrane

d- Not functioning Eustachian tube

All are side effect of corticosteroids -

a- Lymphopenia

b- Osteomalacia aippg.com

c- Increased white cell count

ECG diagram ……………. With ST segment depressed -

a- LBBB b- RBBB

c- Left ventricular hypertrophy

d- W.P.W. sy (accelerated AV conduction)

e- Inferior infarction

Baby 18 months breast feed only, no supplementary food. What deficiently will be
associated -

a- Vitamin A b- Vitamin B

c- Vitamin C d- Vitamin D

e- Vitamin K
8 yrs old boy 5% dehydrated hypokalemia. What is preferred solution in this case?

a- 0.9 Hartmann

b- Ringer La

c- 0.45% saline + glucose

d- 5% dextrose

Parents complain that their child has funny turns, last week not good, he had coryzal
illness. Suddenly his arms get extended and parents say that child not respond like before-

a- Infantile spasm

b- Febrile convulsion

c- Herpes encephalitis

What is good prognosis of schizophrenia that is respond to therapy?

a- Lack of precipitating factors

b- Presence of treasure & affected disorder

c- Schizoid person

d- Auditory hallucinations

Joseph., Oct 27, 2007

#74

Joseph.

Joseph. Guest

A patient taking Serotonin (SSRI) he took accidentally serotonin agonist. Which should
be taken concomitantly to avoid Serotonin syndrome?
a- L - tryptophan b- haloperidol

c- Sertraline d- Citralopam

e- Olanzapine

A patient came with a severe chest pain. ECG was normal. What is correct?

a- Sent him home

b- Heparin IV, nitrates are contraindicated

c- Exercise test to be done immediately

d- ECG should not be repeated unless pain once more

A 22yrs old man, snake bite 2 hours before. No symptoms unless bite marks on dorsum
of his foot. What is appropriate next step in treatment?

a- Give antivenom

b- Incision between marks, leave 5 min. to blood

c- Observe 24 Hours

d- Tourniquet + incision let blood + antivenom

e- Sent him home

A patient with respiratory distress, bronchial carcinoma was diagnosed he has bone
pain…………..

a- Polypeptide PTH like secreted

Severe excruciating abdominal pain. The least likely is -

a- Aortic aneurism

b- Acute appendicitis
c- Ruptured peptic ulcer

d- Renal colic

e- Pancreatitis

Abdominal pain, minimal abdominal rigidity, lumbar pack pain + shock -

a- Leaking aortic aneurism

b- Retrocecal appendicitis aippg.com

c- Lumbar prolapse

Lacunar infarction -

a- Middle cerebral artery /Embolus

b- Cerebral Ultrasonography

Undescended testis in 8 yrs old boy what would you tell his mother as further
information?

a- Orchidopexy reduce incidence of spermatogenesis eff.

b- Most associated with inguinal hernia

c- Reassurance

d- more incidence of malignant tumor

Infant was previously well, respiratory distress, unilateral wheezing, no history of atopy
-

a- Measure 02

b- Expiratory and inspiratory Chest x-ray


A Baby with a history of 3 month recurrent right eye infection, previously respond well
to two days therapy with antibiotics now eye infects with a sticky discharge -

a- Antibiotics should be at least 10 days

b- Gonococcal infection

c- Chlamydial infection

d- Blocked tear duct (Obstruction of lacrimal duct)

Joseph., Oct 27, 2007

#75

Joseph.

Joseph. Guest

Greenish vaginal discharge, all is correct except-

a- Trichomonas vaginalis

b- Giardenella

c- Cervical carcinoma

d- Chlamydia

e- Foreign body in vagina

Patient taking phenytoin wants oral contraceptive pills. What is most appropriate?

a- Microgynomn 30

b- Microgynom 50

Progesterone 14-15 days of the cycle, as a part of Combined OCP is responsible for all of
the following except -
a- Reduced endometrial hyperplasia

b- Premenstrual symptoms (PMS)

c- Increased HDL compared with oestrogen only pills

d- Artificial menstrual blood flow at the end of the course

Syntometrin (Ergometrin) is given for all of the following except -

a- Contraction of the myoepithelial breast duct

b- Physical contraction of uterus in late pregnancy

c- Increase body volume because of its antidiuretic effects

d- Increase breast milk production aippg.com

e- Is responsible for uterine contraction during labour

A female patients 32 weeks pregnant was involved in car accident as a front seat
passenger. On exam. She has a few abdominal bruises due to seat belt pressure, slight
vaginal bleeding. All is appropriate next step except -

a- Observation

b- CTG

c- Induction of labour for possible placental abrubtio development

A bisexual man with one month history of diarrhoea has mouth ulcer, arthralgia, joint
pain - hands as well rash on palmar / solar -

a- Reiter’s disease

b- Behcet’s disease

c- AIDS
A patient has low Na, low k, low sodium, HCO3 reduces plasma osmolarity 186 (N 250)
… urine osmolarity 825 (50 - 1200) -

a- Diabetes insipidus

b- Diabetes Mellitus

c- SIDHA

d- Na depletion

e- Water intoxication

Picture of abdominal plaques and scales, salmoncolour -

a- Eczema b- Pityriasis rosea

c- Psoriasis

A boy school age, felt on outstretched hand. X-ray of ulna and radius fracture, no other
findings. What is treatment?

a- full plaster cast till the wrist

b- Plaster cast till the MCP joints

c- Full plaster cast till distal MCP joints

Spiral fracture of humerous what is common associated?

a- Lesion of n. ulnaris

b- Radialis c- Medianus

d- Axillaries e- n. accesorius

Joseph., Oct 27, 2007

#76
Joseph.

Joseph. Guest

A child with photophobia, red eye (subconjuctival injection) pupil responding well to the
light, vision 6/12 -

a- Glaucoma b- III Nerve palsy

b- Blepharitis d- Conjunctivitis

Brief episodes of monocular vision is -

a- cerebral artery occlusion

b- internal carotid artery aneurism

c- Insilateral carotid artery stenosis

d- Glaucoma

Retinal neovascularisation -

a- Retinal venous thrombosis

b- Glaucoma

c- Malignant melanoma

d- Hypertension

e- Diabetes mellitus

Female standing on one limb -

a- Shortening of one limb

b- Reduced wear bearing aippg.com

c- Proprioceptive in balance
Dip-stick Ca increase in blood dehydration, constipation, Ca stones recurrent what to do
to prevent recurrence?

a- Allopurinolol

b- Spironolactone

c- Steroid

d- Decrease Ca +2 diet

Progesterone n pregnancy-

a- Dilatation of the uterus

b- Dilatation of the ureter and calyces

c- Reduce incidence of infection

A female patient complains of fullness, fat indigestion after fatty meals. US shows
stones in gall bladder asepsis. After surgery no stone left free of symptoms fro a few
weeks than symptoms reccure. What is most likely explanation?

a- Pancreatitis after cholecystitis

b- Stone was left in common bile duct

c- It is a result of removal of the GB (fat indigestion)

d- GB stones were not cause of her symptoms

e- Recurrence of stones

Picture leg a male patient red, oedema, inflamed with pustules diabetic patient -

a- Amoxycillin + vancomycin

b- Fluoxacillin, penicillin
c- Penicillin + vancomycin

d- Vancomycin, Metronidazole, amoxycillin

What is considered as absolute contraindication for OC pills?

a- Smoking

b- Diabetes mellitus

c- Varicose veins

d- Superficial trombophlebitis

e- Migraine with visual aura

Picture, red infection of skin, abd/arm, followed by infection of the chest?

a- TBC

b- Streptococcus

c- Pneumococci

d- Mycobacterium avium

e- Legionela pneumonis

Joseph., Oct 27, 2007

#77

Gina.

Gina. Guest

1. A 14 years old boy,has regular severe breathing problems that often prevent him
from cycling to school. He was diagnosed with asthma some years ago, but treatment with
inhalation corticosteroids and beta-2-mimetics do not seem to help. His pediatrician finds
no allergies, normal lung function, no bronchial hyperactivity and no decreased

lung function is observed during an ergometry test. These complaints and

findings are most characteristic of:

A. Cystic fibrosis

B. Severe asthma

C. Hyperventilation

D. Exercise-induced asthma

2. After one week of bed rest following a nasty ankle sprain, a 64-year-old female has
developed painful discolouration of the leg. You decide to start a home treatment with an
anticoagulant.Your preferred drug in this case would be?

A. Oral aspirin

B. Intravenous heparin

C. Low-molecular heparins sc.

D. Coumarin

3. Jane’s parents are trying to make her stop from biting her fingernails. They
applied a substance with a offensive taste on her nails. Now,when she tries to bite her
nails, the taste makes her stop immediately. This is an example of:

A. Flooding

B. Aversive conditioning

C. Desensitization

D.Repression

E. Rejection
4.Jane is also undergoing therapy for her fear of heights. As part of the therapy, she
does relaxation exercises in which she imagines to stand on a chair, than on a ladderand
finally on top of a roof. Which behavioural therapy is her therapist applying?

A. Systematic desensitization

B. Negative reinforcement

C. Positive reinforcement

D. Flooding

E. Familiarization

5. Freud’s psychoanalysis distinguishes itself from other types of psychotherapy


because ...?

A. The patient becomes familiar with his/her present feelings

B. The patient gains insight into problems he/she was not aware of before

C. Problem behaviour is replaced by wanted behaviour

D. The patient adapts a new way of thinking.

Gina., Oct 28, 2007

#78

dr.sahar

dr.sahar Guest

1. A 14 years old boy,has regular severe breathing problems that often prevent him
from cycling to school. He was diagnosed with asthma some years ago, but treatment with
inhalation corticosteroids and beta-2-mimetics do not seem to help. His pediatrician finds
no allergies, normal lung function, no bronchial hyperactivity and no decreased

lung function is observed during an ergometry test. These complaints and


findings are most characteristic of:

A. Cystic fibrosis

B. Severe asthma

C. Hyperventilation

D. Exercise-induced asthma*****

2. After one week of bed rest following a nasty ankle sprain, a 64-year-old female has
developed painful discolouration of the leg. You decide to start a home treatment with an
anticoagulant.Your preferred drug in this case would be?

A. Oral aspirin

B. Intravenous heparin

C. Low-molecular heparins sc.*****

D. Coumarin

3. Jane’s parents are trying to make her stop from biting her fingernails. They
applied a substance with a offensive taste on her nails. Now,when she tries to bite her
nails, the taste makes her stop immediately. This is an example of:

A. Flooding

B. Aversive conditioning****

C. Desensitization

D.Repression

E. Rejection

4.Jane is also undergoing therapy for her fear of heights. As part of the therapy, she
does relaxation exercises in which she imagines to stand on a chair, than on a ladderand
finally on top of a roof. Which behavioural therapy is her therapist applying?
A. Systematic desensitization******

B. Negative reinforcement

C. Positive reinforcement

D. Flooding

E. Familiarization

5. Freud’s psychoanalysis distinguishes itself from other types of psychotherapy


because ...?

A. The patient becomes familiar with his/her present feelings

B. The patient gains insight into problems he/she was not aware of before******

C. Problem behaviour is replaced by wanted behaviour

D. The patient adapts a new way of thinking.

Freud didn't exactly invent the idea of the conscious versus unconscious mind, but he
certainly was responsible for making it popular. The conscious mind is what you are aware
of at any particular moment, your present perceptions, memories, thoughts, fantasies,
feelings, what have you. Working closely with the conscious mind is what Freud called the
preconscious, what we might today call "available memory:" anything that can easily be
made conscious, the memories you are not at the moment thinking about but can readily
bring to mind. Now no-one has a problem with these two layers of mind. But Freud
suggested that these are the smallest parts!

The largest part by far is the unconscious. It includes all the things that are not easily
available to awareness, including many things that have their origins there, such as our
drives or instincts, and things that are put there because we can't bear to look at them,
such as the memories and emotions associated with trauma.
dr.sahar, Oct 28, 2007

#79

dr.sahar

dr.sahar Guest

A businessman with h/o Depression, had 2 major depre

A businessman with history of Depression, had 2 major depressive episodes, one after
the death of his son, and the second, after his company went bankrupt (2 yrs ago). He has
2 months until retirement and plans a trip to overseas. Pt feels well. A CXR shows
inoperable cancer. What should the physician do?

a. Withhold the information because it is fatal

b. Withhold the information because of his psychiatric diagnostic

c. Withhold the information

d. Tell only the wife who can provide supportive care

e. Tell the truth to the patient

dr.sahar, Oct 28, 2007

#80

Joseph.

Joseph. Guest

23 yrs old girl has abdominal colic, 2 weeks back had viral infection, puerperal rash,
most probable diagnosis is -

a- ITP b- HSP
75 yrs old woman presented with anemia, lethargy most common -

a- Acute lymph leukemia

b- Chronic lymph leukemia

c- Acute myeloid leuk.

Which of the following diseases has poorest prognosis?

a- Hodgkin lymphoma

b- Metastatic choriocarcinoma

c- Metastatic stomach carcinoma

d- Metastatic bronchial carcinoma

A female patient has sore throat, (sore red tongue) MCV 118(75-110N) HB 50(110-150
mg/dl)-

a- haemolytic anaemia

b- Fe deficiency anaemia

c- Pernicious anaemia

d- Infective mononucleosis

e- Thalassemia minor’

What differentiate between haemolytic and FE deficiency anaemia?

a- Polycythemia

b- Normal platelet count

c- Polychromasia

d- Increased heptoglobin

e- Increased reticulocytes
Prodromal symptoms with differentiate SCH from dementia?

a- Ideas reference

b- Level of consciousness

Aboriginals have reduced risk of the following disease when comparing with Australia?

a- Depression b- Anxiety

c- Bipolar aff. Disorder d- Self harm

e- Drug and alcohol abuse

Haemochromatosis diagnosed by -

a- Ferritin level increasing

b- Transferin saturity decreased

Parents with adopted daughter suffering from SCH, ask you what is risk of having their
physiological daughter with SCH?

a- Nil b- < 2%

c- 10% d- 10 - 40%

e- > 50%

You are carrying a medical check for patient with a gastroesophageal reflux. You found
increased oesophageal acidity. Patient taking antacids, smoking 20 cig. /day, drinking 2
cups of wine/day. On Endoscopy multiple ulcers - oesophagus. You should prescribe him -

a- H 2 receptor antagonist

b- antacid
c- Antacid + antibiotic

d- Proton pump inhibitor

e- Fundoplication

Joseph., Oct 31, 2007

#81

Joseph.

Joseph. Guest

Adult man has recurrent UT infection, stricture, urethra, after long and difficult
dilatation manouvre he is in shock -

a- Perforation of ureter

b- Gram negative bacteremia

An 18 yrs old boy, has yellow sclerae, can’t move eye bilaterally outwards, left eye
upward movement presented. Previous he had positive serology for syphilis on many
occasions -

a- Neurosyphilis

Accident, blood from the urethral meatus, fracture of symph. Pubis. What is next?

a- Urethrogram b- IVP

c- US d- Urethral catheter

Adult females, tingling, numbness (at tip fingers) symptoms increased at night, wakes
her up. During the day she can’t carry her shopping bag. Wasting the muscles on thenar
eminence-
a- Carpal tunnel Sy

b- Rheumatoid arthritis

c- Osteoarthritis

Jaundice, lethargy, morning stiffness in the morning, reduced during the day -

a- Rheumatoid arthritis

35 yrs old female present with unilateral exophtalmus mostly -

a- Nasal duct enlargement

b- Lacrimal gland inflame

c- Increased ICT

d- Thyrotoxicosis

e- Antral Ca

65 yrs old patient, tremor only when hold up cup of tea in front of T.V. Tremor as at rest
while asleep. On examination minor increase in muscle tone, no cogwheel rigidity -

a- Levodopa b- Anti - cholinergic

c- Apomorphine s.c.

A baby after 6 hours prolonged labour, “jittery” (shakes) measure -

a- Blood glucose b- Protein

c- O2 d- Lumbar puncture for meningitis

Multiple pregnancies, second baby breech presentation the first baby born with a cord
around neck. All is correct except -
a- Check for vaginal bleeding

b- CTG for foetal heart rate

c- Check the twin position

d- External version and delivery

e- Oxytocin if vertex presentation no distress w. uterine contraction not started

FE deficiency anaemia common all of except -

a- Multiple pregnancies

b- Thalassemia minor

Joseph., Oct 31, 2007

#82

Joseph.

Joseph. Guest

A infant 18 month old bubbling, not speaking, respond to noise. Mild missed child
hearing screening w. was at 9 months. What is next step?

a- Arrange for another child hearing test

b- Audiometrical assessment

c- Hearing test

A lady with a history of Postmaturity of the previous baby was delivered at 41 wks, was
4.400 gms With shoulder Dystocia. Now she is 32 wks pregnant should consider all of the
following except -

a- Induction of labour at 37 wks


b- Mid forceps vaginal delivery, unless vertex presentation, B. representing at the level
of ischial spine

c- Follow up DM at 20 and 30 weeks

d- Follow up baby at 34 wks measure size by US

A baby was normal intra-uterine, normal CTG until last ½ hour of the labour >>>
stillbirth. Labour progress was god no maternal distress. Which of the following test is
least likely to establish cause of death?

a- Coomb’s test

b- Anti RH AB

c- Anticardiolipin

d- Dead foetal parts

e- Autopsy

A nullipara 17 (or 13 weeks now) 2 previous abortion at 8 weeks. Most common cause is

a- Infection

b- Failure of progression of luteal phase

c- Chromosomal abnormality

d- Cervical incompetence

Medical student, started medical course, 3 months back his fellow students tell him he
has yellow sclerae. Bilirubin is high -

a- Hepatitis b- haemolytic disorder

c- Gilbert’s disease

55 yrs old man headache for 2 wks, unilateral, ESR 80, no other abnormality -
a- Aspirin b- Prednisone 15 mg

c- Prednisone 75 mg d- Ergotamine tartarate

e- Warferin

Picture hand, four fingers / deformity / contracture -

a- Rheumatoid arthritis

b- Late osteoarthritis

c- Rupture of extensor tendons

d- Ulnar radiation

A child 9 yrs old previously well has high temp. 39 C, pain, swelling of the knee,
tenderness, inflammation more localized at the upper part of the tibia. Most likely -

a- Osteomyelitis b- Rheumatic fever

c- Septic arthritis d- Fracture humerus

Undescendent testis 1 year old boy -

a- Indirect inguinal hernia (90%)

b- Direct inguinal hernia

c- Localised inguinal superficial pouch

d- Chromosomal

e- More risk of Seminoma/ teratoma

Woman, with abdominal distension, acne, hirsutism, ovarian cyst 5 cm, LH: FSH
increased -

a- Teratoma b- Fibroid
c- Mucocystinous cyst of ovarium

d- Polycystic ovarian Sy. (PCO)

Joseph., Oct 31, 2007

#83

Joseph.

Joseph. Guest

A baby infant, OPD given O2 not responding -

a- Broncho dilatator b- Increase dose of O2

c- Intubate and ventilate

What is more realted to pulmonary hypertension and Cor pulmonale?

a- Bronchogenic carcinoma

b- Pulmonary embolism

c- Emphysema

d- Chronic chest infection

Pleurisy reduced Ca acidosis -

a- Related o rheumatic arthritis

aippg.com

Which of the following disease has normal anion gap acidosis?

a- Addison disease b- Tissue hyperperfussion

c- Renal failure d- Malnutrition


What would be considered normal in newborn?

a- male with a enlarged breast

b- Female with a enlarged clitoris

Which f the following is matched?

a- Dermatomyositis - increased creatinine kinase

b- Ankylosing spondylitis

Mother got a baby with a mongolism disease (Down Sy). She wants to know what are
other risk findings from GP others than cardiac anomalies?

a- Severe mental impairment

b- Cystic fibrosis (except)

c- Multiple myeloma

d- Hypothyroidism

Tension pneumothorax what is initial step in management?

a- Wide bore needle into 2 intercostal spaces

Parents ask about their son phenylketonuria. What is the risk for in next pregnancy?

a- 1 : 4 b- 1 : 2

Joseph., Oct 31, 2007

#84

Joseph.
Joseph. Guest

. A boy came to complain about his boyfriend problem in sexual relationship, dressed in
girl clothes -

a- Transsexualism b- Transvestitism

Catheterization for aortic stenosis, pressure gradient > 55 mg. Hg. What is most
significant to find out?

a- Left ventricular ejection fraction

b- End diastolic volume

c- Pressure of left atrium

d- CV pressure

aippg.com

Infant with a subdural haematoma, and x-ray. Multiple healing sited, what is probable
Diagnosis?

a- Non accidental injury

b- Bone fracture

c- Haemolytic disorder

d- Skull fracture

Defence mechanisms are divided into mature, immature and neurotic. What is
considered as immature?

a- Suppression (mature)

b- Denial (denial)

c- Introjection

d- Projection (narcistic)
e- Sublimation (mature)

Women complains about indigestion, abdominal discomfort, lost 3kg from the last 1
year, anaemis -

a- Cystic fibrosis b- Chron’s disease

c- Ulcerative colitis

A baby 3 months old with a subcapular, strawberry haemangioma, tell parents -

a- Reassure b- Excision

c- Lasser coagulation d- Observation

Otoscelerosis what you will get -

a- Normal tympanic membrane

b- Usually ruptured tympanic membrane

c- Non functioning Eustachian tube

A 23 yrs old man returns from Bali and he got watery diarrhoea (no blood). On physical
examination, he is normal. What is the commonest organism?

a- Giardia b- Rota virus

c- Toxicogenic E. coli d- Cholera

A 2 yrs old child came to you with his mother with history of a mild contaminated would
in his scalp. He has no previous history of immunization. What is your best management?

a- T. Immunoglobulin b- T. Toxoid
c- DPT d- Antibiotic

What is the most common cause of iron deficiency anaemia in children, except -

a- Premature baby b- Twin baby

c- Prolonged feeding of unmodified cows milk

d- Coeliec disease

e- Thalassemia minor

Joseph., Oct 31, 2007

#85

Joseph.

Joseph. Guest

A 7 weeks old child was brought to you by his mother. His mother complains that he is
vomiting intermittently since birth. O/E you get no abnormality except some wt lose.
What is your probable diagnosis?

a- UTI b- Pyloric stenosis

c- Phenylketonuria d- DI

e- GE reflux

Phenylketonuria, which one is except -

a- One in two regarding in sex

b- One in four regarding in sex

c- Female will be affected and male will be carrier

d- male will be affected and female will be carrier


aippg.com

A picture looks like Kaposi’s sarcoma what is the most common organism?

a- Pneumocystic carini b- Streptococci pneumonia

c- Mycoplasma pneumonia d- Tuberculous Bovis

An x-ray of the # of the shaft of the radius and ulna. What is your management?

a- Plaster above the elbow to the proximal MCP joint

A mother came to you with her 15 months old child, complaining of he does not babble
as before and doesn’t say a word. At 8 months of age the child passed the hearing test
and he was born at 34 wks O/E he looks towards the sound and nothing abnormal. What
is your management?

a- Reassure the mother it is developmental variant

b- Do Audiometry

c- The child is suffering from congenital deafness

Anti-D antigen -

a- Should not be given if the child is B+ve

b- It is an active immunity

c- Decreases auto hemolysis

d- Useful if in 1st 24 hours

e- It given in 2nd trimester (decreases of auto immunisal)

01.08.2007

A child came to you with fever, O/E you find neck rigidity, protein was 1.2, sugar was
normal and lymphocyte was 50X10. What is your probable diagnosis?

a- T.B. b- Echo virus


c- Bacterial meningitis d- Typhoid

What carcinoma doesn’t metastasise into the brain?

a- Lung b- Liver

c- Kidney d- Prostate

e- Testis

Which carcinoma usually metastasis to the pelvic bone?

a- Stomach b- Skin

c- Breast d- Lung

e- Colon

Toothpaste like discharge from one nipple and the nipple s inverted. What is your Dg?

a- Ductal carcinoma

b- Intraductal carcinoma

c- Duct ectasia

d- Breast abscess

e- Hyper Prolactinemia

Joseph., Oct 31, 2007

#86

Joseph.

Joseph. Guest
A man came with dyspnoea, O/P the lower lobe of the Rt. Lung was dull and found
bronchial breath sound over that area. What is your diagnosis?

a- Rt. Plural effusion b- Lt. Pleural effusion

c- Rt. Sided consolidation d- Lt. Sided consolidation

e- Haemopneumothorax

The usual treatment of secondary dysmenorrhoea is except -

a- OCP b- NSAID

c- Bromocriptin d- Danazole

You are a medical officer after RTA a patient came to the emergency dyspnoea and
pneumothorax. What is your immediate treatment?

a- Chest pain

b- A wide bore needle in the 2nd intercostal space in the mid clavicular line

c- Antibiotic

d- ECG.

e- Intubation & 02 inhalation

A patient after motor accident with pelvic #. Five days after successful resuscitation he
develops serve dyspnoea. What is your initial management?

a- O2 inhalation

b- I/V heparin

c- Intubation & ventilation

d- Intensive antibiotic therapy


A patient came to you with sweating & agitated, probably opioid toxicity. What is your
Rx?

a- Naltrexone b- Naloxone

c- Diazepam d- Diuretics

A patient with COPD came in the emergency department with severe dyspnoea 28% O2
8L given by mask. After one hour you did arterial blood gas and you found that PO2 level
was 42mm of Hg, PCO2 level was 68mm of Hg. What you will do now?

a- Increase O2 inhalation by mask

b- Ask the pt. For intubation and ventilation

c- Increase I/V fluid

d- Repeat (Again check) arterial blood gas level after one hour

aippg.com

COPD pt. with severe dyspnoea. You are an emergency medical how you understand
that pt. is improving.

a- After measuring the arterial blood gas level and PO2 is increasing and PCO2 is
decreasing

b- According to that patient vocal responds in painful stimuli

Undescended testis related with -

a- Torsion of the testis

b- Spermatogenesis will be perfect if the operation done before 3 yrs.

c- Developmental delay of the child

d- Acute Epididymitis

e- Inguinal hernia
One ECG tracing, what is your diagnosis -

a- WPW syndrome b- Inferior MI

c- Complete heart block d- Normal tracing

e- Anterior MI

Erythema nodosum you will get in all condition except -

a- Sarcoidosis b- Bacterial myocardial infection

c- T.B. d- Yersinia

e- Rheumatic fever

Joseph., Oct 31, 2007

#87

Joseph.

Joseph. Guest

A chest X-ray of a child consolidation in the left side. What is your probable diagnosis?

a- Streptococci pneumonia

b- Mycoplasma pneumonia

c- Tuberculous pneumonia

d- Aspergillus’s pneumonia

e- Viral pneumonia

How do you differentiate ovarian cyst from ascites?

a- By shifting dullness
b- On percussion centrally dull and tympanic in the periphery

c- On percussion centrally tympanic and dull in the periphery

aippg.com

A 22 yrs old woman came to you with secondary amenorrhoea. O/E you found LH: FSH
ratio increase, what is your diagnosis?

a- PCO b- Primary ovarian failure

b- Non functioning Ut. D- Endometriosis

e- Valvo-vaginitis

Microganon-30, how does it work?

a- In the hypothalamus b- In the pituitary gland

c- In the ovary d- In the ut.

e- Both ovary and ut.

A 32 yrs old lady came to you at 8 wks of pregnant. She has a history of abortion, one in
8 wks another one in 10 wks. What is the most common cause of abortion of this lady is ?

a- Cervical incompetence

b- Chromosomal abnormality

c- Valvo vaginitis

d- Ovarian tumor

Acute severe excruciating abdominal pain, which is least likely -

a- Acute MI b- Rupture of aortic aneurysm

c- Acute appendicitis d- Renal colic

e- Mesenteric artery occlusion


A 22 yrs old lady secondary dysmenorrhoea, all of the following are true except -

a- Aspirin b- Danazole

c- OCP d- Bromocriptin

e- Acetaminophen

Wernicke’s encephalopathy, what is the appropriate treatment -

a- I/V thiamine and followed by glucose

b- Oral glucose only

c- I/V glucose

A 4 days old full term baby well at birth. Suddenly collapsed at cot. You are a medical
officer and called by the ward nurse to see the baby. O/E you found that the baby is
peripherally cyanosed, no pulse and respiratory distress. What is your diagnosis?

a- Pulmonary hypertension

b- Fallot’s tetralogy

c- Congenital heart disease

d- Lt. Heart hypoplasia

After splenomegaly you will get all of the following except -

a- Persistent anaemia

b- Normal R.B.C. life span

Contact dermatitis all of the following are true except -

a- mainly plant causes


b- Systemic steroid works well

c- Topical steroid doesn’t work

Joseph., Oct 31, 2007

#88

Joseph.

Joseph. Guest

Neuropathy ulcer in the metatarsal of the foot. What is the most common cause of this
type of ulcer?

a- Diabetic neuropathy

b- Ischaemic ulcer

All of the following causes facial nerve palsy except -

a- Chr. Parotitis b- Ca of the parotid gland

c- # of the base of the skull d- Mastoiditis

e- # of the mandible

After peritoneal fluid aspiration, you get the malignant cell in the fluid. Which of the
following is the most reliable sign in this condition?

a- Palpable lymph node in P/R examination

b- Supraclavicular lymph node palpable

Ventricular tachycardia WOF is the successful resuscitation.

a- Pupillary reaction to the light


b- ECG

Panic attack all are true except -

a- Precipitating factor should be avoided

b- 20% occur in general population

c- Females are more common then males

d- Presided by sweating & hyperventilation

aippg.com

One primigravida lady height is 150 cm. At 38 wks she is in labour, on P/V examination
you think that pelvis is apparently reduced but cervix is fully effaced and 4cm dilated. Baby
is in longitudinal position and head is in 0 station. What is your management?

a- Trial of labour

b- X-ray pelvimetry

c- Start I/V oxytocin

d- Immediate C/S

e- Wait and see

Which one most related with colonic carcinoma?

a- Melanosis coli

b- Ulcerative colitis

c- Colon irradiation

d- Diverticulosis

e- Adenomatous polyp

A picture of big goiter. What is your diagnosis?


a- Multinodular goiter

b- Grave disease

c- Diffuse goiter

d- Thyroid carcinoma

A child can babble, laugh when he looks in the mirror, can roll over and can sit without
support with an arch back. He can also walk with support. Which one is the consistent
with his developmental age?

a- 6 yrs b- 3 months

c- 7 months d- 9 months

e- 12 months

A prime lady at 38 wks present to you with 1000ml of blood loss and lower abdominal
pain. Which one of the following is not favour of abruption placenta?

a- Fetal dead b- Uterus tense & distended

c- Fetal head is high up d- BP is 180/110

e- Normal pulse rate

Febrile convulsion, WOF is correct -

a- It occurs at least 15 minutes

b- Most common in 6 months to 4 yrs of age

c- It may occurs epilepsy in later life

d- It’s most common in male

Joseph., Oct 31, 2007


#89

Joseph.

Joseph. Guest

All of the following can cause severe hypokalaemia except -

a- Pyloric stenosis

b- Chr. Renal failure

c- Crohn’s disease (primary hyperaldosteronism)

Which of the following is the normal phenomenon at birth?

a- Fusion of labia

b- Enlarged clitoris

c- Engorgement of breast bud of male child

d- Enlarge liver more then 5 cm and palpable

e- Cleft plate

A child having diarrhoea and vomiting, which of the following could develop convulsion?

a- Serum Na more then 165 mmol/L

b- Serum Na less then 132mmol/L

c- Serum K more then 7.5mmol/L

d- Serum K less then 2.2mmol/L

Bed redden patient what is the most common cause of diarrhoea?

a- Diverticulitis

b- Fecal impactation
c- Ischaemic colitis

d- Ulcerative colitis

X-ray of tibia fibula # and a 3 cm pieces of bone piers the wound. What is your
management?

a- T.T. followed by antibiotic

b- Debridement is only indicated when the wound is severely contaminated

c- Closed reduction

d- Antibiotic and I/V fluid

Diabetic ulcer what is your appropriate Rx?

a- Amoxicillin + penicillin

b- Ampicillin + Gentamycin + Metronidazol

c- Flucloxacillin + Gentamycin

d- Ampicillin + cloxacillin

Regarding post operative pain management which of the following is true?

a- Pathedine works better then morphine

b- Always consider reduce the dose in older patient

c- Pathedine must be given strictly in wt/kg basis

On x-ray you found that pituitary tumour is greater then 3 cm in the optic chiasma and
prolactin level is 2200mU/I. What is your initial Rx?

a- Danazole

b- Bromocriptine
c- Transfrontal approach surgery

d- Transphenoidal approach surgery

Bleeding from the nipple. What is the most common cause?

a- Intraductal papiloma

b- Ductal carcinoma

c- Paget’s disease

d- Mastitis

A child can see the computer clearly in the school but he can’t see the blackboard
clearly. What is your diagnosis?

a- Myopia

b- Hypermetropia

c- Normal phenomenon

Joseph., Oct 31, 2007

#90

Joseph.

Joseph. Guest

A child came to you with a minor clean contaminated wound but he doesn’t have
any history of previous immunization. What is your management?

a- T. immunoglobulin

b- T.T.

c- D.P.T.
d- Just clean the wound

e- Local antibiotic

Painful red eye but pupillary reflex is intact. What is your diagnosis?

a- Iritis

b- Conjunctivitis

c- Keratitis

d- Glaucoma

Unilateral exophthalmos, what is the cause?

a- Maxillary antral tumour

b- Thyrotoxicosis (Grave’s disease)

c- Simple goiter

Buttock pain, what is the cause?

a- Aorta-iliac A occlusion

b- Femoral A occlusion

c- Popliteal A occlusion

After difficult catheterization suddenly decrease Bp. What is your diagnosis?

a- Internal haemorrhage

b- Septcaemia aippg.com

c- Pylonephritis
After prolonged labour the baby is jittering. What is your diagnosis?

a- Hypoglycaemia

b- Hyperglycaemia

c- Hyper bilirubanimia

d- Hyperkalemia

What is the absolute contraindication of OCP?

a- Focal migraine

b- Varicose vein

c- Smoking

d- Mild hypertension

Twin pregnancy after one delivery you will do all in 2nd delivery except?

a- External cephalic version

b- Check the fetal lie

c- Check the bleeding

d- Check the feta heart sound

e- Check the vaginal laceration

Bee bite what is your management?

a- I/M adrenalin

b- I/V hydrocortisone

c- I/V fluid

d- I/V antibiotic
If you breast feed the baby which vitamin will be deficiency?

a- Vitamin A b- Vitamin K

c- Vitamin D d- Vitamin B12

e- Vitamin E

Joseph., Oct 31, 2007

#91

Joseph.

Joseph. Guest

An obese 8 yrs old boy came to you. O/E his wt. Was 48kg, which is more then 98th
percentile and the height was 140 cm, which is more then 90th percentile, otherwise
normal. WOF is correct in relation to this boy?

a- Advanced bone age

b- Increase blood sugar level

c- Primary hyperaldosteronism

d- Thyroitoxicosis

e- Hypothyroidism

All are true in case of intermittent claudication except -

a- Pain in the thigh buttock & legs

b- Sudden shooting pain post. Of the thigh to the leg

c- Cramping pain in the calf muscle

d- Pain with exercise


e- Sensation of paralysis

A patient came to you with scleritis, enlarge cervical LN. O/E direct bilirubin was normal,
liver enzymes increase but Alk PO4 was normal. GGT also increase what is your diagnosis?

a- Hepatitis A

b- Hepatitis C

c- Gilbert’s syndrome

d- Infectious mononucleosis

e- Cholecystitis

aippg.com

A 35 yrs old lady nullipara, wishes to conceive but she has fibroid ut., bleeding 60mg, Hb
level is 90g/l, O/E ut. is boggy to the level to the umbilicus. What is your management?

a- Correct the anaemia then myomectomy

b- Suppression of the fibroid by GnRH analogue for 3 months then myomectomy

c- Hysterectomy

d- Hysterectomy and Oophorectomy

e- Observe 6 months then myomectomy

3 yrs old child pain in the hip, most common cause is -

a- Transient synovitis

b- Perthes disease

c- Septic arthritis

d- Oesteomyelitis

e- Osgood schlatter disease


Infective endocarditis, what is the most common organism-

a- Staphylococcus aureus

b- Streptococci viridans

c- Streptococci fecalis

d- Coxiella

e- Viruses

Sudden and severe vertigo, collapse, no sign of deafness. What is your diagnosis?

a- Vestibular neuritis

b- Menier’s disease

c- Occasional positional vertigo

d- Migraine

What is the most common cause of death after extensive burn in Australia?

a- Irreversible shock

b- Nutritional imbalance

c- Spreading sepsis

d- Severe chest bleeding with respiratory failure

e- Severe renal failure

What is the side effect corticosteroid except -

a- Lymphopenia

b- Osteomalasia

c- Increase white cell count


d- Growth retardation

A patient came to you with severe chest pain ECG done but it is normal. What is your
management?

a- Thrombolytic therapy should not given if ECG normal

b- Discharge the patient for going home

c- I/V heparin and nitrates are contraindicated

d- Exercise test is to be done immediately

e- ECG should not be repeat unless pain once again

Joseph., Oct 31, 2007

#92

Joseph.

Joseph. Guest

A 22 yrs old man came to you complaining of snake bite 2 hours before. O/E there is
abrasion in the leg otherwise normal. What is your management?

a- Give antivenom

b- Observe for 24 hours

c- Give an incision and lets bleed for 5 minutes

d- Clean the wound and give bandage and let him go

e- Do nothing

A 22 months old child came to you with respiratory distress, unilateral wheeze.
Previously he was well, what is your management?
a- Measure O2 level

b- Do an x-ray chest inspiratory & expiratory

c- Give antibiotic

d- Give bronchodilator

A 3 months old child history of recurrent Rt eye infection. Previously respond to two
days antibiotic therapy. Now again infected and eye is sticky. What is your diagnosis?

a- Incomplete the course of antibiotic

b- Congenital obs. Of nasolacrimal duct

c- Infected with N. gonorrhoea

d- Viral conjunctivitis

Greenish vaginal discharge all are true except -

a- Trichomonas vaginalis

b- Gardnerella

c- Cervical carcinoma

d- Chlamydia

e- Foreign body aippg.com

A patient is taking phenytoin, which contraceptive is suitable for her -

a- Microgynon 30

b- Microgynon 50
A female patient 32 wks of pregnant. She had an accident and she was in the front seat.
She has only abdominal bruises due to the seat belt and slight vaginal bleeding. What
would you do except?

a- Observation

b- Do an urgent Ultrasonography

c- Induction of labour for the fear of placental abruption

d- Monitor the fetal HS

One 8 yrs old boy, 5% dehydration and hypokalemia. What is your Dx?

a- 0.9% Hartmann solution with Na lactate compound

b- 0.45% saline+ glucose

c- 5% dextrose

A bisexual man diarrhoea for one month, has mouth ulcers, artharalgia, one joint of
hand is painful; 2nd toe pain, ulcers on palms and soles, what is the diagnosis?

a- Reiter’s disease

b- Behcet’s disease

c- AIDS

d- Syphilis

e- Reactive arthritis

A picture (silvery scaly red lesion) of abdomen. What is your diagnosis?

a- Eczema

b- Pitoiriasis rosea

c- Psoriasis
X-ray of spiral # of humerous, what should be -

a- Ulnar N. lesion b- Radial N. Lesion

c- Median N. lesion d- Axillary N. Lesion

e- Radial A lesion

Brief episodes of monooccular vision loss -

a- Cerebral A occlusion b- internal carotid A aneurysm

c- Glaucoma d-Ipsilateral C. A. stenosis

Joseph., Oct 31, 2007

#93

Joseph.

Joseph. Guest

Retinal neovascularisation -

a- Glaucoma b- Retinal V thrombosis

c- Malignant melanoma d- Hypertension

e- D. mellitus

Progesterone (physiological change) in pregnancy causes -

a- Dilation of the ureter

b- Dilation of the ureter & pelvis

c- Dilation of the ureter, pelvis & calyces


d- Reduced incidence of infection

e- Stone

A female patient complains of fullness fat indigestion after fatty meals, in an ultrasound
stones in GB, asepsis. After surgery no stones left in the GB and free of symptoms for few
weeks, then symptoms recur. What is the explanation?

a- Pancreatitis after cholecystitis

b- Stone was left in CBD

c- It’s a result of removal of GB- fat indigestion

d- G. stones were not her symptoms

e- There should be recurrence of stone formation after cholecystectomy

aippg.com

23 yrs old girl has abdominal colic, 2 wks back she had viral infection, perpural rash.
Most probable diagnosis is -

a- ITP

b- Ischaemic colitis

c- Henoch-schonlein purpura

d- Glandular fever

A 75 yrs old female presented with lethargy. O/E WBC slightly increased, Hb decrease,
platelets variable, urate & Alk Po4 increase. What is your diagnosis?

a- ALL b- AML

c- CLL/CML d- Hodgkin’s disease

Which of the following has bad prognosis?

a- Hodgkin’s disease
b- Metastatic choriocarcinoma

c- Metastatic bronchial carcinoma

d- Metastatic stomach carcinoma

e- Non Hodgkin’s disease

A female patient has sore throat, white membrane on the tongue. O/Inves MCV is 118
(76 - 110), Hb 150 (110 - 150mg/dl). What is your diagnosis?

a- Haemolytic anaemia

b- Iron deficiency anaemia

c- Pernicious anaemia

d- Infectious mononucleosis

e- Thalassemia minor

What differentiates between haemolytic and iron deficiency anaemia?

a- Polycythaemia b- Polychromasia

c- Normal platelet count

Haemachromatosis is diagnosed by -

a- Ferritin level b- Transferin saturity

c- Hb level d- Fe. Level

Prodromal symptom which differentiates schizophrenia from delirium?

a- Idea of reference

b- Level of consciousness

c- Auditory hallucination
d- Visual hallucination

Aboriginals have increase risk of death compared with other Australians, due to -

a- Depression b- Anxiety

c- Bipolar affective disorder

d- Self harm

e- Drug and alcohol abuse

Oxytocin -

a- Is secreted from ant. pituitary

b- Acts by enzyme oxytenase

c- Causes contraction of various involuntary muscles like ut. & breast

d- Causes contraction of ut. only

e- Increase breast ankle

Joseph., Oct 31, 2007

#94

Joseph.

Joseph. Guest

Action of synthetic Syntocinon except -

a- ADH like action

b- Production of milk

c- Hypertension
d- Water intoxication

aippg.com

Absence of withdrawal bleeding after stopping of oestrogen progesterol pill is due to -

a- Hypothalamic insufficiency

b- Pituitary insufficiency

c- Problem in uterus

d- Primary ovarian failure

Progesterone can cause AOF except -

a- Mild hypertension

b- Amenorrhoea after 3rd injection

c- Infertility may occur up to 6 months after stopping

d- Breakthrough bleeding

In a 65 yrs old woman having ovarian tumour and endometrial hyperplasia WOF is
correct -

a- Fibroma b- Granulosa cell tumor

c- Teratoma

A young patient came with lower abdominal pain, she is ovulating. USG revealed 3 cyst
of 1.7cm, 0.7 cm, 2.2 cm. what will you go?

a- Reassurance and review

b- Tell the patient she has no problem

c- Cystectomy

d- Hysterectomy
e- OCP

Rx of mastalgia -

a- Bromocriptine b- Isotrinitoin

c- Indomethacin

A 50 yrs old woman came to with complaints of slight vaginal bleeding. She also
complains that she has amenorrhoea for 11 months. What is your diagnosis?

a- Endometrial carcinoma

b- Ca. cervix

c- Oestrogenic stimulation of the ovary

d- Vaginal atrophy

e- Cervical polyp

A woman having of 4200gm baby with shoulder Dystocia and still birth, now became
pregnant again. You can do all except -

a- Induction at 37-38th wks

b- USG after 34 wks

c- Blood sugar estimation at 28th wks

d- Midforcep delivery when bony head is in ischial spine

e- C/S

During the follow-up of a patient of hydatidiform mole, what is the most important
thing you will do?

a- Serum beta HCG level assay


b- Avoidance of pregnancy

c- Maintain the blood glucose level

A woman 16 wks of pregnancy having hypertension 110/95mm of Hg, proteinurea +++,


mild oedema what is your Dx?

a- Preclampsia

b- Renal hypertension

c- Glomerulonephritis

Joseph., Oct 31, 2007

#95

Joseph.

Joseph. Guest

A 18 yrs old lady came with hypertension. She has a history of enuresis up to 13 yrs.
WOF is the most likely diagnosis?

a- Reflux nephropathy

b- Hypertension

Erythema nodosum can occur in all of the following condition except -

a- Streptococcal infection

b- TB

c- Rheumatic fever

d- SLE

e- Leprosy
A picture of scale with hairless round area -

a- Tinea capitis

aippg.com

A picture of hand with scaly shiny things on it -

a- Psoriasis

How can we monitor heparin?

a- INR

b- APTT (activated partial thromboplastin time)

c- Fibrinogen

A patient von Willbrand’s disease. What is the most likely she has -

a- Prolonged bleeding time

A 62 yrs man has a long history of COPD and dyspnoea, blood gas showed Pco2 68mm
Hg Po2 60mmHg. He was given 28% o2 therapy. After an hour the blood gas showed 7.37,
Po2 80mm of Hg and Pco2 40mm of Hg but the pts clinical condition didn’t improved.
WOF is next therapy?

a- Bronchodilator

b- Hydrocortisone

c- Intubations and ventilation

About amyloidosis all of them are associated except -

a- Leprosy
b- TB

c- Multiple myeloma

d- Chronic active hepatitis

On 3rd postoperative day serum Na+ 165 mmol/L and urine osm is 250. WOF is the
cause?

a- Thaizide diuretic too high

b- DM

c- Diabetes insipidus

d- SIADH

Where can you find fat embolism?

a- Whole body

b- Upper part of the body

c- Depending sites of the skin

Joseph., Oct 31, 2007

#96

Joseph.

Joseph. Guest

WOF nerve supply the anterior 2/3rd of tongue?

a- Facial nerve

b- Hypoglossal N (XI)
WOF is the side effect of ca-channel blocker?

a- Angioedema

b- Peripheral edema

aippg.com

WOF is not the side effect of prednisolon?

a- Leucopoenia

b- Leucocytes

How would you differentiate between hemolytic anemia and anemia of chronic blood
loss?

a- Decreased hepatoglobulin

b- Raised ferritin

A patient COPD and polycythemia. What is the best measure you can taken to reduce
his attendance to the hospital?

a- Home O2 therapy

b- Phlebotomy

A 55 yrs old woman come to you with the complain of tremor. Her hands are shaking
when she answered the phone but it stopped when she look at her hand. On examination
you find coarse tremor in resting. What is the most appropriate Rx?

a- Benzihexol

b- Propranolol

c- Levadopa

What is the cause of diarrhoea in a bed-ridden pt?


a- Ulcerative colitis

b- Fecal impaction

c- Ischaemic colitis

A pt with 1.8 cm tumor in the pituitary and 3 mm on optic chiasm. What is the initial Rx?

a- Transfrontal resection

b- Transpheniodal resection

c- Bromocriptine

A patient with H/O intermittent diarrhoea, urethral discharge culture is negative, he has
ulcers in the mouth and arthritis and he has sausage shaped 2nd toe. What is the most
likely -

a- Reiter’s disease

b- Behechet’s disease

A patient with miosis and ptosis. What is the most likely cause?

a- Bronchogenic ca

b- Carotid sinus thrombosis

Joseph., Oct 31, 2007

#97

Joseph.

Joseph. Guest
A 60 yrs old man is taking thiazide diuretics and other hypertensive drugs, no found
serum glucose level is high what would be the management?

a- Stop the drug and check the serum level again

b- GTT

c- Starting antidiabetic drug for diabetes

A 23 yrs old lady came with complain of Dysuria. You give antibiotic. After a week shw is
completely symptom free. What will you do next?

a- Check the urine

b- Give some prophylactic antibiotic

c- Give nothing until she has got any problem

Indication of operation in aortic stenosis in a pt with pressure gradient 55% WOF is


likely -

a- Ejection fraction

A pt comes with swelling in the periorbital region and feet. Urine examination shows
red cell cast. What is the most likely Dx?

a- Glomerulonephritis

In aborigine and Torres state islander all of the following can be present except -

a- Depression

b- Anxiety

c- Alcohol abuse

What is the most common cause of the infective endocarditic?


a- Staphylococcus aerious

A 23 yrs old lady come with progressive weakness over both hands and wasting. WOF
will be the most likely Dx?

a- Multiple sclerosis

b- Syringomyelia

Regarding immunology, WOF statement is correct?

a- Killer cells are T-subsets and kill tumor cells

b- B cells produce first IgM and later IgG

c- IgM need the help of complement

In case of heamochromotesis WOF is the most important to make the Dx?

a- Serum Iron

b- Serum ferritin aippg.com

All of them could be the cause of proteinuria except -

a- Glomerulonephritis

b- Polycentric kidney

c- Horse shoe kidney

Joseph., Oct 31, 2007

#98

Joseph.

Joseph. Guest
A lady with H/O headache, she wakes up from the sleep at the same time every day.
What would be the most likely the preventive Rx for her?

a- Methyl sergide

A man came from Bali he has got watery diarrhoea. What is the most likely cause?

a- E.coli

b- Salmonella

c- Shigella

A patient came with bee sting and has got anaphylactic reaction. What would be the
most likely the immediate management?

a- Adrenalin

b- Hydrocortisone

c- ? aippg.com

Which type of hepatitis most likely become chronic?

a- Hep a b- Hep B

c- Hep C

WOF should use immuglobinal injection except -

a- Mumps b- Varicose

WOF Ca metastasis to brain except -

a- Breast b- Lung
c- Melanoma d- Prostate

e- Renal

20 yrs male lift heavy thing over his head and scope twice the next step -

a- Holter monitoring

b- X-ray

c- BP

d- Ultrasound of the heart

e- Echo

WOF acidosis with normal anion gap -

a- Aspirin

Hiv + what does it mean -

a- Salivary spreading

b- He/she will get life long infection and infertility

WOF are diabetes auto neuropathy -

a- Impotency b- Bradycardia

c- Urinary retention d- Diarrhoea

Diarrheas for 2 yrs -

a- Cohn’s disease b- Ulcer disease

c- Viral
VW (Von Willebrands) factors -

a- Prolong bleeding

Anaemia with MCV > 110

a- Intestine infection b- pernicious anemia

WOF are true about duodenal ulcer except -

a- Hunger pain b- Loss of appetite

c- Weight gain

CSF with normal glucose lymph cysts 100

a- Herpes virus b- Echo virus

c- Bacteria

A man with pleuretic chest pain and cough. There are no breath sounds in the right
lower zone, but bronchial breathing is noted in the right middle zone. WOF is most likely?

a- Right lower lobe consolidation

b- Right lower lobe collapse

c- Pleural effusion

d- Right pneumothorax

Joseph., Oct 31, 2007

#99

Joseph.
Joseph. Guest

About twin delivery, after the delivery of 1st baby you will do all of them except -

a- Palpate another baby

b- Per vaginal exam

c- Look any blood come out

d- Do an external version in case of breech

A 32 wks pregnant lady with herpes infection. What is correct -

a- Do caesarean section at term

A 32 wks pregnant lady with mother vehicle accident came to you. On examination you
some laceration on her abdomen otherwise she is normal. You will do all of them except -

a- Immediate delivery of the baby as she may have abruption placentae

In abruption placenta WOF is not correct -

a- Abdomen tender

b- Painless bleeding aippg.com

A 14 yr old girl with phenytoin. She came to you for ocp. WOF is most appropriate?

a- Microgynon 30

b- Microgynon 50

c- Diane 35

About oral pill in which level it act -


a- Pituitary

b- Hypothalamus

About oxytocin -

a- It has an antidiuretic effect

b- ?Smooth

c- Breast milk product

Absolute contraindication of OCP -

a- Thrombophlebitis

b- Migraine with hemiplegic aura

c- Smocking

d- Diabetes

What is the most dangerous sign of CTG?

a- Acceleration due to contraction

b- decelerations due to sleep

c- Very regular heart beat (variation less than 5)

A lady with 8 wks pregnancy passing some hydrophic mole through the vagina. What
would be the most likely management?

a- Measure HCG regularly until it becomes negative

A pregnant lady comes with 17 wks pregnancy with bleeding per vagina. WOF is the
least likely?
a- Ectopic pregnancy

A 8 wks pregnant lady come with vaginal bleeding. You will do USG due to all of the
following reason except -

a- To see the cause of bleeding

b- To determine the gestational age

c- To see the foetal malformation

d- To check foetal viability

A lady delivered a still born baby. WOF you will not do to confirm Dx?

a- Post mortem of the baby

b- Coomb’s test

c- Anti cardiolipid Ab

d- Anti lupus ab

A woman who has previously delivered a 4 kg baby with troublesome delivery. She is
para-2 and gravida 3. You should do all of the following except -

a- Induction of labour in 37-38 wks

b- Do caesarean section at term

c- Check blood glucose level regularly from 30wks

A patient come with 16 weeks pregnancy, she has high blood pressure 180/120 with
proteinuria, what is the most likely cause?

a- She has existing disease


b- She may have multiple pregnancy

A 19 yrs old primi 38 wks gestation with hyper reflexia, hypertension and oedema might
be excepted to have the elevated blood levels except -

a- Uric acid

b- Creatinine

c- Platelate decreased

d- SGOT

e- BUN

A lady came with vaginal bleeding after 2yrs of menopause. WOF is diagnosis?

a- Atrophic vaginitis

Short stature pregnant lady with 38 wks what is next step -

a- X-ray aippg.com

b- Caesarean

A girl with abdominal pain. US examination showed 3 cysts in the right ovary with 2, 3
and 4 cm?

a- Observe and 2-3 months repeat US

b- Reassuring

c- Laproscopy

d- Aspiration

NO bleeding after withdrawing contraceptive pills would be due to -


a- Ovary b- Uterus

c- Pituitary d- Hypothalamus

A patient forms a solid tumour which was solid, uterine curettage shows endometrium
hyperplasia. What is the cause?

a- Cyst b- Teratoma

c- Dysgerminoma d- Granulosa cell tumor

Effect of progesterone on kidney in pregnancy -

a- Relax only ureter

b- Relax both pelvis and ureter

All f the following could be cause of greenish foul smell discharge except -

a- Trichomoniasis b- Gardenella

c- FB d- Cervical cancer

e- ? Chlamydia

A Primigravida 38 wks in labour. On P/V pelvis was reduced. Station 0 cervix 4 cm


dilated, membrane intact -

a- X-ray pelvimetry b- C/S

c- Oxytocin d- Continuous trial labour

Joseph., Oct 31, 2007

#101

Joseph.
Joseph. Guest

A mother of 3 children has mastalgia. She is not responding usual treatment. Next
appropriate treatment -

a- OCP b- Danazole

c- Bromocriptine

A 37 weeks pregnant painful vaginal bleeding (1000ml blood loss). WOF is not
consistent with abruption placentae?

a- Foetal death b- BP 110/80

c- Uterus firm and contract d- Fetal head high in presentation

A patient who has difficulties with pregnancy. She lost her 3 pregnancy in 6, 8, 10 wks.
Before that she had terminate 2 babies at 10 and 12 wks. What could be the cause?

a- Cervical in competence

b- Deficient luteal phase

c- Chromosomal abnormality

Find out in correct association with pregnant lady which cause foetal effect on
pregnancy

a- IUGR - Anticardiolipin antibody

b- Platelate count 100,000 - intracranial haemorrhage

c- Heart block - antibody

25 yrs old woman galactorrhea and amenorrhoea. CT scan shows 3mm on optic
chaisma. Prolactin > 1500. Best Rx-
a- Transfrontal

b- Transphenoidal

c- Bromocriptine

d- OCP

e- Clomiphene aippg.com

A lady 3 days after giving birth becomes irritable and tearful. WOF is correct?

a- Seen in at least 50%

b- not occur in C/S patient

A woman came to you 2 yrs after menopause asking for treatment to reduce the
postmenopausal symptom. HRT oestrogen 1st 28 days and progestrogen 12. You are
explaining all except -

a- She will have premenstrual like symptom

b- Will have withdrawal bleeding in every month

c- Progesterone will increase HDL more than oestrogen

d- Will reverse the action of progesterone

e- Progesterone will augment the action of oestrogen for prevention of osteoporosis

Primary dysmenorrhoea all true except -

a- Bromocriptine

b- Indomethacine

c- Mefenamic acid

d- OCP
Pregnancy with group B streptococcus. Which is not true?

a- Infection in 32 wk is absolute indication for C/S

b- Can lead to neonatal meningitis

c- Can lead o neonatal infection

Pregnancy with carpal tunnel syndrome -

a- Become better as pregnancy

b- Treatment with siling and arm elevation

c- Required surgery

Pregnant woman wake up at night due to pain tingling and numbness in Rt hand. WOF
the cause -

a- carpal tunnel syndrome

b- Ulnar palsy

c- Branchial palsy

26 yrs old Nulliparous married wants baby complaining of severe menorrhagia. O/E
swelling in the abdomen up to umbilicus -

a- Total hysterectomy

b- Open myomectomy after correcting her anaemia with BT

Cleft lip in 2nd child what is the chance?

a- 2% b- 3 - 5%

c- 25% d- 9%
Joseph., Oct 31, 2007

#102

Joseph.

Joseph. Guest

A 2 yrs old boy with minimal contaminated cut injury he didn’t immunized before
what is the most likely treatment?

a- T.T. (Tetanus toxoid)

b- DPT

c- TIG

d- Antibiotics

A 2 yrs old boy came with unilateral wheeze-

a- Foreign body

A boy can roll over, can prone to supine and hold his head. He bounches in supported
standing. He laughs and babbles what is the age -

a- 3 month b- 7 month

c- 10 month

What is normal in a new born baby?

a- Hypertrophy of clitoris

b- Engorgement of breast

A chest x-ray of pneumonia -


a- Mycoplasma

b- Streptococcus A

A baby with jitteriness which will be the immediate management?

a- Serum glucose level

A 3 yrs old boy with hip pain. WOF is the most likely cause?

a- Perth’s disease

b- Tenosynovitis

A 8 wks baby with intermittent vomiting WOF most likely cause -

a- Urinary tract infection

b- Pyloric stenosis

A 9 wks old baby with force full vomiting his serum k 7.7 -

a- Pyloric stenosis

b- Congenital adrenal hyperplasia

All of the following could be the cause of anaemia in infancy except -

a- Prematurity

b- Multiple pregnancies

c- Thalassemia

a boy with weakness of both leg he has a history of viral fever without reflex WOF DX-
a- Gillian Barrie syndrome

A boy was bitten by a 1 meter long black snake WOF management -

a- Observation

b- Anti snake venom and apply tourniquet

aippg.com

Baby multiple healthy -

a- Parents deletion (non-accidental injury)

b- Environment

Joseph., Oct 31, 2007

#103

Joseph.

Joseph. Guest

Convulsion -

a- Hyper Na b- Hypo Na

c- Hyper K d- Hypo K

A child with multiple subperiostial hematoma and calcification -

a- Scurvy b- Accidental injury

c- Unaccidental injury d- Osteomyelitis


A boy cannot see blackboard L 8/20, R 10/20. He can see the letters on the screen of PC.
After wearing hole glasses sight improves -

a- Myopia b- Amblyopia

c- Astigmatism

A child was born normally WT 3.2 kg. Apgar score was 5 in 1 min and 8 in 5 min. He has
delay development. WOF condition is associated with his developmental delay?

a- Paternal uncle has intellectual deficit

b- Father is an alcoholic

c- Sister has febrile seizure

d- Mother has 2 café’s lait spots

e- Paternal grandmother has hypothyroidism

aippg.com

About mental health act WOF is correct-

a- Identical in every state

b- May be life saving

What is the prodormal feature of schizophrenia?

a- Idea of reference

b- Unemployment

c- Dysfunctional parenting

Hypnagogic hallucination WOF correct -

a- It can happen in normal sleep


About diagnosis of Tourette disorder all of the following are essential except -

a- Tic’s bou 3-4 times a day

b- Pt does not distress by tics

c- Always start before the age of 18

WOF is the distinguishing feature of schizophrenia and acute organic brain syndrome -

a- Difficulty in sustaining attention

b- Fluctuation disturbances of consciousness

c- Visual hallucination

d- Auditory hallucination

What is the best prognostic feature of schizophrenia?

a- Presence of affective disorder

All of the following cause of serotonine syndrome -

a- Haloperidol b- Clonazepam

c- Moclobomide d- Dextrometherphan

A bank manager was robbed 3 wks ago WOF most likely -

a- Out burst of anger b- Depression

c- Panic attack

A man is saying, “Wheat is good for health.” Wheat 5 types what is the do -

a- Clang
b- Confabulation

Déjà vu can be seen commonly in WOF lobe -

a- Tempo

b- Front

Joseph., Oct 31, 2007

#104

R.dass

R.dass Guest

The parents of an adopted girl, who has schizophrenic coming to ask about the chance
of their genetic daughter getting schizophrenia. WOF answer you would give?

a- Nil b- Less than 2%

c- 10% d- 30-40%

35 yrs old female comes to you with history of menorrhagia and anxiety. Recently she
feels her husband is detached from her and she is not close to him as she used to be. She
has 2 children living in a rented house and brought all household things on higher
purchases and they have to pay it off regularly. They can’t afford to go out together for
a dinner. Exam normal. What is your next step management?

a- Refer her to a psychiatrist

b- Send her to marriage consoling

c- Have an interview with her husband

d- Advice to have a holiday

e- Refer to a gynecologist
WOF is right -

a- Postpartum blues predispose to postpartum psychosis

b- More than 50 of women suffer from postpartum blues

c- She requires anti-psychotic medication

d- Requires ECT

Narcissistic personality -

a- Does not from any body

36 yrs old bus driver complain he can’t sleep at night apart from he driving stress.
You notice that the driver drinks 6 can bear and 10 cups of coffee weekly. What is next
step?

a- Tell him to go on sickness break

b- Treat for sleep apnea

c- Prescribe diazepam

d- Tell him to cut down on alcohol and change to different coffee

e- Tell him nothing needs to be changed

The most common cause of blood fluid from nipple of breast is -

a- Intraductal papiloma

b- Breast Ca

c- Trauma
The most likely for a male 22yrs ingurino-scrocal swelling with swelling disappeared on
lying down -

a- Direct inguinal hernia

b- Varicocele

c- Saphaena varix

d- Lymphoma

Contacted dermatitis except -

a- Avoiding antigen

b- Steroid IV

c- Don’t use steroids

WOF not related with breast Ca -

a- Contras breast Ca b- Family history

c- Uterine Ca d- Artificial menopause

R.dass, Oct 31, 2007

#105

R.dass

R.dass Guest

The most likely for a breast with toothpaste and nipple retracted -

a- Breast Ca b- Page’s disease

c- Mammary duct dysphasia d- Papillary


Picture a lesion on the back of hand -

a- malignant skin cancer b- Infected solar keratosis

c- Keratoacanthoma

The goals of surgery for mobility of obesity are all the following except -

a- Reduce energy

b- Increased metabolic

50 yrs old female abdominal pain for half day, the most likely diagnosis is -

a- Omentum arterial vessel blocking

Hutchinson melanoma freckle except -

a- Covered area

b- Irregular edges

Picture a big neck swelling -

a- Goiter

b- Graves

aippg.com

X-ray tibia and fibula opening fracture -

a- Debridama internal fix- type I

b- If no contaminations, debridma is unnecessary

c- External fix- type II, III


d- Close cut

Picture - Colles fracture. What is the most likely complication?

a- Malunion b- Wrist stiffness

13. Picture - Figures deformation -

a- Rheumatoid arthritis

b- Contracted tendon

ECG preoperative exam -

a- LBBB b- RBBB

c- WPW

A spinal L4.5 slipped disc produces -

a- Absent ankle jerk

b- Lost of dorsiflaxion

c- Loss of sensation on the medial calf

After falling injury, x-ray showed pelvic injury and blood clots at external urinary
meatus. WOF are appropriate -

a- Flory catheter b- IVP

c- Tap d- Urethrography

Undescended testis in children is most commonly associated with -

a- Infertility b- Indirect inguinal hernia


c- Malignancy d- Torsion

The most common tumor to metatarsals to bone is -

a- Stomach b- Ovarian

c- Breast d- Rectum

e- Colon

R.dass, Oct 31, 2007

#106

R.dass

R.dass Guest

. A diabetic with heel ulcer. The cause is -

a- Blood vessel b- Infection

c- Peripheral never

Management of Wernicke’s syndrome with low plasma sugar should be -

a- IV 5% glucose

b- Vita B1 and then 5% glucose

The advantage of mammography is -

a- Painless

b- Early detection than self examination


Fistula -

a- Two opening between

A patient with acidic urine WOF most common stone can be seen -

a- Uric acid b- Cystine

c- Oxalates

A boy 2 yrs with subclave strawberry naevus -

a- Observe b- Surgery

c- Laser d- Nitigen

aippg.com

A 65 yrs old man with profuse bleeding per rectum the cause is -

a- Diverticulosis

b- Ca rectum

c- Ischemic colitis

Most correct about Otosclerosis -

a- A reddish tinge on the tympanic membrane

b- Normal tympanic membrane

VII nerve least occurs in WOF-

a- Chronic parotids

b- Acoustic neuroma

c- Mastoiditis
d- Base of the skull fracture

Uncomplicated mucosal haemorrhoids WOF is not true -

a- Lowering of a lump

b- Bleeding

c- Server pain

d- Mucous discharge

Immediate management of Rt tension pneumothorax -

a- Intercostals tube

b- Wide bore needle at 2nd intercostals space

c- Endotrachial tube

After splenectomy which one of the following will not occur?

a- Giardia

A middle aged man presents with BP 80/60, HR 120, back pain and slight abdominal
guarding, WOF is most likely diagnosis?

a- Acute pancreatitis

b- Acute myocardial infarction

c- Leaking aortic aneurysm

d- Perforated peptic ulcer

34 yrs old man presents with acute abdomen for 3 days and he has H/O vomiting
following by perinial pain. O/E abdomen distended but non tender, bowel sound absent.
Abdominal x-ray shows multiple air fluid levels. What is the best management before
surgery?

a- 2000ml 4% in 0.45% NaCL

R.dass, Oct 31, 2007

#107

R.dass

R.dass Guest

The common treatment for severe obstructive hypertrophic cardiomyopathy is:

a- Digoxin

b- Captopril

c- Atenolol

d- Verapamil

e- Lasix

A 12 yrs old girl, while watching TV at night time, becomes distressed and short of
breath. On examination she was breathing rapidly. This disorder is due to:

a- Epilepsy

b- Asthma

c- Panic attack

d- Depression

Digoxin gives all of these arrhythmias except:

a- Atrial fibrillation
b- Ventricular tachycardia

c- Ventricular ectopics

d- Nodal rhythm

e- Paroxismal atrial tacycardia

Prolong PR caused by:

a- Digoxin

b- Amiodaron

c- Verapamil

d- Atenolol

aippg.com

A 45 yrs old lady is very depressed and she wants to commit suicide. She feels guilty and
refuses any medical treatment and admission to hospital. Her relatives also disagree to
hospital admission but want to give her medication at home. What is the right
management?

a- Advice antidepressant

b- Close monitoring by family members

c- Admit her to hospital for urgent ECT regardless of their consent

d- Start antidepressant by injection for monitoring action

A young man has been suffering pain in his left eye for the past two days. He complains
of a headache and on examination his temperature is 38.5 c and he also swelling in his left
upper eye lid. What is the right management?

a- Analgesics

b- Intravenous antibiotics

c- Topical chloramphenicol
d- Eye trauma

A 50 yrs old hypertensive man has come to your surgery and complains of failure of
erection in the last couple of weeks. What is your initial management?

a- Reassure the patient that this is normal and can happen in this age bracket

b- Viagra

c- Review his medication

d- Ask about his sexual activity and sexual technique

e- Assess cardiovascular status

Combined HRT have all the advantages except:

a- Prevent hyperplasia of endometrium

b- The production of some perimenopausal symptoms

c- Production of period bleeding at the end of cycle progestrone

d- Augmentation of bone protection by progesterone

A 23 yrs old woman with secondary amenorrhea is found to have high level of
prolactine and on CT scan is found small adenoma in the pituitary which was over 1mm in
chaisma optic. She is planning a pregnancy in 12 months time. What is the best treatment
for this patient?

a- Transfrontal resection

b- Transphenoidal resection

c- Bromocriptine

d- Chemotherapy

What would be seen in a one-sided lesion of the trigeminal nerve?


a- One-sided facial paresis

b- Inability to poke the tongue out from the mouth

c- Inability to raise the palate

d- Displacement of the jaw when the mouth is open

What is inappropriate in the treatment of an acute asthma attack?

a- Prescribing oral steroids

b- Prescribing inhaled steroids earlier to reduce an inflammation

c- Prescribing combination of inhaled Salbutamol and oral theophyline

d- Educate the patient to be more active to decide about his/her medication

e- FEV measure at home

R.dass, Oct 31, 2007

#108

R.dass

R.dass Guest

A 65 yrs old man presents with blurred vision and moderate pain in the left eye. On
examination the eye is red, sclerae is misty, pupil dilated and fixed. What is your
diagnosis?

a- Retinal vein occlusion

b- Glaucoma

c- Cataract

d- Iritis

e- Conjunctivitis
The acute anterior uveitis is most commonly associated with:

a- Rheumatoid arthritis

b- Ankylosing spondylitis

c- Sjorgenis syndrome

d- Reiteris syndrome

e- SLE

Middle age man suffering hypertrophic obstructive carditis has an outflow obstruction
of the left ventricle on ECHO examination. Which drug can help to reduce this gradient?

a- Verapamil

b- Captopril

c- Frusemide

d- Digoxine

e- Prazosines

A 67 yrs old woman is suffering from tremors in her right hand especially when she is
watching TV or answering a phone. On examination a coarse tremor appears in the right
hand while she talking but it disappears when she looks at her hand. There is a slight
spasticity in the right hand without “cogwheel”. What medication is most likely to help?

a- Propranolol

b- Carbidopa/levodopa

s- Segmental

d- Benztropin

e- Observe
What is the characteristic for Granuloma Anullare?

a- Raised irregular border

b- Itchy

c- Usually ulcerate

d- Premalignant

e- Respond to griseofulvin

Bronchogenic adenoma is:

a- Always benign

b- Premalignant

c- Malignant

d- Never obstract bronchus

e- Never diagnosed on Endoscopy

A 24 yrs old man comes to see you because of sharp abdominal and lower back pain.
The sample of urine which he has brought with him is obviously blood-stained. He is
asking you to give him a penitidine injection for pain relief. On examination the abdomen
is intact without quarding or tenderness. What is the next most appropriate step?

a- Send him home

b- Do penitidine and reassess him 1 hour later

c- Do penitidine + buscopan, send him home, review next day

d- Examine the fresh urine sample

e- Send him to the hospital for surgical assessment

Why do Australian women have a longer life span than Australian men?
a- Women more often use the medicare system and look after their health

b- Women are more biologically and genetically safe

c- Men are more prone to accidents

d- Men use the gym more

e- Men are more stressed at work

What is the most important thing about screening programs for some disease?

a- Interference with the natural history of the disease

b- New techniques are available

c- Treatment is available

d- The importance to pick up earlier stages of the disease

A student presents to your surgery with fever, malaise and sore throat for the last 12
days. On examination you find whitish exudate on the tonsils, maculo-papular rash on
moderate lympho-adenopathy. What is your diagnosis?

a- Syphilis

b- Yaws

c- Ep. Barr virus infection

d- Pemphygouse

e- Steven-Johnson’s syndrome

What is not true about eczema?

a- Usually on the flectural surfaces

b- Moisturising agents are helpful

c- Soap and water are helpful


d- Dryness of the skin

e- Family history of asthma and hay fever

R.dass, Oct 31, 2007

#109

R.dass

R.dass Guest

Lesion of the spinal cord at S1 level produces which symptoms?

a- Absent ankle jerk

b- Absent knee jerk

c- Loss of dorsiflaxion of the foot

d- Loss of sensation on the medial aspect of the cal

A 35 yrs old policeman comes to you for jaundice. He is taking sulfasalazine,


Prednisolone for ulcerative colitis. He drinks 60gm alcohol/day. His lab investigations are:

Serum bilirubin 18 (N 17)

Alkaline phosphate increase

GGT increase

SGOT 30 (N.26)

SGPT normal

What is your diagnosis?

a- Fatty liver

b- Alcoholic cirrhosis

c- Bilary stone
d- Sclerosing cholangitis

e- Cholestatis

Causes of hypercalcemia:

a- Vitamin D toxicity

b- Hyperparathyroidism

c- Bony metastasis

Patient with Parkinson’s disease can have all of the following except:

a- Tremor during sleep

b- Mask like face

c- Depression

d- Bradykinesia

e- Waddling gait aippg.com

Right sided hemiplegia involving right side of face is most likely a result of:

a- Middle cerebral artery embolism

b- Carotid artery embolism

c- Subclavian artery

d- Basilar artery

e- Posterior cerebral artery

A young man presents with a sore throat for the past 24 hours and he develops
proteinuria++, and hematuria. What is this?

a- IgA nephropathy
b- Post streptococcal glomerulonephritis

c- Membranous glomerulonephritis

d- Interstitial nephritis

A 60 yrs old patient presents with CCF and Captopril is commenced with history of
hypertension. He is already long term Chlorothiazide, it will result in:

a- Serum creatinine is increased

b- K is increased

c- K is decreased

d- Na is decreased

e- Increase in cholesterol

Which drug can rapidly cross blood brain barrier?

a- Water-soluble

b- Lipid soluble

What is the specific feature of Parkinson’s disease?

a- Abdominal postural reflex

b- Broad based gait

c- Hyperreflexia

d- Hypertonia

e- Loss of downward gaze

What is the commonest way of transmission of Hepatitis C in Australia?

a- Blood transmission
b- IV drug abuse

c- Body fluids

d- Homosexuality

A patient who suffers from chronic arthritis looks pale. He takes NSAID. Hb low, Fe low,
Ferities high, MCV normal. What is the most likely cause?

a- Iron deficiency anemia

b- Anemia of chronic disease

Patient with more than 12cm splenomegaly blood film shows tear drops polkilocytosis.
What is the most likely cause?

a- Myelofibrosis

b- CLL

c- CML

d- Multiple myeloma

R.dass, Oct 31, 2007

#110

R.dass

R.dass Guest

A 27 yrs old male diagnosed with anemia, blood exam shows microcytic hypochromic
anemia. What is your initial investigation?

a- Stool exam

b- Colonoscopy
c- Sigmoidoscopy

d- HB electrophoreses

A patient presented with anemia. On examination it was found that Hb low, A2 high.
What is the most likely cause?

a- Thalassaemia minor

b- Hereditary spherocytosis

What is the common cause of death in a patient with Thalassaemia?

a- Heart failure

b- Renal failure

c- Infection and septicemia

d- Anemia

A young lady is suffering from menorrhagia. She has bruises on her limbs. Her brother
and father have bleeding disorder:

a- Haemophilia A

b- Von Willebrand’s disease

c- Congenital Hypofebrinogenemia

d- Haemophilia B

aippg.com

A woman who has history of Von Willebrand’s disease has cholecystectomy. What is
your initial management?

a- Cryoprecipitate

b- Vita. K
c- Blood transfusion

d- Factor VIII

Which of the following is least likely to cause transmission of HIV and Hep. B virus:

a- Albumin

b- Factor VIII

c- Fresh frozen plasma

d- Platelet concentrates

e- Cryoprecipitate

A young man suffers from fever, mild generalized lymphadenopathy, and macula-
popular rash on the upper part of the body and palm and soles. He also had erosion in
pharynx, Grey tonsilar exudate, perianal lesion. What is your diagnosis?

a- Yaws

b- Secondary syphilis

c- Infectious mononucleosis

Granuloma annular is:

a- Pruritis

b- Ulceration

c- Griseofulvine should be given

d- Premalignant

e- Raised irregular border

43. Australian woman live longer. What is the cause?


a- Genetic determination

b- Visit doctor frequently

Which of the following findings suggests that the patient has gout?

a- High serum acid

b- Birefringence crystals in Synovial fluid

c- High level of uric acid in urine

Rheumatic fever’s clinical feature:

a- Involved only large joints

b- Usually affects small joints

c- Does not cause permanent deformity

Hypertension is the most preventable factor in which if the following conditions:

a- Stroke

b- Coronary artery disease

A patient came to the emergency department claiming that he has got snakebite.
Clinical exams revealed that there was no sign of bite, but there was an area of erosion on
the left foot. What is your management?

a- Administration of anti-venom

b- Observation

c- Apply tourniquet

d- Sent him home


A patient was diagnosed with trigeminal neuralgia. Which of the following clinical
finding is true?

a- Patient has sensory loss

b- Jaw is deviated to the unaffected side, when asked to open it

Regarding emphysema, all of the following clinical findings are true except:

a- Fixed expended

b-

c- Fixed expiratory capacity

d- Decreased ventilator capacity

An alcoholic patient is complaining from oily stool and weight loss. What is your next?

a- Stoll fat estimation

b- Pancreatic function test

c- ERCP/CT

d- Sweet teat

e- X-ray of abdomen

R.dass, Oct 31, 2007

#111

R.dass

R.dass Guest

Thrombolytic therapy is indicated in which of the following conditions?

a- Q wave in ECG
b- Development of recent RBBB

c- Development of LBBB

d- ST depression

In infective endocarditis, WOF is true

a- Vasculitis

WOF true for polycythemia rubravera:

a- Erytropoetin is decreased

b- It is difficult to diagnose cyanosis

c- Plasma volume is contracted

d- ESR is raised

Regarding papillary necrosis of the kidney, all of the following are true except:

a- Alcohol

b- Medullary sponge kidney

c- Analgesic nephropathy

d- Diabetic nephropathy

A pt. with hyperkalemia and increased S. creatinine WOF is the best method to diagnose
whether his renal failure is due to acute or chronic renal failure?

a- Creatinine clearance test

b- Serum urea level

c- Ultrasonography

d- IVP
Pt. developed puffiness in the face following upper respiratory tract infection. His BP is
180/120 mmHg, WOF things can happen:

a- If haematuria is present it is dangerous

b- There is immediate danger of development of LVF

Blood Na level is 120meq/L. What does it mean?

a- Extracellular fluid osmolarity is decreased

b- Total body Na is decreased

c- SIADH

d- Decreased ADH secretion

e- Renal exchange of Na is decreased

Pt. was hypertensive, developed heart failure and has gout and was having Allopurinol
ACE inhibitor, Hydrochlorothiazide and indomethacin. What is the cause?

a- Deplesion of Na in the extracellular fluid

b- Indomethacin toxicity

c- ACE inhibitor toxicity

aippg.com

Hypercalcaemia may occur in all the causes except:

a- Pancreatitis

b- Hyperparathyroidism

c- Renal tubular acidosis

Diabetic autonomic neuropathy- all are true except


a- Bradycardia

b- Nocturnal diarrhoea

c- Urinary retention

d- Impotency

e- Pupil size

Regarding Diabetes mellitus- NIDDM is more related to the following than IDDM

a- Abdominal fat is a risk factor

b- IDDM is more genetically related than NIDDM

c- Oral hypoglycaemic agents reduce insulin resistance

Diabetic ketoacidosis most commonly occurs in

a- Undiagnosed IDDM

b- Known IDDM pt. forgot to take insulin

c- Diabetic pt. with infection

d- Known IDDM c infection

Regarding positive balance diet, WOF is true

a- Olive oil increases LDL

b- Olive oil reduces LDL

c- Taking fish 300gm per day does not decrease LDL

d- Alcohol does not change triglyceride level

An 18 yrs old girl with history of jaundice came with palmer fever and palmer erythema.
Which of the following is true for liver biopsy findings?
a- Periportal lymphocytes and bridging necrosis

b- Fatty change in the liver

A picture of a 23 yrs old girl with infected cystic acne with hirsuitism. Which of the
following is the most appropriate initial treatment?

a- Antiandrogen

b- Isotretinoin for 20wks

c- Isotretinoin with tetracycline

d- Broad spectrum antibiotic

R.dass, Nov 1, 2007

#112

R.dass

R.dass Guest

A photo showing circular raised margined lesions on the face. What is the possible
diagnosis?

a- Microsporum canis

In paracetamol poisoning, the cause of morbidity is-

a- Hepatic injury

b- Metabolic acidosis

c- Metabolic alkalosis

d- Hypoglycemia (cause of mortality)


In tricyclic anti depressant poisoning WOF are true except:

a- HCO3 to prevent arrhythmia

b- Flumazenil should not be given if is due to TCA + Diazepam

c- Bile acid

d- Magnesium

e- Diazepam to

Regarding bronchial asthma all are true except:

a- Total lung volume is decreased

b- Increase in residual volume

c- Decreased tidal volume

d- Decreased vital capacity

aippg.com

WOF are true regarding Asthma except:

a- It increases Br. Sensitivity

b- It does not occur after 50 yrs of age

c- Inspiratory crackles

A pt. with one sided dullness with shifting of trachea on the same side, Diminished
breath sound and respiratory movement. Which of the following is true?

a- Pleural effusion with collapse

A 23 yrs old lady with distended abdomen, glositis, muscle wasting, anaemia,
macrocytosis, diarrhoea no history of surgery. WOF is true:

a- Gluten sensitive enterocolitis


b- Duodenal biopsy is reqd. for confirm the diagnosis

c- Crohn’s disease

d- Ulcerative colitis

e- Pernicious anaemia

A photo of raised irregular border lesion in the leg

a- Granuloma Anullare

An abattoir worker came with H/O fever, myalgia and dark urine. The most likely cause
is

a- Leptospirosis

b- Brucellosis

Regarding eradication of H. Pylori which is true

a- it helps in healing of the ulcer

b- Prevents relapse rate

c- Need for long time therapy

Which of the following is not true regarding transferrin?

a- Increased in iron deficiency anaemia

b- Increase in chronic infection

c- Decrease in nephritic syndrome

d- Increased in pregnancy
Pt. with severe chest pain, absent carotid pulse, pain radiating to the back. What would
you find in chest X-ray?

a- Widening of the mediastinum

In thyrotoxicosis all of the following are present except

a- Limb girdle muscle weakness

b- Small muscle wasting

c- Atrial fibrillation

Regarding Myasthenia gravis which is true

a- Some people does not have antibody

b- Thymectomy is only done when thymoma is present

c- Reflexes are diminished after repeated stimulation

d- EMG always slow

e- Electomyograph

In a pt. with aortic cardiac catheterization shows pressure gradient of 55mmHg. Which
of the following will help you to take decision for operation?

a- Decreased Lt. ventricular ejection fraction

b- Increased LV end diastolic volume

R.dass, Nov 1, 2007

#113

R.dass

R.dass Guest
A pregnant lady presents with mild dyspnoea. O/E splitting of the 2nd Ht. Sound and no
change with respiration, what is the possible cause?

a- ASD

b- VSD

Regarding keratoacanthoma which is true

a- rapidly growing and spontaneous resolution

Regarding Prednisolone, which are true except

a- Osteomalacia

Regarding stent in coronary angioplasty which is true

a- Reduce complication and increase stenosis

b- Reduce complication and reduce stenosis

A lady came with dyspnoea. Blood gas analysis shows- HCO3-23, Pa CO2 29, Pa O2 60,
PH 7.4. Which of the following is true for her condition?

a- Respiratory alkalosis with reduced gas exchange

b- Respiratory alkalosis with normal gas exchange

A 65 yrs old man with profuse bleeding per rectum. The cause is-

a- Diverticulosis

b- Ca. rectum

c- Ischemic colitis
A pt. came from Indonesia. Has diarrhoea treated with Metronidazol for 3 days was not
responding. Stool culture 4 days before shows salmonella typhae. Regarding further
management WOF is true-

a- Amoxycillin

b- No Rx repeat culture after 10 days

c- Add Trimethoprim + Sulfonamide

d- Give Metronidazol

. Most common cause of pseudomembranous colitis is-

a- Ampicillin

b- Vancomycin aippg.com

c- Metronidazol

A man suffering from diarrhoea for 3 months. WOF is not true

a- Camylobactor jejuni

b- Giardia

Regarding Giardia infection, diagnosis is confirmed by which of the following

a- Tropozoide finding from duodenum aspiration

b- Cyst in the stool

c- Histopathology of duodenum shows Giardiasis

A young male pt. came with fever and cough. On x-ray it was diagnosed as an inactive
T.B. M.T. was possible. What is your next step of your management?
a- Triple drug Rx

b- Consecutive 6 sputum culture and wait for the result

c- INH prophylaxis for 12 months

d- Repeat X-ray after 3 months

Anaerobe non-spore bearing bacteria could be isolated from which of the following

a- Brain abscess

b- Hip prosthesis

Pt. is in shock. If you ask for blood bank which of the following would be most
appropriate blood group to be transfused?

a- O Negative

b- Same group uncrossed blood

All of the following are characteristic features of Mycoplasma Pneumonae except

a- Pleuritic chest pain

b- Severe cough

c- Fever

d- Lobber pneumonia

Regarding Lt. Renal artery stenosis all of the following are true except

a- Rx with ACE inhibitor will deteriorate the function of the same kidney

b- Hyperkalemia

R.dass, Nov 1, 2007


#114

R.dass

R.dass Guest

A 35 yrs old lady nullipara wishes to conclave, but has fibroid uterus, bleeding 60ml, Hb
level is 90g/l, O/E ut is boggy to the level of the umbilicus. What is your management?

a- Correct the anaemia the myomectomy

b- Suppression of the fibroid by GnRH analogue for 3 months the myomectomy

c- Hysterectomy

d- Hysterectomy & oophorectomy

e- Observe 6 months the myomectomy

Infective endocarditis: what is the most common organism?

a- Staphylococcus aureus

b- Streptococci viridans

c- Streptococci faecalis

d- Coxiella

e- Viruses

A pt. came to you with severe chest pain ECG done but it is normal, what is your
management?

a- Thrombolytic therapy should not be given if ECG is normal

b- Discharge the patient for going home

c- I/V heparin and nitrates are contraindicated

d- Exercise test is to be done immediately


e- ECG should not be repeated unless pain once again

aippg.com

A 22 months old child came to you with respiratory distress, unilateral wheeze.
Previously he was well. What is your management?

a- Measure O2 level

b- Do an X-ray chest inspiratory and expiratory

c- Give antibiotic

d- Give bronchodilator

A 22 yrs old man came to you complaining of snake bite 2 hours before. O/E there is
abrasion in the leg otherwise normal. What is your management?

a- Give antivenom

b- Observe for 24 hours

c- Give an inclusion and let bleed for 5 minutes

d- Clean the wound and give bandage and let him go

e- Do nothing

A bisexual man diarrhoea for one month, has mouth ulcers, artharalgia, one joint of
hand is painful: 2nd pain, ulcers on palms and soles. What is the diagnosis?

a- Reiter’s disease

b- Behcet’s disease

c- AIDS

d- Syphilis

e- Reactive arthritis
Retinal neovascularisation:

a- Glaucoma

b- Retinal V thrombosis

c- Hypertension

d- Malignant melanoma

e- Diabetes mellitus

A 23 yrs old girl has abdominal colic; 2 weeks back she had viral infection, perpural rash.
Most probable diagnosis is:

a- ITP

b- Ischaemic colitis

c- Henoch-Schonlein purpura

d- Glandular fever

What differentiate between haemolytic and iron deficiency anaemia?

a- Polycythaemia

b- Polychromasia

c- Normal platelet count

Oxytocin:

a- Is secreted from ant pituitary

b- Acts by enzyme oxynase

c- Causes contraction of various involuntary muscles like ut & threat

d- Causes contraction of ut only


Progesterone can cause AOF except:

a- Mild hypertension

b- Amenorrhoea after 3rd injection

c- Infertility may occur up to 6 months after stopping

d- Breakthrough bleeding

In a 35 yrs old woman having ovarian tumour and endometrial hyperplasia WOF is
correct?

a- Fibroma

b- Granulosa cell tumour

c- Teratoma

A 50 yrs old woman came to you with complaint of slight vaginal bleeding. She also
complains that she has amenorrhoea for 11 months. What is your diagnosis?

a- Endometrial carcinoma

b- Ca. cervix

c- Oestrogenic stimulation of the ovary

d- Vaginal atrophy

e- Cervical polyp

A woman 16 weeks pregnant having hypertension 110/95 mm of Hg. proteinurea +++


mild oedema. What is your diagnosis?

a- Preclampsia

b- Renal hypertension

c- Glomerulonephritis
30 yrs cystic fibrosis with chest pain treated with nebulised tobramycin th/DM, recent
weight loss, steathhorrea. Correct management?

a- Decreased carbohydrates diet and stabilising glycaemic control

b- Pancreatic enzyme

c- Continue nebulised tobramycin for another 3 weeks

d- Chest physiotherapy

R.dass, Nov 1, 2007

#115

R.dass

R.dass Guest

Difficult breathing after penicillin for U.R.T.I. has generalized urticaria and blood
pressure 90mm Mg. Correct management?

a- IV I.M adrenalin 1:10,000

b- S.C. adrenalin 1:1000

c- I.V. pneumothorax

d- Airway assessment

e- 500ml IV colloid

Each is important host defence against bacterial meningitis:

a- IgA

b- Ciliary activity

c- Mucosal epithelium
d- Complement

e- IgM

factor least likely to affect prognosis of breast ca:

a- Tumour size

b- Oestrogen receptor status

c- Menopause

d- Involvement of axillary lymph node

e- Percent 8 cell in s-phase

Nephrotic Syndrome commonly caused by except?

a- Post glomerulonephritis

b- Result decrease in plasma volume

c- Plasma cholesterol normal but HDL

d- Result in renal salt wasting

aippg.com

37 yrs old man with depression presents to emergency drowsy and aggressive. He has
taken a bottle of doxepin and scotch together. Important management includes:

a- Admin for ECG monitoring for 4 hours

b- Serial doses of charcoal orally

c- Do serum paracetamol as the only drug screen

d- As the patient is conscious significant T.C.A. overdose can be ruled out

e- Alkalisation of urine will aid excretion of doxepin, and will protect against arrhythmia
A 15 yrs old girl presents with fever and painful left forearm two days after her cat had
clawed the arm. There is regional lymph adenopathy present and temperature is 39.C.
Which of the following statement is true:

a- The implicated bacterium E. coli

b- Most suffer of cat fever are >60

c- Complication may includes involvement live, spleen, central nervous system, bowel
and skin

d- Ampicillin and cephalosporin are inadequate therapy

e- Lymphadenopathy in the field of drainage is invariable

A 71 yrs old woman presents with crushing chest pain, a ECG shown. She is pale and
diaphoretic, peripheral vein. The most likely cause of the hypotension is?

a- Chordae rupture causing acute mitral regurgitation

b- S.B.E.

c- Right ventricular myocardial infarction

d- Second degree AV Block

e- Pericardial tamponade

Insulin resistance is a complication of on-going therapy requiring increasing doses.


Which of the following is the most common cause?

a- Anti-insulin antibody

b- Switching from procine to human insulin

c- Obesity

d- The somoguy effect

e- Non-compliance
Condition that are associated with Lichen planus includes all of the following with the
exception of?

a- Primary biliary chirrhosis

b- Graft versus host disease

c- Chronic active hepatitis

d- Amalgam in dental fillings

e- Chronic myeloid leukemia

Lipid lowering drugs are a complex group of drugs with a spectrum of side effects.
Which of the following indication is correct?

a- Gemfibrozol is indicated for hypercholesterolaemia

b- Simvastatin is effective for hypertrigliceridemia

c- Fish oils are suite for treating hypercholesterolaemia

d- Fluvastatin is effective for combined hypercholesterolaemia and


hypertrigliceridaemia

e- Individual without coronary heart disease with LDL cholesterol level > 4.5 mmol/l
should not receive such drugs because of side effect profiles

Which of the statement concerns infantile spasm is correct?

a- Age of onset is usually 18 months

b- They occur less frequent with physically activity

c- ECG is usually normal

d- Developmental milestone are unaffected

e- Aetiology is frequently idiopathic


A girl infant is brought to your rooms for investigations of abdominal pain manifested by
colic and progressive constipation. Other siblings are well. Which of the following is least
likely to b implicated?

a- Lead poisoning

b- Hypo thyroidism

c- Hirsprung disease

d- Hypocalcaemia

e- Infantile botulism

Louis a 5 year-old-by is brought to you for testing his free lead level, which is
significantly elevated. His physical and loco motor examination is completely normal. On
questioning, the parents state that they have recently moved to an old terrace. The initial
most appropriate management would be to?

a- Commence decontamination of the house

b- Commence a chelating agent

c- Commence d-penlcillamine for 2-6 months

d- Check the free erythrocyte protoporphyrin

e- Check the urinary excretion of the lead after administration of calcium EDTA

Alf. a 74 yrs old man recently admitted for management of respiratory failure asks
about home-oxygen. The following are criteria for long-term continuous oxygen therapy
except:

a- PaO2 < 55mmHg

b- O2 saturation < 88%

c- Treating the hypoxia associated with the sleep apnoea

d- If the patient has mental impairment oxygen should be withheld for safety reasons

e- Congestive heart failure with intractable angina


All of the following statements about inflammatory bowel disease are true except:

a- Pseudopolyps developing in ulcerative colitis do not become malignant

b- Development of fistula between bowel and skin or bladder suggests granulomatous


bowel disease, such as a regional enteritis

c- Severe hypotension with fever, leucocytocis and abdominal distention suggests


development of toxic megacolon which usually requires immediately surgery

d- Occult bleeding occurs in both ulcerative colitis and regional enteritis but gross
bleeding is more characteristic of ulcerative colitis

e- Metronidazole may be to control acute diarrhoea in Crohn’s disease

R.dass, Nov 1, 2007

#116

R.dass

R.dass Guest

Graves disease:

a- Is the major cause of thyrotoxicosis in the elderly

b- Is more common in men than women

c- Is associated with delectable thyroid stimulating antibodies in 50% of cases

d- Usually remains in remission after a course of anti-thyroid drug therapy

e- Ophthalmopathy frequently does not parallel the clinical course of thyrotoxicosis

The pain associated with uterine contractions may be relieved by:

a- Diazepam
b- Intramuscular Pethidine

c- Pudendal block

d- Inhalation of nitrous oxide

e- Caudal block

Causes of non-unlon of fracture include-

a- Very slight banding movements during the healing phase

b- Infection of the fracture haematoma

c- Anoxia

c- Uraemia

e- Paget’s disease

Clouding of consciousness:

a- Is a hallmark of dementia

b- Occurs frequently in Schizophrenic disorder

c- When diagnosed should prompt research for organic aetiology

d- Refers primarily to disorientation in time

e- Is frequently not noticed when it occurs in hospitalized patients

Which of the following are true statements regarding psoriatic arthritis?

a- The commonest type is an oligcarticular, asymmetric arthritis

b- An association with HLA B27 is seen in those patients with sacro-iliitis

c- Pure distal interphalangeal joint change is uncommon

d- Gold therapy is effective in the polyarticular types


e- Periarticular osteoporosis is a radiological feature

aippg.com

In a patient with fibroadenosis of the breast:

a- Duct papillomatosis may be present

b- Pregnancy usually produces relief

c- The breasts should be replaced by prosthetic implants

d- Radiotherapy is a useful palliative treatment

e- The breasts should be replaced by prosthetic implants

Regarding therapy of thyrotoxicosis:

a- Carbimazole acts mainly to inhibit the endocytosis of thyroglobulin

b- Propanolol is particularly useful in the presence of cardiac failure

c- Post-surgical recurrence is best managed by antithyroid drugs and/or radio-iodine

d- Therapeutic radioactive Iodine has been shown to be associated with an increased


incidence of leukemia

e- Electroversion of atrial fibrillation is performed prior to commencement of


antithyroid drugs

Gastro oesophageal reflux:

a- May present with heartburn

b- It always associated with a hiatus hernia

c- If untreated usually leads to stricture formation

Years old patient presents with headache and sudden monocular visual loss. What test
should immediately order?
a- Complete blood count

b- Sedimentation rate

c- Skull x-ray

d- Chest x-ray

e- Urinalysis

A patient presents with a painful arc of abduction at the shoulder joint in the range
from 60-120 degree. Which of the following conditions would cause this?

a- Osteoarthritis of the shoulder joint

b- Partial tear of the rotator cuff muscle

c- Fractured neck of humerus

d- Arthritis of the acromio-clavicular joint

e- Subcaspination calcification

A 37 yrs old man requests a check up. He says he is quite well and has no particular
worries. There is no significant past or family history. The only positive finding on
examination is a BP reading of 160/110. Urinalysis is normal. The doctor should:

a- Arrange for an IVP

b- Commence a trial of Thiazide diuretic

c- Advise patient to return for review in one week for another examination

d- Prescribe a B blocker drug and review in two weeks

e- Arrange chest x-ray and ECG

The most serious side effect of tri-cyclic drugs involves:

a- The genitor-urinary tract


b- The salivary glands

c- The elementary tract

d- The cardiovascular system

e- The respiratory system

A patient aged 60 yrs presents with bleeding per vaginam. Two years previously she had
the same complaint and at that time uterine curettage revealed no abnormality. The best
management would be:

a- Hysterectomy

b- Colposcopy

c- Haemoglobin estimation

d- Culdoscopy

e- Hysteroscopy

Arthritis is common in which of the following conditions

a- Rubella

b- Mumps

c- Measles

d- Infection mononucleosis

e- Varicella

R.dass, Nov 1, 2007

#117

R.dass

R.dass Guest
In which of the following conditions of the colon is malignant change most likely to
occur?

a- Adenomatous polyp

b- Melanosis coli

c- Diverticulitis

d- Familial intestinal polyposis

e- Ulcerative colitis

Bone pain is recognised as an important feature in which of the following conditions?

a- Paget’s disease

b- Multiple myeloma

c- Osteomalacia

d- Primary hyperparathyroidism

e- Sarcoidosis

Features of primary herpetic stomatitis include-

a- vesicles and ulcers on the buccal mucosa as well as tongue and palate

b- Inflamed gums

c- Pain

d- Recent contact with chicken pox

e- Cervical lymph node enlargement

Causes of aseptic necrosis of the hip includes

a- Femoral neck fracture


b- Alcoholism

c- Corticosteroid therapy

d- Sickle cell disease

e- Dysbaric disorders

aippg.com

Patients with major depression often have:

a- Inversion of diurnal rhythm

b- Early morning wakening

c- Diurnal mood variation

d- Abnormal cranial CT scans

e- Diarrhoea

A patient with serous otitis media:

a- Is usually a young man or man

b- May have an allergy which blocks the Eustachian tube

c- Is said to have glue ear

d- Has a bulging red eardrum

e- May require the insertion of a grommet

Raynaud’s Phenomenon may be secondary to-

a- Atherosclerosis obliterans

b- Scleroderma

c- Cold hemaglutination

d- Ergotism
e- Rheumatoid arthritis

Formal thought disorder often occurs in-

a- Agoraphobia

b- Schizophrenic disorder

c- Delirium

d- Histrionic personality disorder

e- Conversion disorder

A patient undergoing an overnight Dexamethasone suppression test (1 mg orally at


midnight) fails to show suppression of her plasma cortisol measured at 8 am. This could be
due to:

a- Endogenous depression

b- Alcoholism

c- Obesity

A 56 yrs old man presents with peritonitis due to ruptured colonic diverticulum. After
successful surgery he is found to be ollguric hypotensive (BP 90/60) and febrile (38.5 C.)
Serum creatinine is 460 mmol/l, urea 32 mmol/l, Na 134 mmol/L, K 3.0 mmol/L and HCO3
16 mmol/L, Haemoglobin is 16.4 G/100ml. Urine Na concentration is 5 mmol/L. Which of
the following statements is/are true?

a- He has established acute tubular necrosis

b- Dialysis must be instituted immediately

c- He is saline and may recover his renal function simply with intravascular volume
repletion

d- Blood cultures should be taken followed by antibiotic therapy

e- Serum K is reduced because he has a metabolic acidosis


R.dass, Nov 1, 2007

#118

R.dass

R.dass Guest

Regarding gas shadows and fluid levels seen on an x-ray incases of intestinal
obstruction-

a- A fluid level seen at the duodenal cap is physiological

b- The distended jejunum is flat and characterless

c- The ileum is characterized by the concertina effect of the valvulae conniventes

d- Obstruction of the ascending colon often causes fluid levels in the small intestine
despite the lieocaecal value

e- The presence of fluid levels in the large intestine is indication an urgent barium meal
and follows through

The following are likely to be normal in a five year old child:

a- A blood pressure of 135/90

b- A palpable systolic thrill

c- A third heart sound

d- A pulse rate of 100/minutes

e- A spit second heart sound

In the management of ingrowing toenail-

a- The success of conservative measures depends a great deal on the patient’s co-
operation
b- The toenail should be cut across convexly

c- The corners of the nail should be cut back

d- Avulsion of the whole nail is the most effective method of treatment

e- Removal of the germinal matrix on the affected side is the surest means of cure

aippg.com

Regarding subarachnoid haemorrhage-

a- Berry aneurysms are responsible for at least half of the cases

b- The aneurysms are due to atherosclerosis

c- The aneurysms are due to syphilis in 20% of cases

d- No age is exempt

e- 20% patients have more than one aneurysm

Congenital lymphoedema-

a- Is also called Milroy’s disease

b- May be familial

c- Is usually complicated by cellulitis or ulceration

d- Is associated with increased length as well as girth of the involved limb

e- Is usually secondary to lymphatic obstruction due to tumour

Management of the lymphodemalous limb may antall-

a- Bed rest and elevation

b- Use of support stocking

c- Use of intermitted compression therapy

d- Intermittent antibiotics
e- Lympho venous

Which of the following is most likely to increase kidney damage?

a- Heart Failure with hypertension

b- Dehydratation and electrolyte imbalance

c- Systemic infection

d- Urinary tract infection

e- Drug toxicity

An alcoholic presents with disorientation, diploma and ataxia of 6 days duration. On


examination is disorientated and presents Nistagmus and bilateral gaze palsy, there is also
symmetrical polyneuropathy. This patient probably has:

a- Hepatic encephalopathy

b- Delirium tremens

c- Alcoholic hypoglycaemia

d- Wernicke’s encephalopathy

e- Korsakoff’s psychosis

The normal daily requirements of potassium in adult is:

a- 5 meq

b- 25 meq.

c- 55 meq.

d- 1 meq.

e- 155 meq
Rapid shallow breathing is found in:

a- Metabolic Acidosis

b- Metabolic Alkalosis

c- Emphysema

d- Restrictive Lung Disease

e- Scoliosis

R.dass, Nov 1, 2007

#119

R.dass

R.dass Guest

Which of the following is not true for third heart sound?

a- Is not heard in Atrial Fibrillation

b- Is present in left heart failure

c- Is diastolic

d- Is normal in children

e- Can be heard in Mitral Insufficiency

Which of the following is the true Best investigation for Bleeding disorders?

a- Bleeding time

b- P.T.

c- PTT

d- Platelets
e- Clinical history

A patient has urinary incontinence, upper motor neurone in the legs, extensor planter
response bilateral, propiception and pain sensation is altered and also has lack of
sensation from the inguinal region below. What is the diagnosis?

a- Spine lesion at T10

b- Spine lesion at L1-L2

c- Cervical compression

d- Syringomyelia

e- B12 deficiency

A 67 yrs old male ha urinary problems, wasting and weakness of biceps in both arms,
spastic presents in both legs and bilateral planter response. What is your diagnosis?

a- Multiple sclerosis

b- Cervical compression with myelopathy

c- Motor neurone disease

d- Subacute combined degeneration

aippg.com

A 60 yrs old man suffered of in and needles in the legs, ataxia and glove and stocking
sensation. What is the diagnosis?

a- Diabetic neuropathy

b- Marie Charcott Thooth

c- Vitamin B 12 deficiency

d- Gillian Barre

e- Diabetic Amyotrophy
A 65 yrs old male complains of pulsatile temporal headache anorexia, loss of weight and
muscle pain, his E.S.R. is 107. He probable has:

a- Dermatomyositis

b- P.A.N.

c- Giant Cell Arteritis

d- Cerebral tumour

e- Ver tebro-basilar insufficiency

Which of the following is not protein anabolic?

a- Growth hormone

b- Thyroxine

c- Insulin

d- Androgens

e- Cortisol

Electrical impulse can cause ventricular fibrillation if it falls on:

a- P wave

b- P-R interval

c- QRS

d- ST interval

e- T wave

Increased jugular venous pressure without displacement of Apex beat can be found in
all of the following except:

a- Mitral Insufficiency
b- Cor- pulmonale

c- Mitral stenosis

d- Myocardial infarction

e- Costrictive pericarditis

Regarding hepatic encephalopathy, which of the following is not true?

a- Improves with Porto-caval shunt

b- Carbohydrate in diet are helpful

c- Protein in the diet is helpful

d- Thiazide diuretics are not indicated

R.dass, Nov 1, 2007

#120

R.dass

R.dass Guest

A homosexual male refers fatigue for 3 months, anorexia and vomiting. Lab results are
as follows:

BIL 15 Albumin 35 ALT: 80

AST 100 HBsAg (+) Liver biopsy: Chronic Active Hepatitis

Regarding this patient which of the following is correct

a- Absence of jaundice is unusual in CAH - 352 D

b- The biopsy result is unexpected with respected to the raised AST and ALT

c- His sexual contacts must be given immunoglobulin

d- There are less than 2% chances that he will develop cirrhosis


e- He may develop Hepatoma

aippg.com

Regarding Q fever - which of the following is not true?

a- Is caused by coxiella Burnett

b- Is spread by tick

c- Infected persons may transmit the condition to others

d- Affect may organs

e- There are abnormal liver function tests

Regarding Echinoccocous in Australia, man os usually:

a- A primary host

b- An intermediate host

c- Only accidentally infested

d- Infested by the parasite entering via unbroken skin

e- Infested by eating infested liver and sheep meat

Treatment of Trycophyton is:

a- Topical Griseofulvin

b- Topical Miconazole

c- Oral Metronidazol

d- Topical Nistatin

e- Oral Thiaoprine

A patient is on treatment or Parkinson’s and develops dyskinesia:


The best management should be:

a- Decrease Levo-dopa

b- Increase Levo-dopa

c- Give anticholinergics

d- Add Adamantine

e- Add Bromocriptine

Which of the following statements about the clinical interview is correct?

a- It is important to question the patient at length on the whole medical history even
though the patient is very ill to ensure at no detail is over looked

b- If the patient is very ill, even though he is conscious, what he says is usually
inaccurate and it is wise not to put too much weight on it

c- The key feature to be established for each symptom is whether or not it is


psychogenic

d- There has been a complete failure of patient to doctor communicate diagnosis at the
conclusion of the interview

e- Emotionally charged detail is often better assessed at a second or third interview

A rough estimate of the extent of the field of vision can be obtained by comparing the
visual fields of the subject under test with those of the examiner. In performing the test,
the examiner sits face to face with the subject.

If you were judging the performance of a clinician, which of the following points of
technique would you consider to be incorrect?

a- The light source is located behind the patient

b- The subject is asked to cover the eye not under test and to look the examiner in the
eye with the other

c- The subject’s right eye is compared with the examiners right eye and left with left
d- The examiner prevents the patient from moving his head by fixing the patient’s
chin centrally with his free hand

e- The examiner’s hand moves in a plane midway between the subject’s face and
his own

In which of the following situations is a turning fork of no value?

a- Testing high tone hearing loss

b- Testing limb vibration sense

c- Testing middle ear conduction function

d- Comparing right and left acoustic nerve function

e- Distinguishing between middle and inner ear deafness

A lump in the neck can be identified as a goitre because:

a- It moves upwards when the tongue is protruded

b- It moves upwards in swallowing

c- It displaces the trachea to one or other side

d- It is tender and fluctuant on palpation

e- There often is fixation of the overlying skin to the lump

Which of the following sets of signs most appropriately describes - Isolated sever
chronic (Mitral) regurgitation

a- Slow rising pulse, apical impulse displaced and sustained, basal ejection systolic
murmur

b- Jerky pulse, apical impulse displaced active and ill sustained apical pan systolic
murmur and third heart sound
c- Large volume collapsing pulse, apical impulse displaced and sustained, early peaking
ejection systolic murmur and early diastolic murmur down the left sternal margin

d- Small volume pulse, apical impulse undisplaced, loud first heart sound at the apex
with opening sound and diastolic murmur

e- Jerky pulse, apical impulse slightly displaced active

R.dass, Nov 1, 2007

#121

R.dass

R.dass Guest

Which question would you judge to be incorrect?

a- She percusses from resonant to less resonant areas of the chest when defining the
upper border of the liver

b- She percusses along a line at right angles to the border of the heart whose size &
position she is attending to define

c- She delivers only 2 or 3 blows in any one position of the pleximeter finger before
moving to the next position

d- She percusses over the distal phalanx of the pleximeter finger & maintains a slight air
gap between the remainder of the finger & the chest wall

e- She compares corresponding areas of the chest on each side rather than percussing
out one side at a time

Which of the following most accurately describes the third heart sound?

a- A high frequency systolic sound related to left ventricular ejection

b- A low frequency early diastolic sound related to rapid early ventricular filling, heard in
some fit young people & also in heart failure
c- A low frequency late diastolic sound related to atrial contraction & a noncompliant
ventricle aippg.com

d- A high frequency early diastolic sound coincident with mitral valve opening

e- A low frequency early diastolic sound heard only in heart failure

Which of the following statements about examination of the abdomen is correct?

a- Abdominal enlargement most marked in the flanks suggests small bowel obstruction

b- The presence of enlarged superficial veins over the abdomen suggests portal
hypertension

c- A midline pulsatile mass suggests an abdominal aneurysm

d- The triangle formed by the 12th rib, the erector spine muscles & the quadratus
lumborum is important in kidney examination

e- An easily seen mass above the inguinal ligament suggests a direct inguinal hernia

Isolated cerebrallar lesions may produce each of the followings except:

a- Truncal ataxia

b- Limb ataxia

c- Nystagmus

d- Hypertonia

e- Intention tremor

In a patient who has had a stroke as a result of a cerebral embolus: Which of the
following areas of clinical examination is least likely to be helpful in determining the basis
of the patients problems?

a- Pulse rate, rhythm & character

b- Auscultation for a mid diastolic murmur at the apex


c- Examination of the retina for haemorrhages closely associated with retinal infarcts

d- Palpation of the calves for thrombosed deep veins

e- An ECG & auscultation of the heart for an atrial gallop & pericardial friction rub, if
there has been a history of prolonged chest pain

Which of the following groupings of clinical features carries the gravest prognosis for a
patient presenting with hypertension regardless of the apparent efficacy of treatment?

a- Arm BP of 200/170, blood pressure known to have been normal 6 months previously,
strong family history of essential hypertension

b- Arm BP 150/120, papilloedema present with pulmonary oedema and ankle oedema,
serum creatinine 0.3 mmol/L. (upper limit of normal 0.09mmol/L)

c- Arm BP 150/120, papilloedema present with pulmonary oedema, serum creatinine


0.28 mmol/L if diabetes also present

d- Arm BP 210/135, rib-notching on x-ray, leg BP 90/40 male patient age 12 years and
systolic murmur audible over precordium

e- Arm BP 160/125, retinal haemorrhages soft and hard exudates L ventricular


hypertrophy present clinically with loud aortic component to the second sound, late onset
mild diabetes present

If you suspect a patient has suddenly developed an embolic blockage of a Femoral


artery -

a- You should warm the leg to induce maximal vasodilatation of skin and collateral
vessels

b- You should withhold pain relief (morphine and omnopon) until a consultant has seen
the patient

c- You should call for surgical help immediately

d- You should elevate the leg to increase venous return

e- The patient and his relations should be informed that amputation is a likely outcome
Patients with peripheral (lower limb) atherosclerosis associated with intermittent
claudication-

a- Should keep their legs and feet as warm as possible at all times of the year

b- Should have an initial angiographic study to establish a base line and subsequently
repeats angiograph because symptoms and signs can be so misleading

c- Should be encouraged to walk within the limits of their claudication pain and
encourage trying to increase their range of daily activities over a period of weeks or
months

d- Should be encouraged to use footwear which is as light as possible in order to reduce


work loads on their calf muscles

e- Should always be given a 6-12 trial of vasodilator drugs because these have been
demonstrated to open up capillary-channels with the majority of patients

A 45-year-old non-smoking lean man experiences the sudden onset of severe anterior
chest pain which radiates to his neck and down both arms. The pain is described as heavy
and lasts 2 hours before being partially relieved by an injection. Which of the following
statement is correct?

a- This man has no identifiable risk factors so that the diagnosis of myocardial infarction
is unlikely

b- The pain is fairly typical of muscular skeletal pain and if it recurs would best be
treated by giving 15mg of morphine by deep IM injection

c- If the pulses in the upper limbs are unequal and on early diastolic murmur is present a
diagnosis of aortic dissection is possible

d- You find that the ECG on admission is within normal limits: this together with the
history strongly suggests that the pain is due to something other than acute myocardial
infarction

e- The ECG on admission shows ST elevation in leads II III and AVF with 1st degree Av
block. Temporary cardiac pacing is indicated because of the risk of complete heart block.
A 19 years old Polynesian girl with the history of the rheumatic fever at the age of 10
years is admitted late one night with a 4 days history of arthralgia and dysponea on
exertion. She has been taking oral ampicillin prescribed by her GP for the past 2 days. On
examination she is febrile with signs of moderate cardiac enlargement but no evidence of
heart failure. A loud apical pansystolic murmur is present. Other examination reveals no
other abnormality. The most appropriate action for you to undertake as the house
surgeon on the evening of admission would be:

a- Continue her ampicillin I/V after taking a series of blood cultures

b- Take a throat swab and blood for serology and after liasion with the laboratory take a
series of blood cultures withhold antibiotic treatment

c- Accept that blood cultures are not likely to be useful and treat her overnight with
penicillin, cloxacillin and Gentamicin to cover the possibility of infective endocarditis

d- Take a throat swab and blood for serology, continue ampicillin and start high dose
aspirin to relieve the arthralgia and reduce her temperature

e- Take a throat swab and blood for serology, withhold antibiotics and commence on
high dose steroid treatment because of the possibility of rheumatic carditis

R.dass, Nov 1, 2007

#122

R.dass

R.dass Guest

Which of the following statements concerning heart failure is most correct?

a- AS cardiac output drops progressive peripheral vasodilatation occurs in an attempt to


maintain peripheral circulation

b- Ventricular filling pressure is usually reduced

c- Digoxin through its inotrophic action produces an increase in cardiac output and a
rise in ventricular filling pressure and myocardial oxygen requirement
d- Diuretics relieve congestive symptoms by reducing filling pressure without altering
cardiac output

e- Dopamine is the drug of choice for treating severe heart failure complicating acute
myocardial infraction

Which of the following sets of clinical findings would be most consistent with isolated
severe mitral stenosis?

a- Normal pulse, displaced overactive L ventricular apical impulse, apical pansystolic


murmur and S3

b- Collapsing pulse, displaced overactive left ventricular apical impulse, early diastolic
murmur down the L sternal edge and apical mid-diastolic murmur

c- Normal pulse, systolic waves in the JVP and a systolic murmur down the L sternal
edge which increases with inspiration

d- Small volume pulse, tapping L ventricular apical impulse, loud S1, opening snap and
long mid-diastolic murmur, pre-systolic accentuation at the apex

e- Slow rising pulse, sustained L ventricular apical impulse, basal ejection systolic
murmur apical S4

A 73 yrs old woman with a long h/o heart failure presents with nausea, vomiting and
abdominal pain increasing over the previous month. On examination she has signs of
moderate congestive heart failure with basal crepitations, JVP 4cm elevated, liver
enlargement 4cm below the costal margin and slightly tender on palpation and moderate
pitting ankle oedema. She sys she has been taking digoxin 0.25mg 1 tablet mane and slow
K 2 tablets mane.

Which of the following statements is most correct?

a- If the serum electrolytes are normal digoxin toxicity is unlikely and the patients GIT
symptoms are most probably related to hepatic congestion

b- The patient should be given Propranolol 40mg tds as prophylactic antiarrthmic


treatment while digoxin toxicity is ruled out
c- Because of the patient’s vomiting the heart failure would be best treated by extra
digoxin and Lasix parenterally

d- The symptoms of Digoxin toxicity and hepatic congestion resulting from heart failure
may be similar

e- The patient may be hypokalemic and the heart failure would be best treated by
substituting aldactone for Lasix and giving extra potassium supplement

A 41 year old businessman, father of three children, presents to the GP on a Sunday


afternoon with a h/o dull retrosternal pain for the first time.

The first episode occurred during a round of golf, but was relieved with rest in about 10
minutes. There after episodes occurred with intermitted frequency and severely in
relation to exertion, but on the morning before he came to the surgery, he had an episode
of pain lasting 20 minutes while shaving.

He was a heavy smoker and had a long h/o wheezy bronchitis. When examined he was
quite undistressed in normal sinus rhythm BP 150/95, no evidence of hear failure. ECG
normal,

Which of the following would you consider most appropriate?

a- Request admission to the local hospital for further management

b- Prescribe anginine tablets to be taken sublingually if the pain recurred

c- Reassure him that there is no sign of coronary problem and ask him to call back if
there is any recurrence

d- Commence him on Propranolol 80mg tds, together with anginine as required for
recurrence of pain

e- Have him exercised by running up and down the 2 flights of steps adjacent to your
office and then repeat ECG aippg.com

Which of the following is true of JVP?

a- It has diffuse character and A&V waves relating to atrial contraction and venous are
often visible
b- Systolic pulsation is characteristic of tricuspid stenosis

c- Regular cannon “a” waves occur with junctional rhythm

d- Irregular cannon a waves occur in complete heart block

e- Normally the venous pressure

A young man aged 19 with a previous h/o rheumatic fever is found to have an enlarged
heart on a routine CXR. His pulse is regular and BP 140/10. The JVP is normal and the liver
is not enlarged. With this information although there may well be more than one murmur
present, which on of the following murmurs would you except to be present?

The level of the diastolic Bp is most likely a reflection of

a- The degree of arterio sclerosis

b- Intravascular volume

c- The resistance offered by pericapillary arterioles

d- Cardiac output

e- Cigarette smoking

Which of the following on its own consider the least important in relation to risk from
coronary artery disease?

a- Alcohol abuse

b- Hypercholesterolaemia

c- Hypertension

d- Family history of coronary artery disease

e- Cigarette smoking
A 34 year old Polynesian woman comes to A&E department with a 2 hour h/o acute
dyspnoea of sudden onset associated with irregular palpitations. No other definite history
can be obtained.

A friend says she thinks the patient may have had rheumatic fever in childhood. On
examination she is very distressed and dyspnoeic, pulse is irregularly irregular: the apical
rate being approximately 120 beats/minute.

The JVP is slightly elevated, liver is not enlarged and there is no oedema. Widespread
fine crepitations are present throughout the lung fields. The apical impulse is undisplaced
and tapping in quality.

On auscultation at the apex, the first heart sound is loud and a diastolic murmur is
audible. Which of the following statements related to the case is least likely to be true?

a- Digitalization and diuretic therapy are appropriate in treatment of this lady

b- Cardioversion is unlikely to be successful

c- When the heart rate is low the diastolic murmur should be more prominent

d- The chest x-ray will show moderate L ventricular dilatation, pulmonary venous
hypertension and interstitial oedema

e- Echocardiogram done when heart rate is slowed will show a reduced (flattened) E-F
diastolic filling slope of the anterior mitral valve leaflet

By which of the following methods may bacteria transfer anti-crobial resistance or


virulence factors

a- Conjunction

b- Transduction

c- Transformation

d- None of the above

e- All of the above

R.dass, Nov 1, 2007


#123

R.dass

R.dass Guest

Bacterial plasmids

a- Consists of RNA

b- Are the factors responsible for bacterial virulence?

c- May be transmissible to other bacteria

d- Are never responsible for resistance to antibiotics

e- Are only found in germ negative bacteria

In which of the following clinical situations has antagonism between antibiotics in


combination been shown to occur?

a- Tobramycin and carbenecillin in pseudomonas aerogenosa septicemia

b- Gentamicin and cephalosporin in Klebsiella pneumonia

c- Tetracycline and penicillin in pneumococcal meningitis

d- Streptomycin and penicillin in enterococcal endocarditis

e- Gentamicin and mefacillin in staph sepsis

Aminoglycoside antibiotics activity

a- Inhibiting cell wall synthesis

b- Damages cytoplasmic membrane

c- Prevents nucleic acid replication or transcription

d- Impairing protein synthesis at ribosomes

e- Impending essential metabolic pathways


Grading of tumours is based on

a- Fine structural appearance

b- Histological appearance

c- Clinical examination

d- Gross examination

e- All of the above

aippg.com

Which of the following gram positive organisms will not respond to penicillin?

a- Bacillus anthracis

b-

c- Corynbacterium diptheriae

d- Staph aurius

e- Clostridium welchii

Infection with a toxin producing strains of corinybacterium diptheriae is usually


associated with -

a- Temperature of 40 c. and follicular tonsillitis

b- Temperature of 38 c. and grey membranes at the site of infection

c- Positive paul-bunnel test

d-

e- Inhibition of synaptic input to spinal motorneurone

Listeria monocytogenes may cause

a- Infections in immunosuppressed patient


b- A malignant fistulae

c- Following anterior

d- Gastroenteritis

e- Encephalitis lehtrogication

In trichophyton fungal infection of the scalp taenia capitia

Which is correct?

a- Exam of the affected scalp under woods light

b- Exam of the affected hair under woods light

c- Exam of the affected hair under fluorescent microscope

d- Exam of the affected skin by fluoroscopy

e- All of the above

A 60 years old female complaining of tremors in her hands while doing her house work.
These tremors disappear on resting her hands on her knees, but when her hands are
grasped the tremors reappear again. What would you give her?

a- Amentadine

b- L-dopa

c- Benzodiazepine

d- Beta blocker (Propranolol)

e- Bromocriptine

Which of the following is the best parameter in the pre-operative assessment of a


patient with bleeding diathesis?

a- Bleeding time
b- C.T.

c- P.T.

d- P.P.T.

e- The clinical history

R.dass, Nov 1, 2007

#124

R.dass

R.dass Guest

A 18 year old girl complaining of menorrhagia, bruising and purpuric rash. The blood
count was found to be normal except for the platelet count which was 20x 109L. On exam
the spleen was 5cm below the costal margin. What is your diagnosis?

a- I.T.P.

b- Henoch-Schonlein purpura

c- S.L.E.

d- QLL

e- Hypersplenism

A 55 years old female Rt. handed, came to the casualty with sudden onset of Rt.
hemiparesis of the face and body but preserved sensation. What is the most likely
diagnosis?

a- Rt. middle cerebral artery thrombosis

b- Lt. middle cerebral artery thrombosis

c- Atherosclerosis of the arteries of the internal capsule

d- Inferior cerebral artery occlusion


aippg.com

A 55 years old man was brought to the casualty by his wife unconscious. His wife said
that her husband is alcoholic and has been drinking a binge of alcohol for 3 days. TWO
week ago he was brought to his home with bruises and unconscious. She said that her
husband’s appetite after that was good and nothing wrong with him. Than he started
to be confused. O/E he found to have palmar erythema and spider nevi and extensor
plantar reflex. What is your diagnosis?

a- Subarachnoid Hge

b- Subdural hematoma

c- Hepatic encephalopathy

d- Wernicke’s encephalopathy

e- Brain tumor

A 27 years old male came with arthritis, urethritis and skin lesions over the palms and
soles also the penia after an attack of gastroenteritis a week before. What is the most
likely diagnosis?

a- Behcet’s disease

b- Reiter’s disease

c- Gonorrhoea

d- Felty’s syndrome

e- He probably has seropositive disease

What is closely related to GFR?

a- Serum bilirubin

b- Serum Uric acid

c- Serum Creatinine & creatinine clearance

d- Serum urea
A young man 24 yrs old on routine exam. He was found to be normal except for having a
Blood pressure 160/100. There is no history of hypertension. What would you do?

a- Start with Thiazide diuretic

b- Repeat the exam. One week later

c- Order an ECG

d- Order an X-ray

e- Order IVP and Catecholamines in urine

Aortic regurge occur with the following except:

a- Marphan’s syndrome

b- Ankylosing syndrome

c- Aortic medial leaflet necrosis

d- Rheumatic fever

e- Prolonged steroid therapy

In cystic fibrosis all are correct except:

a- Sterility and oligospermia

b- Complete villous atrophy

c- Increase NaO1 in the sweat

d- Steatorrhea

Which of the following sets of physical signs if elicited in a patient would specifically
alert you to the possibility of thyrotoxicosis?

a- Warm moist hands and a bruit over a palpable thyroid gland


b- A palpably enlarged thyroid gland with mild stridor on inspiration

c- Atrial fibrillation and mild hypertension

d- Weight loss with a record of persistent slight pyrexia

e- Abnormally brisk tendon reflexes and a coarse muscle tremor

What is the most reliable clinical method of determining whether a thyroid gland is
diffusely enlarged?

a- Careful inspection of the front of the neck in the thyroid area from both sides and
frontal position

b- Palpation of the gland with the fingers of both hands during a swallowing movement,
the examiner standing behind the patient

c- Measurement of the neck circumference 5cm above the supra-sternal notch and
comparing the result with standard values for age and ex sex

d- Palpating the gland with the right hand and during a swallowing movement from a
position in front of and slightly to the left of the patient (assuming a right handed
examiner)

e- Measuring the length and breath of the gland with thyro-calipers and comparing the
results with the standard record

R.dass, Nov 1, 2007

#125

R.dass

R.dass Guest

A rugby player comes to you with a moderately painful swollen left knee joint, injured
during a game the previous day. Which of the following signs would most clearly suggest
the possibility of his having torn the medial meniscus (semi-liner cartilage)?

a- Limitation of the last few degrees of extension by a springy resistance


b- Inability to fully flex the knee because of pain

c- Noticeable wasting of the left quadriceps

d- Audible clicking from within the joint on passive flexion/extension

e- Abnormal anterior posterior range of movement of the tibia on the femur with the
joint flexed at 90c.

An effusion into the pericardium range of demonstrated by -

a- Scintigraphy (radio-nucleotide scanning)

b- Computerized tomography

c- Ultrasound

d- Intracardiac injection of opaque medium (Angiocardiography)

e- All of the above

Osteomyelitis (infection of bone) -

a- Usually involves mid-shaft near the nutrient artery

b- may show no radiological abnormality in the early stages

c- Typically shows an area of low uptake on scintigraphy

d- Should first be examined by ultrasound

e- Can be distinguished from tumour by presence of periosteal new-bone formation

The rate of elimination of alcohol from the body depends largely on -

a- Essentially by the kidney

b- Essentially by respiration

c- Rate of conversion of alcohol to acetaldehyde

d- Rate of conversion of acetic acid to CO2 & H2O


e- Rate of conversion of acetaldehyde to acetic acid

aippg.com

A child who is in good health but his parents c/o of that the child has attack of complete
loss of movement especially by night when he watches the TV. The attack last few seconds
and during which his eyes are opened and he can hear voices. The parents say that one of
their relatives has the same problem. The diagnosis is:

a- Sleep paralysis

b- Grand epilepsy

c- Myoclonic epilepsy

d- Temporal lobe epilepsy

e- Familial periodic paralysis

A 15 year-old-boy is complaining of headache, bluring of vision, absence of public hair,


polyuria and polydypsia. His parent noticed that he has not progressed in growth. What is
the diagnosis?

a- Pituitary adenoma

b- Diabetes insipidus

c- Diabetes mellitus

d- Craniopharyngioma

e- Medulloblastoma

What is associated with heparin induced thrombocytopenia?

a- Neutropenia

b- Tachycardia

c- Anaemia

d- Eosinophilia
e- Thrombosis

Thrombocytopenia due to salicylate therapy may commonly associated with -

a- Splenomegaly

b- Anaemia

c- Leucopenia

d- Eosinophilia

e- G.I.T. bleeding

A 66 yrs old man with weakness and wasting of the upper limbs, decrease biceps jerky
spasticity increase deflexed of legs and no sensory symptoms.

a- motor neurone disease

b- B12 deficiency

c- Cervical Spondylosis

d- Syringomyelia

e- Alcolic neuropathy

A 45 years old man C/O of urinary urgency and 10/E he was found to have flaccid
paralysis of the lower limbs, increased reflexes and extensor planter reflex. This is sensory
loss below the inguinal ligament. No abnormality in the upper limbs, the diagnosis is:

a- MS

b- S.C.D.C.

c- Vord compression at T 10

d- Disc prolapse at L 1-2

e- Disc prolapse at L 4-5


R.dass, Nov 1, 2007

#126

R.dass

R.dass Guest

What is common in autoimmune haemolytic thrombocytopenia purpura and ITP?

a- Splenomegaly

b- Petechiae on buttocks

c- Haemorrhage in the optic fundi

d- Erythema nodosum

e- Purpura rash on the lower bodyparts (extremities)

In hepatitis A

a- IgM means immunity

b- Bilirubinuria occurs before jaundice

c- Will get protection if globulin is given within 24 hours after exposure to infection

d- Presence of transfer agent ………………. is diagnostic and means it will


become fulminant hepatitis

e- There is no chronic stage

Captopril is complicated by hypotension if given with:

a- Low salt diet

b- Give with Thiazide diuretic

c- Beta blocker
d- Give to 73 yrs old man

Dissecting aortic aneurysm of the descending aorta -

a- Atherosclerosis

b- May produce aortic incompetence

c- Can be diagnosed by ultra sound

d- Will need surgery if causes low C.O. symptoms

Short term treatment with high dose of corticosteroids can cause-

a- Cushing’s disease

b- Psychosis

c- Osteoporosis

d- Hypertension aippg.com

Which drug of the following need to be reduced or even contraindicated in renal


failure?

a- Nitrofuratoin

b- Gentamycin

c- Tetracyclines

d- Erythromycin

A child came to the casualty with hypertension, respiratory distress and oedema of the
lower limb. He had a bee sting in his leg, what would you give him?

a- IV fluids

b- Intravascular adrenaline
c- IV hydrocortisone

d- Antihistamines

The second heart sound is increasingly split in:

a- RBBB

b- ASD

c- PS

d- Constrictive pericarditis

Parathyroid adenoma can cause;

a- Renal stones

b- Hypercalcaemia

c- Hyperphosphaturia

d- Hypercalciuria

48 yrs old mother with two children who had common cold recently two weeks ago. She
became irritable, she has been complaining of difficulty during swallowing and sweating.
Investigations showed increased T4, increased T3 resin uptake, TSH normal, Radioiodine
nil (she had tender enlarge thyroid)

What is/are correct?

a- She does not have Grave’s disease

b- She is thyrotoxic

c- If you do radio active iodine uptake you will find it normal or low

d- She has including antithyroid antibodies


R.dass, Nov 1, 2007

#127

R.dass

R.dass Guest

Myxedema is usually associated with:

a- carpal tunnel syndrome

b-

c- Cerebrallar type ataxia

25 yrs old woman with Raynauld’s phenomena and dysphagia, She smokes 20
cigarettes a day and She’s got cough. Which of the following is/are likely to be found?

a- Subcutaneous calcification

b- She probably will develop arthritis if she is HLA 29

c- X-ray shows immobile oesophagus

d- Treat for chronic bronchitis

A female patient 30 yrs old with recurrent attacks of palpitation came to the casualty.
Her pulse was 170/min. The ECG showed normal QRS but P waves were absent. What
is/are the possible treatment?

a- IV verapamil

b- IV digitalis

c- Carotid sinus massage

d- Isoprenaline

aippg.com
Which of the following drug/disease combination is/are true?

a- tetracycline/Pyelonephritis

b- Caphalexin/Meningitis

c- Gentamycin/Lower lobe pneumonia

d- Chloramphenicol/Typhoid fever

A young patient presents with jaundice and his liver function test was abnormal. He also
has arthritis. What is the most likely cause?

a- hepatitis A

b- Hepatitis B

c- Hepatitis C

d- Hepatitis E

e- Infectious mononucleosis

Which of the following viral infection most likely become chronic?

a- Hepatitis A

b- Hepatitis B

c- Hepatitis C

d- Hepatitis E

What is cause for increased life expecting at birth of females than males in Australia?

a- Males exercise more than females

b- Genetic and biological differences

c- Females seek health care facilities more than males

d- Males die more in accidents and violence than the females


e- Employment stress is more for males

A 36 yrs old man who is a known ischeamic heart disease patient, complaints of recently
increased frequency of chest pain. This episode is prolonged chest pain. Which is true
regarding this patient?

a- Thrombolytic therapy is not indicated if ECG is normal

b- Heparin is not indicated if ECG and enzyme are normal

c- Repeat ECG is not indicated if not associated with pain

d- Stress test has to be done urgently

e- Serial enzyme assay has to be done even with chest pain

A 40 yrs old man, who is known hypertensive presents with severe chest pain radiating
to the back. His BP was 180/120. Chest x-ray showed mild cardiomegaly with widening of
mediastinum. He also had soft diastolic murmur along the left sternal border. ECG showed
acute inferior infarction. What is the appropriate management?

a- CTBG.

b- Thrombolys the patient

c- Give morphine, beta blocker and arrange for Trans oesophageal echocardiography

d- Give intravenous heparin

Commonest organism causing spreading cellulitis-

a- Group A streptococcus

b- Group B streptococcus

c- Gram negative bacilli

d- Staphylococcus
R.dass, Nov 1, 2007

#128

R.dass

R.dass Guest

A 22 yrs old man comes with jaundice; on examination he also has cervical
lymphadenopathy. His liver unction test was:

a- SGOT 450.

b- SGPT 400.

c- LKP 200.

d- CCT 250.

e- Bilirubin 80.

Peripheral blood showed atypical lymphocytes. What is the most likely diagnosis?

a- Hepatitis A

b- Hepatitis B

c- Infectious mononucleosis

d- Cytomegalo virus

e- Hepatitis C

What is the causative of herpes zoster?

a- HSV 1.

b- Varcola

b- HSV 2.

d- Varicella
e- EBW

A patient with WPW syndrome has broad complex tachycardia with fibrillation. What is
the management?

a- Digoxin

b- Verapamil

c- DC Cardioversion

d- Beta blocker

e- ACE inhibitors

A 69 yrs old man presents with temporary memory loss and blurring of vision for 10
minutes duration. Where is the lesion?

a- Vertebrobasillar insufficiency

b- Transient global amnesia

c- Carotid artery stenosis

d- Dementia

e- Middle cerebral artery thrombosis

aippg.com

A 65 yrs old man presents with confusion. His plasma sodium was 168-mmol and urine
osmolality 205 (50-200), what is the cause for his results?

a- Addison’s disease

b- Compulsive water drinking

c- Diabetes insipitus

d- SIADH

e- Congestive cardiac failure


Which of the following is not usually a presentation of Mycoplasma pneumonia?

a- Pleuritic chest pain

b- Fever

c- Headache

d- Severe cough

e- Cold

A 60 yr old lady, lost ten kg weight over the past 6 months, what is the least likely
cause?

a- Depression

b- Carcinoma of pancreas

c- Non-insulin dependant diabetes

d- Hypothyroidism

On examination of a 70 yr old lady there was generalized lymphadenopathy and


splenomegaly. Her only compliant was abdominal discomfort. What is the most likely
diagnosis?

a- Acute lymphoblastic leukemia

b- Acute myeloid leukemia

c- Hodgkin’s lymphoma

d- Infectious mononucleosis

All are side effect of corticosteroid except:

a- Hirsuitism
b- Lymphopenia

c- Osteomalacia

d- Leucocytosis

e- Cushing’s syndrome

R.dass, Nov 1, 2007

#129

R.dass

R.dass Guest

Regarding HIV which of the following is correct?

a- Usually develops after two years of infection

b- Most of the opportunistic infection occurs with generalized lymphadenopathy

c- Monthly pentamidine nebulisation reduces the incidence of opportunistic lung


infection

d- Antibodies will develop within a mean period of nine year after infection

In a suspected HIV patient, what is the advise you will give?

a- Usually seroconverts within 3 months

b- Should avoid sexual contact

c- chance of transplacental infection is 75%

d- 25% transplacental

e- 17% breast feeding


12 yr old school girl suddenly collapsed at school. She was brought by ambulance with
dextrose drip 60/ml min. On examination of her dolls eye reflex were present but she was
not responding to painful stimulus. Her vital signs as follows:

Resp. rate 14/min.

Pulse rate 50/min.

SaO2 100%

B/P 180/110(?)

If you are working in a tertiary hospital, what is the next step of management?

a- Arrange for urgent CT scan

b- Give dextrose infusion and start on saline

c- Give steroids

d- Intubate the patient

e- Neurosurgical reference

A hypertensive patient present with haematuria (Glomurular nephritis picture). On


investigation of urine, which of the following will confirm the diagnosis?

a- Erythrocyte cast

b- Granular cast

c- Hyaline cast

The diagnosis of chronic renal failure is confirmed by presence of

a- Haematuria

b- Small size kidney

c- hypertension aippg.com
d- Proteinuria

e- Anaemia

A 48 yr old female, who is smoker complaints of dysponea. She has a past history of
exposure to asbestosis ten years ago. On investigation her total lung capacity was 145% of
normal expectancy. Diffusion capacity was 25%. On examination, she had widespread
wheeze over lung field. What is the diagnosis?

a- Chronic bronchitis

b- Emphysema

c- Asbestosis

d- Asthma

e- Carcinoma of bronchus

A patient present with dyspnoea. On examination, percussion note was dull on the right
lower lobe, and bronchial breath sound on the mid zone. What is the most likely cause?

a- Rt. lower lobe consolidation

b- Rt. pleural effusion

c- Rt. lower lobe collapse

d- Lt. side pneumothorax

Pulmonary embolism is associated with all of the following except-

a- Opening snap

b- Pre systolic crescendo murmur

c- Presence of S3

d- Loud S1

e- Atrial fibrillation
Which of the following statement is correct regarding his management?

a- No antibiotic prophylaxis before tooth extraction

b- Warferin is only indicated if patient has TIA and atrial fibrillation

c- Mitral valvulolasty has to be done

d- Captopril should be given

About keratoacanthoma, which is correct?

a- Slow growing tumour with spontaneous resolution

b- Fast growing tumour with spontaneous resolution

c- Never undergo spontaneous resolution

R.dass, Nov 1, 2007

#130

R.dass

R.dass Guest

Which of the correct combination of antibodies with disease?

a- Anti smooth muscle antibody - primary biliary cirrhosis

b- Anti mitochondrial antibody - Chronic active hepatitis

c- Anti ribonuclear protein - Sjogren’s syndrome

d- Anti acetylcholine receptor antibody - Myasthenia gravis

e- Anti nuclear antibody - Thyroiditis


A chronic inflammatory arthritis patient presents with anemia, and investigation
showed;

Hb 10gms%

Sr.Fe 6 (12-15)

TIBC less than normal

MCV 80 (80-90)

Ferritin 820 (very high)

Transferrin - Low normal range

What is the most likely cause for this patient’s anemia?

a- Anemia of chronic disease

b- He is probably iron deficient

c- Megaloblastic anemia

d- Hemochromatosis

A man presents with pain on the right knee for the past two days duration. On
aspiration of the knee the fluid was cloudy. Microscopic examination showed polymorphs
500/mm cube, No organism seen. What s the probable diagnosis?

a- Rheumatoid arthritis

b- Septic arthritis

c- Reiter’s disease aippg.com

d- Ankylosing spondylitis

Anterior uveitis is commonly found in:

a- Reiter’s syndrome
b- Sjogren’s syndrome

c- Ankylosing spondylitis

d- Rheumatoid arthritis

Which of the following is not typical of duodenal ulcer?

a- Nocturnal pain

b- Pain relived with antacids

c- Hunger pain

d- Loss of appetite

e- Epigastric pain

A female patient has reflex oesophagitis, not responding to H2 receptor antagonist. She
also complaints that her fingers are changing color when immersed in cold water, what is
your next step of management?

a- Ranitidine

b- Fundoplication

c- Omeprazole

d- Pneumatic dilatation of esophagus

e- Octreotide

Regarding Paget’s disease what is true?

a- hypercalciuria is always present

b- Alkaline phosphatase is usually elevated

c- Osteosarcoma is a common complication


Which of the least appropriate statement about chronic asthma?

a- Will not usually present after fifty years of age

b- Hypersensitivity to bronchial stimulus

R.dass, Nov 1, 2007

#131

R.dass

R.dass Guest

A 46 yrs old female present with pain and proptosis of her left eye. What is the probable
diagnosis?

a- Cluster headache

b- Sinusitis

c- Retroorbital tunnel

d- Temporal

A.T. scan result from:

a- Post streptococcal

b- Morphine’s syndrome

c- Ankylosing spondylitis

d- Prolonged steroid therapy

In hepatic encephalopathy which are not correct?

a- Portocaval anastomosis improves the condition

b- We give CHO liberally


c- High protein diet is recommended

d- Constipation does not aggravate the condition

The forth heart sound is heard in;

a- myocardial infarction

b- M.S. severe

c- Severe A.S.

d- T.S.

Coronary insufficiency;

a- aortitis can be a cause

b- Dissecting aneurysm can be a cause

c- Aortic regurge

d- M.S.

Analgesic nephropathy;

a- Haematuria and renal colic

b- The main pathology is peritubular necrosis

c- Papillary necrosis can occur and leads to abstruction

d- Can present with sterile pyuria

aippg.com

A lady presents with headache that occurs premenstrual, and the pain starts from the
occipital spreads forwards the left frontal region. The pain is aggravated on walking and
she is irritable, (photophobia) is present. What is the most likely cause?

a- Tension headache
b- Premenstrual tension

c- Migraine without aura

d- Cluster headache

e- Temporal arteritis

A patient presents with fourth attack of gout with pain in the knee joint. What is the
management of this patient?

a- Paracetamol

b- Probenecid

c- Indomethacin

d- Allopurinol

What is the true about heparin-induced thrombocytopenia?

a- bleeding

b- Thrombosis

c- Neutropenia

d- Eosophilin

All are true regarding hemolytic anemia except;

a- Increased iron absorption

b- Reduced MCV

c- Increased urobilinogen

R.dass, Nov 1, 2007

#132
R.dass

R.dass Guest

Which of the following drug will not increase the pressure gradient in hypertrophy
obstructive cardiomyopathy?

a- Digoxin

b- ACE inhibitors

c- Diuretics

d- Verapamil

e- Vasodilators

Reducing of hypertension is most important in which of the following?

a- Coronary heart disease

b- Ischemic stroke

A 65 yrs old man presents with sudden weakness of Lt. Upper limb. He almost
recovered completely from it. On examination you find only mild weakness of the limb.
Which of the following would be cause for his symptoms?

a- Rt. sided lacunar infarct

b- Lt. sided lacunar infarct

c- Middle cerebral artery infarct

In polycythemia, which is not commonly found?

a- Reduced amount of Leucocytes

b- Bleeding
c- Thrombosis

d- Increased platelet

e- Splenomegaly

Which of the following is more suggestive of tuberculous pleural effusion?

a- Increase lymphocytes

b- Decreased glucose

c- Increased protein

d- Severely blood stained

aippg.com

The features of lower motor neurone lesion are;

a- Muscle weakness, wasting, fasciculation, hyper reflexia

b- Muscle weakness, wasting, hypotonia, fasciculation and decreased reflex

c- Hypotonia, increased muscle mass, fasciculation and decreased reflex

d- Muscle weakness, rigidity and increased reflex and absent abdominal reflex

A 58 yrs old man accompanied by his wife to the hospital with bruises seen on his ---.
She complains that her husband was binge drinking for one week. On examination he was
confused and had extensor planter reflex. What is the probably diagnosis?

a- Hepatic encephalopathy

b- Korsakov’s psychosis

c- Central pontine myelinosis

d- Subdural hematoma

e- Dementia
IgA nephropathy - what is the usual presentation?

a- Hematuria, rashes over the buttocks and legs

R.dass, Nov 1, 2007

#133

R.dass

R.dass Guest

During normal inspiration -

a- Right atrial pressure increases

b- Pulmonary venous flow increases

c- Systemic arterial pressure increases

d- Heart rate decreases

e- Splitting of the second heart sound is increased

The following statements are true -

a- The normal pulmonary artery systolic pressure is 40-60mm Hg

b- A pulmonary capillary wedge pressure greater than 25 mm Hg is usually associated


with pulmonary oedema

c- The hypotension associated with isolated right ventricular infarction is due to a low
left atrial pressure and usually responds to fluid challenge

d- in constrictive pericarditis the left and right ventricular diastolic pressures equalize

e- Following an anterior myocardial infarction cardiogenic shock (BP 80/50) associated


with a pulmonary wedge pressure of 20mm Hg is best treated with inotropic support
The following statements are correct for cardiac muscle -

a- Cardiac muscle differs from smooth and striated muscle because of its inherent
rhythmicity

b- Depolarisation of cardiac muscle is the result of an initial calcium influx

c- Repolarisation of cardiac muscle is the result of potassium efflux

d- Calcium ions are required for electromechanical coupling of the cardiac myosite
aippg.com

e- The energy for contraction of the cardiac, myosite is provided as adenosine


diphosphate

On examination of the jugular venous pressure -

a- With the patient at 45 degree the jugular venous pressure usually lies up to 3 cm
above the sternal angle

b- The Y descent of the venous pressure s associated with right ventricular filling

c- In tricuspid regurgitating the V wave is more prominent but similar in timing to that in
the normal subject

d- Cannon waves may be present with VVI pacing

e- Inspiration causes a rise in the venous pressure in the presence of cardiac tamponade

Endothelial derived relaxation factor (nitric oxide) -

a- Reduces platelet adhesion and aggregation

b- is released during forearm ischaemia and is responsible for flow dependent


vasodilatation

c- is responsible for coronary vasodilatation in response to acetylcholine

d- Acts as a potent vasoconstrictor at high concentrations

e- Promotes mitogenesis and proliferation of smooth muscle cells


A significant reduction in systemic arterial pressure is characteristically observed during
inspiration in the following conditions -

a- Severe asthma attack

b- Positive pressure ventilation

c- Aortic dissection causing aortic regurgitation and cardiac tamponade

d- Massive pulmonary embolism

e- Massive haemorrhage

The following statements are correct for mitral stenosis -

a- The later the opening snap the more severe the stenosis

b- It is approximately equally common in men and women

c- A loud first heart sound suggests a rigid or calcified valve

d- Presystolic accentuation of the diastolic murmur occurs if the patient is in atrial


fibrillation

e- The Graham Steell murmur is due to associated aortic incompetence

The following conditions are associated with disturbances of lipid metabolism that give
rise to an increased risk of coronary artery disease -

a- Familial hyperalphalipoproteinaemia

b- Familial mixed Hyperlipidaemia

c- Hormone replacement therapy in post-menopausal women

d- Heterozygous familial hypercholesterolaemia

e- Hypothyroidism
During the normal cardiac cycle -

a- The aortic and pulmonary valves close synchronously

b- The period of left ventricular isovolumic contraction occurs between the first heart
sound and the onset of the carotid upstroke

c- The V wave of the jugular venous pulse coincides with the carotid pulse

d- The interventricular septum depolarizes from right to left

e- Right ventricular systole is prolonged during expiration

The following agents have been shown to reduce mortality significantly after myocardial
infarction -

a- Captopril

b- Nifedipine

c- Oral nitrates

d- Timolol

e- Simvastatin

R.dass, Nov 1, 2007

#134

R.dass

R.dass Guest

In Fallot’s tetralogy -

a- Right to left shunting is present, but cyanosis may not be detected at birth

b- The second heart sound is widely split

c- The chest x-ray shows plethoric lung fields


d- Paradoxical emboli occur because of right to left shunting of blood in the heart

e- Pulmonary hypertension develops in early life

The following conditions are associated with false positive responses in exercise
tolerance testing -

a- Hypertension

b- Aortic valve disease

c- Left bundle branch block

d- Hypertrophic cardiomyopathy

e- Digoxin therapy

Coarctation of the aorta is -aippg.com

a- Usually congenital but may be acquired

b- Recognised by absent or delayed femoral artery pulses

c- A common cause of heart failure in infancy but an uncommon cause of hypertension


in adults

d- Associated with an increased incidence of bicuspid aortic valve

e- A cause of left to right shunting of blood

In non-rheumatic atrial fibrillation -

a- The risk of embolic stroke is increased 5-fold

b- The risk of stroke is less than in atrial fibrillation due to rheumatic valve disease

c- Excess alcohol intake is an unlikely cause

d- Cardioversion may be performed without anticoagulation if a transthoracic


echocardiogram is normal
e- Paroxysmal atrial fibrillation is associated with a lower risk of stroke

The following statements are correct for Starling’s law -

a- The force of contraction is inversely proportional to the initial length of the cardiac
muscle fibre

b- The law is only true for isolated heart preparations

c- The law means that stroke volume is proportional to left ventricular end diastolic
volume unless the inotropic state of the myocardium or vascular resistance change

d- As a consequence of the law, factors which reduce cardiac filling will increase
contractility

e- The law is dependent on the integrity of the cardiac stretch receptors

A broad complex tachycardia is more likely to be Supraventricular tachycardia with


aberrant conduction than ventricular tachycardia if -

a- Cannon waves are seen in the neck veins

b- Fusion beats are seen on the electrocardiogram

c- Atrioventricular dissociation is seen on the electrocardiogram

d- The QRS complexes are morphologically identical to a bundle branch block pattern
seen when the patient is in sinus rhythm

e- The tachycardia is abolished by carotid sinus massage

During pregnancy -

a- Cardiac output increases

b- A cardiomyopathy may occur

c- New cardiac murmurs are usually innocent

d- Use of anticoagulants is a more difficult problem than in non-pregnant women


e- Pulmonary oedema due to mitral stenosis is characteristically treated by valvotomy

On standing from the supine position -

a- Compensatory changes in the circulation are the result of the stimulation of the
carotid sinus and aortic baroreceptors by a reduced blood pressure

b- Reduced cerebral blood flow causes an increase in cerebral PCO2 and a decrease in
cerebral PO2

c- Total peripheral vascular resistance decreases

d- Cardiac output increases

e- Circulating Angiotensin II decreases

Amiodarone -

a- Causes nausea in about 50% of patients with congestive cardiac failure

b- Causes hypothyroidism or hyperthyroidism in 3-5% of patients

c- Prolongs the action potential duration

d- is preferable to flecainide in the treatment of ventricular tachycardia in patients with


ischaemic heart disease and impaired left ventricular function

e- has a half life of about 10 days

The following statements about the Valsalva manoeuvre are correct -

a- The manoeuvre is performed by straining to inhale against a closed glottis

b- Initially blood pressure increases transiently, then falls because venous return is
impaired

c- A tachycardia is present whilst straining but a Bradycardia occurs after release of the
manoeuvre

d- The heart rate and blood pressure responses are abolished by sympathectomy
e- The cardiovascular responses to the manoeuvre may be impaired in diabetes

R.dass, Nov 1, 2007

#135

R.dass

R.dass Guest

In congestive cardiac failure -

a- ACE inhibitors reduce mortality by up to 40%

b- Digoxin may result in symptomatic improvement in patients in sinus rhythm with a


dilated left ventricle and third heart sound

c- Beta blockade may result in symptomatic improvement in cases of dilated


cardiomyopathy

d- A third sound is associated with ventricular filling

e- Treatment with a combination of nitrates and hydralazine may prolong survival

In the new born infant the following circulatory changes occur -

a- Peripheral vascular resistance rises

b- Pulmonary vascular resistance rises

c- Blood flow across the forearm ovale reverses

d- Blood flow across the ductus arteriosus is initially reduced and reversed immediately
after birth before ceasing entirely in a further day or two

e- Superior vena caval blood flow exceeds inferior vena caval blood flow for the first
time

Atrial myxomas -
a- are more common in the right atrium than in the left atrium

b- do not recur after resection

c- may mimic infective endocarditis

d- are poorly visualized on echocardiography

e- usually arise from a pedicle near to the fossa ovalis

aippg.com

Prinzmetal’s or variant angina is characterized by -

a- Evidence of proximal coronary artery disease in over 50% of cases

b- Marked ST elevation with increased R wave amplitude during anginal attacks

c- Ventricular dysrhythmias during pain

d- Pain at rest or at night

e- A good response to sublingual nitrates

The following metabolic and neurohumoral effects are observed in patients with
congestive cardiac failure -

a- Compensatory reduction on basal metabolic rate

b- Inhibition of aldosterone secretion because of sodium retention

c- Polycythaemia

d- Increased circulating renin concentrations

e- Increased responsiveness of the heart to circulating catecholamines

After splenectomy -

a- Howell-jolly bodies are found in the red cells

b- There is an exaggerated neutrophil and platelet response to inflammation

c- Penicillin prophylaxis against staphylococcal infection should be continued for life


d- Lymphopenia is found

e- Meningococcal vaccine should be administered to patients visiting equatorial Africa

The following are features of myelodysplastic syndrome -

a- Increased numbers of blasts in the bone marrow

b- Atypical monocytes in the peripheral blood

c- Transfusion-dependent anaemia

d- Transformation into treatment-resistant acute myeloid leukaemia (AML)

e- Treatment with busulphan may be effective

In acute leukaemia -

a- The presence of Auer rods confirms the diagnosis of acute lymphoblastic leukaemia
(ALL)

b- The presence of the Philadelphia chromosome confers a good prognosis

c- Disseminated intravascular coagulation (DIC) is common in acute promyelocytic


leukaemia (AML-M3)

d- CNS involvement is common in acute lymphoblastic leukaemia

e- Hypokalaemia in acute monocytic leukaemia is due to renal tubular damage by


lysozyme

Fragmented red cells on the blood film -

a- are found in disseminated intravascular coagulation

b- in thrombotic thrombocytopenic purpura are due to damage to red cells by


intravascular fibrin

c- are found in march haemoglobinuria

d- are usually associated with Polychromasia


e- are associated with increased serum haptoglobins

The following are features of iron-deficiency anaemia -

a- Reduced bone marrow stainable iron

b- reduced serum iron and iron binding capacity (TIBC)

c- a low MCV, MCH and MCHC

d- Target cells and pencil cells in the peripheral blood

e- Correction can almost always be achieved with simple oral iron preparations

R.dass, Nov 1, 2007

#136

R.dass

R.dass Guest

Beta-thalassaemia major -

a- May present with severe anaemia between the ages of 5 and 10 yrs

b- commonly presents with neonatal jaundice

c- can be diagnosed antenatally by Hb electrophoresis on fetal red cells

d- is associated with atrophy of the spleen

e- should be treated with regular oral desferrioxamine to reduce iron overload

Fresh frozen plasma (FFP) -

a- is available in the UK in 300 ml aliquots

b- Is the optional replacement fluid after major burns


c- two units should be given for every four units of SAG-M blood transfused in an
operative situation

d- Contains blood group antibodies

e- is approved for the urgent correction of over-anticoagulation with warferin


aippg.com

A prolonged thrombin clotting time may occur in -

a- warferin therapy

b- prothrombin deficiency

c- disseminated intravascular coagulation (DIC)

d- dysfibrinogenaemia

e- liver disease

The following are recognised to be associated with an inherited tendency to venous


thrombo-embolism -

a- the lupus anticoagulant

b- abnormal structure of factor V causing resistance to degradation by activated protein


C

c- factor IX deficiency

d- anti-thrombin deficiency

e- protein S deficiency

Heparin treatment for pulmonary embolus -

a- should be commenced as a constant intravenous infusion os 40,000 u/24 hours

b- prolongs the prothrombin time (PT)

c- may cause a fall in platelet count


d- may give rise to Osteomalacia

e- if ‘overload’, may be reversed by quinine suphate

Cold agglutinins -

a- cause abnormal red cell indices when blood is processed at room temperature

b- cause rouleaux on the blood film

c- most commonly have anti-Rhesus specificity

d- are associated with lympho-proliferative disorders

e- when associated with haemolysis transfusion of packed red cells are the mainstay of
treatment

Lymphocytosis -

a- in chronic lymphocytic leukaemia (CLL) may be due to T-cell proliferation

b- in pertussis morphologically resembles CLL

c- Presenting in association with skin rashes is frequently of T-cell origin

d- in glandular fever resembles that of CLL

e- Is a racial variant occurring not uncommonly in people of African origin

In a patient with macrocytosis -

a- the presence of target cells is in keeping with a diagnosis of liver disease

b- the finding of Polychromasia should prompt a reticulocyte count

c- the finding of neutrophil hypersegmentation should prompt measurement of serum


vitamin B12

d- the finding is normal if the patient is a neonate


e- the presence of a monocytosis is in keeping with a diagnosis of myeloproliferative
disorder

In blood coagulation -

a- the intrinsic pathway proceeds more rapidly than the extrinsic one

b- the intrinsic system is activated by blood coming into contact with a non-
endothelialised surface e.g. subendothelium

c- calcium ions are required for the intrinsic, but not the extrinsic pathway to proceed

d- platelets play a part in the normal coagulation cascade

e- when the coagulation mechanisms are working normally, blood may take more than
an hour to clot in a dry, plastic tube

The following statements are true about factor VIII -

a- it consists of two parts of approximately equal size

b- synthesis of factor VIIIc and VIII von Willebrand factor (VWF) is controlled by sex-
linked (X chromosomes) genes

c- synthesis of VIIIc and VWF takes place chiefly in the liver

d- in von Willebrand’s disease, there is a reduction in both VIIIc and VIII VWF

e- a factor VIIIc level of 10% (normal range 50-150%) is associated with severe bleeding
diathesis

R.dass, Nov 1, 2007

#137

R.dass

R.dass Guest
In transmitted cytomegalovirus (CMV) infection -

a- the risk of cytomegalovirus transmission can be reduced by leucocyte-depleting blood


filters

b- more than a third of blood donors in the UK are CMV antibody positive

c- premature neonates should receive blood from CMV-negative donors

d- acyclovir is an effective treatment for CMV pneumonitis after bone marrow


transplant

e- measurement of IgM CMV antibodies is the earliest method of diagnosing infection

In a patient with paroxysmal nocturnal haemoglobinuria (PNH) -

a- iron may precipitate acute hemolysis

b- aplastic anaemia is a recognised complication

c- chronic granulocytic leukaemia may supervene

d- the Hess test is commonly abnormal

e- Splenomegaly is commonly present

Cryoprecipitate -

a- is prepared by thawing fresh frozen plasma

b- is free of risk of hepatitis infection

c- is rich in fibrinogen

d- may reverse the platelet defect in Uraemic patients

e- is of therapeutic valve in haemophilia B

aippg.com

Granulocyte colony stimulating factor (G-CSF) -


a- achieves its best physiological effect by intravenous bolus administration rather then
subcutaneous injection

b- administration during chemotherapy improves the effectiveness of cytotoxics against


carcinoma cells

c- can stimulate the proliferation of myeloblasts in acute myeloid leukaemia

d- bone pain as a recognised side-effect

e- increases the number of CD34 positive stem cells in the blood in normal people

In aplastic anaemia -

a- a history of preceding hepatitis A is associated with a relatively benign course

b- immunosuppressive therapy may be effective

c- a leuko-erythroblastic blood picture may be present at diagnosis

d- ferrokinetic studies show predominant accumulation of isotope in the liver

e- a positive Ham’s test may be present

A patient with a first pulmonary embolus has been on continuous i.v. heparin for five
days and has received one dose of 10mg warferin 12 hours before blood is drawn for
coagulation tests: these show an activated partial thromboplastin time (APTT) ratio of 1.7,
and an international normalized prothrombin ratio (INR) of 1.8 -

a- the heparin should be increased

b- the INR result shows the effect of over-heparinisation

c- the patient is unusually sensitive to the effect of warferin

d- the aimed for INR range in this patient is 3-4.5

e- the INR is the patient’s prothrombin time in seconds divided by the normal
control time in seconds
Dietary iron -

a- in a normal diet amounts to about 3 mg/day

b- absorption is reduced by vitamin C

c- absorption is reduced by phosphates

d- is about 90% excreted in health

e- is absorbed in the ferric (Fe 3+)

Increased plasma erythropoietin occurs in -

a- Aplastic anaemia

b- polycythaemia rubra vera (PRV)

c- renal failure

d- massive obesity

e- Hepatoma

A 54 yrs old man has WBC 5.4X 109/l, HB 10.4 g/dl, platelets 135 X 109/l. Differential
count shows: neutrophils 87%, lymphocytes 6%, monocytes 3%, metamyelocytes 3%,
promyelocytes 1%, and nucleated red cells 3/100 white cells. The following are
appropriate investigations to elucidate the cause of the abnormal blood picture -

a- Chest x-ray

b- haemoglobin electrophoresis

c- fasting cholesterol

d- bone marrow aspirate

e- HLA typing

Regarding human blood groups -


a- naturally occurring anti-A and anti B antibodies can be IgM and IgG

b- most people secrete their A or B blood group antigens in saliva

c- The ABO type of an individual can ‘change’ during some illness e.g. acute
leukaemia

d- Group O is the commonest blood in all racial groups

e- Lack of certain blood group antigens protects against Plasmodium vivax

A healthy blood donor with a haemoglobin of 14 g/dl is found to have a positive direct
antiglobulin (Coombs’) test. The following are appropriate investigations -

a- reticulocyte count

b- anti-nuclear factor

c- physical examination for enlarged lymph nodes

d- Drug history

e- Barium enema

R.dass, Nov 1, 2007

#138

R.dass

R.dass Guest

Usually the first sign of salicylate poisoning in children is -

a- Delirium

b- Coma

c- Hyperventilation

d- Hyperpyrexia
e- Convulsion

Which of the following is not associated with neonatal hyperbilirubinaemia?

a- Fetal rubella infection

b- ABO incompatibility

c- Malaena

d- Hereditary spherocytosis

e- Bruising

The commonest of anemia in an infant aged 12 months is -

a- Bleeding from Meckel’s diverticulum

b- Recurrent bleeding from the nose

c- Fibrocystic disease of the pancreas

d- Diet only of milk

e- Folate deficiency aippg.com

A 5 yrs child presents with an acute painful leg left and temperature of 39.5 C. WOF is
the most likely cause?

a- Scurvy

b- Rickets

c- Synovitis of the hip

d- Osteomyelitis

e- Perthe’s disease
A boy aged 8 yrs presents with an acute attack of wheezing. He has a past history of
recurrent episodes of wheezing since the age of 5 yrs and has been well in between
attacks. O/E he has minimal thoracic movement with hyperinflation. Breath sounds and
sibilant rhonchi are barely audible on auscultation. He is cyanosed even on 40% O2.

Which is the most likely diagnosis?

a- Bronchlolitis

b- Inhaled foreign body

c- Cystic fibrosis

d- Asthma

e- Bronchlectasis

Each of the following statements is correct except-

a- Malabsorption in cystic fibrosis is predominantly due to impaired pancreatic exocrine


function

b- Malabsorption of fat predispose to rickets

c- Disaccharides deficiency is a complication of gastroenteritis

d- Steatorrhoea may occur with heavy infestation of Giardia lamblia

e- Patients with coeliac disease usually have fat globules in the stool

A child aged two years whose brother has just recovered from gastro-enteritis has
complained of abdominal pain for 18 hours and has been feverish and irritable. He has
vomited twice since the pain commenced but has not had a bowel action. O/E his
temperature is 38 C. He resists examination of the abdomen and has tenderness and
guarding of the lower abdomen without associated distension. His management should be
-

a- Erect x-ray examination of the abdomen

b- Oral antipyretic

c- Laparotomy
d- White cell count and review

e- Oral Metronidazol

A 7 yr boy complaining of one sided faeculant nasal discharge. The most common cause
is -

a- Foreign body

b- Sinusitis

c- Cold

The commonest cause of Anorectal bleeding in children is -

a- Ulcerative colitis

b- Rectal polyps

c- Intussusception

d- Fissure in ano

e- Peptic ulcer

Which of the following are helpful in the treatment of Atopic eczema, except?

a- Avoidance of wool garments

b- Antibiotics

c- Moisturizing cream

d- Sea-water bath

e- Oral corticosteroids

R.dass, Nov 1, 2007

#139
R.dass

R.dass Guest

What is the most common presentation of chronic renal failure in childhood?

a- Polyuria

b- Increased creatinine clearance

c- Decreased uric acid level in serum

d- Hyperglycaemia

3 yr child was brought to you by his mother with bow legs. O/E there is tibial torsion,
distance between knees is 3 inches. Your action is-

a- Popliteal splints

b- Osteotomy bilateral

c- Vitamin D

d- Reassure the mother

e- Weighing of the outer soles of the shoes

Which newborn is more prone to develop kernicterus, Except?

a- High titre conjugated bilirubin

b- Diabetic mother

c- Premature aippg.com

d- Hypothyroid

e- Small for dates


In patient with asthma severe? The best method of assessing the severity of the disease
would be-

a- Arterial blood gases analysis

b- FEVI

c- The severity of cyanosis

d- The severity of the dyspnoea

e- Tachycardia

Teratoma of the testis characteristically metastasis to-

a- Para-aortic nodes

b- Inguinal nodes

c- Kidneys

d- Vertebral

e- All of the above

A patient diagnosed as craniopharyngioma and tumour mass is 10mm in CT scan. The


best treatment should be -

a- Tamoxifen

b- Bromocryptine

c- Surgery trans-spnoncidal approach

d- Surgery Transfrontal approach

A young man with a history f frequent sore throat with a puffy face, oliguria, blood
pressure reading of 160/120 and pulmonary crepitations. WOF is correct?

a- Haematuria is a grave prognostic sign


b- His main immediate risk is of death from left ventricular failure

c- Increase in oral fluids will result in a diuresis

d- If there is renal tenderness a renal biopsy should be undertaken

e- Dialysis is contra-indicated during the acute phase of the illness

All of the following are lesions in basal ganglions except?

a- Tremor

b- Dystonia

c- Ataxia

d- Chorea

e- Rigidity

WOF statements concerning. Parkinson’s disease is not true -

a- It frequently commences on one side

b- It flexed pasture is characteristic

c- It causes falls without loss of consciousness

d- It causes involuntary stopping whist walking

e- The tremor persists during sleep

Regarding immunology, WOF statement is correct?

a- Killer cells are T-subsets and kill tumour cells

b- B cells produce first IgM and later IgG

c- IgM need the help of complement

d- IgG increases with infectious mononucleosis


e- Staphylococcus aureus needs opsonization

R.dass, Nov 1, 2007

#140

R.dass

R.dass Guest

The valve of giving anti D gamma globulin to an Rh -ve mother is -

a- Elimination of the fetal RBC from the maternal stream

b- Coating the fetal red blood cells thus preventing to maternal

c- Immunological reaction

d- Stimulate the immune response of the mother to produce antibodies against the
fetal blood cells

e- Inhibiting the immune response of the mother to the fetal red blood cells

A 6 yr old boy thought always to be healthy is found to have a loud parsystolic murmur
at the lower left sternal edge. He is not cyanosed. The apex occurs left ventricular and
pulmonary second sound is normal. The ECG shows left ventricular hypertrophy and the
chest x-ray shows mild cardiomegaly and pulmonary plethoric. The most likely diagnosis
is-

a- ASD

b- VSD

c- MI

d- Co-arotaxia of aora

e- Cardiomyopathy aippg.com
WOF statements is true of cerebral palsy?

a- It is present before birth

b- It is a static instead of progressive lesion

c- It is a psychomotor disorder of obscure cause

d- It is an inherited disease

e- it is synonymous with rhethosis

Which of the following will indicate developmental delay?

a- Smiles for the first time at 8 weeks

b- Sits up unsupported at 8 weeks

c- Walks for the first time at 16 months

d- Says about 3 words at 16 months

e- Has day and night urinary control at 5 years

Purulent vaginal discharge Except-

a- Pyometra

b- Monilial vaginitis

c- CIN III

d- Decubitus ulcer

In patients with pernicious anaemia, optimal therapy with vitamin B12 will correct-

a- Gastric acidity to normal levels

b- Thrombocytopenia

c- Malabsorption of vitamin B12 measured by the Schilling test


d- Hyper segmentation of the neutrophils

e- Early and mild peripheral neuropathy

Deficiency in boys significantly associated with -

a- Parental criminality

b- Low average IQ

c- Large family size

d- Inconsistent harsh paternal discipline

e- Lower socio-economic class

In regard to chronic venous disease which of the following comments is/are true?

a- In primary varicose veins, the prevalence of sapheno femoral incompetence is higher


than that of sapheno-popliteal incompetence

b- When sapheno-femoral and sapheno-papliteal incompetence occurs in the patient it


is most likely to be an acquired problem

c- Chronic venous disease due to the post-phlebitis (post-thrombolic) syndrome is often


cured by venous ligation and excisional surgery

d- Venous ulceration is only associated with the post phlebitis (post thrombotic)
syndrome

e- Graduated compression stocking produce an equivalent effect to surgery for both the
symptoms and complications of chronic venous disease

Deep sea fish -

a- Protect against coronary heart disease whet, consumed in quantities at 30g or more
twice per week

b- Contain high levels at n-3 fatty acids


c- When consumed in amounts greater than 400g per week does not affect the skin
bleeding time

d- Is a poor source of protein compared to meat

e- Is useful for treating high plasma cholesterol levels

A 32 yr old woman presents with severe menorrhagia. She wishes to have pregnancies
in the future. Her uterus is enlarged to a size which is equivalent to a ten week pregnancy.
Ultrasonography confirms that this is due to fibromyomata. Which of the following
therapies may be appropriate in her management?

a- Gonadotrophin releasing hormone analogue therapy

b- Vaginal hysterectomy

c- Hysteroscopic endometrial resection

d- Myomectomy

e- Oral oestrogen supplementation

R.dass, Nov 1, 2007

#141

R.dass

R.dass Guest

A 3 yr old child was brought with a history of unilateral nasal discharge and halitosis of 3
days duration. What is your most probable diagnosis?

a- Sinusitis

b- Insilitis

c- FB in the nose

d- Otitis media
e- Gingivostematitis

A mother brought her 6 yr old child to you saying that it was not doing very well in
school. IQ was 80. No other abnormal find. What is your advice to the mother?

a- Send the child to a school for the severely disabled and mental retardet

b- Ask the teacher to give extra lesion to the child after school hours

c- Refer to a psychiatrist

d- Reassure the mother that he may

A six year old child developed oedema of feet and periorbital oedema. The next day he
had vomiting and diarrhoea and developed joint pains. The next day he got a purpuric
rash on the past aspect of him thigh and calf and Gluteal regular. What is he most
appropriate investigation would perform in this child? aippg.com

a- Urine microscopy and culture

b- Xray the joints

c- Stool examination

d- Estimation of immunoglobulins in the serum

A 4 yr old child was brought to you with a smooth round Huctuant swelling at the
junction of the middle and the lower third of the Rt. sternomastoid muscle. It was a
subcutaneous swelling. What is your approach?

a- Hodgkin’s lymphoma

b- Take a chest x-ray

c- Lymph node enlargement

d- Do aspiration of the controls and send for pathology

e- Excision physiotherapy
An elderly woman can read the newspaper, but has halos in bright sunlight?

a- Glaucoma

b- Cataract

c- Presbyopia

d- Macular degeneration

Which of the following drugs decreases renin?

a- Beta blockers

b- ACE inhibitors

c- Spironolactone

d- Hydralazine

e- Centrally acting antihypertensives

Newborn with respiratory distress, faint breath sounds on the left and with a scaphoid
abdomen -

a- Meconium aspiration syndrome

b- Situs inversus

c- Diaphragmatic hernia

Which nerve gives the sensation of taste to the anterior 2/3rd of tongue?

a- Trigeminal nerve

b- Facial nerve

c- Hypoglossal nerve

d- Glossopharyngeal nerve
A patient who underwent a cholecystectomy now comes with jaundice. What is the
investigation of choice?

a- Oral cholecystogram

b- CT scan

c- Ultrasound

d- Sr. creatinine

e- Upper GI studies

R.dass, Nov 1, 2007

#142

R.dass

R.dass Guest

All of the following are true regarding a pseudobulbar palsy except -

a- Wasting and fasciculation of tongue

b- Jaw jerk decreased

c- Loss of sphincter control

d- Gag reflux present

Regarding ACE inhibitors which is not true?

a- Used in the treatment of heart failure

b- Used as first line in the treatment of Hypertension

c- Used in Diabetics

d- Cannot be used in the treatment of Aortic stenosis


Regarding CRF and Calcium metabolism -

a- Ca is increased

b- Ca is decreased

c- There is no relation between CRF and Ca

d- Causes Osteomalacia aippg.com

Most significant complication of massive blood transfusion?

a- Pulmonary oedema

b- Change in acid base balance

c- DIC and coagulation defect

d- Increased IVP

e- No such complications

Regarding CRF and potassium all of the following reduce potassium except (treatment
of hyperkalemia)-

a- Calcium carbonate

b- Glucose and insulin

c- Dialysis

d- Resonium

e- Sodium Bicarbonates

55 yr old patient with dysphagia for solids with a previous history of reflux-

a- carcinoma oesophagus

b- Stricture
c- Scleroderma

d- Achalasia

e- Raynaud’s

An infant came with pneumonia, X-ray showed consolidation of a lobe with round
translucencies and a small pleural effusion. What is the treatment of choice?

a- Crystalline penicillin

b- Flucloxicillin

c- Amoxicillin / clavulanic acid

d- Tetracycline

A new born was peripherally cyanosed and crying lusting. The axillary temperature
recorded was 37.2 degrees what would be the next step?

a- Take an xray

b- Reassure

c- Oxygen

d- Urine culture

34 yrs old lady on phenytoin wants to take OCP’S what can be prescribed-

a- Microgynon 30

b- Microgynon 50

c- Triphasic

d- Oetradiol patches

e- Progesterone only oil


6 months old boy brought by his mother with a temperature of 38.9degrees with
bilateral wheezing his resp. rate was 36/min. other members in the family had a h/o
similar illness. There is family h/o asthma. What is the diagnosis?

a- Asthma

b- Foreign body

c- Bronchiolitis

d- Pneumonia

R.dass, Nov 1, 2007

#143

R.dass

R.dass Guest

16 weeks pregnant lady came for a check up, for the diagnosis of foetal anencephaly all
are true except -

a- Increased alpha feto protein

b- Increased beta HCG

c- Nuchal thickness

d- Decreased alpha feto protein

Regarding tubal pregnancy most suggestive is -

a- Ultrasound showing empty uterus

b- Ultrasound showing tubal mass

c- Negative beta HCG

d- CT scan
10 weeks old child with persistent unilateral eye discharge not responding to antibiotics
but recurring -

a- Nasolacrimal duct obstruction

b- Gonococcus

c- Chlamydia

Unilateral foul smelling, bloodstained discharge from nose -

a- Foreign body

b- Nasal polyps

c- Atopy

d- Rhinitis

10 yrs old came to your surgery with scrotal pain. On examination both testis are in the
scrotum, next management -

a- Do an ultrasound

b- Arrange surgery

c- Write some analgesic and send him home

d- Do nothing it will go away

e- Tell his mother to review back again when the pain recurs

19 yrs old girl had a binge of drinking the previous night with lower abdominal
tenderness and all investigations and tests are normal -

a- Treat as gastroenteritis

b- Tell her it is due to alcohol


c- Not sure of diagnosis come back for review

d- Give analgesic and ant-emetic and send her home

Reversal of non depolarizing skeletal muscle blockage -

a- Pyridostigmine

b- Neostigmine

c- Atropine

d- Benzhexol

Differentiation between schizophrenia and schizophreniform disorders in by -

a- Affective symptoms

b- Duration of symptoms

c- Lack of insight

d- Female and male ratio

A lady with a previous divorce now comes to you with a seductive behaviour -

a- Narcistic

b- Histrionic

c- Borderline

In Australia bush fires are common either accidentally or due to some people lighting
fire deliberately. Which is true regarding pyromaniacs?

a- Done for notoriety and publicity

b- To hide their acts

c- As they like to play with fire


d- Set fire and get panic attacks

e- For satisfaction

R.dass, Nov 1, 2007

#144

R.dass

R.dass Guest

People living near airport have -

a- Explosive personality

b- Insomnia

Regarding panic attacks all are true except-

a- 20% have had at least one attack in their lifetime

b- With out agoraphobia it is equal in mal and female

c- Always avoid precipitating factors

d- Usually occurs in the twenties

Regarding treatment of chronic duodenal ulcer -

a- Eradication of H pylori

b- H2 blockers

c- PPI

d- Selective vagotomy
4 year old boy with fever and malaise, lymphocyte count - normal, platelets decreased,
Hb decreased -

a- ALL

b- Infectious mononucleosis

c- Hodgkin’s disease

d- Read Stenberg cells

aippg.com

Pregnant lady with Group B strep infection. What is true?

a- Penicillin to be given as prophylaxis

b- Bolus dose of penicillin before labour

c- Take a swab and if B strep present then treat

Regarding MI, maximum deaths occur in -

a- With in 1st 2 hours

b- 2 to 12 hrs

c- 12 to 24 hrs

d- 2 to 7 days

e- After discharge

A case of stable angina with chest pain on examination enzymes and ECG are normal
but as you were examining him he belches and says he feels better, what would you do?

a- Admit to coronary care and do ECG monitoring

b- Send him home with appointment to cardiologist

c- Refer to gastroenterologist

d- If enzymes normal then probably no cardiac


With regard to primary health care all are true except -

a- 1/3rd population come to a GP with psychiatric symptoms

b- Most patients are psychotic

c- Only few are referred to psychiatrists

d- Alcoholics and drug abuse frequently overlooked by GP’s

A patient with known Parkinson’s disease for 2 yrs on long term treatment now
comes with tongue protruding out. What would you do?

a- Decrease levodopa + Carbidopa

b- Increase levodopa + Carbidopa

c- Stop levodopa + Carbidopa

d- Treat with chlorpromazine

In a case of twin pregnancy all are true except -

a- Asymmetrical growth retardation

b- Anaemia in pregnancy

c- Premature labour

d- 2nd twin foetal malformation

R.dass, Nov 1, 2007

#145

R.dass

R.dass Guest
60 yrs old female diarrhoea and profuse mucous discharge, what could be the cause -

a- Chron’s disease

b- Ulcerative colitis

c- Villous adenoma

d- Rectal Ca

e- Acute mesenteric ischaemia

How do you differential between Anorectal and colorectal cause of bleeding -

a- Blood mixed with stools

b- Fresh bright bleeding

c- Mucoid discharged

d- Pain during defecation

Most common cause of severe chest pain in pericarditis-

a- Viral pericarditis

b- Tuberculosis

c- Mycoplasma

d- Uraemia

e- MI

All of the following are causes of supraclavicular mass except -

a- Stomach Ca

b- Cervical rib

c- Breast Ca
d- Subclavian thrombosis

e- Subclavian artery aneurysm

Which of the following has worst prognosis?

a- Advanced breast Ca

b- Choriocarcinoma

c- Hodgkin’s lymphoma

d- Non-Hodgkin’s lymphoma aippg.com

e- Prostate carcinoma

Which of the following does not metastasize to brain?

a- Malignant melanoma

b- Prostate Ca

c- Lung Ca

d- Breast Ca

40yr old lady with a 2 cm palpable breast lump on the right side. What is the next step?

a- FNAC

b- Ultrasound

c- Lumpectomy

d- Mammography

e- Radical mastectomy

A lady with palpable breast lump, FNAC showed few malignant cells. regarding
conservative surgery what is true?
a- Assess for oestrogen receptors

b- Bone marrow biopsy

c- Axillary lymph node sampling

d- Mammography

A lady with a breast cancer on left side operated 2 yrs ago now detects a small lump on
the right side. How do you explain the lump?

a- Cancer arising de novo

b- Fibroadenoma

c- Metastasis from the previous one

Facial nerve palsy can be associated with all of the following except?

a- Chronic parotitis

b- Ca parotid

c- Acoustic neuroma

d- # base of the skull

R.dass, Nov 1, 2007

#146

R.dass

R.dass Guest

A middle aged woman the deafness and loss of corneal reflex but with no tinnitus -

a- Vestibular neuronitis

b- Meniere’s disease
c- Acoustic neuroma

d- Multiple sclerosis

Ptosis, dysphagia, ataxia, on the same side and spinothalamic loss on the opposite side -

a- Vertebral artery occlusion

b- Basilar artery occlusion aippg.com

c- MS-midbrain

d- Lateral medullary syndrome

22yrs old lady with diplopia. On examination the right eye, the medial side of the image
is lost (i.e. diplopia on looking laterally). What is the diagnosis?

a- Left 6th nerve palsy

b- Left 3rd nerve palsy

c- Right 6th nerve palsy

d- Posterior cranial fossa tumour

Reading a patient with hepatoma, which of the following is least likely?

a- Hepatitis B

b- Hepatitis C

c- Hemochromatosis

d- CMV

e- Alcoholic cirrhosis

In which of the following cell mediated immunity is lost first followed by loss of humoral
immunity?
a- CLL

b- HIV

c- RA

Which of the following is not a carcinogen?

a- EBV

b- CMV

c- Hep C

d- HIV

A picture of a lesion at the lateral angle of the eye. What is the treatment(It’s a
BCC)?

a- Surgical removal

b- Excision and radiation

c- Cryotherapy

d- Chemotherapy

e- Local steroids

Picture of swelling at the outer angle of the eye. t is described as being hard and present
since birth -

a- Osteoma

b- Sebaceous cyst

Picture of a large swelling on the back near the left scapula -

a- Lipoma
b- Sebaceous cyst

Picture of the face with a non itchy rash like lesion on the cheeks, forehead -

a- SLE

b- Seborrhoeic dermatitis

c- Acne rosacea

d- Dermatomyositis

R.dass, Nov 1, 2007

#147

R.dass

R.dass Guest

Post operative specimen (testicle with epididymis)

a- TB

b- Epididymoorchitis

c- Teratoma

d- Torsion of testis

e- Seminoma

A chest x-ray of a child showing consolidation, neutrophils increased. What could be the
cause?

a- Klebsiella pneumonia

b- Group A streptococcus

c- Staphylococcus
d- Mycoplasma pneumonia

A lesion (looks like an ulcer) diagnosis -

a- Amelanotic melanoma

b- Implantation Dermoid

c- Basal cell carcinoma

ECG- patient is a diabetic and he is dyspnoeic diagnosis-

a Inferior wall MI

b- Anterior wall MI

c- Pericarditis aippg.com

d- WPW syndrome

e- Pulmonary embolism

ECG patient has palpations, otherwise normal -

a- Atrial fibrillation

b- Atrial flutter with variable block

c- WPW syndrome with accelerated beats

ECG patient comes with sweating, palpitations -

a- Ventricular Ectopic

b- Ventricular tachycardia

c- Atrial fibrillation

d- RBBB
How do you treat WPW in a patient with a previous history of collapse?

a- Beta blockers

b- Cardioversion

c- Radiofrequency ablation abnormal tract

d- Surgical ablation

e- Long term verapamil

40y old man with SVT 160/min. Patient not arousable, treatment is -

a- Cardioversion

b- Adenosine

c- Procainamide

d- Verapamil

e- Take an ECG

Child with heart rate 220/min otherwise normal. How do you manage?

a- Cold stimulus

b- Valsalva

c- Verapamil

All of the following are side effects of depot medroxyprogesterone, except -

a- Amenorrhoea

b- Weight gain

c- Depression
d- Used with oestrogen causes stratification and cornification of vagina

e- Hypotension

f- Hypertension

R.dass, Nov 1, 2007

#148

R.dass

R.dass Guest

20yr old man came with pain referring from groin to trip of penis brought a sample of
urine mixed with blood and asks for a shot if Pethidine to relieve his pain. What is the next
step -

a- Give him an injection of Pethidine to relieve his pain

b- KUB

c- Ultrasound abdomen

d- Examine fresh urine sample

12 yr old boy bee sting with wheeze, with swollen lips, tachycardia, restless, immediate
treatment -

a- Adrenaline IM

b- Hydrocortisone IV

c- Oxygen

d- Antihistamine

e- Give NSAID and send him home


Young man with RTA, Bp 90/70, pulse 140 difficulty in breathing -

a- Start two IV line to Hartmann

b- Wide bore needle

c- Tube drainage

aippg.com

RTA respiratory distress mediastinal shift to opposite side with emphysema in the neck.
What is the diagnosis?

a- Tension pneumothorax

b- Hemothorax

c- Cardiac tamponade

d- Bronchial tear

Newborn male with normal genitalia- which is true-

a- 47xxy

b- 46xy with androgen sensitivity

c- Mother treatment with cyptoterone from 8 weeks

16 yr old girl came with her mother with no menstrual breast development is normal
(testicular feminisation)-

a- 45xo

b- 46xy

After MVA a patient is dyspnoiec Bp 100/70, HR 110/min, Breath sounds decreased on


left side, heart sounds normal, JVP raised, next step in management -

a- IV fluids
b- Wide bore thoracotomy

c- Tube thoracotomy

A patient with excruciating chest pain and a diastolic murmur. What does therapy
show?

a- Widening of mediastinum

b- Increased left ventricular size

c- Trachea shifted to left

A child with fever malaise, sore throat………white papillae on the tongue and later
a sandpapery rash……what is true diagnosis?

a- Measles

b- Rubella

c- Scarlet fever

A child with fever of 3 days duration and a rash develops when fever subsides -

a- Rubella

b- Roseola (Measles)

c- Erythema multiforme

d- Steven Johnson syndrome

R.dass, Nov 1, 2007

#149

R.dass

R.dass Guest
An unconscious man (a known COPD case) was brought to the emergency, on
examination there was a bruise on the parietal area and needle mark in the cubital fossa.
His ABG was as follows (PH: 7.26, PCO2: 60, PO2: 50) His previous ABG showed (PH: 7.35,
PCO2: 30, PO2: 60). What is the diagnosis?

a- Narcotic

b- Subdural haemorrhage

16 weeks pregnancy woman with proteinuria 3+, hematuria and hypertension -

a- PIH

b- Pre-existing renal disease

c- Essential hypertension

d- Pregnancy will continue until term

Which of the following is familial -

a- Papillary carcinoma

b- Medullary carcinoma

c- Follicular Ca

d- Anaplastic

e- Secondary Ca of thyroid

A patient with HIV and cough (respiratory symptoms) has a Mantoux 5mm -ve, what is
the next step?

a- INH prophylaxis

b- Zidovudine

c- Interferon
Patient with HIV +ve status, what is true -

a- Life long infectivity aippg.com

b- He has AIDS

c- Can transmit through saliva

d- He should avoid sexual intercourse

Tremor can be present in all of the following except -

a- Hyperthyroidism

b- Hypothyroidism

c- Benign essential tremor

d- Parkinsonism

e- Chronic liver disease

A lady with tremor on lifting the phone and disappears when she looks at her hand with
mild rigidity of the hand but no cog wheel rigidity. What is the treatment?

a- Propranolol

b- Benzhexol

c- Levodopa

Complication of # of Epiphyseal plate -

a- Retardations of longitudinal growth

b- Joint stiffness

c- Malunion

d- Non - union
e- Levodopa

Which of the following does not have any interaction?

a- Warferin & isosorbitrate

b- Verapamil & metoprolol

c- Erythromycin & terfenadine

d- Digoxin & Amiodarone

12yr old weight 90th percentile with a limp -

a- Perthes’s disease

b- Slipped capital femoral epiphyses

c- Tibial synovitis

13 yr old child which would be the appropriate bone age for a normal adult height -

a- 9yr old

b- 13yr old

Regarding mammography, all except -

a- It is painless

b- Can diagnose breast Ca earlier than self examination

c- More diagnostic for 70yr old

d- Definitive diagnosis of Ca breast


A young boy has fever and limp. There is tenderness at one point in the tibia, all
movements are full except flexion which is restricted to 30 degree. What is the diagnosis?

a- Septic arthritis

b- Osteomyelitis

c- Perthes disease

10 days old boy with vomiting, Sr potassium is 7.7, Sr Na is 118.What is the diagnosis?

a- Pyloric stenosis

b- Congenital adrenal hypoplasia

c- SIADH

d- Posterior cranial fossa tumour

6 hours after difficult catheterization, a patient developed fever with chills-

a- Suppurative urethritis

b- Gram negative septicaemia

c- Haemorrhage aippg.com

Regarding hemochromatosis, diagnostic investigation is -

a- Sr. ferritin

b- Transferrin

c- Sr. iron

d- Liver biopsy

In Australia the common cause of iron deficiency is -

a- Nutritional deficiency
Most common cause of bleeding P/R in children -

a- Fistula in ano

b- Fissure in ano

Patient with depression is treated with antidepressants, following a course of treatment


she now wants to stop the drug. What do you advice?

a- Follow up every week

b- Follow up every month

c- Admission to hospital

Child with abdominal mass and with metastasis to skull -

a- Neuroblastoma

b- Wilm’s tumour

A child babbles sits for sometime unsupported, stands with support and holds an object
by the palm. What is the age?

a- 5 months

b- 7 months

c- 1 year

A patient with Dysthymia for 2 years treatment is -

a- SSRI +cognitive +behaviour therapy

b- SSRI +benzodiazepine

c- SSRI
d- Antipsychotic +SSRI

A young patient vomits 1 litre of blood and has 2 times malaena, what is next
appropriate step?

a- Gastroscopy

b- Immediate surgery

c- Barium meal

Old lady with knee swelling and has weekly positive birefringent crystals the crystals
contain -

a- Calcium pyrophosphate dehydrate

b- Calcium hydroxyapatite

c- Urate

A patient is worried about the (sexual dysfunction and depression) side effects of
sertraline, what would be the advice regarding the same to the patient?

a- it is only side effect of antidepressant

b- Both these side effects are not related to sertraline

c- it causes sexual dysfunction but not depressant

R.dass, Nov 1, 2007

#151

R.dass

R.dass Guest
Young man after a RTA was found unconscious, only responds to painful stimuli and eye
cold stimuli what is the GCS score-

a- GCS>3

b- 3 to 6

Child with fever and blisters in the palm and sole and mouth, there is lymphadenopathy
-

a- Coxsackie’s

b- Kawasaki

c- Herpes

All of the following are scaly lesions except -

a- keratoacanthoma

b- Squamous cell Ca

c- Pityriasis rosea

d- Psoriasis

Rash similar to syphilis except -

a- infectious mononucleosis

b- Atopic eczema

c- Discoid eczema

d- Tinea corporis

Pregnant lady with carpal tunnel syndrome. What is true?

a- surgical intervention is rarely needed


b- Splinting in hyperextension

What is typical feature of carpal tunnel syndrome (CTS) repeat?

a- severe pain awakening the patient at night

b- Pain in the ring and little finger

c- Paresthesia restricted to median nerve

d- History of myxedema

3rd day blues what is correct -

a- occurs at least in 50% after delivery

b- More common following “C” section

c- Always lead to psychosis

d- Results in more premature malformed children

Regarding child abuse - aippg.com

a- more common in deformed premature and LBW children

b- Toddlers are more commonly affected

c- Majority patients would have a psychiatric disorder

d- Other children in the family won’t get affected

Confidentially can be broken in which of following situations -

a- child abuse

b- When the police comes to ask

c- Insurance matters
All are hepatotoxic except -

a- paracetamol

b- OCP’S

c- Halothane

d- INH

R.dass, Nov 1, 2007

#152

R.dass

R.dass Guest

Carcinomatous change can occur in all except -

a- Sjogren’s syndrome

b- Thyrotoxicosis

c- Coeliac disease

2yr old child with chronic respiratory infection, what would you do?

a- sweat chloride test

b- X ray chest

c- Barium meal

Child with chronic cough and rectal prolapse diagnosis?

a- congenital megacolon
b- Cystic fibrosis

c- Whooping cough

6 weeks old child on breast feeding with 4 days constipation and thriving well,
diagnosis-

a- Hirschsprung’s disease

b- Normal variant

c- Hypothyroidism

d- Acquired constipation

Which of the following is not associated with brain metastasis?

a- small cell Ca

b- Breast Ca

c- Renal Ca

d- Lung Ca

An elderly patient, known case of Ca rectum, on morphine has chronic constipation and
was treated with Lactulose but was not responding what is the next step?

a- arrange a surgical consultation

b- Enema

c- P/R examination

d- Modify diet

All of the following are associated with Raynaud’s except -

a- RA
b- SLE

c- Scleroderma

d- Dermatomyositis

e- Ankylosing spondylitis

Cause of diarrhoea in an old bed ridden patient -

a- faecal impaction

b- Carcinoma aippg.com

Haemolytic anaemia all are true except -

a- increased urobilinogen

b- lcterus and clear urine

c- MCV decreased

A child ingested washing powder half an hour ago next management -

a- Admit to hospital and possible Endoscopy

b- Charcoal

c- Sent him home

d- Syrup of ipecacuanha

e- Observe

R.dass, Nov 1, 2007

#153

R.dass
R.dass Guest

Mother of an 18 month child was concerned as it was not babbling; audio logical
assessment was done when he was 10 months old. What is the next step?

a- Arrange audiometry

b- Repeat hearing test

c- Reassure the mother

7 ½ yr old girl attained menarche, her mother noticed the breast growth and axillary
hair growth 6 months ago-

a- Premature aneurysms

b- Undiagnosed congenital adrenal hyperplasia

In a young female patient what is the most common cause of subarachnoid


haemorrhage

a- Ruptured aneurysms

b- AV malformations

c- Mycotic aneurysm

A neonate few hours after birth developed cyanosis which was not responding to
oxygen diagnosis -

a- Transposition of great vessels

b- Tetralogy of fallot’s

c- VSD

Lung carcinoma seen in non smokers-


a- Adenocarcinoma

b- Squamous cell carcinoma

A child has facial movements and it is increased on watching TV (question on tics). They
are present even during examination -

a- Myoclonic epilepsy

b- Tics

c- Partial complex epilepsy

In TNM staging which has a better prognosis-

a- T1 N0 M0

b- T1 N1 M0

60 yr old female patient had repeated bilateral Thrombophlebitis and DVT -

a- pancreatic cancer aippg.com

b- Ovarian cancer

c- Protein deficiency

Most common cause of central cyanosis -

a- left to right shunt

b- right to left shunt

c- CO poisoning

Regarding immunology what is correct -

a- IgG associated with Atopic eczema


b- Delayed hypersensitivity is t cell mediated

R.dass, Nov 1, 2007

#154

R.dass

R.dass Guest

Why is it difficult to differentiate beta HCG and LH?

a- FSH high

b- LH high

c- Prolactin high

d- Progresterone

Young hypertensive male with proteinuria, hematuria and upper respiratory infection
for 2 days, diagnosis?

a- IgA nephropathy

b- Glomerulonephritis

c- Nephrotic Syndrome

d- Acute Pyelonephritis

e- Membrane Nephropathy

All of these can cause gynecomastia except -

a- Spironolactone

b- Cimetidine

c- Digoxin aippg.com
d- Labetolol/Frusemide

e- Methyldopa

Which of the following is least likely associated with primary hypothyroidism?

a- 72 y/o with multinodular goitre

b- 28 y/o with menorrhagia

c- 9 y/o with retarded bone age

d- 16 y/o with anovulatory cycles

Which of the following is correct?

a- Alcohol reduces triglycerides

b- Olive oil decrease HDL

c- Increased LDL/HDL ratio is good

d- Taking 300gms of fish daily does not reduce cholesterol

Point prevalence in schizophrenia means the -

a- Current cases at that HDL

b- Cases in one year

c- The total number of cases

Regarding Diabetes mellitus what is true?

a- At least take 100gms of carbohydrate everyday to prevent ketonuria

b- If one parent has diabetes there is 1.8 chance of getting affected

c- You can’t give insulin until level comes to normal


Regarding diabetic foot, all are true except-

a- 50% mortally following amputation

b- If the pulse is present patient is unlikely to have it

c- To have diabetic foot controlling glucose level can help

Regarding 80% carotid artery stenosis, what is true?

a- The incidence of stroke will be halved

b- 40% stroke with out treatment

c- 50% incidence of stroke after endarterectomy

d- 40% death will occur within 30 days

Spiral # of humerus, which nerve damaged?

a- Radial

b- Ulnar

c- Median

d- Volkmann’s ischaemia contracture

R.dass, Nov 1, 2007

#155

R.dass

R.dass Guest

50yr old wakes up at night due to pain in the calf which is relieved by walking, what is
the cause?
a- Ischaemic pain

b- Raynaud’s phenomenon

c- Muscular cramps

d- DVT

e- Intermittent claudication

A patient with pain in the calf worsened by elevation of feet -

a- Ischaemia

b- Noctumal cramps

c- DVT

Which of the following features to be due to arterial ischaemia?

a- Pain along the buttock and thigh after exertion

b- Weakness of the buttock and thigh

c- Shooting pain from buttock and thigh

d- Leg paralysis

Most common feature of rectal carcinoma -

a- Tenesmus

b- Bleeding

c- Incomplete defecation

aippg.com

A patient after MVA sustained a pelvic # and has blood in the external meatus initial
investigation of choice is -

a- Urethrogram
b- CT scan

c- Cystoscopy

d- Catherisation

e- IVU

Regarding uncomplicated haemorrhoids all are true except -

a- Pain

b- Pruritis

c- Bleeding

d- Prolapse

e- Mucus discharge

Regarding PMS what is true?

a- 5% have very severe symptoms

b- 40% have PMS

c- all patients with PMS always have Dysmenrrhoea

A mechanic with penetrating hand injury what is true?

a- Drainage should be done from extensor surface

b- It indicates that there is extension of the palmar abscess into extensor aspect

c- The swelling is due to oedema

d- It indicates the involvement of extensor tendons

Apathetic and dull are the negative symptoms of schizophrenia, which is the negative
symptom?
a- catatonia

b- Blunted affect

A patient who is aware about his own problems during treatment, what is it?

a- Instinct

b- Insight

c- Pseudo altruism

d- Intellectualization

R.dass, Nov 1, 2007

#156

zar

zar Guest

answers

hi

the questions are very helpfull . Thanks

can you plz post answers .

both joseph n R.dass good work .

can you plz post the 30th october amc mcqs questions . it will be great.
thanks n regards

zar, Nov 1, 2007

#157

Joseph.

Joseph. Guest

The change of the severe hypo-glycemia in the infant of a diabetic mother can be
lessened by all the following except -

a- Careful control of the maternal blood glucose levels during pregnancy

b- Maternal intravenous loading with 10% glucose beginning 2 to 4 hr period to the


expected time of delivery

c- Careful glucose monitoring of the infant

d- Early feeding of the infant

e- Maintenance of the infant in a neutral thermal environment

A term, 4200 g female infant is delivered via cesarean section because of cephalopelvic
disproportion. The amniotic fluid was clear and the infant cried almost immediately after
birth. Within the first 15 min. of life, however the infant’s respiratory increased to 80
breaths per minute and she began to have intermittent grunting respirations. The infant
was transferred to the level two nursery and was noted to have an oxygen saturation of
94%. The chest radiograph showed fluid in the fissure, over-aeration, and prominent
pulmonary vascular markings. The most likely diagnosis in this infant is -

a- diaphragmatic hernia

b- Meconium aspiration

c- Pneumonia
d- Idiopathic respiratory distress syndrome

e- Transient tachypnea of the newborn

The infant pictured below presented with syphilis. Clinical features of congenital syphilis
during the first 3 months of life include all of the following except -

a- Maculopapular rash

b- Persistent rhinitis

c- Anemia

d- Interstitial keratitis

e- Hepatosplenomegaly aippg.com

A newborn infant is ready for discharge from the term nursery. In providing instructions
to the parents, you include advice concerning infant automobiles restraints. Each of the
following is true except:

a- Children are safest restrained in the back seat of the car

b- The risk of death is tenfold greater in unrestrained children

c- Many children restrained are used improperly

d- Most children between 1 and 4 yrs of age are not restrained

e- An adult wearing a seat belt can safely hold an infant in his/her arms

Correct statements regarding neural tube defects (anencephaly, meningomyelocele)


include each of the following except –

a- The hereditary pattern is considered multifactorial

b- The prenatal diagnosis can be made by detection of very low levels of alpha-
fetoprotein in the amniotic fluid

c- There is an increased risk in the next pregnancy


d- The risk of a third affected child is approximately 10%

e- Environmental and social factors influence the incidence

An infant born to a heroin addict is likely to exhibit all the following except –

a- Prematurity and low birth weight

b- Onset of withdrawal symptoms within the first 2 days of life

c- Hyperirritability and coarse tremors

d- Vomiting and diarrhea

e- An increased incidence of hyaline membrane disease

A previously healthy full-term infant has several episodes of duskiness and apnea during
the second day of life. Diagnostic considerations should include each of the following
except –

a- Bacterial meningitis

b- Congenital heart disease

c- Seizure disorder

d- Harlequin syndrome

e- Hypoglycemia

In the newborn period, which of the following is the least common sign of meningitis?

a- Lethargy

b- Jaundice

c- Vomiting

d- Nuchal rigidity

e- Hypothermia
Joseph., Nov 2, 2007

#158

Joseph.

Joseph. Guest

A 72 year old man complains of limiting exertional chest tightness. He has clinical
evidence of significant aortic stenosis. Transthoracic echocardiography demonstrates a
peak aortic valve gradient of 80 mmHg with moderate left ventricular function and
probable aortic incompetence Cardiac catheterization is performed. What would be the
most helpful information to obtain from this investigation?

a- Aortic valve gradient

b- Left ventricular function

c- Presence of co-existing coronary artery disease

d- Right heart pressure

e- Severity of co-existing aortic incompetence

A patient with dilated cardiomyopathy and permanent atrial fibrillation (AF) has a
resting heart rate of 110 bpm. 24 hours taped recording show even higher uncontrolled
rates, particularly associated with exercise. He is already taking 187.5 mg of digoxin and
has a normal creatinine. Which one of the following would be the most beneficial
treatment?

a- Addition of beta blocker

b- Addition of verapmil

c- AV node ablation and permanent pacemaker insertion

d- DC cardioversion

e- Increase digoxin to 250 mg.


A 30 year old window cleaner has a one year history of frequent, rapid, irregular
palpitations associated with dizziness, but no actual syncope. He drinks approximately 35
units per week but is on no regular medications. His resting ECG confirms a diagnosis of
Wolff-Parkinson-white syndrome (WPW). Which one of the following is the most
appropriate treatment?

a- Aminodarone

b- Flecainide

c- Radiofrequency ablation of accessory pathway

d- Radiofrequency modification of AV node

e- Sotalol

A 60 year old presents with increasing swelling of ankles, abdominal distension, and
dyspnoea. She has a past medical history of pulmonary, tuberculosis as a child and a left
mastectomy and subsequent radiotherapy 5 years previously. On examination she is
apyrexial, with a sinus tachycardia of 100bpm and blood pressure of 110/160 (paradox 8
mmHg.). She has significant peripheral oedema and ascites. Her jvp is elevated at 8 cm
above the sternal angle and demonstrates a rapid y descent. What is the most likely
diagnosis?

a- Cardiac tamponade

b- Constrictive pericaditis

c- Intra- abdominal neoplasm

d- Severe tricuspid incompetence

e- Superior vena cava obstruction

The following findings are obtained during right and left heart catheterization in a 50 yr
old woman:

Pressures: right atrial = mean 9, right ventricle = 35/2, pulmonary artery = 36/14,
pulmonary capillary wedge = 10, aorta = 120/65.
Saturations (%): superior vena cava = 65, right atrial = 76,

Right ventricle = 77, pulmonary artery = 75, aorta = 97.

She has no significant past medical history, except recent onset of paroxysmal atrial
flutter. What is the most likely diagnosis?

a- ASD – ostium primum aippg.com

b- ASD – ostium secundun

c- Sinus venous defect

d- Tricuspid incompetence

e- VSD

An asymptomatic 5 yr old man is noted to have a systolic murmur. He is in sinus rhythm.


Transthoracic echocardiography demonstrates normal left ventricular function and
dimensions, together with moderate mitral regurgitation secondary to posterior mitral
valve leaflet prolapse. The left atrium is enlarged at 4.5 cm. Which one of the following is
the most appropriate course of action?

a- Amiodarone to maintain sinus rhythm

b- Recommend SBE prophylaxis and discharge

c- Regular clinical follow-up with repeat echocardiography

d- Regular clinical follow-up with repeat echocardiography and anticoagulation with


warferin

e- Transoesophageal echo and early referral for mitral valve replacement

A 45 yr old HGV licence holder is seen in outpatients 6 weeks post uncomplicated


inferior myocardial infarction (MI), for which he received streptokinase. His resting ECG
shows inferior T wave inversion. What would be the next investigations/procedure?

a- Angiography with percutaneous intervention to the right coronary artery

b- Bruce protocol exercise test


c- Echocardiogram

d- Modified Bruce exercise test

e- Thallium scan

a patient presents with severe interscapular pain. A CT scan confirms a diagnosis of


Type B aortic dissection. Clinical examination reveals sinus tachycardia of 100 bpm and
blood pressure of 150/84 mmhg (similar in both arms.) Which one of the following is the
most desired intervention?

a- Intravenous beta blocker

b- Intravenous GTN

c- Oral ACE inhibitor

d- Referral for urgent surgery

e- Transoesophageal

Joseph., Nov 2, 2007

#159

Joseph.

Joseph. Guest

Post anterior MI, a 60 yr old man develops cardiogenic shock with blood pressure of
80/50 mmhg and diminished urine output. A Swan-Ganz catheter is inserted and reveals a
pulmonary capillary wedge pressure (PCWP) of 20 mmhg; systemic vascular resistance
(SVR) 1580 (normal: 900-1200 dyne.s/cm5) and cardiac index (CI) of 1.8 (cardiac output
/body surface area, normal; 2.8 -3.5 l/min/m2). Which one of the following should be the
optimal initial treatment?

a- Dobutamine

b- Dopamine
c- Intravenous GTN

d- Intravenous fluids (gentle)

e- Noradrenaline

A patient has pulmonary embolus proven by spiral CT scanning. 24 hours after initiation
of low molecular weight heparin (LMWH) therapy he becomes hypotensive, tachycardic
and hypoxic without evidence of acute bleeding. His JVP is elevated at 8 cm above the
sternal notch. Which one of the following would be the desired intervention?

a- Charge LMWH to intravenous unfractionated heparin

b- Inotropic support

c- Insertion of inferior vena cava filter

d- Thrombolysis with streptokinase

e- Thrombolysis with tissue plasminogen activator

Six months after prosthetic mitral valve replacement a patient presents with a 2 month
history of rigors, anorexia, fatigue and weight loss. The CRP is elevated at 110.
Transthoracic echo confirms moderate paravalvular regurgitation with vegetation. Which
one of the most likely infecting organism?

a- E.coli

b- Staphylococcus aureus

c- Staphylococcus epidermidis

d- Streptococcus viridans

e- Candida albicans aippg.com

A 40-yr-old smoker is found to have fasting cholesterol of 8.0 mmol/l and triglycerides
of 1.8 mmol/l. He is hypertensive with a strong family history of premature coronary
artery disease. Which one of the following would be the recommended intervention?
a- Atorvastatin

b- Bezafibrate

c- Cholestyramine

d- Diet

e- Pravastatin

A patient with a previous history of myocardial infarction presents with a broad


complex tachycardia. Blood pressure is 110/60 mmhg. Lung fields are clear. What is the
most appropriate initial management therapy?

a- Intravenous digoxin

b- Intravenous lignocaine

c- Oral aminodarone

d- Overdrive pacing

e- Transoesophageal echo and dc cardioversion if no thrombus

Which one of the following physiological variables has the greatest influence toward
increasing cardiac output during the later stages of strenuous exertion in a healthy adult?

a- Coronary vasodilatation

b- Decrease in systemic vascular resistance

c- Increased alveolar ventilation

d- Increased stroke volume

e- Increased ventricular rate

A patient undergoes assessment for suitability of percutaneous trans-septal mitral


valvuloplasty for the treatment of rheumatic mitral stenosis. The presence of which one of
the following features would most likely preclude this form of intervention?
a- Atrial fibrillation

b- Coronary artery disease

c- Heavily calcified mitral valve

d- Pulmonary hypertension

e- Spontaneous contrast in the left atrium, seen on TOE

An asymptomatic 30-yrs-old gym instructor is seen as part of a routine medical


examination. An ECG demonstrates LBBB. Which one of the following clinical signs is most
likely to be present?

a- Displaced apex beat

b- Fourth heart sound

c- Left ventricular heave

d- Reverse splitting of the second heart sound

e- Third heart sound

Joseph., Nov 2, 2007

#160

Joseph.

Joseph. Guest

A 15-yrs-old girl presents with a short history of fever, malaise and a flitting
polyarthritis. Clinical examination reveals a oft apical systolic murmur and pericardial rub.
Investigations demonstrate elevated inflammatory markers (CRP and ESR). Which one of
the following is the most likely diagnosis?

a- Acute rheumatic fever

b- Atrial myxoma
c- Kawasaki disease

d- Subacute bacterial endocarditis

e- Systemic lupus erythematosus

A patient undergoes successful elective DC cardioversion for lone AF. Prior to the
cardioversion he is taking warferin, digoxin and verapamil. Which one of the following
drugs is the most important for the patient to be discharged on until outpatient review in
six weeks?

a- Amiodarone

b- Aspirin

c- Clopidogrel

d- Digoxin

e- Warferin

A 40-yrs-old man is investigated having undergone successful communicate


resuscitation following an episode of ventricular fibrillation.

Echocardiogram demonstrates no structural abnormalities and coronary arteries are


normal at angiography. Which one of the following would be the most appropriate long-
term management?

a- Amiodarone

b- Automated implantable cardiac defibrillator

c- Procainamide

d- Sotalot aippg.com

e- VT stimulation study and ablation

When contemplating coronary bypass grafting, which one of the following is the
preferred option for revascularising the left anterior descending artery (LAD)?
a- Direct endarterectomy

b- Left internal mammary artery

c- Radial artery

d- Right internal mammary artery

e- Saphenous vein graft

A 30-yrs-old man presents with recurrent episodes of chest pain and exertional pre-
syncope. There is a family history of sudden death. Resting ECG demonstrates features of
left ventricular hypertrophy and precordial T wave inversion. Which one of the following is
the most likely diagnosis?

a- Dilated cardiomyopathy

b- Hypertrophic cardiomyopathy

c- Ischaemic heart disease

d- Pulmonary embolic disease

e- Supravalvular aortic stenosis

An infant with trisomy 21 (Down’s syndrome) presents with failure to gain weight
and clinical evidence of heart failure. Which one of the following congenital cardiac
abnormalities is most likely to account for this?

a- Aortic incompetence

b- Endocardial cushion defect

c- Mitral valve prolapse

d- Pulmonary hypertension

e- Secundum ASD
A 70-yrs-old man is found to be hypertensive with repeated blood pressure recording of
> 160/95 mmHg. He is already taking a thiazide diuretic. There are no apparent secondary
causes of hypertension. He has previously undergone coronary artery bypass grafting.
Which one of the following is the most appropriate initial anti-hypertensive treatment?

a- Alpha blocker

b- Aniotensin-converting-enzyme inhibitor

c- Angiotensin II receptor blocker

d- Beta blocker

e- Calcium channel antagonist

A 45-yrs-old man presents with a protracted episode of central chest pain. He is a


current smoker with a strong family history of premature coronary artery disease. His
resting ECG is normal. Which one of these investigations would be the most helpful in
assisting with planning initial management?

a- Angiogram

b- Echocardiogram

c- Exercise test

d- Troponin I

e- Ventilation/perfusion scan

Joseph., Nov 2, 2007

#161

Joseph.

Joseph. Guest

In chronic liver failure (obstructive jaundice) what is true -


a- Vitamin K absorption is reduced

b- Prothrombin cannot be converted to thrombin

In hemochromatosis after venesection all are true except -

a- Skin pigmentation becomes normal

b- Cardiac siderosis is reversible

c- Hepatoma can be cured

153. In hemochromatosis -

a- Size of the liver is decreased following treatment

A patient with perforated peptic ulcer all are correct except -

a- Patient remembers exactly the moment of rupture

b- Back pain

c- Vomiting

d- Board like rigidity

e- Guarding

Concerning cluster headache all are correct except -

a- Vomiting

b- Treat with Methysergide

c- Occurs after 3-4 hrs of failing asleep

d- Occurs at the same time of the day


A young man with needle marks and pin point pupils, gas reflexes are normal, no
extensor planter reflexes -

a- Opiate ingestion

b- Temporoparietal #

A young man with throat infection in child hood and was treated with penicillin. He now
comes with a fear of developing a rash due to penicillin as he was influenced by his
friend’s words. What does he have?

a- Induced delusion

b- Hypochondriasis

A young man with left sucking wound in the chest. After initial airway resuscitation and
circulation management what is the next step?

a- Debridement and closure

b- Pressure bandage

c- IV fluids

Which infection is least likely during childhood?

a- Syphilis

b- Gonorrhoea

c- Streptococcal

d- HIV

e- Herpes

Appropriate estimation of the foetal gestation age is by -

a- Transvaginal ultrasound at 8 weeks


b- Bimanual examination at 8 weeks

c- Ultrasound at 18 weeks

Joseph., Nov 2, 2007

#162

Joseph.

Joseph. Guest

Regarding OCP what is the absolute contraindication?

a- Focal migrainous hemiplegia

b- Hypertension

c- Diabetes

Regarding OCP what is correct?

a- All the progesterone’s except cryproterone have derived from testosterone

Young man with severe joint pain and profuse bleeding from venipuncture site.
Diagnosis?

a- DIC

b- Thrombophlebitis

All are true regarding DIC except -

a- Fibrin Decreased

b- FDP decreased

c- PTT increased
Intermittent claudication commonest site -

a- Profunda Femoris

b- Superficial femoral artery

c- Posterior tibial artery

d- Bifurcation of abdominal aorta

e- External iliac artery

Which defence mechanism is mature?

a- Denial

b- Projection

c- Introjection

d- Humour aippg.com

Undescended testis is most commonly associated with -

a- Torsion of testis

b- Spermatogenesis will be perfect if the operation is done before 3 years

c- Development delay of the child

d- Acute Epididymitis

e- Hernia

G3P0 pregnant lady came to you at 16 weeks pregnancy. She had two spontaneous
abortions at 17 and 18 weeks in the previous pregnancies. How do you treat?

a- Chromosomal analysis

b- Cervical stitch
c- X linked recessive

Regarding Mullerian agencies all of the following is true except -

a- Short vagina

b- Normal breast development

c- Complete recessive

What would be the most likely cause for bloody discharge from the nipple in a 50 year
old lady?

a- Intraductal papiloma

b- Intraductal carcinoma

c- Paget’s disease

d- Fibrocystic disease

The relative marker in Hepatocellular carcinoma is -

a- PSA- Prostate, BHP

b- CA 125 - Ovaru/Breast

c- CEA - Colorectal Ca

d- CA 153 - Breast Ben. Bre. Disease

e- AFP - Fetoprotein Increase 90%

Treatment of Seminoma stage 2 with metastasis to the Para-aortic lymph nodes -

a- Radio therapy to testis and nodes

b- Surgery and radiation to Para aortic nodes

c- Chemotherapy
d- Surgery

70 year old woman with vaginal prolapse with pessary what is the complication?

a- Atrophic vaginitis

b- Cervical carcinoma

c- Decubitus ulcer

d- Carcinoma of vagina

Joseph., Nov 2, 2007

#163

Joseph.

Joseph. Guest

After splenectomy you except all of the following except -

a- Life long increased risk of infection

b- Transient increased platelets to predispose to thrombosis

c- Persistent decrease of RBC life span

d- Decrease in anaemia

Diabetic mother after prolonged labour delivered a jittery baby which responds to
oxygen. What is the diagnosis?

a- Hyperglycaemia

b- Hypoglycaemia

c- Meconium aspiration

d- Hyperbilirubinemia
Splenectomy is most beneficial in which of the following?

a- Haemophilia

b- Chronic ITP

c- Myelofibrosis

A middle aged man unable to fall asleep has night mares and remembers his daily
activities, diagnosis?

a- Anxiety disorder

b- Depression aippg.com

c- OCD

Tourette syndrome; all are correct except -

a- Motor and vocal tics are seen

b- 2 or more bouts per day

c- The person is not distressed with vocal tics

d- Coprolalia is less than 10%

e- Onset always less than 18 years of age

Alcoholism is associated with all of the following except -

a- Substance abuse (benzodiazepine)

b- Depression

c- OCD

Proteinuria is seen in all of the following except -


a- Congenital nephritis

b- RPGN

c- UTI

d- RA

e- Horseshoe Kidney

A 35 yr old lady has a grey greenish vaginal discharge, microscopy shows clue cells, and
she responded well to Metronidazole, diagnosis?

a- Chlamydia

b- Bacterial vaginosis

c- Trichomoniasis

d- Moniliasis

Regarding stammering which is correct?

a- Spontaneous resolution at 16 years

Still birth after a normal delivery, all of the following tests are done except -

a- Kleihaur test

b- Chromosomal analysis

c- Foetal parts

d- Maternal antibodies

e- Foetal post-mortem

Polyhydramnios all are true except -

a- Rh incompatibility
b- Oesophageal atresia

c- Bladder neck obstruction

Polyhydramnios see in all except -

a- Diabetes

b- Multiple pregnancy

c- Cardiac anomalies

d- Hydrops fetalis

Joseph., Nov 2, 2007

#164

Joseph.

Joseph. Guest

Child with grayish tonsilar exudates and atypical lymphocytes -

a- Infectious mononucleosis

b- CMV

Urge incontinence all are true except -

a- Colposuspension

b- Taping the vagina

c- Pelvic exercise

d- Weighted vaginal cones


Regarding Rubella and pregnancy a kindergarten teacher 6 weeks pregnant had one of
her student with rubella. All are true except -

a- Immunoglobulins

b- IgG is increased initially then IgM

c- Repeat SRH after 10 Days

d- If mother infected early than child can have microcephaly, seizures and cataracts

A patient with pyloric stenosis vomiting a lot and vomitus contains undigested food
material taken 2 days back. What is the initial management?

a- Normal saline

b- Hartmann’s

c- Ringer lactate

Daily requirement of potassium -

a- 50mmol

b- 1gm

c- 30mmol

In pyloric stenosis vomiting is -

a- Intermittent

b- Occurs 1hour after food

c- 2 days after food

A man with abdominal pain xray showed multiple fluid levels. What is the management?

a- 2 litres RL aippg.com
b- 2 litres Hartmann’s before surgery

c- 2 litres Hartmann’s after surgery

d- 2 litres Hartmann’s After surgery

Regarding antibiotic prophylaxis what is true -

a- Single does is sufficient when compared to multidose

b- Urologic surgery needs a culture for the specific antibiotic

A 4 yr child constipated, rectum empty and sphincter tone lax, mass at the apex of
rectum -

a- Hirschsprung’s

b- Acquired megacolon

c- Hypothyroidism

Hypercalcaemia seen in all except -

a- Hyperparathyroidism

b- Pancreatitis

c- RTA

3 yr old girl with periorbital oedema, protein 3+, all are true except -

a- Prednisolone will improve in 90% cases

b- Most of the cases are caused by focal segmental glomerulosclerosis

c- Penicillin prophylaxis should be given

d- Prognosis is good
A young man with scrotal mass which disappeared on lying down -

a- Hernia

b- Varicocele

A young girl with fever and oedema of the eye lid but there was no chemosis or
proptosis, what is the treatment?

a- IV flu cloxacillin

b- Topical chloramphenicol

c- Antibiotics

Pooled immunoglobulin is given in all of the following except -

a- Hepatitis A

b- Mumps

c- Rabies

d- Tetanus

A girl with neck stiffness, fever, lumbar puncture showed few neutrophils 200
lymphocytes, protein 2.1. What is the management?

a- Bed rest and paracetamol

b- Ceftriaxone

c- Prednisolone

Joseph., Nov 2, 2007

#165

Joseph.
Joseph. Guest

Adult patient ……………LP shows 85% neutrophils, 200 RBC’S, 2.1 glucose.
What is the diagnosis?

a- Bacterial meningitis

b- Subarachnoid haemorrhage

A child with high fever, how do you assess the severity?

a- Increased WBC count

b- Pallor, irritability

c- Neck stiffness always present with in 6 hours

Grand mother with BPD, which is true?

a- There is less than 10% incidence in grandchild

Postoperative pain management -

a- Morphine is better than Pethidine

Regarding HSP -

a- Do microscopy of urine

b- Abdominal x rays

MS most erroneously diagnosed as -

a- Conversion disorder

b- Ataxia
c- Neuropathy aippg.com

Regarding CTG which is the most ominous sign?

a- Early deceleration

b- Late deceleration

c- There is loss of variability during uterine contractions

d- Beat to beat variation more than 5

A young patient with abdominal pain, back pain and also has impotence. He was
investigated on several occasions. What is the likely diagnosis?

a- Hypochondriasis

b- Munchausen’s syndrome

c- Somatisation

d- Factitious disorder

Regarding perforated peptic ulcer all are true except -

a- Guarding and rigidity present

b- Patient with shock at the onset

c- On percussion the liver is dull

Question on Prader willi syndrome -

a- Genetic syndrome

b- Associated with obesity and Mental retardation

c- Hypotonia

d- Hypogonadism
Fragile X- mental retardation -

a- X-linked condition

b- 1 in 2000 males

c- 2nd to Down’s syndrome in males

d- The first marker of the condition was a small gap, or fragile site, evident near the tip
of the long arm of X chromosome

e- Anticipation

Affected (Heterozygous) woman:

a- Early menopause

b- May have learning difficulties

c- Fank mental retardation

Affected males -

a- Macroorchidism (enlarged tests)

b- Large ears

c- Prominent fans

c- A high-pitched vice

d- Mental retardation

Joseph., Nov 2, 2007

#166

Joseph.

Joseph. Guest
A 35 yr old school teacher came to you with a H/O. difficulty to asleep, irritability and
anxiety. When he goes to bed all minor mistakes he made in the day came to his mind &
argues with those in himself. These thoughts are intrusive & give him much distress
though. He knows these thoughts are irrational. He tries to get rid of them but unable to
do that. He is perfectionist and wants things to be done in the ways he likes. Which of the
following drugs would be most beneficial to Him?

a- Benzodiazepines

b- Non benzodiazepine hypnotic

c- SSRI

d- Atypical anti psychotic

e- Anti psychotic/ anti depressant

Which of the following is a mature defense mechanism?

a- Pseudo altruism

b- Projection

c- Repression

d- Humour

e- Reaction formation

Primary Psychiatric health care is important for all of the following reasons except -

a- One third of all patients present with mental illness to Psychiatrist

b- It needs referral to Psychiatrist for a small number of patients

c- Most of the patients present with psychotic symptoms

d- Psychiatric patients commonly have medical problems

e- Drugs & Alcoholics are commonly overlooked by primary health care providers
18 yrs old boy dressed as female came to you with his boyfriend for problems in sexual
relationship. He felt uncomfortable with his sex since childhood. He had been cross
dressing in several occasions. What is the phenomenon called in Psychiatry?

a- gender identity disorder

b- Transsexuality

c- Transvestism aippg.com

d- Masochism

e- Homosexualism

A lady came to you for counseling after her 2nd marriage break-up. She was severdy
depressed and anxious but looked very seductive during the interview. She was very
suggestive when she was talking about her previous sexual relationship. Her behaviour is
most probably due to -

a- Borderline personality

b- Histrionic personality

c- Antisocial personality

d- Dependent personality

e- Substance abuse

Panic disorder agora phobia all are correct except -

a- More frequent in men

b- Equally frequent in male and female

c- Starts in the 20s

d- Should avoid trigger factors


Effects of noise in airport cause in the near by -

a- Depression

b- Schizophrenia

c- Sleeplessness

d- Aggressive psychosis

Middle aged lady presented with nausea, dyspepsia and abdominal distension. She has
a past history going to many doctors & treated for disorders. She has been treated by a
rheumatologist for aches and pains by cardiologists for Palpitation and gave her
Propranolol, neurologist for epilepsy. You find her tense and anxious. Examinations show
normal findings. There are scars from appendectomy operation. What’s the
appropriate diagnosis in this case?

a- Conversion disorder

b- Hypochondriasis

c- Sumatisation disorder

d- Munchausen’s syndrome

e- Neurotic anxiety

Joseph., Nov 2, 2007

#167

Joseph.

Joseph. Guest

A logia is a negative symptom of Schizophrenia. Which of the following symptoms is


another negative symptom of schizophrenia?

a- Auditory hallucination
b- Delusion of reference

c- Affective flattering

d- Bizarre behaviour

e- Tangentiality aippg.com

Which is not correct about Tourette’s disorder?

a- Onset is before 18 yrs

b- Patient is not distressed by Tic

c- Tics occur many times a day, usually every day

d- More common in males than females

e- Both motor and vocal Tics are present

A high school teacher aged around 40, complains that he finds difficulty in falling asleep
as he goes over the events of the day repeatedly in his head. He is known to be a
perfectionist. Although he knows this ritual is intrusive he can’t resist it. Which of the
following condition he is suffering from?

a- Hypochondriasis

b- OCD

c- Schizophrenia

d- Delusional disorder

e- Personality disorder

Two weeks before a patient was discharged from hospital after she recovered from a
major depression. But now she develops a relapse but doesn’t to take medicine.
What’s the next stage of management?

a- Involuntary hospital admission


b- Transfer to a community based care

c- Tell her that recurrence rate is high & keep her under obstruction

d- Weekly visit for follow-up

e- Refer her to a psychiatrist

What’s the difference between schizophrenia and schizophreniform disorder?

a- Duration

A 12 yr old boy frequently protrudes his tongues while watching TV WOF is correct?

a- Habitual tics

b- Sydenham’s Chorea aippg.com

c- Hunting ton’s Chorea

In ectopic pregnancy earliest rupture occurs at -

a- Ampulla

b- Isthmus

c- Interstitium

Non contraceptive benefits of OCP are all except -

a- Reduce incidence of benign breast disease

b- Reduce incidence of ovarian carcinoma

c- Reduce incidence of endometrial carcinoma

d- Reduce incidence of cervical carcinoma

e- Reduce incidence of PCO


Ectopic pregnancy can be confidently diagnosed by -

a- Serum Beta HCG study

b- USG of pelvis

c- Laparoscopy

d- Finding of empty uterus

Earliest diagnosis of gestational age can be done by -

a- 8 weeks intra vaginal USG

b- 8 weeks abdominal/pelvic USG

c- Bimanual examination at 8 wks

d- B-HCG measurement at 16 wks

Joseph., Nov 2, 2007

#168

Joseph.

Joseph. Guest

A 35 yrs old lady had a greenish grey vaginal discharge. Microscopic exam of vaginal
smear showed due cells. What could be her condition?

a- Moniliasis

b- Gardnerella vaginitis

c- Trichomoniasis aippg.com

d- Chlamydial infections

e- Staphylococcal vaginitis
Bronchial asthma in pregnancy -

a- Should not be treated with steroids

b- Should not be treated with bronchodilators

c- Usually not increased in incidence of bronchospasm

d- Needs caesarean section

e- Increased risk of premature labour

A female patient married for 2 yrs. Came to you for consultation for infertility. She has
regular periods. She has regular intercourse for past 6 months. On exam she has
retroyarted uterus and no other abnormality. What may be the management?

a- Do corrective surgery

b- Manipulation and ring pessary

c- Reassurance /give her a temp. chart/explain about ovulation time, fertile period and
review after 4 weeks

d- Do hysterosalphingography

Carpal tunnel syndrome in pregnancy. Which of the following is the correct response?

a- Diuretic treatment

b- Surgical intervention

c- Splinting of the hand in mid extension position

d- Immediate termination of pregnancy

e- ACE inhibitor therapy

A pregnant lady with 16th wks pregnancy developed proteinuria+++, BP 104/95mm of


Hg, serum creatinine 1.5. Which of the following may be the diagnosis?
a- Pre eclampsia

b- Pre existing renal disease

c- Glomerulonephritis

Which of the following is caused by anti cardiolipin antibody?

a- Repeated abortion

A 26 yrs old lady with irregular cycles, every cycle in 4-10 wks intervals, her LMP at that
time way 6/52. On examination for her amenorrhoea FSH 3iu(N13), LH- 850 (N4-10,
follicular phase, 20-100mid cycle), prolactin - 450(N-50-500). Which of the following is the
most probable diagnosis of her amenorrhoea?

a- Premature ovarian failure

b- Poly cystic ovarian disease aippg.com

c- Stress due to recent disease

d- Early pregnancy

e- LH secreting pituitary

Risk of breast cancer, all are true except -

a- Artificial early menopause

b- Cancer in the other breast

c- Cancer in the body of uterus

d- First pregnancy after 35

e- Family H/O breast cancer

Which of the following is the absolute Contra indication of the one of OCP?
a- Migraines aura

b- Smoking

c- Hypertension (severe)

d- Diabetics mellitus

e- Gallbladder disease

Joseph., Nov 2, 2007

#169

Joseph.

Joseph. Guest

The time of ovulation is accurately diagnosed by which of the following?

a- Serial measurement of oestrogen in the proliferating phase

b- Urinary LH

c- Regular basal body temp

d- Serial LH measurement

e- Estimation of progresterone in the mid luteal phase

Repeated second mid trimester miscarriage. Patient is 3rd time pregnant what is the
best management?

a- Bed rest aippg.com

b- Cercelage

The most common cause of jaundice in pregnancy -

a- Hepatitis B
b- Cholestasis

c- Obstruction

Pregnant lady developed heart failure; she had fixed and wide splitting 2nd heart sound.
What may be the cause?

a- ASD

b- VSD

c- Pulmonary stenosis

d- Coarctation of aorta

e- PDA

All are correct regarding Mullerian agencies except -

a- Patient have a short vagina

b- X-inked recessive

c- Normal breast development

A lady with 39 wks gestation, cervix 8 cm dilated, CTG shows foetal distress. What is the
management?

a- Induction by oxytocin

b- Foetal scalp PH measurement

c- Repeat CTG

d- Immediate C/S

After long term use of OCP there is increased risk of -

a- Endometrial Ca
b- Breast Ca

c- Cervical Ca

d- Ovarian Ca

A pregnant lady of 30 wks gestation was found to be positive for Group B streptococcus.
WOF is true?

a- Treat the patient with immediately with penicillin

b- Treatment with IV penicillin during labour will significantly reduce the risk of
Neonatal infection aippg.com

c- Treat with doxycycline

d- Do not treat because 20% of women are carriers

e- They are normal vaginal flora

A man developed inguinoscrotal swelling which disappear on lying down, what could be
probable cause?

a- Direct inguinal hernia

b- Varicocele

c- Hydrocele

d- Sapheno varix

e- Tumour of the testis

A 40 yrs old man presents with acute severe excruciating abdominal pain. Which is least
likely?

a- Acute MI

b- Ruptured aortic aneurysm

c- Renal colic
d- Acute appendicitis

e- Biliary colic

f- Perforated peptic ulcer

Patient with an accident has decreased breath sound on the left side, trachea deviated
to the right side, surgical emphysema of neck and left side of chest. Which of the following
is the correct diagnosis?

a- Haemothorax

b- Tension pneumothorax

c- Cardiac tamponade

d- Rupture bronchus

e- Laceration of bronchus

Joseph., Nov 2, 2007

#170

Joseph.

Joseph. Guest

What is the most common cause of blindness world wide?

a- Trachoma

b- Cataract

c- Glaucoma

d- Xerophthalmia

e- Onchocercosis
Antibiotic prophylaxis in surgery, which is not true?

a- Multiple drugs are preferred than single drug

Picture of specimen of swollen testis -

a- TB testis

b- Semonima

c- Terratoma of testis

d- Torsion of testis

e- Infection

Picture of a large lump on back of a man below the inferior border of left scapula -

a- Lipoma

b- Dermoid

c- Sebaceous cyst

d- Buffalo hump

e- Hemangioma

Picture, tumour on the lateral side of Rt. eye brow since 30 yrs which does not move on
palpation. WOF is the most probable?

a- Osteoma

b- Lipoma

c- Sebaceous cyst

Which of the following nerve is damaged after # of lower part of humerus?

a- Radial nerve
b- Median nerve

c- Ulner nerve

d- Brachial nerve

Undescended testis, which of the following has the most likely association?

a- Infection

b- Inguinal hernia

c- Malignancy

d- Torsion of testis

e- Varicocele

Patient developed intestinal obstruction with vomiting and dehydration X-ray showed
multiple levels. Which of the following is the best option?

a- 2000 cc Hartmann soln. starting preoperatively

b- 2000 cc Hartmann soln. during operation

c- 2000 cc 5% dextrose in water starting preoperatively and continued through

Which of the following is not a feature of uncomplicated haemorrhoids?aippg.com

a- Mucoid discharge

b- Pain

c- Pruritis

d- Blood in stool

Old bed ridden lady with occasional Diarrhoea, O/E faecal mass is felt in rectum -

a- Ca rectum
b- Stool impaction aippg.com

c- Laxative use

d- Chronic diarrhoea

68 yrs old man,(picture given) developed this condition for 6 week (keratoacanthoma
/BCC). What is the best management?

a- Obstruction

b- Local excision

c- Wide excision

d- Medial treatment

WOF does not cause a supraclavicular palpable mass?

a- Breast cancer

b- Sub clavian arterial aneurism

c- Sub clavian venous thrombosis

d- Cervical rib

e- Stomach cancer

On 7th post operative day patient developed chest pain and breathing difficulty. What
may be the cause?

a- Atelectasis

b- Pneumonia

c- Pulmonary embolism

d- Sepsis
WOF needs immediate intervention?

a- Surgical emphysema

b- Tension pneumothorax

c- Haemothorax

d- Fractured ribs/ flail chest

e- Open pneumothorax

Joseph., Nov 2, 2007

#171

Joseph.

Joseph. Guest

Most common cause of bloody discharge from the nipple is -

a- Intraductal papiloma

b- Paget’s disease of the ectasia

c- Mammary duct ectasia

d- Intraductal carcinoma aippg.com

e- Fat necrosis

Facial nerve palsy occurs with all of the following except -

a- Chr. Parotitis

b- Mastoiditis

c- Ca parotid gland

d- Acoustic neuroma
A man develops dysphagia o solids and liquids. WOF is the most likely?

a- Achalasia

b- Tumour of oesophagus

c- Oesophageal spasm

d- Stristure

A 45 yrs old man states that his father and younger brother died of bowel cancer. He is
worried about himself and wants to exclude the possibility of the disease. Which
screening tests will you conduct?

a- Occult blood in stool for 3 specimens

b- Sigmoidoscopy

c- Sigmoidoscopy + 3 stool specimen for occult blood

d- Barium enema

e- Colonoscopy

Single thyroid nodule is most commonly seen in -

a- Part of a multi nodular goiter

b- Ca thyroid

c- Thyroid cyst

d- Thyroid adenoma

e- Hashimoto’s disease

A patient with BP 90/60, pulse 100/min. JVP 0.5 cm above the sternum & tachycardia.
WOF is correct?
a- Cardiac failure

b- Tension pneumothorax

c- Hypovolumic shock

d- Cardiac tamponade

e- Vasovagal syncope

A lady presented with a 2 cm malignant breast lump. WOF suggests that conservative
breast surgery can be contemplated?

a- Hormone receptor status

b- Peau de orange of the breast

c- Palpation of axillary lymph node

d- Mammography

e- Metastasis of bone

WOF does not metastasize to brain?

a- Ca breast

b- Ca lung

c- Ca prostate

d- Ca kidney

A big inguinal mass present for 3 months (with superficial ulceration). WOF is a
possible?

a- Venous thrombosis

b- Metastatic lymph node aippg.com

c- strangulated inguinal hernia


d- Aneurysm of epigastric artery

50 yrs old smoker waken up at night due to pain in calf and relieved by walking around.
most likely cause is -

a- Ischaemic pain

b- Raynaud’s phenomenon

c- Muscle cramp

d- DVT

e- Intermittent claudication

A elderly man feels pain in his leg after lifting it for some time above the bed. WOF is
correct?

a- Arterial ischaemia

b- Venous stasis

c- Muscle spasm

d- Disc prolapse

Commonest site of occlusion in patient with intermittent claudication?

a- Abdominal aortic bifurcation

b- External iliac artery

c- Superficial femoral artery

d- Profunda Femoris artery

e- Posterior tibial artery

WOF cause the most metastasis to bone?


a- Ca breast

b- Ca stomach

c- Ca lung

d- Melanoma

e- Ca colon

Joseph., Nov 2, 2007

#172

Joseph.

Joseph. Guest

When prophylactic antibiotic is given?

a- 1 hour before operation

b- Per operative antibiotic

c- After 24 hours

Picture of an ulcer with 6 months duration -

a- Basal cell ca aippg.com

b- Amelanotic melanoma

c- Tubercular ulcer

A patient developed pyloric stenosis. He vomited undigested food taken in the previous
meal. He was dehydrated. WOF will be his best replacement fluid?

a- Normal saline

b- Dextrose 5% in water
c- Dextrose 4% in N/5 (1/5 normal) saline

d- Hartmann soln.

e- Hypertonic saline

A carpenter injured the palm of his hand with a screw driver and then developed an
abscess, which cause a swelling on the dorsum of his hand. Which of the following
statement is correct?

a- It’s an extension of the abscess to the extensor tendons

b- Incise the swelling on the dorsum of the hand

c- Swelling is due to oedema

d- It indicates involvement of the extensor tendons

Carpal tunnel syndrome. WOF is true?

a- Tingling sensation along the median nerve

b- Pain at night

c- Pain of the whole hand

d- Pain during carrying weight

Cholecystitis is commonly associated with -

a- Stone in the gall bladder

b- Stone in the bile duct

c- Stone in the outflow tract

d- Stone in the hepatic duct


Patient developed a mass in the upper abdomen with a H/O trauma few days back.
WOF is the best investigation?

a- US of the abdomen

b- CT scan

c- Oral cholecystography

d- ERCP

A young man presents with a full thickness burn on his thigh, the size of a 20c coin, you
should -

a- Excise and do pedicle graft

b- Excise and do a primary suture

c- Excise and do a split skin graft

d- Excise and do dressings

e- Excise and do full thickness graft with micro vascular surgery

All of the following are true regarding arterial ischaemia except -

a- Pain in thigh, buttock and legs

b- Sudden shooting pain along the back of thigh to the leg

c- Cramping pain in the calf muscle

d- Sensation of paralysis aippg.com

e- Pain with exercise

Patient developed RT. hypochondriac pain and jaundice. He had a history of


cholecystectomy few yrs back. WOF is the best investigation to do?

a- USG
b- CT

c- ERCP

d- Oral examination

e- Plain xray of abdomen

The most likely early symptoms of acute pancreatitis?

a- Vomiting

b- Sweating

c- Faintness

d- Pain

e- Constipation

A discrete mobile rubbery mass in the left breast of a 22 yr old woman is most likely to
be -

a- Carcinoma

b- Fibro adenoma

c- Intra Ductal papiloma

d- Fibro adenosis

e- Fat necrosis

Joseph., Nov 2, 2007

#173

Joseph.

Joseph. Guest
Metastatic spread of testicular tumour mainly occurs to -

a- Lung

b- Kidney

c- Para aortic lymph nodes

d- Inguinal lymph nodes

e- Liver

WOF is the cause of hypoprothrombinaemia in obstructive jaundice?

a- Vitamin K Malabsorption

b- Calcium absorption aippg.com

c- Decreased fibrinogen

d- Liver failing to convert prothrombin to thrombin

A patient presented with tension pneumothorax, what is the immediate management?

a- Intercostal tube

b- Xray followed by intercostals tube

c- Large bore needle at 2nd intercostals space

d- Thoracotomy

Child cam bounch back, can roll over, sit with arched back, can stand with support, no
pincer grasp developed. What may be his probable age?

a- 5 months

b- 7 months

c- 9 months

d- 11 months
e- 15 months

Child was delivered normality with wt. 3.2 kg. Apgar score was 5 in 1 minute, 8 in 5
minutes. Now he has delayed development. Which one of the following can be associated
with his delayed development?

a- maternal uncle has intellectual deficit

b- father is alcoholic

c- Sister had febrile seizures

d- Mother has 2 café au late spots aippg.com

e- Paternal grandma has hypothyroidism

X-ray of the baby shows opacities /consolidation. What may be the cause?

a- Mycoplasma pneumoniae

b- Beta streptococcus

c- Streptococcus pneumoniae

d- Staphylococcus aureus

e- Klebseilla pneumoniae

A full term baby is transferred to postnatal ward one hour after delivery. On arrival she
is noticed to have ctanosed hands and feet. On examination you see that the child is
responsive and cries lustily on handling. Resp. rate is 40/min. rectal temperature is 37.5
degree Celsius. Heart, lungs are clinically normal. WOF statements is appropriate?

a- Oxygen should be given

b- An immediate -ray chest should be arranged

c- Reassure the mother that the baby is OK

d- Infant has early signs of resp. distress


e- The infant has most probably had a convulsion

13yrs old boy with height of 9 yrs old. Which one of the following will support a good
prognosis for further growth of the child?

a- Bone of the child is 13 years

b- Bone age of the child is 9 years

After recovery from fever a child developed a neck swelling which is translucent and is
located in the posterior triangle of the neck. What’s the possible Dx?

a- Cystic hygroma

b- Tubercular lymadenitis

c- Branchial cyst

d- Bacterial lymphadenitis aippg.com

e- Metastatic lymph nodes

Child came with fever & neck stiffness O/E CSF protein was 1.1gm/L, glucose - normal,
Lymphocyte - 100/L, Neutrophil - 50/L. No organism was found. Most likely diagnosis was
-

a- Bacterial meningitis

b- Herpes simplex meningitis

c- Echo virus

d- TB meningitis

e- Typhoid

A 9 month old child has been irritable and febrile for 48hrs. Now he has generalized
erythematous rash and his temp. is 37.2 degree centigrade. What’s the most likely Dx?
a- Herpes simplex 1 infection

b- Rubella

c- Scarlet fever

d- Roseola infantum

e- Erythema infectiosum

Joseph., Nov 2, 2007

#174

Joseph.

Joseph. Guest

A 4 yr old boy was brought to you with fever of 2 days. He was limping and had
tenderness and swelling over the upper end of tibia. What may be the possible cause?

a- Septic arthritis

b- Irritable hi

c- Osteomyelitis

d- Cellulitis

e- Thrombophlebitis

The most common cause of blood stained stool in an otherwise normal infant is -

a- Anal fissure

b- Fistula in ano aippg.com

c- Haemorrhoids

d- Meckel diverticulitis

e- Intussusception
What would mostly be associated with undescended testis?

a- Inguinal hernia

b- Testicular tumour

c- Infertility

d- Torsion of testis

e- Infection

18 months old child was brought by her mother she doesn’t babble. Hearing test
was done at 10 months and was normal. What’s the next step of management?

a- Reassure that the baby is normal

b- Audiogram

c- Check development

A 9 day baby with projectile vomiting and dehydration. Lab tests show:

K+ 7.5, Na+ 125, CI-80, Hco3-20. What could be the diagnosis?

a- Pyloric stenosis

b- Congenital adrenal hyperplasia

c- Galactosemia

d- Sepsis

e- Gastro esophageal reflux

8 yrs old child with headache, morning vomiting, ataxia & nystagmus for 6 wks. His
school had episode of Chicken Pox 2 wks ago. He has a family H/O migraine. What could
be the possible diagnosis/
a- Migraine

b- Post. Fossa tumour/Medulloblastoma

c- Encephalitis aippg.com

d- Varicella cerebellitis

2 yrs old child with 1 month history of unilateral foul smelling sero-sanginous nasal
discharge. Most probable cause is -

a- Foreign body

b- Nasal polyp

c- Sinusitis

d- Tonsillitis

Mother brought her 12yrs. old child with hip pain and limping. He is in the 19th centile
of height and also he is not growing in height. What may be the probable cause?

a- Hip arthrosis

b- Perthe’s disease

c- Transient Synovitis

d- Slipped upper femoral epiphysis

e- Tuberculosis

8 wks old baby with unilateral eye discharge, subsides with antibiotics, but recurs.
Pathology shows polymorph but no organism. What may be the cause?

a- Gonococcal conjunctivitis

b- Blocked naso lacrymal duct

c- Allergic conjunctivitis
d- Chlamydial infection

e- Syphilis

A 4 months old baby with repeated chest infection, systolic murmur over the left sternal
border, not putting on weight and breathlessness while feeding. Apex beat in the mind
axillary line. What may be the diagnosis?

a- VSD

b- ASD

c- Coarctation of Aorta

d- Fallot’s Tetralogy

e- Severe aortic stenosis

3 hours, after birth a new born was found to have respiratory distress and reduced
breath sound over the left side of chest. Abdomen was scaphoid, what may be the
diagnosis?

a- Congenital diaphragmatic hernia

b- Tension pneumothorax aippg.com

c- Staphylococcal pneumonia

d- Sliding hernia

A child was referred by Kindergarten teacher & the parent with the complaint of over
activity, not cooperative enough with other children and destructive behaviour. But when
kept alone, was found to be playing happily with toys and other children. Which of the
following is true?

a- It is a normal variant

b- Poor parenting

c- ADHD
d- Autism

e- Borderline disorder

Joseph., Nov 2, 2007

#175

Joseph.

Joseph. Guest

A child had accidentally ingested dish washing powder. He developed ulcers in mouth
drooling of saliva. What would be the next management?

a- Reassure the parents that nothing will happen & send them home

b- Admission to hospital with possible Endoscopy

c- Ask them to bring the baby back if any problem arises

New born with cataract and jaundice. 2 of his brothers/sisters had the same problem.
What may be the DX?

a- Rubella embryopathy

b- Galactosemia

c- Toxoplasmosis aippg.com

d- Glaucoma (congenital)

e- CMV infection

A 2yrs old boy is seen by his mother to have a swelling in his right groin for a few hours.
He has no other symptoms. Exam. Shows no swelling, both testes are in the scrotum but
right spermatic cord is thicker. What is the management?

a- Arrange for a surgical consultation for inguinal herniotomy


b- Reassure the patient that the baby is OK

c- Reexamine him at weekly intervals

d- Arrange to review the child after some months

e- Rt. Orchidopexy

An 8 yrs old obese boy was brought to doctor. O/E his wt. was 48kg. which is more than
98th percentile and height was 140 cm. which is more than 90th percentile. He is
otherwise normal. Which of the following is the correct statement?

a- Advanced bone age

b- Increased blood sugar level

c- Primary hyperaldosteronism

d- Thyrotoxicosis

e- Hypothyroidism

A 9 month old baby with fever was brought to doctor. O/E their was temp. 37.5 degree
cent. He had cough, runny nose and resp. distress. There was wide spread bilateral
wheeze on auscaultation. 2 other siblings in the house have H/O cold. Which of the
following may be cause in this baby?

a- Acute bronchiolitis

b- Croup

c- Asthma

d- Pneumonia

e- Inhaled foreign body

A 2 yrs old girl is noted by her father having palpitation (about 160/min.) she is
otherwise normal. She developed same symptoms twice before. Which of the following is
the initial management?
a- Administration of beta blocker

b- Verapamil

c- ACE inhibitor

d- Cardioversion

e- Cold water applied to face

All of the following are features of cluster headache except -

a- More in males

b- Starts 3 to 4 hrs. after falling asleep aippg.com

c- Headache present for several days in each episode

d- Vomiting

Which one of the following is used to overcome the muscle relaxant effects of
anaesthesia?

a- Neostigmine

b- Physostigmine

c- Atropine

d- Fluphrenthixole

e- Succinylchline

Patient with Parkinson’s disease getting levodopa+ Carbidopa developed twitching


movement of face. What’s the management?

a- Benzhexol hydrochloric

b- Reduce the existing dose

c- Add anticholinergic
d- Add antipsychotic

e- Increase the dose of existing drug

Antiphospholipid antibody is associated with -

a- Recurrent abortion

b- Increased complement

Joseph., Nov 2, 2007

#176

Joseph.

Joseph. Guest

Which nerve carries taste sensation of anterior 2/3rd of tongue?

a- Facial nerve

b- Trigeminal nerve

c- Glossopharyngeal nerve

d- Hypoglossal nerve aippg.com

Patient developed pain after elevating the leg above the bed. What may be the cause?

a- Venous insufficiency

b- Arterial insufficiency

c- Thrombosis

d- Varicosities of veins
A person wakes up at night with calf pain. When he gets out of bed and walks about he
is relived of the pain. WOF is correct?

a- Ischaemic pain

b- Rest pain

c- Muscle cramp

d- Schiatic pain

e- Disc prolapse

Patient developed joint swelling, Microscopic exam. of joint fluid showed positive
birefringent crystals. WOF is true?

a- Calcium pyrophosphate dehydrate crystal

b- Calcium diphosphate urate crystal

c- Monosodium urate crystal

d- Gout

WOF joints are not involved in Rheumatoid arthritis?

a- Cervical spine

b- First carpo metacarpal joint

c- Distal inter phalangeal joint

d- Metacarpo phalangeal joint

WOF drugs are not hepatotoxic?

a- Halothen

b- Digoxine

c- Warferin
d- Tetracycline aippg.com

e- CC4

WOF does not cause facial nerve palsy?

a- Ca parotid gland

b- Chronic parotitis

c- Mastoiditis

d- Acoustic neuroma

WOF can be associated with massive blood transfusion?

a- Volume overload

b- Coagulation defect

When a man was asked to look at his left side he developed double vision in doing go.

a- Left 3rd nerve

b- Left 6th nerve

c- Optic nerve

d- Lesion at optic chaisma

e- Right 6th nerve

WOF is true of end arterectomy in a patient with more than 80% stenosis of internal
carotid artery?

a- Death rate will reduce by more than 50%

b- If no surgery is done more than 70 % will die in one month

c- Death 24 hours is more than 30%


d- There is 10% reduction in mortality

What is the most common cause of painful pericarditis?

a- Uraemia

b- Viral infection aippg.com

c- Rheumatic arthritis

d- SLE

e- Tuberculosis

Joseph., Nov 2, 2007

#177

Joseph.

Joseph. Guest

Death rate is highest in acute MI -

a- In first 24 hours

b- In first 3 hours

c- In 1-7 days

d- In the first month

A patient presented with ptosis of left eye, left side sensory loss of face. Left side gag
reflex was absent, right sided hemi paresis and incoordation of right upper and lower
limb. Where is the lesion?

a- Internal carotid artery

b- Vertebral artery
c- Middle cerebral artery

d- Post. cerebral artery aippg.com

e- Multiple sclerosis

Regarding acute Hepatitis B infection WOF is true?

a- Hepatitis B surface antigen

b- hepatitis C antigen

Patient developed cardiac arrhythmia. Ventricular rate was 110/min., BP was 90/40
with irregular ventricular contraction on ECG. Patient had impaired consciousness. What is
the immediate management?

a- Adenosine

b- Cardioversion

c- IV lignocaine

d- Valsalva maneuver

e- Cardiac massage

ECG strip given -

a- Lateral MI b- Inferior MI

c- RVH d- LVH

ECG strip given -

a- Sick sinus syndrome

b- WPW syndrome

c- Atrial fibrillation
d- Atrial flutter with variable block

ECG strip of a diabetic patient given -

a- Multiple U waves

A 53 yrs old patient came with chest pain. P/H of taking Nitroglycerine for recurrent
chest pain with improvement. This time ECG was done and was normal & cardiac
enzymes/ troponin was negative. Pain decreased with belching. WOF will be his best
treatment?

a- Refer to gastroenterologist

b- Immediate angioplasty

c- Admit to ICU (?2 hrs.) for continuous monitoring

d- Send home with cardiologist out patient appointment

e- No heart pain & ECG being normal reassure & send him home

Picture of face of a lady with wide spread non pruritic rash, diagnosis?

a- Rosacea b- SLE

b- Dermatitis d- Psoriasis

What does it mean regarding sero conversion to HIV?

a- Life long infectivity

b- Prohibition of sex

c- Treatment with Zudovudin

d- He has AIDS
WOF does not mimic secondary syphilis?

a- Eczema

b- Psoriasis

c- Pityriasis rosea

d- Dermatophytoid

HIV patient had Mantoux positive. WOF is the correct response?

a- INH prophylaxis

b- INH should not be given due to hepatotoxicity

c- Mantoux test is positive due to immuno suppression

d- Treatment with anti TB drug aippg.com

WOF is not pre malignant?

a- Aphthus ulcer

b- Leucoplakia

c- Solar keratosis

d- Sinus tract in chr. Osteomyelitis

e- Skin irradiation in infancy/Thyroid irradiation in infancy

Infectious mononucleosis -

a- Resembles acute HIV

b- Liver function tests usually impaired

c- Diagnosed by anti body test


Joseph., Nov 2, 2007

#178

Joseph.

Joseph. Guest

COR pulmonary secondary to Emphysema. Best management -

a- O2 inhalation b- Digoxine

c- Frusemide d- Bronchodilator

Patient developed tinnitus, vertigo, diminished corneal reflex. What’s the possible
diagnosis?

a- Mennier’s disease aippg.com

b- Acoustic neuroma

55 yrs old lady can’t see properly while in out door. Every thing looks hazy, but she
can read news paper. WOF is the correct response?

a- Cataract b- Glaucoma

c- Myopia d- Macular degeneration

e- Diabetic retinopathy

WOF is true of Paget’s disease?

a- High output heart failure b- Bone pain usually

c- Hypercalcemia d- Hyperphosphataemia
A patient with rheumatoid arthritis developed anaemia which is normocytic
normochromic. Ferritin level was high. What’s the diagnosis?

a- Iron deficiency anaemia

b- Anaemia of chronic disease

c- Haemolytic anaemia

Renin is developed by -

a- Beta blocker b- Alpha blocker

c- ACE inhibitor

A 55 yrs old hypertensive patient developed severe chest pain radiating to back and
dyspnoea of 3 hrs duration. O/E BP was 230/140, absent right carotid pulse, diastolic
murmur along the left sternal border. ECG & cardiac enzymes were normal. What can be
the chest x-ray finding?

a- Widening of superior mediastinum

b- Calcification of aorta

c- Pulmonary oedema

d- Pericardial effusion

e- Cardiomegaly

What is the cause of increased life expectancy of woman in Australia?

a- Males exercise more than females

b- Genetic make up

c- Females seek healthcare facilities more than males

d- Males die more in accidents & violence than females

e- Employment stress is more for males


In WOF conditions hypoxia can be corrected by giving 100% oxygen?

a- lung disease

b- Cyanotic heart disease

c- Methemoglobinemia

d- Carbon monoxide poisoning

A 58 yrs old man was brought to hospital accompanied by his wife with bruise on his
head. She complained that he was binge drinking for past one week. O/E he was confused
& had extensor planter reflexes. WOF may be the probable dx?

a- Hepatic encephalopathy

b- Korsokov’s psychosis aippg.com

c- Central pontine myelinosis

d- Subdural haematoma

e- Dementia

Joseph., Nov 2, 2007

#179

Joseph.

Joseph. Guest

A man has lost ankle jerk and loss of sensation on the lateral aspect of leg and sole of
foot. WOF nerve is affected?

a- L-2 b- L-3

c- L-4 d- S-1
e- S-2

WOF does not mimic secondary syphilis?

a- Eczema b- Psoriasis

c- Pityriasis d- Scarlet fever

e- Dermatofitoid skin condition aippg.com

Regarding haemolytic anaemia -

a- MCV- decreased b- Bilirubin in urine

c- Iron absorption - increased

d- Heptoglobin - increased

e- Haemoglobinurea

WOF is not present after splenectomy for hereditary spherocytosis?

a- Normal life span of RBC.

b- Transient leucocytes

c- Anaemia resolved

d- Persistence of Osmotic fragility

e- Persistence of Spherocytosis

A patient was treated with anti H. pyloric regimen for recurrent duodenal ulcer. WOF is
the proper and quick test to evaluate the effectiveness of therapy?

a- Serology b- Urea breath test

c- Biopsy d- Serum Gastrin level

e- Ba-meal X-ray
A patient developed sudden dyspnoea. Physical exam showed decreased left upper
chest movement. There was hyper resonance to percussion & decreased breath sound on
the left side. What may be the diagnosis?

a- Atelectasis b- Left lung consolidation

c- Left pneumothorax d- Left pleural effusion

e- Right sided emphysema

A young patient presented with haematuria, proteinurea (more than 3.5 gm) 3 days
after a sore throat. WOF is the most likely diagnosis?

a- Nephrotic syndrome

b- Streptococcal glomerulonephritis aippg.com

c- IgA nephropathy

d- Acute pyelonephritis

e- Membranous nephropathy

A young patient presented with uncomplicated chronic duodenal ulcer. WOF is the best
management?

a- Eradication of H. pylori

WOF does not cause carcinoma?

a- Hepatitis C b- EB virus

c- Cytomegalovirus

Right side of chest is swollen and moves less than the left side. Right side is more
resonant but breath sound is louder in the left side. The most likely disorder is -
a- Consolidation on left side

b- Pneumothorax on the right side

c- Collapse on the right side

d- Consolidation on the right side

e- Pleural effusion on the left side

A young man presented with dyspnoea, tachypnoea, hyperinflated left hemithorax with
evidence of mediastinal shift to the right. Breath sound is less on the left. BP is 109/60.
What’s your immediate management?

a- Immediate fluid resuscitation to maintain BP

b- Intubate and secure the air way

c- Insert a wide bore needle in the 2nd intercostal space in the mid clavicular line
aippg.com

d- Insert a needle in the 4th space in the m8id axillary line

A patient with Parkinson’s disease getting levodopa + Carbidopa developed


twitching movement of face. What’s the management?

a- Reduce dose of levodopa

b- Give Benzhexol hydrochloride

c- Increase the dose of levodopa

Joseph., Nov 2, 2007

#180

Joseph.

Joseph. Guest
Renal calculi is associated with all of the following except -

a- Depression b- Hypercalcuria

c- Diarrhoea d- Lytic bone lesion

e- Hyper calcinosis aippg.com

On diagnostic evaluation a patient was found to have normal T3 and T4 but had
decreased TSH. Which of the following drugs can not be give?

a- Propranolol

b- Iodine containing contrast medium

Smoking increased malignancy, all of the following are associated with except -

a- Ca pancreas b- Ca oesophagus

c- Endometrial Ca d- Ca bladder

e- Lung cancer

Patient of chronic renal failure developed osteopenia. WOF is the treatment of choice?

a- Phosphate binder

b- CaCo3

A patient with aortic stenosis had a gradient of 50 mm of Hg. What’s the most
important factor to assess the severity of the disease?

a- End diastolic pressure

b- Left ventricular ejection fraction

c- Presence of coronary disease

d- Cardiac output
WOF combination does not cause drug interaction?

a- Warferin + Nitroglycerine

b- Erythromycin + Terfenadin

c- Warferin + Cimetidine

d- ACE inhibitor + Amerloride

Two days after a moderate MI a 65 yrs old man developed abdominal pain and
diarrhoea with passage of blood. Abdomen is slightly tender. Plain X-ray shows distended
intestine but no fluids level. Serum amylase is slightly elevated. Temp. is 38 degree C. The
most likely diagnosis is -

a- Ischaemic colitis b- Ca of sigmoid colon

c- Diverticulitis d- Acute pancreatitis aa

e- Heparin induced bleeding

A 30 yrs old woman comes to you for a general check up. WOF you will do as the first
thing?

a- Complete body scan]

b- Complete blood examination

c- X-ray chest aippg.com

d- Detailed history and physical examination

e- Ask about her general health concern

Which of the following condition is most benefited from splenectomy?

a- Henoch Schonlein purpura

b- Myelofibrosis
c- Chronic ITP

d- Sickle cell anaemia

Hepatitis B -

a- Epidemiologically similar to hepatitis E

b- Immunization should be given to Gluteal region

c- If it infects in infancy most probably leads to chronic course

d- Both Hep S Ag and Hep E Ag are necessary for the diagnosis

Picture of perianal haematoma was given. A netball player developed sudden onset of
perianal pain came after 24 hrs. What’s the appropriate management?

a- Incision and drainage under local anaesthesia

b- Hot saline bath at 2 hrs intervals

c- Rectal sphincterectomy under GA

d- Antibiotics

Which of the following organism can not be transmitted by blood transfusion except?

a- Hep B b- Hep A

c- Treponema palladium

d- Malaria e- Hep C

CT scan of the brain a space-occupying lesion asked for the diagnosis-

a- Infarction b- Haemorrhage

6. CT scan of the abdomen was given showed a large cyst in the hypochondrium -

a- Pancreatic cyst b- Ascities


b- Ovarian cyst d- Splenic cyst

aippg.com

In which situation you will give TPA with in 4 hrs -

a- All anterior MI

b- All inferior cyst

Joseph., Nov 2, 2007

#181

R.dass

R.dass Guest

With respect to combined the oral contraceptive pill. Which of the following is not true?

a- It reduces the incidence of benign breast disease

b- It reduce the incidence of endometrial cancer

c- It reduce the incidence of ovarian cancer aippg.com

d- It reduce the plasma concentration of triglycerides

e- It reduce the incidence of iron deficiency anemia

Which of the following is associated with a reduced risk of breast cancer?

a- Nulliparous

b- Early menarche

c- late menopause

d- First child under 20 years of age

e- If you breasts feed your children


You discover that a pregnant woman of 30 weeks gestation is positive for streptococcus
B. Which of the following is true?

a- Treat the patient with penicillin immediately

b- Treatment with IV penicillin during labour will significantly reduce the risk of severe
neonatal disease

c- Treat with doxycycline

d- Do not treat because 25% of women are carriers

e- They are part of normal vaginal flora 10-30%

A woman that in a previous pregnancy had a child affected by anencephaly is pregnant


again. With respect to managements of this pregnancy which of the following is not
correct?

a- Amniocentesis at 18 weeks

b- CVB at 11 weeks

c- Ultrasound at 11 weeks to look for malformations

d- Ultrasound at 11 weeks looking for Nuchal thickness

e- Maternal screening at 16 weeks

36 years old primigravida is worried about Down’s syndrome. Which of the following
investigations is the most specific and at the same time has the lowest risk for the foerus?

a- Maternal screening at 16 weeks gestation

b- Amniocentesis at 18 weeks

c- CVB at 12 weeks

d- Ultrasound at 11 weeks to look for Nuchal thickening

e- Cordiocentesis at 24 weeks
In which part of the body is the majority of the sperm blocked?

a- Cervix

b- Vagina

c- Uterus

d- Fallopian tubes aippg.com

e- Isthmus

A woman of 48 years that had a CIN lesion removed, 10 years ago and has recently had
a hysterectomy but has her ovaries left. Which of the following is not true?

a- Woman that have had this type of operation have less chances of getting cancer of
the ovaries than woman that have not had this operation

b- She will need to have vaginal vault smears every 2 years

c- She will enter menopause at the normal age perhaps just a little earlier

d- Oestrogen therapy will be beneficial to her when she enters menopause

e- She will not need progesterone in HRT

A 52 yrs old woman that has had a hysterectomy and post operative DVT comes to see
you complaining of severe hot flushes. Which of the following would you recommend?

a- The combined oral contraceptive pill b- Oestrogen transdermal patches 50 microgram

c- Low dose oestrogen therapy per os

d- Oestrogen and progesterone transdermal patch

e- Progesterone therapy only

A 26 yrs old Nulliparous woman who was recently married and wants to have children
comes to see you complaining of severe menorrhagia. On examination you find that
uterus was completely deformed by uterine fibroids and that her uterus can be felt just
under her umbilicus, and haemoglobin is decreased (4 gram %). Which of the following
would you recommend?

a- Total hysterectomy

b- Open myomectomy after controlling her anemia with blood transfusion

c- Give 3 months therapy with gonadotropin agonists then do an open myomectomy

d- Do a transcervical myomectomy

e- Prescribe her the oral contraceptive pill

A young woman with infertility, recommended to do laparoscopy. She comes to her GP


looking for information about laparoscopy. All the following are true except:

a- A minimum risk for laparoscopy due to damage to the internal organ

b- Risk for damage to aorta or vena cava which can lead to death

c- You can find diagnosis polycystic ovarian syndrome aippg.com

d- Better for diagnosis of endometriosis

e- Pain in the shoulder is common past operatively

R.dass, Nov 3, 2007

#182

R.dass

R.dass Guest

A 50 yrs old woman whose last period was 2 years ago comes with 3 days vaginal
bleeding without abdominal pain. What is the most likely cause?

a- Early stage of cervical cancer

b- Endometrial cancer
c- Occasional follicular hypertrophy

d- Vaginal atrophy aippg.com

e- Anovulatory cycles

Regarding endometrial ablation all is true except:

a- Difficult to operate if the uterus is bigger than 10cms

b- Cause amenorrhoea in 60-80% of the patient

c- Postop you can use just oestrogen without progesterone

d- Is more safe to use roller blade

e- Required long hospital stay

An 18 yrs old female who has multiple partners during the last year comes to you
because of a vaginal discharge. All of the following are true except:

a- Take HIV test

b- Take lower endocervical swab for gonorrhoea

c- Take a swab from the posterior fornix for Chlamydia

d- Treat sexual partner

e- Cervical smear cytology

A 28 yrs old woman G2 PI with a normal vaginal delivery in the past. Now pregnant at 37
weeks and comes to the delivery ward for induction due to pre-eclampsia. The baby lies
longitudinal, cephalic, not engaged. At this time you rupture the membrane and give the
patient a syntocynon-infusion. In related to the start of first uterine contraction the foetus
has Bradycardia of 60. What do you do next?

a- Caesarean section

b- Do abdominal CTG
c- Do vaginal examination

d- Just observe

e- Oxytocin

A woman after 10 hours of obstructive labour delivers by caesarean section. Three days
later she develops fever 37.9c. What is the least likely?

a- UTI

B- Wound haematoma

c- DVT

d- Endometrial infection

e- Engorgement of the breast

A mother brings her 4 year old girl with intermittent yellow vaginal discharge. In the
examination you find that the vulva is normal structurally except little red and the girl
trends to put her hand o the area, with sign of scratching. What is the most likely
diagnosis?

a- Foreign body

b- Monilial infection

c- Gardenella vaginalis

d- Syphilis

e- Sexual abuse

The most benefit from mammogram screening for breast cancer is:

a- Earlier confirmation lump then self palpitation

b- Woman with family history of breast cancer and first degree relatives

c- 50-54 year are group


d- 35-50 year are group aippg.com

e- Woman over 65 yrs of age

A 50 yrs old woman, bleeding after menopause, is taking Warfarin drugs. This is least
likely with:

a- CIN 3

b- Cervical carcinoma

c- Endometrial carcinoma

d- Vaginal atrophy

e- Anovulatory cycles

In an Rh negative mother, al of the following are true except:

a- Immunoglobin during the second trimester can be helpful

b- In ABO incompatibility, Anti-Rh immunoglobin is not necessary

c- Immunoglobin should be given during the first 72 hours after delivery

All of the following treatment regimes regarding candidiasis can be used except:

a- Imidazole

b- Metronidazole

c- Ketocanazole

d- Nystatin

e- Ictracondazole

R.dass, Nov 3, 2007

#183
R.dass

R.dass Guest

A young couple wants to have a child. The husband’s mother had bipolar disorder.
You tell them:

a- The risk is less than 10% in offspring

b- They should adopt a child as the risk is too high

c- They should seek expert genetic counselling prior to having a child

d- Should undergo amniocentesis to decide whether termination of pregnancy

About progestogen in HRT, does all except:

a- decrease the risk of endometrial cancer

b- Decrease risk of cervical cancer

c- Causes premenstrual cancer aippg.com

d- Decreases HDL

e- Augments effect of oestrogen in bone protection

Combine HRT has all the advantages except:

a- Prevent hyperplasia of endometrium

b- The production f some perimenopausal symptoms

c- Production of period bleeding at the end of cyclical progesterone

d- Augmentation of bone protection by progesterone

In post/menopausal woman, what is the least likely cause of greenish foul smelling
vaginal discharge?
a- Trichomonads vaginalis

b- Gardnerella vaginalis

c- Chlamydia

d- Cervical carcinoma

e- Foreign body

A 19 yrs old primi, 38 weeks gestation with dryper reflexia, hypertension and oedema
might be excepted to have elevated blood levels of all of the following except:

a- Uric acid

b- Creatinine

c- Platelets

d- SGOT

e- BUN

A 45 yrs old female who has been into post menopausal period for 6 months duration
started complaining about intermittent bouts of flushes, insomnia, anger, depression and
emotional outbursts. Your treatment for her would be:

a- Reassurance

b- Progesterogen only pills

c- Oestrogen only pills

d- Combination pills

e- Non of the above

Progesterone challenge test includes a 5 days course of medroxyprogesterone acetate


(MPA) 10mg daily. If there is no withdrawl bleeding after stopping MPA, this will indicate:

a- Hypothalamic incompetence
b- Pituitary failure

c- Ovarian failure

d- Uterine pathology

e- Thyroid disorder

Cardiotocography involves a continuous recording of the fetal heart rate. Abnormal


result may mean the presence of:

a- Spontaneous acceleration of the fetal heart rate with uterine contraction

b- Beat to beat variation during sleep

c- Fetal heart rate 140 per minute aippg.com

d- Slowing of the fetal heart rate in relation to movements

e- Short-term variation of greater than 5 beats

Regarding with Asthma in pregnancy. Following statements are true:

a- There is no significant aggravation during pregnancy

b- It affects 0.4% to 1.3% of all gestations

c- Medical treatment includes Terbutaline and Cromolyn sodium

d- Aminophylline is safe in pregnancy

e- No evidence for teratogenicity by using corticosteroids

The highest failure rate of method of contraception is:

a- Condom

b- IUCD

c- OC pills

d- Diaphragm and spermicides


e- Injection depot medroxyprogesterone acetate

R.dass, Nov 3, 2007

#184

R.dass

R.dass Guest

Carcinoma in situ of the cervix:

a- may have extended into endocervical glands

b- Is visible to the naked eye aippg.com

c- Is a malignant invasive lesion

d- Can be visualized colposcapically

e- Can be treated by local destruction

C.I.N. III:

a- Includes cases of severe dysplasia

b- Includes cases of carcinoma in situ

c- Is premalignant

d- Could be treated by superficial cervical cautorisation

e- Often returns to normal spontaneously

Signs of a ruptured octopic pregnancy are:

a- rebound tenderness in the abdomen

b- Heavy vaginal bleeding


c- Cervical excitation

d- Localized tenderness in the pelvic

e- Tachycardia

A patient with an ectopic pregnancy:

a- Always has amenorrhoea

b- Has a positive pregnancy test in 95 percentage of cases

c- May complain of fainting attacks

d- May complain of shoulder tip pain

e- Usually presents with cardiovascular collapse

Aetiological factors in ectopic pregnancy include:

a- Gonococcal salpingitis

b- Tubal surgery

c- Intra-uterine contraceptive devices

d- Tuberculous salpingitis

e- Progestogen only oral contraceptives

Useful procedures in making diagnosis of ectopic pregnancy:

a- Examination under anesthesia

b- Ultrasound scans of the pelvic

c- A pregnancy test

d- Laparoscopy

e- Abdominal X-ray
A pregnant lady with HTN her initial BP was 170/110 and now increases to 170/125, you
will give her treatment all of the following except:

a- ACE inhibitors

b- Frusemide aippg.com

c- Intravenous hydralazine

A lady at 16th week of gestation with in her 2nd pregnancy she has history of birth of a
baby was 4.7kgs and was born by midforcep delivery with fracture or her Clavicle, what
will you do for better management?

a- 3 hour glucose tolerance test

b- Estimation of blood glucose, Ketone and Sugar

Pregnant diabetic patient with IDDM, WOF is correct except-

a- Strict control of blood glucose from 10th week onward

A patient at 17th week gestation present with pain at right iliac fossa, likely-

a- Ectopic pregnancies

b- Appendicitis

c- Red degeneration of myoma

d- Ovarian tension

R.dass, Nov 3, 2007

#185

R.dass
R.dass Guest

Which is the earliest site of rupture of ectopic pregnancy?

a- Midportion (isthmus)

b- Fibriated ampulla

Regarding endometrial Ablation: WOF is not true

a- Amenorrhoea occur in only 60-80% of cases

b- Danazole given prior to ablation therapy aippg.com

c- Ablation will be difficult if length of the uterus is more than 10cm

d- Roller ball scooping is better than Electro-dissection

e- After ablation only estrogen can be given without progesterone

A 68 years old boy lady was taking Warfarin due to MI came with single episode of mild
vaginal bleedings, WOF is the diagnosis-

a- Cervical Ca

b- Endometrial polyp

c- Warfarin side effect

d- Endometrial Ca

A lady around 20 years of age, she was having Prolactin level 2200 IU/L. FSH normal,
TSH normal, Adenoma - 1.8 and it extends up to 3mm from optic chiasma, your initial
treatment will be:

a- Transphenoidal surgery

b- Transfrontal surgery
c- Bromocriptin therapy

A pregnant lady with a well balanced diet, WOF she will be deficient-

a- Vitamin C

b- Vitamin D

c- Vitamin A

d- Phospher

e- None of the above

A lady of 22 years old, she has hirsutism, oligomenorrhea, endometrial, shows - Atypia
and hyperplasia, of endometrium, she was married and sexually active, WOF is correct
except:

a- Clomiphen citrate on 5-9 days/28

b- Medroxyprogesterone 5mgs daily for 25 days

c- Medroxyprogesterone 10mgs daily for 14 days

A pregnant lady at 38 weeks of gestation present with vaginal bleeding 1000mls and
abdominal pain. WOF is least likely:

a- Fetal demise

b- Uterus tense and contracts

c- Head up with longitudinal lie

A primi gravida with ruptured membrane for 6 weeks. Oxytocin drip was given: Cervical
dilation - 4cms, position - mento anterior, no progress of labor. Regarding obstructed
labor which is least likely:

a- Anterior lip of cervix dilated


b- Head 1 cm above ischial spine

c- Per abdominal examination head 4cm above per vaginal examination 5cm below the
ischial spine

d- Presentation mento posterior

To reduce the cervical Ca dramatically in Australia WOF is appropriate:

a- More availability of Papsmear

b- More availability of cervical screenings

c- Colposcopy aippg.com

d- Early treatment of CIN

e- To invent vaccine to Human papiloma virus

A man 25 years old married a woman of 22 years old for 3 years FSH, LH level of the
body is normal, but she is not conceiving, All of the following condition are appropriate for
IVF except:

a- Mild endometriosis treated

b- Sperm count<1,000,000/mm

c- IVF with microinjection

R.dass, Nov 3, 2007

#186

R.dass

R.dass Guest

Mammography is-
a- Painless

b- Can detect Ca earlier than self breast examination

c- It is more beneficial in 30-40 years age group

d- Difficult in defection after 60years aippg.com

For induction of ovulation you can do all except:

a- HMG

b- Oral Clomiphene citrate

c- GnRh + HMG

d- BHCG

What is the purpose of screening in general population?

a- To detect the disease in early stage and protection

b- Natural history of the disease of the disease can be changed in early detection

A woman came after delivery of a child. What contraceptive she will take for the next
one year, which failing rate that is <2% except:

a- She will breast feed the baby and she will be Amenorrhea

b- OCP

A primigravida is in labour for 10 hours. Bishop’s pre-induction cervical scoring


indicates cervix as 2cm dilatation, 40% effacement, posterior position, consistency firm
and head station as + 1. Fetal is in longitudinal lie and cephalic presentation. Which of the
following is the most appropriate method?

a- Surgical rupture of membranes followed by intravenous Syntocinon

b- Intravenous Syntocinon until cervix is 3cm and then surgical rupture of membranes
c- Intravenous prostaglandin E2 gel 1-2 mg

d- Intracervical Foley’s catheter

e- Intravaginal prostaglandin F2 alpha

The following are the conditions causing Polyhydramnios except:

a- Anencephaly

b- Spin bifida

c- Hydrops fetalis

d- Duodenal atresia

e- Maternal heart disease

The diagnosis of pre-invasive lesion is confirmed in a 50 years old lady. You would
perform which of the following: (TYPE-J)

a- Cone biopsy

b- Cautery

c- Large-loop excision of transformation zone (LIETZ)

d- Colposcopy and punch biopsy

e- Curettage

A 23 years old patient is diagnosed as cervical erosion. She also has troublesome mucus
discharge per vagina and is causing distress to the patient. The most appropriate
management is:

a- Reassurance

b- Vagina pessary

c- Cervical cauterization by diathermy


d- Hysterectomy aippg.com

e- Broad spectrum antibiotics

Clinical management of abnormal uterine bleeding in a middle aged patient who has no
apparent pelvic pathology includes the followings except:

a- Non-steroidal anti-inflammatory drugs (NSAIDs)

b- Endometrial abalation by laser

c- Monophasic contraceptive pills

d- Dilatation and currttage (D&C)

e- Danazol

On third day after birth, a 2 kg baby had bilirubin level of 250 mmol (<200). Which of
the following is correct?

a- Most likely cause is immaturity liver enzymes

b- Usually exacerbate by breast feeding

c- Kernicterus is common if untreated

d- Is an indication for urgent exchange transfusion

R.dass, Nov 3, 2007

#187

R.dass

R.dass Guest

32 weeks pregnant mother comes for the first time with B/P of 180/125,
proteinuria>8g, with no oedema. All of the following are true except:
a- MgSo4 infusion should be started prophylacticaly

b- Methyldopa 500mg tds should be given

c- She should be delivered within 48 hrs

d- Betamethesone should be given Aippg.com

e- Continuous CTG monitoring has to be done

On examination of a pregnant woman, the foetus was term, with vertex presentation,
longitudinal lie and head at 3 station. ARM done, clear liquor, and she was also started on
Oxytocin drips, five minutes after commencing oxytocin, on auscultation with pinards
fetoscope FSH was <60/min corresponding with uterine contraction. What is your next
step of action?

a- Do a carditocography

b- Do a fetal scalp blood sampling

c- Do another PV examination

d- Do an ultrasound examination

A 26 years lady comes with lower abdominal pain, and she had her last menstrual
period (regular cycle) two weeks ago. Ultrasound scan was done and reported as, Uterus-
normal, No fibroid seen. Both ovaries- normal size and texture, Lt. Ovaries has 3 cyst
measuring 0.7cm, 2.2cm, and 3.0cm. “No neovascularisation seen”

What is your next step of management in this patient?

a- Reassurance her that nothing is wrong

b- Laparoscopy and aspiration of the 2.2cm cyst

c- Rescan the patient in two wks time

The most common site for endometriosis is:

a- Broad ligament
b- Round ligament

c- Utero sacral ligament

d- Recto vaginal pouch

e- Infundebulo pelvic ligament

The most accurate method of assessing excepted due date is:

a- Vaginal ultrasound at (8) weeks

b- Bimanual vaginal exam (8) weeks

c- Bimanual exam (18) weeks

d- Abdominal ultrasound at (18) weeks

e- X-ray

Within the last ten years the most modern advancement in management and treatment
of diabetic pregnancy is:

a- Ultrasound examination of malformed fetus

b- Home blood glucose monitoring

c- Examination of glycosalated haemoglobin in RBC

d- Paediatrician can correct congenital heart disease during neonatal period

Relative contraindication to contraceptives includes of AOF except:

a- Previous pulmonary thrombosis

b- DVT

c- Migraine headache

d- Previous pregnancy with eclampsia

e- Hypertension
Increase breast milk production could be result from:

a- Increase water intake

b- A strong sucking infant

c- Message breast during and (n between) after the feed

d- Let down reflex aippg.com

e- Stilbestrol

The most common association with lumbar epidural:

a- Hypotension d- Dural tap

b- Hypertension e- Faetal Bradycardia

c- Headache

Ashmann’s Syn. The most diagnostic procedure:

a- Uterine sound

b- colposcopy

c- Laptscopy

d- Hystrosalpingography

e- d and c

R.dass, Nov 3, 2007

#188

R.dass

R.dass Guest
A female taken contraceptive is also on warferin was given amoxyllin for mild UTI
suddenly develop slight vaginal bleeding most likely explaination is:

a- Amoxyllin interfere with contraceptive absorption

b- Amoxyllin interfere with warferin through bacteria in the gut which affect absorption
of warferin

c- Amoxyllin inhibit bacteria flora which stop vitamin K absorption

d- Used extra method of contraception aippg.com

e- Stop warferin

A 26 yrs old female with 20wks pregnancy comes to you with concern. She had 2
previous abortions for which there is no pain and she didn’t notice any bleeding. What
is the best management?

a- Salbutamol

b- Bed rest

c- Stitch the cervix

d- NT androgens

e- Progesterone

The most significant cause of infertility in a male with the following semen analysis is:

a- Abnormal from more than 70%

b- Volume 1.8ml

c- Mobility 10%

d- Sperm count 10 million

e- Fructose low in the serum


WOF is in correct concerning endometrium abalation:

a- Progesterone

b- High estrogen could be given as replacement after abalation

c- Oral contraceptive

Concerning Hydriomios WOF is incorrect:

a- Acute hydriomios is more common than chromic hydriomios

b- Associated commonly with multiple pregnancy

c- Associated more commonly with diabetes

d- Associated with Rh incompatibility

AOF can cause secondary amenorrhoea at the level of hypothalamus except:

a- Excessive weight loss

b- Heavy exercise

c- Prolactinemia

d- Sheehan’s Syn.

e- Stress

Concerning oxytoxin drip WOF is not correct:

a- Uterine rupture

b- Water intoxication

c- Faetal distress hypoxia

d- Hypotension? Hypertension

e- Tonic uterine contraction


When does the fertilized ovun nidus, itself in to the endometrium?

a- 2 to 5 days after ovulation

b- 5 to 7 days after ovulation

c- 6 hrs after fertilization

d- 10 days after ovulation

e- 5 to 7 days after fertilization

WOF site is the location where most of reduction of the sperm count occurs:

a- Vagina

b- Cervix aippg.com

c- Endometrium

d- Follopian tube

e- Comu

Concerning uterine abalation most effective method is:

a- Progestrone

b- High estrogen could be given as replacement after abalation

c- Oral contraceptive

R.dass, Nov 3, 2007

#189

R.dass

R.dass Guest
Concerning Bulimia Nervosa common persistant (presentation)? Association is:

a- Mark obesity

b- Constipation

c- Depression aippg.com

d- Very good psychotherapy and behavioural therapy

e- Good training and well control diet

Childhood Autism commonly associated with AOF except:

a- Sign develop early in childhood

b- Look people as mechanical objects

Dequaviean’s tenosynovitis WOF is true:

a- NV is compressed median

b- Pain on movement of the thumb

c- Cause pain in the wrist

d- Swelling at the wrist

After mastectomy and radiation treatment of breast cancer oedema of the hand is due
to:

a- Obstruction of lymphatic system

b- Excision of lymph nodes in the axilla

Soon after splenectomy in a patient with congenital spherocytosis peripheral blood film
will show:
a- Leucocytosis

b- Decreases survival of RBC

c- Increase platelets count

d- Spherocytes can be seen

AOF are contraindication in pregnancy except:

a- Lithium

b- Thalidomide

c- Nitrofuratoin

d- Sulfurxazole

e- Dalatin

35 yrs old female with PPH of fresh blood per vagina BP 95/50mmHg, pulse 129/min,
uterus small and well contracted, placenta membrane seems to be intact the most likely
cause is:

a- Ruptured uterus

b- Laceration of vagina and cervix

c- retained pieces of placenta

Management of primary Dysmenorrhoea include all except:

a- Mefanamic acid

b- Indomethacin

c- Bromocryptine aippg.com

d- Oestrogen and progestrone


Which of the most appropriate method to detect gestational age

a- Bimanual examination at 8weeks

b- Transvaginal examination ultrasound at 6 wks

c- Abdominal ultrasound at 18 wks

Which of the following drugs in high dose in late pregnancy causes neonatal
neurological problem?

a- Benzodiazepam

b- Antipsychotic drug

c- Anticonvulsant

d- Narcotic analgesic

e- TOA

R.dass, Nov 3, 2007

#190

R.dass

R.dass Guest

Risk of breast cancer AOF are true except:

a- Artificial early menopause

b- Cancer in the other breast

c- Cancer in the body of uterus

d- First pregnancy after 35 yrs aippg.com

e- First H/O breast cancer


Regarding pulmonary embolism AOF true except:

a- Massive pulmonary embolism usually is consistent with clinically proved DVT

b- Symptoms depends on site, size and number of embolism

c- Emboli in pulmonary artery may cause

The following are of use in treating essential hypertension:

a- Frusemide

b- Hethyl dops

c- Labetalol

d- Propranolol

e- Bed rest

An unstable lie is associated with:

a- Prematurity

b- Placenta pracvia

c- Uterus didelhys

d- A fundal fibroid

e- Grand multiparity

Patients with rheumatic heart disease who become pregnant:

a- Should always be given iron and folate supplements

b- Commonly develop pre-eclampsia

c- Commonly develop atrial fibrillation

d- Should have operative procedures covered by prophylactic antibiotics


e- Usually remain asymptomatic

Pregnant patients diagnosed as having mitral sterosis:

a- Have usually been diagnosed before the pregnancy began

b- should usually have a termination of pregnancy

c- should be considered for mitral valvotomy during pregnancy

d- should have an elective C. S. performed at 37 wks

e- should not routinely be given ergometrine in the 3rd stage of labour

A high head at term could be true to:

a- Wrong dates

b- Placenta praevia

c- Cephato-pelvic disproportion

d- Placental abruption

e- An occipito-posterior position

The following drugs are effective in stopping premature labour:

a- Pethidine

b- Ritodnine

c- Ethyl alcohol

d- Diazepam

e- Aspirin

A 17 yrs old girl with soft fluctuant mass 4cm inside the introitus which is laterally
located:
a- Mullerian

b- Wolffian aippg.com

c- Bartholin’s

d- Squamocolumnar

e- Hymenal

Treatment of endometriosis:

a- Bromcergocryptine

b- GnRh agonist

c- Danazol

d- Oral contraception

e- Pregnancy

R.dass, Nov 3, 2007

#191

R.dass

R.dass Guest

Distinguish ascities from ovarian cyst:

a- Shifting dullness

b- Tympony laterally and dullness anteriorly

c- Tympony anteriorly and dullness laterally

d- Fluid wave aippg.com

e- Diminish intestinal peristalsis


Sequence of event pubertal girl:

a- Breast budding, accelerated growth, public hair, axillary hair

b- Accelerated growth, breast budding, public hair, axillary hair

c- Breast budding, public hair, accelerated growth, axillary hair

d- Breast budding, axillary hair, public hair, accelerated growth

e- Breast budding, public hair, axillary hair, accelerated growth

A 65 yrs old woman has decreased interest in sexual activity. The most explanation is:

a- Decreased lubrication

b- Prone to vaginitis

c- Vaginal atrophy

d- Relaxation of pelvic muscular support

e- Sexual orgasm is lost

Perinatal death:

a- No. of stillbirth /1000

b- No. of neonatal death /1000

c- No. of stillbirth and neonatal death /1000

d- No. of stillbirth

e- No. of neonatal death

Cervical Ca is more common in:

a- Nulliparous
b- late menopause

c- Divorced woman

d- Sex first time at 27 years

e- Early sex expenence

Which is the commonest site from anaerobes?

a- Brain abscess

b- Endocarditis

c- Infected prostheses

What is the most common mode of transmission of Hep C in Australia?

a- vertical transmission

b- IV drug users

c- Food

d- Sexual contact

e- Droplet

A farmer suddenly had undulant fever for 2-3 days associated with abrupt onset of
headache, severe myalgia, jaundice, and Petechial rash on the skin. Hepatosplenomegaly
is present. Which of the following is the most appropriate diagnosis?

a- Brucellosis

b- Yellow fever

c- Leptospirosis

d- Malaria aippg.com

e- Anthrax
Advantages of coronary angioplasty stent are:

a- Decreases the risk of acute complications

b- Decreases risk of restenosis

c- Both a + b

d- Increases the risk of acute complications

e- Increases risk of restenosis

R.dass, Nov 3, 2007

#192

R.dass

R.dass Guest

Concerning renal artery stenosis all are true except:

a- Commonly associated with atherosclerosis

b- Hypekalemia

c- Hypokalemia

A patient comes to the ER severely shocked; pulse is 120. What is the next step in
management?

a- Wait for blood cross matching

b- Give O-ve blood immediately

c- Give iv fluids

Which of the following is the unlikely sign of Mycoplasma pneumonia?


a- Pleuritic chest pain

b- fever

c- Cough

d- Headache

All are true about lactose intolerance except:

a- Increased glucose<1mmol after 50gm of glucose

b- Steatorrhea

c- Seen more in Asian people aippg.com

d- Normal small bowel histology

In a patient with cardiac arrest which of the following should be given first?

a- Adrenaline

b- Lignocaine

c- Atropine

d- Sodium bicarbonate

e- Isoprenaline

A male patient with COPD has an infective exacerbation and received 28% oxygen. He
then became drowsy and ABG analysis showed PaCO2 61, PaO2 68. What is the first step
in management?

a- Immediate intubation and ventilation

b- Decrease O2 and Repeat assessment and ABG analysis 30mins later

c- Give immediate antibiotic cover against gram negatives

d- Give higher O2 concentration


e- Start aminotryptillin I/V infusion

A patient with COPD was confused and was given 10ml/min O2 in the ambulance. He is
now unresponsive. Which is the most likely result of an ABG taken now?

a- PO2=40, PCO2=100, Ph=7.35

b- PO2=100, PCO2=10, Ph=7.40

c- PO2=60, PCO2=45, Ph=7.50

d-v PO2=40, PCO2=100, Ph=7.15

e- PO2=100, PCO2=60, Ph=7.25

13 yrs old girl with irregular bleeding came after 1year of menarche. which persist for 10
days, you will do all of the following except:

a- Clomiphen citrate given for ovulation if she desire for pregnancy

b- estrogen

c- Progesterone

d- OCP

R.dass, Nov 3, 2007

#193

R.dass

R.dass Guest

A 16 yrs old girl present with scanty pubic and axillary hair, her pubic hair develops at 13
yrs of ago, she has no breast enlargement, she is shy and do not allow to her perineum
examination. You will do all of the followings investigations except:

a- CT scan of sela tersica


b- Pelvic USG through abdomen

c- Prolactin level

d- FSH level

e- LH level aippg.com

Effect of progesterone. All of the following except:

a- Increase HDL

b- Increase LDL

c- Fluid retention

d- Change of voice

Postmenopausal woman came to you, gave him her prophylactic HRT, estrogen
0.625mgs and Progesterone for 14 days. You will tell her all of the following are effect of
progesterone except:

a- It will prevent her change of endometrial carcinoma

b- Cardio-profile effect of estrogen will be corrected

c- It will augment ostioporetic effect of estrogen

18 yrs old lady with HTN and taking ACE inhibitors and well controlled; he wants to be
pregnant. You will give her all of the following except:

a- Change the ACE inhibitors and use other anti-hypertensive drugs

A lady with cystic fibrosis, she wants to be pregnant. You will advice her-

a- She should not be pregnant

b- She should adopt a child


c- Through it is risky for her she can be pregnant with modern therapy if she strongly
desire

A woman at 18 wks of twin gestation, during counseling you will tell her all except:

a- Frequent visit

b- Review during third trimester, she may develop

c- if any pain develops, she will come to the hospital

d- B-methasone to 30th week

Mullerian agenesis, which is not true:

a- karyatype 46XX

b- Normal pubic hair

c- Normal breast form

d- X-linked recessive

A 21 yrs old 12wks gestation, on examine uterus is 14 wks size. Something appeared on
the introitus, suction D&C was done & removed a tissue which is hydrophic, WOF is true:

a- HCG level determination weekly till it becomes normal

b- Telling the patient not to get pregnant and give OCP

Danazol effect:

a- Cleft lip aippg.com

b- Cardiac anomalies

c- Female psuedohermaphroditism

d- Limb malformation
WOF can procedure retention of urine except:

a- Fibroids

b- Haematocolpus

c- Retroverted uterus

d- Wart obstructing the genital tract

e- Spinal cord lesion

R.dass, Nov 3, 2007

#194

R.dass

R.dass Guest

A 25 yrs old male present for infertility on examine is tall, small testis gypaecomastia,
Dx?

a- Hlinefelter syndrome

b- Testicular syndrome

c- Down syndrome aippg.com

d- Turner syndrome

e- Marfan syndrome

HCG is a glycoprotein with Alpha & beta subunit, which is true:

a- ACTH like effect

b- Is principally produced by syncitiotrophoblast

c- Peaks at 28-32 wks


d- It will continue to be high at 16wks

Asymptomatic bacteriuria, which is true

a- WBC at 100,000

b- 30% will be symptomatic

c- Presents in 5-6% of female population

d- Needs short course antibiotics

e- Streptococcus faecalis is the predominant organism

You are addressing a class of nurses about a pregnant woman who had a previous
caesarean section for fetal distress. What would be your response when you are asked
about the chances of this pregnancy being a normal delivery?

a- 90%

b- 75%

c- 50%

d- 20%

e- 10%

A primigravid patient was in the first stage of labour for 14 hours duration. After third
days of delivery, the patient developed pyrexia of 38°c. Which of the following is the
commonest cause for this rise of temperature?

a- Engorged breast

b- Upper genital tract injuries leading to endometritis

c- Breast abscess

d- Episiotomy with haematoma formation

e- Pulmonary embolism
A woman who has primary postpartum haemorrhage is examined and noted that her
uterus is small and well controlled. Her blood pressure is 85/50mmHg and there was no
missing cotyledons or fragmented membranes detected during post delivery check. The
reason for her bleeding would be:

a- Related placental tissue

b- Ruptured uterus

c- Vagino-cervical tears

d- Clotting disorder aippg.com

e- Uterine atony

Painless jaundice in pregnancy is manifested by:

a- Intrahepatic cholestasis of pregnancy

b- Acute fatty degeneration of pregnancy

c- Hepatitis B

d- Severe hyperemesis gravidarum

e- Elevated progresterone level during pregnancy

Commonest organism causing bartholinitis is:

a- Staphylococcus

b- Streptococcus

c- Associated with Neisseria gonorrhoeae

d- Clostridium welchii

e- Coliform organisms
Primary dysmenorrhoea can be effectively treated by:

a- Paracetamol

b- Prostaglandin inhibitors

c- Cervical dilatation

d- Oral contraceptive pills

e- Hysterectomy

A woman who had a child with previous marriage married to another man for 1 year.
Her husband had left sided orchidectomy last 8 years ago. They have normal sex life but
they do not have children yet. What would be the cause of male infertility in this
situation?

a- Spermatocoele

b- Diabetes mellitus aippg.com

c- Varicocoele

d- Past history of testicular torsion

e- Encysted hydrocoele of the cord

R.dass, Nov 3, 2007

#195

R.dass

R.dass Guest

After delivery, an infant develops prominent neurological depression for a prolong


period. Which of the following medications used in the late pregnancy will produce this
condition?

a- Phenytoin
b- Carbamazepine

c- Sodium valproate aippg.com

d- Phenobarbitone

e- Benzodiazepine

Antepartum hemorrhage (APH) in primigravida can be best diagnosed by:

a- vaginal examination

b- Localization of placenta by ultrasound

c- Inspection of cervix with Sims speculum

d- Cross-match blood

e- Amniocentesis

A primigravida is in labour for 10 hours. Bishop’s pre-induction cervical scoring


indicates cervix as 2cm dilatation, 40% effacement, posterior position, consistency firm
and head stations as+1. Fetus is in longitudinal lie and cephalic presentation. Which of the
following is the most appropriate method?

a- Surgical rupture of membranes followed by intravenous Syntocinon

b- Intravenous Syntocinon until cervix is 3cm and then surgical rupture of membranes

c- Intravaginal prostaglandin E2 gel 1-2mg

d- Intracervical Foley’s catheter

e- Intravaginal prostaglandin F2 alpha

The followings are the conditions causing polyhydramnios except:

a- Anencephaly

b- Spina bifida
c- Hydrops fetalis

d- Duodenal atresia

e- Maternal heart disease

The diagnosis of pre-invasive lesion is confirms in a 50 years old lady:

You would perform which of the following : (TYPE J)

a- Cone biopsy

b- Cautery

c- Large-loop excision of transformation zone (LIETZ)

d- Colposcopy and punch biopsy

e- Curettage

A 23 yrs old patient is diagnosed as cervical erosion. She also has troublesome mucus
discharge per vagina and is causing distress to the patient. The most appropriate
management is:

a- Reassurance

b- vaginal pessary

c- Cervical cauterization by diathermy

d- Hysterectomy aippg.com

e- Broad Spectrum antibiotics

Clinical management of abnormal uterine bleeding in a middle aged patient, who has no
apparent pelvic pathology includes the followings except:

a- Non-steroidal anti-inflammatory drugs(NSAIDs)

b- Endometrial abalation by laser


c- Monophasic contraceptive pills

d- Dilatation and currttage (D & C)

e- Danazol

Many women with cystic fibrosis are now reaching child bearing age. Gynaecologists
should advice them that:

a- Contraception is not necessary because fertility rates are low

b- They should never reproduce as their child has a very great chance of having the
disease

c- They should not conceive as the antibiotics usually used to treat their respiratory
infections are contraindicated in pregnancy

d- Pregnancy may in fact, be lift-threatening but successful delivery may usually be


accomplished if strongly desired

e- When they deliver, general anaesthesia should be employed to minimize stress of


delivery

A 32 yrs old ready to discharge from hospital after having a baby. She is breast feeding
and asks for contraception method with less than 2% failure. She wants to have another
baby after breastfeeding this one for 12 months. What would you advice her?

a- Full breastfeeding is a safe contraceptive method provided no periods happen and


the baby is not given additional food apart from breast milk

b- IUD

c- Mini pill (progesterone only)

d- Depot provera

e- Dianne 35- if she gets periods and fails to breastfeed the baby

In twin delivery, after the first one is born, cord is tighted. You do all except:
a- Check there is no bleeding from the uterus

b- Check if the second twin’s lie aippg.com.

c- Commence external diversion if second twin is in breech position

d- Commence oxytocin if second twin is vertex and contractions have not resumed
spontaneously

e- CTG to be performed to monitor the heart rate if second twin

R.dass, Nov 3, 2007

#196

R.dass

R.dass Guest

All of the following CTG changes are considered to the normal except:

a- Acceleration with control

b- Acceleration with sound stimulation aippg.com

c- Very regular rhythm, with beat to beat variation <5-min

d- Decrease variability during sleep

e- Acceleration with foetal movements

A 16 yrs old girl had height spurt at the age of 13 when in grade 5. with breast and pubic
hair development during puberty but does not have a period. She is too shy for the most
sensitive investigation except:

a- pelvic ultrasound

b- Chromosomal studies

c- Intra vaginal ultrasound


d- Progesterone, FSH/LH level

e- CT pituitary gland

A 13 yrs girl had first period a year ago. She has used the Monophasic CP since then and
has had only 2 periods at irregular intervals. Your advice would be:

a- Continue OCP and review in six months

b- Change to triphasic OCP

c- Surge of oral contraceptive pill

d- She will need stimulation of ovulation in her fertility years if she wants to become
pregnant

A 23 yrs old woman with 2 years old child is found to have a very high level of prolactin
and on CT scan is found adenoma in the anterior pituitary of 1.6mm in diameter about
3mm away from chiasma in optic and she doesn’t have visual disturbance. What is the
best treatment?

a- Transphenoidal surgery

b- Transnasal surgery

c- Bromocriptine

d- Radiotherapy

e- Chemotherapy

OCP with phenytonin what is correct:

a- Microgyunone 50 plus progesterone

b- Oestrogen 35 plus progesterone

c- Progesterone

d- Oestrogen 125-75-50 progesterone


e- Oestrogen 15 plus progesterone

When labour becomes obstructed after 24 hours duration, the following may be
excepted?

a- Maternal Bradycardia

b- Foetal Bradycardia aippg.com

c- Transverse arrest of the foetal head

d- Maternal hypotension

e- Pronounced moulding of the foetal head

Ectopic pregnancy commonly present as:

a- The patient may be faint with a tender abdomen

b- Shoulder tip pain

c- Mass inn LIE or RIE

d- Sudden severe pain, peritonism and shock

e- Hyperpyrexia, hypotension and tachycardia

Pulmonary disease commonly seen after the delivery is:

a- Asthma

b- Amniotic fluid embolism

c- Bronchitis

d- Atelectasis

e- Bronchopneumonia

Rate of the Caesarean section is decreased in all of the following diseases except:
a- Dystocia

b- fetal distress

c- Breech

d- Other unstable lie

e- Fetal abnormalities

Which of the following is most common cause of low milk production?

a- Low intake of water

b- Mother is very tired

c- Infant is put to breast very frequency

d- Infant is put to breast for less time and there is no proper let down reflex aippg.com

R.dass, Nov 3, 2007

#197

R.dass

R.dass Guest

On third day after birth, a 2kg baby had bilirubin level of 250 mmol(<260). Which of the
following is correct?

a- Most likely cause is immaturity of liver enzymes

b- Usually exacerbated by breast feeding

c- Kernicterus is common if untreated

d- Is an indication for urgent exchange transfusion


32 wks pregnant mother comes for the first time with B/P of 180/125, proteinuria>8g,
with no oedema. All of the following are true except:

a- MgSo4 infusion should be started prophylacticaly

b- Methyldopa 500mg tds should be given

c- She should be delivered within 48 hrs

d- Betamethesone should be given

e- Continuous CTG monitoring has to be done aippg.com

A 22 yrs old married for one year has come to consult you because of infertility. She is
been having regular intercourse for the last six months. What is your management
regarding this patient?

a- Gamete intra fallopian transfer (GIFT)

b- Give her a temperature chart and explain her about ovulation time, fertile period,
and review her after 4 wks

c- Do a hysterosalphingography

All of the following can cause Hyperprolactinemia except:

a- pituitary adenoma

b- Phenothiazines

c- Breast engorgement

d- Oestrogen

e- Renal failure

A 26 yrs old lady with irregular cycles for a longtime had cycles once in 4-10 wks
intervals. Her LMP at that time was 6/52 before. On routine examination for her
amenorrhea:
FSH 3IU (13)

LH 850 IU (4-10 follicular phase, 20-100 midcycle)

Prolactin 450 (50-100)

What is the most likely cause for amenorrhea?

a- Premature ovarian failure

b- Polycystic ovarian disease

c- Stress due to recent marriage

d- Early pregnancy

e- LH secreting pituitary aippg.com

A 42 yrs old lady had menorrhagia and was anaemic, for which she under went a D&C
three wks ago and no abnormality was seen. She has never had hormonal treatment
before. All of the following can be given to her except:

a- Tranexamic acid

b- Prostaglandin synthetase inhibitors

c- Norethesterone

d- LHRH

e- GnRH

All of the following are side effect of progesterone except:

a- Weight gain

b- Menstrual irregularities

c- Amenorrhea after 2-3 injections

d- Reduced chance of fertility for 6-12 months after last injection


R.dass, Nov 3, 2007

#198

R.dass

R.dass Guest

In the drug treatment in Psychiatry enzyme Cytochrome P450 is important especially for
the antidepressant drugs. Which of the following statement is true regarding enzyme
P450 -?

a- it explains the interaction between the antidepressant and the other drugs

b- Causes rush symptoms in the brain

c- They are elevated by the neuroleptic drugs aippg.com

d- They make the neuroleptic drugs lipid soluble

The use of which of the following widely used antipsychotic drug has recently been
suspended in Australia for its effect on heart causing prolongation of the QT interval:

a- Clozapine

b- Thioridazine

c- Fluphenazine

d- Chlorpromazine

e- Risperidone

Elderly abuse is becoming increasing in Australia and is a great concern. Which of the
following statement regarding elderly abuse is true?

a- Elderly female are more likely to be abused than elderly male

b- More often by unknown person

c- The abusers are more to be mentally ill


d- Mentally ill are less likely to be abused

e- Physical illness of the elderly reduce the risk of abuse

A 35 yrs schoolteacher came to you with the history of difficulty in sleep, irritability and
anxiety. When he goes to bed all minor mistakes he did in the day comes in to his mind
and he argues with them himself. These thoughts come intrusively in to his mind and give
him too much distress though he realizes that these thoughts are irrational. He tries to get
rid of the thought but is unable to do that. He has to resolve the arguments before he can
get in to sleep. He is perfectionist and wants things to be done in a way he likes. Which of
the following drug would be most beneficial for him?

a- Benzodiazepine

b- Non Benzodiazepine hypnotic

c- SSRI

d- Atypical antipsychotic

e- Antipsychotic/Antidepressant

A 45 yrs old man was hospitalized after cycle accident where he hit a lamp-post. He has
fracture of the femur and bruises over the chest. There is no apparent injury in the head.
On 3rd day he became suddenly confused and disoriented. What is the most probable
cause?

a- Subdural Haematoma

b- Alcohol withdrawal aippg.com

c- Acute anxiety state

d- Schizophrenic psychosis

e- Hypersensitivity to drugs

60 Years old patient presents with headache and sudden monocular visual loss. What
test should immediately order?
a- Complete blood count

b- Sedimentation rate

c- Skull x-ray

d- Chest x-ray

e- Urinalysis

A patient presents with a painful arc of abduction at the shoulder joint in the range
from 60-120 degree. Which of the following conditions would cause this?

a- Osteoarthritis of the shoulder joint

b- Partial tear of the rotator cuff muscle

c- Fractured neck of humerus

d- Arthritis of the acromlo-clavicular joint

e- Subcaspination calcification

A 37 yrs old man requests a check up. He says he is quite well and has no particular
worries. There is no significant past or family history. The only positive finding on
examination is a BP reading of 160/110. Urinalysis is normal. The doctor should:

a- Arrange for an IVP

b- Commence a trial of Thiazide diuretic

c- Advise patient to return for review in one week for another examination

d- Prescribe a B blocker drug and review in two weeks

e- Arrange chest x-ray and ECG

The most serious side effect of tri-cyclic drugs involves:

a- The genitor-urinary tract


b- The salivary glands

c- The elementary tract

d- The cardiovascular system

e- The respiratory system

A patient aged 60 yrs presents with bleeding per vaginam. Two years previously she had
the same complaint and at that time uterine curettage revealed no abnormality. The best
management would be:

a- Hysterectomy

b- Colposcopy aippg.com

c- Haemoglobin estimation

d- Culdoscopy

e- Hysteroscopy

R.dass, Nov 3, 2007

#199

R.dass

R.dass Guest

Arthritis is common in which of the following conditions

a- Rubella

b- Mumps

c- Measles aippg.com

d- Infection mononucleosis

e- Varicella
In which of the following conditions of the colon is malignant change most likely to
occur?

a- Adenomatous polyp

b- Melanosis coli

c- Diverticulitis

d- Familial intestinal polyposis

e- Ulcerative colitis

In the diagnosis of hypogonadism the following are true -

a- Low testosterone with high FSH more than LH levels indicate a primary seminiferous
tubule dysfunction

b- Low testosterone with raised FSH & LH indicate a pituitary or hypothalamic lesion

c- Low testosterone with low normal FSH & LH indicate a primary testicular lesion

d- Low FSH levels with a decreased sperm count indicate isolated FSH deficiency

e- High serum testosterone and FSH & LH indicate a disorder of androgen action

In the diagnosis of hypogonadism the following are true -

a- An extra X chromosome is diagnosed by finding a Bar body in a buccal sinear

b- The karyatype may be obtained by examining the peripheral blood lymphocyte

c- In Klinefelter’s (KF) syndrome the testicles are small but firm

d- In KF, unlike other hypogonads aggression may be a feature

e- A normal XY karyatype is compatible with a diagnosis of KF

The following statements are true -


a- Orchitis in pre-pubertal boys due to mumps resolves completely, usually without
sequelae

b- Spironolactone decreases testosterone synthesis and blocks action of testosterone


on the receptor aippg.com

c- Ketoconazole blocks steroid but not testosterone synthesis

d- Androgen deficiency in cirrhosis treated by testosterone may result in worsening of


gynaecomastia

e- Androgen therapy in chronic renal failure improves sexual performance if there is


hypogonadism

The following are true in crypto rchidism -

a- Unilateral cryptorchidism may be associated with bilateral seminiferous tubule


dysfunction

b- Bilateral cryptorchidism may result in a low serum count with normal testosterone
levels

c- Bilateral cryptorchidism when associated with low testosterone levels is associated


with other syndromes like KF

d- Ultra sound abdominal scan may be helpful in localsing a non palpable testis

e- The risk of malignancy in undescended testis is greater than the normal population,
even after orchiopexy

The first heart sound intensity is likely to be variable in -

a- Atrial fibrillation

b- Atrial flutter

c- Multiple ectopic beats

d- Complete heart block

e- Left bundle branch block


A giant A wave in the jugular venous pulse are commonly seen in -

a- Fallot’s tetralogy

b- Tricuspid stenosis

c- Constrictive pericarditis

d- Systemic hypertension

e- Pulmonary hypertension

Pulmonary hypertension is a recognized complication of -

a- Thrombo-embolic disease

b- Polycythemia rubra vera

c- Life at high altitude

d- Chronic alveolar hypoventilation

e- Patent ductus arteriosus

Following might be a clue to the etiology of hypertension -

a- His mother had eclampsia

b- His father had gout

c- A systolic murmur in the abdomen

d- A history of haemoglobinuria aippg.com

e- History of attack of muscular weakness

Constrictive pericarditis usually associated with -


a- A history of rheumatic fever

b- Orthopnoea

c- Swelling of the abdomen

d- Pulses paradoxus aippg.com

e- Expanslle pulsation of the liver

In cardiac tamponade the following occur -

a- A rising blood pressure

b- A high venous pressure

c- Increase pulsation in jugular veins

d- Progressive Bradycardia

e- Pallor cyanosis and cold extremities

The following conditions are likely to cause death of a patient before the age of the ten
if untreated -

a- Biscuspid aortic valve

b- Transposition of great arteries

c- Ventricular septal detect < 0.5 cm D

d- Tetralogy

e- Coarctation of the descending aorta

The following are likely to be present in primary pulmonary hypertension -

a- Polycythemia

b- Anginal pain

c- Prominent A wave in JVP


d- Dilation of the pulmonary artery

e- Dominant S in Vi

A large uncomplicated PDA accompanied by -

a- Cyanosis

b- Wide pulse pressure

c- Finger clubbing

d- Diminished pulmonary 2nd sound

e- Capillary pulsation

In a child with rheumatic fever each one of the following taken by itself would justify the
diagnosis of carditis -

a- Tachycardia

b- A soft systolic murmur

c- An enlarge heart

d- High fever

e- Heart blocks

A small pulse is example in -

a- Aortic stenosis

b- Mitral stenosis

c- PDA

d- Beri beri disease

e- Cor pulmonale
A systolic murmur best heard in the pulmonary area contributes to the diagnosis of -

a- Fallot’s tetralogy

b- Eisenmenger’s syndrome

c- Mitral incompetence

d- Tricuspid incompetence aippg.com

e- Marfan’s syndrome

Recognized unwanted effects of the digitalis glycosides include -

a- Accelerated AV conduction

b- Anorexia

c- A decrease in blood coagulability

d- Gynecomastia

e- Ventricular ectopic

Characteristic features of Fallot’s tetralogy include -

a- Mental subnormality

b- Forcible apex beat

c- Fixed split of 2nd heart sound

d- Metabolic alkalosis

R.dass, Nov 3, 2007

#201

R.dass

R.dass Guest
In atrial flutter -

a- There is usually an atrial rate about 150 per minute

b- Cardiac pain occur

c- Ventricular rate may be reduced by carotid sinus pressure

d- Organic disease of the heart is usually present aippg.com

e- Sinus rhythm can be restored by DC shock

Advanced mitral stenosis characterized by-

a- peripheral cyanosis

b- Finger clubbing

c- Reduced pulse pressure

d- Left ventricular enlargement

e- An accelerated 1st heart sound

Secondary prevention after myocardial infarction -

a- Warferin is of proven valve

b- Aspirin is of proven valve

c- Propranolol is of a proven valve

d- Coronary interventions are of proven valve

e- Ca channel blockers are of proven valve

Dobutamine -

a- Should be given intravenously


b- Has no dopamonergic effect

c- Causes the release of adrenalin

d- Had marked chronotropic effect at therapeutic doses

e- Is a peripheral vasodilator

Takayasu’s arteritis -

a- Is an autoimmune disease

b- Both sexes are equally affected

c- Diagnosed by ophthalmolgical examination

d- Never involve pulmonary artery

e- Can be diagnosed by arteriography

Ventricular tachycardia -

a- May not need treatment always

b- Requires DC shock in almost all cases

c- Associated with hyperkalemia

d- Should not be treated with Amiodarone aippg.com

e- Magnesium sulphate is the treatment of choice in trosade pointes

Causes of R wave in v1 lead include -

a- Wolff Parkinson White syndrome

b- True posterior myocardial infarction

c- Left anterior hemiblook

d- Hyperkalemia
e- Right ventricular hypertrophy

Marfan’s syndrome -

a- Equally common in male and female

b- Cardiac involvement may be absent

c- Mitral valve is never affected

d- Cased dislocation of the lens

e- Dissection of aorta is a recognised complication

Regarding iron therapy -

a- Ascorbic acid decreases iron absorption

b- The advantage of slow release preparations is that higher doses can be administered

c- If there is poor response to oral iron parenteral iron increases the response

d- 200 mg of FeSO4 contains more elemental iron than Fe gluconate

e- Maximal iron absorption occurs in the distal jejunum

Adverse effects of thiazides include -

a- Thrombocytopenia aippg.com

b- Hypercholesterolaemia

c- Impotence

d- Peripheral neuropathy

e- Hypochloraemic alkalosis

R.dass, Nov 3, 2007


#202

R.dass

R.dass Guest

Adverse effects of amiadarone include -

a- Hyperthyroidism

b- Peripheral neuropathy

c- Dystonia

d- Hypothyroidism aippg.com

e- Hepatitis

Sodium valproate -

a- Causes fetal hepato cellular damage

b- Causes tremors of fingers

c- Causes weight gain

d- Is a potent enzyme inducer

e- The blood levels correlate well with activity of the drug in the body

Generally recognized contraindications for therapy with streptokinase include -

a- Age > 75 years

b- IDDM

c- History of peptic ulcer

d- Bradycardia needing pacing

e- Bronchial asthma
Interferon -

a- Is a synthetic antiviral agent

b- is capable of inhibiting viral replication

c- Enhances MHC class 11 expression at the cell surface

d- its action is on viral infected cells

e- Is useful in the management of active chronic hepatitis B virus infection

Oral will be required for the treatment of -

a- Anaemia of Prematurity

b- Thalassaemia in infancy

c- Coeliec disease in a child

d- Sideroblastic anaemia aippg.com

e- Acute post-streptococcal glomerulonephritis

Recognized unwanted effects of therapy with tricyclic antidepressants include -

a- urinary retention

b- Diarrhoea

c- Cardiac dysrhythmlas

d- Postural hypotension

e- Cholestatic jaundice

The following drugs are correctly paired with a recognized unwanted effect -

a- Nitrofurantoin and peripheral neuropathy


b- Indomethacin and peripheral oedema

c- Cotrimoxazole and vitamin B12 deficiency

d- Metoclopramide and oculogyric crisis

e- Amiodarone and photosensitivity

Carbimazole -

a- Crosses the placental barrier

b- Is a recognized cause of agranulocytosis

c- Given in excessive doses leads to an increase in the size of the thyroid gland

d- Inhibits the thyroid trapping of Iodine

e- Should not be given in Association with Propranolol

A major route of metabolism for the following drug involves genetically determined
acetylation -

a- hydralazine

b- Phenelzine

c- Diazepam

d- Isoniazid

e- Dapsone

Thiazide diuretics -

a- Act on the loop of Henle distal to the major sit of sodium reabsorption

b- Case hypokalaemia by blocking the odium potassium exchange site in the distal
tubule

c- Increase the renal excretion of calcium


d- Increase the renal excretion of magnesium aippg.com

e- Elevate LDL cholesterol in long-term use

R.dass, Nov 3, 2007

#203

R.dass

R.dass Guest

Which of the following is the treatment of alcohol withdrawal hallucinosis?

a- Diazepam

b- Chlorpromazine

c- Haloperidol

d- Antidepressant

35 yrs old woman came to you with the history of menorrhagia and anxiety. Recently
she feels her husband is detached from her and she is not close to her husband as she
used to be. She has two children living in a rented house and bought all household things
on higher purchase and they have to pay it off regularly. They can not afford to go out
together for dinner. On physical examination no abnormality was found. What is your next
step of management?

a- Refer her to a Psychiatrist for treatment

b- Send her for marriage counseling

c- Talk to her husband

d- Advice them to take a holiday

e- Refer her to a Gynaecologist


A depressive patient is under treatment for two years and he is in remission. He wants
to stop the drug. You will tell him at this stage-

a- you can not stop the drug

b- You may have another episode of depression

c- You have to continue the drug life long in low dose

d- You can withdraw the drug gradually by 2 weeks

e- This is good time to take an alternative medicine

Which of the following is the Prodromal feature of Schizophrenic patient at risk?

a- Ideas of reference

b- Unemployment

c- Dysfunctional parenting

d- History of childhood abuse

e- Parietal lobe tumour

In mini-mental state examination the use of “Proverb” and expansion of the meaning is
assessed in which of the following disease -

a- Depersonalisation

b- Dementia

c- Delusional disorder

d- Dysthymic disorder

e- Personality disorder

Which of the following is true for Borderline personality disorder?

a- Lack of social relationship


b- Mood instability

c- Actively avoid social situation

d- History of conduct disorder

Naltrexone is used in heroin addiction as it is-

a- Short acting

b- Effective orally

c- Can increase tolerance of heroin

d- Should be used within first 3 days of the opioid use

e- It is ineffective in medically prescribed opioid addiction

Psychiatric primary health care is important for all following reasons except -

a- One third of all patients present with mental illness to Psychiatrist

b- It needs referral to Psychiatrist for a small number of patients

c- Most of the patients present with psychotic symptoms

d- Psychiatric patients concomitantly have medical problems

e- Drugs and Alcoholic are commonly overlooked by the primary health care provider

aippg.com

An 18-year-boy dressed as a female came to you with his boyfriend for problem in
sexual relationship. He always felt uncomfortable with his sex since childhood. He is taking
oestrogen for the last two years. He has been cross-dressing in several occasions since he
left home. What is this phenomena in Psychiatry called?

a- Gender identity disorder

b- Transsexualism

c- Transvestism
d- Masochism

e- Homosexualism

Defense mechanism is very important in Psychiatry. It is of three types “Mature”,


“Neurotic” and “Immature”. Which of the following is an Immature defense mechanism?

a- Identification

b- Introspection

c- Projection

d- Humour

e- Sublimation

R.dass, Nov 3, 2007

#204

R.dass

R.dass Guest

All of the following are true for Tourette syndrome except -

a- Both motor and vocal tics

b- Tics occurs in bouts several times in a day

c- Patient is not usually distressed by the tic

d- Less than10% has Coprolalia (Vocal Tics)

e- Onset always before the age of 18 years

aippg.com

An anxious mother of a 6 yrs old who has stolen a pencil case from his classmate in the
school came to you for consultation. She said he is doing well in the school and he never
did it before. The family is well off and is no family problem. What is your next appropriate
step?

a- In this stage you should adopt an “wait and see” policy

b- Refer her to the family child counseling service

c- A friendly trip to the nearest police station would be a good lesson for the boy

d- Inform the police to prevent further occurrence

e- Punish the boy

A young Schizophrenic patient comes for his antipsychotic medication stops at the front
of your office salutes you repeatedly. What is the most likely cause of his action?

a- Mannerism

b- Therapeutic behaviour

c- Obsessive compulsive disorder

d- Epilepsy

e- Migraine

A lady came to you with acute paralysis of the limbs following severe distress. She was
diagnosed as suffering from an acute conversion disorder. The following is true regarding
the management of this patient-

a- get her a wheelchair and refer her to the social welfare department

b- Graded physiotherapy to recover her physical disability

c- Refer her to psychiatry for psychodynamic study to resolve the conflict

d- Admission to the hospital in the psychiatry department

e- Refer her to physician for treatment

Antipsychotic drugs sometimes causes hyperprolactinaemia which results in all except -


a- Increased libido

b- Decreased libido

c- Osteoporosis in female

d- Galactorrhea in female

e- Gynaecomastia

THE MCQS HAVE ARRIVE – AMC MCQS

PSYCHIATRY

1.A lady with a previous divorce now comes to you with a seductive behaviour

a. Narcistic

b. Histrionic

c. Borderline

2.In Australia bush fire are common either accidentally or some people lighting fire
deliberately. Which is true regarding pyromaniacs?

a. Done for notoriety or publicity

b. To hide their acts

c. As they like to play with fire

d. Set fire and get panic attacks

e. For satisfaction
3.Depersonalization may occur in

a. Schizophrenia

b. Ecstatic religious experience

c. Depression

d. Post traumatic disorder

e. All of the above

4.A middle aged lady present to you with nausea dyspepsia abdominal distention. She
had a past history of going to many doctors and being treated for many disorders. she has
been treated by a rheumatologist for aches and pains, cardiologist for her palpitations and
gave her propanolol without improvement , a neurologist for her epilepsy. On
examination you find a tense anxious woman in spite of her daily dose of benzodiazepine.
There are scar from appendectomy and hysterectomy operations. What is the most
appropriate diagnosis?

a. Munchausen syndrome

b. Conversion disorder

c. Hypochondriasis

d. factitious disorder,

e. Neurotic anxiety

5.Many psychopharmacologic substance are lipophilic the advantage is

a. better absorbed with meals

b. promptly binds with free fatty acids


c. more resistant to be metabolized by thae liver

d. the pass rapidly to the brain and to the blood

e. a drug which has a long life and needs to be given once a day daily

6. Lithium:

a. loses it effectiveness over the time

b. is non addictive

c. works better in women than men

d. interaction with NSAIDS

7. morbid jealousy

a. alcoholism

b. schizophrenia

c. depression

d. obsessive compulsive

e. mania

8. A 40 yr old male school teacher who has recently separated from his wife was;
prescribed imipramine 25 mg for his depression. He had a road accident in which he hurt
his child. He was admitted to hospital for taking an overdose of imipramine with alcohol.
He does not give consent to go for psychiatric counselling or to be admitted hospital for
further treatment. Your next step in the management of this patient is:

a. Ask him to go to alcoholics anonymous


b. Increase the dose of Amitriptyline to 125 mg and send him home

c. Change to another drug

d. Retain him in the hospital against his using he Mental health act

e. Contact his wife and try to persuade her to return to him

f. Alcohol counselling

9.A 46 year old widow consults you for advice on management of insomnia, indifferent
appetite; recent weight loss of 5 kg., headaches and tearfulness. 4 weeks ago her only son
died after subarachnoid haemorrhage. Since then she had experienced auditory
hallucination, of her son's voice calling out her name and had felt his presence in their
living room. Your initial management of this woman would involve:

a. Grief counselling and a brief course of benzodiazepine

b. Grief counselling and a brief course of TCA

c. Grief counselling and a brief course of phenothiazine’s

d. Referral to psychiatrist

e. Referral for hospitalisation and treatment

10.Depersonalization may occur in:

a. schizophrenia

b. ecstatic religious experience

c. depression

d post traumatic disorder

e.A11 of the above


11. A 36 yr old engineer is due to give an speech soon and is reluctant to be exposed to
people. At the same time, he does not want to sign cheques or write anything in public.
He also thinks his, boas is controlling what he is doing although he knows that there is no
motive for that because he is a very good worker. He realizes there is no reason to act this
way, Lately he has been avoiding to go to the canteen with his mates and takes alcohol to
cope with the situation. What he is suffering from?

a. Social phobia

b. agoraphobia

d. antisocial personality disorder

e. panic disorder

f. paranoid disorder

12. Deinstitutionalisation means:

a. Transferring a patient from a psychiatric hospital to a community based facility.-

b . Put him in prison hospital

c. discharge the patient to a nursing home

d. Put patient in a psychiatric hospital

e. psychiatric patients are not permitted to vote

13.Lithium:

a. loses its effectiveness over the time

b. Is non-addictive
c. works better in women than men

d. Interaction with NSAIDS causes increase in lithium levels

e. Lithium toxicity

14.Cognitive therapy has good results in:

a. depression

b- chronic pain

c. social phobia

d. bulimia

e. all of the above

15. Many psychopharmacologic substance are lipophilic. The advantage is:

a. better absorbed with meals

b. promptly bind to free fatty acids

c. more resistant to be metabolised by the liver they pass rapidly to the blood and to
the brain

e. a drug has a long life and needs to be given once a day only

16. Psychiatric treatment in women is more commonly found in:

a. first trimester of pregnancy

b. second trimester

c. third trimester
d. puerperium

e.6 weeks after the first control

17.Regarding depression, what is true?

a. onset in winter, disappear in summer

b. -onset in autumn, disappear in summer

c. onset in summer; disappear in winter

d. onset in summer, disappear in autumn

e. onset in spring, disappear in winter

18. Despite obvious to the contrary, a 16 w old female with anorexia nervosa, thinks she
is overweight. This is an example of :

a. delusion

b. obsession

c. over valued idea

d. rumination

e. hallucination

19. A 35 year old female secretary, unmarried, complains of feeling of mass or lump on
her neck:
a. psychological (Globus Hystericus)

b. inflamed tonsils

c. lingual thyroid

d. hyperthyroidism

e. goitre

20.In generalized anxiety disorder, what Will develop in the next follow :

a. Schizophrenia

b. bipolar manic depression.

depressive illness

mania

phobia.

21. Using the WHO criteria, what is the most common symptoms of schizophrenia?

a. Flat affect

b. Auditory Hallucination

c. Visual hallucination

d. Lack of insight

e. Suicidal ideation

22. Exposure to stimuli and response prevention is the treatment of


a. Obsessive-compulsive disorder

b. Agoraphobia

c. Depression

d. Bipolar manic depression

e. Schizophrenia

23.Morbid jealousy, associated with:

a. alcoholism

b. schizophrenia

c. depression

d. obsessive compulsive

e. mania

24.WOF may be treated with ECT

a. puerperal depression

b. catatonic schizophrenia

c. mania

d. acute schizophrenia

e. all of the above


25.A 1 yr old child of normal intelligence becomes withdrawn and shows deterioration
in his school work. What is most likely:

a. schizophrenia

b. response to stress *in the family

c. normal variance of adolescence

d. Migraine

26. Anorexia nervosa is characterized by all of the following except.

1. 90% of those affected are females

2. raised FSH is pathognomonic

3. not eating or recognizing hunger

27. Fluoxetine is a new antidepressant unrelated to TC.A but with similar pharmacologic
action. What is its mechanism of action?

a. inhibits uptake of dopamine

b. inhibits uptake of norepephrine

c. inhibits uptake of serotonin

d. inhibit uptake of epinephrine


28.Apatient maintained on lithium carbonate was given an NSAIDS and developed then
Most likely explanation for this :

- lithium toxicity

- toxicity to NSAIDs

29. In married couple, the most likely cause of inhibited sexual excitement is?

a. fear of pregnancy

b. marital discord

c. empty nest syndrome

d. organic disorder

30.A Malaysian university student who is working as a waiter, whose girlfriend went
home to attend her father's funeral became irritable, restless and unable to sleep. He
complains that his companions are talking behind his back and plotting against him what is
your diagnosis?

a. reactive psychosis

b. grief reaction

c. onset schizophrenia

d. suspicious cannabis

e. panic psychosis
31. A 45 yr old salesman comes to you with complaints of difficulty of getting to sleep,
waking up with nightmares & loss of weight. What is your diagnosis'?

- anxiety

- depression

32. The most common cause of vaginismus :

-- Phobia

- schizophrenia

33. In borderline personality. What is correct?

a. antisocial with difficulty in establishing a relationship

b. histrionic personality

c. social withdrawal

d. identification problem –

e. tendency to self harm

34. What is correct of obsessive rumination?

a. compulsive hand washing

b. repetitive, persistent sexual fantasy c.


35. A 35 yr old man maintained on lithium was admitted for an infection. HIV test taken
and done without his consent turned up positive. What is correct in this case?

a. stop lithium and start AZT

b. Tell the patient he has AIDS and extract a history of his sexual contacts/partners

c. It is unlawful to do the test without consent and pre treatment counselling

d. Inform the staff of the patient condition

36. A 65 yr old Greek lady living in Australia since 34 yrs ago has been living alone. She
now seems to answer voices and does strange things. The most likely

a. paranoid delusions

b. migrant psychosis

c. Capgras syndrome

d. Paraphilia

e. Frigoli

37. A 23 yr old male unemployed is complaining about his nose. He says that he can't
get a good job because of his nose. He also said he can't get a girl friend , and is
demanding about a reduction of his nose. The doctors an opinion is that the nose looks
normal. What is your management?

a. refer to p1'astic surgeon

b. refer to psychiatry
d. try a short course of...

e. explain him very gently that his nose looks normal and try to find out what social
causes and difficulties about his work.

38. What is delusion?

a. False Belief

b. False thought

c. Sexual Fantasy

39. Most appropriate re :post partum psychosis is:-

a. family counselling

b. ask mother to stop breastfeeding

c. admit to hospital

d. psychiatric treatment to the mother

e. treat and send home

40. A 85 YO old man with progressive loss of memory for recent events, unable to do
any calculations is disoriented for time, WOF is correct?

1. frontal lobe tumour

2. Alzheimer’s

3. Multi infarct Dementia

4. Due to old age


41. A young girl with running nose; diarrhoea. sweating; lacrimation, and abdominal
pain. WOF is the cause'?

a. heroin withdrawal

b. cocaine

c. LSD

d. marijuana

e. amphetamine

42. Endogenous depression. WOF is true?

a. Early morning waking

b. Difficulty in initial sleep

c. Dreams with content involving death patient

d. Somnolence in the early part of day

e. None of Above

43. Basis of Dopamine theory of Scz:

a. Phenothiazine can cause Parkinson like syndrome

b. Amphetamine can cause schizophrenia

c. Anticholinergic drug can cause delirium.


c. Tricyclic Antidepressants have anticholinergic effects.

d. Imipramine helps in obsessive neurosis.

44 Acute alcoholic hallucination can be differentiated from delirium tremens by the fact
that:

A. It occurs following ingestion of small amount of ethanol whereas DT is a withdrawal


syndrome.

c. Unilateral tinnitus is pathognomic of alcohol hallucination.

d. Unilateral auditory hallucination is path gnomic of alcoholic, hallucination.

e. The sensorial remain clear despite the presence of terrifying auditory hallucination.

f. The hallucination of alcoholic is visual and unpleasant.

45. WOF is characteristic of schizophrenia'?

a auditory hallucination

b. Depersonalisation

d. antisocial behaviour

e. flight of ideas

f. all of the above

46 A 70 yr old male taking Aldomet for hypertension for the last 10 Years starts to
isolate himself and is suffering from deafness: He feels that people are speaking about
him. What is most likely diagnosis:
a. Symptoms of Aldomet

b. reaction to deafness

c. atherosclerotic dementia

d. senile dementia

e. senile paraphrenia

MEDICINE

1. In dislocation of shoulder, which nerve is liable to be injured?

a. Radial

b. Axillary

c. Musculocutaneous

d. Ulnar

e. Brachial plexus

2. Which of the following is likely to damage an artery?

a. Anterior Dislocation shoulder

b. Posterior dislocation shoulder

c. Anterior dislocation hip


d. Posterior dislocation hip

e. Posterior dislocation knee

3. In Colles Fracture, the distal segment is displaced:

a. Backwards and ulnarwards

b. Backwards and radially

c. Forwards and ulnarwards

d. Forwards and radially

e. Minimally displaced

4. An 18-month-old is brought to you by the mother for breast enlargement. Tanner 3.


No pubic hair or axillary hair is seen. No other abnormality is seen. What is your diagnosis?

a. Precocious puberty

b. Idiopathic thelarche

c. Congenital adrenal hyperplasia

d. Breast tumour

e. Mastitis

5. A 21-year-old child is brought to you for having loose motions for 3 days. The stool
contains undigested food material. No vomiting. Thriving well. What is he suffering from?
a. Cystic fibrosis

b. Coeliac disease

c. Gastroenteritis

d. Toddlers diarrhoea

e. IBS

6. A large man presents with big face broad hands thick and oily skin and hoarse voice.
What is the initial investigation of choice?

a. GTT + GH

b. Insulin challenge test + GH

c. GH+ cortisol

d. GH

e. GTT

7. Regarding heparin induced thrombocytopenia, what is true?

a. Purpura

b. Joint bleeding

c. Ecchymosis

d. Thrombosis

e. Hematuria
8. Regarding ADHD, what is correct?

a. Age 5-7 years

b. Reading difficulties

c. Social withdrawal

d. Females affected more than Males

e. Prevalence is 15%

9. All the following can cause dysphagia except:

a. Oesophageal cancer

b. Achalasia

c. Oesophageal varices

d. Barrett’s oesophagus

e. Hiatus hernia

10.A 6 week old is brought to you for not having passed motion for last 4 days. He has
been feeding well, and is gaining weight. Otherwise well and active. What is the most
probable explanation?

a. Hirsprung’s disease

b. Normal variant

c. Hypothyroidism

d. Acquired constipation
e. Foreign body

11. A man returns after a trip to India. Now has 3 days diarrhoea with blood in stools.
Looks run out and dehydrated. Noted to have fever. What is most correct?

a. Giardia infection

b. Amoebiasis

c. E. coli gastroenteritis

d. Viral diarrhoea

e. Cholera

12. In a 40 years old lady, all the following may be the cause of menorrhagia except:

a. Hormonal response to uterus

b. Subserous fibroid

c. Intermural fibroid

d. Adenomyosis

e. Endometriosis

13. 3 month old comes with unilateral eye discharge. What is the most possible cause:

a. Chlamydia infection

b. Gonococcus

c. Ophthalmia neonatorum

d. Blocked nasolacrimal duct


e. Normal variant

*14. A 6-month-old presents with SOB. There was a birthday party at home yesterday.
On examination, child is wheezing. What is most initial investigation?

a. Sweat test

b. Bronchogram

c. CXR

d. Inspiratory and Expiratory Rontogram

e. PEFR measurement

15. A mother brings her 3-year-old baby girl with unilateral nasal discharge off and on
for last 3 months. Secretion is foul smelling and at times copious. Baby is uncooperative.
What is the initial investigation of choice?

a. CXR

b. indirect laryngoscopy

c. Examination under GA

16. A 6-year-old girl wakes up at night c/o sever pain in her calves. Mother rubs her
calves and she feel fine, goes to sleep. This has been happening for a number of days now.
At daytime she is active and playing. All of the following are true except:

a. Ca and CK will be abnormal

b. No investigations are needed

c. Family Hx will be positive


d. Reassurance is the treatment.

17. Which vaccination should not be given to a child who is under remission on
chemotherapy for CLL?

a. Polio

b. MMR

c. HiB

d. DPT

18.Solitary mobile carcinoma breast lump. No axillary lymphadenopathy. Which of the


following is a prognostic marker?

a. Age > 45

b. Neu2 / HER oncogene status

c. Oestrogen level

19. All can occur with chronic limb ischemia except:

a. Great Toe gangrene

b. Pallor

c. Ulcer on medial side of leg above malleolus

d. Rest pain

e. Intermittent claudication
*22. 40 year old with acute painful testis:*

a. Varicocele

b. Hydrocele

c. Epididymoorchitis

d. Torsion

e. Tumour

23. WOF is true regarding a 2cm kidney stone in the pelvis seen in KUB examination.:

a. Expectant treatment is right

b. It is a urate stone

c. Lithotripsy

d. Open surgery

24. Patient can’t dorsiflex or evert his foot. What is wrong?

a. L4

b. L5

c. Tibial nerve

d. Peroneal nerve

e. Sciatic nerve

25. Body dysmorphia is seen in AOF except:


a. Anorexia nervosa

b. Acromegaly

c. Non dominant parietal lobe lesion

d. Dominant parietal lobe lesion

e. Narcissistic personality disorder

26. 6 year old child comes to you with up rolling if eyes, neck stiffness, protruded
tongue, rigid body and face tilted to one side. Had vomiting and was being treated by the
GP. What is the cause?

a. Huntington’s chorea

b. Epilepsy

c. Reaction to metoclopramide

d. Gastritis

27. Lung cancer associated with non-smokers:

a. Sq. cell Ca.

b. Adenocarcinoma

c. Small cell Ca.

d. Oat cell Ca.

e. Large cell Ca.

28. All of the following cause gynecomastia except :


a. spironolactone

b. cimetidine

c. digoxin

d. alpha methyldopa

e. labetolol

29.A young female presents with 24 hour h/o sore throat. Later she develops
proteinuria and some hematuria. What is the most probable cause?

a. Glomerulonephritis

b. IgA Nephropathy

c. Nephrotic syndrome

d. HTN

30.What is the most important step in psychotherapy?

a. Detailed history

b. Isolation

c. History from friends and relatives.

d. Developing relationship

e. Investigations

31. When do we use Psychodynamic Psychotherapy?


a. Obsessive Compulsive Disorder

b. Bipolar Disorder

c. Schizophrenia

d. Depression

e. Anorexia Nervosa

32. You are asked to examine a person. He is cut off from the world since he was teen.
Lives alone and wants to be alone. No friends. Family wants to support but he refuses to
get any help. He doesn’t have any thought disorder or perceptional symptoms, but his
affect is blunt. What would be the most probable diagnosis?

a. Chronic schizophrenia

b. Major depression

c. Narcissistic Personality

d. Bipolar disorder

33.Regarding Billing’s method of contraception, AOF are true except:

a. Cervical mucus has to be palpated

b. Unsafe sex, if it becomes more

c. Safe sex after 2-3 days of becoming more

d. Since it considers the cervical mucus, it is same contraception in regular, irregular and
lactational menstrual periods.

34.Regarding Levonorgestral containing IUCD, AOF are true except:


a. Light periods

b. Amenorrhoea

c. Decreased risk of cervical carcinoma

d. Decreased risk of sexually transmitted diseases

e. Decreased risk of ectopic pregnancy

35.Spleenectomy in spherocytosis will do all the following except:

a. Normalize anaemia

b. Normalize spherocytosis

36. An 8-year-old child presents to the ED with bilateral tonsillitis with greyish white
exudates. He has cervical lymphadenopathy, fever and hepatosplenomegaly. What is the
probable cause?

a. EBV

b. CMV

c. Diphtheria

d. Bacterial

37. A young child with 2-week h/o dry cough with whop, and some times vomits. What
is true about this patient?

a. Will have cough for the next 4 weeks

b. Amoxicillin should be started


c. D & T vaccine should be given immediately

38. A patient with pyloric stenosis, vomiting for 4 days now presents to ED with mother.
He is dull lethargic, skin turgor is lost, and BP is 90/50. What is the most initial choice of
fluid in this patient?

a. Normal saline

b. Hartman’s solution

c. Ringer’s lactate solution

d. Dextrose 5% and Normal saline

39.Regarding diffuse fibrosing alveolitis what is true?

a. Decreased FEV1 / FEV

b. Decreased TLC

c. Decreased Expansion

d. Decreased O2 tension

e. Hypercarbia

40.Psychiatric side effect of corticosteroids?

Withdrawal causes delirium

41. A patient is suffering from cyclical mastalgia. Routine treatment fails. What is your
next management?
a. NSAID

b. Bromocriptine

c. Danazol

d. Progesterone

e. Lasix

42.Regarding CPR, WOF statement is most correct?

a. Adrenaline should be given every 10 minutes

b. Bicarbonate is mandatory

c. ECM only pumps ½ of the cardiac output in one stroke

d. ECG is necessary before cardio-version.

43. Patient with grandiose character. Doesn’t care about others. Feels himself to be
unique:

a. Histrionic personality

b. Schizoid

c. Narsistic

d. Delusional

e. Borderline
44.Picture of CT Scan showing two white spots just lateral to the midline. A patient 59
year old presents with sudden onset of left hand weakness. What could be the possible
cause?

a. SDH

b. SAH

c. Intra cerebral haemorrhage

d. Cerebral infarct

45. WOF has the least risk of having congenital malformation at birth.

a. Chromosomal abnormality AD, AR, Sex linked recessive

b. Rubella at 18 weeks

46. What is the most likely cause of post coital bleeding?

a. Endometrial Carcinoma

b. Vaginal candidiasis

c. Cervical polyp

d. Fibroid

47. Regarding thyroid cancer, all are true except:

a. Enlarged thyroid
b. Solitary nodule

c. Hot nodule

d. Change in voice

e. Tracheal compression

48. Patient presents with fever, tachycardia (Pulse 110/mt), Tremors, and palpable &
tender thyroid. On investigation, there is no radioactive iodine uptake. What is the
possible treatment?

a. Carbimazole

b. I131

c. Beta-Blocker + Paracetamol

d. Thyroxine

49. A women has bilateral suppurative inflammatory lesion on the axilla and groin.
What is true?

a. Actinomycosis

b. Fungal infection – Mycosis Fungoides

c. Suppurative Hidradenitis

d. Multiple lymphadenopathy with abscess

e. Pilonidal sinus
50. An old man who was operated for aortic aneurysm 3 weeks ago, presents to the ED
with irregular pulse, restlessness, abdominal distension and not having passed flatus and
motions for last 2 days. WOF is the most possible cause?

a. Rupture of suture site

b. Mesenteric artery embolism

c. Urinary retention

d. Diverticulosis

51 Why do we do an ultra sound in a patient with sign symptoms of biliary disease?

a. To visualize stones in the GB or CBD

b. To visualize pancreatic cancer

c. To see the dilation of bile tree

d. To locate the hepatic cancer

e. To visualize the GB pathology

52. A seventy-year lady who is alcoholic and smokes > 20 cigarettes a day, presents to
you with tiredness and fatigue. She also complains of weakness in limbs. Her Hb was 98,
MCV 110, and platelets were decreased. What is the most possible cause?

a. AoCD

b. IDA

c. Alcoholism

d. Pernicious Anaemia
e. Auto immune Haemolytic Anaemia

53. A young female on oral contraceptive misses one tablet while in the mid cycle. Had
coitus the same day, and takes the ‘missed’ tablet 12 hours later. Then continues with rest
of her tablets. She has spotting on the third day of coitus. What will be your advice?

a. Continue with the OCP.

b. Change to different contraceptive drug.

54. AOF are the features of basal ganglion except:

a. Ataxia

b. Tremor

c. Bradykinesia

d. Chorea

e. Rigidity

55.Most common cause of bowel obstruction in Australia assuming that he as not


undergone any abdominal surgery, is:

a. Volvulus

b. Intersuseption

c. Groin hernia
d. Internal hernia

e. Tumours

56. A patient presents with wasting of small muscles of hand with thenar sparing. What
is the most probable diagnosis?

a. Ulnar N

b. Median N

c. Axillary N

d. Musculocutaneous N

57. Picture on Page 206 of AMC. Picture of scalp showing white scaly lesion.

a. Psoriasis

b. Alopecia Areata

c. Pediculosis

d. Tinea Capitis

58. A young couple wants to adopt abstinence as contraception. Assuming the female is
having a regular monthly cycle of 28 days, you would advice them to avoid sex on AOF
days except:

a. Day 4 – 17

b. Day 8 – 17
c. Day 10 – 19

d. Day 12 – 19

e. Day 17 - 21

59. Most common cause of cholecystitis in Australia

a. Gall stone at out flow

b. Gall stone in CBD

c. Pancreatitis

d. Tumour

e. Hepatitis

60.WOF is expected to happen after splenectomy for spherocytosis?

a. RBC life span improves

b. Leukopenia

c. Reduction in anaemia

d. Reduction of spherocytes

e. Reduction in fragility of RBCs.

61.What is the cause of majority of HbsAg patients in Chinese population?

a. Blood product
b. IVDU

c. Child birth

d. Sexual spread

e. Haemophilia

62. A 60 years old female patient of RA takes 5mg of Prednisolone tabs BD for last 10
years. Now she comes to you complaining sudden onset of pain and swelling of her right
knee, which is warm and tender. Your management would be:

a. Increase dose of NSAID

b. Decrease dose of NSAID

c. IV Antibiotics & hospitalisation

d. Knee X Ray for unrecognised trauma

e. Knee aspiration for cytology.

63. A 12 year old is having pain in his hip. His mother noticed a limp. He is afebrile, and
the general examination is normal. What is the most probable cause of his symptoms?

a. Slipped upper femoral epiphysis

b. Perthes disease

c. Non specific synovitis

d. Osteomyelitis

64. Photo of viral wart on the eyelid of a young female.


a. Wart

b. Molluscum contagiosum

c. Herpes simplex

d. Skin tag

65. AOF cause gynecomastia, except:

a. Spironolactone

b. Digoxin

c. Methyldopa

d. Labetolol

66. A 41 week primi comes to you complaining of no fetal movements for last 24 hrs.
FHS is 140/mt, CTG is normal, and rest of the examination is normal. You send her home.
Next day she rings you stating that there is again no fetal movement for the last 24 hrs.
What will be you next step of management?

a. Tell her that all her examinations are fine, & she should not worry

b. Ask her to wait for another 24 hrs.

c. Admit labour ward and induce labour.

d. Immediate LSCS

67. Picture of supracondylar fracture (AMC Book): A young girl fall on her out stretched
hand. What will be your management?

a. Neurological study to check the Median Nerve involvement

b. Fasciotomy
c. # Reduction and assessment of circulation

d. Back slab and review in 24 hours

68. Many people attended a dinner party on a week end. 10% had diarrhoea. Most of
them recovered spontaneously, but few needed hospitalisation for severe dehydration.
What is the most probable cause?

a. Giardia

b. Salmonella

c. Clostridium

d. Shigella

69.Complication of # of epiphyseal plate

a. Retardation of longitudinal growth

b. Joint stiffness

c. Malunion

d. Non-union

e. Avascular necrosis

70.4 day old full term baby normal at birth. Suddenly collapse at cot. O/E baby is
peripherally cyanosed , no pulse and respiratory distress. Diagnosis?

a. Pulmonary Hypertension

b. Fallot’s Tetralogy
c. PDA

d. CHD

e. Lt Heart hyperplasia.

f. Transposition of great vessels

MEDICINE, SURGERY , OBSTETRICS AND PSYCHIATRY

1. An elderly woman can read the newspaper, but has halos in bright sunlight.

A. Cataract

B. Glaucoma

C. Presbyopia

D. Macular Degeneration

2. Which of the following drugs decrease renin:

A. Beta Blockers

B. ACE Inhibitors

C. Spironolactone
D. Hydralazine

E. Centrally acting antihypertensives.

3. Newborn with respiratory distress , faint breath sounds on the left and with a
scaphoid abdomen

A. Meconium aspiration syndrome

B. Situs inversus

C. Diaphragmatic hernia

4. Which nerve gives the sensation of taste to the anterior 2/3rd of tongue.

A. Trigeminal nerve

B. Facial nerve

C. Hypoglossal nerve

D. Glossopharyngeal nerve
5. A Patient who underwent a cholecystectomy, now comes with jaundice .what is the
investigation of choice.

A. Oral cholecystogram

B. CT scan

C. Ultrasound

D. Sr. Creatinine

E. Upper GI Studies

6. Regarding pseudobulbar palsy AOF is true …except

A. Wasting and fasciculation of tongue

B. Jaw jerk decreased

C. Loss of sphincter control


D. Gag reflex present

7. Regarding ACE Inhibitors, which is not true :

A. Used in the treatment of heart failure

B. Used as first line in the treatment of Hypertension

C. Used in Diabetics

D. Cannot be used in the treatment of Aortic stenosis.

8. Regarding CRF and Calcium metabolism

A. Ca is decreased

B. Ca is increased

C. There is no relation between CRF and Ca

D. Causes Osteomalacia.
9. Most significant complication of massive blood transfusion. ( *)

A. Pulmonary oedema

B. Change in acid base balance

C. DIC and coagulation defect

D. Increased CVP

10. Regarding CRF and Potassium all of the following reduce potassium except
(treatment of hyperkalemia) *

A. Calcium Carbonate

B. Glucose and Insulin

C. Dialysis

D. Resonium
E. Sodium Bicarbonate

11. 55year old patient with dysphagia for solids with a previous history of reflux

A. Carcinoma oesophagus

B. Stricture

C. Scleroderma

D. Achalasia \

E. Raynaud’s syndrome

12. An infant came with pneumonia , X ray showed consolidation of a lobe with round
translucencies and a small pleural effusion. What is the treatment of choice?

A. Crystalline penicillin

B. Flucloxicillin

C. Amoxicillin / clavulanic acid


D. Tetracycline

13. A new born was peripherally cyanosed and crying lustily ..the axillary temperature
recorded was 37.2degrees what would be the next step( *)

A. Take an x ray

B. reassure

C. oxygen

D. urine culture

14. 34 year old lady on phenytoin wants to take OCP’S what can be prescribed

A. Microgynon 30

B. Microgynon 50

C. Triphasic

D. Oetradiol patches

E. Progesterone only pill


15. 6 months old boy brought by his mother with a temperature of 38.9degrees with
bilateral wheezing .his resp. rate was 36/min .other members in the family had a h/o
similar illness. There is family h/o asthma. What is the diagnosis?

A. Asthma

B. Foreign body

C. Bronchiolitis

D. Pneumonia

16. 16 weeks p

regnant lady came for a check up ,for the diagnosis of foetal anencephaly all are true
except

A. increased alpha feto protein

B. increased beta HCG

C. nuchal thickness
D. decreased alpha fetoprotein

17. Regarding tubal pregnancy most suggestive is (*)

A. –ve beta HCG

B. ultrasound showing empty uterus

C. ultrasound showing tubal mass

D. CT scan

18. 10 weeks old child with persistent unilateral eye discharge responding to antibiotics
but recurring

A. nasolacrimal duct obstruction

B. gonococcus

C. Chlamydia

19. unilateral foul smelling ,bloodstained discharge from nose

A. foreign body
B. nasal polyps

C. Atopy

D. Rhinitis

20. 10 year old came to your surgery with scrotal pain .on examination both testis are in
the scrotum , next management :

A. do an ultrasound

B. arrange surgery

C. write some analgesic and send him home

D. do nothing it will go away

E. tell his mother to review back again when the pain recurs

21. 19 year old girl had a binge of drinking the previous night with lower abdominal
tenderness and all investigations and testis are normal (*)
A. treat as gastroenteritis

B. tell her it is due to alcohol

C. not sure of diagnosis ;come back for review

D. Give analgesic and antiemetic and send her home.

22. Reversal of non depolarising skeletal muscle blockade

A. Pyridostigmine

B. Neostigmine

C. Atropine

D. Benzhexol

23. differentiation between schizophrenia and shcizophreniform disorders is by (*)


A. affective symptoms

B. duration of symptoms

C. lack of insight

D. female and male ratio

24. A lady with a previous divorce now comes to you with a seductive behaviour

A. Narcicistic

B. Histrionic

C. Borderline

25. In Australia bush fires are common either accidentally or due to some people
lighting fire deliberately, which is true regarding pyromaniacs.

A. Done for notoriety and publicity

B. To hide their acts


C. As they like to play with fire

D. Set fire and get panic attacks

E. For satisfaction.

26. People living near airport have(*)

A. Explosive personality

B. Insomnia

C. Agitated

D. Depression

27. Regarding panic attacks all are true except (*)

A. 20% have had at least one attack in their lifetime.


B. With out agoraphobia it is equal in male and female

C. Always avoid precipitating factors

D. Usually occurs in the twenties

28. Regarding treatment of chronic duodenal ulcer

A. eradication of H pylori

B. H2 blockers

C. PPI

D. Selective vagotomy

29. 4 year old boy with fever and malaise , lymphocyte count –normal, platelets
decreased ,Hb decreased

A. ALL
B. Infectious mononucleosis

C. Hodgkin’s disease

30. Pregnant lady with Group B strep infection, what is true.

A. Penicillin to be given as prophylaxis

B. Bolus dose of penicillin before labour

C. Take a swab and if B strep present then treat.

31. Regarding MI , maximum deaths occur in (*)

A. with in 1st 2 hrs

B. 2 to 12hrs

C. 12 to 24hrs

D. 2 to 7days
E. after discharge

32. A case of stable angina with chest pain. On examination enzymes, ECG are normal
but as you were examining him he belches and says he feels better - what would you do
(*)

A. Admit to coronary care Unit and do ECG monitoring.

B. send him home with appointment to cardiologist

C. refer to gastroenterologist

D. if enzymes normal then probably no cardiac

33. With regard to primary health care all are true except(*)

A. 1/3rd of population come to a GP with psychiatric symptoms.

B. Most patients are psychotic

C. Only few are referred to psychiatrists


D. Alcoholics and drug abuse frequently overlooked by GP’s

34. A patient with known Parkinson’s disease for 2 yrs on long term treatment now
comes with tongue protruding out. What would you do ?

A. Decrease levodopa +carbidopa

B. Increase levodopa + carbidopa

C. Stop levodopa +carbidopa

D. Treat with chlorpromazine

35. In a case of twin pregnancy all are true except

A. asymmetrical growth retardation

B. anaemia in pregnancy

C. premature labour
D. acute polyhydramnios

E. 2nd twin foetal malformation.

36. 60 yr old female with diarrhoea and profuse mucous discharge .what could be the
cause

A. Cron’s disease

B. ulcerative colitis

C. villous adenoma

D. rectal Ca

E. acute mesenteric ischaemia

37. How do you differentiate between anorectal and colorectal cause of bleeding(*)

A. Blood mixed with stools


B. Fresh bright bleeding

C. Mucoid discharge

D. Pain during defecation

38. most common cause of severe chest pain in pericarditis

A. viral pericarditis

B. tuberculosis

C. Mycoplasma

D. Uraemia

E. MI

39. all of the following are causes of supraclavicular mass except (*)

A. stomach Ca
B. cervical rib

C. breast Ca

D. subclavian thrombosis

E. subclavian artery aneurysm

40. which of the following has worst prognosis

A. advanced breast Ca

B. choriocarcinoma

C. Hodgkin’s lymphoma

D. Non-Hodgkin lymphoma

E. Prostate carcinoma
41. Which of the following does not metastasise to brain

A. Malignant melanoma

B. Prostatic Ca

C. Lung Ca

D. Breast Ca

42. 40 yr old lady with a 2 cm palpable breast lump on the right side .What is the next
step.(*)

A. FNAC

B. Ultrasound

C. Lumpectomy

D. Mammography

E. Radical mastectomy
43. A lady with a palpable breast lump, FNAC showed few malignant cells regarding
conservative surgery what is true? (*)

A. Assess for oestrogen receptors

B. Bone marrow biopsy

C. Axillary lymph node sampling

D. Mammography

44. A lady with a breast cancer on left side operated 2yrs ago now detects a small lump
on the right side .how do you explain the lump

A. Cancer arising de novo

B. Fibroadenoma

C. Metastasis from the previous one


45. facial nerve palsy can be associated with all of the following except

A. chronic parotitis

B. Ca parotid

C. Acoustic neuroma

D. # base of the skull

46. A middle aged woman with deafness and loss of corneal reflex but with no tinnitus

A. vestibular neuronitis

B. Meniere’s disease

C. Acoustic neuroma

D. Multiple sclerosis

47. Ptosis ,dysphagia ,ataxia ,on the same side and spinothalamic loss on the opposite
side:

A. vertebral artery occlusion


B. basilar artery occlusion

C. MS – midbrain

D. Posterior communicating artery syndrome (PICA)

48. 22yr old lady with diplopia. On closing the right eye ,the medial side of the image is
lost ( i.e diplopia on looking laterally ) what is the diagnosis

A. left 6th nerve palsy

B. left 3rd nerve palsy

C. right 6th nerve palsy

D. posterior cranial fossa tumour

49. Regarding a patient with hepatoma , which of the following is least likely

A. Hepatitis B
B. Hepatitis C

C. Hemochromatosis

D. CMV

E. Alcoholic cirrhosis

50. In which of the following cell mediated immunity is lost first followed by loss of
humoral immunity (*)

A. CLL

B. HIV

C. RA

51. which of the following is not a carcinogen (*)

A. EBV

B. CMV
C. Hep C

D. HIV

52. Picture of a lesion at the lateral angle of the eye what is the treatment ( it’s a BCC) :

A. Surgical removal

B. Excision and radiation

C. Cryotherapy

D. Chemotherapy

E. Local steroids

53. Picture of swelling at the outer angle of the eye .it is described as being hard and
present since birth:

A. Osteoma

B. Sebaceous cyst
C. Lipoma

54. picture of a large swelling on the back near the left scapula

A. lipoma

B. sebaceous cyst

C. 2ndary breast

55. Picture of the face with a non itchy rash like lesion on the cheeks, forehead.

A. SLE

B. Seborrheic dermatitis

C. Acne rosacea

D. Dermatomyositis

56. post operative specimen ( testicle with epididymis )


A. TB

B. Epididymoorchitis

C. Teratoma

D. Torsion of testis

E. seminoma

57. A chest x ray of a child showing consolidation > neutrophils increased. What could
be the cause

A. Klebsiella pneumonia

B. Group B streptococcus

C. Staphylococcus

D. Mycoplasma pneumonia

58. A lesion (looks like an ulcer) diagnosis?


A. Amelanotic melanoma

B. Implantation dermoid

C. Basal cell carcinoma

59. ECG- patient is a diabetic and he is dyspnoeic .diagnosis(*)

A. Inferior wall MI

B. Anterior wall MI

C. Pericarditis

D. WPW syndrome

E. Pulmonary embolism

60. ECG patient has palpitations, otherwise normal (*)


A. Atrial fibrillation

B. Atrial flutter with variable block

C. WPW syndrome with accelerated beats

61. ECG patient comes with sweating , palpitations

A. Ventricular ectopic

B. Ventricular tachycardia

C. Atrial fibrillation

D. RBBB

62. How do you treat WPW in a patient with a previous history of collapse?

A. Beta blockers

B. Cardioversion
C. Radiofrequency ablation abnormal tract

D. Surgical ablation

D. Long term verapamil

63. 40 yr old man with SVT 160/min ..Patient not arousable , Treatment is

A. cardioversion

B. adenosine

C. procainamide

D. verapamil

E. take an ECG

64. A child with heart rate of 220/min otherwise normal , how do you manage

A. cold stimulus
B. Valsalva

C. Verapamil

D. DC shock

65. All of the following are side effects of depot medroxyprogesterone , except(*)

A. Amenorrhoea

B. Weight gain

C. Depression

D. Used with oestrogen causes stratification and cornification of vagina

E. hypotension

66. 20yr old man came with pain referring from groin to loin to tip of penis brought a
sample of urine mixed with blood and asks for a shot of pethidine to relieve his pain. What
is the next step

A. give him an injection of pethidine to relieve his pain


B. KUB

C. Ultrasound abdomen

D. Examine fresh urine sample

67. 12yr old boy with bee sting with wheeze ,with swollen lips ,tachycardia , restless,
immediate treatment(*)

A. adrenaline IM

B. hydrocortisone IV

C. oxygen

D. antihistamine

E. give NSAID and send him home

68. Young man with tachycardia, BP 90/70,Pulse 140 difficulty in breathing (*)

A. start two IV line to Hartmann


B. wide bore needle

C. tube drainage

69. RTA respiratory distress mediastinal shift to opposite side with emphysema in the
neck .what is the diagnosis

A. Tension pneumothorax

B. Hemothorax

C. Cardiac tamponade

D. Bronchial tear

70. Newborn male with normal genitalia , which is true

A. 47xxy

B. 46xy with androgen insensitivity

C. mother treatment with cyproterone from 8 weeks


71. 16yr old girl came with her mother with no menstrual , breast development is
normal (testicular feminisation)

A. 45xo

B. 46xy

C. Turner syndrome

72. After MVA a patient is dyspnoeic BP 100/70, HR 110/min, Breath sounds decreased
on left side, heart sounds normal, JVP raised, next step in management (*)

A. IV fluids

B. Wide bore thoracostomy

C. Tube thoracostomy

73. A patient with excruciating chest pain and a diastolic murmur .what does the X-ray
show (*)

A. widening of mediastinum
B. increased left ventricular size

C. trachea shifted to left

74. A child with fever malaise, sore throat …white papillae on the tongue and later a
sandpapery rash...what is the diagnosis

A. measles

B. rubella

C. scarlet fever

75. A child with fever of 3 days duration and a rash develops when fever subsides

A. Rubella

B. Roseola

C. Erythema multiforme
76. An unconscious man (a known COPD case) was brought to the emergency, on
examination there was a bruise on the parietal area and needle mark in the cubital fossa.
His ABG was as follows (PH: 7.26, PCO2: 60, PO2: 50) His previous ABG showed (PH: 7.35,
PCO2: 30, PO2: 60) .what is the diagnosis? (*)

A. Narcotic

B. Subdural haemorrhage

C. Subarachnoid haemorrhage

77. 16 weeks pregnant woman with proteinuria 3 +, hematuria, and hypertension (*)

A. PIH

B. Pre-existing renal disease

C. Essential hypertension

D. Pregnancy will continue until term

78. Which of the following is familial


A. papillary carcinoma

B. medullary carcinoma

C. follicular Ca

D. anaplastic

E. secondary Ca of thyroid

79. A patient with HIV and cough (respiratory symptoms) has a Mantoux 5mm +ve ,
what is the next step

A. INH prophylaxis

B. Zidovudine

C. Interferon

80. Patient with HIV +ve status ..what is true (*)

A. Life long infectivity


B. He has AIDS

C. Can transmit through saliva

D. He should avoid sexual intercourse

81. Tremor can be present in all of the following except

A. hyperthyroidism

B. hypothyroidism

C. benign essential tremor

D. Parkinsonism

E. chronic liver disease

82. A lady with tremor on lifting the phone and disappears when she looks at her hand
with mild rigidity of the hand but no cogwheel rigidity. What is the treatment?

A. Propranolol
B. Benzhexol

C. Levodopa

83. Complication of # of epiphyseal plate

A. retardation of longitudinal growth

B. joint stiffness

C. malunion

D. non-union

E. avascular necrosis

84. which of the following does not have any interaction

A. warfarin & Isosorbitrate

B. verapamil & metoprolol


C. erythromycin & terfenadine

D. digoxin & amiodarone

85. 12yr old child weight 90th percentile with a limp

A. Perthes’s disease

B. slipped capital femoral epiphyses

C. tibial synovitis

86. 13yr old child which would be the appropriate bone age for a normal adult height

A. 9yr old

B. 13yr old

C. 18yr old

87. Regarding mammography, what is true?


A. It is painless

B. Can diagnose breast Ca earlier than self-examination

C. More diagnostic for 70yr old

D. Definitive diagnosis of Ca breast

88. 10 days old boy with vomiting, serum potassium is 7.7; serum Na is 118.what is the
diagnosis.

A. Pyloric stenosis

B. congenital adrenal hyperplasia

C. SIADH

D. Posterior cranial fossa tumour

89. A young boy has fever and limp. There is tenderness at one point in the tibia; all
movements are full except flexion which is restricted to 30degrees. what is the diagnosis?
A. Septic arthritis

B. Osteomyelitis

C. Perthes disease

90. 6 hours after difficult catheterisation, a patient developed fever with chills

A. suppurative urethritis

B. gram negative septicaemia (bacteremia)

C. haemorrhage

91. Regarding hemochromatosis , diagnostic investigation is

A. Sr. ferritin

B. Transferrin

C. Sr .Iron
D. Liver biopsy

92. In Australia the common cause of iron deficiency is

A. Nutritional deficiency

B. Malabsorption

C. Diarrhoea

93. most common cause of bleeding P/R in children

A. fistula in ano

B. fissure in ano

C. haemorrhoid

94. Patient with depression is treated with antidepressants; following a course of


treatment she now wants to stop the drug .What do you advice?
A. Follow up every week

B. Follow up every month

C. Admission to hospital

95. Child with abdominal mass and with metastasis to skull

A. neuroblastoma

B. Willm’s tumour

C. retinoblastoma

96. A child babbles , sits for sometime unsupported ,stands with support and holds an
object by the palm .what is the age (*)

A. 5 months

B. 7 months

C. 1year
D. 18 mon.

97. A patient with dysthymia for 2years ..treatment is (*)

A. SSRI + SSRI+ benzodiazepine

B. SSRI cognitive + behaviour therapy

C. SSRI

D. Antipsychotics +SSRI

98. A young patient vomits 1Litre of blood and has 2 times malaena. what is next
appropriate step

A. Gastroscopy

B. Immediate surgery

C. Barium meal
99. old lady with knee swelling and has weekly positive birefringent crystals ,the crystals
contain

A. calcium pyrophosphate dihydrate

B. calcium hydroxyapatite

C. urate

100. A patient is worried about the (sexual dysfunction and depression) side effects of
sertraline. What would be the advice regarding the same to the patient?

A. It is only side effect of antidepressant

B. both these side effects are not related to sertraline

101. Young man after a RTA was found unconscious only responds to painful stimuli and
eye cold stimuli what is the GCS score

A. GCS >3
B. 3 to 6

C. 6 to 9

102. child with fever and blisters in the palm and sole and mouth ,there is
lymphadenopathy

A. Coxsackie’s

B. Kawasaki

C. Herpes

103. All of the following are scaly lesions except

A. Keratoacanthoma

B. Squamous cell Ca

C. Pityriasis rosea

D. Psoriasis

104. Rash similar to syphilis except


A. Infectious mononucleosis

B. Atopic eczema

C. Discoid eczema

D. Tinea corporis

105. Pregnant lady with carpel tunnel syndrome .what is true?

A. Surgical intervention is rarely needed

B. Splinting in hyperextension

C. Surgical correction

106. what is typical feature of carpal tunnel syndrome

A. severe pain awakening the patient at night

B. pain in the ring and little finger


C. paresthesia restricted to median nerve

D. history of myxedema

107. 3rd day blues what is correct? (*)

A. Occurs at least in 50% after delivery

B. More common following “C “section

C. Always lead to psychosis

D. Results in more premature malformed children

108. regarding child abuse

A. more common in deformed ,premature and LBW children

B. toddlers are more commonly affected

C. majority patients would have a psychiatric disorder


D. other children in the family wont get affected

109. confidentiality can be broken in which of the following situations

A. child abuse

B. when the police comes to ask

C. insurance matters

110. all are hepatotoxic except

A. paracetamol

B. OCP’S

C. Halothane

D. INH
111. carcinomatous change can occur in all except

A. Sjogren’s syndrome

B. thyrotoxicosis

C. coeliac disease

112. 2yr old child with chronic respiratory infection, what would you do?

A. Sweat chloride test

B. X ray chest

C. Barium meal

113. Child with chronic cough and rectal prolapse, diagnosis?

A. Congenital megacolon

B. Cystic fibrosis
C. Whooping cough

114. 6 weeks old child on breast feeding with 4 days constipation and thriving well
diagnosis? (*)

A. Hirschsprung’s disease

B. normal variant

C. hypothyroidism

D. acquired constipation

115. Which of the following is not associated with brain metastasis?

A. Small cell Ca

B. Breast Ca

C. Renal Ca

D. Lung Ca
116. An elderly patient ,known case of Ca rectum , on morphine has chronic
constipation and was treated with lactulose but was not responding .what is the next step

A. Arrange a surgical consultation

B. Enema

C. P/R examination

D. Modify diet

117. All of the following are associated with Raynaud’s except

A. RA

B. SLE

C. Scleroderma

D. Dermatomyositis

E. Ankylosing spondylitis
118. cause of diarrhoea in an old bed ridden patient

A. faecal impaction

B. Carcinoma

C. Constipation

119. Haemolytic anaemia all are true except

A. Increased urobilinogen

B. Icterus and clear urine

C. MCV decreased

120. A child ingested washing powder half an hour ago next management(*)

A. Admit to hospital and possible endoscopy


B. Charcoal

C. Sent him home

D. Syrup of ipecacuanha

E. Observe

121. Mother of an 18 month child was concerned as it was not babbling .audiological
assessment was done when he was 10 months old .what is the next step

A. Arrange audiometry

B. Repeat hearing test

C. Reassure the mother

122. 71/2 yr old girl attained menarche; her mother noticed the breast growth and
axillary hair growth 6 months ago.

A. premature puberty

B. undiagnosed congenital adrenal hyperplasia


C. turner syndrome

123. In a young female patient what is the most common cause of subarachnoid
haemorrhage?

A. Ruptured aneurysms

B. Av malformations

C. Mycotic aneurysms

D. trauma

124. A neonate few hours after birth developed cyanosis which was not responding to
oxygen. NO murmer is there. Diagnosis? (*)

A. Transposition of great vessels

B. Tetralogy of fallot’s

C. VSD

D. ASD
125. Lung Carcinoma seen in non smokers

A. Adenocarcinoma

B. Squamous cell carcinoma

C. Small cell ca.

D. Basal cell ca.

126. A child has facial movements and it is increased on watching TV (question on tics).
They are present even during examination.

A. Myoclonic epilepsy

B. Tics

C. Partial complex epilepsy

D. Panic attack

127. In TNM staging which has a better prognosis


A. T1 N0 M0

B. T1 N1 M0

C. T1 NI MI

128. 60 yr old female patient had repeated bilateral thrombophlebitis and DVT cause

A. pancreatic Cancer

B. ovarian cancer

C. protein c deficiency

129. Most common cause of central cyanosis

A. Left to right shunt

B. Right to left shunt


C. CO poisoning

130. Regarding immunology what is correct?

A. Ig G associated with atopic eczema

B. Delayed hypersensitivity is T cell mediated

131. Why is it difficult to differentiate beta HCG and LH

A. FSH high

B. LH high

C. Prolactin high

D. Progesterone low
132. Young hypertensive male with proteinuria, hematuria and upper respiratory
infection for 2 days .Diagnosis?

A. Ig A nephropathy

B. Glomerulonephritis

C. Nephrotic syndrome

D. Acute pyelonephritis

E. Membrane nephropathy

133. All of these can cause gynecomastia except

A. Spironolactone

B. Cimetidine

C. Labetolol

D. Digoxin
E. methyldopa

134. which of the following is least likely associated with primary hypothyroidism

A. 72y/o with multinodular goitre

B. 28 y/o with menorrhagia

C. 9y/o with retarded bone age

D. 16y/o with anovulatory cycles

135. which of the following is incorrect :

A. alcohol reduces triglycerides

B. olive oil decreases HDL

C. increased LDL/HDL ratio is good

D. Taking 300 gms of fish daily does not reduce cholesterol


136. Point prevalence in schizophrenia means the

A. Current cases at that time

B. Cases in one year

C. The total number of cases

137. Regarding Diabetes mellitus what is true? (*)

A. At least take 100gms of carbohydrate everyday to prevent ketonuria.

B. If one parent has diabetes there is 1:8 chance of getting affected.

C. You can’t give insulin until level comes to normal.

138. Regarding diabetic foot ,all are true ,except (*)

A. 50% mortality following amputation


B. If the pulse is present patient is unlikely to have it.

C. To have diabetic foot controlling glucose level can help

139. Regarding 80% carotid artery stenosis. What is true? (*)

A. The incidence of stroke will be halved

B. 40% stroke with out treatment

C. 50% incidence of stroke after endarterectomy

D. 40% death will occur within 30days

140. Spiral # of humerus which nerve damaged

A. Radial

B. Ulnar

C. Median
D. Volkmann’s ischemia contracture

141. 50 yr old patient wakes up at night due to pain in the calf which is relieved by
walking. What is the cause?

A. Ischemic pain

B. Raynaud’s phenomenon

C. Muscular cramps

D. DVT

E. Intermittent claudication

142. A patient with pain in the calf worsened by elevation of feet

A. Ischemia

B. Nocturnal cramps

C. DVT
143. Which of the following feature is unlikely to be due to arterial ischemia

A. Pain along the buttock and thigh after exertion

B. Weakness of the buttock and thigh

C. Shooting pain from buttock and thigh

D. Leg paralysis

144. Most common feature of rectal carcinoma

A. Tenesmus

B. Bleeding

C. Incomplete defecation

D. rectal prolapse
145. A patient after MVA sustained a pelvic # and has blood in the external meatus
,initial investigation of choice is (*)

A. Urethrogram

B. CT scan

C. Cystoscopy

D. Catherisation

E. IVU

146. Regarding uncomplicated haemorrhoids all are true except

A. Pain

B. Pruritus

C. Bleeding

D. Prolapse

E. Mucus discharge
147. Regarding PMS what is true?

A. 5% have very severe symptoms

B. 40% have PMS

C. all patients with PMS always have dysmenorrhoea

148. Patient mechanic with penetrating hand injury. What is true? (*)

A. Drainage should be done from extensor surface

B. It indicates that there is extension of the palmar abscess into extensor aspect

C. The swelling is due to oedema

D. It indicates the involvement of extensor tendons

149. Apathetic and dull are the negative symptoms of schizophrenia. Which is the other
negative symptom (*)

A. Catatonia
B. Blunted affect

C. Cataplexy

150. A patient who is aware about his own problems during treatment .what is it ? (*)

A. Instinct

B. Insight

C. Pseudo altruism

D. Intellectualisation

151. In chronic liver failure (obstructive jaundice) what’s true

A. Vitamin K absorption is reduced

B. Prothrombin cannot be converted to thrombin

C. Chronic hepatitis
152. In hemochromatosis after venesection all are true except

A. Skin pigmentation becomes normal

B. Cardiac siderosis is reversible

C. Hepatoma can be cured

153. In hemochromatosis

A. size of the liver is decreased following treatment

154. A patient with perforated peptic ulcer all are correct except

A. Patient remembers exactly the moment of rupture

B. Back pain

C. Vomiting
D. Board like rigidity

E. Guarding

155. Concerning cluster headache all are correct except

A. vomiting

B. treat with methysergide

C. occurs after 3-4 hrs of falling asleep

D. occurs at the same time of the day

156. A young man with throat infection in child hood and was treated with penicillin
..He now comes with a fear of developing a rash due to penicillin as he was influenced by
his friends words. What does he have?

A. Induced delusion

B. Hypochondriasis
157. young man with needle marks and pin point pupils ,gag reflexes are normal ,no
extensor plantar reflexes(*)

A. opiate ingestion

B. temporoparietal #

158. A young man with left sucking wound in the chest .After initial airway resuscitation
and circulation management what is the next step(*) C.

A. Debridement and closure

B. Pressure bandage

C. IV fluids

159. Which infection is least likely during childbirth?

A. Syphilis

B. Gonorrhoea
C. Streptococcal

D. HIV

E. Herpes

160. Appropriate Estimation of the foetal gestational age is by (*)

A. Transvaginal ultrasound at 8 weeks

B. Bimanual examinations at 8 weeks

C. Ultrasound at 18 weeks

D. Transvaginal ultrasound at 18 weeks

161. Regarding OCP what is the absolute contraindication?

A. Focal migrainous hemiplegia


B. Hypertension

C. Diabetes

162. Regarding OCP what is correct? (*)

A. All the progesterone’s except cyproterone have derived from testosterone

B. Derive from oestrogen

C. Derive from progesterone

163. Young man with severe joint pain and profuse bleeding from venipuncture site.
Diagnosis? (*)

A. DIC

B. Thrombophlebitis

C. Clotting defect

164. All are true regarding DIC except (*)


A. Fibrin decreased

B. FDP decreased

C. PTT increased

165. intermittent claudication commonest site

A. profunda femoris

B. superficial femoral artery

C. posterior tibial artery

D. bifurcation of abdominal aorta

E. external iliac artery

166. Which defence mechanism is mature?

A. Denial

B. Projection
C. Introjection

D. Humour

167. Undescended testis is most commonly associated with (*)

A. Torsion of testis

B. Spermatogenesis will be perfect if the operation is done before 3 years

C. Development delay of the child

D. Acute epididymitis

E. Hernia

168. G3P0 pregnant lady came to you at 16 weeks pregnancy .She had two spontaneous
abortions at 17 and 18 weeks in the previous pregnancies .how do you treat?

A. Chromosomal analysis
B. Cervical stitch

C. Complete bed rest

169. Regarding Mullerian agenesis all of the following is true except

A. Short vagina

B. Normal breast development

C. X linked recessive

170. What would be the most likely cause for bloody discharge from the nipple in a 50
year old lady?

A. Intraductal papilloma

B. Intraductal carcinoma

C. Paget’s disease

D. Fibrocystic disease
171. The relative marker in Hepatocellular carcinoma is

A. PSA

B. CA125

C. CEA

D. CA153

E. AFP

172. Treatment of seminoma stage 2 with metastasis to the para-aortic lymph nodes

A. Radio therapy to testis and nodes

B. Surgery and radiation to Para aortic nodes

C. Chemotherapy

D. Surgery
173. 70 year old woman with vaginal prolapse with ring pessary what is the
complication (*)

A. atrophic vaginitis

B. cervical carcinoma

C. decubitus ulcer

D. carcinoma of vagina

174. After splenectomy you expect all of the following except (*)

A. Life long increased risk of infection

B. Transient increased platelets to predispose to thrombosis

C. Persistent decrease of RBC life span

D. Decrease in anaemia
175. Diabetic mother after prolonged labour delivered a jittery baby which responds to
oxygen ..what is the diagnosis

A. Hyperglycaemia

B. Hypoglycaemia

C. Meconium aspiration

D. Hyperbilirubinemia

176. Splenectomy is most beneficial in which of the following

A. Haemophilia

B. Chronic ITP

C. Myelofibrosis

177. A middle aged man unable to fall asleep has night mares and remembers his daily
activities diagnosis?
A. Anxiety disorder

B. Depression

C. OCD

D. Insomnia

178. Alcoholism is associated with all of the following except

A. Substance abuse (benzodiazepine)

B. Depression

C. OCD

179. Proteinuria is seen in all of the following except

A. congenital nephritis

B. RPGN

C. UTI
D. RA

E. Horseshoe kidney

180. Tourette syndrome ; all are correct except

A. Motor and vocal tics are seen

B. 2 or more bouts per day

C. the person is not distressed with vocal tics

D. coprolalia is less than 10%

E. onset always less than 18 years of age

181. A 35 year old lady has a grey greenish vaginal discharge, microscopy shows clue
cells, and she responded well to metronidazole .diagnosis (*)

A. Chlamydia
B. Bacterial vaginosis

C. Trichomoniasis

D. Moniliasis

182. Regarding stammering which is correct

A. spontaneous resolution at 16years

B. resolution after 2 year.

C. No resolution in life

D. Operative measure

183. Still birth after a normal delivery. all of the following tests are done except

A. Kleihaur test

B. Chromosomal analysis
C. Foetal parts

D. Maternal antibodies

E. Foetal post-mortem

184. Polyhydramnios all are true except

A. Rh incompatibility

B. Oesophageal atresia

C. bladder neck obstruction

D. Anencephaly

185. Polyhydramnios see in all except

A. Diabetes

B. Multiple pregnancy
C. Cardiac anomalies

D. Hydrops fetalis

186. Child with greyish tonsillar exudate and atypical lymphocytes

A. Infectious mononucleosis

B. CMV

C. ALL

187. Urge incontinence all are true except (*)

A. Colposuspension

B. Taping the vagina

C. Pelvic exercise
D. Weighted vaginal cones

188. (Regarding Rubella and pregnancy) a kindergarten teacher 6 weeks pregnant had
one of her student with rubella .all are true except

A. immunoglobulins

B. IgG is increased initially then IgM

C. Antibody measure and repeat SRH after 10 days

D. If mother infected early then child can have microcephaly, seizures and cataracts

189. A patient with pyloric stenosis vomiting a lot and vomitus contains undigested food
material taken 2 days back. What is the initial management?

A. Normal saline

B. Hartmann’s

C. Ringer lactate
190. daily requirement of potassium

A. 50mmol

B. 1gm

C. 30mmol

191. In pyloric stenosis vomiting is

A. Intermittent

B. Occurs 1hour after food

C. 2 days after food

192. A man with abdominal pain x ray showed multiple fluid levels .what is the
management

A. 2 litres RL

B. 2 litres Hartmann’s before surgery


C. 2 litres Hartmann’s after surgery

D. 2 litres Hartmann’s after surgery

193. regarding antibiotic prophylaxis what is true

A. single dose is sufficient when compared to multidose

B. urologic surgery needs a culture for the specific antibiotic

194. 4 year old child constipated ,rectum empty and sphincter tone lax ,mass at the
apex of rectum

A. Hirschsprung’s

B. acquired megacolon

C. hypothyroidism

195. hypercalcemia seen in all except


A. hyperparathyroidism

B. pancreatitis

C. RTA

196. 3 year old girl with periorbital oedema ,protein 3+, all are true except

A. prednisolone will impr

Guest, Dec 9, 2005

#1

pguest

pguest Guest

thanks

thanks niruben,i have already cleared my mcq exam but i am sure it will be very helpful
to the rest.

i have copied n posted the questions on another site, given u the credit, i hope u dont
mind.

pguest, Dec 9, 2005

#2
Guest

Guest Guest

STICKY TOPICS

I would have preferred to keep the questions here...i think admin should move this page
to sticky topics...

Guest, Dec 12, 2005

#3

Guest

Guest Guest

1-b

2-c or e

3-e

4-a

5-???

6-b

7-a

8d

9-d

10-e

11-a

12-a
13-b and d

14-c or e

15-????

16-d

17-a

18-c

19-a

20-C ??

21-d

22-a

23-a

24-???

25-IS the age 10, if yes then i think the answer is b

26-b

27-c

28-litiunm toxicity

29-b

30-a

31-anxiety

32-phobia

33-d and e

34-b

35-c

36-????
37-b or e

38-a

39-c

40-2

41-a

42-a or b

43-a

44-A???

45-a???

46-b????

Guest, Dec 13, 2005

#4

sadiqaa

sadiqaa Guest

answers..psychiatry

1...b

2...e

The diagnostic criteria for pyromania are:

deliberate and purposeful firesetting on more than one occasion;

tension or emotional arousal before the act;

intense interest, curiosity or fascination about fire (which can include fire equipment
and the consequences of fire);
pleasure, gratification or relief when setting or witnessing fires and their aftermath;

the firesetting is not done for another motive such as financial gain, anger or revenge,
to gain recognition or to relieve boredom, and is not done in response to a delusion or
hallucination or due to impaired judgment (such as through intoxication); and

the firesetting is not better accounted for by conduct disorder, antisocial personality
disorder or a manic episode.

3...c

4...a

5...d

6...b

7...b

8...d

9...a

10...c

11...f

12...a

13...b

14...e

15...e

16...e 6 weeks post partum

17...b...not sure ,this is what happens in seasonal dysthymic disorder

18...a Delusion

A delusion is commonly defined as a fixed false belief and is used in everyday language
to describe a belief that is either false, fanciful or derived from deception.

19... :roll:
20...e

21...d

22...a

23...b...also in alcoholism and cocaine addiction

24...e

25...b

26...b

27...c

28...developed what :?

29...a????

32....fear is a common cause

33...a

35...c

37...b

38...a

39...d

41...a

43...a

44...a

45...a

46...b

sadiqaa, Aug 7, 2007

#5
sadiqaa

sadiqaa Guest

MEDICINE

1...b....in ant.dislocations

3...c

4...c

5...d

6...a

7...d

8...a

9...c

10...b

11...c

12...b

13...d

14...d

15...c

16...b

17...b

18...c

19...c

22...c

23...c
24...d

26...c

27...e...

28...e

29...a

30...d

31...e....not sure

32...b.......not sure

34...d

35...b

36...c

37...a

38...d

39...c

41...c

43...c

44...c

46...c

47...c

48...c

49...c

50...b

51...c

52...d...????
53...advice to stop taking pills from the this pack that will initiate bleeding and start
another .

54...a

55...c

56...a

57...d

58...c

59...a???

60...c

61...???

62...e

63...

sadiqaa, Aug 7, 2007

#6

Guest

Guest Guest

MEDICINE

63...a

65...d

66...c ....its 41 weeks!

67...c..b/c volkmann contractures r common in this injury

68...b
69...a though all epiphyseal # dont cause growth retardation..it depends on severity of
injury, invovement of growth plate

70...e....if it is "hypoplasia"

Guest, Aug 8, 2007

#7

Guest

Guest Guest

MEDICINE, SURGERY , OBSTETRICS AND PSYCHIATRY

1...b

2...a

3...c

4...b

5...c...US ?????

6...all r false except d

7...d...afterload reducers r not used in AS.

8...d

9...b

10...a

11...a

12...c caused in this age group by S. pnemoniae, H.influenzae and staphlococcus.

13...b...excessive crying causes cyanosis

14...b

15...c????
16...c

17...c???

18...a

19...a

20...b...b/c the most common cause of scrotal pain in infants and children is torsion f
testes

22...b ...

23...b

24...b

25...c

26...b

28...d????

30...b

Guest, Aug 8, 2007

#8

Guest

Guest Guest

31...a

32...c ????? anyone??

33...b

34...c

35...e

37...b
38...e

39...d

40...a

41...b

42...a

43...c

44...a

45...a

Guest, Aug 10, 2007

#9

Guest

Guest Guest

46...d ...c can also be true b/c sometimes acoustic neuroma patients dont complain of
tinnitis.

47...D

PICA =posterior inf cerebellar artery .

Lateral medullary syndrome (also called Wallenberg's syndrome and posterior inferior
cerebellar artery syndrome) is a disease in which the patient has difficulty with swallowing
or speaking or both owing to one or more patches of dead tissue (known as an infarct)
caused by interrupted blood supply to parts of the brain.

This syndrome is characterized by sensory deficits affecting the trunk and extremities on
the opposite side of the infarct and sensory and motor deficits affecting the face and
cranial nerves on the same side with the infarct. Other clinical symptoms and findings are
ataxia, facial pain, vertigo, nystagmus, Horner's syndrome, diplopia and dysphagia. The
cause of this syndrome is usually the occlusion of the posterior inferior cerebellar artery
(PICA) at its origin.
The affected persons have difficulty in swallowing (dysphagia) resulting from
involvement of the nucleus ambiguus, and slurred speech (dysphonia, dysarthria).
Damage to the spinal trigeminal nucleus causes absence of pain on the ipsilateral side of
the face, as well as an absent corneal reflex.

The spinothalamic tract is damaged, resulting in loss of pain and temperature sensation
to the opposite side of the body. The damage to the cerebellum or the inferior cerebellar
peduncle can cause ataxia.

48...c

49..d

50...b

Guest, Aug 10, 2007

#10

Guest

Guest Guest

46...d ...c can also be true b/c sometimes acoustic neuroma patients dont complain of
tinnitis.

47...D

PICA =posterior inf cerebellar artery .

Lateral medullary syndrome (also called Wallenberg's syndrome and posterior inferior
cerebellar artery syndrome) is a disease in which the patient has difficulty with swallowing
or speaking or both owing to one or more patches of dead tissue (known as an infarct)
caused by interrupted blood supply to parts of the brain.

This syndrome is characterized by sensory deficits affecting the trunk and extremities on
the opposite side of the infarct and sensory and motor deficits affecting the face and
cranial nerves on the same side with the infarct. Other clinical symptoms and findings are
ataxia, facial pain, vertigo, nystagmus, Horner's syndrome, diplopia and dysphagia. The
cause of this syndrome is usually the occlusion of the posterior inferior cerebellar artery
(PICA) at its origin.

The affected persons have difficulty in swallowing (dysphagia) resulting from


involvement of the nucleus ambiguus, and slurred speech (dysphonia, dysarthria).
Damage to the spinal trigeminal nucleus causes absence of pain on the ipsilateral side of
the face, as well as an absent corneal reflex.

The spinothalamic tract is damaged, resulting in loss of pain and temperature sensation
to the opposite side of the body. The damage to the cerebellum or the inferior cerebellar
peduncle can cause ataxia.

48...c

49..d

50...b

Guest, Aug 10, 2007

#11

Guest

Guest Guest

51...b

52...a

54...a

57...d???

62...c
63...a

65...e

66...d

67...O2 then adrenaline

69...a

Guest, Aug 10, 2007

#12

Guest

Guest Guest

57. A chest x ray of a child showing consolidation > neutrophils increased. What could
be the cause

A. Klebsiella pneumonia

B. Group B streptococcus

C. Staphylococcus

D. Mycoplasma pneumonia

57...if the child is upto 2 months old its group B strep.


if he is older then staph is the right answer

Guest, Aug 10, 2007

#13

Guest

Guest Guest

74...c

75...b

76...a

77...b

78...b

79...a

80...a

81...b

82...a

83...a

84...a

86...b

87...b

88...b

89...b

90...b
2009 SEP 12TH RESIT QUE

1) Post op Delirium--alcohol withdrawal


2)COPD--LFT explanation and further mx
3) Hydrocele, transilluminable DD and further mx, 22year old boy
4)Repeated candidiasis 23year old lady
5)Downs syndrome, first suupect with examination, explain to mum
6)TIA mx 45year old lady with weakness of left leg DM type1
7) Depression, 65year old male with wt loss constipation llq pain, all ix normal take psychological hystro and DD
8) Acute pyelonephritis. 40 year old female with flush and flu like symptom.

AMC MCQ 30/11/2007 SYDNEY RECALLS AND PREPARATION TIPS

 Hi,
I was sitting the AMC MCQ offshore exam in Sydney 30/10/2007. I will write down my
recalls, some other information about the exam and a few other general things people might
find helpful. Good luck with your preparation.

The exam itself seemed pretty hard to me, at least there were not very many recalls (I guess
only 20 questions!). I was often struggling and rarely feeling 100% sure about my answers,
especially with the mastery questions. A lot of topics were the same but changed in a way
that was quite confusing. And even if I have prepared by reading the common topics from
recalls I was usually unsure about my answer! Psychiatry seemed to be a major part
including the mastery questions. A lot of mastery questions were also O&G.

No pictures from the Anthology book, no ECGs, no CTGs, only 1 or 2 CXRs, and maybe
6-10 pictures in total. Bloods were always given in a way where you could see the normal
range.

We were 10 people siting the exam in Sydney. We were able to bring the bags in the room
and leave them at the entrance, some lollies were provided but drinks were not allowed at
the computer. The air conditioning was on and I can only advise you guys to bring a
sweater!

For preparation I did the AMC MCQ book twice and the anthology book pictures twice. I
read mainly the Oxford handbook of clinical medicine, surgery and specialities to prepare
in advance for about 2 months besides working full time, and repeated old papers and
questions for the last 2 weeks with literature search in a medical library and internet
resource as up to date. I only did a few old exams but read throughout the main topics
which I didn’t know the answers to. And by the way, I just got my results and I did
well.

Now to the questions, I must say it is really hard to recall them, as there are so many and
you really don’t have much time to go through them twice. I just managed to finish my
questions 3 minutes before the time ran out! So this is just a try to recall the questions and
write them from my perspective as in what I read into the question! So please keep that in
mind and don’t only rely on these recalls! And don’t rely on the answers. I must
admit I had to guess a lot in this exam! And even if I might some notes right after the exam
I just managed to type them down for you a few weeks later. So don’t get too fixed on
the exact words of the questions!

Medicine and general:

1. Picture of mouth with –what looked to me like a- small cyst underneath the tounge?
a) Ranula
b) haemangioma

2. Picture of hand with flexion contracture mainly IV and V th. digit. Dx (Diagnosis)?
a) Duyptren’s contracture

3. Female 28yr, neck stiffness, no focal or other neurology, head CT normal, CSF picture:
200 red cells, protein slightly increased to 70, Dx?
a) SAH
b) Meningitis
c) nothing serious/normal headache

4. CSF: picture bacterial meningtitis. Dx?


a) Bacterial meningitis

5. Blood picture with increased AP, ALT (>AST), AST, hep B and C negative. Dx?
a) viral hepatitis

6. Blood picture increased AP, ALT, Bilirubin. Dx?


a) Cholelithiasis
b) Cholecystitis

7. 85 years, ductus hepaticus mass or palpable gallbladder, also cardiomegaly and


hepatomegaly in medical history. Tx (treatment)?
a) biliary shunt
b) whipple op

8. Bloods with increased ESR

9. Cant recall question anymore but it contained all this: patient with rash on tonsils. Dx:
viral - swab, ASO titre, WCC, fever , LN

10. Anaemia blood picture with decreased iron and increased transf. Dx?
a) Normocytic anaemia

11. Status asthmaticus proper treatment in ED?


a) Rehydrate, inh. Salbutamol, parenteral steroids
b) rehydrate, salbutamol inh., ipratropium inh.
c) theophylline and…
12. Vit. B12 deficiency picture (at least my impression). Sx (symptoms)?
a) PNP

13. Leucaemia definitive diagnosis? LAD, fatigue, increased LUC, Hb 110 and

14. PE (pulmonary embolism) proven with V/Q scan in a young girl. Tx?
a) Iv heparin
b) Warfarin
c) Thrombolysis

15. 80yr old lady with PE post a long distance flight. Tx?
a) Iv heparin
b) Warfarin
c) Thrombolysis

16. PE postv a rectal surgery. Tx?


a) Iv heparin
b) Warfarin
c) Thrombolysis

17. Acute drop of thrombocytes to 50.

18. Patient develops oedema and HTN (hypertension), Hx: ESRF (end stage renal failure)
and diabetes mellitus. Treatment?
a) Start ACE-I
b) Thiazide
c) beta-blocker

19. Tx with thiazide and ACE-I in combination. Can cause the following changes:
a) Insulin increase
b) insulin decrease
c) glucose increase
d) potassium decrease

20. Alcohol, now acute confusion

21. Wernicke patient develops low BSL in ED. Tx?


a) Thiamine, ten Glucose infusion
b) 10ml of 50% (or 20ml of 10%) Glucose stat
c) Glucose infusion
d) Glucagon

22. Alcohol

23. Culture came back positive for salmonella infection on the 4th day after travelling.
Patient already received a 3-day treatment of metronidazole. His symptoms haven’t
changed since. Mx (management)?
a) Metronidazole for 3 weeks
b) do nothing

24. Alcohol abuse now with acute nausea and vomiting. Abdominal pain also to back. In
ED Shock. Dx?
a) Pancreatitis

25. 10 years history of acidity/reflux, well controlled with medicine, no history of weight
loss. Now dysphagia/swallowing preoblems especially for meat. Dx?
a) Oesophagus carcinoma
b) stricture of esophagus
c) barrets esophagus
d) side effects of drugs

26. Bleeding ulcer on endoscopy and test positive for helicobacter pylori. Mx?
a) Adrenalin injection on endoscopy
b) iv omeprazole

27. Best test to re-check post 7days of triple therapy for H. pylori?
a) Urease breath test

28. Fluids resuscitation for burn victim with 15% surface affected, 80kg? (my addit
4x15x80=4800ml)
a) give 3 l Hartmans and 2 l dextrose
b) same with blood products and Hartmanns
c) N/Saline and Hartmanns

29. Burns with…. Mx?


a) Intubation

30. 15yr old girl with Crohn’s disease. Comes with faecal soiling. No pain/ no blood.
On examination you will find?
a) Vesicle or so
b) Fistula
c) Fissure

31. Rash on legs

32. Patient with haemoptysis and haematuria and Epistacsis. Dx?


a) Wegener’s

33. Patient with haemoptysis and haematuria. Investigation?


a) Antimembr. Ab
b) ANA

34. 78 yr old wife with cognitive changes, confusion. Ix?


a) CT
35. Fecal impectation rectum

36. Alcoholic male found behind bar, previous episode of binch drinking, speech…? Dx?

37. Man /boy collapsed during rugby/football game on the field. No contact to other players
before on field. Now loss of consciousness for 30 minutes. Dx?
a) Vasovagal
b) SAH
c) Seizure

38. congestive heart failure

39. Long-standing smoker, age around 80 presents with frothy sputum, pleuritic chest pain,
tachycardic, clear chest on auscultation, post flight. Dx?
a) PE
b) Cancer

40. Long-standing smoker, age around 80 presents with haemoptysis, pleuritic chest pain,
tachycardic, clear chest on auscultation, post flight…some other question about it

41. Longstanding smoker presents with neurological signs on hands and arms

42. Patient with PE szenario. ECG normal but tachycardia 100bpm. What do you expect to
see on CXR?
a) Nothing
b) Effusion

43. WPW history. Now tachycardic with (no ECG was given, only description) broad
complexes in ECG and it looks like AF. Tx?
a) Electrocardioversion
b) Digoxin

44. ECG ST elevation in V1-V3, SVT (only describtion, no ECG given). Most likely
diagnosis?
a) PE

45. Patient with SVT and low BP and decreased level of conciousness. Treatment?
a) Verapamil
b) Sotalol
c) Valsalva maneuver
d) Cold water

Surgery:
1. Patient with rectus muscle pain and mass on the side of abdomen when strained. Nothing
to palpate when lying down. Dx?
a. Rectus strain
b. hernias with several names which I never heard of

2. Ruptured AAA

3. Wound of patient’s fist/hand 14 hours post injury. What to do after debridement?


a. Suture and antibiotics
b. Suture and drain
c. Leave open

4. Empty rectum but blood (? Can’t remember what I meant by that in my notes)

5. Bowel motions every 4-5 days only?


a. Normal

6. Mass in right upper quadrant that moves with breathing. Dx?

7. When do you do an explorative laparotomy post accident at work (explained the whole
szenario)?
a. Retroperitoneal air
b. (?retroperitoneal) Bleed
c. blood collection in liver
d. blood collection in spleen

8. Patient with fracture presents with pain on forearm and hand, increased on extension,
slight swelling. Extension fixation. Cause?
a. Haematoma
b. Damage median nerve
c. Carpaltunnel syndrome

9. MVA (motor vehicle accident) with fractured ribs 2-7 (bilateral?). Now presents with
difficulty breathing and pain. Mx?
a. Strapping ribs
b. Nerval block of ribs at…
c. Intubate and ventilate

Paediatrics:

1. Picture of child with generalised icterus. Which investigation would you perform to
confirm the diagnosis? (Nothing like bili or so given)

2. Pictures of a child’s back showing pink scaly patch, about 2 x 1 cm?

a. Rosacea
3. Picture of CXR of a child with left upper lobe consolidation and some round shaped
lesions, (?pneumatocele). (Not 100% sure if this was the question but I think it was Which
treatment?
a. Flucloxacillin

4. CSF given. I think viral picture. Child post birth, developes vesicles somewhere. Tx?
a. Acyclovir

5. Child with 3 days of fever then develops a rash. Dx?


a. 3 day fever

6. Child with scaly rash on neck, macular, mouth pink tounge, sore throat. Dx?
a. Scarlet fever
b. Mumps

7. Child younger than1 yr., soft palpable mass behind nipple. Tx?
a. Reassurance and monitoring

8. Leucaemia, lethargy for 3 months, Lab. shows leuocytosis with mainly functioning
leucos. What is the investigation to confirm most likely diagnosis?
a. LN aspiration
b. Philadelphia chromosome

9. Child 13 or 18 months old, SSRI ingested, as per parents “quite some amountâ€. How
can you evaluate the risk of toxicity?
a. ECG
b. EUC
c. Level

10. Asthma 10-13yr old child, 2 attacks in the last year, now for general check-up in your
surgery. Mother states child had cough all winter. Mother concerned. Currently on ventolin
PRN. Mx?
a. Start PEF monitoring and measure regularly
b. start preventer (name given)

11. Asthma exacerbation again in child. Mx?


a. Fluticasone
b. salmeterol inh.
c. Ipratropium
d. 6hrly ventolin

12. Ikterus

13. Preterm baby develops respiratory distress a few hours after birth. Dx?

a. RDS
14. Toddler was at birthday party of family member. Sudden onset of cough and wheeze.
Mx?
a. In-and epiratory CXR
b. Bronchoscopy

15. Similar question, can’t recall the difference: Toddler was at birthday party of family
member. Sudden onset of cough and wheeze. Mx?
a. In-and epiratory CXR
b. Bronchoscopy

16. 2 yr. old swallowed a 2.5cm long screw. AXR shows the screw in the RUQ. Mx?
a. R/V in 2 days
b. Endoscopy
c. Follow-up ultrasound

17. Child with burns and stridor. Mx?


a. Intubation

18. CF (cystic fibrosis) sweat test. Child with recurrent respiratory infections, no weight
gain

19. *Milestones question. Child can roll, hold head up, sit bending forward, babble, grasp
palm, stand when hold. Age?
a. 5 months
b. 7 months
c. 10 months
d. 12 months

20. SIDS. Parents are coming to you telling you this is all they are doing now and want
advice. What can be most effective after 1st affected child to prevent?
a. Baby alarm
b. Stop smoking
c. Breast feeding only
d. Lay supine
e. Lay on belly

21. What is APGAR if child HR 140, moves all limbs, cries, arms cyanosed and maybe a
few more hints?
a. 9

22. Lateral CXR of a child with stridor. Dx?


a. Epiglotitis
b. Laryngomalazia
c. Croup

23. Child, bowel obstruction


24. Neonate with respiratory distress

25. Intussuseption szenario

26. Child 3 yr. old with laceration on scalp. Clean and not contaminated. Child not
immunised. Mx?
a. Antibiotics
b. Tetanustoxin
c. Tetanusimmunoglobulin
d. TDPertussis vaccination
e. TD vaccination

27. Hirschsprungs

28. Child with decreased femoral pulses, Dx?


a. Coarctation
b. Hypoplastic left heart

29. Child 12 hours post birth, cyanotic, no murmur

30. Neonate at 1.5hours. Nurse states did turn blue on hands and feet. On examination
normal heart and lung examination. What is true?
a. Seizure
b. Start giving oxygen
c. Early resp. distress signs

31. Child around 3 yrs with abdominal mass and haematuria. Dx?
a. Wilms tumour

32. Child 14yr, pain on side of knee. Limb shortened, limps, hib abducted. (Note: not given
obese in history) Dx?
a. Slipped epiphysis
b. Perthes disease
c. Osteochondrosis

33. Child with cough and dyspnoea. On auscultation crepitations on the right and dull on
percussion over right middle lobe. Dx?
a. Effusion
b. Pneumonia

34. Child with haemophilus meningitis and pneumonia gets exposure to other child (age
<4yr) without immunisation. Mx of the exposed child?
a. Amoxicillin orally for 4 days
b. Rifampicin for 4 days
c. Cefachlor oral for 4 days
d. Chloramphenicol shot im
e. Penicillin iv

35. Child born and not able to take breath. What u will do first?
a. phryngeal suction
b. intubation and ventilation
c. wait and watch
d. x-ray

36. Parents asking about risk for their future child, as mother’s brother has
arachnodactily?
a. The risk for the child is same as in normal population
b. 50% change of having
c. 50% change of being a carrier

37. parents came to physician, they have a child with the cleft palate and want to know
what is the probability of cleft palate in their second child
a. 3%
b. 7%
c. 0.4%
d. 4. 4%

O&G:

1. Galactorrhoea 4 months post breast-feeding was stopped, child now 8 months old. Tx?
a. Reassurance
b. Bromocriptine

2. Women 30-32 weeks gestation comes in with symptoms of preeclamsia. Fetus seems
well. Management?
a. Hydralazine
b. Induce labour
c. CS

3. Girl on OCP (oral contraceptive pill) for 2 months, not working as she has no regular
menstruation and co. Mx?
a. Continue for 2 more months

4. 15 yr old girl has collected information about depot contraception and requests it.
Management?
a. Re-educate her about options and risks, then give it to her if she still insists on it
b. Ask mother for permission
c. It is contraindicated in that age group

5. Vagina dry, cyclic (pre)menopause


6. Vaginal/vulva swelling and pain Dx?
a. Barthol. Abscess
b. Bart. Cyst

7. Rash vaginal or so Dx?


a. Secondary to syphilis
b. Ulcer vulva

8. Vaginal itch

9. Absolute contra indication (CI) for IUD?


a. Previous cervix
b. STD
c. Uterus malformation

10. IUC using female, now amennorhoea. Result shows she is pregnant. Mx of IUD?
a. Leave in
b. Take out immediately by pulling on the wires you can see
c. Scope

11. 45 yr old female with positive FNA for cancer. What do you do to decide for
conservative Tx?
a. Check LN axillary manually to exclude spread
b. Mammogram
c. US of breast (comment: no option for MRI or CT given)

12. Mother 18 weeks of gestation with bacterial vaginosis. Mx?


a. Tell her to check again at 28 weeks and if still there treat with…
b. Metronidazole now
c. No need for action

13. Mother, GBS (Group B strep.) positive at 30 weeks gestation. When to give IV
penicillin?
a. In labour
b. Perform CS and give during CS
c. Start with first cut of CS
d. To baby after birth

14. Primary amenorrhoea in a 18yr old girl. No breast development. Mx?


a. Karyotyping

15. Parents with 3 previous miscarriages. Mx?


a. Karyotype of both parents

16. Uterus umbifig. What’s true about pregnancy?


a. Will never be pregnant
b. Tell her normal chances
c. Most likely infertile
d. Increased risk for miscarriage

17. Mother who is breast-feeding gets mastitis. Mx?


a. Stop breastfeeding until she is well to prevent contamination
b. Stop on affected side
c. Stop feeding from affected side and use breast pump there

18. Mother Hepatitis B positive

Psychiatry: (there were many psych. questions but those are the ones I just can’t
remember!)

1. Middle aged women presents low mood. Sounds like depression. Started St. John’s
Worth 300mg and it helped a bit but still not enough and still depressed. Not suicidal. Mx?
a. Increase St. Johns Worth to 600mg
b. Add SSRI (name and dose given, think Fluoxetine 75mg)
c. Wash out period for St. John’s Wrth, then start SSRI (name and dose given)

Ethics & Co:

1. Primary prevention means?


a. Decreased prevalence
b. Dcreased incidence and preterm death

2. Old man with dementia with new diagnosis of untreatable cancer. His wife is the carer
and asks you not to tell him about diagnosis as he would get depressed and then she
couldn’t care for him anymore at home. Which allows you most not to tell the diagnosis
to the patient?
a. You made the decicion that patient would not understand the diagnosis
b. Wife can’t care for him anymore if he hears diagnosis
c. Treatment of dementia plus depression is very difficult

3. Old man lives with daughter and son in law. Comes to you as a GP with bruises. He tells
you about some abuse/violence by the son in law but asks you not to tell the son in law or
his daughter. Mx?
a. Call police and tell them
b. Cll daughter to talk about it
c. Tll ptient to come for regular visits at you surgery from now on to check-up on him

Surgery at night, an emergency operation is getting ready to start. The surgeon walks in
ataxic with symptoms of being drunk/current ETOH abuse. You talk him but he replies
“mind your own businessâ€. Then what u will do next
4. Mx?
a. Call security to stop him and take him out of theatre (?or police)
b. Call your bosses/ CMO stat
c. Call ethic commun. Or board stat
d. Call someone tomorrow morning
e. Find other surgeon to replace him/ talk to another colleague that he will take charge

5. Study about HIV. Blind study.


a. Tell people the test results
b. As it is a blind study you can’t tell the patients but advise them to do HIV test
themselves afterwards

6. GP can be involved in clinical study. What is your main responsibility?


a. To be familiar with the Helsinky
b. Make sure it fits the ethics
c. Make sure your patients get paid

7. which one can decrease the prevalence of a disease in the population? herd immunity/
treatment/ health programme
Guest, Dec 2, 2007
#1

Guest Guest

thx, good on you


Guest, Dec 3, 2007
#2

Guest Guest

how did u sit Offshore MCQ in Sydney? Venues changed? anywell, thaxs for sharing your
expenrience.
Guest, Dec 3, 2007
#3

Guest Guest

need help

hi there .i am keen the exam plz help me with the notes


Guest, Mar 6, 2008
#4

Guest Guest

would abyone like to discuss the answers???


Guest, Mar 7, 2008
#5

nyeinoo Guest

want to check the answers.

yes i want to check the answer. how can i do that?


do u have email?
nyeinoo, May 24, 2008
#6

nyeinoo Guest

want to discuss

it is my email nyeinoo@gmail.com
want to discuss on 30.11.07
can send the answer with number. i will send u back after i get email from u. thanks
nyeinoo, May 24, 2008
#7

Guest Guest

answers

may u please send mw za answers of obs & gyne


muhfa85@gmail.com
tanks in advance
Guest, May 31, 2008
#8

NEELAM Guest

AMC BOOKS

do u still have amc books. iam looking for amc books. my email id
neelampsingh77@rediffmail.com
Cheers

Neelam
NEELAM, Jun 7, 2008
#9

AMCdoctor Guest

what is the answer

what is the anwseer for them


5 a 3 weeks old baby presented with jaundice which began at 4 days of life. Total bilirubin
is 200( conjugated bilirubin 120)WOF diagnosis

A Congenital hypothyroidism
B Sepsis
C Rh incompatibility
D Biliary atresia
E Breast feed

6 A mother brought his 9 years old boy to the GP , She was concerned that her son is too
fat. On examination , he had hight (90 Centile) , weight (95 centile) . Which of the
following investigation would you find in this boy
A Serum growth hormone level
B Serum cortisol level
C Serum parathyroid hormone level
D Serum TSH level
E Serum aldosterone level
AMCdoctor, Jun 12, 2008
#10

pradeepsingh_79 Guest

Thanks for the help

Thanks for the help.


Pradeep
pradeepsingh_79, Aug 30, 2008
#11

Guest Guest
hi thanks very much, its really of great help...
Guest, Nov 28, 2008
#12

Guest Guest

hi,thanks.can anyone send the answers of these qs on drpotdars@yahoo.co.in


Guest, Apr 1, 2009
#13

Guest Guest

Hi there

I'm planning to take AMC in September, I'm in Newcastle looking for astudy group
Guest, May 14, 2009
#14

Guest Guest

what do you guys think about the canadaQbank ,is it a good idea to do this 2 mnths
beforeamc mcq or not?
Guest, May 14, 2009
#15

anisha Guest

hi

hi can you post answers of these to me on dranu78@yahoo.co.in


thanks!!!!
anisha, Jan 13, 2010
#16

sandeep83 Guest

hey so nice of u to post questions ..thnks. can u mail me answers too..


sandeep_12_06@yahoo.co.in
sandeep83, Mar 8, 2010
#17

Guest Guest

Hello

Thanks for your useful information about AMC!


As "Mr angelitainca" , most of the people who passed AMC exam say OHCM(Oxford
Hand Book of Clinical Medicine), OHCS(surgery), Oxford hand book of clinical
specialities , GP books(Sold by AMC) are quite enough to sit the exam if you practiced
AMC old questions.

See you,
I'ld like to share and receive information regarding AMC coz I might sit AMC at Perth in
November 2010.

COLLECTION OF MCQS FOR AMC PART 1

1. A WOMEN WHO HAS PREVIOUSLY DELIVERED A 4KG BABY WITH TROUBLESOME


DELIVERY. SHE IS PARA €“2 AND GRAVIDA 3. YOU SHOULD DO ALL OF THE FOLLOWING
EXCEPT

A) DO CAESAREAN SECTION AT TERM****

B) CHECK BLOOD GLUCOSE LEVEL REGULARLY FROM 30 WEEKS

C) DO X-RAY PELVIMETRY

D) INDUCTION OF LABOUR IN 37-38 WEEKS

2. NO BLEEDING AFTER WITHDRAWING CONTRACEPTIVE PILLS WOULD BE DUE TO

A) OVARY

B) UTERUS

C) PITUITARY
D) HYPOTHALAMUS

3. ALL OF THE FOLLOWING COULD BE CAUSE OF GREENISH FOUL SMELL DISCHARGE


EXCEPT

A) TRICHOMONIASIS

B) GARDENELLA

C) FB

D) CERVICAL CANCER

4.BENIGH OR MALIGNANT NEOPLASIA WILL CAUSE A WATERY AND PINK OR BLOOD


STAINED DISCHARGE

GARDENELLA WILL CAUSE A GREY FUOL SMALING DISCHARGE

5.A PRIMIGRAVIDA 38 WKS IN LABOUR. ON P/V PELVIS WAS REDUCED. STATION 0


CERVIX 4 CM DILATED, MEMBRANE INTACT

A) X-RAY PELVIMETRY

B) C/S

C) OXYTOCIN

D) CONTINIOUS TRIAL LABOUR****

6. A PT WHO HAS DIFFICULTIES WITH PREGNANCY. SHE LOST HER 3 PREGNANCY IN 6, 8,


10 WEEKS. BEFORE THAT SHE HAD TERMINATE 2 BABY AT 10 AND12 WKS. WHAT COULD
BE THE CAUSE

A) CERVICAL IN COMPETENCE

B) DEFICIENT LUTEAL PHASE


7. FIND OUT IN CORRECT ASSOCIATION WITH PREGNANT LADY WHICH CAUSE FOETAL
EFFECT ON PREGNANCY

A) IUGR- ANTICARDIOLIPIN

B) PLATELET COUNT 100,000 €“ INTRACRANIAL HAEMORRHAGE

C) HEART BLOCK €“ANTIBODY

8. ALL OF THE FOLLOWING COULD BE THE CAUSE OF ANEAMIA IN INFANCY EXCEPT

A) PREMATURITY

A CHILD WITH MULTIPLE SUBPERIOSTIAL HAMOTOMA AND CALCIFICATION

A) SCURVY****

B) ACCIDENTAL INJURY

C) UNACIDENTAL INJURY

D) OSTEOMYELITIS

SUBPERIOSTEAL HEMORRHAGE IS A TYPICAL FINDING OF INFANTILE SCURVY. THE


LOWER ENDS OF THE FEMUR AND TIBIA ARE THE MOST FREQUENTLY INVOLVED SITES.
THE SUBPERIOSTEAL HEMORRHAGE IS OFTEN PALPABLE AND TENDER IN THE ACUTE
PHASE.

9.A CHILD WAS BORN NORMALLY WT3.2 KG. APGER SCORE WAS 5 IN 1 MIN AND 8 IN 5
MIN. HE HAS DELAYED DEVELOPMENT. WOF CONDITION IS ASSOCIATED WITH HIS
DEVELOPMENTAL DELAY?

A) PATERNAL UNCLE HAS INTELLECTUAL DEFICIT

B) FATHER IS A ALCOHOLIC

C) SISTER HAS FEBRILE SEIZURE


D) MOTHER HAS 2 CAF這S LAIT SPOTS

E) PATERNAL GRANDMOTHER HAS HYPOTHYROIDISM

.10. A MIDDLE-AGED MAN PRESENTS WITH BP80/60, HR120, BACK PAIN AND SLIGHT
ABDOMINAL GUARDING, WOF IS MOST LIKELY DIAGNOSIS

A) ACUTE PANCREATITIS

B) ACUTE MYOCARDIAL INFARCTION

C) LEAKING AORTIC ANEURYSM****

D) PERFORATED PEPTIC ULCER

.11. 34 YR OLD MAN PRESENTS WITH ACUTE ABDOMEN FOR 3 DAYS AND HE HAS H/O
VOMITING FOLLOWING BY PERINEAL PAIN. O/E ABDOMEN DISTENDED BUT NON TENDER,
BOWEL SOUND ABSENT. ABDOMINAL X €“RAY SHOWS MULTIPLE AIR FLUID LEVELS.
WHAT IS THE BEST MANAGEMENT BEFORE SURGERY?

A)2000 ML 4% IN 0.45% NACL

C) 2000ML OF HARTMAN SOLUTION BEFORE OPERATION****

D) 2000ML OF HARTMAN SOLUTION DURING SURGERY

E) 2000ML OF 5% DEXTROSE

12.FOLLOWING THE BIOCHEMISTRY OF A PATIENT WHO UNDER RX OF VOMITING

NA-117 CL 86 K+3.2 SERUM OSMOLALITY 900

MOST LIKELY CAUSE

A)SODIUM DEPLETION

B)WATER INTOXICATION
C)SIADH

D)DI

IT CAN NOT BE SAIDH AS SERUM OSMOLARITY IS TOO HIGH

CAN NOT BE DI AS NA IS TOO LOW

CAN NOT BE WATER INTOXICATION AS OSMOLALITY IS TOO HIGH AGAIN

I DON'T KNOW ABOUT SODIUM DEPLETION

.13.NA-168 SERUM OSM 200ML CL-10

MOST LIKELY CAUSE

A) DI

B) SIADH

C) ACUTE TUBULAR NECROSIS

14..55 YEAR OBESE LADY CAME WITH GENERALISED PAIN, PULSE IRREGULAR, BP 90/60
MMOF HG ABDOMEN WAS TENDER AND REBOUND TENDERNESS PRESENT WITH
DISTENSION. P/R DENOTE BLOOD STAINED

A) RUPTURED DIVERTICULITIS***

B) MESENTERIC ISCHEMIA

C) CA RECTUM

.15. ALL THE FOLLOWING ARE CAUSEOF ISCHEMIC PAIN EXCEPT

A) NUMBNESS IN BUTTOCK

B) CALF PAIN

C) PARALYSIS OF THIGH MUSCLE


D) SUDDEN SHOOTING PAIN IN THIGH****

16.A PT WITH ASCITIS ,PARACENTESIS SHOWS MALIGNANT CELL. O/E WHAT WILL BE
FOUND

A) SPLENOMEGALY

B) HEPATOMEGALY

C) PALPABLE NODE P/R

D) SUPRACLAVICULAR NODE ****

MALIGNANT ASCITES IS SEEN MOST COMMONLY IN PATIENTS WITH OVARIAN,


ENDOMETRIAL, BREAST, COLON, GASTRIC, AND PANCREATIC CANCER

17€¢ A PATHOLOGIC LEFT-SIDED SUPRACLAVICULAR NODE (VIRCHOW NODE)


SUGGESTS THE PRESENCE OF UPPER ABDOMINAL MALIGNANCY

18.A ONE-MONTH-OLD BABY PRESENTED WITH VOMITING AND WT. LOSS FOR LAST
TWO WEEKS. ON EXAMINATION BABY FOUND VERY WEEK & LETHARGIC, THERE WAS NO
MASS IN THE ABDOMEN. WHAT WILL YOU DO NEXT?

A)BA MEAL FOLLOW THROUGH

B)GASTRIC EMPTING STUDY

C)ULTRASONOGRAPPHY

D)URINE ANALYSIS

E)ENQUIRY ABOUT FEEDING ****** TAKE DETAILED HISTORY FIRST


19.A 62 YRS. MALE, SMOKER WITH BACKGROUND OF COPD WAS NORMAL 2 DAYS AGO,
NOW NEIGHBORS DISCOVERED HIM AS CONFUSED, DIFFICULTY IN RESPIRATION, COUGH
& ASTERISTIX IN HANDS. WHAT IS THE DX?

A)UREMIC ENCEPHALOPATHY

B)HEPATIC ENCEPHALOPATHY *******

C)CO2 NARCOSIS

D)RESPIRATORY ACIDOSIS

20.A 68 YRS. OLD MAN PRESENTS WITH COUGH, DYSPNOEA, AND RECURRENT
HAEMORRGHIC PLEURAL EFFUSION. HE HAS SILICOSIS; HE ALSO WORKED IN A MINE FOR
10 YRS. NOW HE WORKS IN A HYDROCARBON PLANT, WHAT COULD BE THE DX?

A)SILICOSIS

B)TUBERCULOSIS

C)MESOTHELIOMA ***** DUE TO ASBESTOSIS

D)ASTHMA

E)METASTATIC CARCINOMA

21.A MAN WEIGHING 84 KG., HEIGHT 179 CM. SMOKER, AND DRINKS ALCOHOL 4-6
GLASS ON ROUTINE EXAMINATION FOUND TO HAVE BP OF 150/[SNIP] MM OF MERCURY,
HE DOESN€™T WANT TO USE HIS SEATBELT WHILE DRIVING HIS CAR. WHICH OF THE
FOLLOWING IS ONE OF THE MOST IMPORTANT MEASURES FOR HIS MANAGEMENT?

A) REDUCE 14 KG. OF WEIGHT IMMEDIATELY

B) STOP SMOKING

C) CHORTHIAZIDE EVERYDAY
D) STOP ALCOHOL *****

E) ASK HIM TO WEAR SEATBELTS NOT SURE

22..A MAN PRESENTED WITH FEVER, COUGH AND SORE THROAT. HIS SPLEEN WAS
REMOVED BEFORE. WHAT WILL YOU GIVE HIM?

A)HAEMOPHILLUS VACCINE

B)PNEUMOCOCCAL VACCINE

C)PENICILLIN I/V *****

D)ORAL ANTIBIOTIC

E)REGULAR ANTIBODY ESTIMATION

23.A MAN PRESENTED WITH FEVER, COUGH AND SORE THROAT. HIS SPLEEN WAS
REMOVED BEFORE. WHAT WILL YOU GIVE HIM?

A)HAEMOPHILLUS VACCINE

B)PNEUMOCOCCAL VACCINE

C)PENICILLIN I/V

D)ORAL ANTIBIOTIC

E)REGULAR ANTIBODY ESTIMATION

24.WHICH OF THE FOLLOWING WILL ASSIST IN THE DIAGNOSIS OF INFANTILE


MENINGITIS?

A) HYPERTHERMIA

B) BULGING OF FONTANALES IS NECESSARY


C) HYPOTHERMIA

D) NECK STIFFNESS

E) APNOEIC SPELL **** NOT SURE

25. A PICTURE OF INTRA-ORAL SWELLING, REDNESS WITH LITTLE PUS EXUDATES SAID
TO BE OF SUBMANDIBULAR GLAND, WHICH OF THE FOLLOWING MEASURE YOU WILL
TAKE?

A)SIALOGRAM *****

B)M.R.I.

C)INTRA ORAL DENTAL X-RAY

D)ULTRASONOGRAPHY

E)ANTIBIOTIC

26.-A PICTURE -- BLACK LITTLE INDURATIONS PRESENTED IN THE INNER ASPECT OF THE
LITTLE TOE, THE LADY SAID SHE DID NOTICED IT VERY RECENTLY. WHICH OF THE
FOLLOWING IS THE MOST LIKELY.

A) CONGENITAL BENIGN NAEVUS

B) CARCINOMA

C) VASCULAR MALFORMATION ****

D) INFECTION

27-A PATIENT IN MND LEFT A NOTE , NOT TO INTUBATE, DO NOT FEED BY


NASOGASTRIC FEEDING. NOW THE PATIENT CAN€™T EAT, CAN€™T SPEAK , HE WAS
SENT TO THE DIET AND NUTRITIONAL DEPARTMENT. WHAT WILL YOU DO?
A) DO INTUBATION

B) INSERT A NASOGASTRIC FEEDING TUBE

C) GIVE PARENTERAL FEEDING *****

D) SET A CENTRAL VENOUS LINE

E) SEND FOR TERMINAL CARE

28. IN DIABETIC KETO- ACIDOSIS WHICH OF THE FOLLOWING IS RESPONSIBLE FOR


HYPERKAELEMIA?

A) LACK OF SODIUM

B) LACK OF INSULIN ****

C) LACK OF ELECTROLYTES

D) LACK OF GLUCOSE

29-A 28 WEEKS PREGNANT LADY PRESENTS TO YOU BRUISES AND MARKS OF VIOLENCE
ON HER BODY INCLUDING HER ABDOMEN. ON QUESTIONING SHE ADMITS THAT HER
PARTNER BEATS HER. WHAT WILL YOU ADVICE TO HER?

A)CALL THE POLICE

B)TREAT HER ACCORDINGLY **** THATS IS TO SEND HER TO A GYANEAC

C)GIVE HER SOME ADDRESSES OF SAFE HOUSE AND COUNSELING

D)ASK HER NOT TO TELL ANYBODY

E)MARRIAGE COUNSELING WITH INCLUDING HER HUSBAND *** THIS IS ALSO POSSIBLE
30-WHICH OF THE FOLLOWING COULD BE THE OUTCOME OF AVOIDEND PERSONALITY
DISORDER?

A)DELUSIONAL DISORDER

B)SOCIAL PHOBIA *****

C)AGGRESSIVE BEHAVIOR

D)PERSECUTORY DELUSION

E)ANXIETY DISORDER

31-A MOTHER CAME WITH HER 5 YRS. OLD WITH ASTHMA. SHE STARTED WITH
FLIXOTIDE INHALATION 4 HOURLY. NOW SHE FEARS OF LONG TERM SIDE EFFECTS OF THE
FLIXOTIDE (FLUTICASONE PROPIONATE), WHAT WILL YOU TELL TO HER?

A) LINEAR GROWTH RETARDATION

B) CANDIDIASIS *****

C) OSTEOPOROSIS

D) WT. GAIN

E) OSTEOMALACIA

32.-A MAN VISITING FROM INDONESIA PRESENTING WITH FOR FEVER & SORE THROAT
FOR LAST 3 DAYS. ON EXAMINATION THERE WAS LYMPHADENOPATHY, BUT NO
SPLENOMEGALI, LYMPHOPHENIA, & MONOSPOT TEST WAS NEGATIVE. WHICH OF THE
FOLLOWING YOU WILL DO?

A)EBV SEROLOGY

B)BLOOD CULTURE *****

C)HIV TEST
D)THROAT SWAB CULTURE

E)GIVE HIM PENICILLIN

33-A MAN COMPLAINING OF REPEATED BLURRING OF VISION FOR FEW WEEKS. NOW
HE COMPLAINS OF BLURRING OF VISION FEEL LIKE A CURTAIN COMING DOWN OVER HIS
VISION. WHICH OF THE FOLLOWING CAN BE THE CAUSE?

A) AMAUROSIS FUGAX ****

B) RETINAL DETACHMENT

C) CAROTID ARTERY OCCLUSION

D)CENTRAL VEIN THROMBOSIS

33.STEPHEN, AGED 18 YRS, PRESENTS WITH SPONTANEOUS EPISTAXIS . WHAT IS THE


FIRST STEP IN MANAGING STEPHEN?

A. DIRECT PRESSURE TO THE LOWER NOSE FOR TWO MINUTE INTERVALS ****→ Q ASK
FIRST STEP MX

B. POSITION STEPHEN SO THAT HE IS SITTING AND LEANING FORWARD (POSITIONING IS


THE FIRST STEP, IF DIRECT PRESSURE IS APPLIED.. IT MUST BE FOR 10 MINUTES WITHOUT
INTERRUPTING, NOT 2 MIN INTERVALS.)

C. APPLICATION OF TOPICAL LOCAL ANAESTHETIC

D. CAUTERY OF BLEEDING VESSELS

E. NASAL PACKING WITH GAUZE

BULIMIA'S COLLECTION, JAN 24, 2007

#1

BULIMIA'S COLLECTION

BULIMIA'S COLLECTION GUEST


1.ALL OF THE FOLLOWING CAN CAUSE EXCRUCIATING ABDOMINAL PAIN EXCEPT:

A) APPENDICITIS

B) M.I.

C) RENAL COLIC****

D) RUPTURE OF ABDOMINAL AORTA

E) MESENTERIC ARTERY OCCLUSION

2. YOU MAY DO ALL AFTER MEMBRANE RUPTURE FOR 2 DAYS AT 32 TH WEEK


GESTATION EXCEPT

A) TAKE A SWAB

B) GIVE STEROIDS

C) CHECK FIBRONECTIN IN VAGINAL FLUID

D) INDUCE LABOUR EVEN THERE IS ON EVIDENCE OF INFECTION

3) REGARDING OCP, ALL OF THE FOLLOWING ARE CORRECT EXCEPT:

A) INCREASE TRIGLYCERIDE***

B) REDUCE OVARIAN CANCERS

C) REDUCE BENIGN BREAST DISEASES

D) REDUCE ENDOMETRIAL CANCERS

4.A PT WITH 1.8 CM TUMOR IN THE PITUITARY AND 3MM ON OPTIC CHIASM. WHAT IS
THE INITIAL RX
A) TRANSFRONTAL RESECTION

B) TRANSPHENOIDAL RESECTION

C)BROMOCRIPTINE****

HERE IT DOESN'T SAY THAT IT IS A PROLACTINOMA. IF IT IS A PROLACTINOMA THEN WE


CAN GIVE BROMOCRIPTINE.

5. REGARDING IMMUNOLOGY, WOF STATEMENT IS CORRECT?

A) KILLER CELLS ARE T- SUBSETS AND KILL TUMOR CELLS

B) B CELLS PRODUCE FIRST IGM AND LATER IGG****

C) IGM NEED THE HELP OF COMPLEMENT

STATEMENT C IS ALSO CORRECT.

6. A 14 YR OLD GIRL WITH PHENYTOIN. SHE CAME TO YOU FOR OCP .WOF IS MOST
APPROPRIATE

A) MICROGYNON 30****

B) MICROGYNON 50

A) DIANE 35

SINCE AEDS ARE ENZYME INDUCERS AND OCP METABOLISM IS INCREASED, A HIGHER
DOSE IS MOST APPROPRIATE.

7. A 8 WEEKS PREGNANT LADY COME WITH VAGINAL BLEEDING. YOU WILL DO USG DUE
TO ALL OF THE FOLLOWING REASON EXCEPT
A) TO SEE THE CAUSE OF BLEEDING

B) TO DETERMINE THE GESTATIONAL AGE

C) TO SEE THE FOETAL MALFORMATION (MOLAR PREGNANCY)****,

D) THE POSITION OF THE SAC IN THE UTERUS

E) CHECK THE VIABILITY OF THE FETUS

BULIMIA'S COLLECTION, JAN 24, 2007

#2

BULIMIA'S COLLECTION

BULIMIA'S COLLECTION GUEST

Q.1

GEORGE IS A 19YRS OLD MAN WHO PRESENTS WITH DYSURIA. ON EXAM. THERE IS A
THICK YELLOW URETHRAL DISCHARGE. WHICH ONE OF THE FOLLOWING IS CORRECT WITH
REGARD TO GONOCOCCAL INFECTION?

A. URINE PCR IS NOW THE GOLD STANDARD FOR DIAGNOSING GONOCOCCAL


URETHRITIS.

B. SPREAD TO INVOLVE THE PROSTATE AND EPIDIDYMIS IS COMMON.

C. THE INCUBATION PERIOD IS USUALLY 14 TO 21 DAYS.

D. CEFTRIAXONE IM AS A SINGLE DOSE IS AN EFFECTIVE TREATMENT. *****

E. PHARYNGEAL GONORRHOEA IS DIAGNOSED BY FINDING GRAM €“VE DIPLOCOCCI


ON A SMEAR.

INCUB PERIOD IS 2 TO 5 DAYS

CULTURE IS CONFIRMATORY

EPIDIDYMO ORCHITIS IS A COMPLICATION


Q.2

BETTY, WHO IS AGED 29 YRS AND NULLIPAROUS, COMES TO YOU FOR A ROUTINE PAP
SMEAR. THE CERVIX LOOKS HEALTHY AND THERE ARE NO ABNORMALITIES ON CLINICAL
EXAM. A WEEK LATER SHE RETURNS FOR THE PATHOLOGY RESULT WHICH IS REPORTED
AS: €˜ABNORMAL CELLS ARE PRESENT, CONSISTENT WITH A DIAGNOSIS OF CARCINOMA
IN SITU CIN 3€™. WHAT IS THE IMPLICATION OF THIS RESULT?

A. BETTY REQUIRES A CONE BIOPSY

B. THE SMEAR SHOULD BE REPEATED AFTER BETTY HAS APPLIED CLINDAMYCIN


VAGINAL CREAM FOR 7 DAYS

C. BETTY HAS A MALIGNANCY OF THE CERVIX AND HYSTERECTOMY IS REQUIRED

D. BETTY HAS MALIGNANCY OF THE CERVIX AND REQUIRES RADIOTHERAPY FOLLOWED


BY HYSTERECTOMY

E. BETTY REQUIRES A COLPOSCOPY *******

Q.3

FEMALE ANDROGEN INSUFFICIENCY SYNDROME INCLUDES ALL OF THE FOLLOWING


EXCEPT:

A. PERSISTENT FATIGUE

B. REDUCED BODY HAIR

C. DECREASED LIBIDO

D. BLUNTED MOTIVATION

E. DYSPHORIA *****?

Q.4
ALL OF THE FOLLOWING ARE EFFECTIVE FIRST LINE TOPICAL TREATMENT FOR SEVERE
DANDRUFF EXCEPT:

A. SELENIUM SULPHIDE

B. ZINC PYRITHIONE ****

C. BETAMETHASONE

D. MICONAZOLE

E. KETOCONAZOLE

MILD TOPICAL STEROIDS , ANTIFUNGALS ARE USED. SO WHY NOT MICONAZOLE?

Q.5

IN ASSESSING A PATIENT FOR ADULT-ONSET ASTHMA, WHICH OF THE FOLLOWING IS


FALSE:

A. THERE IS USUALLY A HISTORY OF PAST OR PRESENT CIGARETTE SMOKING

B. THERE IS USUALLY A LONG HISTORY OF ATOPIC DISEASE *****

C. ATTACKS CAN BE TRIGGERED BY CHEMICAL AND PHYSICAL AGENTS

D. THE ASTHMA TENDS TO BE MORE CHRONIC THAN EPISODIC

E. THE ASSOCIATED COUGH CAN BE HARSE, SUFFUSING AND NON PRODUCTIVE

LATE ONSET ASTHMA IS NON ATOPIC

Q.6

WHICH OF THE FOLLOWING FACTORS WOULD NOT BE A POOR PROGNOSTIC INDICATOR


FOR JOINT REPLACEMENT IN HIP OSTEOARTHRITIS IN A 65 YRS OLD MAN?
A. AGE

B. A BMI OF 33

C. ASSOCIATED DIABETES

D. PRESENCE OF OSTEOPOROSIS

E. A PREVIOUS JOINT REPLACEMENT ***********?

NOT SURE

Q.7

TOM, AGED 18YRS, PRESENTS WITH A TYPICAL FIRST EPISODE OF SCHIZOPHRENIA.


WHICH OF THE FOLLOWING ANTIPSYCHOTIC MEDICATIONS IS THE TREATMENT OF
CHOICE FOR HIM?

A. CHLORPROMAZINE

B. HALOPERIDOL

C. THIORIDAZINE

D. OLANZAPINE ***********

E. CLOZAPINE

Q.8

THE DIAGNOSIS OF ACUTE GONORRHOEA IN A MALE IS IDEALLY MADE BY:

A. GONOCOCCAL COMPLEMENT FIXATION TEST

B. VDRL REACTION

C. DARK GROUND ILLUMINATION OF URETHRAL PUS ****

D. GRAM STAIN AND CULTURE OF URETHRAL PUS


E. PROSTATIC MASSAGE

Q.9

A COMMON SIDE EFFECT OF THE ATYPICAL ANTIPSYCHOTIC DRUG, OLANZAPINE, IS :

A. NEUTROPENIA

B. HYPOTENSION

C. SEXUAL DYSFUNCTION

D. WEIGHT GAIN **************

E. PARKINSONISM

Q.10

A TRAUMATIC PERFORATION OF THE EAR THAT HAS OCCURRED IN WET CONDITIONS


SUCH AS SWIMMING OR WATER-SKIING WILL OFTEN :

A. BE ASSOCIATED WITH A PURULENT DISCHARGE

B. BE COMPLICATED BY A STAPHYLOCOCCAL INFECTION

C. REQUIRE A SHORT COURSE OF ORAL ANTIBIOTICS *********

D. NOT HEAL SPONTANEOUSLY

E. REQUIRE SURGICAL REPAIR

NOT SURE

Q.11

WHEN SOMEONE IS REFERRED TO AS BEING IN THE PRE-CONTEMPLATION STAGE WITH


REGARD TO CHANGE IN BEHAVIOUR, THIS MEANS S/HE IS :
A. RESISTANT TO CHANGE

B. PREPARING FOR CHANGE

C. RECEPTIVE TO CHANGE

D. LOOKING FORWARD TO SPECIFIC ADVICE

E. HAS NOT YET CONSIDERED CHANGE ****

THANK U ATOOSA FOR THE EXPLANATION

Q.12

EPISTAXIS IS UNLIKELY TO ARISE FROM :

A. INJURY TO THE TURBINATES

B. SPONTANEOUS BLEEDING FROM LITTLE;S AREA

C. ANTICOAGULATION THERAPY

D. ENLARGE ADENOIDS **********

E. NASAL FRACTURE

Q.13

STEPHEN, AGED 18 YRS, PRESENTS WITH SPONTANEOUS EPISTAXIS . WHAT IS THE FIRST
STEP IN MANAGING STEPHEN?

A. DIRECT PRESSURE TO THE LOWER NOSE FOR TWO MINUTE INTERVALS**********

B. POSITION STEPHEN SO THAT HE IS SITTING AND LEANING FORWARD

C. APPLICATION OF TOPICAL LOCAL ANAESTHETIC

D. CAUTERY OF BLEEDING VESSELS

E. NASAL PACKING WITH GAUZE

POSITION SHOULD BE SITTING UP AND HEAD DOWNWARDS.


Q.14

WHAT IS THE CAUSE OF THE GREATER LIFE EXPECTANCY AT BIRTH OF FEMALES THAN
MALES IN AUSTRALIA

A. MALES EXERCISE MORE THAN FEMALES

B. GENETIC AND BIOLOGICAL DIFFERENCES

C. FEMALES SEEK HEALTH CARE FACILITIES MORE THAN THE MALES *********

D. MALES DIE MORE IN ACCIDENTS AND VIOLENCE THAN THE FEMALES

E. EMPLOYMENT STRESS IS MORE FOR MALES

???

Q.15

WHICE ONE OF THE FOLLOWING SITUATIONS WILL NOT REQUIRE PROPHYLACTIC


ANTIBIOTICS TO MANAGE A WOUND:

A. AN ELDERLY DEBILITATED PATIENT

B. THE WOUND IS MORE THAN 8 HRS OLD

C. THE PATIENT HAS ALCHOLIC LIVER DISEASE

D. A DEEP WOUND TO THE HAND

E. A LARGE SUPERFICIAL ABRASION ON THE THIGH OF A 22 YRS OLD ********

BULIMIA'S COLLECTION, JAN 24, 2007

#3

BULIMIA'S COLLECTION

BULIMIA'S COLLECTION GUEST


1.A WOMEN WHO HAS PREVIOUSLY DELIVERED A 4KG BABY WITH TROUBLESOME
DELIVERY. SHE IS PARA €“2 AND GRAVIDA 3. YOU SHOULD DO ALL OF THE FOLLOWING
EXCEPT

A) DO CAESAREAN SECTION AT TERM

B) CHECK BLOOD GLUCOSE LEVEL REGULARLY FROM 30 WEEKS

C) DO X-RAY PELVIMETRY ****

D) INDUCTION OF LABOUR IN 37-38 WEEKS

I DON'T THINK X RAY PELVIMETRY IS OF USE HERE( FOR AN EXPECTED BIG BABY).

IF IT PROGRESS TO TERM THEN THE POSSIBILITY IF CESAREAN SECTION IS MOST LIKELY.

2. NO BLEEDING AFTER WITHDRAWING CONTRACEPTIVE PILLS WOULD BE DUE TO

A) OVARY

B) UTERUS

C) PITUITARY ****

D) HYPOTHALAMUS ****

THIS IS POST PILL AMENORRHOEA WHICH IS DUE TO OVER SUPRESSION OF


HYPOTHALAMIC PITUTARY AXIS BY THE ESTORGEN AND PROGESTERONE OF OCP. SO I
THINK BOTH C AND D ARE CORRECT.

3. ALL OF THE FOLLOWING COULD BE CAUSE OF GREENISH FOUL SMELL DISCHARGE


EXCEPT

A) TRICHOMONIASIS

B) GARDENELLA

C) FB
D) CERVICAL CANCER

? B AND D

4. A PRIMIGRAVIDA 38 WKS IN LABOUR. ON P/V PELVIS WAS REDUCED. STATION 0


CERVIX 4 CM DILATED, MEMBRANE INTACT

A) X-RAY PELVIMETRY

B) C/S

C) OXYTOCIN

D) CONTINIOUS TRIAL LABOUR****

I THINK CHOICE D IS APPROPRIATE SINCE X RAY PELVIMETRY IS OBSOLETE NOW A DAYS


AND IF THE PROGRESS IS WELL GO FOR A TRIAL.

5. A PT WHO HAS DIFFICULTIES WITH PREGNANCY. SHE LOST HER 3 PREGNANCY IN 6, 8,


10 WEEKS. BEFORE THAT SHE HAD TERMINATE 2 BABY AT 10 AND12 WKS. WHAT COULD
BE THE CAUSE

A) CERVICAL IN COMPETENCE **********

B) DEFICIENT LUTEAL PHASE

CERVICAL INCOMPETENCE CAUSES MID TRIMESTER ABORTIONS ONLY. SO THE CHOICE


IS B ONLY.
6.. FIND OUT IN CORRECT ASSOCIATION WITH PREGNANT LADY WHICH CAUSE FOETAL
EFFECT ON PREGNANCY

A) IUGR- ANTICARDIOLIPIN

B) PLATELET COUNT 100,000 €“ INTRACRANIAL HAEMORRHAGE ********

C) HEART BLOCK €“ANTIBODY

FOR AN ICH THE COUNT SHOULD BE LESS THAN 40,000.

7.. ALL OF THE FOLLOWING COULD BE THE CAUSE OF ANEAMIA IN INFANCY EXCEPT

A) PREMATURITY

B) MULTIPLE PREGNANCY

MAY BE A

8. A CHILD WITH MULTIPLE SUBPERIOSTIAL HAMOTOMA AND CALCIFICATION

A) SCURVY****

B) ACCIDENTAL INJURY

C) UNACIDENTAL INJURY

D) OSTEOMYELITIS
9. A CHILD WAS BORN NORMALLY WT3.2 KG. APGER SCORE WAS 5 IN 1 MIN AND 8 IN 5
MIN. HE HAS DELAYED DEVELOPMENT. WOF CONDITION IS ASSOCIATED WITH HIS
DEVELOPMENTAL DELAY?

A) PATERNAL UNCLE HAS INTELLECTUAL DEFICIT

B) FATHER IS A ALCOHOLIC

C) SISTER HAS FEBRILE SEIZURE

D) MOTHER HAS 2 CAF這S LAIT SPOTS ***********

E) PATERNAL GRANDMOTHER HAS HYPOTHYROIDISM

NEUROFIBROMA? .

10. A MIDDLE-AGED MAN PRESENTS WITH BP80/60, HR120, BACK PAIN AND SLIGHT
ABDOMINAL GUARDING, WOF IS MOST LIKELY DIAGNOSIS

A) ACUTE PANCREATITIS

B) ACUTE MYOCARDIAL INFARCTION

C) LEAKING AORTIC ANEURYSM****

D) PERFORATED PEPTIC ULCER

11. 34 YR OLD MAN PRESENTS WITH ACUTE ABDOMEN FOR 3 DAYS AND HE HAS H/O
VOMITING FOLLOWING BY PERINEAL PAIN. O/E ABDOMEN DISTENDED BUT NON TENDER,
BOWEL SOUND ABSENT. ABDOMINAL X €“RAY SHOWS MULTIPLE AIR FLUID LEVELS.
WHAT IS THE BEST MANAGEMENT BEFORE SURGERY?

A)2000 ML 4% IN 0.45% NACL

C) 2000ML OF HARTMAN SOLUTION BEFORE OPERATION****

D) 2000ML OF HARTMAN SOLUTION DURING SURGERY


E) 2000ML OF 5% DEXTROSE

12.FOLLOWING THE BIOCHEMISTRY OF A PATIENT WHO UNDER RX OF VOMITING

NA-117 CL 86 K+3.2 SERUM OSMOLALITY 900

MOST LIKELY CAUSE

A)SODIUM DEPLETION

B)WATER INTOXICATION

C)SIADH

D)DI

13.55 YEAR OBESE LADY CAME WITH GENERALISED PAIN, PULSE IRREGULAR, BP 90/60
MMOF HG ABDOMEN WAS TENDER AND REBOUND TENDERNESS PRESENT WITH
DISTENSION. P/R DENOTE BLOOD STAINED

A) RUPTURED DIVERTICULITIS

B) MESENTERIC ISCHEMIA *******

C) CA RECTUM
IRREGULAR PULSE SUGGESTIVE OF AF AND AN EMBOLUS CAUSING MESENTRIC
ISCHEAMIA.

14.. ALL THE FOLLOWING ARE CAUSEOF ISCHEMIC PAIN EXCEPT

A) NUMBNESS IN BUTTOCK

B) CALF PAIN

C) PARALYSIS OF THIGH MUSCLE

D) SUDDEN SHOOTING PAIN IN THIGH****

15.. A PT WITH ASCITIS ,PARACENTESIS SHOWS MALIGNANT CELL. O/E WHAT WILL BE
FOUND

A) SPLENOMEGALY

B) HEPATOMEGALY ********

C) PALPABLE NODE P/R

D) SUPRACLAVICULAR NODE

BULIMIA'S COLLECTION, JAN 24, 2007

#4

GUEST

GUEST GUEST
1. A WOMEN WHO HAS PREVIOUSLY DELIVERED A 4KG BABY WITH TROUBLESOME
DELIVERY. SHE IS PARA €“2 AND GRAVIDA 3. YOU SHOULD DO ALL OF THE FOLLOWING
EXCEPT

A) DO CAESAREAN SECTION AT TERM

B) CHECK BLOOD GLUCOSE LEVEL REGULARLY FROM 30 WEEKS

C) DO X-RAY PELVIMETRY ***

D) INDUCTION OF LABOUR IN 37-38 WEEKS

IF FIRST ONE IS VAGINAL DELIVERY NO NEED TO DO PELVIMETRY HERE.

2. NO BLEEDING AFTER WITHDRAWING CONTRACEPTIVE PILLS WOULD BE DUE TO

A) OVARY

B) UTERUS

C) PITUITARY ***

D) HYPOTHALAMUS ***

BOTH

3. ALL OF THE FOLLOWING COULD BE CAUSE OF GREENISH FOUL SMELL DISCHARGE


EXCEPT

A) TRICHOMONIASIS

B) GARDENELLA

C) FB

D) CERVICAL CANCER
4 A PRIMIGRAVIDA 38 WKS IN LABOUR. ON P/V PELVIS WAS REDUCED. STATION 0
CERVIX 4 CM DILATED, MEMBRANE INTACT

A) X-RAY PELVIMETRY

B) C/S

C) OXYTOCIN

D) CONTINIOUS TRIAL LABOUR ***

5. A PT WHO HAS DIFFICULTIES WITH PREGNANCY. SHE LOST HER 3 PREGNANCY IN 6, 8,


10 WEEKS. BEFORE THAT SHE HAD TERMINATE 2 BABY AT 10 AND12 WKS. WHAT COULD
BE THE CAUSE

A) CERVICAL IN COMPETENCE

B) DEFICIENT LUTEAL PHASE

INCOMPLETE RECALL

6. FIND OUT IN CORRECT ASSOCIATION WITH PREGNANT LADY WHICH CAUSE FOETAL
EFFECT ON PREGNANCY

A) IUGR- ANTICARDIOLIPIN

B) PLATELET COUNT 100,000 €“ INTRACRANIAL HAEMORRHAGE

C) HEART BLOCK €“ANTIBODY ***

ANTI-RO ANTIBODY ASSOCIATED CONGENITAL COMPLETE HEART BLOCK

7. ALL OF THE FOLLOWING COULD BE THE CAUSE OF ANEAMIA IN INFANCY EXCEPT


A) PREMATURITY

B) MULTIPLE PREGNANCY

INCOMPLETE RECALL

8 A CHILD WITH MULTIPLE SUBPERIOSTIAL HAMOTOMA AND CALCIFICATION

A) SCURVY ***

B) ACCIDENTAL INJURY

C) UNACIDENTAL INJURY

D) OSTEOMYELITIS

9 A CHILD WAS BORN NORMALLY WT3.2 KG. APGER SCORE WAS 5 IN 1 MIN AND 8 IN 5
MIN. HE HAS DELAYED DEVELOPMENT. WOF CONDITION IS ASSOCIATED WITH HIS
**DEVELOPMENTAL DELAY?

A) PATERNAL UNCLE HAS INTELLECTUAL DEFICIT -ASSOCIATED WITH MENTAL


RETARDATION(FRAGILE X)

B) FATHER IS A ALCOHOLIC (FETAL ALCOHOL SYNDOM ASSOCIATED WITH ALCOHOLIC


MOTHER)

C) SISTER HAS FEBRILE SEIZURE

D) MOTHER HAS 2 CAF這S LAIT SPOTS (NORMAL IQ-NEUROFIBROMA BUT 2 SPOTS


MAY BE PRESENT WITH NORMAL PERSON)

E) PATERNAL GRANDMOTHER HAS HYPOTHYROIDISM( WHAT ABOUT THE MOTHER


CONDITION? )
10. A MIDDLE-AGED MAN PRESENTS WITH BP80/60, HR120, BACK PAIN AND SLIGHT
ABDOMINAL GUARDING, WOF IS MOST LIKELY DIAGNOSIS

A) ACUTE PANCREATITIS

B) ACUTE MYOCARDIAL INFARCTION

C) LEAKING AORTIC ANEURYSM ***

D) PERFORATED PEPTIC ULCER

11. 34 YR OLD MAN PRESENTS WITH ACUTE ABDOMEN FOR 3 DAYS AND HE HAS H/O
VOMITING FOLLOWING BY PERINEAL PAIN. O/E ABDOMEN DISTENDED BUT NON TENDER,
**BOWEL SOUND ABSENT. ABDOMINAL X €“RAY SHOWS ***MULTIPLE AIR FLUID
LEVELS. WHAT IS THE BEST MANAGEMENT BEFORE SURGERY?

A)2000 ML 4% IN 0.45% NACL

C) 2000ML OF HARTMAN SOLUTION BEFORE OPERATION

D) 2000ML OF HARTMAN SOLUTION DURING SURGERY

E) 2000ML OF 5% DEXTROSE

INSTINAL OBS . BEFORE OPERATION WE USE NORMAL SALINE.

12..FOLLOWING THE BIOCHEMISTRY OF A PATIENT WHO UNDER RX OF VOMITING

NA-117 CL 86 K+3.2 SERUM OSMOLALITY 900

MOST LIKELY CAUSE

A)SODIUM DEPLETION

B)WATER INTOXICATION***

C)SIADH ***

D)DI
13.NA-168 SERUM OSM 200ML CL-10

MOST LIKELY CAUSE

A) DI ***

B) SIADH

C) ACUTE TUBULAR NECROSIS

14.55 YEAR OBESE LADY CAME WITH GENERALISED PAIN, PULSE IRREGULAR, BP 90/60
MMOF HG ABDOMEN WAS TENDER AND REBOUND TENDERNESS PRESENT WITH
DISTENSION. P/R DENOTE BLOOD STAINED

A) RUPTURED DIVERTICULITIS

B) MESENTERIC ISCHEMIA ***

C) CA RECTUM

15. ALL THE FOLLOWING ARE CAUSEOF ISCHEMIC PAIN EXCEPT

A) NUMBNESS IN BUTTOCK

B) CALF PAIN

C) PARALYSIS OF THIGH MUSCLE

D) SUDDEN SHOOTING PAIN IN THIGH ***

16. A PT WITH **ASCITIS ,PARACENTESIS SHOWS ***MALIGNANT CELL. O/E WHAT WILL
BE FOUND
A) SPLENOMEGALY ---LYMPHOMA WITH MATASTASIS

B) HEPATOMEGALY--2NDARY MALIGNANCY

C) PALPABLE NODE P/R --COLON CARCINOMA

D) SUPRACLAVICULAR NODE --INTRA ABD. MALIGNANCY

ALL MAY BE POSSIBLE

GUEST, JAN 24, 2007

#5

GUEST

GUEST GUEST

A 16 YEAR OLD GIRL PRESENTS TO HER GP WITH SECONDARY AMENORR

T/F-- Q1. A 16 YEAR OLD GIRL PRESENTS TO HER GP WITH SECONDARY AMENORRHOEA
OF 7 MONTHS DURATION. SHE HAS NO MEDICAL HISTORY OF NOTE AND DENIES ANY
SEXUAL ACTIVITY.

I) ANDROGEN INSENSITIVITY SYNDROME IS A POSSIBLE CAUSE F

II) GIVEN THE HISTORY, A PREGNANCY TEST WOULD BE OF LITTLE VALUE F

III) THYROID FUNCTION TESTS SHOULD BE INCLUDED IN THE INITIAL

INVESTIGATIVE TESTS. T

IV) A PROGESTIN CHALLENGE FOLLOWED BY A WITHDRAWRAL BLEED IS


SUGGESTIVE OF AN ANOVULATORY DISORDER. T?

V) LOW LEVELS OF FSH AND LH ARE SUGGESTIVE OF A HYPOTHALAMIC DISORDER T

Q2. EARLY PREGNANCY LOSS:

I) IS MORE COMMON IN WOMEN WITH A RETROVERTED UTERUS F

II) SHOULD BE DETECTABLE FROM LACK OF FETAL HEART ACTIVITY ON USS

(ULTRASOUND SCAN) AT 5 WEEKS. F

III) IS GREATLY INCREASED IN SYPHILITIC MOTHERS F

IV) IS ASSOCIATED WITH AUTOIMMUNE THYROID DISEASE T

V) REQUIRES UTERINE EVACUATION IF THERE IS INCOMPLETE ABORTION T

Q3. THESE GYNAECOLOGICAL CONDITIONS AND POTENTIAL TREATMENTS ARE

CORRECTLY MATCHED:

I) HYPERPROLACTINAEMIA + CABERGOLINE ( BROMOCRIPTIN IS USE)

II) HIRSUTISM + SPIRONOLACTONE F


III) FIBROIDS + GNRH AGONISTS ?

IV) DYSFUNCTIONAL UTERINE BLEEDING + NSAID'S F

V) DETRUSOR INSTABILITY + OXYBUTININ F

Q4. REGARDING CONGENITAL DIAPHRAGMATIC HERNIAS.

I) OVER 80% OCCUR ON THE RIGHT F

II) THEY ARE ASSOCIATED WITH PULMONARY HYPOPLASIA AND PULMONARY

HYPOTENSION. T

III) THEY MAY BE DIAGNOSED ANTENATALLY. T

IV) THEY INVARIABLY PRESENT WITH RESPIRATORY DISTRESS AT ABOUT 6

WEEKS OF AGE. F

V) AFFECTED BABIES MAY BENEFIT FROM INHALED NITRIC OXIDE. F

5. REGARDING SYMPTOMATIC PRE-ECLAMPSIA

I) IT MAY DEVELOP FROM 12 WEEKS GESTATION F

II) DIURESIS IS A PRODROMAL SYMPTOM PRIOR TO A FIT F?


III) HYPOREFLEXIA IS A RECOGNISED SIGN. F

IV) THE CIRCULATING BLOOD VOLUME IS DECREASED. F

V) DIAZEPAM IS THE TREATMENT OF CHOICE FOR IMMINENT ECLAMPSIA. F

Q6. OLIGOHYDRAMNIOS IS ASSOCIATED WITH:

I) POTTERS SYNDROMET

II) ANENCEPHALY F

III) RHESUS ALLOIMMUNIZATION F

IV) POST MATURITY T

V) AMNIOTIC BANDS ?

Q7. ANAPHYLAXIS

I) IS A TYPE I HYPERSENSITIVITY REACTION T

II) IS MEDIATED BY IGG F


III) HISTAMINE CAUSES BRONCHOCONSTRICTION T

IV) STEROIDS ARE USED FOR THEIR IMMEDIATE EFFECTS ON THE IMMUNE PROCESS F

V) EPINEPHRINE (ADRENALIN) CAUSES BRONCHODILATATION MEDIATED VIA ITS

BETA AGONIST EFFECTS T

Q8. REGARDING PLACENTA PREVIA

I) MOST PATIENTS PRESENT WITH VAGINAL BLEEDING T

II) THE MATENAL MORTALITY RATE IS ABOUT 3% F

III) SMOKING IS A RISK FACTOR ??

IV) IT IS MORE COMMON IN MULTIPAROUS WOMEN T

V) IMMADIATE DELIVERY IS ALWAYS INDICATED F

Q9. IN THE MANAGEMENT OF POST PARTUM HAEMORRHAGE:

I) UTERINE ATONIA IS THE COMMONEST CAUSE T

II) PATIENTS SHOULD BE SAT UPRIGHT TO AID BREATHING. F


III) UTERINE PACKING IS AN INITIAL MEASURE F

IV) PLATELETS SHOULD BE GIVEN EARLY F

V) IF DUE TO INCOMPLETE PLACENTAL EXPULSION REQUIRES URGENT

EXPLORATION OF THE UTERINE CAVITY T

Q10. AN INCREASED RISK OF ECTOPIC PREGANANCY IS SEEN WITH:

I) PREVIOUS CHLAMYDIA INFECTION?

II) PREVIOUS CANDIDA INFECTION ?

III) CONCEPTION WHILST ON COCP (COMBINED ORAL CONTRACEPTIVE PILL) T

IV) CONCEPTION WHILST ON POP (PROGESTERONE ONLY PILL) ?

V) ARTIFICIAL INSEMINATION T

Q19. REGARDING GENITOURINARY PROLAPSE:

I) ENTEROCELE DESCRIBES SMALL BOWEL HERNIATION INTO THE VAGINA. T

II) ENTEROCELE WILL INEVITABLY ACCOMPANY PROCIDENTIA ?


III) SURGICAL REPAIR IS CONTRAINDICATED IN ELDERY PATIENTS F

IV) RING PESSARIES MAY BE LEFT IN PLACE INDEFINITELY F

V) POSTNATAL PELVIC FLOOR EXERCISES HAVE A PROVEN BENEFIT IN AVOIDING

LATER PROLAPSET

Q20. A 29 YEAR OLD PRIMIGRAVID WOMAN PRESENTS TO A+E AT 17 WEEKS OF

PREGNANCY WITH A FEVER, COUGH AND BREATHLESSNESS. SHE SMOKES 5

CIGARETTES A DAY BUT DOES NOT DRINK ALCOHOL. THE FOLLOWING ARE TRUE:

I) IF CXR AND ECG ARE NORMAL, P.E (PULMONARY EMOBLISM) MAY BE EXCLUDED. F

II) A POSITIVE D-DIMERS RESULT WILL CONFIRM P.E. F

III) AN ECG SHOWING THE 'S1Q3T3' PATTERN IS PATHOGNOMONIC OF P.E IN

THIS CASE. T

IV) IF HEPARIN THERAPY IS INITIATED IT SHOULD BE CONTINUED THROUGHOUT

THE PREGNANCY. T

V) WARFARIN IS CONTRAINDICATED IN BREASTFEEDING MOTHERS. F


Q21. COMPLICATIONS IN LABOUR:

I) MECONIUM ASPIRATION IS MORE COMMON IN POST-TERM PREGNANCY T

II) FORCEPS OR VENTOUSE MAY ONLY BE USED IF THE CERVIX IS FULLY DILATED T

III) EPIDURAL ANALGESIA INCREASES THE RISK OF ASPIRATION OF GASTRIC

CONTENTS T

IV) FACE PRESENTATION HAS AN INCREASED INCIDENCE OF CORD PROLAPSE ??

V) SUSPECTED PLACENTA PRAEVIA SHOULD BE CONFIRMED BY DIGITAL EXAMINATION F

Q22. REGARDING PAIN RELIEF IN LABOUR:

I) ENTONOX IS GIVEN AT A CONCENTRATION OF 50% NITROUS OXIDE IN OXYGEN T?

II) PETHIDINE IS CONTRAINDICATED IN A MOTHER TAKING TRICYCLIC

ANTIDEPRESSANTS ?

III) PETHIDINE IS CONTAINDICATED IN PRE-ECLAMPSIAF

IV) BUPIVICAINE IS COMMONLY USED IN LABOUR EPIDURAL ANAESTHESIA ?


V) HEADACHE IS AN IMMEDIATE SIGN OF ACCIDENTAL DURAL TAP IN EPIDURALS ?

Q23. THE MENSTRUAL CYCLE

I) OVULATION COINCIDES WITH THE LH (LEUTINISING HORMONE) PEAK T

II) IN THE FOLLICULAR PHASE, RISING OESTRADIOL INITIALLY PROVIDES

NEGATIVE FEEDBACK ON FSH LEVELST

III) INCREASING PROGESTERONE LEADS TO THINNING OF CERVICAL MUCUS T

IV) A NORMAL 21 DAY PROGESTERONE IS AN INDIRECT INDICATOR OF OVULATION T

V) THE AVERAGE LENGTH OF EACH MENSTRUAL CYCLE TENDS TO DECREASE WITH

INCREASING AGE BETWEEN THE AGES 20 AND 50 ?

Q24. BREECH POSITIONS:

I) OCCUR IN LESS THAN 5% OF LABOURS?

II) MOST COMMONLY OCCUR AS THE 'EXTENDED BREECH' T


III) ARE MORE COMMON IN PRIMIPAROUS WOMENT

IV) EXTERNAL CEPHALIC VERSION IS SUCCESSFUL IN CONVERTING ON AVERAGE

< 1/3 OF BREECH PRESENTATIONS ?

V) REQUIRE DELIVERY AT 38 WEEKS T

Q25. POLYCYSTIC OVARY SYNDROME

I) IS THE COMMONEST CAUSE OF ANOVULATORY INFERTILITY T

II) REQUIRES THE PRESENCE OF OVARIAN CYSTST

III) IS ASSOCIATED WITH INSULIN RESISTANCE T

IV) MAY BE ASSOCIATED WITH INCREASED CARDIOVASCULAR MORBIDITY T

V) SERUM CONCENTRATIONS OF TESTOSTERONE ARE INCREASED T

Q26. IN THE DIAGNOSIS OF INFERTILITY:


I) AXILLARY PIGMENTATION WOULD SUPPORT A DIAGNOSIS OF PCOS T

II) A DAY 21 PROGESTERONE LEVEL OF >30 NMOL/L SUGGESTS THAT OVULATION

HAS OCCURRED T

III) DAILY TEMPERATURE CHARTING, IF FOLLOWED CORRECTLY, IS 90%

SENSITIVE FOR DETECTING IF AND WHEN OVULATION HAS OCCURED F

IV) SHEEHANS SYNDROME IS A DIFFERENTIAL DIAGNOSIS IN PATIENTS WHO

EXPERIENCE INFERTILITY DESPITE OVULATION OCCURRING F

V) WHEN COMPARED TO THE GENERAL POPULATION THERE IS AN INCREASED

PREVALENCE OF COELIAC DISEASE IN INFERTILE PATIENTS.F?

27. CERVICAL CANCER:

I) IS MOST COMMON IN WOMEN BETWEEN 20 AND 40 YEARS OF AGE. T

II) IS MORE COMMON IN WOMEN WHO HAVE HAD MANY SEXUAL PARTNERS T

III) DOES NOT OCCUR WITHOUT PRE-EXISTING GRADE III CERVICAL

INTRAEPITHELIAL NEOPLASIA (CIN) F


IV) COMMONLY PRESENTS WITH A PAINFUL ABDOMENF

V) RARELY RESPONDS TO RADIOTHERAPY TREATMENT F

GUEST, JAN 24, 2007

#6

BULIMIA

BULIMIA GUEST

:ROLL:

WHERE ARE ATOOSA & RATAN'S NAMES ????

STILL NOT FAIR

:LOL: :D :P

I HAVE TO ASK ATOOSA-- HER MCQS ARE TOO GOOD & HARD- SO ALSO SCARY!!!

I DON'T KNOW WHERE DID SHE GET THOSE ??

BULIMIA, JAN 25, 2007

#7

BULIMIA'S COLLECTION
BULIMIA'S COLLECTION GUEST

Q.1

THE DRUGS OF CHOICE FOR TREATMENT OF ERYSIPELAS IS:

A. PENICILLIN****

B. METHICILLIN

C. AMPICILLIN

D. TETRACYCLINE

E. NONE OF THE ABOVE

Q.2

WHICH OF THE FOLLOWING CRITERIA ENABLE A CLEAR DISTINCTION TO BE MADE


BETWEEN HEMORRHAGE AND THROMBOSIS IN A PATIENT WITH A CVA?

A. THE PROGRESS OF THE CLINICAL FEATURES

B. THE DEGREE OF LOSS OF CONSCIOUSNESS

C. THE ABRUPTNESS OF ONSET

D. THE PRESENCE AND ABSENCE OF HEADACHE

E. NONE OF THE ABOVE*****AMCQ

Q.3

THE NEPHRITIC SYNDROME CAN BE CAUSE BY ALL EXCEPT

A. MINIMAL CHANGE DISEASE


B. AMYLOIDOSIS****

C. MEMBRANOUS NEPHROPATHY

D. RENAL VEIN THROMBOSIS

E. DIABETIC NEPHROPATHY

Q.4

WHICH OF THE FOLLOWING STATEMENTS CONCERNING BEE STING ALLERGY IS


CORRECT?

A. IF A PATIENT HAS HAD A LIFE-THREATENING EPISODE IN THE PAST, HE OR SHE IS AT


RISK OFA FUTURE ONE.*****

B. A MILD REACTION IN THE PAST PUTS A PATIENT AT RISK OF A FUTURE LIFE


THREATING EPISODE

C. WASPS ONLY STING ONCE

D. A BEE STING IS SMALLER IN VOLUME THAN A WASP STING

E. A RAISED SERUM IGG PERSISTS FOR YRS IN THOSE AT RISK OF ANAPHYLAXIS

Q.5

ANDREW, AGED 60 YRS, COMPLAINS OF TRACES OF BLOOD IN HIS STOOL FOR THE PAST
FEW WEEKS. WHICH OF THE FOLLOWING IS THE MOST LIKELY CAUSE?

A. DIVERTICULOSIS

B. CANCER OF THE RECTUM***

C. HAEMORRHOIDS

D. ANGIODYSPLASIA

E. COLITIS
Q.6

CYANOSIS IS LEAST LIKELY TO OCCURE IN:

A. EBSTEIN€™S ANOMALY

B. TOF

C. TRICUSPID ATRESIA

D. VSD****

E. TRANSPOSITION OF THE GREAT VESSELS

Q.7

WHICH OF THE FOLLOWING PATHOGENS CAUSING GASTROINTESTINAL INFECTION IS


NOT ASSOCIATED WITH BLOODY DIARRHEA

A. LISTERIA MONOCYTOGENES

B. CAMPYLOBACTER

C. YERSINIA ENTEROCOLITICA

D. SALMONELLA****

E. SHIGELLA

Q.8

ESR IS RAISED IN ALL OF THE FOLLOWING EXCEPT

A. MULTIPLE MYELOMA

B. POLYCYTHAEMIA RUBRA VERA****

C. MACROCYTIC ANAEMIA

D. PREGNANCY
E. CHRONIC HEPATITIS

Q.9

WHICH OF THE FOLLOWING IS THE MOST COMMON PATHOLOGICAL CONDITION


ASSOCIATED WITH AORTIC ANEURYSM

A. ATHEROSCLEROSIS*****

B. SYPHILITIC AORTITIS

C. TRAUMA

D. RHEUMATIC AORTITIS

E. CYSTIC MEDIAL NECROSIS

Q.10

WHICH OF THE FOLLOWING FEATURES IS FOUND IN BERI-BERI

A. CARDIAC FAILURE, GLOSSITIS, DERMATITIS

B. CARDIAC FAILURE, NEUROPATHY, DERMATITIS

C. GLOSSITIS, DERMATITIS, MEMORY LOSS

D. GLOSSITIS, NEUROPATHY, MEMORY LOSS

E. CARDIAC FAILURE, NEUROPATHY, MEMORY LOSS*****

BULIMIA'S COLLECTION, JAN 26, 2007

#8

BULIMIA

BULIMIA GUEST
THAT WAS RATAN'S COLLECTION-- MR PHOTOCOPY MACHINE

HE HAD TO TYPE IT & HELPED TO U

BULIMIA, JAN 26, 2007

#9

GUEST

GUEST GUEST

1. A CHILD IS BROUGHT TO YOU WITH A FEVER. ON EXAMINATION YOU FIND NECK


RIGIDITY, PROTEIN IS 1.2, SUGAR IS NORMAL AND LYMPHOCYTE IS 50X10. WHAT IS YOUR
PROBABLE DIAGNOSIS?

A) TUBERCULOSIS

B) ECHO VIRUS

C) BACTERIAL MENINGITIS *******

D) TYPHOID

E) RHINO VIRUS

2. A PATIENT AFTER A MOTOR TRAFFIC ACCIDENT WITH A PELVIC FRACTURE. FIVE DAYS
AFTER SUCCESSFUL RESUSCITATION, HE DEVELOPS SEVERE DYSPNOEA. WHAT IS YOUR
INITIAL MANAGEMENT?

A) OXYGEN INHALATION

B) I/V HEPARIN

C) INTUBATION AND VENTILATION *********


D) INTENSIVE ANTIBIOTIC THERAPY.

3. A PATIENT WITH COPD CAME INTO THE EMERGENCY DEPARTMENT WITH SEVERE
DYSPNOEA, 28% O2 8L GIVEN BY MASK. AFTER ONE HOUR, YOU DID ARTERIAL BLOOD
GASES AND FOUND THAT PO2 LEVEL WAS 42MM OF HG, PCO2 WAS 68MM OF HG. WHAT
WILL YOU DO NEXT?

A) INCREASE O2 INHALATION BY MASK

B) ASK THE PATIENT FOR INTUBATION AND VENTILATION *****

C) INCREASE I/V FLUID

A) REPEAT ARTERIAL BLOOD GAS LEVEL AFTER ONE HOUR.

4. YOU WILL GET ERYTHEMA NOSODUM IN ALL THE FOLLOWING CONDITIONS, EXCEPT:

A) SARCOIDOSIS

B) BACTERIAL MYOCARDIAL INFARCTION *****

C) TUBERCULOSIS

D) YERSINIA

E) RHEUMATIC FEVER.

5. HOW DOES MICROGYNON 30 WORK?

A) IN THE HYPOTHALAMUS

B) IN THE PITUITARY GLAND

C) IN THE OVARIES ****

D) IN THE UTERUS
E) ON BOTH OVARIES AND UTERUS.

6. A PRIMIGRAVIDA WOMAN€™S HEIGHT IS 150CM. AT 38 WEEKS GESTATION SHE IS


IN LABOUR. ON PELVIC EXAMINATION, YOU THINK THAT THE PELVIS IS APPARENTLY
REDUCED BUT THE CERVIX IS FULLY EFFACED AND 4CM DILATED. THE BABY IS IN THE
LONGITUDINAL POSITION AND HEAD IS AT 0 STATION. WHAT IS YOUR MANAGEMENT
PLAN:

A) TRIAL OF LABOUR

B) X-RAY PELVIMETRY

C) START I/V OXYTOCIN

D) IMMEDIATE CAESAREAN SECTION ***8

E) WAIT AND SEE.

7. WITH A DIABETIC ULCER, WHAT IS YOUR APPROPRIATE RX:

A) AMOXICILLIN + PENICILLIN

B) AMPICILLIN + GENTAMYCIN + METRONIDAZOL.

C) FLUCLOXACILLIN + GENTAMYCIN *****

D) AMPICILLIN + CLOXACILLIN.

8. AFTER A DIFFICULT CATHETERISATION, BLOOD PRESSURE SUDDENLY DECREASES.


WHAT WOULD BE YOUR DIAGNOSIS?

A) INTERNAL HAEMORRHAGE ****

B) SEPTICAEMIA
C) PYLONEPHRITIS.

9. AFTER THE FIRST DELIVERY IN A TWIN PREGNANCY, YOU WILL DO ALL OF THE
FOLLOWING IN THE SECOND DELIVERY, EXCEPT:

A) EXTERNAL PODALIC VERSION *****

B) CHECK THE FETAL LIE

C) CHECK THE BLEEDING

D) CHECK THE FETAL HEART SOUND

E) CHECK THE VAGINAL LACERATION/S.

10. 23 YEAR-OLD GIRL HAS ABDOMINAL COLIC, 2 WEEKS BACK SHE HAD VIRAL
INFECTION, PURPURAL RASH. MOST PROBABLE DIAGNOSIS IS:

A) ITP

B) ISCHAEMIC COLITIS

C) HENOCH-SCHONLEIN PURPURA ****

D) GLANDULAR FEVER

11. WHAT DIFFERENTIATES BETWEEN HAEMOLYTIC AND IRON DEFICIENCY ANAEMIA?

A) POLYCYTHAEMIA

B) POLYCHROMASIA ****

C) NORMAL PLATELET COUNT


12. A 50 YEAR-OLD WOMAN CAME TO YOU WITH COMPLAINT OF SLIGHT VAGINAL
BLEEDING. SHE ALSO COMPLAINS THAT SHE HAS AMOENORRHOEA FOR 11 MONTHS.
WHAT IS YOUR DIAGNOSIS?

A) ENDOMETRIAL CARCINOMA

B) CA. CERVIX

C) OESTROGENIC STIMULATION OF THE OVARY

D) VAGINAL ATROPHY

E) CERVICAL POLYP

13. 30 YEAR OLD CYSTIC FIBROSIS WITH CHEST PAIN TREATED WITH NEBULISED
TOBRAMYCIN TH./ DM , RECENT WEIGHT LOSS, STEATHHORREA. CORRECT
MANAGEMENT?

A) DECREASED CARBOHYDRATED DIET AND STABILISING GLYCAEMIC CONTRO

B) PANCREATIC ENZYME

C) CONTINUE NEBULISED TOBRAMYCIN FOR ANOTHER 3 WEEKS

D) CHEST PHYSIOTHERAPY

WHAT IS THE QUESTION

14. 37 YEAR-OLD MAN WITH DEPRESSION PRESENTS TO EMERGENCY, DROWSY AND


AGGRESSIVE. HE HAS TAKEN A BOTTLE OF DOXEPIN AND SCOTCH TOGETHER. IMPORTANT
MANAGEMENT INCLUDE:

A) ADMIN FOR ECG MONITORING FOR 4 HOURS

B) SERIAL DOSES OF CHARCOAL ORALLY


C) DO SERUM PARACETAMOL AS THE ONLY DRUG SCREEN

D) AS THE PATIENT IS CONSCIOUS SIGNIFICANT T.C.A. OVERDOSE CAN BE RULED OUT

E) ALKALISATION OF URINE WILL AID EXCRETION OF DOXEPINE, AND WILL PROTECT


AGAINST ARRHYTHMIA ***

15. A FIFTEEN YEAR-OLD GIRL PRESENTS WITH FEVER AND PAINFUL LEFT FOREARM
TWO DAYS AFTER HER CAT HAD CLAWED THE ARM. THERE IS REGIONAL LYMPH
ADENOPATHY PRESENT AND TEMPERATURE IS 39OC. WHICH OF THE FOLLOWING
STATEMENT IS TRUE?

A) THE IMPLICATED BACTERIUM E.COLI

B) MOST SUFFER OF CAT FEVER ARE >60

C) COMPLICATION MAY INCLUDE INVOLVEMENT LIVER, SPLEEN, CENTRAL NERVOUS


SYSTEM, BOWEL AND SKIN *****

D) AMPICILLIN AND CEPHALOSPORIN ARE INADEQUATE THERAPY

E) LYMPADENOPATHY IN THE FIELD OF DRAINAGE IS INVARIABLE

16. LOUIS, A 5 YEAR-OLD BOY IS BROUGHT TO YOU FOR TESTING HIS FREE LEAD LEVEL,
WHICH IS SIGNIFICANTLY ELEVATED. HIS PHYSICAL AND LOCOMOTOR EXAMINATION IS
COMPLETELY NORMAL. ON QUESTIONING, THE PARENTS STATE THAT THEY HAVE
RECENTLY MOVED TO AN OLD TERRACE. THE INITIAL MOST APPROPRIATE MANAGEMENT
WOULD BE TO?

A) COMMENCE DECONTAMINATION OF THE HOUSE

B) COMMENCE A CHELATING AGENT ******

C) COMMENCE D-PENICILLAMINE FOR 2-6 MONTHS

D) CHECK THE FREE ERTHROCYTE PROTOPORPHYRIN


E) CHECK THE URINARY EXCRETION OF LEAD AFTER ADMINISTRATION OF CALCIUM
EDTA

17. ALF, A 74 YEAR-OLD MAN RECENTLY ADMITTED FOR MANAGEMENT OF


RESPIRATORY FAILURE ASKS ABOUT HOME-OXYGEN. THE FOLLOWING ARE CRITERIA FOR
LONG-TERM CONTINUOUS OXYGEN THERAPY EXCEPT:

A) PAO2 <55MMHG

B) O2 SATURATION <88%

C) TREATING THE HYPOXIA ASSOCIATED WITH THE SLEEP APNOEA *****

D) IF THE PATIENT HAS MENTAL IMPAIRMENT OXYGEN SHOULD BE WITHHELD FOR


SAFETY REASONS

E) CONGESTIVE HEART FAILURE WITH INTRACTABLE ANGINA

18. ALL OF THE FOLLOWING STATEMENTS ABOUT INFLAMMATORY BOWEL DISEASE ARE
TRUE EXCEPT:

A) PSEUDOPOLYPS DEVELOPING IN ULCERATIVE COLITIS DO NOT BECOME MALIGNANT

B) DEVELOPMENT OF FISTULA BETWEEN BOWEL AND SKIN OR BLADDER SUGGESTS


GRANULOMATOUS BOWEL DISEASE, SUCH AS A REGIONAL ENTERITIS *****8

C) SEVERE HYPOTENSION WITH FEVER, LEUCOCYTOCIS AND ABDOMINAL DISTENTION


SUGGESTS DEVELOPMENT OF TOXIC MEGACOLON WHICH USUALLY REQUIRES
IMMEDIATE SURGERY

D) OCCULT BLEEDING OCCURS IN BOTH ULCERATIVE COLITIS AND REGIONAL ENTERITIS


BUT GROSS BLEEDING IS MORE CHARACTERISTIC OF ULCERATIVE COLITIS

E) METRONIDAZOLE MAY HELP TO CONTROL ACUTE DIARRHOEA IN CROHN€™S


DISEASE
19. JOSE, 6 MONTHS-OLD BOY IS REFERRED BACK TO YOU BY THE LOCAL HOSPITAL
WHERE HE HAS PRESENTED SEVERAL TIMES OVER THREE MONTHS DUE TO A SERIES OF
FALLS AT HOME. THE QUESTION OF NON-ACCIDENTAL INJURY (CHILD ABUSE) HAS BEEN
RAISED. YOUR INITIAL ASSESSMENT WOULD INCLUDE:

A) DEVELOPMENTAL MILESTONE ASSESSMENT

B) A SKELETAL SURVEY WITH RADIOGRAPH OF AP CXR, AP PELVIS AP VIEWS OF


EXTREMITIES, AP LATERAL VIEWS OF SKULL *****

C) CONFRONTATION OF

D) PROMPT REFERRAL TO A PAEDIATRICIAN

E) FUNDUSCOPY NONE OTHER SEEMS RITE

GUEST, JAN 27, 2007

#10

GUEST

GUEST GUEST

1. A CHILD IS BROUGHT TO YOU WITH A FEVER. ON EXAMINATION YOU FIND NECK


RIGIDITY, PROTEIN IS 1.2, SUGAR IS NORMAL AND LYMPHOCYTE IS 50X10. WHAT IS YOUR
PROBABLE DIAGNOSIS?

A) TUBERCULOSIS

B) ECHO VIRUS

C) BACTERIAL MENINGITIS *******

D) TYPHOID

E) RHINO VIRUS
2. A PATIENT AFTER A MOTOR TRAFFIC ACCIDENT WITH A PELVIC FRACTURE. FIVE DAYS
AFTER SUCCESSFUL RESUSCITATION, HE DEVELOPS SEVERE DYSPNOEA. WHAT IS YOUR
INITIAL MANAGEMENT?

A) OXYGEN INHALATION

B) I/V HEPARIN

C) INTUBATION AND VENTILATION *********

D) INTENSIVE ANTIBIOTIC THERAPY.

3. A PATIENT WITH COPD CAME INTO THE EMERGENCY DEPARTMENT WITH SEVERE
DYSPNOEA, 28% O2 8L GIVEN BY MASK. AFTER ONE HOUR, YOU DID ARTERIAL BLOOD
GASES AND FOUND THAT PO2 LEVEL WAS 42MM OF HG, PCO2 WAS 68MM OF HG. WHAT
WILL YOU DO NEXT?

A) INCREASE O2 INHALATION BY MASK

B) ASK THE PATIENT FOR INTUBATION AND VENTILATION *****

C) INCREASE I/V FLUID

A) REPEAT ARTERIAL BLOOD GAS LEVEL AFTER ONE HOUR.

4. YOU WILL GET ERYTHEMA NOSODUM IN ALL THE FOLLOWING CONDITIONS, EXCEPT:

A) SARCOIDOSIS

B) BACTERIAL MYOCARDIAL INFARCTION *****

C) TUBERCULOSIS

D) YERSINIA

E) RHEUMATIC FEVER.
5. HOW DOES MICROGYNON 30 WORK?

A) IN THE HYPOTHALAMUS

B) IN THE PITUITARY GLAND

C) IN THE OVARIES ****

D) IN THE UTERUS

E) ON BOTH OVARIES AND UTERUS.

6. A PRIMIGRAVIDA WOMAN€™S HEIGHT IS 150CM. AT 38 WEEKS GESTATION SHE IS


IN LABOUR. ON PELVIC EXAMINATION, YOU THINK THAT THE PELVIS IS APPARENTLY
REDUCED BUT THE CERVIX IS FULLY EFFACED AND 4CM DILATED. THE BABY IS IN THE
LONGITUDINAL POSITION AND HEAD IS AT 0 STATION. WHAT IS YOUR MANAGEMENT
PLAN:

A) TRIAL OF LABOUR

B) X-RAY PELVIMETRY

C) START I/V OXYTOCIN

D) IMMEDIATE CAESAREAN SECTION ***8

E) WAIT AND SEE.

7. WITH A DIABETIC ULCER, WHAT IS YOUR APPROPRIATE RX:

A) AMOXICILLIN + PENICILLIN

B) AMPICILLIN + GENTAMYCIN + METRONIDAZOL.

C) FLUCLOXACILLIN + GENTAMYCIN *****

D) AMPICILLIN + CLOXACILLIN.
8. AFTER A DIFFICULT CATHETERISATION, BLOOD PRESSURE SUDDENLY DECREASES.
WHAT WOULD BE YOUR DIAGNOSIS?

A) INTERNAL HAEMORRHAGE ****

B) SEPTICAEMIA

C) PYLONEPHRITIS.

9. AFTER THE FIRST DELIVERY IN A TWIN PREGNANCY, YOU WILL DO ALL OF THE
FOLLOWING IN THE SECOND DELIVERY, EXCEPT:

A) EXTERNAL PODALIC VERSION *****

B) CHECK THE FETAL LIE

C) CHECK THE BLEEDING

D) CHECK THE FETAL HEART SOUND

E) CHECK THE VAGINAL LACERATION/S.

10. 23 YEAR-OLD GIRL HAS ABDOMINAL COLIC, 2 WEEKS BACK SHE HAD VIRAL
INFECTION, PURPURAL RASH. MOST PROBABLE DIAGNOSIS IS:

A) ITP

B) ISCHAEMIC COLITIS

C) HENOCH-SCHONLEIN PURPURA ****

D) GLANDULAR FEVER
11. WHAT DIFFERENTIATES BETWEEN HAEMOLYTIC AND IRON DEFICIENCY ANAEMIA?

A) POLYCYTHAEMIA

B) POLYCHROMASIA ****

C) NORMAL PLATELET COUNT

12. A 50 YEAR-OLD WOMAN CAME TO YOU WITH COMPLAINT OF SLIGHT VAGINAL


BLEEDING. SHE ALSO COMPLAINS THAT SHE HAS AMOENORRHOEA FOR 11 MONTHS.
WHAT IS YOUR DIAGNOSIS?

A) ENDOMETRIAL CARCINOMA

B) CA. CERVIX

C) OESTROGENIC STIMULATION OF THE OVARY

D) VAGINAL ATROPHY

E) CERVICAL POLYP

13. 30 YEAR OLD CYSTIC FIBROSIS WITH CHEST PAIN TREATED WITH NEBULISED
TOBRAMYCIN TH./ DM , RECENT WEIGHT LOSS, STEATHHORREA. CORRECT
MANAGEMENT?

A) DECREASED CARBOHYDRATED DIET AND STABILISING GLYCAEMIC CONTRO

B) PANCREATIC ENZYME

C) CONTINUE NEBULISED TOBRAMYCIN FOR ANOTHER 3 WEEKS

D) CHEST PHYSIOTHERAPY

WHAT IS THE QUESTION


14. 37 YEAR-OLD MAN WITH DEPRESSION PRESENTS TO EMERGENCY, DROWSY AND
AGGRESSIVE. HE HAS TAKEN A BOTTLE OF DOXEPIN AND SCOTCH TOGETHER. IMPORTANT
MANAGEMENT INCLUDE:

A) ADMIN FOR ECG MONITORING FOR 4 HOURS

B) SERIAL DOSES OF CHARCOAL ORALLY

C) DO SERUM PARACETAMOL AS THE ONLY DRUG SCREEN

D) AS THE PATIENT IS CONSCIOUS SIGNIFICANT T.C.A. OVERDOSE CAN BE RULED OUT

E) ALKALISATION OF URINE WILL AID EXCRETION OF DOXEPINE, AND WILL PROTECT


AGAINST ARRHYTHMIA ***

15. A FIFTEEN YEAR-OLD GIRL PRESENTS WITH FEVER AND PAINFUL LEFT FOREARM
TWO DAYS AFTER HER CAT HAD CLAWED THE ARM. THERE IS REGIONAL LYMPH
ADENOPATHY PRESENT AND TEMPERATURE IS 39OC. WHICH OF THE FOLLOWING
STATEMENT IS TRUE?

A) THE IMPLICATED BACTERIUM E.COLI

B) MOST SUFFER OF CAT FEVER ARE >60

C) COMPLICATION MAY INCLUDE INVOLVEMENT LIVER, SPLEEN, CENTRAL NERVOUS


SYSTEM, BOWEL AND SKIN *****

D) AMPICILLIN AND CEPHALOSPORIN ARE INADEQUATE THERAPY

E) LYMPADENOPATHY IN THE FIELD OF DRAINAGE IS INVARIABLE

16. LOUIS, A 5 YEAR-OLD BOY IS BROUGHT TO YOU FOR TESTING HIS FREE LEAD LEVEL,
WHICH IS SIGNIFICANTLY ELEVATED. HIS PHYSICAL AND LOCOMOTOR EXAMINATION IS
COMPLETELY NORMAL. ON QUESTIONING, THE PARENTS STATE THAT THEY HAVE
RECENTLY MOVED TO AN OLD TERRACE. THE INITIAL MOST APPROPRIATE MANAGEMENT
WOULD BE TO?

A) COMMENCE DECONTAMINATION OF THE HOUSE

B) COMMENCE A CHELATING AGENT ******

C) COMMENCE D-PENICILLAMINE FOR 2-6 MONTHS

D) CHECK THE FREE ERTHROCYTE PROTOPORPHYRIN

E) CHECK THE URINARY EXCRETION OF LEAD AFTER ADMINISTRATION OF CALCIUM


EDTA

17. ALF, A 74 YEAR-OLD MAN RECENTLY ADMITTED FOR MANAGEMENT OF


RESPIRATORY FAILURE ASKS ABOUT HOME-OXYGEN. THE FOLLOWING ARE CRITERIA FOR
LONG-TERM CONTINUOUS OXYGEN THERAPY EXCEPT:

A) PAO2 <55MMHG

B) O2 SATURATION <88%

C) TREATING THE HYPOXIA ASSOCIATED WITH THE SLEEP APNOEA *****

D) IF THE PATIENT HAS MENTAL IMPAIRMENT OXYGEN SHOULD BE WITHHELD FOR


SAFETY REASONS

E) CONGESTIVE HEART FAILURE WITH INTRACTABLE ANGINA

18. ALL OF THE FOLLOWING STATEMENTS ABOUT INFLAMMATORY BOWEL DISEASE ARE
TRUE EXCEPT:

A) PSEUDOPOLYPS DEVELOPING IN ULCERATIVE COLITIS DO NOT BECOME MALIGNANT

B) DEVELOPMENT OF FISTULA BETWEEN BOWEL AND SKIN OR BLADDER SUGGESTS


GRANULOMATOUS BOWEL DISEASE, SUCH AS A REGIONAL ENTERITIS *****8

C) SEVERE HYPOTENSION WITH FEVER, LEUCOCYTOCIS AND ABDOMINAL DISTENTION


SUGGESTS DEVELOPMENT OF TOXIC MEGACOLON WHICH USUALLY REQUIRES
IMMEDIATE SURGERY
D) OCCULT BLEEDING OCCURS IN BOTH ULCERATIVE COLITIS AND REGIONAL ENTERITIS
BUT GROSS BLEEDING IS MORE CHARACTERISTIC OF ULCERATIVE COLITIS

E) METRONIDAZOLE MAY HELP TO CONTROL ACUTE DIARRHOEA IN CROHN€™S


DISEASE

19. JOSE, 6 MONTHS-OLD BOY IS REFERRED BACK TO YOU BY THE LOCAL HOSPITAL
WHERE HE HAS PRESENTED SEVERAL TIMES OVER THREE MONTHS DUE TO A SERIES OF
FALLS AT HOME. THE QUESTION OF NON-ACCIDENTAL INJURY (CHILD ABUSE) HAS BEEN
RAISED. YOUR INITIAL ASSESSMENT WOULD INCLUDE:

A) DEVELOPMENTAL MILESTONE ASSESSMENT

B) A SKELETAL SURVEY WITH RADIOGRAPH OF AP CXR, AP PELVIS AP VIEWS OF


EXTREMITIES, AP LATERAL VIEWS OF SKULL *****

C) CONFRONTATION OF

D) PROMPT REFERRAL TO A PAEDIATRICIAN

E) FUNDUSCOPY NONE OTHER SEEMS RITE

GUEST, JAN 27, 2007

#11

GUEST

GUEST GUEST

Q.1

ALL OF THE FOLLOWING TESTS SHOULD BE A PART OF 1ST ANTENATAL SCREEN EXCEPT:

A. FULL BLOOD EXAMINATION

B. BLOOD GROUP AND ANTIBODIES


C. DATING USG

D. SYPHILIS SEROLOGY

E. HEPATITIS B SURFACE ANTIGEN

Q.2

EVE, 9WKS PREGNANT, PRESENTS WITH A H/O 24 HRS OF INTERMITTENT DARK BLOOD
LOSS STAINING HER UNDERWEAR. ON P/V EXAM. UTERUS IS THE EXPECTED SIZE AND CX
OS IS CLOSED AND NON TENDER. YOU SHOULD ADVICE HER THAT :

A. SHE IS LIKELY TO HAVE MISCARRIED

B. B. APPROX. HALF OF ALL PREGNANT WOMEN SUFFER FROM BLEEDING DURING 1ST
TRIMESTER

C. SHE IS REQUIRES ADMISSION TO HOSPITAL FOR D & C

D. SHE SHOULD HAVE AN USG EXAM.

E. THERE IS A HIGHER INCIDENCE OF CONGENITAL MALFORMATION DURING BLEEDING


OCCURRED IN 1ST TRIMESTER.

Q.3

WHICH OF THE FOLLOWING ARE NOT A RISK FACTOR FOR PRIMARY OPEN ANGLE
GLAUCOMA?

A. DM

B. HYPERMETROPIA

C. STEROID THERAPY

D. POSITIVE FAMILY HISTORY

E. AGE GREATER THAN 40 YRS

Q.4
WEBER€™S TEST IS USED IN THE CLINICAL ASSESSMENT OF HEARING . WHICH OF THE
FOLLOWING STATEMENTS ABOUT WEBER€™S TEST IS CORRECT

A. IT IS OFTEN UNRELIABLE

B. IT IS PERFORMED WITH 128 HZ TUNING FORK

C. IN UNILATERAL CONDUCTIVE DEAFNESS, SOUND IS LOUDEST IN THE NORMAL EAR

D. IN UNILATERAL SENSORINEURAL DEAFNESS , SOUND IS REFERRED TO THE EFFECTED


EAR

E. EAR WAX MUST BE ABSENT FOR THE TEST TO BE ACCURATE

Q.5

14 YRS OLD BOY PRESENTS WITH LONG H/O RTI AND RECURRENT VIRAL
GASTROENTERITIS. BUT HIS BACTERIAL RESP.INFECTIONS RESPOND TO ANTIMICROBIAL
THERAPY. HIS ILLNESS RECURE QUICKLY. OTHER THAN SUFFERING RECURRENT ALLERGIC
RHINITIS AND VERNAL CONJUNCTIVITIS, HE IS WELL. THE MOST LIKELY CAUSE OF HIS
IMMUNOGLOBIN DEFICIENCY IS

A. IGA

B. IGG

C.IGG2

D. IGE

E. IGM

Q.6

OLD MAN WITH HEART FAILURE SIGNS, DYSPNEA, MILD ICTERUS, LOWER LIMB
WEAKNESS, SMOOTH HEPATOMEGALY

A. BUD CHARI SYND

B. CIRRHOSIS

C. VENO OCCLUSIVE DIS.


D. IVC THROMBOSIS WITH PUL. EMBOLISM

Q.7

REGARDING PSYCHIATRIC SIDE EFFECTS OF STEROIDS, WHICH IS CORRECT

A. MANIA IS MORE COMMON WHEN SYMP. ARE SEVERE

B. DEPRESSION IS COMMON WHEN SYMP. NOT SEVERE

C. DELIRIUM AFTER WITHDRAWAL

Q.8

LADY WITH IVF AT 38 WKS CAN€™T FEEL FOETAL MOVEMENTS FOR PAST 24 HRS, CTG
ARE NORMAL€”NEXT STEP

A. INDUCTION OF LABOUR

B. OXYTOCIN CHALLENGE TEST

C. CHECK USG

D. AMINOCENTESIS

Q.9

ABSOLUTE CONTRAINDICATION IN ECT

A. HEART BLOCK

B. MI 2 WKS AGO

C. ITRA CRANIAL LESION

D. RAISE ICP

E. H/O GRANDMAL SEIZURES

Q.10
IN UTERINE CANCER ALL OF THE FOLLOWING IS ARISK FACTOR EXCEPT

A. H/O SMOKING

B. UNOPPOSED ESTROGEN THERAPY

C. MULTIPARITY

D. NONE OF THE ABOVE

GUEST, JAN 27, 2007

#12

DR. HONEY

DR. HONEY GUEST

.1. IN A PATIENT WITH MYOCARDIAL INFARCTION WAS FOUND A NEW SYSTOLIC


MURMUR ON EXAMINATION. CARDIAC EJECTION FRACTION WAS 55%. WHICH OF THE
FOLLOWING IS MOST PROBABLE CAUSE:

A. AORTIC REGURGITATION

B. PAPILLARY MUSCLE DYSFUNCTION

C. MITRAL VALVE STENOSIS

D. PAPILLARY MUSCLE RUPTURE ***

E. TRICUSPID VALVE REGURGITATION

2.AN OBESE PATIENT WITH DIABETES MELLITUS IS UNDER ANTI-HYPERTENSION


TREATMENT. HIS BLOOD PRESSURE IS 160/100MMBG ON EXAMINATION. WHICH OF THE
FOLLOWING IS YOUR INITIAL CONSIDERATION FOR THIS PATIENT:

A. DECREASED PROTEIN IN HIS DIET

B. CONCURRENT HYPERTENSIVE THERAPY


C. GIVE DIURETICS

D. CONTROL SUGAR INTAKE IN THE DIET

E. IDEAL WEIGHT ****

3. AT WHICH LEVEL OF CHOLESTERAL YOU CONSIDER TO GIVE LIPID-LOWERING STATINS


(EG, SIMVASTATIN, PRAVASTATIN)

A. 6 MMOL/L

B. 5.5 MMOL/L

C. 5 MMOL/L

D. 4.5 MMOL/L ****JM

E. 4 MMOL/L

4. WHICH OF THE FOLLOWING IS THE *MOST COMMON CHARACTERISTIC OF PLEURA


EFFUSION OF TB:

A. GLUCOSE DECREASED OR ABSENT

B. MONOCYTE ---MAY BE LYMPHOCYTE

C. BLOOD STAINED

D. PROTEIN <2G

E. FIND TB BACILLUS

5. WHICH FOLLOWING GROUP IS THE MOST AT RISK OF HIV INFECTION:


A. HETEROSEXUAL

B. HOMOSEXUAL

C. INTRAVENOUS DRUG USER ***

D. BLOOD TRANSFUSION

E. HAEMOPHILIAS

6. WHICH OF THE FOLLOWING GROUP IS LEAST LIKE OF INFECTION OF HIV:

A. HETEROSEXUAL

B. BLOOD TRANSFUSION ***DUE TO SCREENING

C. HOMOSEXUAL

D. HAEMODIALYSIS

E. HAEMOPHILIACS

7. DENGUE FEVER, ALL FOLLOWINGS ARE CORRECT EXCEPT:

A. ARBOVIRUS

B. MOSQUITO TRANSMISSION

C. CHILDREN GET LEAST SEVERE ILLNESS

D. THERE IS NO SPECIFIC TREATMENT

E. AIR DROPLET INFECTION ***


8. A PATIENT WITH ???MYCOBACTERIA INFECTION WHICH OF THE FOLLOWING IS MOST
APPROPRIATE TREATMENT

A. COTRIMOXAZOLE

B. TETRACYCLINE

C. AMOXYCILLINE

D. METRONIDAZOLE

E. ERITHROMYCIN

?? WHICH TYPE OF BACTERIA/ TB/ PNEUMONIAE

9. WHAT IS COMPATIBLE WITH CRITICAL ILLNESS:

A. INCREASED CORTISOL , INCREASED TSH

B. BOTH CORTISOL AND TSH DECREASED

C. INCREASED CORTISOL, DECREASED TSH***

D. DECREASED CORTISOL, INCREASED TSH

E. NORMAL CORTISOL, INCREASED TSH

10.FOR AN ELDERLY MAN, WHICH ABOVE FOLLOWING BLOOD SUGAR LEVEL NEED
FURTHER INVESTIGATION

A. 5 MMOL/L

B. 5.5 MMOL/L

C. 6 MMOL/L

D. 6.5 MMOL/L
E. 7 MMOL/L ***

11. A YOUNG PATIENT COMES FROM OVERSEAS WITH DIARRHOEA, NO BLOOD.


TEMPERATURE 37.9C, STOOL EXAMINATION SHOWED FEW SALMONELLAS. WHAT IS YOUR
MANAGEMENT:

A. OBSERVATION AND REPEAT STOOL EXAMINATION 3 DAYS LATER **?

B. BROAD SPECTRUM ANTIBIOTIC LIKE AMOXICILLIN

C. TRIMETHOPRINE PLUS SULPHASALAZINE

D. COTRIMOXAZOLE PLUS TRIMETHOPRIM

E. REASSURE

12.. PATIENT WITH SUPPOSED HEPATOMA. WHICH OF THE FOLLOWING QUESTIONS IS


MOST HELPFUL FOR DIAGNOSIS:

A. PRESENT LIVER CIRRHOSIS ***

B. ALCOHOL LIVER DISEASE

C. ACUTE HEPATITIS

D. CHOLANGITIS

E. FAMILY HISTORY OF LIVER HEPATOMA

13. IN ANAEMIA PATIENT WITH INCREASED TRANSFERRIN. ALL OF THE FOLLOWING IS


CORRECT EXCEPT:

A. INCREASED SERUM FERRITIN***


B. DECREASED SERUM FERRITIN

C. INCREASED TOTAL IRON BINDING CAPACITY

D. INCREASED TRANSFERRIN

E. DECREASED SERUM IRON

14. IN HAEMOLYTIC ANAEMIA, ALL IS CORRECT EXCEPT:

A. INCREASED UROBILINOGEN

B. SERUM UROBILIN

C. ERYTHROCYTE COUNT***

D. DECREASED MCV

E. INCREASED SERUM IRON

15. A 67 YEAR-OLD ALCOHOLIC MAN HAS BACKACHE, ESR 120MM/H. WHICH OF THE
FOLLOWING IS MOST LIKELY DIAGNOSIS:

A. MULTIPLE SCLEROSIS

B. ACUTE LYMPHATIC LEUKAEMIA

C. HODGKIN DISEASE

D. NON-HODGKIN DISEASE

E. MULTIPLE MYELOMA***

16. A 67 YEAR-OLD PATIENT WITH LONG STANDING ALCOHOLISM. LAB: HB 9.8; ***ESR
120: LEUKOCYTES 10000 AND LEFT SHIFT. WHICH OF THE FOLLOWING IS DIAGNOSIS:
A. VIRAL HEPATITIS

B. MULTIPLE MYELOMA ***

C. IRON DEFICIENCY ANAEMIA

D. FATTY LIVER

HIGH ESR> GAINT CELL ARTERITIS, POLYMYALGIA RHEUMATICA, TEMPORAL ARTERITIS,


MULTIPLE MYELOMA, TUBERCULOSIS, DEEP ABSCESS, BACTERIAL ENDOCARDITIS, ACUTE
OSTEOMYELITIS

17. A 28 YEAR-OLD FEMALE WITH LONG STANDING ALCOHOLISM, HISTORY OF BOWEL


IRRITATION. LAB: HB 9.8; ***MCV 110. WHICH IS THE MOST LIKELY DIAGNOSIS:

A. WHIPPLE€™S DISEASE

B. COLORECTAL CARCINOMA

C. IRRITATIVE BOWEL SYNDROME

D. ULCERATIVE COLITIS

E. COELIAC DISEASE ***

18. ALL FOLLOWING SIGNS ARE PRESENT IN PARKINSON DISEASE EXCEPT:

A. LOSS OF POSTURAL REFLEXES

B. MYOCLONUS

C. DECREASED REFLEXES *

D. MUSCLE WEAKNESS

E. BROAD WIDE GAIT ***


19. A 35 YEAR-OLD FEMALE WITH FACIAL PAIN, ASSOCIATED SENSORY DELICATE, NO
TRIGGER ZONE. WHAT IS THE DIAGNOSIS:

A. TRIGEMINAL NEURALGIA

B. RAMSAY €“ HUNT SYNDROME

C. BELL€™S PALSY

D. MULTIPLE SCLEROSIS ***

E. TOLOSA €“ HUNT SYNDROME

20. THESE ARE ALL PHENYTOIN SIDE EFFECTS EXCEPT:

A. NYSTAGMUS

B. OSTEOMALACIA

C. STIMULATE OSTEOPOROSIS

D. LYMPHOMA

E. DEAFNESS ***

21. PATIENT 64 YEAR-OLD, SUDDENLY OCCURRED HEMIPARESIS AND PARALYSIS OF


SOFT PALATE AND HORNER€™S SYNDROME ON THE SAME SIDE OF THE BODY. WHERE IS
THE LESION:

A. MIDDLE CEREBRAL ARTERY

B. CAROTID ARTERY OCCLUSION


C. VERTEBRO-BASILAR ARTERY ***

D. GIANT CELL ARTERITIS

E. SUBCLAVIAN STEAL SYNDROME

22. PATIENT 64 YEAR-OLD, SUDDENLY OCCURRED HEMIPARESIS AND PARALYSIS OF


SOFT PALATE AND HORNER€™S SYNDROME ON THE SAME SIDE OF THE BODY. WHERE IS
THE LESION:

A. MIDDLE CEREBRAL ARTERY

B. CAROTID ARTERY OCCLUSION

C. VERTEBRO-BASILAR ARTERY ***

D. GIANT CELL ARTERITIS

E. SUBCLAVIAN STEAL SYNDROME

23. A 55 YEAR-OLD PATIENT HAS BLOOD PRESSURE 200/130MMHG. HIS LEFT KIDNEY IS
SMALL AND URINE TESTS AND KIDNEY FUNCTIONS ARE NORMAL. WHAT IS YOUR NEXT
INVESTIGATION:

A. INTRAVENOUS PYELOGRAPHY

B. RETROGRADE

C. RENAL CT SCAN

D. RENAL SCAN---ISOTOPE SCAN ***

DX RAS> DEFINITIVE IVX-- RENAL ARTERIOGRAPHY


24. PATIENT HAS SUDDEN DYSPNEA. PHYSICAL EXAMINATION FOUND LEFT UPPER
CHEST RESPIRATORY MOVEMENTS INCREASED. IN LEFT LOWER PART PERCUSSION
DULLNESS AND DECREASED RESPIRATORY SOUNDS. WHAT IS THE DIAGNOSIS:

A. ATELECTASIS

B. PNEUMONIA

C. PLEURAL EFFUSION***

D. PULMONARY THROMBOSIS

E. PNEUMOTHORAX

25. POST-OPERATIVE THIRD DAY SERUM NA 165MMOL/L AND URINE OSMO IS 250
(NORMAL 50 €“ 1000). THE MOST LIKELY CAUSE IS:

A. THIAZIDE DIURETIC DOSE IS TOO HIGH

B. DIABETES MELLITUS

C. DIABETES INSIPIDUS ***

D. SIADH

E. ACUTE RENAL FAILURE

DR. HONEY, FEB 7, 2007

#13
DR. HONEY

DR. HONEY GUEST

1. AN OBESE PATIENT WITH DIABETES MELLITUS IS UNDER ANTI-HYPERTENSION


TREATMENT. HIS BLOOD PRESSURE IS 160/100MMBG ON EXAMINATION. WHICH OF THE
FOLLOWING IS YOUR INITIAL CONSIDERATION FOR THIS PATIENT:

A. DECREASED PROTEIN IN HIS DIET

B. CONCURRENT HYPERTENSIVE THERAPY

C. GIVE DIURETICS

D. CONTROL SUGAR INTAKE IN THE DIET

E. IDEAL WEIGHT*****

2. AT WHICH LEVEL OF CHOLESTERAL YOU CONSIDER TO GIVE LIPID-LOWERING STATINS


(EG, SIMVASTATIN, PRAVASTATIN)

A. 6 MMOL/L

B. 5.5 MMOL/L

C. 5 MMOL/L****

D. 4.5 MMOL/L

E. 4 MMOL/L
3. WHICH OF THE FOLLOWING IS THE MOST COMMON CHARACTERISTIC OF PLEURA
EFFUSION OF TB:

A. GLUCOSE DECREASED OR ABSENT

B. MONOCYTE

C. BLOOD STAINED

D. PROTEIN <2G

E. FIND TB BACILLUS

4. WHICH FOLLOWING GROUP IS THE MOST AT RISK OF HIV INFECTION:

A. HETEROSEXUAL****

B. HOMOSEXUAL

C. INTRAVENOUS DRUG USER

D. BLOOD TRANSFUSION

E. HAEMOPHILIAS

HIV IS TRANSMITTED PRIMARILY THROUGH SEXUAL CONTACT (OVER 70%);


WORLDWIDE, IT IS MORE COMMON IN HETEROSEXUAL MEN AND WOMEN THAN IN
HOMOSEXUAL MEN.

PARENTERAL TRANSMISSION OCCURS LARGELY AMONG INTRAVENOUS DRUG USERS;


TRANSMISSION BY CONTAMINATED BLOOD PRODUCTS IS RARE IN THE UNITED STATES,
THOUGH THIS REMAINS A SERIOUS PROBLEM IN DEVELOPING COUNTRIES.
5. WHICH OF THE FOLLOWING GROUP IS LEAST LIKE OF INFECTION OF HIV:

A. HETEROSEXUAL

B. BLOOD TRANSFUSION

C. HOMOSEXUAL

D. HAEMODIALYSIS****

E. HAEMOPHILIACS

6. DENGUE FEVER, ALL FOLLOWINGS ARE CORRECT EXCEPT:

A. ARBOVIRUS

B. MOSQUITO TRANSMISSION

C. CHILDREN GET LEAST SEVERE ILLNESS****

D. THERE IS NO SPECIFIC TREATMENT

E. AIR DROPLET INFECTION****

THE PROGNOSIS FOR UNCOMPLICATED DENGUE FEVER IS VERY GOOD, AND ALMOST
100% OF PATIENTS FULLY RECOVER. HOWEVER, AS MANY AS 6-30% OF ALL PATIENTS DIE
WHEN DHF OCCURS. THE DEATH RATE IS ESPECIALLY HIGH AMONG THE YOUNGEST
PATIENTS (UNDER ONE YEAR OLD).
7. A PATIENT WITH MYCOBACTERIA INFECTION WHICH OF THE FOLLOWING IS MOST
APPROPRIATE TREATMENT

A. COTRIMOXAZOLE

B. TETRACYCLINE****

C. AMOXYCILLINE

D. METRONIDAZOLE

E. ERITHROMYCIN****

8. WHAT IS COMPATIBLE WITH CRITICAL ILLNESS:

A. INCREASED CORTISOL , INCREASED TSH****

B. BOTH CORTISOL AND TSH DECREASED

C. INCREASED CORTISOL, DECREASED TSH

D. DECREASED CORTISOL, INCREASED TSH

E. NORMAL CORTISOL, INCREASED TSH

9.FOR AN ELDERLY MAN, WHICH ABOVE FOLLOWING BLOOD SUGAR LEVEL NEED
FURTHER INVESTIGATION

A. 5 MMOL/L

B. 5.5 MMOL/L

C. 6 MMOL/L

D. 6.5 MMOL/L****
E. 7 MMOL/L

10. A YOUNG PATIENT COMES FROM OVERSEAS WITH DIARRHOEA, NO BLOOD.


TEMPERATURE 37.9C, STOOL EXAMINATION SHOWED FEW SALMONELLAS. WHAT IS YOUR
MANAGEMENT:

A. OBSERVATION AND REPEAT STOOL EXAMINATION 3 DAYS LATER

B. BROAD SPECTRUM ANTIBIOTIC LIKE AMOXICILLIN

C. TRIMETHOPRINE PLUS SULPHASALAZINE

D. COTRIMOXAZOLE PLUS TRIMETHOPRIM

E. REASSURE

11. PATIENT WITH SUPPOSED HEPATOMA. WHICH OF THE FOLLOWING QUESTIONS IS


MOST HELPFUL FOR DIAGNOSIS:

A. PRESENT LIVER CIRRHOSIS****

B. ALCOHOL LIVER DISEASE

C. ACUTE HEPATITIS

D. CHOLANGITIS

E. FAMILY HISTORY OF LIVER HEPATOMA

THE MOST COMMON CAUSES OF HCC ARE ALCOHOLIC CIRRHOSIS, STEROID USE, AND
HEMOCHROMATOSIS.
12. IN ANAEMIA PATIENT WITH INCREASED TRANSFERRIN. ALL OF THE FOLLOWING IS
CORRECT EXCEPT:

A. INCREASED SERUM FERRITIN****

B. DECREASED SERUM FERRITIN

C. INCREASED TOTAL IRON BINDING CAPACITY

D. INCREASED TRANSFERRIN

E. DECREASED SERUM IRON

IT IS IRON DEFICIENCY

13. IN HAEMOLYTIC ANAEMIA, ALL IS CORRECT EXCEPT:

A. INCREASED UROBILINOGEN****

B. SERUM UROBILIN

C. ERYTHROCYTE COUNT

D. DECREASED MCV

E. INCREASED SERUM IRON

14 A 67 YEAR-OLD ALCOHOLIC MAN HAS BACKACHE, ESR 120MM/H. WHICH OF THE


FOLLOWING IS MOST LIKELY DIAGNOSIS:

A. MULTIPLE SCLEROSIS

B. ACUTE LYMPHATIC LEUKAEMIA

C. HODGKIN DISEASE

D. NON-HODGKIN DISEASE
E. MULTIPLE MYELOMA****

15. A 67 YEAR-OLD PATIENT WITH LONG STANDING ALCOHOLISM. LAB: HB 9.8; ESR 120:
LEUKOCYTES 10000 AND LEFT SHIFT. WHICH OF THE FOLLOWING IS DIAGNOSIS:

A. VIRAL HEPATITIS

B. MULTIPLE MYELOMA

C. IRON DEFICIENCY ANAEMIA

D. FATTY LIVER

16. A 28 YEAR-OLD FEMALE WITH LONG STANDING ALCOHOLISM, HISTORY OF BOWEL


IRRITATION. LAB: HB 9.8; MCV 110. WHICH IS THE MOST LIKELY DIAGNOSIS:

A. WHIPPLE€™S DISEASE

B. COLORECTAL CARCINOMA

C. IRRITATIVE BOWEL SYNDROME

D. ULCERATIVE COLITIS

E. COELIAC DISEASE

17 ALL FOLLOWING SIGNS ARE PRESENT IN PARKINSON DISEASE EXCEPT:


A. LOSS OF POSTURAL REFLEXES

B. MYOCLONUS

C. DECREASED REFLEXES

D. MUSCLE WEAKNESS

E. BROAD WIDE GAIT****

18. A 35 YEAR-OLD FEMALE WITH FACIAL PAIN, ASSOCIATED SENSORY DELICATE, NO


TRIGGER ZONE. WHAT IS THE DIAGNOSIS:

A. TRIGEMINAL NEURALGIA

B. RAMSAY €“ HUNT SYNDROME

C. BELL€™S PALSY

D. MULTIPLE SCLEROSIS

E. TOLOSA €“ HUNT SYNDROME

19. THESE ARE ALL PHENYTOIN SIDE EFFECTS EXCEPT:

A. NYSTAGMUS

B. OSTEOMALACIA

C. STIMULATE OSTEOPOROSIS

D. LYMPHOMA

E. DEAFNESS
20. PATIENT 64 YEAR-OLD, SUDDENLY OCCURRED HEMIPARESIS AND PARALYSIS OF
SOFT PALATE AND HORNER€™S SYNDROME ON THE SAME SIDE OF THE BODY. WHERE IS
THE LESION:

A. MIDDLE CEREBRAL ARTERY

B. CAROTID ARTERY OCCLUSION

C. VERTEBRO-BASILAR ARTERY

D. GIANT CELL ARTERITIS

E. SUBCLAVIAN STEAL SYNDROME

21. PATIENT 64 YEAR-OLD, SUDDENLY OCCURRED HEMIPARESIS AND PARALYSIS OF


SOFT PALATE AND HORNER€™S SYNDROME ON THE SAME SIDE OF THE BODY. WHERE IS
THE LESION:

A. MIDDLE CEREBRAL ARTERY

B. CAROTID ARTERY OCCLUSION

C. VERTEBRO-BASILAR ARTERY****

D. GIANT CELL ARTERITIS

E. SUBCLAVIAN STEAL SYNDROME

22. A 55 YEAR-OLD PATIENT HAS BLOOD PRESSURE 200/130MMHG. HIS LEFT KIDNEY IS
SMALL AND URINE TESTS AND KIDNEY FUNCTIONS ARE NORMAL. WHAT IS YOUR NEXT
INVESTIGATION:

A. INTRAVENOUS PYELOGRAPHY

B. RETROGRADE
C. RENAL CT SCAN

D. RENAL SCAN****

23. PATIENT HAS SUDDEN DYSPNEA. PHYSICAL EXAMINATION FOUND LEFT UPPER
CHEST RESPIRATORY MOVEMENTS INCREASED. IN LEFT LOWER PART PERCUSSION
DULLNESS AND DECREASED RESPIRATORY SOUNDS. WHAT IS THE DIAGNOSIS:

A. ATELECTASIS****

B. PNEUMONIA

C. PLEURAL EFFUSION

D. PULMONARY THROMBOSIS

E. PNEUMOTHORAX

24.POST-OPERATIVE THIRD DAY SERUM NA 165MMOL/L AND URINE OSMO IS 250


(NORMAL 50 €“ 1000). THE MOST LIKELY CAUSE IS:

A. THIAZIDE DIURETIC DOSE IS TOO HIGH

B. DIABETES MELLITUS

C. DIABETES INSIPIDUS****

D. SIADH

E. ACUTE RENAL FAILURE


25. STAGHORN CALCULI CAN BE FOUND IN THE ENTIRE FOLLOWING SITUATION EXCEPT:

A. RECURRENT PYELONEPHRITIS

B. IMMOBILISATION

C. IDIOPATHIC HYPERCALCIURIA

D. PRIMARY HYPERPARATHYROIDISM

26. A 7 YR WITH ENURESIS . WHAT IS THE MOST LIKELY CAUSE?

A. UTI

B. FAMILY DISORDER

C. O.M

D. VESICO -URETERIC REFLUX

E. ECTOPIC URETERS

27. A 6 YR BOY FOUND TO HAVE A SYSTOLIC MURMUR ON THE LEFT STERNAL BORDER.
HIS WEIGHT IS ON THE 3RD CENTILE. WHAT IS YOUR ADVICE TO H IS PARENTS?

A. THAT HE HAS ATRIAL SEPTAL DEFECT

B. THAT HE HAS VENTRICULAR SEPTAL DEFECT

C. THAT HE SHOULD BE REFERRED TO A CARDIOLOGIST

D. THAT THE MURMUR WILL REMIT SPONTANEOUSLY

E. THAT SURGERY IS ESSENTIAL TO CORRECT HIS CONDITION


28. WOF IS THE MOST LIKELY CAUSE OF MENINGITIS IN THE 4 Y.O CHILD?

A. E.COLI

B. HAEMOPHILUS INFLUENCE

C. ADENO-VIRUS

D. N. MENINGITIDIS

E. STREP. PNEUMONIAE

29. A 2 Y.O PRESENTED WITH CHRONIC CONSTIPATION. AXR SHOWER HEAVY FAECAL
LOADING OF THE COLON . P/R WAS UNEVENTFUL. WHAT IS THE MOST APPROPRIATED
NEXT STEP?

A. DOUBLE CONTRACT BARIUM ENEMA

B. RECTAL BIOPSY

C. ANAL DILATION

D. AXR

E. LAXATIVES

30. 5 YO PRESENTED WITH A MASS IN THE LEFT LOWER ABDOMINAL QUARANT.


OTHERWISE , HE IS WELL. WHAT IS THE MOST LIKELY CAUSE?

A. NEUROBLASTOMA

B. ACUTE LEUKAEMIA

C. NEPHROBLASTOMA (WILM€™S TUMOUR)

D. HODGKIN€™S DISEASE

E. NON-HODGKIN LYMPHOMA
31. A FULL TERM NEONATE WAS BORN AT 30/4, 1500 G WAS NOTICED TO BE
CYANOSED ON THE HANDS AND FEET . O/E THE INFANT WAS RESPONSIVE AND JITTERY
ON HANDING. THE RESPIRATORY RATE IS 40/ MIN AND LUNG AND HEART ARE CLINICALLY
NORMAL . WOF IS CORRECT?

A. THE INFANT MOST PROBABLY HAS A CONVULSION

B. O¬2¬¬ SHOULD BE GIVEN

C. BLOOD SUGAR LEVEL SHOULD BE CHECKED

D. BLOOD CALCIUM LEVEL SHOULD BE CHECKED

E. THE INFANT€™S RECTAL TEMPERATURE SHOULD BE CHECKED

32. A WOMAN PRESENTED WITH VAGINAL BLEEDING AT 8/40 OF HER PREGNANCY.


BLOOD LOSS WAS ESTIMATED AS 1L. O/E HER BP IS 90/60, PULSE IS 130/1€™. THE FETUS
IS DEAD. ALL THE FOLLOWING IS RELEVANT TO HER TREATMENT EXCEPT:

A. RESUSCITATE HER WITH I/V FLUIDS UNDER CVP CONTROL

B. DO U/S TO CHECK ECTOPIC PREGNANCY

C. ORDER A COAGULATION PROFILE

D. DO U/S TO CHECK PLACENTA PRAEVIA

E. INSECT A URINARY CATHETER

33. A 34Y.O, G2 P1 PRESENTS AT TERM WITH MILD PRE-ECLAMPSIA AND A BISHOP


SCORE OF 4. WOF IS INCORRECT IN HER MANAGEMENT?

A. RUPTURE THE MEMBRANES

B. RUPTURE THE MEMBRANES THEN PUT UP SYNTOCINON INFUSION


C. PUT UP SYNTOCINON INFUSION IN HARTMAN€™S SOLUTION

D. APPLY PROSTAGLANDIN PE2 ¬INTRA-VAGINALLY

34. A 8 WKS PREGNANT WOMAN COMES INTO CONTACT WITH A CHILD WHO WAS
RECENTLY DIAGNOSED WITH RUBELLA. ALL OF THE FOLLOWING IS TRUE EXCEPT

A. GIVE HER ANTI-RUBELLA SPECIFIC IG

B. IF INFECTION OCCURS THE FETUS MAY SUFFER FROM MICROCEPHALY, HEARING


LOSS AND CARDIAC ABNORMALITIES

C. ORDER IGG AND IGM ANTIBODIES

D. REPEAT TESTING FOR IGG AND IGM AT 14-16/40 EVEN IF INITIAL TEST WAS (-)

E. VACCINATE HER AGAINST RUBELLA

35. A 42 YO WOMAN ON O/C COME TO SEE YOU B/O MID-CYCLE SPOT BLEEDING.
WHAT IS THE MOST LIKELY CAUSE?

A. REDUCED OESTROGEN LEVEL

B. CERVICAL ECTROPION(EROSION)

C. CERVICAL INTRA-EPITHELIAL NEOPLASIA

D. ENDOCERVICAL CANCER

E. ATROPHIC VAGINITIS
36. A 50 YR WOMAN HAS BEEN ON TAMOXIFEN FOR 5 YRS BREAST CANCER . PRESENT
WITH MID €“CYCLE VAGINAL BLEEDING . WHAT IS THE MOST LIKELY CAUSE?

A. METASTASES FROM THE BREAST

B. ENDOMETRIAL CANCER

C. ENDOMETRIAL HYPERPLASIA

D. ENDOMETRIAL HYPOPLASIA BECAUSE OF TAMOXIFEN

E. DECREASED OESTROGEN LEVEL BECAUSE OF TAMOXIFEN

37.PROGESTION AGAINST ATHOROSCLEROSIS MEDIATED BY HRT. MAY BE DUE TO ALL


THE FOLLOWING EXCEPT

A. DECREASE TOTAL CHOLESTEROL LEVEL

B. DECREASE LDL LEVEL

C. DECREASE HDL LEVEL

D. DECREASE TOTAL TRIGLYCERIDES LEVELS

E. PROTECTIVE EFFECT ON VASCULAR WALL ENDOTHELIUM

38.A 28 YO WOMAN WITH 18/12 H/O POST PILL AMENORRHOEA [RESENTS B/O
INFERTILITY .ALL OF THE FOLLOWING INVESTIGATIONS MAY NEED TO BE CARRIED OUT
EXCEPT

A. S. PROLACTIN

B. CT SCAN EVEN IF THE PROLACTIN LEVEL IS NORMAL

C. S. OESTROGEN

D. FSH
E. LAPAROSCOPY FOR PCO

39. A YOUNG WOMAN PRESENTS B/O ACUTE PELVIC PAIN AFTER 8 WKS OF
AMENORRHOEA .B_HCG IS (-) AND ON U/S . SHE IS FOUND TO HAVE AN 1.8CM CYST,
WHAT IS THE MOST APPROPRIATED ACTION

A. ASK TO SEE HER AGAIN AFTER 3 WKS

B. REFER HER FRO LAPAROSCOPY

C. REFER HER FOR SURGICAL REMOVE OF THE CYST

D. PRESCRIBE ORAL CONTRACEPTIVE PILL

E. PRESCRIBE HER CLOMIPHENE

40. A 24 YRS WOMAN PRESENTS B/O ACUTE PELVIC PAIN AND SPOTTING AFTER 8 WKS
OF AMENORRHOEA . B-HCG IS +, IN THE PAST SHE HAS HAD APPENDICECTOMY AND
CHOLECYSTECTOMY . WHAT IS THE LEAST LIKELY CAUSE OF HER PAIN

A. ECTOPIC PREGNANCY

B. RED DEGENERATION OF A FIBROID

C. TORTES OVARIAN CYST

D. PELVIC ABSCESS

E. INEVITABLE ABORTION
41. A WOMAN IS BEING INVESTIGATION FOR INFERTILITY . HER GYNAECOLOGIST
ADVISED LAPAROSCOPY AND SHE ASKS YOUR OPINION ABOUT THE PROCEDURE . YOU
MAY TELL HER ALL THE FOLLOWING EXCEPT

A. VISCERAL DAMAGE IS POSSIBLE BUT UNLIKELY TO HAPPEN

B. VENA CAVAL OR AORTIC RUPTURE CAN CAUSE CARDIAC ARREST AND DEATH

C. LAPAROSCOPY IS DIAGNOSTIC OF PCO

D. SHOULDER PAIN IS COMMON AFTER THE PROCEDURE

E. SERIOUS HEART OR LUNG DISEASE USUAL, DO NOT CONTRAINDICATE THE


PROCEDURE

42. A YOUNG MAN WITH BILATERAL WEAKNESS AND WASTING OF THENAR,


HYPOTHENAR AND INTEROSSEI OF HAND , WHICH OF THE FOLLOWING IS THE MOST
POSSIBLE CAUSE

A. BILATERAL ULNAR NERVE PALSIES

B. BILATERAL MEDIAN NERVE PALSIES

C. MULTIPLE SCLEROSIS

D. BRAIN STEM INFARCTION

43. A LADY PRESENTS AN ENGORGED LONG SAPHENOUS VEINS AND ENGORGED


TRIBUTARIES IN THE CALF AND THIGH , WHICH OF THE FOLLOWING IS THE
MANAGEMENT?

A. LIGATION OF SAPHENOUS VEIN

B. LIGATION OF STRIPPING VEIN

C. AMPUTATE THE LEG IF VARICES ARE SEVERE

D. SCLEROTHERAPY
E. REMOVE VARICES VEIN UNDER LOCAL ANAESTHESIA

44. MANAGEMENT OF BEDRIDDEN IN A PATIENT INCLUDE ALL OF THE FOLLOWINGS


EXCEPT:

A. KEEP DRY THE PRESSURE AREA

B. USE A WATER BED

C. USE SHEEP SKIN ON THE BED

D. KEEP VENTILATION ON BOTH SIDES OF FEET

E. CONSTANT MOVEMENT BY TURNING THE PATIENT

45. A WOMAN PRESENTS SUDDEN ONSET OF ABDOMINAL PAIN, VOMITING FOR 8 HRS.
O/E: THERE WERE IRREGULAR PULSE , TERDERNESS OF ABDOMEN, BOWEL SOUND
ABSENT AND BLOODY DISCHARGE FROM RECTUM . WHICH OF THE FOLLOWING IS THE
MOST LIKELY DIAGNOSIS?

A. ULCERATION COLITIS

B. COLON CANCER

C. DIVERTICULITIS

D. MESENTERIC DISEASE

E. CROHN €˜S DISEASE

46. FIVE DAYS AFTER APPENDECTOMY , A PATIENT DEVELOPS 39C OF FEVER, WHICH OF
THE FOLLOWING IS THE MOST LIKELY CAUSE OF THE FEVER?

A. APPENDIX ABSCESS
B. WOUND INFECTION

C. THROMBOPHLEBITIS

D. PULMONARY ATELECTASIS

E. BRONCHOPNEUMONIA

47. LITHOGENESIS OF BILE IS INCREASE BY ALL OF THE FOLLOWING EXCEPT:

A. AGE

B. WEIGHT LOSS

C. MALABSORPTION

D. CROHN€™S DISEASE

E. HYPERLIPIDAEMIA

48. A MAN, MIDDLE AGE, COMPLAINS THAT THERE WAS A MILD ABDOMINAL PAIN
WITH AN ABDOMINAL DISTENSION , WHICH WAS RELIED BY PASSING FLATUS, THERE WAS
SEMIFORMAL STOOL MOTION . WHICH OF THE FOLLOWING IS THE MOST POSSIBLE
DIAGNOSIS?

A. IRRITABLE BOWEL SYNDROME

B. CANCER OF COLON

C. COELIAC DISEASE

D. DIVERTICULAR DISEASE

49. IN A PATIENT WITH SPHEROCYTOSIS, ALL OF THE FOLLOWING MAY HAPPEN AFTER
SPLENECTOMY EXCEPT
A. LIFE SPAN OF RBC WILL BECOME NORMAL

B. REDUCE THE RATE OF HAEMOLYSIS

C. INCREASE RISK OF INFECTION

D. INCREASE PLATELET

E. INCREASE STAPHYLOCOCCUS INFECTION

50. IN THIRD DAY AFTER HEAD SURGERY , A PATIENT HAS REDUCED LEVELS OF SERUM
NA+ K + AND CL -, THE URINE OSMOLALITY IS NORMAL, BUT BLOOD OSMOLALITY IS LOW,
WHICH OF THE FOLLOWING IS THE MOST LIKELY CAUSE?

A. WATER INTOXICATION

B. INAPPROPRIATE ADH SYNDROME

C. ACUTE RENAL FAILURE

D. CHORIC RENAL FAILURE

E. DIABETIC INSIPIDUS

51. WHICH OF THE FOLLOWING METASTATIC CARCINOMA HAVE THE WOST


PROGNOSIS:

A. PROSTATE

B. BREAST

C. STOMACH

D. UTERUS

E. COLON
52. WHAT IS CORRECT IN NORMAL GRIEF?

A. EFFECTIVE REACTIVITY IS PRESERVED

B. SUICIDAL RISK IS INCREASED

C. PROGRESSION TO PATHOLOGICAL GRIEF IS COMMON

D. MAY PRECIPITATE SCHIZOPHRENIA

E. MAY PRECIPITATE DELUSIONAL DISORDER

53. A 24 YEARS OLD MAN WAS APPREHENDED BY THE POLICE IN PUBLIC PARK BECAUSE
OF AGGRESSIVE BEHAVIOUR. HE CLAIMED TOTAL AMNESIA OF THE EVENT. WHAT IS THE
MOST LIKELY EXPLANATION FRO HIS BEHAVIOUR:

A. GLOBAL AMNESIA

B. BRIEF PSYCHOTIC REACTION

C. ALCOHOL AND DRUG INTOXICATION

D. ORGANIC BRAIN DISORDER

E. EPILEPSY

54. WHICH OF THE FOLLOWING IS THE BEST POST-OPERATION CARE FOR REVERSING
THE EFFECT OF NEUROMUSCULAR BLOCKING ANAESTHESIA :

A. ATROPINE

B. PHYSOSTIGMINE

C. PANCURONIUM BROMIDE
D. INTUBATION & ADEQUATE VENTILATION

E. SUXAMETHONIUM

55. ALL THE FOLLOWING ARE FEATURES OF BULIMIA NERVOSA EXCEPT

A. HIRSUTISM

B. TEETH EROSION

C. AMENORRHOEA

D. BILATERAL PAROTID SWELLING

E. HOARSENESS OF VOICE

DR. HONEY, FEB 7, 2007

#14

BOOBY

BOOBY GUEST

1. YOU ARE CALLED TO THE WARD TO VISIT A CHILD WHO WAS BORN AT TERM BY

NORMAL VAGINAL DELIVERY BUT ONLY WEIGHT 1800 G . YOU WILL EXCEPT ALL OF THE

FOLLOWING EXCEPT:

A. IDIOPATHIC RESPIRATORY DISTRESS SYNDROME

B. HYPOGLYCAEMIA

C. MECINOUM ASPIRATION ***

D. HYPOTHERMIA DUE TO LOW BODY WEIGHT


2. A MOTHER BRINGS HER 6 MONTHS OLD CHILD TO SEE YOU BECAUSE SINCE HIS LAST

DTP VACCINATION 2 WEEKS AGO,HE NO LONGER SLEEPS WELL AT NIGHT AND CRYS
WHEN

SHE TRYS TO LEAVE HIM. WHICH OF THE FOLLOWING ARE YOU GOING TO TELL THE
MOTHER:

A. REASSURE THE MOTHER THAT THIS BEHAVIER IS NORMAL FOR HIS AGE AND WILL
PASS ***?

B. THE CHILD HAS MUST HAVE ANY MORE VACCINATION

C. THIS IS A REACT TO THE VACCINATION

D. TAKE A URINE SAMPLE FOR EXAMINATION

3. JAUNDICE APPEARING *16 HR AFTER BIRTH ( IN A FULL TERM BABY ). MOST LIKELY

CAUSE:

A. EXCESSIVE DESTRUCTION OF NORMAL RED BLOOD CELLS

B. ISO-IMMUNISATION***

C. LIVER IMMATURITY

D. INFECTION

E. PHYSIOLOGICAL

4.A CHILD WITH BIRTH WEIGHT 3125 G ( IN 75 TH PERCENTILE ) . AFTER ONE MONTH
HIS WEIGHT IS IN THE 10TH PERCENTILE.HE PASSES LOTS OF URINE AND BOWEL OPENS

TWICE A DAY. HE FEEDS 6 HOURLY , 210 G FORMULARY MILK. WHAT IS THE FIRST

STEP OF MANAGEMENT:

A. CHECK THE STRENGHT OF THE FORMULA***

B. DO A SWEAT TEST

C. TEST FEEDING THE CHILD

D. EXAMINE THE STOOLS MICROSCOPICALLY

E. STOOL CULTURE

5.WHAT IS THE MOST LIKELY TO BE ACCOSSIATED WITH IN A CHILD WITH ATOPIC

ECZEMA :

A. COW MILK

B. IGG ACCOSSIATED

C. EFFECTS ONLY THE EXTENSOR SURFACES **

D. CAUSE DRY SKIN

E. ONLY CASE IN THE NAPPY AREA

6. A 3 Y OLD BOY WITH NORMAL GROSS AND FINE MOTOR SKILLS, IS ONLY ABLE TO

PRONOUNCE 3 WORDS IN INCOMPREHENSIBLE MANNER . ALL COULD BE RESPONSIBLE


EXCEPT:

A. AUTISM

B. MILD MENTAL RETARDATION


C. DEAFNESS

D. CLEFT PALATE***

E. DYSPHONIA

7.WHAT IS NOT TRUE ABOUT HELICOBACTER PYLORI?

A. IT IS ASSOCIATED WITH GASTRIC CARCINOMA

B. IT IS ASSOCIATED WITH GASTRIC LYMPHOMA

C. GASTRITIS

D. GASTRO-OESOPHAGEAL REFLUX***

E. PREPYLORIC ULCERS

8. REGARDING ANGIOPLASTY ALONE VERSUS STENT INSERTION, WHICH ONE IS TRUE?

A. DECREASED RESTENOSIS BUT NO INCREASE IN ACUTE COMPLICATIONS

B. STENTS SHOULD ONLY BE USED IF ANGIOPLASTY HAS FAILED

C. STENTS ARE CONTRAINDICATED IN SEVERE MULTIVESSEL DISEASE

D. DECREASED COMPLICATIONS BUT NOT DECREASED RE-STENOSIS RATE ***

9. A MAN , 56Y OLD, HAD A *SWOLLEN KNEE. THE ASPIRATION SHOWS CLOUDY FLUID,

NEUTROPHILS++++ AND *NO BACTERIA ON MICROSCOPY AND CULTURE. WHAT IS THE


MOST
LIKELY DIAGNOSIS?

A. SEPTIC ARTHRITIS ???

B. REITER'S***

C. GOUT

D. RHEUMATOID ARTHRITIS

10. WITH REGARDS TO COLORECTAL CARCINOMA WHICH IS CORRECT?

A. OVER 50% OF CANCERS DETECTED WITH SCREENING ARE DUKES A

B. FLEXIBLE SIGMOIDOSCOPY PICKS UP MORE THAN 40%

C. FAECAL OCCULT BLOOD TESTING IS GOOD SCREENINING FOR THE POPULATION ***

D. 20% WILL BE MISSED BY FAECAL OCCULT BLOOD BECAUSE THE CANCER IS NOT
BLEEDING AT THE TIME****

E. MOST OF CANCERS DO NOT ARRISE FROM VILLOUS ADENOMATA

11. REGARDING A SERUM LIPID PROFILE AND DIET WHAT IS TRUE:

A.

B.OLIVE OIL WILL INCREASE THE LDL/HDL RATIO

C.POSITIVE ENERGY BALANCES INCREASE LDH

D.100-300G OF FISH PER WEEK WILL HAVE NO EFFECT ON PLASMA OR MEMBRANES

E. AN INCREASE IN DIETARY SATURATED FATTY ACIDS WILL INCREASE VLDL ***


12. PULMONARY HYPERTENSION AND COR PULMONALE ARE FREQUENTLY SECONDARY
TO:

A. EMPHYSEMA ***

B. BRONCHIECTASIS

13. A 60Y OLD WOMAN WITH ALONG HISTORY OF SMOKING PRESENTED WITH
DYSPNOEA.

ON EXAMINATION THERE WERE BIBASAL CREPITATIONS. FEV1 AND FVC WERE BOTH

SEVERELY REDUCED BUT THE RATIO (NOT GIVEN BUT CALCULATED) WAS 80%. WHAT IS

THE MOST LIKELY DIGNOSIS?

A. FIBROSING ALVEOLITIS***

B. INTERSTITIAL LUNG DISEASE

C. BRONCHOGENIC CARCINOMA

D. DECREASED LUNG COMPLIANCE

14. PHOTO OF A DIABETIC LEG THAT WAS RED AND SHINY. WHAT IS THE BEST
TREATMENT.

A. PENICILLIN AND FLUCLOXACILLIN ***

B. PENICILLIN AND GENTAMICIN


C. PENICILLIN, GENTAMICIN AND METRONIDAZOLE

D. METRONIDAZOLE

E. AMPICILLIN AND GENTAMICIN

15. A 36 YEAR OLD PRIMIGRAVIDA IS WORRIED ABOUT DOWN'S SYNDROME. WHICH OF


THE

FOLLOWING INVESTIGATIONS IS THE *MOST SPECIFIC AND AT THE SAME TIME HAS THE
*LOWEST RISK

FOR THE FETUS?

A. MATERNAL SCREENING AT 16 WEEKS GESTATION

B. AMNIOCENTESIS AT 18 WEEKS

C. CVB AT 12 WEEKS

D. ULTRASOUND AT 11 WEEKS TO LOOK FOR NUCHAL THICKENING ***

E. CORDIOCENTESIS AT 24 WEEKS

16.A WOMAN OF 48 YEARS THAT HAD A CIN LESION REMOVED 10 YEARS AGO AND HAS

RECENTLY HAD A HYSTERECTOMY BUT HAS LEFT HER OVARIES LEFT. WHICH OF THE

FOLLOWING IS *NOT TRUE?

A. WOMEN THAT HAVE HAD THIS TYPE OF OPERATION HAVE LESS CHANCE OF GETTING

CANCER OF THE OVARIES THAN WOMEN THAT HAVE NOT HAD THIS OPERATION ***

B. SHE WILL NEED TO HAVE VAGINAL VAULT SMEARS EVERY 2 YEARS

C. SHE WILL ENTER MENOPAUSE AT THE NORMAL AGE, PERHAPS JUST A LITTLE EARLIER.

D. OESTROGEN THERAPY WILL BE BENEFICIAL TO HER WHEN SHE ENTERS MENOPAUSE


17. A 52 YEAR OLD WOMAN THAT HAS HAD A HYSTERECTOMY AND A POST OPERATIVE

DVT, COMES TO SEE YOU COMPLAINING OF SEVERE HOT FLUSHES . WHICH OF THE

FOLLOWING WOULD YOU RECOMMEND?

A. THE COMBINED ORAL CONTRACEPTIVE PILL

B. OESTROGEN TRANSDERMAL PATCHES

C. LOW DOSE OESTROGEN THERAPY PER OS ***

D. OESTROGEN AND PROGESTERONE TRANSDERMAL PATCH

E. PROGESTERONE THERAPY ONLY

18. A 18 Y OLD FEMALE WHO HAS HAD MULTIPLE PARTNER DURING LAST YEAR,
COMING

TO YOU DUE TO VAGINAL DISCHARGE.ALL OF THE FOLLOWING IS TRUE EXCEPT:

A. TAKE HIV TEST.

B. TAKE ENDOCERVICAL SWAB FOR GONNORRHEA

C. TAKE A SWAB FROM THE POSTERIOR FORNIX FOR CHLAMYDIA

D.TREAT SEXUAL PARTNER***

E. CERVICAL SMEAR CYTOLOGY

19. A 28-Y OLD WOMAN, G2, P 1 WITH A NORMAL VAGINAL DELIVERY IN THE PAST
.NOW PREGNANT IN 37 WEEK AND COMES TO THE DELIVERY WARD FOR INDUCTION
DUE TO

PRE-ECLAMPSIA.THE BABY LIES LONGITUDINAL, CEPHALIC, NOT ENGAGED. AT THIS

TIME YOU RUPTURE THE MEMBRANE AND GIVE THE PATIENTE A SYNTOCYNON-
INFUSION.

IN RELATION TO THE START OF FIRST UTERINE CONTRACTION THE FETUS HAS

BRADYCARDIA OF 60 .WHAT DO YOU DO NEXT:

A. CESARIAN SECTION

B. DO ABDOMINAL CTG ***?

C. DO VAGINAL EXAMINATION ??

D. JUST OBSERVE

E. OXYTOCIN

20. A WOMAN AFTER 10 H OF OBSTRUCTIVE LABOUR DELIVERS BY CESARIAN SECTION.

3 DAYS AFTER SHE DEVELOPS FEVER 37.9C WHAT IS THE **LEAST LIKELY:

A. UTI

B. WOUND HAEMATOMA

C. DVT

D. ENDOMETRIAL INFECTION

E. ENGORGEMENT OF THE BREAST ***?


21. A MOTHER BRINGS HER 4 YEAR OLD GIRL WITH INTERMITTENT YELLOW VAGINAL

DISCHARGE. IN THE EXAMINATION YOU FIND THAT THE VULVA IS NORMAL


STRUCTURELY

EXCEPT LITTLE RED AND THE GIRL TEND TO PUT THE HAND TO THE AREA, WITH SIGN

OF SCRATCHING. WHAT IS THE MOST LIKELY DIAGNOSIS:

A. FOREIGN BODY ***

B. MONILIAL INFECTION.

C. GARDARELLA VAGINALIS

C. SYPHILIS

E. SEXUAL ABUSE

22. A WOMAN WHO HAS PREVIOUSLY DELIVERED A 4 KG BABY WITH TROUBLESOME, IS

NOW *33WKS PREGNANT WITH BREECH PRESENTATION. YOU SHOULD DO ALL THE
FOLLOWING

*EXCEPT

A. DO X-RAY FOR PELVIMETRY

B. CESAREAN SECTION IF THE BABY > 4 KG *??

C. MAY BE YOU CAN TRY A NORMAL DELIVERY IF THE BABY IS <3.8KG, IS IN FRANK

OR COMPLETE BREECH

D. DO A US TO MEASURE FETAL SIZE AND POSITION


23. 37 WEEKS OF PREGNANCY, WITH VAGINAL BLEEDING OF 900ML. HAEMOGLOBIN

DECREASED (6.8) BP 130/80. EXAMINATION UTERUS IS TENDER AND FETAL DEAD IS

OBSERVED. MANAGEMENT SHOULD INCLUDE ALL THE FOLLOWING *EXCEPT:

A. US

B. ANTICOAGULATION PROFILE

C. IF RH NEGATIVE, GIVE HAEMOGLOBIN

D. BLOOD TRANSFUSION AND IMMEDIATE SURGERY***

E. OXYTOCIN INFUSION

8. REGARDING THE INTERACTION BETWEEN GP'S IN AUSTRALIA AND PSYCHIATRIC

DISORDERS, WHICH OF THE FOLLOWING IS TRUE?

A. ONE THIRD OF PATIENTS THAT PRESENT FOR PRIMARY CARE HAVE A PSYCHIATRIC

DISORDER

B. MOST OF THE PATIENTS ARE NOT REFERRED TO A PSYCHIATRIST

C. ALCOHOL AND DRUG ABUSE ARE COMMONLY OVERLOOKED BY GP"S

24.. WHICH OF THE FOLLOWING IS A BAD PROGNOSTIC FACTOR FOR A WOMAN WITH

DIABETES MELLITUS AND A HISTORY OF DEPRESSION OF HAVING A RELAPSE?

A. HYPERTENSION

B. PREVIOUS H.R.T.
C. PREVIOUS HISTORY OF ALCOHOL ABUSE

D. PREVIOUS GOOD RESULTS WITH ELECTROCONVULSIVE THERAPY

???

25. A 20 YEAR OLD WOMAN WITH IDDM AND REPEATED EPISODES OF "ACCIDENTAL"
OVERDOSE

OF INSULIN WAS REFERRED TO THE PSYCHIATRIC DEPARTMENT BY THE DEPARTMENT


OF

GENERAL MEDICINE. YOU FIND THAT SHE IS A SEVERE BORDERLINE CASE. WHICH OF

THE FOLLOWING IS TRUE?

A. THE PHYSICIANS AND PSYCHIATRISTS SHOULD WORK TOGETHER CLOSELY ON THIS

CASE TO REDUCE THE RISK OF COUNTER TRANSFERENCE ***

B. RECOMMEND IMMEDIATE PSYCHOTHERAPY

C. GET A COURT ORDER TO MAKE SURE THAT SHE TREATS HER DIABETES PROPERLY

D. DO NOT PRESCRIBE A PHENOTHIAZINE

E. CHANGE HER THERAPY FROM INSULIN TO ORAL HYPOGLYCAEMICS

BOOBY, FEB 8, 2007

#15

DR.SANDRA

DR.SANDRA GUEST

Q1. WHAT IS THE DRUG OF CHOICE TO CONTROL SUPRAVENTRICULAR TACHYCARDIA?

A) ADENOSINE ***

B) PROPRANOLOL
C) VERAPAMIL

D) DIGOXIN

Q2. A 5 YEAR OLD CHILD IS RUSHED TO CASUALTY REPORTEDLY ELECTROCUTED WHILE


PLAYING IN A PARK. THE CHILD IS *APNEIC AND IS VENTILATED WITH BAG MASK. THERE
ARE BURNS ON EACH HAND. WHAT WILL BE THE NEXT STEP IN THE MANAGEMENT?

A) CHECK PULSES

B) START CHEST COMPRESSIONS

C) INTUBATE ***

D) CHECK OXYGEN

Q3. RECURRENT RESPIRATORY TRACT INFECTIONS MAY OCCUR IN ALL OF THE


FOLLOWING EXCEPT:

A) VENTRICULAR SEPAL DEFECT

B) TETROLOGY OF FALLOT ***

C) TRANSPOSITION OF GREAT ARTERIES

D) TOTAL ANOMALOUS VENOUS RETURN

Q4. A 6 MONTH OLD BOY WEIGHTING 3.2 KG PRESENTS WITH RECURRENT VOMITING
AND POLYURIA INVESTIGATIONS SHOW BLOOD UREA 60 MG/DL CREATININE 0.7 MG/DL,
CALCIUM 12.8 MG/DL, PHOSPHATE 3 MG/DL, PH 7.45, BICARBONATE 25 MEQ/L AND PTH
140 PG/ML (NORMAL<60 PG/ML). DAILY URINARY CALCIUM EXCRETION IS REDUCED.
ULTRASOUND ABDOMEN SHOW BILATERAL NEPHROCALCINOSIS. THE MOST LIKELY
DIAGNOSIS IS:

A) BARTTER SYNDROME

B) MUTATION OF THE CALCIUM SENSING RECEPTOR.

C) PSEUDO-PSEUDOHYPOPARATHYROIDISM
D) PARATHYROID ADENOMA ***

Q5. A FEMALE CHILD HAS RECENTLY LEARNED TO EAT WITH SPOON WITHOUT SPILLING;
TO DRESS AND UNDRESS HERESELF WITH SUPERVISION; AND TO UNDERSTAND THAT SHE
IS A GIRL. THESE SKILLS ARE FIRST MASTERED BETWEEN THE AGES OF:

A) 2 & 3 YEARS ***

B) 3 & 4 YEARS

C) 4 & 5 YEARS

D) 5 & 6 YEARS

DR.SANDRA, FEB 10, 2007

#16

DR.SANDRA

DR.SANDRA GUEST

1. FALSE SENSE OF PERCEPTION WITHOUT ANY EXTERNAL OBJECT OR STIMULUS IS


KNOWN AS:

1. ILLUSION.

2. IMPULSE.

3.HALLUCINATION.

4. PHOBIA.

2. A YOUNG FEMALE PRESENTS WITH HISTORY OF DYSPNOEA ON EXERTION. ON


EXAMINATION, SHE HAS WIDE, FIXED SPLIT OF S2 WITH EJECTION SYSTOLIC MURMUR
(III/VI) IN LEFT SECOND INTERCOSTALS SPACE. HER EKG SHOWS LEFT AXIS DEVIATION. THE
MOST PROBABLE DIAGNOSIS IS :
1. TOTAL ANOMALOUS PULMONARY VENOUS DRAINGE.

2. TRICUSPID ATRESIA.

3. OSTIUM PRIMUM ATRIAL SEPTAL DEFECT.

4. VENTRICULAR SEPTAL DEFECT WITH PULMONARY ARTERIAL HYPERTENSION.

3. A 62 YEARS OLD MAN WITH CARACIOMA OF LUNG PRESENTED TO EMERGENCY


DEPARTMENT WITH RESPIRATORY DISTRESS. HIS EKG SHOWED ELECTRICAL ALTERNANS.
THE MOST LIKELY DIAGNOSIS IS :

1. PNEUMOTHORAX.

2. PLEURAL EFFUSION.

3. CARDIAC TAMPONADE.

4. CONSTRICTIVE PERICARDITIS.

4. ATRIAL FIBRILLATION MAY OCCUR IN ALL OF THE FOLLOWING CONDITIONS, EXCEPT:

1. MITRAL STENOSIS.

2. HYPOTHYROIDISM.

3. DILATED CARDIOMYOPATHY.

4. MITRAL REGURGITATION.

5. A PATIENT WITH RECENT-ONSET PRIMARY GENERALIZED EPILEPSY DEVELOPS DRUG


REACTION AND SKIN RASH DUE TO PHENYTOIN SODIUM, THE MOST APPROPRIATE
COURSE OF ACTION IS :

1. SHIFT TO CLONAZEPAM.

2. RESTART PHENYTOIN SODIUM AFTER 2 WEEKS.

3. SHIFT TO SODIUM VALPROATE.

4. SHIFT TO ETHOSUXIMIDE.
6. A PATIENT UNDERGOING SURGERY SUDDENLY DEVELOPS HYPOTENSION. THE
MONITOR SHOWS THAT THE END TIDAL CARBON DIOXIDE HAS DECREASED ABRUPTLY BY
15MMHG. WHAT IS THE PROBABLE DIAGNOSIS?

1. HYPOTHERMIA.

2. PULMONARY EMBOLISM

3. MASSIVE FLUID DEFICIT

4. MYOCARDIAL DEPRESSION DUE TO ANESTHETIC AGENTS.

7. WHICH OF THE FOLLOWING MARKER IN THE BLOOD IS THE MOST RELIABLE


INDICATOR OF RECENT HEPATITIS B- INFECTION?

1. HBSAG

2. LGG ANTI - HBS

3. LGM ANTI - HBC

4. LGM ANTI €“ HBE

8. THE RISK OF DEVELOPING INFECTIVE ENDOCARDITIS IS THE LEAST IN A PATIENT


WITH:

1. SMALL VENTRICULAR SEPTAL DEFECT

2. SEVERE AORTIC REGURGITATION.

3. SEVERE MITRAL REGURGITATION

4. LARGE ATRIAL SEPTAL DEFECT.

9. A 20 YEAR OLD MALE HAD PAIN ABDOMEN AND MILD FEVER FOLLOWED BY
GASTROENTERITIS. THE STOOL EXAMINATION SHOWED PRESENCE OF PUS CELLS AND
RBCS ON MICROSCOPY. THE ETIOLOGICAL AGENT RESPONSIBLE IS MOST LIKELY TO BE:
1. ENTEROINVASIVE E. COLI.

2. ENTEROTOXIGENIC E. COLI.

3. ENTEROPATHIOGENIC E. COLI.

4. ENETROAGGREGATIVE E. COLI.

10. IN TRIPLE SCREENING TEST FOR DOWN'S SYNDROME DURING PREGNANCY ALL OF
THE FOLLOWING ARE INCLUDED EXCEPT.

1. SERUM BETA HCG

2. SERUM OESTRIOL

3. MATERNAL SERUM ALFA FETOPROTEIN

4. ACETYL CHOLINESTERASE

11.AN OPTIC NERVE INJURY MAY RESULT IN ALL OF THE FOLLOWING EXCEPT.

1. LOSS OF VISION IN THAT EYE.

2. DILATATION OF PUPIL.

3. PTOSIS

4. LOSS OF LIGHT REFLEX.

12.FATHER TO SON INHERITANCE IS NEVER SEEN IN CASE OF:

1. AUTOSOMAL DOMINANT INHERITANCE

2. AUTOSOMAL RECESSIVE INHERITANCE

3. X- LINKED RECESSIVE INHERITANCE

4. MULTIFACTORIAL INHERITANCE
13. A 3- YEAR OLD BOY IS DETECTED TO HAVE BILATERAL RENAL CALCULI. METABOLIC
EVALUATION CONFIRMS THE PRESENCE OF MARKED HYPERCALCIURIA WITH NORMAL
BLOOD LEVELS OF CALCIUM, MAGNESIUM, PHOSPHATE, URIC ACID AND CREATININE. A
DIAGNOSIS OF IDIOPATHIC HYPERCALCIURIA IS MADE. THE DIETARY MANAGEMENT
INCLUDES ALL, EXCEPT

1. INCREASED WATER INTAKE

2. LOW SODIUM DIET

3. REDUCED CALCIUM INTAKE

4. AVOID MEAT PROTEINS.

14. ALL OF THE FOLLOWING ARE CHARACTERISTIC FEATURES OF KWASHIORKOR,


EXCEPT :

1. HIGH BLOOD OSMOLARITY.

2. HYPOALBUMINEMIA.

3. EDEMA.

4. FATTY LIVER.

15. A 55-YEAR-OLD MALE ACCIDENT VICTIM IN CASUALTY URGENTLY NEEDS BLOOD.


THE BLOOD BANK IS UNABLE TO DETERMINE HIS ABO GROUP, AS HIS RED CELL GROUP
AND PLASMA GROUP DO NOT MATCH. EMERGENCY TRANSFUSION OF PATIENT SHOULD
BE WITH:

1. RBC CORRESPONDING TO HIS RED CELL GROUP AND COLLOIDS/CRYSTALLOID.

2. WHOLE BLOOD CORRESPONDING TO HIS PLASMA GROUP.

3. O POSITIVE RBC AND COLLOIDS/ CRYSTALLOID.

4. AB NEGATIVE WHOLE BLOOD.


16. ALTHOUGH MORE THAN 400 BLOOD GROUPS HAVE BEEN IDENTIFIED, THE ABO
BLOOD GROUP SYSTEM REMAINS THE MOST IMPORTANT IN CLINICAL MEDICINE
BECAUSE. :

1. IT WAS THE BLOOD GROUP SYSTEM TO BE DISCOVERED.

2. IT HAS FOUR DIFFERENT BLOOD GROUPS A, B, AB, O(H).

3. ABO (H) ANTIGENS ARE PRESENT IN MOST BODY TISSUES AND FLUIDS.

4. ABO (H) ANTIBODIES ARE INVARIABLY PRESENT IN PLASMA WHEN PERSONS RBC
LACKS THE CORRESPONDING ANTIGEN.

17.AFTER A MINOR HEAD INJURY A YOUNG PATIENT WAS UNABLE TO CLOSE HIS LEFT
EYE AND HAD DROOLING OF SALIVA FROM LEFT ANGLE OF MOUTH. HE IS SUFFERING
FROM :

1. VIITH NERVE INJURY.

2. VTH NERVE INJURY.

3. IIIRD NERVE INJURY.

4. COMBINED VIITH AND IIIRD NERVE INJURY.

18. WHICH ONE OF THE FOLLOWING DOES NOT PRODUCE CYANOSIS IN THE FIRST YEAR
OF LIFE. :

1. ATRIAL SEPTAL DEFECT.

2. HYPOPLASTIC LEFT HEART SYNDROME.

3. TRUNCUS ARTERIOSUS.

4. DOUBLE OUTLET RIGHT VENTRICLE.

19. THE FOLLOWING DRUG IS INDICATED IN THE TREATMENT OF PITYRIASIS


VERSICOLAR :

1. KETOCONAZOLE.
2. METRONIDAZOLE.

3. GRISEOFULVIN.

4. CHLOROQUINE.

20. A PATIENT WAS ADMINISTERED EPIDURAL ANAESTHESIA WITH 15ML 1.5%


LIGNOCAINE WITH ADRENALINE FOR HERNIA SURGERY. HE DEVELOPED HYPOTENSION
AND RESPIRATORY DEPRESSION WITHIN 3 MINUTES AFTER ADMINISTRATION OF BLOCK.
THE COMMONEST CAUSE WOULD BE.

1. ALLERGY TO DRUG ADMINISTERED.

2. SYSTEMIC TOXICITY TO DRUG ADMINISTERED.

3. PATIENT GOT VASOVAGAL SHOCK.

4. DRUG HAS ENTERED THE SUB ARACHNOID SPACE.

21. A 60 YEAR OLD HYPERTENSIVE PATIENT ON ANGIOTENSIN II RECEPTOR


ANTAGONISTS (LOSARTAN) IS POSTED FOR HERNIA REPAIR SURGERY. THE ANTI-
HYPERTENSIVE DRUG SHOULD BE :

1. CONTINUED TILL THE DAY OF OPERATION.

2. DISCONTINUED 24 HRS. PREOPERATIVELY.

3. DISCONTINUED ONE WEEK PREOPERATIVELY.

4. ADMINISTERED IN AN INCREASED DOSAGE ON THE DAY OF OPERATION.

22. A 'MALIGNANT PUSTULE' IS A TERM USED FOR

1. AN INFECTED MALIGNANT MELANOMA

2. A CARBUNCLE

3. A RAPIDLY SPREADING RODENT ULCER.

4. ANTHRAX OF SKIN
23. A WARTHIN'S TUMOUR IS:

1. AN ADENOLYMPHOMA OF PAROTID GLAND

2. A PLEOMORPHIC ADENOMA OF THE PAROTID

3. A CARCINOMA OF THE PAROTID.

4. A CARCINOMA OF SUBMANDIBULAR SALIVARY GLAND

24. HYPOCALCAEMIA CHARACTERIZED BY ALL EXCEPT.

1. NUMBNESS AND TINGLING OF CIRCUMORAL REGION.

2. HYPERACTIVE TENDON REFLEXES AND POSITIVE CHVOSTEK'S SIGN.

3. SHORTENING OF Q-T INTERVAL IN ECG.

4. CARPOPEDAL SPASM

25. WHICH OF THE FOLLOWING NOT TRUE OF GAS GANGRENE.

1. IT IS CAUSED BY CLOSTRIDIUM PERFRINGENS

2. CLOSTRIDIUM PERFRINGENS IS A GRAM-NEGATIVE SPORE-BEARING BACILLUS.

3. GAS GANGRENE IS CHARACTERIZED BY SEVERE LOCAL PAIN CREPITUS AND SIGNS OF


TOXEMIA.

4. HIGH DOSE PENICILLIN AND AGGRESSIVE DEBRIDEMENT OF AFFECTED TISSUE IS THE


TREATMENT OF ESTABLISHED INFECTION.

OBS AND GYNAE

G1. THE COMMONEST CAUSE OF OCCIPITO- POSTERIOR POSITION OF FETAL HEAD


DURING LABOUR IS:

1.MATERNAL OBESITY
2.DEFLEXION OF FETAL HEAD

3.MULTIPARITY

4.ANDROID PELVIS.

G2. THE COMMONEST CAUSE OF BREECH PRESENTATION IS

1. PREMATURITY

2. HYDROCEPHALUS

3. PLACENTA PREVIA

4. POLYHYDRAMNIOS

G3. THE COMMONEST CONGENITAL ANOMALY SEEN IN PREGNANCY WITH DIABETES


MELLITUS IS

1. MULTICYSTIC KIDNEYS

2. OESOPHAGEAL ATRESIA

3. NEURAL TUBE DEFECT

4. ENALAPRIL

G4. USE OF WHICH OF THE FOLLOWING DRUG IS CONTRA-INDICATED IN PREGNANCY.

1. DIGOXIN

2. NIFEDIPINE

3. AMOXYCILLIN

4. EALAPRIL
G5. A 21 YEAR OLD PRIMIGRAVIDA IS ADMITTED AT 39 WEEKS GESTATION WITH
PAINLESS ANTEPARTUM HAEMORRHAGE. ON EXAMINATION UTERUS IS SOFT, NON
TENDER AND HEAD ENGAGED. THE MANAGEMENT FOR HER WOULD BE:

1. BLOOD TRANSFUSION AND SEDATIVES.

2. A SPECULUM EXAMINATION

3. PELVIC EXAMINATION IN OT

4. TOCOLYSIS AND SEDATIVES.

G6. THE TREATMENT OF CHOICE FOR STAGE 1 CANCER LARYNX IS:

1. RADICAL SURGERY.

2. CHEMOTHERAPY.

3. RADIOTHERAPY.

4. SURGERY FOLLOWED BY RADIOTHERAPY.

G7. THE FOLLOWING STATEMENTS REGARDING TURNER SYNDROME ARE TRUE EXCEPT.

1. OCCURRENCE OF TURNER SYNDROME IS INFLUENCED BY MATERIAL AGE.

2. MOST PATIENTS HAVE PRIMARY AMENORRHOEA.

3. MOST PATIENTS HAVE SHORT STATURE.

4. EDEMA OF HANDS AND FEET IS AN IMPORTANT FEATURE DURING INFANCY.

G8. PEROCONCEPTIONAL USE OF THE FOLLOWING AGENT LEADS TO REDUCED


INCIDENCE OF NEURAL TUBE DEFECTS :

1. FOLIC ACID.

2. IRON.

3. CALCIUM.
4. VITAMIN A.

G9. USE OF TAMOXIFEN IN CARCINOMA OF BREAST PATIENTS DOES NOT LEAD TO THE
FOLLOWING SIDE EFFECTS.

1. THROMBOEMBOLIC EVENTS

2. ENDOMETRIAL CARCINOMA

3. CATARACT

4. CANCER IN OPPOSITE BREAST

G10. THE LEVEL OF ALPHA FETOPROTEIN IS RAISED IN ALL OF THE FOLLOWING EXCEPT.

1. CIRROHOSIS OF LIVER.

2. HEPATOCELLUR CARCINOMA

3. YOLKSAC TUMOR

4. DYSGERMINOMA

G11.WHAT WILL BE THE BMI OF A WOMAN WHOSE WEIGHT IS 89 KG AND HEIGHT IS


172 CM:

1 27.

2 30.

3 33.

4 36.

Q12. A 36-YEAR-OLD FACTORY WORKER DEVELOPED ITCHY, ANNULAR SCALY PLAQUES


IN BOTH GROINS.

APPLICATION OF A CORTICOSTEROID OINTMENT LED TO TEMPORARY RELIEF BUT THE


PLAQUES CONTINUED TO EXTEND AT THE PERIPHERY. THE MOST LIKELY DIAGNOSIS IS:
1 ERYTHEMA ANNULARE CENTRIFUGUM.

2 GRANULOMA ANNULARE.

3 ANNULAR LICHEN PLANUS.

4 TINEA CRURIS.

Q13. A 16-YEAR-OLD BOY PRESENTED WITH ASYMPTOMATIC, MULTIPLE,


ERYTHEMATOUS, ANNULAR LESIONS WITH A COLLARETTES OF SCALES AT THE PERIPHERY
OF THE LESIONS PRESENT ON THE TRUNK. THE MOST LIKELY DIAGNOSIS IS:

1 PITYRIASIS VERSICOLOR.

2 PITYRIASIS ALBA.

3 PITYRIASIS ROSEA.

4 PITYRIASIS RUBRA PILARIS.

Q14. A 40-YEAR-OLD WOMAN PRESENTS WITH A 2-YEAR HISTORY OF ERYTHEMATOUS


PAPULOPUSTULAR LESIONS ON THE CONVEXITIES OF THE FACE. THERE IS A BACKGROUND
OF ERYTHEMA AND TELANGIECTASIA. THE MOST LIKELY DIAGNOSIS IN THE PATIENT IS:

1 ACNE VULGARIS.

2 ROSSACEA.

3 SYSTEMIC LUPUS ERYTHEMATOSUS.

4 POLYMORPHIC LIGHT ERUPTION.

Q15. A 27-YEAR OLD SEXUALLY ACTIVE MALE DEVELOPS A VESICULOBULLOUS LESION


ON THE GLANS SOON AFTER TAKING TABLET PARACETAMOL FOR FEVER. THE LESION
HEALED WITH HYPERPIGMENTATION. THE MOST LIKELY DIAGNOSIS IS:

1 BEHCET€™S SYNDROME.

2 HERPES GENITALIS.
3 FIXED DRUG ERUPTION.

4 PEMPHIGUS VULGARIS.

PAEDIATRICS

PA1. IN NEONATAL SCREENING PROGRAMME FOR DETECTION OF CONGENITAL


HYPOTHYROIDISM, THE IDEAL PLACE AND TIME TO COLLECT THE BLOOD SAMPLE FOR TSH
ESTIMATION IS :

1. CORD BLOOD AT TIME OF BIRTH.

2. HEAL PAD BLOOD AT THE TIME OF BIRTH.

3. HEAL PAD BLOOD ON 4 DAY OF BIRTH.

4. PERIPHERAL VENOUS BLOOD ON 28 DAY.

PA2. A THREE YEARS OLD BOY PRESENTS WITH POOR URINARY STREAM. MOST LIKELY
CAUSE IS :

1. STRICTURE URETHRA.

2. NEUROGENIC BLADDER.

3. URETHRAL CALCULUS.

4. POSTERIOR URETRAL VALVES.

PA3. THE TREATMENT OF CHOICE FOR PRIMARY GRADE V VESICO-URETERIC REFLUX


INVOLVING BOTH KIDNEYS IN A 6 MONTH OLD BOY IS:

1. ANTIBIOTIC PROPHYLAXIS.

2. URETERIC REIMPLANTATION.

3. CYSTOSCOPY FOLLOWED BY SUBURETERIC INJECTION OF TEFLON.

4. BILATERAL URETEROSTOMIES.
PA4. REGARDING NEONATAL CIRCUMCISION, WHICH ONE OF THE FOLLOWING IS TRUE :

1. IT SHOULD BE DONE WITHOUT ANAESTHESIA , AS IT IS HAZARDOUS TO GIVE


ANAESTHESIA .

2. IT SHOULD BE DONE WITHOUT ANESTHESIA, AS NEONATES DO NOT PERCEIVE PAIN


AS ADULT.

3. IT SHOULD BE DONE UNDER LOCAL ANAESTHESIA ONLY.

4. GENERAL ANAESTHESIA SHOULD BE GIVEN TO NEONATE FOR CIRCUMCISION AS THEY


ALSO FEEL PAIN AS ADULT.

PA5. THE TREATMENT OF CHOICE IN ATTENTION DEFICIT HYPERACTIVITY DISORDER IS :

1. HALOPERIDOL.

2. IMIPRAMINE.

3. METHYLPHENIDATE.

4. ALPRAZOLAM.

PA6. HYPOCHLOREMIA HYPOKALEMIA AND ALKALOSIS ARE SEEN IN:

1. CONGENITAL HYPERTROPHIC PYLORIC STENOSIS

2. HIRSCHSPRUNG'S DISEASE.

3. ESOPHAGEAL ATRESIA

4. JEJUNAL ATRESIA

PA7. A NEWBORN BABY HAD NORMAL APGAR SCORE AT BIRTH AND DEVELOPED
EXCESSIVE FROTHING AND CHOKING ON ATTEMPTED FEEDS. THE INVESTIGATION OF
CHOICE IS:

1. ESOPHAGOSCOPY

2. BRONCHOSCOPY
3. MRI CHEST

4. X-RAY CHEST AND ABDOMEN WITH THE RED RUBBER CATHETER PASSED PER ORALLY
INTO ESOPHAGUS

PA8. A NEW BORN BABY HAS BEEN REFERRED TO THE CASUALTY AS A CASE OF
CONGENITAL DIAPHRAGMATIC HERNIA. THE FIRST CLINICAL INTERVENTION IS TO:

1. INSERT A CENTRAL VENOUS PRESSURE LINE.

2. BAG AND MASK VENTILATION

3. INSERT A NASOGASTRIC TUBE.

4. VENTILATE WITH HIGH FREQUENCY VENTILATOR

PA9.ONE YEAR OLD MALE CHILD IS PRESENTED WITH POOR URINARY STREAM SINCE
BIRTH. THE INVESTIGATION OF CHOICE FOR EVALUATION IS:

1. VOIDING CYSTOURETHROGRAPHY (VCUG)

2. USG BLADDER

3. INTRAVENOUS UROGRAPHY

4. UROFLOWMETRY

PA10. EISENMENGER SYNDROME IS CHARACTERIZED BY ALL EXCEPT:

1. RETURN OF LEFT VENTRICLE AND RIGHT VENTRICLE TO NORMAL SIZE.

2. PULMONARY VEINS NOT DISTENDED.

3. PRUNING OF PERIPHERAL PULMONARY ARTERIES.

4. DILATATION OF CENTRAL PULMONARY ARTERIES.

PA11. WHICH ONE OF THE FOLLOWING IS THE COMMON CAUSE OF CONGENITAL


HYDROCEPHALUS IS?

1 CRANIOSYNOSTOSIS.

2 INTRA UTERINE MENINGITIS

3 AQUEDUCTAL STENOSIS

4 MALFORMATIONS OF GREAT VEIN OF GALEN.

PA12. THE MOST COMMON MALIGNANT NEOPLASM OF INFANCY IS:

1 MALIGNANT TERATOMA.

2 NEUROBLASTOMA.

3 WILMS€™ TUMOR.

4 HEPATOBLASTOMA.

PA13. THE MOST COMMON PRESENTATION OF A CHILD WITH WILM€™S TUMOR IS:

1 AN ASYMPTOMATIC ABDOMINAL MASS.

2 HEMATURIA.

3 HYPERTENSION.

4 HEMOPTYSIS DUE TO PULMONARY SECONDARY.

PA14. THE PROTECTIVE EFFECTS OF BREAST MILK ARE KNOWN TO BE ASSOCIATED


WITH:

1 IG M ANTIBODIES.

2 LYSOZYME.

3 MAST CELLS.

4 IG A ANTIBODIES.
PA15. ALL OF THE FOLLOWING STATEMENTS ARE TRUE ABOUT CONGENITAL RUBELLA
EXCEPT:

1 IT IS DIAGNOSED WHEN THE INFANT HAS IGM ANTIBODIES AT BIRTH.

2 IT IS DIAGNOSED WHEN IGG ANTIBODIES PERSIST FOR MORE THAN 6 MONTHS.

3 MOST COMMON CONGENITAL DEFECTS ARE DEAFNESS, CARDIAC MALFORMATIONS


AND CATARACT.

4 INFECTION AFTER 16 WEEKS OF GESTATION RESULTS IN MAJOR CONGENITAL


DEFECTS.

PSYCHIATARIES

PS1. A MIDDLE AGED MAN PRESENTED WITH PAIN IN BACK, LACK OF INTEREST I
RECREATIONAL ACTIVITIES, LOW MOOD, LETHARGY, DECREASED SLEEP AND APPETITE
FOR TWO MONTHS. THERE WAS NO HISTORY SUGGESTIVE OF DELUSIONS OF
HALLUCINATIONS. HE DID NOT SUFFER FROM ANY CHRONIC MEDICAL ILLNESS. THERE
WAS NO FAMILY HISTORY OF PSYTCHIATRIC ILLNESS. ROUTINE INVESTIGATIONS
INCLUDING HAEMOGRAM, RENAL FUNCTION TESTS, LIVER FUNCTION TESTSM
ELECTROCADIOGRAM DID NOT REVEAL ANY ABNORMALITY. THIS PATIENT SHOULD BE
TREATED WITH :

1. HALOPARIDOL.

2. SERTRALINE.

3. ALPRAZOLAM.

4. OLANZAPINE.

PS2. THE FOLLOWING IS A SCHNEIDER'S FIRST RANK SYMPTOM :

1. PERSECUTORY DELUSION.

2. VOICES COMMENTING ON ACTIONS.

3. DELUSION OF GUILT.

4. INCOHERENCE.
PS3. AN ELDERLY HOUSE WIFE LOST HER HUSBAND WHO DIED SUDDENLY OF
MYOCARDIAL INFARCTION COUPLE OF YEARS AGO. THEY HAD BEEN STAYING ALONE FOR
ALMOST A DECADE WITH INFREQUENT VISITS FROM HER SON AND GRANDCHILDREN.
ABOUT A WEEK AFTER THE DEATH SHE HEARD HIS VOICE CLEARLY TALKING TO HER AS HE
WOULD IN A ROUTINE MANNER FROM THE NEXT ROOM. SHE WENT TO CHECK BUT SAW
NOTHING. SUBSEQUENTLY SHE OFTEN HEARD HIS VOICE CONVERSING WITH HER AND
SHE WOULD ALSO DISCUSS HER DAILY MATTERS WITH HIM. THIS HOWEVER, PROVOKED
ANXIETY AND SADNESS OF MOOD WHERE SHE WAS PREOCCUPIED WITH HIS THOUGHT.
SHE SHOULD BE TREATED WITH:

1. CLOMIPRAMINE.

2. ALPRAZOLAM.

3. ELECTROCONVULSIVE THERAPY.

4. HALOPERIDOL.

PS4. YAWNING IS A COMMON FEATURE OF

1. ALCOHOL WITHDRAWAL

2. COCAINE WITHDRAWAL

3. CANNABIS WITHDRAWAL

4. OPIOID WITHDRAWAL

PS5.A 40 YEAR OLD MALE, WITH HISTORY OF DAILY ALCOHOL CONSUMPTION FOR THE
LAST 7 YEARS, IS BROUGHT TO THE HOSPITAL EMERGENCY ROOM WITH ACUTE ONSET OF
SEEING SNAKES ALL AROUND HIM IN THE ROOM, NOT RECOGNIZING FAMILY MEMBERS,
VIOLENT BEHAVIOR AND TREMULOUSNESS FOR FEW HOURS. THERE IS HISTORY OF HIS
HAVING MISSED THE ALCOHOL DRINK SINCE 2 DAYS. EXAMINATION REVEALS INCREASED
BLOOD PRESSURE, TREMORS, INCREASED PSYCHOMOTOR ACTIVITY, FEARFUL AFFECT,
HALLUCINATORY BEHAVIOR, DISORIENTATION, IMPAIRED JUDGEMENT AND INSIGHT. HE
IS MOST LIKELY TO BE SUFFERING FROM:

1. ALCOHOLIC HALLUCINOSIS.
2. DELIRIUM TREMENS.

3. WERNICKE ENCEPHALOPATHY.

4. KORSAKOFF€™S PSYCHOSIS.

PS6. A 45 YEAR MALE WITH A HISTORY OF ALCOHOL DEPENDENCE PRESENTS WITH


CONFUSION, NYSTAGMUS AND ATAXIA. EXAMINATION REVEALS 6TH CRANIAL NERVE
WEAKNESS. HE IS MOST LIKELY TO BE SUFFERING FROM:

1. KORSAKOFF€™S PSYCHOSIS.

2. WERNICKE€™S ENCEPHALOPATHY.

3. DE CLERAMBAULT SYNDROME.

4. DELIRIUM TREMENS.

PS7. A 25YEAR OLD FEMALE PRESENTS WITH 2YEAR HISTORY OF REPETITIVE,


IRRESISTIBLE THOUGHTS OF CONTAMINATION WITH DIRT ASSOCIATED WITH REPETITIVE
HAND WASHING. SHE REPORTS THESE THOUGHTS TO BE HER OWN AND DISTRESSING;
BUT IS NOT ABLE TO OVERCOME THEM ALONG WITH MEDICATIONS. SHE IS MOST LIKELY
TO BENEFIT FROM WHICH OF THE FOLLOWING THERAPIES:

1. EXPOSURE AND RESPONSE PREVENTION.

2. SYSTEMATIC DESENSITIZATION.

3. ASSERTIVENESS TRAINING.

4. SENSATE FOCUSING.

PS8. AN 18 YEAR OLD BOY CAME TO THE PSYCHIATRY OPD WITH A COMPLAINT OF
FEELING CHANGED FROM INSIDE. HE DESCRIBED HIMSELF AS FEELING STRANGE AS IF HE
IS DIFFERENT FROM HIS NORMAL SELF. HE WAS VERY TENSE AND ANXIOUS YET COULD
NOT POINT OUT THE PRECISE CHANGE IN HIM. THIS PHENOMENA IS BEST CALLED AS:

1. DELUSIONAL MOOD.

2. DEPERSONALIZATION.
3. AUTOCHTHONOUS DELUSION.

4. OVER VALUED IDEA.

PS9. DRY MOUTH DURING ANTIDEPRESSANT THERAPY IS CAUSED BY BLOCKADE OF:

1. MUSCARINIC ACETYLCHOLINE RECEPTORS.

2. SEROTONERGIC RECEPTORS.

3. DOPAMINERGIC RECEPTORS.

4. GABA RECEPTORS.

PS10. AN 18-YEAR-OLD STUDENT COMPLAINS OF LACK OF INTEREST IN STUDIES FOR


LAST 6 MONTHS. HE HAS FREQUENT QUARRELS WITH HIS PARENTS AND HAS FREQUENT
HEADACHES. THE MOST APPROPRIATE CLINICAL APPROACH WOULD BE TO:

1. LEAVE HIM AS NORMAL ADOLESCENT PROBLEM.

2. RULE OUT DEPRESSION.

3. RULE OUT MIGRAINE.

4. RULE OUT AN OPPOSITIONAL DEFIANT DISORDER.

PS11. PRESERVATION IS:

1. PERSISTENT AND INAPPROPRIATE REPLETION OF THE SAME THOUGHTS.

2. WHEN A PATIENT FEELS VERY DISTRESSED ABOUT IT.

3. CHARACTERISTIC OF SCHIZOPHRENIA.

4. CHARACTERISTIC OF OBSESSIVE COMPULSIVE DISORDER (OCD)

PS12. ONE OF THE FOLLOWING USUALLY DIFFERENTIATES HYSTERICAL SYMPTOMS


FROM HYPOCHONDRIACAL SYMPTOMS:
1. SYMPTOMS DO NOT NORMALLY REFLECT UNDERSTANDABLE PHYSIOLOGICAL OR
PATHOLOGICAL MECHANISMS.

2. PHYSICAL SYMPTOMS ARE PROMINENT WHICH ARE NOT EXPLAINED BY ORGANIC


FACTORS.

3. PERSONALITY TRAITS ARE SIGNIFICANT.

4. SYMPTOMS RUN A CHRONIC COURSE.

Q13. .55 IN WHICH OF THE FOLLOWING CONDITIONS SPLENECTOMY IS NOT USEFUL?

1 HEREDITARY SPHEROCYTOSIS.

2 PORPHYRIA.

3 THALASSEMIA.

4 SICKLE CELL DISEASE WITH LARGE SPLEEN.

Q14. THE FOLLOWING IS IDEAL FOR THE TREATMENT WITH INJECTION OF SCLEROSING
AGENTS.

1 EXTERNAL HEMORRHOIDS.

2 INTERNAL HEMORRHOIDS.

3 PROLAPSED HEMORRHOIDS.

4 STRANGULATED HEMORRHOIDS.

Q15. IN AN ADULT MALE, ON PER RECTAL EXAMINATION, THE FOLLOWING


STRUCTURES CAN BE FELT ANTERIORLY EXCEPT:

1 INTERNAL ILIAC LYMPH NODES.

2 BULB OF THE PENIS.

3 PROSTATE.

4 SEMINAL VESICLE WHEN ENLARGED.


23. 37 WEEKS OF PREGNANCY, WITH VAGINAL BLEEDING OF 900ML. HAEMOGLOBIN

DECREASED (6.8) BP 130/80. EXAMINATION UTERUS IS TENDER AND FETAL DEAD IS

OBSERVED. MANAGEMENT SHOULD INCLUDE ALL THE FOLLOWING ****EXCEPT:

A. US

B. ANTICOAGULATION PROFILE

C. IF RH NEGATIVE, GIVE HAEMOGLOBIN

D. BLOOD TRANSFUSION AND IMMEDIATE SURGERY

E. OXYTOCIN INFUSION****

DR.SANDRA, FEB 10, 2007

#17

DR.SANDRA

DR.SANDRA GUEST

Q1. WHAT IS THE DRUG OF CHOICE TO CONTROL SUPRAVENTRICULAR TACHYCARDIA?

A) ADENOSINE****

B) PROPRANOLOL

C) VERAPAMIL

D) DIGOXIN

Q2. A 5 YEAR OLD CHILD IS RUSHED TO CASUALTY REPORTEDLY ELECTROCUTED WHILE


PLAYING IN A PARK. THE CHILD IS APNEIC AND IS VENTILATED WITH BAG MASK. THERE
ARE BURNS ON EACH HAND. WHAT WILL BE THE NEXT STEP IN THE MANAGEMENT?
A) CHECK PULSES

B) START CHEST COMPRESSIONS****

C) INTUBATE

D) CHECK OXYGEN

* FIRST, REMOVE THE PATIENT FROM THE CIRCUIT.

* PATIENTS WHO ARE IN ARREST THEN REQUIRE BASIC AND ADVANCED CARDIAC LIFE-
SUPPORT REGIMENS. IN ELECTRICALLY INDUCED ARREST, NO UNDERLYING DISEASE
CAUSED THE ARREST. THEREFORE, PROTRACTED EFFORTS OF RESUSCITATION ARE MET
WITH SUCCESS MORE OFTEN THAN WITH DISEASE-CAUSED ARREST.

* PATIENTS WHO ARE UNCONSCIOUS BUT NOT IN ARREST REQUIRE CAREFUL


VENTILATORY OBSERVATION AND ASSISTANCE, IF INDICATED.

* PATIENTS WITH BURNS ABOVE THE NECK REQUIRE SUPPLEMENTAL OXYGEN AND
INTUBATION BECAUSE OF THE HIGH PROBABILITY OF AIRWAY AND LUNG DAMAGE.

* SECONDARY BLUNT TRAUMA OFTEN IS ENCOUNTERED OWING TO FALLS CAUSED BY


INVOLUNTARY MUSCULAR CONTRACTION. IT IS DEALT WITH IDENTICALLY TO ANY OTHER
BLUNT TRAUMA.

Q3. RECURRENT RESPIRATORY TRACT INFECTIONS MAY OCCUR IN ALL OF THE


FOLLOWING EXCEPT:

A) VENTRICULAR SEPAL DEFECT

B) TETROLOGY OF FALLOT****
C) TRANSPOSITION OF GREAT ARTERIES

D) TOTAL ANOMALOUS VENOUS RETURN

IN VSD :FREQUENT RESPIRATORY INFECTIONS MAY OCCUR SECONDARY TO THE


PULMONARY CONGESTION.

SO IN TF WE DON'T HAVE PULMONARY CONGESTION. AS A RESULT I THINK THE RIK OF


RESPIRATORY INFECTION IS LOW, BUT I AM NOT SURE

Q4. A 6 MONTH OLD BOY WEIGHTING 3.2 KG PRESENTS WITH RECURRENT VOMITING
AND POLYURIA INVESTIGATIONS SHOW BLOOD UREA 60 MG/DL CREATININE 0.7 MG/DL,
CALCIUM 12.8 MG/DL, PHOSPHATE 3 MG/DL, PH 7.45, BICARBONATE 25 MEQ/L AND PTH
140 PG/ML (NORMAL<60 PG/ML). DAILY URINARY CALCIUM EXCRETION IS REDUCED.
ULTRASOUND ABDOMEN SHOW BILATERAL NEPHROCALCINOSIS. THE MOST LIKELY
DIAGNOSIS IS:

A) BARTTER SYNDROME

B) MUTATION OF THE CALCIUM SENSING RECEPTOR.

C) PSEUDO-PSEUDOHYPOPARATHYROIDISM

D) PARATHYROID ADENOMA****

Q5. A FEMALE CHILD HAS RECENTLY LEARNED TO EAT WITH SPOON WITHOUT SPILLING;
TO DRESS AND UNDRESS HERESELF WITH SUPERVISION; AND TO UNDERSTAND THAT SHE
IS A GIRL. THESE SKILLS ARE FIRST MASTERED BETWEEN THE AGES OF:

A) 2 & 3 YEARS

B) 3 & 4 YEARS****

C) 4 & 5 YEARS

D) 5 & 6 YEARS

DR.SANDRA, FEB 10, 2007


#18

DR.SANDRA

DR.SANDRA GUEST

1. 37 WEEKS OF PREGNANCY, WITH VAGINAL BLEEDING OF 900ML. HAEMOGLOBIN

DECREASED (6.8) BP 130/80. EXAMINATION *UTERUS IS TENDER AND FETAL DEAD IS

OBSERVED. MANAGEMENT SHOULD INCLUDE ALL THE FOLLOWING ****EXCEPT:

A. US

B. ANTICOAGULATION PROFILE

C. IF RH NEGATIVE, GIVE HAEMOGLOBIN

D. BLOOD TRANSFUSION AND IMMEDIATE SURGERY

E. OXYTOCIN INFUSION****

THIS Q. GOES IN FAVOUR OF ABRUBTIO PLACENTA. BECAUSE PLACENTA PRAEVIA IS


PAINLESS AND THERE IS NO OTHER HISTORY. IN CASE OF UT RUPTURE, RISK FACTORS ARE-
- PREVIOUS C/S, OBS . LABOUR, HIGH FORCEPS DELIVERY, INTERNAL VERSION, BREECH
EXTRACTION AND MOTHER MAY PRESENT AS UNEXPLAINED TACYCARDIA, SUDDEN
MATERNAL SHOCK, CESSATION OF CONTRACTIONS, DISAPPEARANCE OF PRESENTING
PART, FETAL DISTRESS, CONTINUOUS PPH WITH A WELL-CONTRACTED UT. WE CAN DO
ALL ACCORDING TO THIS Q. IN CASE OF ABRUPTIO PLACENTA EXCEPT IMMEDIATE
SURGERY BUT WE NEED TO RESUSCITATION TO MOTHER. FETAL DEATH BEST
MANAGEMENT IS MATERNAL RESUSCITATION FOLLOWED BY INDUCTION OF LABOUR,
AIMING FOR A VAGINAL DELIVERY. IF FETUS IS ALIVE, IMMEDIATE DELIVERY BY C/S.

DR.SANDRA, FEB 10, 2007

#19
DR.SANDRA

DR.SANDRA GUEST

Q1. A PATIENT PRESENTS WITH LOWER GASTROINTESTIANAL BLEED. SIGMOIDOSCOPY


SHOWS ULCERS IN THE SIGMOID. BIOPSY FROM THIS AREA SHOWS ULCERS IN THE
SIGMOID. BIOPSY FROM THIS AREA SHOWS FLASKSHAPED ULCERS. WHICH OF THE
FOLLOWING IS THE MOST APPROPRIATE TREATMENT ?

A) INTRAVENOUS CEFTRIAXONE

B) INTRAVENOUS METRONIDAZOLE ***

C) INTRAVENOUS STEROIDS AND SULPHASALAZINE

D) HYDROCORTISONE ENEMA

Q2. A 25-YEAR OLD FARMER PRESENTED WITH HISTORY OF HIGH GRADE FEVER FOR 7
DAYS AND ALTERED SENSORIUM FOR 2 DAYS. ON EXAMINATION, HE WAS COMATOSED
AND HAD CONJUNCTIVAL HE4MORRHAGE. URGENT INVESTIGATIONS SHOWED A
HEMOGLOBIN OF 11 GM/DL, SERUM BILIRUBIN 8 MG/DL AND UREA 78 MG/DL.
PERIPHERAL BLOOD SMEAR WAS NAGATIVE FOR MALARIAL PARASITE. WHAT IS THE
MOST LIKELY DIAGNOSIS ?

A) BRUCELLOSWIS

B) WEIL'S DISEASE ***

C) ACUTE VIRAL HEPATITIES

D) Q FEVER

Q3. A 50-YEAR OLD LADY PRESENTED WITH HISTORY OF PAIN UPPER ABDOMEN,
NAUSEA AND DECREASED APPETITE FOR 5 DAYS. SHE HAD UNDERGONE
CHOLECYSTECTOMY 2 YEARS BACK. HER BILIRUBIN WAS 10 MG/DL, SGOT 900 IU/I SGPT
700 IU/I AND SERUM ALKALINE PHOPHATASE WAS 280 IU/I. WHAT IS THE MOST LIKELY
DIAGNOSIS ?
A) ACUTE PANCREATITIS

B) ACUTE CHOLAGITS

C) ACUTE VIRAL HEPATITIS ***

D) POSTERIOR PENETRATION OF PEPTIC ULCER

Q4. A 70- YEAR OLD MALE PATIENT PRESENTED TO THE EMERGENCY DEPARTMENT
WITH PAIN IN EPIGASTRIUM AND DIFFICULTY IN BREATHING FOR 6 HOURS. ONE
EXAMINATION, HIS HEART RATE WAS *56 PER MINUTE AND THE BLOOD PRESSURE WAS
106/60 MM HG. CHEST EXAMINATION WAS NORMAL. THE PATIENT HAS BEEN TAKING
OMEPRAZOLE FOR GASTROESOPHAGEAL REFLUX DISEASE FOR LAST 6 MONTHS. WHAT
SHOULD BE THE INITIAL INVESTIGATION ?

A) AN ECG ANSWER INF WALL MI***

B) AN UPPER GI ENDOSCOPY

C) URGENT ULTRASOUND OF THE ABDOMEN

D) AN X-RAY CHEST

Q5. A 30-YEAR OLD DELIVERED A HEALTHY BABY AT 37 WEEK OF GESTATION. SHE WAS
A KNOWN CASE OF CHRONIC HEPATITIS B INFECTION. SHE WAS POSITIVE FOR HBS AG BUT
NEGATIVE FOR HBEAG. WHICH OF THE FOLLOWING IS THE MOST APPROPRIATE
TREATMENT FOR THE BABY ?

A) BOTH ACTIVE AND PASSIVE IMMUNIZATION SOON AFTER BIRTH.****

B) PASSIVE IMMUNIZATION SOON AFTER BIRTH AND ACTIVE IMMUNIZATION AT 1 YEAR


OF AGE.

C) ONLY PASSIVE IMMUNIZATION SOON AFTER BIRTH.


D) ONLY ACTIVE IMMUNIZATION SOON AFTER BIRTH

DR.SANDRA, FEB 10, 2007

#20

DR.JOVA.

DR.JOVA. GUEST

A 17 YEAR OLD HIGH SCHOOL SENIOR COMES TO THE CLINIC BECAUSE OF PAIN DURING
HER MENSES FOR THE PAST 2 YEARS. SHE DESCRIBES THE PAIN AS CRAMPY, FELT IN THE
LOWER ABDOMEN AND RADIATING TO THE BACK. SHE SAYS IT IS MOST SEVERE THE
SECOND DAY OF THE FLOW AND SUBSIDES THEREAFTER. HER MEDICAL HISTORY SHOWS
THAT MENARCHE WAS AT AGE 14 YEARS. PHYSICAL EXAMINATION IS NORMAL. THE MOST
APPROPRIATE NEXT STEP IS TO

A) BEGIN A THERAPEUTIC TRIAL OF ASPIRIN

B) BEGIN A THERAPEUTIC TRIAL OF IBUPROFEN

C) DETERMINE SERUM CA-125 CONCENTRATION

D) DO DIAGNOSTIC LAPAROSCOPY

E) DO A DILATATION AND CURETTAGE

VIEW ANSWER B: BEGIN A THERAPEUTIC TRIAL OF IBUPROFEN

DYSMENORRHEA, "PAINFUL MENSTRUATION" IS THE CHIEF COMPLAINT OF THIS

17 Y/O PT. SHE EXPERIENCES A TYPICALLY


LOWER ABDOMINAL PAIN CONSISTENTLY ON THE SECOND DAY OF MENSTRUATION,
AND

APPARENTLY AT NO OTHER INTERVAL. THE

PAIN IRRADIATION TO THE BACK IS PROBABLY A "RED HERRING" AS NOTHING ELSE

IN THE HX SUGGEST OTHERWISE.

OTHER REMOTE POSSIBILITES ON A DIFFERENTIAL DX OF DYSMENORRHEA

(SECONDARY):

* CHRONIC PID - PT NOT SEXUALLY ACIVE (?)

* INTRAUTERINE DEVICE

* LEIOMYOMAS - PT IS TOO YOUNG

* ENDOMETRIOSIS - " "

- INTERESTINGLY ENOUGH THIS PT'S MENARCHE WAS 2 YEARS AGO, PRIMARY

DYSMENORRHEA USUALLY PRESENTS 2 YEARS AFTER

THE ONSET OF MENSES.

*AN NSAID TRIAL COULD PROVE BENEFICIAL SINCE THE CAUSE FOR THIS TYPE OF

"PAINFUL MENST" IS THOUGHT TO BE SECONDARY

TO PG-INDUCED MYOMETRIAL CONTRACTIONS.

ESTROGEN THERAPY IS AN ALTERNATIVE, ALTHOUGH BEGINNING IBUPROFEN A FEW

DAYS BEFORE THE ANTICIPATED MENSES CAN OFTEN HELP.

IF THE PT WERE OLDER THE ENDOMETRIOSIS (A KNOWN CAUSE OF 2ARY

DYSMENORRHEA) WOULD HAVE BEEN GIVEN GREATER CONSIDERATION, AND

PARTICULARLY IF SHE HAD A HX OF "INFERTILITY."


- CA-125 IS OFTEN A "FALSE +" IN PRE-MENOPAUSAL WOMEN !!

DR.JOVA., JUN 27, 2008

#21

DR.JOVA.

DR.JOVA. GUEST

12 YEAR OLD GIRL IS REFERRED TO THE CLINIC BY THE SCHOOL NURSE FOR EVALUATION
OF SCOLIOSIS. HER SCOLIOSIS WAS DETECTED DURING A ROUTINE SCREENING
EXAMINATION AT THE SCHOOL AND APPEARS MILD (CURVE LESS THAN 10 DEGREES). SHE
IS ATHLETIC AND IS OTHERWISE IN GOOD HEALTH. DURING THE PHYSICAL EXAMINATION,
PARTICULAR ATTENTION SHOULD BE GIVEN TO:

A) ARM LENGTH

B) BLOOD PRESSURE

C) BODY WEIGHT

D) CARDIAC EXAMINATION

E) STAGE OF PUBERTAL DEVELOPMENT

VIEW ANSWER E: STAGE OF PUBERTAL DEVELOPMENT

THE TERM SCOLIOSIS DENOTES LATERAL CURVATURE OF THE SPINE, WHICH IS ALWAYS
ASSOCIATED WITH SOME ROTATION OF THE INVOLVED VERTEBRAE. SCOLIOSIS IS
CLASSIFIED BY ITS ANATOMIC LOCATION, IN EITHER THE THORACIC OR LUMBAR SPINE,
WITH RARE INVOLVEMENT OF THE CERVICAL SPINE. THE CONVEXITY OF THE CURVE IS
DESIGNATED RIGHT OR LEFT. THUS, A RIGHT THORACIC SCOLIOSIS WOULD DENOTE A
THORACIC CURVE IN WHICH THE CONVEXITY IS TO THE RIGHT; THIS IS THE MOST
COMMON TYPE OF IDIOPATHIC CURVE. POSTERIOR CURVATURE OF THE SPINE (KYPHOSIS)
IS NORMAL IN THE THORACIC AREA, ALTHOUGH EXCESSIVE CURVATURE MAY BECOME
PATHOLOGIC. ANTERIOR CURVATURE IS CALLED LORDOSIS AND IS NORMAL IN THE
LUMBAR AND CERVICAL SPINES. IDIOPATHIC SCOLIOSIS GENERALLY BEGINS AT ABOUT 8
OR 10 YEARS OF AGE AND PROGRESSES DURING GROWTH. IN RARE INSTANCES,
INFANTILE SCOLIOSIS MAY BE SEEN IN CHILDREN 2 YEARS OF AGE OR LESS.

IDIOPATHIC SCOLIOSIS IS ABOUT FOUR OR FIVE TIMES MORE COMMON IN GIRLS THAN
IN BOYS. THE DISORDER IS USUALLY ASYMPTOMATIC IN THE ADOLESCENT YEARS, BUT
SEVERE CURVATURE MAY LEAD TO IMPAIRMENT OF PULMONARY FUNCTION IN LATER
YEARS. IT IS IMPORTANT TO EXAMINE THE BACK OF ANY ADOLESCENT COMING IN FOR A
ROUTINE PHYSICAL EXAMINATION IN ORDER TO IDENTIFY SCOLIOSIS EARLY. THE
EXAMINATION IS PERFORMED BY HAVING THE PATIENT BEND FORWARD 90 DEGREES
WITH THE HANDS JOINED IN THE MIDLINE. AN ABNORMAL FINDING CONSISTS OF
ASYMMETRY OF THE HEIGHT OF THE RIBS OR PARAVERTEBRAL MUSCLES ON ONE SIDE,
INDICATING ROTATION OF THE TRUNK ASSOCIATED WITH LATERAL CURVATURE.

DISEASES THAT MAY BE ASSOCIATED WITH SCOLIOSIS INCLUDE NEUROFIBROMATOSIS,


MARFAN'S SYNDROME, CEREBRAL PALSY, MUSCULAR DYSTROPHY, POLIOMYELITIS, AND
MYELODYSPLASIA. NEUROLOGIC EXAMINATION SHOULD BE PERFORMED IN ALL
CHILDREN WITH SCOLIOSIS TO DETERMINE WHETHER THESE DISORDERS ARE PRESENT.

FIVE TO 7% OF CASES OF SCOLIOSIS ARE DUE TO CONGENITAL VERTEBRAL ANOMALIES


SUCH AS A HEMIVERTEBRAL OR UNILATERAL VERTEBRAL BRIDGE. THESE CURVES ARE
MORE RIGID THAN THE MORE COMMON IDIOPATHIC CURVE (SEE BELOW) AND WILL
OFTEN INCREASE WITH GROWTH, ESPECIALLY DURING ADOLESCENCE.

EIGHTY PERCENT OF SCOLIOSIS IS IDIOPATHIC. SINCE 30% OF FAMILY MEMBERS ARE


ALSO AFFECTED, SIBLINGS OF AN AFFECTED CHILD SHOULD BE EXAMINED.

IDIOPATHIC INFANTILE SCOLIOSIS, OCCURRING IN CHILDREN 2-4 YEARS OF AGE, IS


QUITE UNCOMMON IN THE UNITED STATES; IT IS MORE COMMON IN GREAT BRITAIN. IF
THE CURVATURE IS LESS THAN 30 DEGREES, THE PROGNOSIS IS EXCELLENT, AS 70%
RESOLVE SPONTANEOUSLY. IF THE CURVATURE IS MORE THAN 30 DEGREES, THERE MAY
BE PROGRESSION, AND THE PROGNOSIS IS THEREFORE GUARDED.
POSTURAL COMPENSATION OF THE SPINE MAY LEAD TO LATERAL CURVATURE FROM
SUCH CAUSES AS UNEQUAL LENGTH OF THE LOWER EXTREMITIES. SCIATIC SCOLIOSIS MAY
RESULT FROM PRESSURE ON THE SPINAL CORD OR ROOTS BY INFECTIOUS PROCESSES OR
HERNIATION OF THE NUCLEUS PULPOSUS; THE UNDERLYING CAUSE MUST BE SOUGHT.
THE CURVATURE WILL RESOLVE AS THE PRIMARY PROBLEM IS TREATED.

TREATMENT OF SCOLIOSIS DEPENDS ON CURVE MAGNITUDE, SKELETAL MATURITY,


AND RISK OF PROGRESSION. CURVATURES OF LESS THAN 20 DEGREES USUALLY DO NOT
REQUIRE TREATMENT UNLESS THEY SHOW PROGRESSION

DR.JOVA., JUN 27, 2008

#22

DR.JOVA.

DR.JOVA. GUEST

WHICH OF THE FOLLOWING TUMOR CHARACTERISTICS CONFERS A POOR PROGNOSIS


IN PATIENTS WITH BREAST CANCER?

A. ESTROGEN RECEPTOR-POSITIVE

B. GOOD NUCLEAR GRADE

C. LOW PROPORTION OF CELLS IN S-PHASE

D. OVEREXPRESSION OF ERBB2 (HER-2/NEU)

E. PROGESTERONE RECEPTOR-POSITIVE

THE ANSWER IS D.
PATHOLOGIC STAGING REMAINS THE MOST IMPORTANT DETERMINANT OF OVERALL
PROGNOSIS. OTHER PROGNOSTIC FACTORS HAVE AN IMPACT ON SURVIVAL AND THE
CHOICE OF THERAPY. TUMORS THAT LACK ESTROGEN AND/OR PROGESTERONE
RECEPTORS ARE MORE LIKELY TO RECUR. THE PRESENCE OF ESTROGEN RECEPTORS,
PARTICULARLY IN POSTMENOPAUSAL WOMEN, IS ALSO AN IMPORTANT FACTOR IN
DETERMINING ADJUVANT CHEMOTHERAPY. TUMORS WITH A HIGH GROWTH RATE ARE
ASSOCIATED WITH EARLY RELAPSE. MEASUREMENT OF THE PROPORTION OF CELLS IN S-
PHASE IS A MEASURE OF THE GROWTH RATE. TUMORS WITH MORE THAN THE MEDIAN
NUMBER OF CELLS IN S-PHASE HAVE A HIGHER RISK OF RELAPSE AND AN IMPROVED
RESPONSE RATE TO CHEMOTHERAPY. HISTOLOGICALLY, TUMORS WITH A POOR NUCLEAR
GRADE HAVE A HIGHER RISK OF RECURRENCE THAN DO TUMORS WITH A GOOD NUCLEAR
GRADE. AT THE MOLECULAR LEVEL, TUMORS THAT OVEREXPRESS ERBB2 (HER-2/NEU) OR
THAT HAVE A MUTATED P53 GENE PORTEND A POORER PROGNOSIS FOR PATIENTS. THE
OVEREXPRESSION OF ERBB2 IS ALSO USEFUL IN DESIGNING OPTIMAL TREATMENT
REGIMENS, AND A HUMAN MONOCLONAL ANTIBODY TO ERBB2 (HERCEPTIN) HAS BEEN
DEVELOPED

DR.JOVA., JUN 27, 2008

#23

DR.JOVA.

DR.JOVA. GUEST

STEM CELL GENE THERAPY HAS BEEN ATTEMPTED IN PATIENTS WITH A VARIETY OF
MEDICAL CONDITIONS. TO DATE, THIS TECHNIQUE HAS HAD THE MOST CLINICAL SUCCESS
IN WHICH OF THE FOLLOWING CONDITIONS?

A. THALASSEMIA

B. GAUCHER DISEASE

C. FANCONI'S ANEMIA

D. SEVERE COMBINED IMMUNE DEFICIENCY (SCID)


E. WISCOTT-ALDRICH SYNDROME (WAS)

THE ANSWER IS D.

STEM CELL THERAPY HAS HELD ENORMOUS PROMISE FOR THE LAST TWO DECADES.
STEM CELLS ARE UNDIFFERENTIATED PROGENITORS THAT CAN DEVELOP INTO HIGHLY
SPECIALIZED CELLS THAT FORM THE VARIOUS ORGANS. TOTIPOTENT STEM CELLS CAN
FORM A PLACENTA AND CAN DEVELOP INTO A COMPLETE EMBRYO. PLURIPOTENT STEM
CELLS ARE CAPABLE OF FORMING TISSUES DERIVED FROM THE THREE MAJOR GERM-LINE
LAYERS: ENDODERM, MESODERM, AND ECTODERM. MULTIPOTENT STEM CELLS ARE THE
PROGENITORS OF CELLS IN PARTICULAR TISSUES. STEM CELLS ARE SELF-RENEWING AND
MAY DIFFERENTIATE. THE CONCEPT OF CORRECTING INBORN GENETIC DEFECTS IN STEM
CELLS HAS BEEN THE CENTRAL THEME IN MOST GENE THERAPY STRATEGIES FOR
CORRECTING CONGENITAL DISORDERS. THE SUCCESS OF BONE MARROW
TRANSPLANTATION IN THE TREATMENT OF MANY CONGENITAL AND ACQUIRED
HEMATOPOIETIC DISORDERS HAS STIMULATED NUMEROUS ATTEMPTS AT GENE THERAPY
FOR THOSE DISORDERS. A VARIETY OF METHODS HAVE BEEN DEVELOPED TO TRANSFER
NEW GENETIC MATERIAL INTO CELLS. DIRECT INJECTION OF DNA AND GENE TRANSFER
USING VIRUSES ARE TWO OF THE MAIN METHODS. ADENOVIRAL AND ADENOVIRUS-
ASSOCIATED VIRUSES ARE EFFECTIVE VECTORS BUT DO NOT INTEGRATE INTO THE
GENOME AND DO NOT OFTEN PRODUCE CONTINUED TRANSCRIPTION. RETROVIRAL AND
LENTIVIRAL VECTORS HAVE BEEN MODIFIED TO REMOVE THE PATHOGENIC SEQUENCES,
LEAVING ONLY THE SEQUENCES THAT ARE IMPORTANT FOR INTEGRATION. IN 2000, THE
FIRST SUCCESSFUL STEM CELL GENE THERAPY WAS REPORTED. CORD BLOOD CD34+ STEM
CELLS FROM X-SCID NEWBORNS WERE TRANSDUCED WITH A C RETROVIRUS VECTOR.
APPROXIMATELY [SNIP]% OF CIRCULATING T LYMPHOCYTES CONTAINED THE VECTOR IN
TRANSDUCED PATIENTS, COMPARED WITH LESS THAN 0.1% IN MYELOID CELLS,
REPRESENTING SELECTIVE EXPRESSION. MANY OF THE PATIENTS RESPONDED TO CLINICAL
VACCINATIONS AND HAVE REMAINED HEALTHY. HOWEVER, TWO RECIPIENTS DEVELOPED
ACUTE HEMATOLOGIC DISORDERS RESEMBLING LEUKEMIA, INDICATING ABERRANT
INTEGRATION OF THE VECTOR INTO THE PATIENT'S GENOME. FURTHER STUDIES HAVE
BEEN SUSPENDED AS RESEARCHERS ATTEMPT TO DISCOVER WAYS TO AVOID RANDOM
INTEGRATION OF VECTORS INTO SITES THAT ARE ONCOGENIC.
DR.JOVA., JUN 27, 2008

#24

DR.JOVA.

DR.JOVA. GUEST

THE TUMOR THAT MOST COMMONLY METASTASIZES TO THE HEART IS

A. MELANOMA

B. NON-HODGKIN'S LYMPHOMA

C. HODGKIN'S LYMPHOMA

D. BREAST

E. RENAL CELL

THE ANSWER IS A.

MELANOMA IS THE TUMOR MOST LIKELY TO METASTASIZE TO THE HEART, ALTHOUGH


THE MOST COMMON PRIMARIES ORIGINATING FROM TUMORS OF THE HEART ARE
BREAST AND LUNG OWING TO THE HIGH INCIDENCE OF THESE TUMORS

DR.JOVA., JUN 27, 2008

#25

DR.JOVA.

DR.JOVA. GUEST
AN 84 YEAR OLD WOMAN IS BROUGHT TO THE OFFICE BY HER DAUGHTER, WHO IS
YOUR PATIENT. THE MOTHER HAS JUST MOVED IN WHITH THE FAMILY BECAUSE SHE CAN
NO LONGER TAKE CARE FOR HERSELF DUE TO PROGRESSIVE, LONG-STANDING DEMENTIA.
THE DAUGHTER HOPES YOU WILL HELP TAKE CARE OF HER MOTHER. ON PHYSICAL
EXAMINATION THE MOTHER HAS NO EVIDENCE OF ANY OTHER CHRONIC DISEASE. SHE
DOES NOT RESPOND TO YOUR WORDS OR TO THE PHYSICAL EXAMINATION. YOU NOTICE
THAT SHE SMELLS OF URINE. ON EXAMINATION OF THE PELVIS THERE IS A DIFFUSE
ERYTHEMATOUS RASH EXTENDING OVER THE PRINEUM TO THE MEDIAL PAGES
BILATERALLY. YOU SUSPECT THE RASH RELATES TO URINARY INCONTINENCE. THE
DAUGHTER IS PRESENT AT THE MOTHER'S EXAMINATION. THE BEST OPENING REMARK TO
THE DAUGHTER IS:

A) "YOU SHOULD TAKE BETTER CARE OF YOUR MOTHER"

B) "HOW LONG HAVE YOU LEFT YOUR MOTHER IN THIS CONDITION?"

C) "YOUR MOTHER NEEDS TO WEAR DIAPERS"

D) "THIS RASH SHOULD RESPOND TO CLEANSING WITH MILD SOAP AND DRYING WITH A
CLEAN TOWEL THREE TIMES A DAY"

E) "THIS IS A RASH CAUSED BY URINE. MY NURSE WLL INSERT A FOLEY CATHETER"

ANSWER D: "THIS RASH SHOULD RESPOND TO CLEANSING..."

AMC QUESTION BANK CLINICALS/MCQS

3 JUNE 2006, MELBOURNE ( RETEST)

QUESTION 1:
A YOUNG STUDENT HAS A HEART MURMUR COMING AND GOING, HE COMES TO YOU
AS A GP FOR DIAGNOSIS. HE HAS NO CARDIAC SYMPTOMS, NO SOB. EVERYTHING IS
NORMAL, NO FAMILY HISTORY OF HEART PROBLEM. GENERAL HEALTH IS FINE.

TASKS:

1. DO A CARDIOVASCULAR EXAMINATION. ( PLEASE DO NOT COVER ABDOMINAL


EXAMINATION AND LOWER LIMBS EXAMINATION.)

2. PRESENT WHAT YOU HAVE FOUND TO YOUR EXAMINER.

3. PRESENT YOUR DIAGNOSIS TO YOUR EXAMINER AND ANSWER EXAMINER.

MY APPROACH: DO THE PHYSICAL EXAMINATION FOLLOWING THE TEXTBOOK. IT WAS A


MEDICAL STUDENT WITH NO MURMUR AT ALL. QUESTIONS FROM EXAMINER:

1. WHAT OTHER PHYSICAL EXAMINATION DO YOU WANT TO DO? ( I STOPPED AFTER I


AUSCULTATED ROLE PLAYER.) VALSAVA EXAMINATION.

2. IF A PATIENT HAS A MURMUR COMING AND GOING, WHAT DO YOU THINK? €“
FUNCTIONAL MURMUR?

QUESTION 2:

A YOUNG STUDENT PRESENTS WITH DRY COUGH, MOSTLY IN THE MORNING OR AT


NIGHT, GENERAL HEALTH OK, NON SMOKER.

TASKS:

1. DO A RESPIRATORY EXAMINATION,

2. REPORT YOUR FINDING TO YOUR EXAMINER.

3. ANSWER EXAMINER€™S QUESTION.


MY APPROACH:

AGAIN, FOLLOW THE TEXTBOOK€™S INSTRUCTION. IT WAS A MEDICAL STUDENT WITH


NO POSITIVE FINDING. I TOLD THE EXAMINER THAT THERE WAS NO POSITIVE FINDING.
THEN SHE ASKED ME WHAT WAS THE DIAGNOSIS- ASTHMA.

IMMEDIATELY SHE ASKED ME WHAT ELSE I WANTED TO DO. I SAID SPIROMETRY.

SHE HANDED OVER A SPIROMETRY RESULT: FEV1 DECREASE, FVC DECREASE, AFTER
BRONCHODILATOR, FEV1 INCREASE BUT LOWER THAN NORMAL, FVC INCREASE BUT
LOWER THAN NORMAL. I SAID IT WAS CONSISTENT WITH THE DIAGNOSIS WITH ASTHMA.

QUESTION 3: (SAME QUESTION FROM APRIL MELBOURNE 2006)

A YOUNG LADY DRIVER HAS A FEW CAR ACCIDENTS BECAUSE SHE CAN NOT SEE THE
CAR COMING FROM THE INTERSECTION.

PLEASE DO A VISUAL EXAMINATION.

TELL YOUR EXAMINER WHAT IS YOUR FINDING.

MAKE A DIAGNOSIS AND MANAGEMENT PLAN.

PLEASE NOTE THAT MRI IS THE GOLD STANDARD OF DIAGNOSIS.

QUESTION 4: (SAME QUESTION FROM NOVEMBER, 2005, MELBOURNE)

YOU ARE AN ED INTERN, A 45 YEARS OLD MAN PRESENTED WITH RIGHT UPPER
QUADRANT ABDOMINAL PAIN FOR 6 HOURS, NAUSEA BUT NO VOMITING, THIS IS THE
FIRST ATTACK.

ON EXAMINATION: TEMPERATURE 38.3 C, OTHER VITAL SIGNS NORMAL.

1. PLEASE EXPLAIN TO YOUR PATIENT WHAT IS THE MOST LIKELY DIAGNOSIS

2. DO THE ESSENTIAL INVESTIGATION.


3. MANAGEMENT PLAN

QUESTION 5

A YOUNG STUDENT PRESENTS WITH POOR SLEEPING, EXAMINATION IS COMING, HE IS


WORRIED ABOUT THE EXAM. RECENTLY, HIS FATHER PASSED AWAY UNEXPECTEDLY DUE
TO HEART ATTACK.

PLEASE TAKE A HISTORY,

COUNSEL THIS PATIENT,

MAKE A MANAGEMENT PLAN

MY APPROACH:

I ASKED ALL THE QUESTIONS RELATED TO DEPRESSION ANXIETY, SUCH AS DRINKING,


EATING, WEIGHT, SEX DRIVE, USUAL INTEREST, THE IMPACT OF HIS FATHER€™S DEATH
ON HIS LIFE. AND ALSO THE PAST PSYCHIATRIC HISTORY, DRUG AND ALCOHOL HISTORY,
MEDICAL HISTORY, SOCIAL HISTORY INCLUDING PERFORMANCE AT SCHOOL, FAMILY
RELATIONSHIP, EMPLOYMENT,

EXPLAIN TO HIM: THE REACTION OF BEREAVEMENT. EMOTIONAL SUPPORT,

MANAGEMENT: SLEEPING PILL, MEDICAL CERTIFICATE, PSYCHOLOGIST COUNSELLING.

QUESTION: HOW SHOULD I USE SLEEPING PILL, WHAT IS THE PREVENTION OF HEART
PROBLEM.

QUESTION 6:
A 2 YEAR OLD GIRL PRESENTED WITH HEADACHE, FEVER FOR 2 DAYS, FATHER IS
CONCERNED, HE BROUGHT HIS DAUGHTER TO YOUR GP SETTING,

HER FATHER HAS FAMILY HISTORY OF MIGRAINE.

PLEASE ASK FOR EXAMINATION FINDING FROM EXAMINER. ( NO FURTHER HISTORY


TAKING)

MANAGE THIS CASE.

MY APPROACH: ASK FOR GENERAL APPEARANCE. VITAL SIGN, ENT, ANY RASH? NECK
STIFFNESS, KERNIG SIGN? OTHER SYSTEM. €“ ONLY PHALANGITIS, AND BULGING EAR
DRUM. SIGN OF DEHYDRATION, NOT DRINKING WELL _ FROM FATHER, BUT EXAMINER
SAID BABY LOOKED WELL?

MY MANAGEMENT PLAN:

EXPLAIN TO FATHER THAT IT MAY BE VIRAL INFECTION HOWEVER I CAN NOT EXCLUDE
MENINGITIS OR OTHER SERIOUS INFECTION, SO PLEASE OBSERVE AND SEND TO ED IF
RASH, FEVER INCREASE, DROWSY, NOT DRINKING WELL OR EATING WELL.

QUESTION 7

24 YEARS OLD LADY PRESENTS WITH VAGINAL BLEEDING 2 HOURS AFTER 8 WEEKS
AMENORRHOEA. GENERAL HEALTH OK.

SHE IS OBESE. YOU ARE A GP.

PLEASE TAKE A HISTORY

ASK FOR PHYSICAL EXAMINATION. ( INVESTIGATION NOT AVAILABLE)

DISCUSS WITH EXAMINER ABOUT YOUR MANAGEMENT PLAN.


MY APPROACH: SHE IS NOT PLANNING FOR FAMILY, PARTNER IS USING CONDOM, SHE
IS NOT ON PILL, BLEEDING MORE THAN USUAL. LMP 8 WEEKS AGO. REGULAR PERIOD, NO
PAIN, NO STD HISTORY. NO FAMILY HISTORY OF BLEEDING TENDENCY. NOT ON ANY
REGULAR MEDICATION. PAP SMEAR 6 MONTHS AGO, NORMAL RESULT. NO PREGNANCY
HISTORY. NO ABORTION HISTORY.

NOW SHE HAS NO ABDOMINAL PAIN. ONLY PER VAGINAL BLEEDING. HAS STOPPED
NOW.

PHYSICAL EXAMINATION: HAEMODYNAMICALLY STABLE. NO TENDERNESS OR MASS IN


PELVIC, SPECULUM: OS CLOSE, NO BLEEDING, NO POLYPS OR ULCER.

EXAMINER ASKED: WHAT IS YOUR DIAGNOSIS IF BETA HCG POSITIVE? THREATEN


MISCARRIAGE.

WHAT OTHER INVESTIGATION DO YOU WANT? €“ U/S

WHAT IF BETA HCG NEGATIVE? POLYCYSTIC OVARIAN DISEASE. OR OTHER BLEEDING


TENDENCY, OR PROBLEMS IN THE UTERUS SUCH AS PID€¦.

QUESTION 8

61 YEARS OLD BRICKLAYER(MALE) PRESENTS WITH TWO YEARS HISTORY OF BACK PAIN
AND THIGH PAIN? (NOT REMEMBER BILATERAL OR WHICH SIDE)

PAST HISTORY: OSTEOPOROSIS

YOUR TASK:

1. HISTORY TAKING

2. ASK FOR PHYSICAL EXAMINATION FINDING

3. YOUR PROVISIONAL DIAGNOSIS AND MANAGEMENT PLAN


MY DIFFERENTIAL DIAGNOSIS:

1. SCIATICA €“ FRACTURE OR DISC PROLAPSE €“ MALIGNANCY OR OSTEOPOROSIS

2. OSTEOARTHRITIS

3. CLAUDICATION

4. TRAUMA?

I ASKED THE HISTORY CAREFULLY, FIRST, PAIN QUESTION.

THEN I ASKED DURING THE LAST TWO YEARS, HAS HE BEEN TO ANY DOCTOR? HE TOLD
ME THAT A DOCTOR TOLD HIM THAT HE HAD GOT SPONDYLOSIS OF SPINE (X-RAY) THEN I
KNEW IT MIGHT BE OSTEOARTHRITIS.

THEN I ASKED EXAMINER THE HAND EXAMINATION AND SHOULDER, NECK, BACK, HIP,
KNEE ETC€¦

THE EXAMINER TOLD ME : HEBERDON NODES ON HAND..

THEN I GOT A CLUE€¦Â€¦

MANAGEMENT X-RAY AND REFERRAL TO RHEUMATOLOGIST€¦

I AM NOT SURE I AM RIGHT OR NOT?

AMC MELBOURNE 29/04/06

I NEVER REALIZED HOW DIFFICULT IT WILL BE TO MAKE A RECALL AFTER THE EXAM. I
DO NOT REMEMBER ALL THE TASKS AND SOME OF THE EXAMINERS QUESTIONS. STEMS
WERE ALSO MUCH LONGER. SO PLEASE BE CAREFUL WHEN YOU READ THESE QUESTIONS
AND FIND YOUR OWN ANSWERS!!

NEW QUESTIONS

1.18 Y.O PATIENT COMES TO YOUR GP PRACTICE FOR INSURANCE PURPOSES. HE HAD
ASTHMA WHEN HE WAS 11.

TASK DO SPIROMETRY, CALCULATE FEV1, FVC AND RATIOS. DISCUSS RESULTS WITH THE
PATIENT.

TO ANSWER PROPERLY I€™D LOOK UP FOR SPIROMETRY FROM


WWW.NATIONALASTHMA.ORG.AU/HTML/MANAGEMENT/SPIRO_RES/VID_SPIRO.ASP

VIDEO IS THERE

FEV1, FVC INTERPRETATION, (NORMAL VALUES WERE PROVIDED BY EXAMINER)

PT ASKED ONE QUESTION €“ DO I HAVE ASTHMA?

OTHER CANDIDATES HAD NORMAL RESULT OF SPIROMETRY AND REASSURED THE PT

.2. PHOTO DORSUM OF HAND. (NOT FROM AMC BOOK €“SCALY LESIONS ON WHOLE
DORSAL SURFACE OF HAND, RED).PT IS A BRICK LAYER STARTED NEW JOB 6 MONTHS
AGO, COMES TO YOUR GP PRACTICE

TASK- TAKE HX, DISCUSS CONDITION WITH PATIENT.

RASH HAS BEEN THERE FOR 3 MONTHS, 3 MONTHS AFTER HE STARTED HIS WORK, AT
WORK DOES EVERYTHING WITH HANDS, NO GLOVES. RASH VERY ITCHY, HOLIDAYS MADE
IT BETTER. PT LOVES HIS JOB. BROTHER HAS PSORIASIS (TO CONFUSE YOU).
MY ANSWER: CONTACT DERMATITIS. EXAMINER EXPECTED TO HEAR ORAL STEROIDS AS
TX , NOT ONLY TOPICAL STEROIDS AND GLOVES AS PROTECTION. I TOLD THE ROLE-PLAYER
HE DOES NOT HAVE TO CHANGE THE JOB, WHICH HE LIKED. THE EXAMINER ASKED ABOUT
COMPLICATIONS. (MY ANSWER CELLULITIS, TREAT WITH ORAL FLUCLOXACILLIN AFTER
CHECKING BSL AND LYMPH NODES)

I€™D READ AT LEAST FROM MURTAGH AND PT€™S EDUCATION RE: THIS TOPIC

3. 50 Y O PATIENT WANTS SOME ADVICE ABOUT LOOSING WEIGHT,BMI 34, CUSHING,


HYPOTHYROIDISM EXCLUDED. SHE WILL ASK YOU ABOUT MEDICATIONS.

TAKS €“ DISCUSS WITH PATIENT. NO FURTHER HX

MY ANSWER €“STILL TALKED RE:FAMILY (ANYBODY WITH DIABETES, ANGINA, CVA?-


SHE SAID NO). REASSURED-, VERY GOOD AND IMPORTANT THAT YOU CAME, GOOD
DECISION. SPOKE RE:CARDIOVASCULAR RISK FACTORS EXPLAINED THE RISK OF INCREASED
WEIGHT €“HEART, DM, CVA AGAIN.

ADVISED RE: DIET ,OFFERED DIETICIAN.

EXERCISE

(ALL THE INFO FROM PT€™S EDU AND WWW.HEARTFOUNDATION.COM.AU)

AND SPOKE ABOUT OSTEOPOROSIS IF SHE DOES NOT EXERCISE

NO SMOKING AND, SAFE LEVELS OF ALCOHOL

RE MEDICATIONS :SAID DON€™T KNOW MUCH ABOUT, ONLY THEY CAN GIVE YOU
HIGH BP. SHE WAS NOT INTERESTED ANYWAY (IN CONTRAST TO THE STEM!!!)

LOOSE YOUR WEIGHT SLOWLY, IN A SENSIBLE WAY.

EXAMINER ASKED ABOUT SURGERY €“ LAP BANDING OF STOMACH

4. 50Y O MAN PROBLEMS WITH VISION, CAN€™T SEE THE SIGNS WHEN DRIVING CAR
SIGNS ON THE LEFT AND RIGHT BUMPING INTO THINGS AND PEOPLE. YOU€™ RE GP.
TASK €“ EXAMINE EYES.TELL DX,

WHAT OTHER INVESTIGATION WOULD YOU DO.

(LOOKS LIKE OLD QUESTION €“ MYOPIA €“MEL NOV 2004), BUT IN THIS EXAM THE
MAN IS 50 Y.O.€¦.

MY ANSWER: ROLE-PLAYER HELPED ME, SAYING I CAN€™T SEE FROM LEFT AND RIGHT.

EXAMINATION SNELLEN CHART- NORMAL,

VISUAL FIELDS- BITEMPORAL HEMIANOPIA

FUNDOSCOPY-EXAMINER SAID NORMAL.

ACCOMMODATION €“ COULDN€™T SEE A THING BUT SAID NORMAL

PUPILS REFLEXES €“EXAMINER SAID NORMAL.

EYE MOVEMENT €“ NORMAL

EXAMINER ASKED SUMMARISE FINDINGS €“ BITEMPORAL HEMIANOPIA.

WHAT€™S THE CAUSE €“ BRAIN TU, POSSIBLY OPTIC CHIASM.

WHAT OTHER TESTS?- I SAID CT AND P, BP, FULL NEURO EXAMINATION AND BASELINE
BLOODS.

LOOK UP TALLEY.-THEORY-FROM THE BOOK, EXAMINATION FROM THE DVD-


INVALUABLE FOR ANY EXAMINATION (ONLY HIP& BACK NOT THERE)

5. 41 WEEKS (CAN€™T REMEMBER POSSIBLY PRIMI ) IN LABOUR. STATION 0. BABY IN


OTPOSITION, CERVIX EFFACED , 3CM DILATION. PASSED MECONIUM. YOU€™RE GP
WORKING INOBS WARD OF COUNTRYSIDE HOSPITAL €“ NO OBSTETRICIAN AVAILABLE.

TASK- TAKE HX FROM PATIENT, ASK EXAMINATION FINDINGS, DISCUSS SITUATION WIH
PT.

(SIMILAR TO SYD SEPT 2005, OR SYD MAY 2004 BUT CHANGED)


5. A LOT OF INTERACTIONS WITH THE EXAMINER AT THIS STATION WHICH MAKES IT
HARDER.

MY ANSWER: I ASKED HOW LONG IN LABOUR €“ PT SAID 3 HRS, ARE YOU
COMFORTABLE €“ DEHYDRATION, PAIN, - PT NO ANCARE, ESP DM ( SWEET DRINK AT 28
WEEKS) €“ PT ALL OK

CONTRACTIONS? €“ EXAMINER SAID EVERY 3-4 MIN,GETTING STRONGER

EXAMINER SAID MANAGE

I SAID - CALL ANOTHER DR IN HOSPITAL FOR 2ND OPINION. CHECK PT GA,HYDRATION,


P, BP, T,

-PUT CTG (EXAMINER SAID€“ REACTIVE) AND LEAVE IT ON.

-FETAL SCALP ELECTRODE- EXAMINER NOT AVAILABLE- COUNTRYSIDE.

-ASKED RE:MECONIUM EXAMINER GREEN , YOU CAN€™T TELL THICK OR THIN

-I WANT TO RE-EXAMINE PT- EXCLUDE CORD PROLAPSE, ASSES CERVIX, PRESENTING


PART.

EXAMINER €“ CERVIX 10CM NOW, BABY BETWEEN OT AND OA,STATION +1 TALK TO
PT

TALKING WAS REASSURANCE- CANAL FOR BABY OPEN, BABY TURNING INTO GOOD
POSITION, MOVING DOWN. GREEN STUFF PASSED BUT DOES NOT MEAN THERE€™S A
PROBLEM (41 WEEKS), I€™LL KEEP AN EYE.

EXAMINER ALSO WANTED TO HEAR - SUCTION AIRWAYS OF BABY, AUGMENTATION


NOT REQUIRED.

SIMILAR CASE IN GREER €“ PROBLEM BASED OBS&GYNE.

OLD QUESTIONS/PAST RECALL PAPERS INCOMPLETE AND CONFUSING


6. 50 Y O WOMEN POSTMENOPAUSAL COME TO YOUR GP PRACTICE FOR THE RESULTS.
RESULT OF MICROCYTIC ANEMIA ATTACHED

NO FURTHER HX TO BE TAKEN.

TASK ASK EXAMINER EXAMINATION FINDINGS,

DISCUSS CONDITION WITH PT (SAME AS SYD MARCH 2005 RETEST)

6. I ASKED FOR IRON STUDIES-

EXAMINER GAVE ME A PAPER --LOW FE, LOW FERRITIN, HIGH TRANSFERRIN, LOW FE
SATURATION.OBVIOUS IDA

I ASKED FOR TISSUE TRANSGLUTAMINASE FOR CELIAC €“

EXAMINER €“ NORMAL AND NOTHING ELSE AVAILABLE. TALK TO PT.

I EXPLAINED IDA, MAYBE DUE TO DIET,MAYBE OTHER CAUSE. THE WOMAN DID NOT
WANT ANY TESTS. THE CASE WAS REALLY €ŒDEALING WITH DIFFICULT PT/CONVINCE
HER TO HAVE THE TEST€.

EXPLAINED WHY LOOSING IRON MAYBE DIET, MAYBE ULCER,MAYBE STH MORE
SERIOUS, WE HAVE TO FIND THE CAUSE.

PT ASKED- CAN IT BE STH ELSE??-NOT LIKELY WE NEED TO FIND THE CAUSE.

CAN YOU GIVE ME IRON? €“ OK, MAY IMPROVE YOUR SYMPTOMS, BUT STILL NEED TO
FIND THE CAUSE.

COLONOSCOPY/GASTROSCOPY INVASIVE BUT DONE UNDER SEDATION SO YOU WILL


COMFORTABLE, NO PAIN. IF LOSING IRON DUE TO ULCER IT€™S EASILY TREATED, IF
STHN ELSE/STHN NASTY BETTER TO KNOW EARLY. GO HOME/ SPEAK TO HUSBAND/COME
BACK WITH DECISION IN FEW DAYS. YOU REALLY NEED THE TESTS.

IDA FROM P.DEVITT


7.30 YO WOMAN ON OCP NO PERIOD FOR 2 MONTHS, COMES TO YOU AS GP. NO IX
RESULT AVAILABLE

TASK FURTHER HX, DISCUSS CONDITION WITH PT.

(SAME AS ADEL SEPT 2004,BUT HIDDEN POINTS WERE PT IS 85KG, WANTS TO BE


PREGNANT IN 2 YRS)

7. HX €“ ON MICROGYNON 30(FORGOTTEN TO ASK FOR HOW LONG!), DID NOT


LOOSE/PUT ON WEIGHT RECENTLY, NOW 85 KG, PERIODS REGULAR BEFORE, NO
HEADACHE/VISION PROBS/NIPPLE DISCHARGE, NO DRY SKIN/COLD INTOLERANCE, NO
ACNE/FUNNY HAIR ON FACE/FHX OF PCOS,NO GYNE PROCEDURE, DOES NOT .SMOKE, NO
MEDS ONLY PILL.

I HAVE TO ASK YOU SENSITIVE QQ- DID YOU HAVE SEX OVER LAST 2 MONTHS. YES.

PE €“ EXAMINER ALL NORMAL

PREGNANCY TEST- UNAVAILABLE

MY ANSWER : EXPLAINED €“ EXCLUDE PREGNANCY FIRST , IF NEGATIVE DON€™T


WORRY, 2 MONTHS NOT SIGNIFICANT, NO TEST NECESSARY NOW

IF YOU CONCERNED WE CAN STOP PILL AND WAIT (USE BARRIER PROTECTION) OR
CHANGE PILL TO HIGHER ESTROGEN OR TRIPHASIC.

OTHER OPTION I€™D RECOMMEND - CONTINUE SAME PILL (AS 2 MONTHS NO


PERIOD- NO REASON TO WORRY), SEE IF PERIOD BACK AND LOOSE WEIGHT OVER NEXT 2
YRS. YOU€™RE OVERWEIGHT PREGNANCY MAY BE COMPLICATED BY HIGH BP AND DM.

PT ASKED €ŒWILL I GET PREGNANT€- WE HAVE TO EXCLUDE PREGNANCY NOW AND


WAIT FOR PERIOD TO COME BACK. SEE ME IN 2-3 WEEKS.

8.YOU€™RE INTERN ED, WOMEN CAME 3 DAYS AGO-PREVIOUSLY ON PROZAC


STOPPED 3 DAYS AGO, STARTED ON ZOLOFT. TODAY SHE€™S VOMITING, JITTERY, DIZZY.
TASK- TAKE FURTHER HISTORY, PE FROM EXAMINER, DISCUSS CONDITION WITH PT.

( SAME AS SYD AUG 2003)

8. STILL CONFUSING THIS QUESTION, IN TAKING HX I REPEATED WHAT WAS IN STEM, PT


UNWELL ON PROZAC BEFORE GIVEN ZOLOFT, UNWELL TODAY. DID NOT TAKE ANY OTHER
MEDS.

PE €“ P-NORMAL, BP NO POSTURAL DROP, NOT DEHYDRATED, CVS< RESP>ABDO NAD,


BRISK REFLEXES

URINE NO KETONES. IX NOT AVAILABLE.

I EXPLAINED ALL DUE TO SE OF ANTIDEPRESSANT, THERE€™S STHN WE CALL


SEROTONIN SYND, BUT USUALLY WHEN 2 ANTIDEPR. MIXED FROM DIFFERENT GROUPS,
THESE 2 ARE SSRI€™S . SO MY DX IS SE OF ANTIDEPRESSANT.

EXAMINER- WHAT WILL YOU WRITE IN CHART?

IV LINE, NSALINE, METOCLOPRAMIDE.

PT ASKED I CAN€™T SLEEP. I ANSWERED I€™LL ASK SENIOR (DID NOT KNOW IF I
COULD GIVE VALIUM)

9. YOU€™RE GP. YOUR PATIENT WAS 29CM AT 30 WEEKS, NOW 4 WEEKS LATER
SHE€™S 30 WEEKS. FIGURES MAY HAVE BEEN DIFFERENT BUT STILL DECREASED
GROWTH AROUND 28/34 WEEKS

TASK-TAKE FURTHER HX, ASK PE AND IX, DISCUSS RESULTS WITH PT.

(SIMILAR TO SYD MAY 2004)

9. IN HX NO HEADACHE, BLURRED VISION, TUMMY PAIN, NO HX OF INFECTION, ANCARE


NORMAL €“USS 18 WEEKS, SWEET DRINK 28 WEEKS. GOOD DIET, NO SMOKING, NO IV
DRUGS.

BABY KICKING LESS (I FORGOT TO ASK HOW MANY KICKS/DAY NOW!!!)


PE €“ P,BP NORMAL HEART SOUND 140/MIN. URINE DIPSTICK NO PROTEIN

I ASKED FOR IX - CTG (EXAMINER SAYS REACTIVE), DOPPLER (EXAMINER-WHAT YOU'RE


LOOKING AT? - S/D RATIO, WHICH ARTERY ?- UMBILICAL)

BPP - (EXAMINER- WHICH PARAMETERS YOU'RE LOOKING AT ?) TONE, BREATHING ETC

USS - EXAMINER SAYS- BABY 10TH PERCENTILE ASYMMETRIC

WHAT'S YOUR DX? - IUGR

TALK TO PT. STARTED EXPLAINING AND BELL RANG.

FUNNY OTHER CANDIDATES GOT SMALL FOR DATES FROM IX, TO OTHERS EXAMINER
SAID- NO IX AVAILABLE.

THIS QUESTION STILL CONFUSING.

IUGR FROM GREER.

OLD QUESTIONS REPEATED

10. 2 Y O BOY BROUGHT BY FATHER TO YOUR GP PRACTICE 50KM FROM HOSPITAL. BOY
HAD SORE THROAT LAST 2 DAYS, TODAY DROWSY, DEHYDRATED, P-HIGH, BP LOW, T-40,
PETECHIAL RASH, NO NECK STIFFNESS. PHOTO FROM AMC BOOK.

TASK - EXPLAIN CONDITION TO FATHER, MANAGE THE CASE.

(SAME AS MEL FEB 2006 !!)

10. MY ANSWER: IT'S URGENT, SO I'LL EXPLAIN VERY BRIEFLY.

BLOOD POISONING, VERY DANGEROUS IF NOT TREATED,SAME BUG AS THE ONE


CAUSING

MENINGITIS. ANY ALLERGIES TO PENICILLIN? PT - NO


I DO IV LINE, TAKE BLOOD FOR CULTURE, GIVE BENZYLPENICILLIN 600MG IV, CALL
AMBOS, CALL REGISTRAR IN HOSPITAL.

YOU' RE WIFE PREGNANT? PT -NO. OK, SO YOU'LL BOTH NEED MEDS FOR PREVENTION
2 TABS. DAILY FOR 2 DAYS. THINK IF ANYONE KISSED, SHARED FOOD WITH CHILD OVER
LAST 10 DAYS. I ALSO NEED TO NOTIFY DHS.

VERY GOOD YOU CAME, VERY IMPORTANT, HOPE THE CHILD WILL BE FINE.

PT ASKED- CAN IT BE STH ELSE? NOT LIKELY AS CHILD REALLY SICK NEEDS TO GO TO ED
NOW, YOU CAN GO WITH THE CHILD IN AMBULANCE.

GUIDELINES RCH

11. FATHER OF 5 MONTHS OLD BOY COMES TO YOUR GP PRACTICE RE: LUMPS IN RT
GROIN AND ANOTHER ONE AROUND BELLY BUTTON. LUMPS COME AND GO.

TASK EXPLAIN DIAGNOSIS AND MANAGE THE CASE

(SAME AS MEL NOV 2005)

11. MY ANSWER: GOOD YOU CAME, YOU CHILD HAS 2 HERNIAS-WEAKENING OF


MUSCLES IN TUMMY, EXPLAINED ABOUT TESTIS TRAVELING FROM TUMMY TO
SCROTUM,CHANNEL DID NOT CLOSE, CAN BE DANGEROUS IF BOWEL GETS STUCK
OUTSIDE.

I AM CALLING SURGEON NOW, FOR APPOINTMENT AS EARLY AS POSSIBLE.

OPERATION POSSIBLY WITHIN 2 WEEKS RE;LUMP IN GROIN,ABOUT LUMP AROUND


BELLY BUTTON SURGEON WILL DECIDE IF HE WILL REPAIRS BOTH TOGETHER. NORMALLY
THE ONE AROUND BELLY-BUTTON CAN WAIT UNTIL 2/4 YEARS

IF BABY CRYING UPSET, CHECK TUMMY IF LUMP-HARD - GO TO ED ASAP.

PT ASKED ANY SE OF ANESTHETICS.? EVERY MEDICATION HAS SE, BUT BENEFIT


OUTWEIGHS RISK.
12. MOTHER 4 WEEKS BABY COMES TO YOUR GP PRACTICE, SAYS BABY VOMITING
EVERYTHING AFTER FEEDING. PUT ON WEIGHT SINCE WAS BORN. SHE WANTS TO
CHANGE TO BOTTLE FEEDING.

TASK TAKE FURTHER HX, PE, DISCUSS MX WITH MOTHER

(SAME AS ADEL OCT 2005)

12. HX -BABY VOMITING NOT ON FURTHER QUESTIONING NOT SO MUCH, ONLY A BIT
ON THE CLOTHES, SHE PROPS BABY UP AFTER FEEDING . BABY HUNGRY 1 HR AFTER
FEEDING (OTHER CANDIDATES GOT 2-3 HRS) THEN SETTLES.

VOMIT ONLY MILK €“ NO BLOOD OR GREEN STUFF.

CHILD HAPPY, DEVELOPMENT OK, PUT ON WEIGHT. DID NOT FEEL ANY LUMP.

PE €“ NORMAL, NO LUMP IN TUMMY.

STILL SEND YOU FOR USS TO EXCLUDE PYLORIC STENOSIS IF RESULT OK BABY HAS
REFLUX.

I EXPLAINED GASTRO-ESOPH. REFLUX/DRAW A PICTURE/, VERY COMMON, GETS BETTER


WITH TIME. CAN LAST UP TO 9 MONTHS.

PT ASKED

AM I DOING STH WRONG?-

CONTINUE BREASTFEEDING AS BREAST MILK BETTER FOR BABY AND NOT THE CAUSE OF
THIS PROBLEM.

YOU CAN THICKEN FEEDS WITH CAROBEL/GAVISCON, KEEP BABY PRONE 30-60MIN,
ELEVATE THE BED 20DEGREES, AND GIVE SMALLER, MORE FREQUENT FEEDS.

PT ASKED - COMPLICATIONS OF REFLUX? €“ BELL RANG

GORD €“ FROM PT€™S EDUCATION

13.40 YO WOMAN (2 EPISODES OF HEADACHES WITH NAUSEA, PHOTOPHOBIA, HAD CT


SCAN DONE EXCLUDED BRAIN TU), COMES TO OUR GP PRACTICE FOR ADVICE.
TASK- EXPLAIN MIGRAINE, DISCUSS FURTHER MX

(SAME AS SYD AUG 2004)

13. MY ANSWER: GOOD NEWS, THE STRESS MAY BE GONE,AS YOU HAVE NO CANCER,
WHAT YOU€™VE GOT IS MIGRAINE.

EXPLAINED RE:VESSELS CLOSING AND OPENING.

DIFFERENT CAUSES, USUALLY MULTIPLE FACTORS, CHINESE FOOD, RED WINE ETC., SO
WRITE A DIARY. PT WAS TEACHER A LOT OF STRESS IN LIFE.

MANAGEMENT €“ ASPIRIN(ASKED IF NO PAIN I TUMMY)/ METOCLOPRAMIDE

IF NOT BETTER ERGOTAMINE (ASKED HER NO PROBS WITH HEART OR BP)

PROPHYLAXIS NOT REQUIRED AS YET.

MIGRAINE FROM MURTAGH

14. YOUNG MAN COMES TO ED 2ND EPISODE OF SEVERE PAIN IN THE BACK, PAIN
RADIATING TO GROIN, WENT AWAY NOW. 1ST EPISODE 2 MONTHS AGO.

TASK TAKE FURTHER HX, ASK IX, DISCUSS DX WITH PT

(SAME SYD MARCH 2006!!)

14. PAIN OCCURRED SUDDENLY, VERY SEVERE, PT MOVING AROUND TO MAKE IT


BETTER

NO CHANGE IN URINE COLOR.

IX ASKED FOR DIPSTICK, KUB AND HELICAL CT, DR GYA SHOWED CT WITH STONE.

EXPLAIN THE PT - SEDIMENT IN URINE, STONE BUILDS UP AROUND PAIN,PAIN HAPPENS


WHEN STONE MOVING.

DR GYA- WHAT€™S MX. STONE IS 5MM.


ANSWERED BORDERLINE, MAYBE PT GOES HOME AND WEES THRU MESH, OR
LITHOTRIPSY. I€™LL TAKE BLOOD TESTS AND ASK CONSULTANT.

CAN HE GO HOME? CONSULTANT WILL DECIDE. (BETTER ANSWER WAS €ŒNOT SURE
AS HE HAS NOT PASSED THE STONE LAST 2 MONTHS€)

WHERE DO YOU REFER? TO SURGEON.

RENAL COLIC FROM P. DEVITT, SENANYAKE- COMMON CASES, PT€™S EDUCATION

15.SHOULDER EXAMINATION GP SETTING. BASKETBALL PLAYER HAD ANTERIOR


DISLOCATION 6 MONTHS AGO, COMES FOR CHECK UP.

TASK- EXAMINE SHOULDER GIVE RUNNING COMMENTARY, ANSWER QUESTIONS.

(SAME SYD MARCH 2004)

15. EXAMINED THE SHOULDER AS FROM DVD , EVERYTHING NORMAL, APPREHENSION


TEST, IMPINGEMENT TEST NEGATIVE

ASKED FOR PIN TO CHECK PATCH ON THE DELTOID (EXAMINER SAYS NORMAL)

QUESTIONS FROM EXAMINER- WHICH NERVE? AXILLARY

WHICH MUSCLE FOR ABDUCTION? DELTOID

PT ASKED CAN I PLAY? YES BUT NO DUNKING

EXAMINER ASKED WHERE YOU REFER? SPORT MEDICINE CLINIC

WHAT DO YOU DO FOR RECURRENT ? BELL RANG.

16. YOUNG WOMEN GAVE BIRTH TO CHILD 4 WEEKS AGO, HAS ANOTHER BABY
30MONTHS. RECENTLY UNWELL, TIRED (CAN€™T REMEMBER WHAT ELSE) YOU€™RE
GP

TASK TALK TO MOTHER, DISCUSS CONDITION(SAME AS SYD JULY 2004)


PT TIRED NO TIME FOR HERSELF, HUSBAND SUPPORTIVE, APPETITE, SLEEP, ENERGY
DOWN, INTEREST GONE, MOOD LOW. NO HALLUCINATIONS, DOS NOT THINK BABY HAS
SPECIAL POWER, NO HX OF DEPRESSION IN PAST. (SOME CANDIDATES GOT 10KG WEIGHT
LOSS). (I FORGOT RISK ASSESSMENT!)

DX EXPLAINED POSTNATAL DEPRESSION/WE DON€™T KNOW WHY IT


HAPPENS,/MAYBE HORMONE CHANGES. COMMON - HAPPENS TO 1/7 WOMEN AFTER
BIRTH

COME BACK WITH HUSBAND, TRY TO FIND TIME FOR YOURSELF, SEE OLD FRIENDS,
RELAXATION,

PT DOES NOT WANT TO SEE COUNSELOR, PSYCHIATRIST. STILL REFERRED HER TO


PANDA

PT ASKED €“ SHOULD I BREASFEED? UP TO YOU - BREAST MILK GOOD FOR BABY, BUT
TAKES A LOT OF YOUR ENERGY SO YOU€™LL HAVE TO THINK AND DECIDE.

COME BACK IN FEW DAYS IF NOT BETTER MAYBE START ANTIDEPRESSANT

RECALL 29TH APRIL 2006 MELBOURNE

1. POST NATAL DEPRESSION

THE 28 YEARS OLD WOMAN CAME TO YOUR GP PRACTICE AFTER DELAY 4 DAYS FROM
APPOINTMENT. SHE JUST GAVE BIRTH TO BABY 6 WEEKS AGO HAS ANOTHER 10 YEARS
OLD CHILD AT HOME. SHE COMPLAINS DOESN€™T HAVE ENOUGH ENERGY AND
CAN€™T ORGANIZE HERSELF.
TAKE RELEVANT HISTORY

EXPLAIN THE CONDITION TO THE PATIENT

MANAGEMENT

2. IRON DEFICIENCY ANEMIA

WOMAN 48 YEARS OLD MENOPAUSE CAME TO YOUR CLINIC TO GET RESULT FROM HER
BLOOD TEST (ATTACHED, SUGGESTED IRON DEFICIENCY ANEMIA).

ASK RELEVANT HISTORY

EXPLAIN THE CONDITION TO THE PATIENT

MANAGEMENT

3. SIDE EFFECT OF ANTIDEPRESSANT

WOMAN CAME TO ED FEELS NAUSEA, VOMITING, JITTERY AND UNWELL. SHE€™S


BEEN ON FLUOVOXETINE FOR 3 YEARS AND THEN SHE CAME TO ED. THE DOCTOR GAVE
HER SERTRALINE.

TAKE RELEVANT HISTORY

MANAGEMENT

4. SEPTICEMIA

FATHER OF 4 YEARS OLD CHILD CAME TO YOUR SURGERY BECAUSE HIS CHILD LOOKS
VERY ILL, FEVER AND DEVELOPED RASH (AS PICTURE IN AMCQ BOOK PAGE 118).O/E GA:
LOOKS ILL, VS: BP 100/60 TEMP : 40 DEGREE RR: 98

EXPLAIN THE CONDITION TO THE PATIENT

MANAGEMENT
YOU KNOW THE SONG GUYS, EXPLAINING CONDITION TO THE PATIENT, NOTIFY HEALTH
DEPARTMENT, PROPHYLAXIS FOR HOUSEHOLD CONTACT.

5. RENAL STONE

28 YEARS OLD CAME TO YOUR SURGERY AFTER HAVING SEVERE ABDOMINAL PAIN 2
HOURS AGO. THE PAIN RADIATES TO HIS RIGHT GROIN AND TESTICLE. HE HAD SIMILAR
PAIN 2 MONTHS BACK.

EXPLAIN THE CONDITION TO THE PATIENT ,ORDER THE NECESSARY INVESTIGATION,


MANAGEMENT

EXAMINER IN THIS STATION WAS VERY NICE. I EXPLAINED TO THE PATIENT ABOUT
STONE IN HIS URINE PASSAGE. I SAID TO EXAMINER I WANT TO ORDER FOR FBE, U&E,
MSU CULTURE AND MICROSCOPY, ABDOMINAL X-RAY.

EXAMINER ASKED:

€¢ WHAT ELSE THE INVESTIGATION YOU NEED TO ORDER? CT KUB

€¢ WHAT STONES DO YOU THINK IT IS? CALCIUM, PHOSPHATE AND URIC ACID.

€¢ WHAT FROM THE BLOOD YOU WANT TO LOOK? (EXAMINER POINTED THE STONES
SO I UNDERSTAND WHAT HE WANTS) SERUM CALCIUM AND PHOSPHATE.

ROLE PLAYER ASKED :

€¢ WHY DO YOU THINK I€™VE GOT THIS CONDITION? BECAUSE THERE€™S
POSSIBILITY THAT YOU DIDN€™T DRINK ENOUGH WATER.

€¢ WHAT ELSE? (EXAMINER STOPPED HIM AND SAID OKAY YOU FINISHED EARLIER
YOU CAN WAIT OUTSIDE.

6. MANAGEMENT OF MIGRAINE
THE WOMAN 38 YEARS OLD CAME TO YOUR SURGERY WITH SEVERE HEADACHE AND
VOMITING. SHE HAD A HISTORY OF MIGRAINE 2 MONTHS AGO. THE CT BRAIN RESULT
DOESN€™T SUGGEST ANY TUMOR.

TAKE RELEVANT HISTORY (NO MORE THAN 2 MINUTES)

EXPLAIN THE CONDITION TO THE PATIENT

MANAGEMENT

I CONGRATULATED THE PATIENT AND EXPLAINED TO THE PATIENT ABOUT


ENLARGEMENT OF BLOOD VESSEL ON THE SCALP AND TRIED DRAWING THE PICTURE AS
WELL. THEN I GAVE HER AN ADVICE ABOUT LIFESTYLE MODIFICATION REDUCE WINE,
CAFFEINE, SMOKING, CHEESE, ETC AND PARACETAMOL AND ANTIEMETIC TABLET.

EXAMINER ASKED:

€¢ WHAT IF SHE DOESN€™T GET BETTER? I€™LL REFER TO NEUROLOGIST AND THEY
MIGHT GIVE HER SUMATRIPTAN OR ERGOTAMINE. YOU AS A GP CAN GIVE HER SHE SAID.

7. BITEMPORAL HEMIANOPSIA

THE MAN 48 YEARS OLD CAME TO YOUR SURGERY. HE COMPLAINED OF HAVING VISUAL
DISTURBANCES. TWO DAYS AGO HE HAD ACCIDENT BECAUSE HE DIDN€™T SEE A TRUCK
COMING FROM THE SIDE OF THE ROAD.

EXAMINE THE PATIENT EYES

ORDER NECESSARY INVESTIGATION

I CHECKED FOR THE VISUAL ACUITY, VISUAL FIELD AND FUNDUSCOPY.THAT REAL
PATIENT HAD VISUAL LOST ON THE LATERAL SIDE.

I SAID TO THE EXAMINER THAT HE HAS BITEMPORAL HEMIANOPSIA PROBABLY DUE TO


COMPRESSION OF OPTIC CHIASM. (PITUITARY ADENOMA)
EXAMINER ASKED:

€¢ WHAT ARE THE SIGN YOU LOOKED ON THE PATIENT?

€¢ WHAT INVESTIGATION YOU WANT TO ORDER? FBE, PROLACTIN LEVEL, BLOOD
SUGAR, LIPID PROFILE, CT BRAIN, 24 HOUR SERUM CORTISOL LEVEL, TFT,€¦.

€¢ WHAT ELSE?

I WASN€™T SURE ABOUT THIS STATION BECAUSE EXAMINER KEPT ASKING WHAT
ELSE? THEN THE BELL RANG.

8. CONTACT DERMATITIS

THE 32 YEARS OLD MAN CAME TO TOUR SURGERY. PRESENTED WITH HAND (THE
PICTURE ATTACHED: SWELLING WITH SIGN OF DERMATITIS AND INFECTION BETWEEN
THE FINGERS)

TAKE RELEVANT HISTORY

EXPLAIN THE CONDITION TO THE PATIENT

MANAGEMENT

FROM HISTORY THE PATIENT SAID IT CAME SUDDENLY, HE DOESN€™T HAVE ANY
ALLERGY, HAY FEVER OR ASTHMA, AS WELL AS FAMILY HISTORY OF THOSE CONDITIONS.
HE IS A BUILDER AND RECENTLY HE WORKED WITH CEMENT. I EXPLAINED TO HIM THAT
HE HAD AN ALLERGY PROBABLY DUE TO CEMENT COMPLICATED BY BACTERIA. I GAVE
HIM PREDNISOLONE AND ANTIBIOTIC. I TALKED ABOUT REFERRING HIM TO ALLERGIC
CLINIC TO HAVE A TEST DONE AND AVOIDANCE TO THE MATERIALS THAT YOU ALLERGIC
TO. THEN REPORT TO YOUR EMPLOYER REGARDING HIS CONDITION. THERE€™S
POSSIBILITY THAT THEY€™LL MOVE YOU TO ANOTHER DEPARTMENT

ROLE PLAYER ASKED:

€¢ HOW LONG YOU GOING TO PUT ME ON ANTIBIOTIC?

I HAVEN€™T ANSWERED AND BELL RANG€¦..


9. GASTROOESOPHAGEAL REFLUX.

THE MOTHER OF 1 MONTH OLD BABY CAME TO YOUR SURGERY BECAUSE THE BABY
ALWAYS VOMITS AFTER FEEDING. THE BABY GROWS AND DEVELOPS WELL. MOTHER
THINKS THERE€™S SOMETHING WRONG WITH HER MILK.

TAKE RELEVANT HISTORY

ASK EXAMINER EXAMINATION FINDING

MANAGEMENT

I TOOK FURTHER HISTORY, THE BABY€™S WELL SO FAR. THEN EXPLAINING ABOUT GE
REFLUX AND DRAWING A PICTURE. I REASSURED MOTHER THAT€™S NOTHING WRONG
WITH THE BABY AND THE BREAST MILK. AND ROLE PLAYER ASKED ARE YOU SURE
THAT€™S MY BABY€™S OKAY? BANG! I REALIZED I FORGOT TO DO PE. I SAID SORRY TO
EXAMINER THEN ASKED GA,VS, ENT, HEART AND LUNG, ABDOMINAL EXAMINATION. ALL
NORMAL. THEN I SAID TO THE PATIENT YOUR BABY€™S NORMAL, USUALLY THIS
CONDITION WILL CLEAR UP BY ITSELF WHEN SOLID IS INTRODUCED ABUT 6 MONTHS OLD.
IF THE CONDITION DOESN€™T GET BETTER YOU CAN ADD CORN FLOUR TO YOUR MILK
OR GAVISCON. I€™LL FOLLOW HER UP IN 1 WEEK, BUT IF YOU HAVE ANY CONCERN
COME BACK TO SEE ME.

10. EXAMINE THE SHOULDER AND NERVE INJURY.

THE 18 YEARS OLD STUDENT PLAYING IN BASKETBALL COMPETITION. HE HAD


SHOULDER DISLOCATION ON THE RIGHT AND NERVE INJURY. HE ALREADY HAD AN
OPERATION FIXING IT UP 6 MONTHS AGO. HE CAME TO YOUR SURGERY ASKING YOU
WHEN HE CAN GO BACK PLAYING BASKETBALL AGAIN.

EXAMINE THE PATIENT

ADVISE THE PATIENT ABOUT RETURNING TO THE COMPETITION


THE EXAMINER WAS REALLY OLD AND VERY NICE. I STARTED EXAMINE THE SHOULDER.

EXAMINER ASKED:

€¢ WHAT NERVE COULD BE INJURED? AXILLARY NERVE. GO ON EXAMINE THE


PATIENT. I EXAMINED FOR SENSATION ON DELTOID MUSCLE AND POWER (ABDUCTION)

€¢ WHAT ELSE? THE BELL RANG€¦ I WAS THINKING ABOUT RADIAL NERVE AS WELL,
BUT NOT SURE. SO PLEASE CHECK THE BOOKS.

11. THE WOMAN 26 YEARS OLD CAME TO YOUR SURGERY COMPLAINED OF STOPPING
HER PERIOD AFTER TWO COURSES OF ORAL CONTRACEPTIVE PILL.

ASK RELEVANT HISTORY

ASK EXAMINER EXAMINATION FINDING AND ORDER NECESSARY INVESTIGATION

EXPLAIN TO THE PATIENT

FROM FURTHER HISTORY THE WOMAN STOPPED HER PERIOD SUDDENLY, DOESN€™T
HAVE TUMMY PAIN AND VAGINAL DISCHARGE. NO PAST HISTORY OF STD,
HYPERTENSION, DM. FAMILY HISTORY (-). I ASKED EXAMINER FOR PREGNANCY TEST
URINE AND BLOOD, FBE, TFT, FSH, LH, PROLACTIN LEVEL, SWAB. EXAMINER SAID GOOD
BUT THEY ARE NOT AVAILABLE. HUH? OKAY, I SAID TO THE PATIENT I€™M WAITING THE
RESULT OF THE TEST I RAN. IF THERE€™S ABNORMALITY I€™LL REFER YOU TO
SPECIALIST STRAIGHT AWAY. IF THEY COME BACK NORMAL WE€™LL WAIT FOR
ANOTHER 3 MONTHS IF YOUR PERIOD DOESN€™T COME I€™LL REFER YOU TO SP. IT
COULD BE DUE TO STRESS OR SIDE EFFECT OF OCP.

I WASN€™T SURE ABOUT THIS CASE. CHECK IT AGAIN IN THE BOOK.


12. YOU ARE HMO IN OBSTETRIC AND GYNECOLOGY DEPARTMENT. 41 WEEKS
PRIMIGARAVIDA CAME TO ED. SHE IS IN LABOUR. O/E CERVIX DILATED 3 CM LOP (IN THE
MORNING LOT, MEANS TRANVERSE) AND MECONIUM STAINED. OBSGYNE REGISTRAR
AND CONSULTATNT ARE NOT THERE. YOU HAVE TO MANAGE HER BY YOURSELF.

ASK RELEVANT HISTORY

ASK EXAMINER ABOUT EXAMINATION FINDING

MANAGE THE PATIET

13. PRIMIGARAVIDA WOMAN 28 YEARS OLD CAME IN FOR ROUTINE CHECK UP. FOUR
WEEKS BEFORE YOU CHECKED FUNDAL HEIGHT 26 CM. NOW SHE CAME TO YOU AND HER
FUNDAL HEIGHT 30 CM

TAKE HISTORY

ASK EXAMINER EXAMINATION FINDING AND ORDER THE NECESSARY INVESTIGATION

(EXAMINER ONLY WILL GIVE YOU WHAT YOU ASK)

MANAGE THE PATIENT

14. WEIGHT REDUCTION

WOMAN 43 YEARS OLD BMI 33. SHE CAME TO YOU FOR ADVICE ON WEIGHT
REDUCTION

ADVICE THE PATIENT

ANSWER PATIENT QUESTIONS REGARDING MEDICATION TO LOSE HER WEIGHT.

15. UMBILICAL AND GROIN HERNIA

A FATHER OF 4 MONTHS OLD BABY CAME TO YOU COMPLAINING OF A LUMP IN HIS


RIGHT SIDE OF THE ABDOMEN AS NOTICED BY THE FATHER IN THE BATH COMES AND GO,
OTHERWISE THE BABY IS FINE.O/E : THERE€™S UMBILICAL HERNIA 1CM, BOTH TESTIS
ARE IN THE SCROTUM

ASK FEW CLARIFYING QUESTIONS


MANAGE THE CONDITION

I EXPLAINED AND DREW A PICTURE AS USUAL. THEN REFERRING THE PATIENT TO A


SURGEON AND SAYING POSSIBILITY OF TWISTING IF IT€™S NOT CORRECTED AND
SHOULD BE IN TWO WEEKS TIME. THE UMBILICAL HERNIA, IT€™LL REDUCE BY ITSELF
WHEN THE CHILD GETS OLDER BUT IF IT DOESN€™T WHEN THE CHILD 5 YEARS OLD,
NEED TO HAVE OPERATION DONE AND DON€™T PUT ANYTHING ON IT.

16. SPIROMETRY

28 YEARS OLD MAN WITH A HISTORY OF ASTHMA CAME TO YOU. HE HAS SYMPTOM
FREE FOR 2 YEARS. HE HASN€™T BEEN USING VENTOLIN IN THE PAST 2 YEARS.

DO SPIROMETRY

EXPLAIN TO THE PATIENT THE RESULT COMPARE TO THE EXPECTED VALUE

I GAVE UP IN THE MIDDLE OF THIS STATION SO DEFINITELY I FAILED THIS STATION . I


HAD NO IDEA AT ALL. FELT REALLY BAD. READ IT IN THE TALLEY O€™CONNOR.

CLINICAL EXAM MELBOURNE FEB 11, 2006

O&G

1- 25 YO FEMALE PATIENT WITH MORE THAN TEN YEARS€™ HISTORY OF DM TYPE 1.

TASK: PRE PREGNANCY COUNSELLING

PT ASKED:
- HOW WOULD DM AFFECT MY PREGNANCY?

- SHOULD I KEEP TAKING INSULIN?

- WHAT ARE THE EFFECTS ON MY BABY?

- WOULD I NEED A CS?

EXAMINER ASKED:

- WHAT KINDS OF BLOOD TEST ARE YOU GOING TO ORDER?

2- YOUNG UNIVERSITY FEMALE STUDENT WANTS TO START TAKING OCP.

TASK: OCP COUNSELING

PT ASKED:

- WHAT TYPE OF OCP IS SUITABLE FOR ME?

- ANY SIDE EFFECTS ASSOCIATED WITH OCP?

3- A PRIMI 38 WKS HAD A CONVULSION WHILE SITTING IN THE WAITING ROOM WITH
HER MUM TO SEE YOU, A REMOTE GP.

TASK: MANAGE THIS PT AND ANSWER THE QUESTIONS RAISED BY HER MUM

HER MUM ASKED:

- WHAT€™S WRONG WITH MY DAUGHTER?

- HOW ARE YOU GOING TO DEAL WITH MY DAUGHTER?

- ANY HARMFUL EFFECTS ON MY DAUGHTER OR HER BABY?


PAEDIATRICS:

4- YOUNG MOTHER COMES TO SEE YOU, A GP, ENQUIRING ABOUT CIRCUMCISION.

TASK: COUNSELLING

PT ASKED:

- WHAT ARE THE BENEFITS OF CIRCUMCISION?

- WHAT ARE THE CONTRA-INDICATIONS?

- WHAT ARE THE COMPLICATIONS?

5- A 9 YO CHILD IS JUST DIAGNOSED DIABETIC TYPE 1. HIS FATHER WANTS TO KNOW


MORE ABOUT IT.

TASK: COUNSELING.

FATHER ASKED:

- WHAT€™S THE MANAGEMENT TO MY SON?

- DOES MY SON NEED INSULIN FOR THE REST OF HIS LIFE?

- CAN MY SON GO TO CAMP WITH SCHOOL?

- HOW ABOUT HIM OVERSTAYING WITH HIS MATES ON WEEKENDS?

6- YOU, A GP WORKING AT REMOTE AREA, ARE SEEING A YOUNG BOY WHO PRESENTED
WITH FEVER AND RASH (PICTURE IS PRESENTED).

TASK: MANAGEMENT AND COUNSELLING HIS FATHER

EXAMINER ASKED:

- WHAT IS YOUR PROVISIONAL DIAGNOSIS.? (SEPTICAEMIC MENINGITIS)

- WHAT€™S YOUR MANAGEMENT?


- IF ALLERGIC TO PENICILLIN WHICH AB WOULD YOU GIVE?

- IF IT€™S HARD TO ESTABLISH IV ACCESS, WHAT ARE YOU GOING TO DO?

PSYCHIATRY:

7- YOUNG FEMALE COMES TO SEE YOU JUST BECAUSE HER PARENTS ASKED HER TO DO
SO. SHE DENIES ANY ABNORMALITY.

TASK: HX, DX, AND MX

EXAMINER ASKED:

- WHAT€™S YOUR DX? WHY? (ANOREXIA NERVOSA)

- WOULD YOU ADMIT THIS PATIENT? WHY?

8- 1ST EPISODE ACUTE PSYCHOSIS: NURSE NOTICED WRIST INJURY, LIVES WITH 3
FRIENDS

NOTE: NO MORE HX REQUIRED

- PATIENT WANTED TO GO HOME TO SLEEP

- PATIENT HAS AUDITORY HALLUCINATION

- NO SUICIDAL IDEATIONS

- VERY AGITATED AND SWEARS ALL THE TIME

- NO INSIGHT

TASK: FOCUSED HX AND MX.

PATIENT ASKED:
- CAN YOU GIVE ME SOMETHING TO GET SOME SLEEP?

EXAMINER ASKED:

- WHAT DO YOU THINK HE HAS?

- WOULD YOU ADMIT HIM?

- WHAT IF HE REFUSES TO BE ADMITTED?

- WHAT WILL YOU GIVE HIM?

- WHAT ARE THE SIDE EFFECTS OF HALOPERIDOL?

MEDICINE

9- A 60 YO LADY HAS BEEN TOLD BY A PREVIOUS GP THAT SHE HAD A MURMUR, SHE
WANTS TO TRAVEL OVERSEES.

TASK: EXAMINE HER CVS AND REPORT TO THE EXAMINER YOUR FINDINGS.

DX: AORTIC STENOSIS

10- A 67 YO LADY WAKES UP IN THE MORNING WITH WEAK RT ARM AND SLURRED
SPEECH THAT LASTED FOR ONE HOUR (TIA CASE)

TASK: EXAMINE THE RELEVANT SYSTEM/S

11- AN ASTHMATIC PATIENT WAS AT A PARTY LAST NIGHT AND TRIED WALNUT BUTTER
FOR THE FIRST TIME,

HE GETS SWOLLEN LIPS AND DIFFICULTY BREATHING, AND THEN GETS BETTER AFTER
TAKING VENTOLIN.

TASK: COUNSELING
PATIENT ASKED:

- AM I GOING TO GET THESE REACTIONS EVERY TIME I HAVE WALNUT BUTTER?

- WHAT ELSE SHOULD I NOT TRY?

- IS THERE A WAY TO CURE OR PREVENT THESE REACTIONS?

- CAN I GO CAMPING?

12- 45 YO MALE PATIENT, REFERRED BY HIS GP TO SEE YOU, FEELS TIRED, CARDIAC
PROBLEMS, ABNORMAL OF LIVE FUNCTION TEST, BRADYCARDIA, NON ALCOHOLIC, NO HX
OF DARK URINE,

TASK IX, DX, MX,

DX: HAEMOCHROMATOSIS

SURGERY

13- 50 YO MALE PATIENT WITH A SWELLING IN HIS SCALP

TASK: EXAMINE, DX & ADVICE ON MANAGEMENT.

DX: LIPOMA

14- 65 LADY HAD PAIN FOR 3 HRS IN THE MORNING AND BLOODY DIARRHOEA,
NAUSEA,

HX OF AF AND APPENDICECTOMY
TASK HX, DD

MESENTERIC ARTERY OCCLUSION / ISCHAEMIC COLITIS

15- AN OBESE MALE PRESENTS TO YOU WITH PAIN IN THE LEFT LOWER CHEST,

TASK: HX, DD

PNEUMOTHORAX / PLEURISY / PE / PNEUMONIA / MI

16- A MIDDLE AGED PATIENT FELL OF HIS MOTOR BIKE AND FRACTURED HIS CLAVICLE,

TASK: MANAGEMENT.

PATIENT ASKED:

- CAN I RIDE MY MOTOR BIKE?

- CAN I DRIVE MY CAR THEN?

- HOW CAN I HAVE A SHOWER?

EXAMINER ASKED:

- WHAT ARE YOU GOING TO DO FOR HIM?

- HOW TO DO THE SLING?

- FOR HOW LONG SHOULD THE PATIENT USE THE SLING?


MELBOURNE 11/02/2006

MEDICINE

1./ HEART EXAMINATION.

2/ A WOMAN CAME TO YOU BECAUSE SHE HAD SLURRED SPEED AND UPPER LIMB
WEAKNESS 2 HOURS AGO. NOW, SHE IS NORMAL. YOUR TASKS ARE:

€¢ TO DO PHYSICAL EXAMINATION?

€¢ TO DISCUSS THE DDX AND TESTS THAT SHOULD BE DONE TO CONFIRM THE
DIAGNOSIS WITH THE EXAMINER?

YOU DO THE UPPER LIMB EXAMINATION AND TALK TO THE PATIENT FOR 1 SECOND TO
PROVE SHE DOES NOT HAVE ANY SPEAKING PROBLEM. THE DIAGNOSIS IS TIA AND TO
DIFFERENTIATE WITH OTHER CAUSES, E.G. HEART BLOCK, HYPOGLYCAEMIA.

3/ A 35 YO MAN CAME TO YOU BECAUSE HE HAS DEVELOPED THE CHEST PAIN THAT
ONLY RADIATED TO HIS LEFT ARM AND HIS NECK. THE PAINFUL AREA IS IN THE BORDER
BETWEEN THE LEFT CHEST AND ABDOMEN. YOUR TASKS ARE.

€¢ TO TAKE THE HISTORY?

€¢ TO TAKE THE PHYSICAL EXAMINATION AND ORDER THE TESTS TO HAVE DDX?

THE SYMPTOM IS ONLY THE PAIN RADIATING TO THE LEFT ARM. THE REST IS NORMAL.

THE PHYSICAL SIGN HAS NOT BEEN DETECTED ANY ABNORMAL SIGN. SO, DDX ARE

A) ANGINA OR MI → ECG, TROPONIN ENZYME.

B) PE OR PNEUMOTHORAX→ CHEST X-RAY AND CT SCAN. CTSCAN IS THE BEST SINCE IT


CAN PROVE PE OR PNEUMOTHORAX QUICKLY AND ACCURATELY.

C) HERPES ZOSTER
4/ A MAN COMES TO YOU SINCE RIGHT AFTER EATING THE NUT, HE DEVELOPED
THROAT IRRITATION THAT WAS SUBSIDED AFTER INHALING ANTIHISTAMINE (I CANNOT
REMEMBER THE NAME OF MEDICATION). YOUR TASKS ARE:

€¢ TO DISCUSS WITH HIM ABOUT HIS PROBLEM, AS WELL AS ANOTHER SERIOUS
PROBLEM?

€¢ MX?

THIS IS A CASE OF NUT ALLERGY. YOU MUST REFER HIM TO THE IMMUNOLOGIST. YOU
ALSO WARN HIM ABOUT ANAPHYLACTIC SHOCK WITH SERIOUS SYMPTOMS AND SIGNS,
TALK TO HIM ABOUT EPI-PEN, THE INFORMATION OF NUT ALLERGY IN THE WALLET OR
BRACELET TO LET THE AMBULANCE OFFICER KNOWS HIS NUT PROBLEM.

Q: WILL I ALLERGY WITH OTHER FOOD?

A: IT€™S VERY GOOD QUESTION BUT THE SPECIALIST WILL DISCUSS WITH YOU WHEN
YOU MEET HIM.

5/ A MAN HAS MIGRATED TO A NEW AREA. HE COMES TO YOU WITH LIVER


ENLARGEMENT. THE US SHOW FATTY LIVER. HE DOES NOT DRINK, AND SMOKING. YOUR
TASK ARE

€¢ TO TAKE THE HISTORY?

€¢ TO ASK THE PHYSICAL EXAMINATION AND TEST RESULTS FROM THE EXAMINER?

€¢ DDX?

THIS CASE IS HAEMACHROMATOSIS→ BECAUSE HE ALSO HAS A HEART PROBLEM AND


SERUM IRON INCREASED?????- I WAS NOT VERY GOOD FOR THIS CASE.
SURGERY

1/A WOMAN COMES TO THE HOSPITAL BECAUSE OF A SERIOUS ABDOMINAL PAIN. HER
FAECES IS BROWN COLOUR. YOUR TASKS ARE:

€¢ TO TAKE THE HISTORY?

€¢ TO ASK THE PHYSICAL EXAMINATION, THE TEST RESULTS FROM THE EXAMINER?

€¢ MX?

THE WOMAN HAS 9/10 ABDOMINAL PAIN. HER ABDOMEN IS TOTALLY DISTENDED AND
RIGID. SHE DID NOT TAKE ASPIRIN OR NSAIDS OR OTHER MEDICATION THAT CAN CAUSE
GASTRIC BLEEDING. SHE ONLY HAS TAKEN THE MEDICATION FOR HER HEART. HER PULSE
IS IRREGULAR- IRREGULAR →GUT NECROSIS. IT WILL BE CONFIRMED BY CTSCAN AND
BARIUM ENEMA WITHOUT PREPARATION.

MX:

1. CALL SURGEON TO ADMIT TO THEATRE (CRITICAL ERROR IF YOU DO NOT KNOW).

2. MORPHINE, FLUID AND OTHER RESUSCITATION.

3. BLOOD TESTS AND OTHERS FOR PREPARING SURGERY.

2/ A MAN HAS HAD A LUMP IN HIS HEAD FOR 2 YEARS. YOUR TASKS ARE:

€¢ TO DO PHYSICAL EXAMINATION?

€¢ TO GIVE DDX?

IT IS A CASE OF LIPOMA. YOU MUST EXPLAIN TO THE PATIENT THAT THE LUMP IS NOT
LIKELY TO BE A CANCER LESION BECAUSE HE HAS HAD FOR 2 YEARS AND YOU CANNOT
FIND OUT ANY LYMPH NODES IN HIS NECK AND HEAD. REMOVING IT AND STUDYING
UNDER MICROSCOPE WILL GIVE AN ACCURATE ANSWER. REFER TO SPECIALIST.
3/ CLAVICULAR BROKEN BONE WITH X-RAY FILM THAT IS VERY CLEAR CUT. YOUR TASKS
ARE:

€¢ TO EXPLAIN THE PROBLEM?

€¢ TO DISCUSS THE COMPLICATION AND MANAGEMENT?

€¢ TO PERFORM THE SLING?

PAEDIATRICS

1/ A 2 YO BOY COMES TO YOU BECAUSE OF HYPOTENSION, COLD TEMPERATURE, AND


RASH THAT IS ILLUSTRATED IN THE PICTURE. HE HAS NO NECK STIFFNESS.

YOUR TASKS ARE:

€¢ TO EXPLAIN TO HIS FATHER WHAT HAS HAPPENED TO HIM?

€¢ MX?

THIS IS A CASE OF SEPTICAEMIA. EXPLAIN TO HIS PARENT ABOUT THE DISEASE. THEN,
YOU WILL CALL AMBULANCE IMMEDIATELY. DURING WAITING FOR THE AMBULANCE,
YOU DRAW OUT HIS BLOOD INTO THE TUBE, AND THEN INJECT CEFTRIAXONE 50MG/ KG
(EXPLAIN WHY YOU DO NOT USE PENICILLIN BECAUSE OF THE ANAPHYLAXIS IN SOME
PEOPLE.)

Q: DURING WAITING THE AMBULANCE, IF THE BOY€™S VEIN COLLAPSES, WHAT WILL
YOU DO?

A: FLUID TRANSFUSION INTO THE TIBIA, AND IM ANTIBIOTIC ARE THE BASIC
PROCEDURES IN THE COUNTRY-SIDE CLINIC.

Q: WHAT ELSE WILL YOU DO?

A: I WILL INFORM TO THE AUTHORITIES AND PRESCRIBE RIFAMPICIN TO HIS RELATIVE.


Q: WHAT IS THE S/E OF RIFAMPICIN?

2/ A NEW CASE WITH 2 EXAMINERS.

A BOY WHO DEVELOPS DM HAS BEEN DISCHARGED. YOUR TASKS TO DISCUSS WITH HIS
FATHER ARE:

€¢ WHAT IS THE KIND OF DM IN HIS SON?

€¢ MX

€¢ COMPLICATIONS? HOW TO AVOID?

PATIENT EDUCATION IS A GOOD SOURCE. YOU MUST SAY HIS SON€™S LIFE WILL BE
NORMAL IF THE SON FOLLOWS THE ADVICE FROM THE SPECIALIST, THE DIETICIAN AND
PHYSIOTHERAPIST (VERY IMPORTANT TO KEEP BMI IN THE NORMAL RANGE). HE CAN GO
OUT TO CAMP, TO PLAY SPORT WITH HIS TEACHERS€™ NOTICE. TALK BRIEFLY ABOUT
THE COMPLICATION, E.G. HYPER/HYPOGLYCAEMIA. DISCUSS SHORT AND LONG ACTING
INSULIN BRIEFLY AND LEAVE THE ANSWER FOR DM SPECIALIST SINCE HE WILL PRESCRIBE
SHORT OR LONG ACTING INSULIN.

DO NOT SCARE WHEN YOU SEE 2 EXAMINERS. LIKE THIS CASE, IT WAS EASY TO TALK.

3/ A MOTHER COMES TO YOU BECAUSE SHE WANT HER MALE BABY WILL HAVE
CIRCUMCISION?

CANDIDATE SHOULD ASK WHAT SHE KNOWS ABOUT THE CIRCUMCISION? THE MOTHER
WILL SAY SHE HAS HEARD FROM HER FRIEND.

CANDIDATE MUST SAY CIRCUMCISION IS NOT COMMON PROCEDURE IN AUSTRALIA


ANY MORE.
OBSTETRICS & GYNAECOLOGY

1/ THE DM YOUNG WOMAN COMES TO YOU BECAUSE SHE WANTS TO CONCEIVE. SHE
HAS USED INSULIN FOR A LONG TIME AND HER DM HAS BEEN CONTROLLED VERY WELL.
SHE HAS JUST MARRIED. YOUR TASKS ARE:

€¢ TO TAKE HISTORY?

€¢ COUNSELLING?

€¢ TO DISCUSS WHAT SHE WANTS TO KNOW?

DURING THE CONVERSATION, SHE IS NORMAL, NO PROBLEM OCCURS. THEN, SHE ASKS

Q: CAN I CONCEIVE? YES

Q: WHAT SHOULD I DO?

A:I WILL REFER YOU TO DM CLINIC SINCE YOU NEED A DM SPECIALIST, A DIETICIAN AS
WELL AS A PHYSIOTHERAPIST TO PREPARE YOU FOR YOUR PREGNANCY?

Q: WHAT WILL HAPPEN TO ME DURING THE PREGNANCY IF DM IS NOT CONTROLLED


WELL?

Q: WHAT WILL HAPPEN TO MY BABY IF DM IS NOT CONTROLLED WELL?

2/ A YOUNG WOMAN COMES TO YOU BECAUSE OF CONTRACEPTIVE PILL. YOUR TASKS


ARE:

€¢ TO DISCUSS WHAT SHE WANTS TO KNOW?

€¢ IF SHE WANTS, YOU CAN OUTLINE OTHER CONTRACEPTIVE METHODS?

YOU MUST DISCUSS ABOUT THE S/E, COMPLICATION OF THE PILL. DO NOT FORGET TO
MAKE SURE SHE IS NOT PREGNANT.
Q: CAN YOU CHOOSE THE PILLS THAT ARE SUITABLE FOR ME?

A: I CANNOT. I WILL INTRODUCE THE ADVANTAGES AND DISADVANTAGES OF EVERY


KIND OF CONTRACEPTION PILLS. THEN YOU SELECT THE PILL THAT IS THE MOST SUITABLE
FOR YOU.

Q: WHICH KIND OF PILLS YOU WILL PRESCRIBE FOR ME?

A: I WILL PRESCRIBE THE LOWEST DOSE OF PILL FOR YOU, SUCH AS MICROGYNON 20.

3/ A 35TH WEEK PREGNANT WOMAN GOT THE CONVULSION IN THE OUTPATIENT


CLINIC. NOW SHE IS CONSCIOUS. YOUR TASKS ARE:

€¢ TO TAKE THE HISTORY AND TO EXPLAIN WHAT HAPPENED TO HER MOTHER?

€¢ TO TAKE SOME SIGNS AND SYMPTOMS FROM THE EXAMINER?

€¢ TO MANAGE THE SITUATION?

THE MOTHER SAID HER DAUGHTER HAD HAD ANTENATAL CARE VERY CAREFULLY
WITHOUT ANY ABNORMALITY, ESP. BLOOD PRESSURE OR PROTEINURIA. SHE DID NOT
CONTRACT WITH EPILEPSY OR HYPOGLYCAEMIA. THE EXAMINER SAYS EVERY THING WAS
NOT AVAILABLE YET. YOU MUST KNOW THE RULE IN THIS CASE THAT IS THE PREGNANT
WOMAN MUST BE TREATED AS A CASE OF ECLAMPSIA UNTIL YOU CAN PROVE OTHER
CAUSES, ESP. EPILEPSY OR HYPOGLYCAEMIA (SEE THAT RULE IN THE ECLAMPSIA TOPIC
FROM THE ROYAL WOMEN HOSPITAL WEBSITE).

PSYCHIATRY

1/ A TEENAGER GIRL COMES TO YOU BECAUSE OF TIREDNESS. PHYSICAL EXAMINATION


IS NORMAL. YOUR TASKS ARE

€¢ TO TAKE THE HISTORY?

€¢ TO REPORT THE PSYCHIATRIC PROBLEMS TO THE EXAMINER?

€¢ TO DIAGNOSE AND MANAGE THE CASE?


THIS IS A CASE OF ANOREXIA NERVOSA IN THE YOUNG GIRL WHO EATS 3 MEALS/DAY
WITH ONLY VEGETABLE. SHE THINKS HER THIGHS ARE QUITE BIG. SHE HAS HAD
AMENORRHOEA. HER BMI IS 15 AND THE SIGNS OF DEHYDRATION. OTHERWISE IS
NORMAL.

REPORT TO EXAMINER ABOUT HER BEHAVIOUR DURING THE CONVERSATION, E.G. NOT
EYE CONTACT, SHE TRIED TO COVER HER BODY.

THE IMPORTANT SIGN FROM THE EXAMINER IS DEHYDRATION. SO, DX IS ANOREXIA


NERVOSA AND YOU MUST ADMIT HER IMMEDIATELY TO HOSPITAL BECAUSE OF
DEHYDRATION.

2/ A MAN WHO DEVELOPS PSYCHOTIC PROBLEMS WITH HALLUCINATION, DELUSION


AND ILLUSION. HE WAS ADMITTED TO HOSPITAL BECAUSE OF THE LACERATION IN HIS
LEFT WRIST. NOW, HE WANTS TO GO HOME AND SLEEPS WITH SLEEPING MEDICATION.
YOUR TASKS ARE

€¢ TO TALK TO THE PATIENT TO FIND OUT THE PROBLEMS?

€¢ MX AND TO DISCUSS WITH THE EXAMINER ABOUT HIS CONDITION?

INTRODUCTION, THEN YOU SHOULD TALK TO THE PATIENT TO SATISFY 3 THINGS:

€¢ YOU ARE HERE TO HELP HIM.

€¢ YOU DO EVERYTHING TO CALM HIM DOWN.

€¢ YOU PERSUADE HIM HARDLY TO STAY IN THE HOSPITAL.

THE PERFORMER DISAGREES AND ALWAYS WANTS TO GO HOME WITH SLEEPING PILLS.
SO, TALK TO THE EXAMINER ABOUT HIS RISK OF SUICIDE (HE CUT HIS WRIST BEFORE HE
WAS ADMITTED TO THE HOSPITAL), THEN YOU CALL CAT TEAM TO ADMIT HIM
INVOLUNTARY TO THE HOSPITAL.
Q: DURING WAITING FOR CAT TEAM, IF THE PATIENT BECOMES VIOLENT, WHAT WILL
YOU DO?

A: I WILL STAY WITH HIM DURING WAITING CAT TEAM. IF HE BECOMES VIOLENT, I WILL
RUN AWAY BECAUSE I MUST BE SAFE FIRST.

WHEN I HAVE MORE PEOPLE (IDEALLY, 6 PEOPLE (IN MURTAG)), I WILL COME BACK TO
INJECT HALOPERIDOL TO HIM.

Q: HOW TO LIMIT THE S/E OF HALOPERIDOL?

A: I HAVE TO USE THE MINIMUM DOSE.

11-FEB 2006 CLINICAL EXAMINATION MELBOURNE

PAEDIATRICS

1. 38/40 PREGNANT WOMAN COMES TO SEE YOU AS A GP AND ASKS ABOUT NEONATAL
CIRCUMCISION.

TASK: TELL YOUR PATIENT ABOUT THE MEDICAL OPINION AND ANSWER HER
QUESTIONS.

1ST PREGNANCY, NO RELIGIOUS REASON. JUST HUSBAND HAD IT DONE WHEN HE WAS
A CHILD.
Q €˜S FROM THE ROLE PLAYER:

WHAT ARE THE DISADVANTAGE AND ADVANTAGE OF CIRCUMCISION?

WHAT ARE THE CONTRAINDICATIONS?

WHEN IS THE BEST TIME TO HAVE IT DONE?

Q€™S FROM THE EXAMINER:

IF THE PATIENT INSISTS ON HAVING IT DONE FOR HER CHILD, WHAT ARE YOU GOING TO
DO?

ANSWER: IT IS PATIENT€™S CHOICE. I€™LL REFER€¦Â€¦.

CIRCUMCISION

2. YOU ARE A HMO IN THE HOSPITAL. A 9 YRS OLD BOY IS JUST DIAGNOSED OF TYPE I
DM. HE IS STILL IN THE HOSPITAL.

TASK: TALK TO THE FATHER AND ANSWER HIS CONCERNS.

HAVE YOU HEARD ABOUT DM? €“ YES. MY UNCLE IS A DIABETIC. I KNOW WHAT IT IS. I
JUST WANT TO KNOW HOW LONG HE NEEDS INSULIN.

CAN MY CHILD HAVE NORMAL LIFE?

CAN HE GO FOR CAMPING?

HOW ABOUT PLAYING SPORT IN SCHOOL?

DIABETES TYPE I JUVENILE €“ COUNSELLING


3. GP SETTING, 50 KM AWAY FROM CITY. A FATHER BROUGHT HIS 2 YRS OLD CHILD TO
SEE YOU. TEMP 40€™C. RASH (PICTURE FROM AMC€™S MCQ BOOK PAGE 118). OVER
THE LASE A FEW HOURS, THE CHILD WAS NOT INTERESTED IN EATING, LETHARGY AND
DROWSY. NO NECK STIFFNESS. NO HISTORY TAKING AND PHYSICAL EXAMINATION.

TASK: MANAGE THE CASE.

Q€™S FROM EXAMINER:

IF THE CHILD IS ALLERGY TO PENICILLIN, WHAT WILL YOU DO?

THE CHILD IS DEHYDRATED, HOW ARE YOU GOING TO GIVE FLUID? €“ PUTTING IV
CANNULA. WHAT ELSE? €“INTRAOSSEOUS INFUSION.

LETHARGIC FEBRILE 2 YR OLD

O& G

1. GP SETTING. YOU WERE INTERVIEWING A MOTHER OF 38/40 PREGNANT LADY WHO


JUST HAD A FIT IN THE WAITING ROOM. NOW SHE IS NOT FITTING, BUT A BIT DROWSY.

TASK: TALK TO THE MOTHER, TAKE THE FOCUS HISTORY AND ASK EXAMINER THE
FINDING, MANAGE THE SITUATION.

HISTORY AND EXAMINATION FINDINGS: 1ST EPISODE OF FIT. BP 180/110, URINE


DIPSTICK €“ PROTEIN +++.

Q€™S FROM EXAMINER:


IF YOU DON€™T HAVE MG SO4, WHAT ARE YOU GOING TO GIVE HER?

Q€™S FROM THE MOTHER:

CAN SHE HAVE ANY FIT AGAIN?

ANY DANGERS TO THE BABY?

WILL SHE HAVE HIGH BLOOD PRESSURE AFTER DELIVERY?

ARE YOU GOING TO THE HOSPITAL WITH HER?

ECLAMPSIA

2. GP SETTING. A 24 YRS OLD LADY, MARRIED, ASKED FOR DIFFERENT TYPES OF OCP.

TASK: HISTORY TAKING, ASK EXAMINER THE PHYSICAL FINDING AND GIVE HER ADVICE.

IN THIS CASE, NO CI OF OCP. SHE CAN HAVE COMBINED OCP. TELL HER 7 DAY€™S
RULE, NOT PROTECTING HER FROM STD

CONTRACEPTION REQUEST

3. GP SETTING. A 26 YRS OLD HAS 15 YRS HISTORY OF WELL CONTROLLED IDDM. SHE
WANTED TO GET PREGNANT.

TASK: GIVE HER ADVICE AND ANSWER HER QUESTIONS.

Q€™S FROM THE ROLE PLAYER:


I WANT TO KNOW HOW DM AFFECTS MY PREGNANT AND HOW PREGNANT AFFECTS
MY DM.

IF I BECOME PREGNANT, ARE YOU GOING TO LOOK AFTER ME?

Q€™S FROM THE EXAMINER:

WHEN ARE YOU GOING TO REFER HER TO ENDOCRINOLOGIST, BEFORE PREGNANT OR


AFTER?

WHAT DO YOU NEED TO CHECK BEFORE SHE BECOMES PREGNANT?

PRE-PREGNANCY COUNSELLING €“ DIABETIC

PSYCHIATRY

1. GP SETTING. A MOTHER OF 16 YRS OLD GIRL CONCERNED ABOUT HER


DAUGHTER€™S WEIGHT. LAST WEEK THE WEIGHT WAS 45KG BUT NOW IS 40 KG. NOW
YOU ARE SEEING THE GIRL.

TASK: TAKE A HISTORY AND ASK EXAMINER THE FINDING

TELL EXAMINER ABOUT YOUR DIAGNOSES AND THE MANAGEMENT.

FINDING €”BMI 14, FINE HAIR, DEHYDRATION, POSTURAL BP DROP. NO IX RESULT.

ANOREXIA NERVOSA

2. GP SETTING, A YOUNG MAN HAD A RIGHT WRIST INJURY AND LIVED WITH OTHER
YOUNG PEOPLE. YOU DID A LIMITED MENTAL EXAMINATION AS FOLLOWING.
HE HAD PROBLEMS WITH HIS PREVIOUS EMPLOYER, POOR SLEEP, LACK OF INSIGHT,
DENIED ANY SUICIDAL IDEATION. HE WAS HAVING HALLUCINATION AND PARANOID.
(LONG STEM).

NO MORE HISTORY TAKING AND EXAMINATION.

TASK: MANAGE THE CASE.

Q€™S FROM THE EXAMINER:

WHAT DO YOU THINK ABOUT THE DIAGNOSIS?

PATIENT REFUSES TO GO TO HOSPITAL, WHAT ARE YOU GOING TO DO?

UNDER WHICH LAW YOU CAN ADMIT THE PATIENT?

WHILE YOU ARE WAITING FOR AMBULANCE, WHAT MEDICATION ARE YOU GOING TO
GIVE? €“ DIAZEPAM. WHICH ROUTE? -IM.

FOR OTHER SYMPTOMS, WHICH MEDICATION ARE YOU GOING TO GIVE? €“
HALOPERIDOL.

THIS IS A YOUNG MAN, IF YOU GIVE HALOPERIDOL, WHAT DO YOU EXPECT? €“
EXTRAPYRAMIDAL SIDE EFFECT. WHAT MEDICATION TO REVERT IT?

WHAT ARE YOU GOING TO DO IF THE CAT TEAM COMES LATE?

PSYCHOTIC EPISODE WITH VIOLENCE €“ CRISIS MANAGEMENT

MEDICINE AND SURGERY

1. GP SETTING. A MIDDLE AGE LADY WANTS TO GO FOR OVERSEAS AND COMES FOR
CHECK UP. SHE HAS HEART MURMUR FOR SEVERAL YEARS, WHICH WAS TOLD BY HER
PREVIOUS GP. SHE WAS ASYMPTOMATIC.
TASK : DO THE PHYSICAL EXAMINATION AND TELL THE EXAMINER ABOUT YOU
DIAGNOSIS.

PANSYSTOLIC MURMUR AT THE MITRAL VALVE AREA.

Q€™S FROM EXAMINER:

WHAT IS THE CAUSE OF MR?

CARDIAC MURMUR

2. ONE HOUR AGO A 46 YRS OLD LADY HAD A DIFFICULTY IN SPEAKING AND WEAKNESS
IN RIGHT ARM. WHEN SHE WAS IN THE WAITING ROOM THE SYMPTOMS RESOLVED.
PATIENT €˜S MOTHER HAD A STROKE.

TASK: EXAMINE THE RELEVANT SYSTEMS.

DYSPHASIA & UPPER LIMB WEAKNESS

3. GP SETTING. A PATIENT HAS A LUMP FOR SEVERAL YEARS. SLIGHTLY INCREASE IN SIZE
AND ASYMPTOMATIC.

TASK: TAKE A FOCUS HISTORY AND DO AN EXAMINATION.

Q€™S FROM EXAMINER:

WHAT ELSE DO YOU NEED TO EXAMINE? €“ LYMPH NODES.

CAN IT BECOME CANCER?

WHAT IS THE COMPLICATION OF THE LUMP? €“ INFECTION

HOW DO YOU REMOVE IT?


SUBCUTANEOUS LUMP

4. GP SETTING. A PATIENT COMES FROM OTHER CITY. A LETTER FROM HIS PREVIOUS GP
SAID THAT HIS LFT€™S WAS ABNORMAL. HE WAS NOT AN ALCOHOL DRINKER, HAD NO
HISTORY OF INFECTIOUS LIVER DISEASE. HE HAD PERMANENT PACEMAKER INSERTION A
FEW YEARS AGO FOR THE BRADYCARDIA. LFT€™S WAS GGT , ALT,AST ALL INCREASED.
YOU HAVE ORDERED SOME TESTS.

TASK: ASK THE EXAMINER ABOUT THE IX RESULT AND TALK TO THE PATIENT.

Q€™S FROM THE EXAMINER:

HOW OFTEN IS THE VENESECTION DONE?

WHAT DIET?

HOW ABOUT THE SIBLINGS AND CHILDREN OF THE PATIENT?

ABNORMAL LIVER FUNCTION TESTS

5. A YOUNG MAN HAD A HISTORY OF ECZEMA AND ASTHMA. LAST NIGHT IN THE PARTY
AFTER EATING A WALNUT HE STARTED HAVING WHEEZING, SOB AND URTICARIA. HE HAD
PEANUT ALLERGY AT 7, 9 AND 17 YEARS OLD. YOUR DIAGNOSIS WAS NUT ALLERGY.

TASK: EXPLAIN TO HIM ABOUT THE DIAGNOSIS AND HOW TO PREVENT IT HAPPENING
AGAIN, WHAT HE CAN DO NEXT TIME. WHAT TEST DO YOU CONFIRM THE DIAGNOSIS.

Q€™S FROM ROLE PLAYER:

IF IT HAPPENS AGAIN, CALL AN AMBULANCE FIRST OR GIVE EPIPEN FIRST?


NUT INDUCED ANAPHYLAXIS

6. HMO IN ED. A 35 YR OLD MAN COMPLAINS OF LEFT CHEST PAIN.

TASK: HISTORY TAKING AND GIVE YOUR DDX.

PLEURITIC CHEST PAIN

7. ACUTE ABDOMEN PAIN.

TASK: TAKE A FOCUS HISTORY. (PT ON DIGOXIN, SOME BRIGHT RED BLOOD IN THE
STOOL)

ASK EXAMINER THE FINDING ( IRREGULAR PULSES, GENERALISED TENDERNESS OF


ABDOMEN, SLIGHT REBOUND, OCCASIONAL BOWEL SOUNDS.)

TELL WHAT YOU ARE GOING TO DO

ACUTE ABDOMEN

8. A 25 YR OLD MAN FELL ON OUTSTRETCHED HAND. YOU DID THE EXAMINATION.


THERE WAS TENDERNESS OVER THE RIGHT CLAVICLE.

TASK: READ X RAY

TELL WHAT YOU ARE GOING TO DO AND MANAGE THE CONDITION WITH THE
MATERIALS PROVIDED.
ALL MATERIALS INCLUDED A SLING AND THE ONES FOR FIGURE 8.

Q €˜S FROM THE ROLE PLAYER:

HOW LONG DO I HAVE IT ON? €“ 3 WEEKS

DO I HAVE A LUMP OVER THE COLLAR BONE?

CAN I GO TO WORK?

FRACTURED CLAVICLE

12/11/2005 MELBOURNE AMC CLINICAL EXAM RECALLED BY LING

MEDICINE AND SURGERY

1. A 40 YEARS OLD LADY, AN IMPORTANT PLAYER IN ORCHESTRA, C/O PAIN, SAUSAGE


SWELLING AND STIFFNESS IN BOTH HANDS RECENTLY AND WAS TESTED FOR ANA, ESR
AND RF AND THE RESULTS SHOWED THAT SHE HAS EARLY RHEUMATOID ARTHRITIS.
TODAY SHE COMES TO YOU (A GP) FOR THE RESULT AND TELL HER YOUR MX.

PATIENT WAS SHOCKED AND KNOWS THAT IT€™S A VERY SEVERE SITUATION BECAUSE
HER MOTHER IS 62 Y/O AND HAS RA FOR MANY YEARS ( I EXPLAIN €˜ EARLY STAGE,
MODERN MEDICATIONS AND MORE THERAPY WILL KEEP THE FUNCTION WELL, IT VARIES
INDIVIDUALLY, VARIOUS RANGES. I WILL REFER YOU TO THE RHEUMATOLOGIST€™ AND
TOTALLY FORGOT THE MAGIC WORDS: EARLY INTERVENTION AND MULTI-DISCIPLINARY
TEAM MANAGEMENT.

QUESTIONS:

CAN I CONTINUE MY CAREER (YES)


WHAT MEDICATIONS YOU WOULD GIVE ME ( FIRST BROFEN AND TAKE REST)

SHE SAID, I REALLY CAN€™T TAKE ANY REST, ANY OTHER MEDICATION CAN RELIEVE
THE PAIN QUICKER STEROIDS-SHE€™S SHOCKED, €˜DR, NO STEROIDS. MY MOM
TOOK IT AND I KNOW THERE ARE A LOT OF SE. (YES, BUT IN SHORT TERM. IT SHOULD BE
OK, BUT ANYWAY I WILL REFER YOU TO THE SPECIALIST.) GIVE LEAFLET AND SUGGESTED
THE INTERNET. THIS CASE LOOKS EASY BUT I WAS NOT SURE THE DETAILS OF THE
DISEASE- MODIFYING AGENTS.( I FAILED THIS STATION.)

2. A 28 Y/O MALE FELL HIS LEFT WRIST ON THE FLOOR YESTERDAY AND STILL SORE
TODAY. HE CAME TO ED AND YOU€™RE A ED HMO. TASK: FOCUSED P/E, ORDER YOUR
INX AND MX

ON P/E, TENDERNESS IN THE SNIFFBOX-------I€™LL DO A X-WAY

EXAMINER ASKED WHAT€™S YOUR POSSIBLE DX? (SCAPHOID #)

HERE IS THE X-WAY ( IT€™S A SCAPHOID #). THIS IS A PICTURE FROM AMC WHICH IS
TOO CLEAR TO BE MISSED.

TALK TO THE PATIENT. (I EXPLAINED AND REFER HIM TO THE ORTHOPAEDICS SURGEON)

HIS QUS:

WHAT ARE THEY GOING TO DO FOR ME? (THEY WILL PUT A PLASTER FOR YOU)

HOW LONG DO I NEED TO HAVE IT ( ABOUT 6-8 WEEKS. BUT THEY WILL ASSESS YOU
REGULARLY. FIRST THEY MAY SEE YOU IN THE NEXT 24 HOURS TO CHECK THE PLASTER,
THE BLOOD CIRCULATION AND THEN THEY MAY CHECK IN 2 WEEKS WHEN THE SWELLING
FADES AWAY AND SOMETIMES THE BONE MAY DIE BECAUSE OF THE POOR BLOOD
CIRCULATION.)

YOU MEAN MY HAND WILL RETURN TO NORMAL AFTER 8 WEEKS. (IT DEPENDS. THE
SURGEON WILL LOOK AFTER YOU AND I THINK YOU WILL BE FINE.)

(THEN I ASKED €˜DO YOU HAVE ANY OTHER CONCERNS) NO.

THEN EXAMINER REMAINDERED ME TO WASH MY HANDS.


3. POORLY-CONTROLLED DM FOR SEVERAL YEARS COMES TO YOU FOR CHECK UP. DO
RELEVANT P/E ON HIS LOWER LIMBS. YOU MAY BE INTERRUPTED BY THE EXAMINER
BECAUSE OF TIME SAVING.

EXAMINER ASKED:

WHAT DO YOU WANT TO CHECK FIRST (PERIPHERAL VASCULAR DISEASE), GO AHEAD.


(I DID INSPECTION AND PALPATION FOR PULSES.)

WHAT€™S OTHER CHANGE? (PERIPHERAL NEUROPATHY),GO AHEAD ( I STARTED


WITH PAIN SENSATION CHECK FOR STOCKING DISTRIBUTION WHICH DISAPPEARED
BELOW THE MID-TIBIA, THEN VIBRATION LOSS BELOW KNEE AND ALSO PROPRIOCEPTION
LOSS ON BOTH BIG TOE JOINTS ON A ROLE PLAYER.

I HESITATED AND ASKED WHETHER I NEED TO DO THE POWER AND TONE. DO AS YOU
WANT. (SO WHEN I WAS DOING THE REFLEXES THE BELL RANG).

4. A 60 Y/O MALE, WHO IS YOUR INFREQUENT PATIENT, CAME TO YOU (GP) TODAY
WANTED TO ENQUIRY ABOUT HIS WIFE BECAUSE HE FOUND SHE IS GETTING MORE
FORGETFUL RECENTLY. HIS WIFE, A 60 Y/O LADY, YOUR FREQUENT PATIENT, WAS SEEN BY
YOU 3 DAYS AGO, COMPLAINING THAT HER HUSBAND FOUND HER MORE FORGETFUL
THAN BEFORE. YOU TOOK HX AND P/E HER AND FOUND SHE WAS NORMAL.

TASK EXPLORE THE HUSBAND REGARDING THE MARITAL ISSUE

RESPONSE TO THE HUSBAND€™S CONCERN

FROM THE BEGINNING, HE COMPLAINED HIS WIFE AND ASKED €˜WHAT DID YOU FIND
WRONG WITH MY WIFE. SHE DIDN€™T TELL ME ANYTHING ABOUT IT€™ (I AM SORRY I
CAN€™T TELL YOU ANY THING ABOUT YOUR WIFE€™S CONDITION WITHOUT HER
WRITTEN PERMISSION. IT€™S PATIENT€™S CONFIDENTIALITY.)

O ( HE€™S NOT HAPPY) WHY NOT? I AM HER HUSBAND. IT€™S ME TO LOOK AFTER
HER ALL THE TIME? (I CAN UNDERSTAND. BUT I CAN€™T. WHAT DO YOU FIND WRONG
WITH YOUR WIFE?)

O SHE €˜S TERRIBLY FORGETFUL, A LOT OF TROUBLE. I AM AFRAID SHE MAY MAKE
FIRE AT HOME WHEN SHE COOKS. I THINK SHE NEEDS TO STAY IN HOSTEL. DOCTOR, I
SHOULD DO SOMETHING EARLIER TO PUT HER ON THE WAITING LIST.( I UNDERSTAND.
BUT IT MUST BE ASSESSED BY THE AGED CARE ASSESSMENT TEAM. I CAN€™T DO THE
ASSESSMENT.)

O I KNOW. THAT€™S WHY I CAME TO YOU. I NEED YOUR REFERRAL. (OH, TERRIBLE
CASE.)

O (I KNOW. BUT ABOUT THIS ISSUE SHOULD ASK YOUR WIFE TO MAKE THE DECISION.
SHE IS CONSCIOUS NOW. SHE HAS THE RIGHT TO MAKE HER OWN DECISION) (THIS TIME
HE DIDN€™T COME BACK STRAIGHTAWAY. AND I CAN ASK QU: HOW ABOUT YOUR
RELATION WITH YOUR WIFE?)

O QUITE GOOD. WE€™VE BEEN MARRIED FOR 30 YEARS. SOMETIMES, UPS AND
DOWNS, BUT I THINK EVERY COUPLE ARE THE SAME. (DO YOU THINK IT IS AS FRESH AS
BEFORE, LIKE THE EARLY TIME?)

O ( LAUGHING) NO, DR, WE€™VE BEEN MARRIED FOR 30 YEARS ALREADY. (DO YOU
HAVE ANY CHILDREN? HOW DO THEY THINK ABOUT YOUR WIFE?)

O THEY AGREE WITH ME. (I GAVE UP THE MARITAL ISSUE. CAN YOU COME TO SEE ME
NEXT TIME WITH YOUR WIFE TOGETHER?)

O I DON€™T THINK SHE WOULD COME. LAST TIME I FORCE HER TO COME. (ANYWAY, I
CAN€™T GIVE YOU ANY INFORMATION ABOUT YOUR WIFE. I HAVE TO REPEAT AGAIN
AND AGAIN. FINALLY THE BELL RANG.)

5. A 60 Y/O MALE C/O EPISODES OF PALPITATION AND DIZZINESS RECENTLY. HIS BP


155/96. I CAN€™T REMEMBER THE OTHER DETAILS. (? ED OR GP SETTING)

TASK: FOCUSED HX

P/E AND INVESTIGATION FROM EXAMINER

READ AN ECG AT 7TH MINUTE

THIS IS A QUITE CONFUSING CASE. I THINK I ASKED TOO MUCH HX.

HX, PALPITATIONS SOMETIME HAPPEN AT NIGHT, NOT RELATED TO EXERTION, NO


CHEST PAIN (?). NO TINNITUS AND SPINNING FEELING. NO TARGET ORGAN DAMAGE
SYMPTOMS (VISUAL PROBLEM, SOB, PAIN IN CALF OR BODY WEAKNESS). NO HX OF
HEART ATTACK OR STROKE OR THYROID DISEASE. NO MEDICATIONS. SMOKING BY 20/DAY
AND ALCOHOL BY 4-5 GLASSES OF WINE SINCE TEENAGE. DRINKING A LOT OF CAFFEINE.

P/E: BP155/96, P150, REGULARLY, HEART AND LUNGS ARE ALL NORMAL.

EXAMINER ASKED: WHAT€™S THE LIKELY CAUSE (TACHYCARDIA)

WHICH ONE (MAYBE SINUS TACHYCARDIA, NOT SURE). HOW IS THE ECG? (ECG WAS
GIVEN. IT€™S AN ATRIAL FLUTTER.)

WHICH DEGREE OF THE BLOCK(4 TO 1).( BELL RANG).

6. A 51 Y/O CAME TO ED WITH SEVERE RIGHT UPPER ABDO PAIN.

TASK: FOCUSED HX , DDX

ANSWER PATIENT€™S QU

AGAIN, A TYPICAL ACUTE CHOLECYSTITIS CASE.

7. A 30 Y/O MALE CAME TO ED, C/O TERRIBLE HEADACHE SINCE YESTERDAY AND
GETTING WORSE.

TASK: FOCUSED HX

P/E FROM EXAMINER

MX

QU: WHAT€™S THE REASONS FOR MY HEADACHE?(DDX)( THEN TALK TO THE


EXAMINER)

P/E (T: 38.5℃, NECK STIFFNESS, NO RASHES)

WHAT ARE YOU GOING TO DO FOR THE PATIENT (CONSULT MY REG OR SENIOR DR.
ADMIT THE PATIENT, GIVE AB.)

WHAT INVESTIGATION( FBE, BLOOD CULTURE)


WHAT ELSE? (CT HEAD). WHY? (RULE OUT BLEEDING). NO BLEEDING, WHAT ELSE?
(LUMBER PUNCTURE). WHAT TESTS ARE YOU GOING TO DO WITH THE FLUID (PRESSURE,
BACTERIA STAINING, CULTURE AND BIOCHEMISTRY). OK, PATIENT HAS A QU.

HOW IS ABOUT MY TWO KIDS (9 AND 11 YEARS OLDS) ( I WILL GIVE ALL YOUR FAMILY
MEMBERS RIFIMPICIN FOR PREVENTION), THE BELL RANG. I HADN€™T FINISHED THE
WHOLE MX ( NOTIFICATION).

8. A YOUNG MALE HAD BLOODY DIARRHEA FOR 6 MONTHS AND HAD COLONOSCOPY
DONE TODAY. UNFORTUNATELY, THE SURGEON JUST SAID THE CHANGES EXIST IN THE
WHOLE LARGE BOW WHICH SUGGESTS ULCERATIVE COLONITIS WITHOUT GIVING A
REPORT. THE PATIENT BROUGHT THE PICTURE OF HIS COLON CHANGE TO YOU (A GP).
TASK:TALK TO PATIENT.

EXPLAIN ( LIFETIME SITUATION, MULTIDISCIPLINARY MX, IT CAN BE WELL-CONTROLLED)

ADMIT TO HOSPITAL AT THIS MOMENT. QU FROM ROLE PLAYER:

HOW ABOUT MY DIET ( LOW FIBRE DIET)

ANY OPERATION NOW ( NOT THIS MOMENT, SOMETIMES MAY NEED SURGEON)

CAN IT BE OTHER SITUATIONS ACCORDING TO THE PICTURE? (THE PICTURE SUGGESTS


INFLAMMATION, CAN BE CAUSED BY INFECTIVE INFLAMMATION, LIKE GERM INFECTION
OR CROHN€™S DISEASE. BUT THE DISTRIBUTION OF THE INFLAMMATION CAN TELL THE
DIFFERENCE).

WHAT€™S THE COMPLICATIONS (BLEEDING, PERFORATION AND HIGHER RISK OF


BOWEL CA. OTHER ORGANS CAN BE AFFECTED, SUCH AS EYE PROBLEMS, MOUTH ULCER,
SKIN CHANGES AND LIVER PROBLEMS.

O+G

9. A 28 YEARS OLD LADY COME TO YOU FOR ADVICE OF NEXT PREGNANCY REGARDING
THAT SE HAD AN ABORTION 2 YEARS AGO.

TASK: RELEVANT HX

ADVERSE EFFECT TO THE NEXT PREGNANCY AND EVALUATION。


I ASKED AND GIVE POSSIBLE EFFECT ON THE FUTURE PREGNANCY:

HOW MANY WEEKS OF PREGNANCY(14/52), WHICH HOSPITAL (RWH),

ANY FEVER AFTER THAT(NO),WHEN GET HER FIRST PERIOD AFTER THE PROCEDURE( A
MONTH);

ANY CHECK UP BY ANY OBSTETRICIAN (YES), HOW ABOUT THE PERIODS SINCE THEN (
AS NORMAL AS BEFORE, ABOUT 4 WEEKS A CYCLE).

BLOOD GROUP (€ŒO€ NEGATIVE), ANY ANTI-D AFTER THE PROCEDURE (NOT
SURE),

SAME PARTNER (YES),

ANY DISEASES: DM, HTN, BLOOD CLOTTING PROBLEMS, SMOKING, ALCOHOL DRUGS

QU FROM ROLE PLAYER: WILL IT AFFECT MY FERTILITY?

10. A 50 Y/O LADY C/O HOT FLUSH AND IRREGULAR PERIODS FOR 11 MONTHS. NO
WEIGHT LOSS OR DISCHARGE. TWO CHILDREN (20 AND 22 Y/O). ALL P/E ARE NORMAL.

TASK: MX (NO FURTHER HX AND P/E)

I WAS ON THE WRONG TRACK AND GAVE HER OCP, SO I FAIL THIS STATION EASILY AND
COMPLETELY.

11. (GP SETTING) A 25 Y/O PH D STUDENT WAS 8 WEEKS OF PREGNANCY. SHE


COMPLAINED RIGHT LOWER INTERMITTENT ABDO PAIN FOR A WEEK.

TASK: FOCUSED HX, P/E AND INX FROM EXAMINER, MX

HX: A DULL PAIN AT 5-6/10, NO FEVER, NO DISCHARGE, NO BURNING SENSATION OR


FREQUENCY PASSING URINE, NO DIARRHEA, NO PREGNANCY REACTION SYMPTOMS.1ST
PREGNANCY, PLANNED, NO PHX OF PID OR STD. PAP SMEAR IS NORMAL. NO SMOKING
OR ALCOHOL.

P/E: VITAL SIGNS ARE NORMAL. ONLY SUPERFICIAL TENDERNESS IN THE RIGHT LOWER
ABDO. US: NORMAL PREGNANT SAC IN UTERUS, WHICH IS COMPATIBLE WITH 8 WEEKS
OF GESTATION. A CYST OF 5 CM WAS FOUND IN THE RIGHT OVARY, NOT TORSION, NOT
RUPTURED. SPECULUM P/E: OS IS CLOSED AND PV P/E SHOWED TENDERNESS IN THE
RIGHT ADNEX.

MX: I WILL REFER YOU TO THE OBSTETRICIAN.

PATIENT ASKED:- THEY WILL REMOVE THE CYST NOW? (I DON€™T THINK SO).

PAEDIATRICS

12. IMMUNIZATION ADVICE FOR A MOTHER WITH A 6 WEEKS OLD CHILD (HER FIRST
CHILD). THE CHILD IS ALL NORMAL WITH P/E AND GROWING WELL. ROLE PLAYER€™ QU:

FIRST SHE ASKED ME TO TALK THROUGH THE SCHEDULE OF VACCINES FROM 6 WEEKS
TO PRE-SCHOOL

IS THERE ANY REACTION AFTER THE VACCINES ? (YES, SOME CHILDREN MAY HAVE
MILD FEVER OR UNWELL). -HOW TO PREVENT IT? (PARACETOMAL).

WHAT IS THE COMPLICATION OF PERTUSIS VACCINE (ENCEPHALOPATHY). IS THERE


ANY CONDITION YOU SHOULD NOT GIVE VACCINE (CHILDREN WITH ENCEPHALOPATHY
CAUSED BY PERTUSIS MUST NOT BE GIVEN ANY PERTUSIS VACCINE)

SOME PARENTS SUGGESTED TO USE DILUTED PROTEIN (A NATURAL WAY OF


IMMUNIZATION). IS THAT THE SAME AS THE IMMUNIZATION SCHEDULE (I HAVEN€™T
HEARD THIS BEFORE AND NOT SURE ITS EFFECT BUT I WILL CHECK THIS FOR YOU.

THEN SHE TOLD ME SHE HAS NO QU FOR ME. I SAID THAT €˜I AM NOT SURE ABOUT
THE DILUTED VACCINE. IS IT RECOMMENDED?€™ THE EXAMINER TOLD ME €˜NO, BUT
YOU SAID YOU WILL CHECK THE BOOK FOR HER. SO THAT€™S FINE

13. A YOUNG MOTHER BOUGHT HER 4 YEARS OLD DAUGHTER TO YOUR SUBURBAN
CLINIC, C/O THAT HER DAUGHTER HAS SOME RASHES IN HER GENITAL AREA IN THE LAST
SEVERAL WEEKS AND IT€™S GETTING WORSE. SHE DEVOICED FOR A WHILE AND
LOOKING AFTER HER DAUGHTER. AT WEEKEND HER DAUGHTER STAYS WITH HER EX-
HUSBAND BUT SHE REFUSED TO GO THERE YESTERDAY. SHE SUSPECTED HER DAUGHTER
WAS SEXUALLY ABUSED BY HER EX-HUSBAND. SHE HAS A NEW BOYFRIEND MOVED IN 5
MONTHS AGO.
TASK: GIVE THE MOTHER YOUR MX PLAN.

(IT LOOKS EASY. I NEED TO CALL GATEHOUSE NOW. THE QUESTION HERE IS HOW TO
EXPLAIN THIS 2-3 SENTENCE IN 8 MINUTES. I KEPT REPEATING THAT THEY WILL TAKE HX,
DO P/E AND DO INVESTIGATIONS. NOT SURE WHAT ELSE THEY WANT.)

14. A FATHER CAME TO YOU, WORRYING ABOUT HIS 5 MONTHS OLD SON. HE FOUND
ONE LUMP IN HIS RIGHT GROIN AREA, WHICH COMES OUT WHEN HE€™S CRYING AND
GOES BACK WHEN HE€™S SLEEPING AND ANOTHER LUMP IN HIS TUMMY BUTTON

TALK TO THE FATHER ABOUT THE MX (TOTALLY REPEATED CASE)

PSYCHIATRY

15. A 60 Y/O LADY WITH RIGHT LOWER ABDO PAIN FOR 6 WEEKS AND LOST 5 KG IN THE
LAST 6 WEEKS. SHE WENT TO SEE THE DOCTOR (P/E WAS NORMAL) AND WAS
INVESTIGATED, INCLUDING, BLOOD TESTS AND COLONOSCOPY, WHICH ARE NORMAL.
TODAY SHE CAME FOR THE RESULTS. ( GP SETTING)

TASK: TAKE HX FOR THE PSYCHIATRY CAUSE

GIVE THE PATIENT YOUR DDX

EXPLAIN THE NATURE OF HER CONDITION AND MX.

HX: THE RL ABDO PAIN FOR 18 MONTHS AND GETTING WORSE IN THE LAST 6 WEEKS.
NO PARTICULAR EVENT 6 WEEKS AGO. POOR SLEEP AND ALWAYS WAKES UP AT 3 AM
AND CAN€™T GO BACK TO SLEEP. APPETITE IS

BACK TO TOP
GOOD SAMARITIAN

GUEST

POSTED: THU SEP 28, 2006 4:43 PM POST SUBJECT:

---------------------

VADIMK, OCT 15, 2006

#1

VADIMK

VADIMK GUEST

BACK TO TOP

GORANI

GUEST

POSTED: FRI SEP 29, 2006 12:53 PM POST SUBJECT:

--------------------------------------------------------------------------------

ALL OF THE FOLLOWING ARE THE SIGNS OF ISCHAEMIA OF THE LOWER LIMB EXCEPT:

A .ULCER AT HALLUCES

B .INTERMITTENT CLAUDICATION
C .REST PAIN

D. ULCER ABOVE MEDIAL MALLEOLUS ***

E .HAIR LOSS ON LOWER LIMB

2. A PAINFUL LUMP IN THE GROIN OF SUDDEN ONSET IN AN OTHERWISE HEALTHY


YOUNG MAN IS MOST LIKELY TO BE?

A. DIRECT INGUINAL HERNIA

B .INDIRECT INGUINAL HERNIA

C .FEMORAL HERNIA

D. ENLARGED GROIN LYMPH NODE

E. SPIGELIAN HERNIA

I'LL GO FOR INDIRECT INGUINAL HERNIA(MORE LIKELY TO STRANGULATE)..DONT KNOW


ABT SPIGELIAN HERNIA..

3.A PATIENT HAD UNDERGONE A PARTIAL GASTRECTOMY 24 HOURS AGO. HIS FLUID
INPUT IS 2 LT AND URINEOUTPUTS IS 2 LT., NASO GASTRIC ASPIRATION- 700 ML AND
DRAIN 500 ML. HIS K IS 3.5, NA 130 AND CL IS 80.WHICH OF THE FOLLOWING IS THE MOST
APPROPRIATE FLUID REPLACEMENT REGIMEN?

A..2L N. SALINE + 1L 5% DEXTROSE + 50 MMOL KCI

B.2L N. SALINE + 2L 5% DEXTROSE + 50 MMOL KCI

C.3L N. SALINE + 1L 5% DEXTROSE + 50 MMOL KCI

D.3L N. SALINE + 2L 5% DEXTROSE + 100 MMOL KCI****

E.2L N. SALINE + 2L 5% DEXTROSE + 100 MMOL KCI

AGREEE
4. A 46 YEARS OLD WOMAN HAD LEFT MASTECTOMY DONE FOR BREAST CANCER TWO
YEARS AGO.NOW SHE COMES TO YOU WITH A PAINLESS, FIRM AND MOBILE LUMP IN HER
RIGHT BREAST. SHE IS OTHERWISE WELL. WHAT IS THE MOST LIKELY DIAGNOSIS?

A.NEW PRIMARY CANCER IN THE RIGHT BREAST

B METASTASES FROM THE PREVIOUS BREAST CANCER ***

C.BENIGN GROWTH

D.METASTASIS FROM ANOTHER MALIGNANT TUMOUR ELSEWHERE

E.LYMPH NODE SWELLING

THERE MUST BE SOME REMNANTS LEFT WHICH GAVE RISE TO METASTASIS

5. AN X RAY SHOWING DISLOCATION OF SHOULDER. WHICH OF THE FOLLOWING IS


CORRECT?

A. POSTERIOR DISLOCATION OF THE SHOULDER.

B.SENSORY LOSS OF THE LATERAL ASPECT OF THE ARM.**

C.WEAKNESS OF THE HAND

D.SENSORY LOSS OF THE MEDIAL ASPECT OF THE ARM.

E.FRACTURE OF THE HUMERUS

6 WHICH OF THE FOLLOWING IS MOST SUGGESTIVE OF DISRUPTION OF DEEP LAYERS


OF A POST OPERATIVE ABDOMINAL WOUND?

A. PERSISTENT PAIN IN WOUND

B.SEROSANGUINEOUS DISCHARGE

C. PERSISTENT ABDOMINAL DISTENSION

D.GAPING IN THE SKIN AFTER THE REMOVAL OF THE SUTURES ***

E.UNEXPLAINED FEVER AND TACHYCARDIA

NOT SURE
I WANT TO DISUSS ABOUT IUGR IN GEST. DM -

WILLIAM'S OBSTETRICS SAYS THAT THERE IN UNEXPLAINED FETAL DEMISE NOT KNOWN
TO BE DUE TO CAUSES LIKE IUGR,PLACENTAL INSUFFECIENCY ETC.

SO GUYS THERE IS "NO IUGR" IN GEST DM

PAED

A 14 YEARS OLD GIRL HAS BREAST ENLARGEMENT, NORMAL GROWTH OF PUBIC AND
AXILLARY HAIR BUT SHE IS STILL

NOT MENSTRUATING. WHICH OF THE FOLLOWING IS CORRECT?

A.SHE IS LIKELY TO START MENSTRUATING TWO YEARS AFTER THE START OF PUBIC
HAIR GROWTH

B.SHE IS LIKELY TO START MENSTRUATING TWO YEARS AFTER THE START OF AXILLARY
HAIR GROWTH

C.SHE IS LIKELY TO START MENSTRUATING WITHIN TWO YEARS FROM THE TIME OF HER
BREAST BUDDING.

D..SHE IS LIKELY TO START MENSTRUATING WITHIN TWO YEARS FROM GROWTH SPURT.

E.THIS IS A CASE OF DELAYED PUBERTY ***

THELARCHE IS FIRST TO COME(WHICH HAS ALREADY DEVELOPED).LAST TO DEVELOP IS


MENARCHE..SO THIS IS A CASE OF PUBERTAL DELAY

A 16 YEARS OLD GIRL PRESENTS WITH FEVER, HEADACHE AND RASH ON HER BODY. SHE
ALSO HAS PROFUSE BLEEDING FROM ALL VENIPUNCTURE SITES. WHAT IS THE MOST
PROBABLE DIAGNOSIS?

A.THROMBOCYTOPENIA

B.'DISSEMINATED INTRAVASCULAR COAGULATION

C.HENOCH SCHONLEIN PURPURA ***


D.ALLERGIC REACTION

WHY FEVER IN THROMBOCYTOPENIA?MOREOVER AGE GROUP BELONGS TO HSP.ALSO


KNOW THAT HSP IS ONE THE MOST COMMON CAUSE OF PURPURA IN THIS AGE GROUP

A 12 MONTHS OLD BABY HAS FEVER. RECENTLY, TWO OTHER FAMILY MEMBERS HAD
AN UPPER RESPIRATORY TRACT INFECTION. ON EXAMINATION OF THE BABY YOU FIND
THAT HIS BODY TEMPERATURE IS 37.5 CTAND CHEST AUSCULTATION REVEALS BILATERAL
WHEEZING. WHAT IS THE MOST LIKELY CAUSE?

A.ACUTE BRONCHIOLITIS ***

B.VIRAL CROUP

C.BRONCHIAL ASTHMA

D.PNEUMONIA

E.FOREIGN BODY

AGE GROUP SUGGESTS IT

A NEONATE WITH COUGH AND FEVER IS FOUND TO BE GRUNTING. CHEST X RAY


SHOWS ROUND LESIONS WITH LITTLE PLEURAL EFFUSION. WHICH OF THE FOLLOWING IS
THE BEST THE TREATMENT?

A.CRYSTALLINE PENICILLIN ??

B.GENTAMYCIN

D.AMOXICILLIN

E.FLUCLOXACILLIN ***

AS STAPH IS COMMON CAUSE OF PNEUMONIA IN THIS AGE GRP.PLZ CORRECT ME

A TEN YEARS OLD GIRL HAS A HISTORY OF REPEATED EPISODES OF HEMIPARESIS FROM
WHICH SHE RECOVERS COMPLETELY. WHAT IS THE MOST LIKELY DIAGNOSIS?

A.MIDDLE CEREBRAL ARTERY OCCLUSION


B.PETIT MAL EPILEPSY **

C.MIGRAINE ??

D.SUBARACHNOID HAEMORRHAGE

MIGRAINE IS UNLIKELY IN THIS AGE GRP BUT IT RARELY PRESENTS WITH


HEMIPARESIS...CAN HEMIAPARESIS BE PRESENT IN PETIT MAL EPILEPSY..IF YES THEN THIS
IS THE ANSWER

THE PARENTS WITH THEIR 10 YEARS OLD CHILD COME TO SEE YOU WITH COMPLAINTS
THAT THE CHILD HAS ENURESIS SINCE BIRTH. THE CHILD HAS NO PROBLEMS DURING THE
DAY TIME. PHYSICAL EXAMINATION DOES NOT REVEAL ANY OTHER ABNORMALITY.
WHICH OF THE FOLLOWING STATEMENTS IS TRUE IN MORE THAN 20 % OF SUCH CASES?

A.HE HAS CHRONIC PYELONEPHRITIS

B.HE HAS CONGENITAL ABNORMALITIES OF THE URINARY TRACT

C.THERE IS AN ASSOCIATED FAMILY HISTORY ****

D.HE HAS SMALL URINARY BLADDER.

E.IT IS ASSOCIATED WITH SPINA BIFIDA OCCULTA

ENURESIS IS FREQUENTLY FOUND IN PATIENTS WITH DISTURBED FAMILY ENV.

A 5 YEARS OLD BOY PRESENTS WITH VOMITING EVERY MORNING AND MORNING
HEADACHE FOR THE LAST TWO WEEKS. WHAT IS THE MOST LIKELY CAUSE?

A.MENINGITIS

B.MIGRAINE

C.CRANIOPHARYNGIOMA **

D.MEDUJLOBLASTOMA
E.WILM'S TUMOUR

WILL CHECK AND TELL BUT MOST LIKELY THE ANSEWER IS CRANIOPHA..

AN ADOLESCENT BOY COMPLAINS OF SLIGHT PAIN IN THE RIGHT KNEE WHICH GETS
WORSE AFTER EXERCISE. ON

EXAMINATION YOU FIND A LUMP ON THE TIBIA TUBERCLE WITH SLIGHT TENDERNESS.
THE BOY CAN'T REMEMBER IF

HE HAS HAD ANY RECENT TRAUMA. WHICH OF THE FOLLOWING STATEMENTS IS


CORRECT?

A.REASSURE THAT THERE IS NOTHING WRONG

B.BIOPSY SHOULD BE DONE TO EXCLUDE MALIGNANCY ***

C.KNEE ASPIRATE SHOWS ORGANISM

D. OSTEOCHONDRITIS IS THE MOST LIKELY CAUSE

A 6 DAY OLD BABY WHO IS SUSPECTED TO HAVE DOWN'S SYNDROME PRESENTS WITH
NON PROJECTILE BILESTAINED VOMITING FOR THE LAST TWO DAYS. ON ABDOMINAL
EXAMINATION YOU FIND AN OLIVE SHAPEDPALPABLE MASS IN THE EPIGASTRIUM. WHICH
OF THE FOLLOWING IS THE MOST LIKELY DIAGNOSIS?

A.VOLVULUS

B.DUODENAL ATRESIA.***

C.INTUSSUSCEPTION

D.MALABSORPTION
A TWO YEARS OLD CHILD PRESENTS WITH FAILURE TO THRIVE FOR THE LAST 6 MONTHS
AND RIGHT BUTTOCK WASTING. HE PASSES BULKY STOOLS 2-3 TIMES A DAY. WHICH OF
THE FOLLOWING IS THE MOST APPROPRIATE INVESTIGATION?

A.C13 BREATH TEST

B.DUODENAL BIOPSY ***

C.STOOL CULTURE

D.STOOL MICROSCOPY

E.SWEAT TEST

A DIABETIC MOTHER GIVES BIRTH TO A CHILD WEIGHT 4240 GM AND WITH AN APGAR
SCORE OF 7/10. ON EXAMINATION YOU FIND CYANOSIS OF HIS HANDS AND FEET AND THE
CHILD IS JITTERY. AXILLARY TEMPERATURE MEASURE SHOWS A READING OF 37.2 C. HIS
CONDITION IMPROVED AFTER MECONIUM ASPIRATION AND 100% O2. WHICH OF THE
FOLLOWING IS THE MOST LIKELY CAUSE?

A.HYPOGLYCAEMIA

B.HEART DISEASE ***

C.LUNG DISEASE.

D.HYPERGLYCAEMIA

E.HIS JAUNDICE IS CAUSED BY BREAST MILK.

CYANOTIC HEART DISEASE MAY BE THE CAUSE WHICH IS COMMON IN BABY OF


DIABETIC MOTHER

A SIX YEARS OLD BOY PRESENTS WITH DELAYED DEVELOPMENT OF SPEECH AFTER A
PERIOD OF NORMAL DEVELOPMENT. ON EXAMINATION YOU FIND THAT HE AVOIDS EYE
CONTACT AND REVEALS A HISTORY OF UNUSUAL LIKING FOR DINOSAURS. WHICH OF THE
FOLLOWING IS THE MOST LIKELY DIAGNOSIS?

A.AUTISM ***
B.ADHD

C.DEAFNESS

REGRESSION OF NORMAL MILESTONES WHICH ARE ALREADY DEVELOPED IS A FEATURE


OF AUTISM

A YOUNG WOMAN WITH ANOVULATORY CYCLES FOR THE LONG TIME IS MOST LIKELY
TO DEVELOP WHICH OF THE FOLLOWING?

A.CERVICAL CANCER

B.ENDOMETRIAL CANCER

C.OVARIAN CANCER ***

D.VAGINAL CANCER

E.CANCER OF THE VULVA

IN WHICH OF THE FOLLOWING CONDITIONS YOU WILL FIND DECREASED AMNIOTIC


FLUID?

A.FOETUS'S BLADDER NECK OBSTRUCTION OF THE FOETUS(RENAL AGENESIS)

B.T.O FISTULA

C.RHESUS INCOMPATIBITY

D.MULTIPLE PREGNANCY

EXPLANATION-SINCE BABY BATHES IN AMNIOTIC FLUID IT ABSORBS AND EXCRETS ONLY


AMNIOTIC FLUID.SO IN RENAL AGENESIS NO URINE IS PRODUCED THEREFORE AMNIOTIC
FLUID IS REDUCED(OLIGOHYDRAMNIOS)..IN ESOPHAGEAL ATRESIA AMNIOTIC FLUID IS
NOT ABSORBED CAUSING POLYHYDRAMNIOS
A 52 YEARS OLD OBESE POST MENOPAUSAL WOMAN PRESENTS WITH NO SIGNS OF
MENOPAUSE AND AMENORRHOEA. WHAT COULD BE THE CAUSE?

A LOW OESTROGEN AND LOW FSH

B HIGH OESTROGEN AND HIGH FSH

C. NORMAL OESTROGEN AND HIGH FSH

D.HIGH OESTROGEN AND LOW FSH ***

E.LOW OESTROGEN AND HIGH FSH

SINCE HOT FLUSHES AND OTHER SYMPTOMS ARE PRODUCED DUE TO HIGH AMT OF
FSH..HERE THE CASE IS REVRSED

A 16 YEARS OLD STUDENT ASKS FOR A PRESCRIPTION FOR CONTRACEPTIVE PILL. SHE
SAYS SHE DOESN'T HAVE SEXUAL PARTNER. WHICH OF THE FOLLOWING IS THE MOST
APPROPRIATE MANAGEMENT?

A. ORAL CONTRACEPTIVE PILL

B.MINIPILL

C.DEPOT

D.ORAL CONTRACEPTIVE PILL + CONDOMS

E.CALL HER PARENTS.

WHY DOES SHE NEED A CONTRACEPTIVE PILL IF SHE HAS NO PARTNER????

OCP IS THE BEST BET FOR EMERGENCIES

A PREGNANT WOMAN IN 1ST TRIMESTER IS FOUND TO HAVE CIN III. WHAT IS YOUR
NEXT STEP?

A. TERMINATE THE PREGNANCY


B. CONE BIOPSY

C.HYSTERECTOMY

D.COLPOSCOPY

E.OBSERVE

NOT SURE

14. A PREGNANT WOMAN IN 1ST TRIMESTER PRESENTS WITH SKIN DIMPLING OVER
THE RIGHT BREAST. EXAMINATION AND CLINICAL TESTS REVEALS CANCER OF THE RIGHT
BREAST. WHICH OF THE FOLLOWING IS YOUR MANAGEMENT?

A.TERMINATE THE PREGNANCY

B.START RADIOTHERAPY

C.WAIT TILL DELIVERY AND THEN START THE TREATMENT ???

D.SURGERY

IF DIMPLING IS PRESENT IT MEANS IT IS T4..WHY SURGERY IF T4...

A 19 YEARS OLD FEMALE COMES TO YOUR CLINIC FOR CONSULTATION. SHE HAS NEVER
HAD PERIODS. ON EXAMINATION HER BREAST DEVELOPMENT IS NORMAL. PUBIC AND
AXILLARY HAIR GROWTH IS ALSO NORMAL. THE

DEVELOPMENT OF HER GENITALS IS ALSO IN NORMAL RANGE. WHAT IS THE MOST


LIKELY CAUSE OF HER AMENORRHEA?

A. TURNER'S SYNDROME

B. PROLACTINOMA

C. ABSENT UTERUS ***D. PITUITARY LESION

E. LESION IN HYPOTHALAMUS
YOU CANT BE SURE WHETHER IT IS IMPERFORATE OR NOT

A 42 YEARS MAN WHO IS A PATIENT OF YOURS IS ARRESTED FOR ATTACKING HIS WIFE
IN A SUPERMARKET. THE POLICE COME TO YOU AND ASK YOU TO WRITE A REPORT
ABOUT THIS PATIENT. WHICH OF THE FOLLOWING IS TRUE FOR GOOD PRACTICE?

A.YOU WILL WRITE A REPORT ABOUT THE PATIENT'S BAD MENTAL STATE

B.YOU WILL TELL THE PATIENT NEVER TO COME TO YOUR SURGERY AGAIN

C.YOU WILL WRITE A GENERAL, UNBIASED AND BALANCED REPORT ABOUT HEALTH OF
THE PATIENT.

D.YOU WILL TELL THE POLICE THAT YOU DON'T WANT TO GIVE ANY REPORTS

E. ARRANGE FOR A MEETING WITH THE PATIENT

PLEASE TELL ME WHERE TO READ ETHICS FROM GUYS!!!

WHICH OF THE FOLLOWING WILL CAUSE SEROTONIN SYNDROME IF GIVEN TOGETHER


WITH SSRI LIKE FLUOXETINE EXCEPT?

A.L-TRYPTOPHAN

B.HALOPERIDOL

C.MOCLOBEMIDE

'D. CHLORPROMAZINE

E.CITALOPRAM

NO IDEA...WHAT IS SERTONIN SYNDROME


IN AUSTRALIA , ALMOST EVERY YEAR THERE ARE SEVERAL INCIDENCES OF BUSH FIRE.
USUALLY IT IS CAUSED BY

YOUNG PEOPLE AND TEENAGERS. WHICH OF THE FOLLOWING IS TRUE?

A. JUVENILE PYROMANIA IS THE MAIN CAUSE OF FIRES IN AUSTRALIA . ****

-B. IT IS DUE TO ACCIDENTAL FIRE CAUSED BY YOUNGSTER WHO GET SCARED AFTER
THE FIRE STARTS TO

SPREAD UNCONTROLLABLY AND THEY RUN AWAY.

C.IT HAS NO SERIOUS CONSEQUENCES

D.YOUNGSTERS LIKE TO IGNITE AND PLAY WITH FIRE.

E.IT IS DONE TO HIDE CRIMES

A SCHIZOPHRENIC WOMAN THINKS THAT SHE IS PERFECTLY WELL AND DENIES ANY
ILLNESS. SHE ALSO REFUSES TO

TAKE MEDICATION AND THINKS THAT SHE CAN TALK WITH ANGELS. WHAT IS THIS
CALLED IN PSYCHIATRY ?

A.LACK OF INSIGHT

B.OVERVALUED IDEA

C.REJECTION

D.DELUSION

E..FORMAL THOUGHT****
WHICH IS THE BEST NERVE IN GRAFTING?

A) MEDIAN NERVE ***

B)RADIAL NERVE

C) DIGITAL NERVE

THICKER NERVES ARE BETTER FOR GRAFTS

PATIENT PRESENTS WITH SUDDEN RUPTURE OF EXTENSOR POLLICIS LONGUS


TENDON,DX?

A) TENDONITIES***

B) TENOSYNOVITIS

C) RA

NOT VERY SURE BUT MORE LIKELY

A PATIENT WITH CANCER BREAST DEVELOP OSTEOPRPOSIS.WHAT IS THE TREATMENT

A) BISPHOSPHONATE ***

B) CYCLICAL ESTROGEN AND PROGESTERONE

C) CONTINUOUS ESTROGEN AND PROGESTERONE

INCREASE DIATERY CALCIUM 2000G / DAY


YOU ARE CALLED TO REVIEW A PATIENT WHO DEVELOPED RESTLESSNESS AND
DIFFICULTY BREATHING EIGHT HOURS

AFTER THYROIDECTOMY. WHAT WILL YOU DO FIRST?

A. ENDOTRACHEAL INTUBATION.

B.REMOVE SUPERFICIAL SUTURES

C.REMOVE SUPERFICIAL AND DEEP SUTURES ***

D.SALBUTAMOL INHALATION AS PATIENT PROBABLY HAS ASTHMA

A 43 YEARS OLD MAN WAS BROUGHT TO THE EMERGENCY DEPARTMENT WITH SEVERE
UPPER ABDOMINAL PAIN

AND SIGNS OF SHOCK. PHYSICAL EXAMINATION REVEALS ABDOMINAL WALL RIGIDITY


AND GUARDING. WHICH OF

THE FOLLOWING IS THE MOST LIKELY DIAGNOSIS?

A.PERFORATED GASTRIC ULCER ***

B.PENETRATING DUODENAL ULCER

C. MESENTERIC ISCHEMIA

D. ACUTE PANCREATITIS

PATIENT IS IN SHOCK WITH UPPER ABDOMINAL PAIN..GUARDING AND RIGIDITY IS


SUGGESTIVE OF PERITONITIS..

A YOUNG MALE FELL FROM A HEIGHT BUT SUSTAINED MINOR INJURIES. AFTER TWO
HOURS HE LOST

CONSCIOUSNESS AND WAS BROUGHT TO YOU IN NORTHERN QUEENSLAND HOSPITAL


WITH LIMITED RADIOGRAPHIC

FACILITIES BY HIS FRIEND. ON EXAMINATION YOU FIND THAT HIS PUPILS ARE FIXED AND
DILATED. WHAT IS OUR
NEXT STEP?

A.ARRANGE FOR TRANSPORTATION TO ANOTHER HOSPITAL BY HELICOPTER.

B.GIVE IV MANNITOL

C GIVE IV CORTICOSTEROID D. D BURR HOLES ***

CASE OF EDH

AN ELDERLY MAN CAME TO SEE YOU WITH COMPLAINTS OF SUDDEN PAIN IN THE
LOWER BACK WHILE LIFTING A HEAVY BOX. ON EXAMINATION YOU FIND THAT THE
LUMBAR SPINE HAS LIMITED MOVEMENT WITH PAIN IN HIS LOWER BACK ON
MOVEMENT. THERE ARE NO NEUROLOGICAL ABNORMALITIES. WHAT IS THE NEXT
INVESTIGATION THAT YOU'LL DO?

A.X RAY OF THE LUMBAR SPINE ***

B.CTSCAN

C.MRI OF THE SPINE

D.LUMBAR PUNCTURE

A 70 YEARS OLD WOMAN HAD UPPER GIB. SHE WAS ON TREATMENT WITH NSAID FOR
KNEE PAIN WHICH WAS

STOPPED A FEW MONTHS AGO. YOU DO AN ENDOSCOPY OF UPPER GL TRACT AND FIND
A DUODENAL ULCER WITH

RECENT BLEEDING. WHAT IS THE BEST MANAGEMENT?

A.OMEPRAZOL I.V. + AMPICILLIN + METRONIDAZOLE ***

B.OMEPRAZOL I.V.

C.HIGHLY SELECTIVE VAGOTOMY

D.VAGOTOMY
E.RANITIDINE I.V. + AMPICILLIN + METRONIDAZOLE

HIGHLY SELECTIVE VAGOTOMY WILL ONLY STOP THE ACID SECRETION.IDEALLY


SOMATOSTATIN INFUSION OR APPLICATION OF ADRENALINE LOCALLY SHOULD BE DONE
OTHERWISE CONVENTIONAL THERAPY FOR ULCER IS RECOMMENDED

WHICH OF THE FOLLOWING IS MOST COMMON CAUSE OF ACUTE PANCREATITIS?

A.GALLSTONES,

B.CHOLECYSTITIS

C.ALCOHOL ABUSE ***

D.SPICY FOOD

E.GASTROENTERITIS

WHAT IS THE MOST COMMON CAUSE OF DEATH AFTER BURN INJURY IN AUSTRALIA ?

A.SHOCK***

B.RESPIRATORY FAILURE

C.CARDIAC FAILURE

D.RENAL FAILURE

MOST LIKELY HYPOVOLEMIC SHOCK

PATIENT PRESENTS WITH SUDDEN RUPTURE OF EXTENSOR POLLICIS LONGUS


TENDON,DX?

A) TENDONITIES

B) TENOSYNOVITIS
C) RA

PROBABLY

A PATIENT WITH CANCER BREAST DEVELOP OSTEOPRPOSIS.WHAT IS THE TREATMENT

A) BISPHOSPHONATE

B) CYCLICAL ESTROGEN AND PROGESTERONE

C) CONTINUOUS ESTROGEN AND PROGESTERONE

D)INCREASE DIATERY CALCIUM 2000G / DAY

OBVIOUS CHOICE

YOU ARE CALLED TO REVIEW A PATIENT WHO DEVELOPED RESTLESSNESS AND


DIFFICULTY BREATHING EIGHT HOURS

AFTER THYROIDECTOMY. WHAT WILL YOU DO FIRST?

A. ENDOTRACHEAL INTUBATION.

B.REMOVE SUPERFICIAL SUTURES

C.REMOVE SUPERFICIAL AND DEEP SUTURES

D.SALBUTAMOL INHALATION AS PATIENT PROBABLY HAS ASTHMA

THIS IS HYPOPARATHYROIDISM SECONDARY TO SURGERY LEADING TO HYPOCALCEMIA


AND EVENTUALLY LIFE THREATENING LARYNGEAL EDEMA
A YOUNG MALE FELL FROM A HEIGHT BUT SUSTAINED MINOR INJURIES. AFTER TWO
HOURS HE LOST

CONSCIOUSNESS AND WAS BROUGHT TO YOU IN NORTHERN QUEENSLAND HOSPITAL


WITH LIMITED RADIOGRAPHIC

FACILITIES BY HIS FRIEND. ON EXAMINATION YOU FIND THAT HIS PUPILS ARE FIXED AND
DILATED. WHAT IS OUR

NEXT STEP?

A.ARRANGE FOR TRANSPORTATION TO ANOTHER HOSPITAL BY HELICOPTER.

B.GIVE IV MANNITOL

C GIVE IV CORTICOSTEROID D.

D BURR HOLES

THIS IS EPIDURAL HEMORRHAGE - SO NEED BURR HOLE OPERATION. CLUE - LUCID


INTERVAL FOLLOWED BY UNCONCIOUSNESS.

AN ELDERLY MAN CAME TO SEE YOU WITH COMPLAINTS OF SUDDEN PAIN IN THE
LOWER BACK WHILE LIFTING A HEAVY BOX. ON EXAMINATION YOU FIND THAT THE
LUMBAR SPINE HAS LIMITED MOVEMENT WITH PAIN IN HIS LOWER BACK ON
MOVEMENT. THERE ARE NO NEUROLOGICAL ABNORMALITIES. WHAT IS THE NEXT
INVESTIGATION THAT YOU'LL DO?

A.X RAY OF THE LUMBAR SPINE

C.MRI OF THE SPINE

D.LUMBAR PUNCTURE

SINCE THE PATIENT IS MORE THAN 50 YEARS OF AGE - THIS IS A RED FLAG SIGN
(DAVIDSON 19TH PAGE [SNIP]). SO PATENT HAS TO BE EVALUATED WITH INVESTIGATION.
PATIENT NEEDS BOTH XRAY AND MRI(DAVIDSON PAGE [SNIP]). SINCE IT IS THE NEXT
INVESTIGATION IN THE QUESTION - I GO FOR XRAY.

A 70 YEARS OLD WOMAN HAD UPPER GIB. SHE WAS ON TREATMENT WITH NSAID FOR
KNEE PAIN WHICH WAS

STOPPED A FEW MONTHS AGO. YOU DO AN ENDOSCOPY OF UPPER GL TRACT AND FIND
A DUODENAL ULCER WITH

RECENT BLEEDING. WHAT IS THE BEST MANAGEMENT?

A.OMEPRAZOL I.V. + AMPICILLIN + METRONIDAZOLE

B.OMEPRAZOL I.V.

C.HIGHLY SELECTIVE VAGOTOMY

D.VAGOTOMY

E.RANITIDINE I.V. + AMPICILLIN + METRONIDAZOLE

BLEEDING HAS STOPPED. SO NOW THE PATIENT NEEDS TREATMENT FOR HEALING OF
ULCER AS WELL AS ERADICATION THERAPY FOR H. PYLORI.

WHICH OF THE FOLLOWING IS MOST COMMON CAUSE OF ACUTE PANCREATITIS?

A.GALLSTONES,

B.CHOLECYSTITIS

C.ALCOHOL ABUSE

D.SPICY FOOD

E.GASTROENTERITIS
WHAT IS THE MOST COMMON CAUSE OF DEATH AFTER BURN INJURY IN AUSTRALIA ?

A.SHOCK

B.RESPIRATORY FAILURE

C.CARDIAC FAILURE

D.?RENAL FAILURE

ITS A GUESS. I ASSUME THAT BEING A COUNTRY WITH GOOD MEDICAL FACILITIES -
FLUID RESUSCITATION SHOULD BE AVAILABLE AND SHOCK SHOULD BE TREATED. BUT
RENAL FAILURE IN BURN CAN BE CAUSED BY SHOCK, MYOGLOBINURIA,
HEMOGLOBINURIA ETC.

PAED

A 14 YEARS OLD GIRL HAS BREAST ENLARGEMENT, NORMAL GROWTH OF PUBIC AND
AXILLARY HAIR BUT SHE IS STILL

NOT MENSTRUATING. WHICH OF THE FOLLOWING IS CORRECT?

A.SHE IS LIKELY TO START MENSTRUATING TWO YEARS AFTER THE START OF PUBIC
HAIR GROWTH

B.SHE IS LIKELY TO START MENSTRUATING TWO YEARS AFTER THE START OF AXILLARY
HAIR GROWTH

C.SHE IS LIKELY TO START MENSTRUATING WITHIN TWO YEARS FROM THE TIME OF HER
BREAST BUDDING.

D..SHE IS LIKELY TO START MENSTRUATING WITHIN TWO YEARS FROM GROWTH SPURT.

E.THIS IS A CASE OF DELAYED PUBERTY

REFERENCE ROBERTSON PAGE 156


A 16 YEARS OLD GIRL PRESENTS WITH FEVER, HEADACHE AND RASH ON HER BODY. SHE
ALSO HAS PROFUSE BLEEDING FROM ALL VENIPUNCTURE SITES. WHAT IS THE MOST
PROBABLE DIAGNOSIS?

A.THROMBOCYTOPENIA

B.'DISSEMINATED INTRAVASCULAR COAGULATION

C.HENOCH SCHONLEIN PURPURA

D.ALLERGIC REACTION

MENINGOCOCCAL INFECTION LEADING TO DIC

A 12 MONTHS OLD BABY HAS FEVER. RECENTLY, TWO OTHER FAMILY MEMBERS HAD
AN UPPER RESPIRATORY TRACT INFECTION. ON EXAMINATION OF THE BABY YOU FIND
THAT HIS BODY TEMPERATURE IS 37.5 CTAND CHEST AUSCULTATION REVEALS BILATERAL
WHEEZING. WHAT IS THE MOST LIKELY CAUSE?

A.ACUTE BRONCHIOLITIS

B.VIRAL CROUP

C.BRONCHIAL ASTHMA

D.PNEUMONIA

E.FOREIGN BODY

VIRAL AILMENT CAUSED BY RSV IN CHILDREN LEADING TO WHEEZING. SOME OF THESE


CHILDREN WILL GO ON TO DEVELOP ASTHMA WHEN ADULT.

A NEONATE WITH COUGH AND FEVER IS FOUND TO BE GRUNTING. CHEST X RAY


SHOWS ROUND LESIONS WITH LITTLE PLEURAL EFFUSION. WHICH OF THE FOLLOWING IS
THE BEST THE TREATMENT?

A.CRYSTALLINE PENICILLIN
B.GENTAMYCIN

D.AMOXICILLIN

E.FLUCLOXACILLIN

I AM NOT SURE. BUT CONSIDERING THAT E.COLI AND GM NEGATIVE ORGANISM ARE
THE COMMONEST BACTERIAL ETIOLOGY IN THIS GROUP - I GO FOR GENTAMYCIN.

A TEN YEARS OLD GIRL HAS A HISTORY OF REPEATED EPISODES OF HEMIPARESIS FROM
WHICH SHE RECOVERS COMPLETELY. WHAT IS THE MOST LIKELY DIAGNOSIS?

A.MIDDLE CEREBRAL ARTERY OCCLUSION

B.PETIT MAL EPILEPSY

C.MIGRAINE

D.SUBARACHNOID HAEMORRHAGE

DON'T KNOW. ANYONE?

THE PARENTS WITH THEIR 10 YEARS OLD CHILD COME TO SEE YOU WITH COMPLAINTS
THAT THE CHILD HAS ENURESIS SINCE BIRTH. THE CHILD HAS NO PROBLEMS DURING THE
DAY TIME. PHYSICAL EXAMINATION DOES NOT REVEAL ANY OTHER ABNORMALITY.
WHICH OF THE FOLLOWING STATEMENTS IS TRUE IN MORE THAN 20 % OF SUCH CASES?

A.HE HAS CHRONIC PYELONEPHRITIS

B.HE HAS CONGENITAL ABNORMALITIES OF THE URINARY TRACT

C.THERE IS AN ASSOCIATED FAMILY HISTORY

D.HE HAS SMALL URINARY BLADDER.

E.IT IS ASSOCIATED WITH SPINA BIFIDA OCCULTA


A 5 YEARS OLD BOY PRESENTS WITH VOMITING EVERY MORNING AND MORNING
HEADACHE FOR THE LAST TWO WEEKS. WHAT IS THE MOST LIKELY CAUSE?

A.MENINGITIS

B.MIGRAINE

C.CRANIOPHARYNGIOMA

D.MEDUJLOBLASTOMA

E.WILM'S TUMOUR

MORNING HEADACHE AND MORNING VOMITING SUGGESTIVE OF BRAIN TUMOR.


MEDULLOBLASTOMA IS A COMMON CHILDHOOD TUMOR

AN ADOLESCENT BOY COMPLAINS OF SLIGHT PAIN IN THE RIGHT KNEE WHICH GETS
WORSE AFTER EXERCISE. ON

EXAMINATION YOU FIND A LUMP ON THE TIBIA TUBERCLE WITH SLIGHT TENDERNESS.
THE BOY CAN'T REMEMBER IF

HE HAS HAD ANY RECENT TRAUMA. WHICH OF THE FOLLOWING STATEMENTS IS


CORRECT?

A.REASSURE THAT THERE IS NOTHING WRONG

B.BIOPSY SHOULD BE DONE TO EXCLUDE MALIGNANCY

C.KNEE ASPIRATE SHOWS ORGANISM

D. OSTEOCHONDRITIS IS THE MOST LIKELY CAUSE

THIS IS OSGOOD SCHLATTER DISEAS. IT IS SELF LIMITING. ONLY CONSERVATIVE


TREATMENT IS REQUIRED IN MOST OF THE CASES. OSTEOCHONDRITIS PRESENTS WITH
LOCKED KNEE, KNEE EFFUSION ETC.(JOHN MURTAG 739)
A 6 DAY OLD BABY WHO IS SUSPECTED TO HAVE DOWN'S SYNDROME PRESENTS WITH
NON PROJECTILE BILESTAINED VOMITING FOR THE LAST TWO DAYS. ON ABDOMINAL
EXAMINATION YOU FIND AN OLIVE SHAPEDPALPABLE MASS IN THE EPIGASTRIUM. WHICH
OF THE FOLLOWING IS THE MOST LIKELY DIAGNOSIS?

A.VOLVULUS

B.DUODENAL ATRESIA.

C.INTUSSUSCEPTION

D.MALABSORPTION

COMMON COMPLICATION OF DOWN'S SYNDROME

A TWO YEARS OLD CHILD PRESENTS WITH FAILURE TO THRIVE FOR THE LAST 6 MONTHS
AND RIGHT BUTTOCK WASTING. HE PASSES BULKY STOOLS 2-3 TIMES A DAY. WHICH OF
THE FOLLOWING IS THE MOST APPROPRIATE INVESTIGATION?

A.C13 BREATH TEST

B.DUODENAL BIOPSY

C.STOOL CULTURE

D.STOOL MICROSCOPY

E.SWEAT TEST

IT SEEMS LIKE A COELIAC DISEAS. BUT ONLY THING THAT IS PUZZLING IS ONE SIDED
BUTTOCK WASTING WHICH SHOULD BE BOTH SIDED(ROBERTSON CLINICAL EXAMPLE
PAGE 571).

A DIABETIC MOTHER GIVES BIRTH TO A CHILD WEIGHT 4240 GM AND WITH AN APGAR
SCORE OF 7/10. ON EXAMINATION YOU FIND CYANOSIS OF HIS HANDS AND FEET AND THE
CHILD IS JITTERY. AXILLARY TEMPERATURE MEASURE SHOWS A READING OF 37.2 C. HIS
CONDITION IMPROVED AFTER MECONIUM ASPIRATION AND 100% O2. WHICH OF THE
FOLLOWING IS THE MOST LIKELY CAUSE?

A.?HYPOGLYCAEMIA

B.HEART DISEASE

C.LUNG DISEASE.

D.HYPERGLYCAEMIA

E.HIS JAUNDICE IS CAUSED BY BREAST MILK.

HYPOGLYCEMIA IS COMMON IN INFANT OF DIABETIC MOTHERS, SO IS POLYCYTHEMIA.


PROBABLY THE CHILD WAS JITTERY DUE TO HYPOGLYCEMIA. NOT SURE THOUGH.

A SIX YEARS OLD BOY PRESENTS WITH DELAYED DEVELOPMENT OF SPEECH AFTER A
PERIOD OF NORMAL DEVELOPMENT. ON EXAMINATION YOU FIND THAT HE AVOIDS EYE
CONTACT AND REVEALS A HISTORY OF UNUSUAL LIKING FOR DINOSAURS. WHICH OF THE
FOLLOWING IS THE MOST LIKELY DIAGNOSIS?

A.AUTISM

B.ADHD

C.DEAFNESS

OBS

A YOUNG WOMAN WITH ANOVULATORY CYCLES FOR THE LONG TIME IS MOST LIKELY
TO DEVELOP WHICH OF THE FOLLOWING?

A.CERVICAL CANCER

B.ENDOMETRIAL CANCER

C.OVARIAN CANCER

D.VAGINAL CANCER
E.CANCER OF THE VULVA

IN WHICH OF THE FOLLOWING CONDITIONS YOU WILL FIND DECREASED AMNIOTIC


FLUID?

A.FOETUS'S BLADDER NECK OBSTRUCTION OF THE FOETUS

B.T.O FISTULA

C.RHESUS INCOMPATIBITY

D.MULTIPLE PREGNANCY

A 52 YEARS OLD OBESE POST MENOPAUSAL WOMAN PRESENTS WITH NO SIGNS OF


MENOPAUSE AND AMENORRHOEA. WHAT COULD BE THE CAUSE?

A LOW OESTROGEN AND LOW FSH

B HIGH OESTROGEN AND HIGH FSH

C. NORMAL OESTROGEN AND HIGH FSH

D.HIGH OESTROGEN AND LOW FSH €¢

E.LOW OESTROGEN AND HIGH FSH

A 16 YEARS OLD STUDENT ASKS FOR A PRESCRIPTION FOR CONTRACEPTIVE PILL. SHE
SAYS SHE DOESN'T HAVE

SEXUAL PARTNER. WHICH OF THE FOLLOWING IS THE MOST APPROPRIATE


MANAGEMENT?

A. ORAL CONTRACEPTIVE PILL

B.MINIPILL

C.DEPOT

D.ORAL CONTRACEPTIVE PILL + CONDOMS

E.CALL HER PARENTS.


A PREGNANT WOMAN IN 1ST TRIMESTER IS FOUND TO HAVE CIN III. WHAT IS YOUR
NEXT STEP?

A. TERMINATE THE PREGNANCY

B. CONE BIOPSY

C.HYSTERECTOMY

D.COLPOSCOPY

E.?OBSERVE

14. A PREGNANT WOMAN IN 1ST TRIMESTER PRESENTS WITH SKIN DIMPLING OVER
THE RIGHT BREAST. EXAMINATION AND CLINICAL TESTS REVEALS CANCER OF THE RIGHT
BREAST. WHICH OF THE FOLLOWING IS YOUR MANAGEMENT?

A.TERMINATE THE PREGNANCY

B.START RADIOTHERAPY

C.WAIT TILL DELIVERY AND THEN START THE TREATMENT

D.?SURGERY

A 42 YEARS MAN WHO IS A PATIENT OF YOURS IS ARRESTED FOR ATTACKING HIS WIFE
IN A SUPERMARKET. THE POLICE COME TO YOU AND ASK YOU TO WRITE A REPORT
ABOUT THIS PATIENT. WHICH OF THE FOLLOWING IS TRUE FOR GOOD PRACTICE?

A.YOU WILL WRITE A REPORT ABOUT THE PATIENT'S BAD MENTAL STATE

B.YOU WILL TELL THE PATIENT NEVER TO COME TO YOUR SURGERY AGAIN

C.YOU WILL WRITE A GENERAL, UNBIASED AND BALANCED REPORT ABOUT HEALTH OF
THE PATIENT.

D.YOU WILL TELL THE POLICE THAT YOU DON'T WANT TO GIVE ANY REPORTS

E. ARRANGE FOR A MEETING WITH THE PATIENT


WHICH OF THE FOLLOWING WILL CAUSE SEROTONIN SYNDROME IF GIVEN TOGETHER
WITH SSRI LIKE FLUOXETINE EXCEPT?

A.L-TRYPTOPHAN

B.HALOPERIDOL

C.MOCLOBEMIDE

'D. CHLORPROMAZINE

E.CITALOPRAM

DONT KNOW

IN AUSTRALIA , ALMOST EVERY YEAR THERE ARE SEVERAL INCIDENCES OF BUSH FIRE.
USUALLY IT IS CAUSED BY

YOUNG PEOPLE AND TEENAGERS. WHICH OF THE FOLLOWING IS TRUE?

A. ?JUVENILE PYROMANIA IS THE MAIN CAUSE OF FIRES IN AUSTRALIA .

B. IT IS DUE TO ACCIDENTAL FIRE CAUSED BY YOUNGSTER WHO GET SCARED AFTER THE
FIRE STARTS TO

SPREAD UNCONTROLLABLY AND THEY RUN AWAY.

C.IT HAS NO SERIOUS CONSEQUENCES

D.YOUNGSTERS LIKE TO IGNITE AND PLAY WITH FIRE.

E.IT IS DONE TO HIDE CRIMES

A SCHIZOPHRENIC WOMAN THINKS THAT SHE IS PERFECTLY WELL AND DENIES ANY
ILLNESS. SHE ALSO REFUSES TO
TAKE MEDICATION AND THINKS THAT SHE CAN TALK WITH ANGELS. WHAT IS THIS
CALLED IN PSYCHIATRY ?

A.LACK OF INSIGHT

B.OVERVALUED IDEA

C.REJECTION

D.DELUSION

E..FORMAL THOUGHT DISORDER [/B]

YOU ARE CALLED TO REVIEW A PATIENT WHO DEVELOPED RESTLESSNESS AND


DIFFICULTY BREATHING EIGHT HOURS

AFTER THYROIDECTOMY. WHAT WILL YOU DO FIRST?

A. ENDOTRACHEAL INTUBATION

B.REMOVE SUPERFICIAL SUTURES

C.REMOVE SUPERFICIAL AND DEEP SUTURES

D.SALBUTAMOL INHALATION AS PATIENT PROBABLY HAS ASTHMA

THIS IS HYPOPARATHYROIDISM SECONDARY TO SURGERY LEADING TO HYPOCALCEMIA


AND EVENTUALLY LIFE THREATENING LARYNGEAL SPASM(NOT EDEMA)
CAUSE OF ACUTE PANCREATITIS

GALL STONE: AT LEAST 30-40% OF CASES. IN SOME SERIES IT HAS BEEN PLACED AS HIGH
AS 90%

ALCOHOLISM: AT LEAST 25% OF CASES

LADY TALKING WITH THE ANGELS

A SCHIZOPHRENIC WOMAN THINKS THAT SHE IS PERFECTLY WELL AND DENIES ANY
ILLNESS. SHE ALSO REFUSES TO

TAKE MEDICATION AND THINKS THAT SHE CAN TALK WITH ANGELS. WHAT IS THIS
CALLED IN PSYCHIATRY ?

A.LACK OF INSIGHT

B.OVERVALUED IDEA

C.REJECTION
D.DELUSION

E..FORMAL THOUGHT DISORDER

ACTUALLY IT IS LACK OF INSIGHT.

I AM QUOTING FROM OXFORD CORE PSYCHIATRY P18

"AS S TECHNICAL TERM INSIGHT MEANS A CORRECT AWARENESS OF ONE'S OWN


MENTAL CONDITION.

IT IS BEST DESCRIBED IN TERMS OF FOUR CRITERIA:

1. AWARENESS OF ONESELF AS PRESENTING PHENOMENA THAT OTHER PEOPLE


CONSIDER ABNORMAL.

2. RECOGNITION THAT THESE PHENOMENA ARE ABNORMAL.

3. ACCEPTANCE THAT THESE ABNORMAL PHENOMENA ARE CAUSED BY ONE'S OWN


MENTAL ILLNESS

4. AWARENESS THAT TREATMENT IS REQUIRED.

BACK TO TOP

GORANI
GUEST

POSTED: FRI SEP 29, 2006 1:00 PM POST SUBJECT:

--------------------------------------------------------------------------------

YOU ARE CALLED TO REVIEW A PATIENT WHO DEVELOPED RESTLESSNESS AND


DIFFICULTY BREATHING EIGHT HOURS

AFTER THYROIDECTOMY. WHAT WILL YOU DO FIRST?

A. ENDOTRACHEAL INTUBATION.

B.REMOVE SUPERFICIAL SUTURES

C.REMOVE SUPERFICIAL AND DEEP SUTURES

D.SALBUTAMOL INHALATION AS PATIENT PROBABLY HAS ASTHMA

A 70 YEARS OLD WOMAN HAD UPPER GIB. SHE WAS ON TREATMENT WITH NSAID FOR
KNEE PAIN WHICH WAS
STOPPED A FEW MONTHS AGO. YOU DO AN ENDOSCOPY OF UPPER GL TRACT AND FIND
A DUODENAL ULCER WITH

RECENT BLEEDING. WHAT IS THE BEST MANAGEMENT?

A.OMEPRAZOL I.V. + AMPICILLIN + METRONIDAZOLE

B.OMEPRAZOL I.V.

C.HIGHLY SELECTIVE VAGOTOMY

D.VAGOTOMY

E.RANITIDINE I.V. + AMPICILLIN + METRONIDAZOLE

THE TREATMENT FOR NSAID INDUCED ULCER INCLUDES GIVING PPI AND IF PAIN CAN
NOT BE TOLERATED GIVE NSAID+PPI. NO MENTION OF GIVING ANTIBIOTICS. --- TORONTO
NOTES

A TWO YEARS OLD CHILD PRESENTS WITH FAILURE TO THRIVE FOR THE LAST 6 MONTHS
AND RIGHT BUTTOCK WASTING. HE PASSES BULKY STOOLS 2-3 TIMES A DAY. WHICH OF
THE FOLLOWING IS THE MOST APPROPRIATE INVESTIGATION?

A.C13 BREATH TEST

B.DUODENAL BIOPSY*****

C.STOOL CULTURE

D.STOOL MICROSCOPY

E.SWEAT TEST

CELIAC DSE DOES HAVE FAILURE TO THRIVE, MUSCLE WASTING AND FLAT BUTTOCKS. --
--NELSON'S PEDIATRICS
A TEN YEARS OLD GIRL HAS A HISTORY OF REPEATED EPISODES OF HEMIPARESIS FROM
WHICH SHE RECOVERS COMPLETELY. WHAT IS THE MOST LIKELY DIAGNOSIS?

A.MIDDLE CEREBRAL ARTERY OCCLUSION

B.PETIT MAL EPILEPSY

C.MIGRAINE ****

D.SUBARACHNOID HAEMORRHAGE

ATYPICAL MIGRAINE CAN PRESENT WITH HEMISENSORY LOSS OR HEMIPARESIS ---


ADAMS

A YOUNG WOMAN WITH ANOVULATORY CYCLES FOR THE LONG TIME IS MOST LIKELY
TO DEVELOP WHICH OF THE FOLLOWING?

A.CERVICAL CANCER

B.ENDOMETRIAL CANCER

C.OVARIAN CANCER

D.VAGINAL CANCER

E.CANCER OF THE VULVA

A SCHIZOPHRENIC WOMAN THINKS THAT SHE IS PERFECTLY WELL AND DENIES ANY
ILLNESS. SHE ALSO REFUSES TO

TAKE MEDICATION AND THINKS THAT SHE CAN TALK WITH ANGELS. WHAT IS THIS
CALLED IN PSYCHIATRY ?

A.LACK OF INSIGHT ------?????


B.OVERVALUED IDEA

C.REJECTION HAVILA- YR COMMENT?

D.DELUSION

E..FORMAL THOUGHT DISORDER-----????

PATIENT PRESENTS WITH SUDDEN RUPTURE OF EXTENSOR POLLICIS LONGUS


TENDON,DX?

A) TENDONITIES-----??

B) TENOSYNOVITIS TRICK AGAIN

C) RA-----????

WHAT IS THE MOST COMMON CAUSE OF DEATH AFTER BURN INJURY IN AUSTRALIA ?

A.SHOCK

B.RESPIRATORY FAILURE

C.CARDIAC FAILURE

D.?RENAL FAILURE

A 14 YEARS OLD GIRL HAS BREAST ENLARGEMENT, NORMAL GROWTH OF PUBIC AND
AXILLARY HAIR BUT SHE IS STILL

NOT MENSTRUATING. WHICH OF THE FOLLOWING IS CORRECT?

A.SHE IS LIKELY TO START MENSTRUATING TWO YEARS AFTER THE START OF PUBIC
HAIR GROWTH

B.SHE IS LIKELY TO START MENSTRUATING TWO YEARS AFTER THE START OF AXILLARY
HAIR GROWTH
C.SHE IS LIKELY TO START MENSTRUATING WITHIN TWO YEARS FROM THE TIME OF HER
BREAST BUDDING. *****( THIS ONE ALSO CORRECT)

D..SHE IS LIKELY TO START MENSTRUATING WITHIN TWO YEARS FROM GROWTH SPURT.

E.THIS IS A CASE OF DELAYED PUBERTY *******

A 16 YEARS OLD GIRL PRESENTS WITH FEVER, HEADACHE AND RASH ON HER BODY. SHE
ALSO HAS PROFUSE BLEEDING FROM ALL VENIPUNCTURE SITES. WHAT IS THE MOST
PROBABLE DIAGNOSIS?

A.THROMBOCYTOPENIA

B.'DISSEMINATED INTRAVASCULAR COAGULATION******( COOLDUDE-U R WRONG)

C.HENOCH SCHONLEIN PURPURA

D.ALLERGIC REACTION

MENINGOCOCCAL INFECTION LEADING TO DIC

A NEONATE WITH COUGH AND FEVER IS FOUND TO BE GRUNTING. CHEST X RAY


SHOWS ROUND LESIONS WITH LITTLE PLEURAL EFFUSION. WHICH OF THE FOLLOWING IS
THE BEST THE TREATMENT?

A.CRYSTALLINE PENICILLIN******

B.GENTAMYCIN HAVILA- YOUR COMMENT

D.AMOXICILLIN

E.FLUCLOXACILLIN****

A TEN YEARS OLD GIRL HAS A HISTORY OF REPEATED EPISODES OF HEMIPARESIS FROM
WHICH SHE RECOVERS COMPLETELY. WHAT IS THE MOST LIKELY DIAGNOSIS?

A.MIDDLE CEREBRAL ARTERY OCCLUSION


B.PETIT MAL EPILEPSY

C.MIGRAINE ??????

D.SUBARACHNOID HAEMORRHAGE

AN ADOLESCENT BOY COMPLAINS OF SLIGHT PAIN IN THE RIGHT KNEE WHICH ***GETS
WORSE AFTER EXERCISE. ON EXAMINATION YOU FIND A LUMP ON THE TIBIA TUBERCLE
WITH SLIGHT TENDERNESS. THE BOY CAN'T REMEMBER IF

HE HAS HAD ANY RECENT TRAUMA. WHICH OF THE FOLLOWING STATEMENTS IS


CORRECT?

A.REASSURE THAT THERE IS NOTHING WRONG

B.BIOPSY SHOULD BE DONE TO EXCLUDE MALIGNANCY ----????SOMETHING WRONG-


THE BOY HAS SYMPTOM

C.KNEE ASPIRATE SHOWS ORGANISM

D. OSTEOCHONDRITIS IS THE MOST LIKELY CAUSE

A TWO YEARS OLD CHILD PRESENTS WITH ***FAILURE TO THRIVE FOR THE LAST 6
MONTHS AND **RIGHT BUTTOCK WASTING. HE PASSES ***BULKY STOOLS 2-3 TIMES A
DAY. WHICH OF THE FOLLOWING IS THE MOST APPROPRIATE INVESTIGATION?

A.C13 BREATH TEST

B.DUODENAL BIOPSY----???

C.STOOL CULTURE

D.STOOL MICROSCOPY

E.SWEAT TEST ----------------??


A DIABETIC MOTHER GIVES BIRTH TO A CHILD WEIGHT 4240 GM AND WITH AN APGAR
SCORE OF 7/10. ON EXAMINATION YOU FIND CYANOSIS OF HIS HANDS AND FEET AND THE
CHILD IS JITTERY. AXILLARY TEMPERATURE MEASURE SHOWS A READING OF 37.2 C. HIS
CONDITION IMPROVED AFTER MECONIUM ASPIRATION AND 100% O2. WHICH OF THE
FOLLOWING IS THE MOST LIKELY CAUSE?

A.HYPOGLYCAEMIA******** ( WE ARE SURE- COOLDUDE)

B.HEART DISEASE

C.LUNG DISEASE.

D.HYPERGLYCAEMIA

E.HIS JAUNDICE IS CAUSED BY BREAST MILK.

A ***YOUNG WOMAN WITH **ANOVULATORY CYCLES FOR THE LONG TIME IS MOST
LIKELY TO DEVELOP WHICH OF THE FOLLOWING?

A.CERVICAL CANCER

B.ENDOMETRIAL CANCER ( AGE IS OLDER)

C.OVARIAN CANCER *********

D.VAGINAL CANCER

E.CANCER OF THE VULVA

A 52 YEARS OLD ***OBESE POST MENOPAUSAL WOMAN PRESENTS WITH **NO SIGNS
OF MENOPAUSE AND **AMENORRHOEA. WHAT COULD BE THE CAUSE?

A LOW OESTROGEN AND LOW FSH

B HIGH OESTROGEN AND HIGH FSH*****

C. NORMAL OESTROGEN AND HIGH FSH

D.HIGH OESTROGEN AND LOW FSH

E.LOW OESTROGEN AND HIGH FSH


A 16 YEARS OLD STUDENT ASKS FOR A PRESCRIPTION FOR CONTRACEPTIVE PILL. SHE
SAYS SHE DOESN'T HAVE

SEXUAL PARTNER. WHICH OF THE FOLLOWING IS THE MOST APPROPRIATE


MANAGEMENT?

A. ORAL CONTRACEPTIVE PILL**** ( HERE IN OZ-- SEXUALLY ACTIVE SINCE 11YROF AGE-
- COOL-DUDE-- FOLLOW HER REQUEST)

B.MINIPILL ( BY THE WAY WHAT IS THAT?-- HAVEN'T READ)

C.DEPOT

D.ORAL CONTRACEPTIVE PILL + CONDOMS

A PREGNANT WOMAN IN 1ST TRIMESTER IS FOUND TO HAVE CIN III. WHAT IS YOUR
NEXT STEP?

A. TERMINATE THE PREGNANCY

B. CONE BIOPSY

C.HYSTERECTOMY HELP

D.COLPOSCOPY

EOBSERVE

14. A PREGNANT WOMAN IN 1ST TRIMESTER PRESENTS WITH SKIN DIMPLING OVER
THE RIGHT BREAST. EXAMINATION AND CLINICAL TESTS REVEALS CANCER OF THE RIGHT
BREAST. WHICH OF THE FOLLOWING IS YOUR MANAGEMENT?

A.TERMINATE THE PREGNANCY

B.START RADIOTHERAPY

C.WAIT TILL DELIVERY AND THEN START THE TREATMENT*** ( I CAN AGREE WITH
COOL-DUDE)
WHICH OF THE FOLLOWING WILL CAUSE SEROTONIN SYNDROME IF GIVEN TOGETHER
WITH SSRI LIKE FLUOXETINE EXCEPT?

A.L-TRYPTOPHAN

B.HALOPERIDOL

C.MOCLOBEMIDE

'D. CHLORPROMAZINE

E.CITALOPRAM

A SCHIZOPHRENIC WOMAN THINKS THAT SHE IS PERFECTLY WELL AND DENIES ANY
ILLNESS. SHE ALSO REFUSES TO

TAKE MEDICATION AND THINKS THAT SHE CAN TALK WITH ANGELS. WHAT IS THIS
CALLED IN PSYCHIATRY ?

A.LACK OF INSIGHT *****( MOST LIKELY)

B.OVERVALUED IDEA

C.REJECTION

D.DELUSION

E..FORMAL THOUGHT DISORDER ****

A TEN YEARS OLD GIRL HAS A HISTORY OF REPEATED EPISODES OF HEMIPARESIS FROM
WHICH SHE RECOVERS COMPLETELY. WHAT IS THE MOST LIKELY DIAGNOSIS?

A.MIDDLE CEREBRAL ARTERY OCCLUSION

B.PETIT MAL EPILEPSY

C.MIGRAINE I THINK HEMIPLEGIC MIGRAINE NOT SURE

D.SUBARACHNOID HAEMORRHAGE
A 70 YEARS OLD WOMAN HAD UPPER GIB. SHE WAS ON TREATMENT WITH NSAID FOR
KNEE PAIN WHICH WAS

STOPPED A FEW MONTHS AGO. YOU DO AN ENDOSCOPY OF UPPER GL TRACT AND FIND
A DUODENAL ULCER WITH

RECENT BLEEDING. WHAT IS THE BEST MANAGEMENT?

A.OMEPRAZOL I.V. + AMPICILLIN + METRONIDAZOLE

B.OMEPRAZOL I.V.

C.HIGHLY SELECTIVE VAGOTOMY

D.VAGOTOMY

E.RANITIDINE I.V. + AMPICILLIN + METRONIDAZOLE

THE TREATMENT FOR NSAID INDUCED ULCER INCLUDES GIVING PPI AND IF PAIN CAN
NOT BE TOLERATED GIVE NSAID+PPI. NO MENTION OF GIVING ANTIBIOTICS. --- TORONTO
NOTES

ALL PATIENTS WITH PROVEN ACUTE OR CHRONIC DUODENAL ULCER AND THOSE WITH
GASTRIC ULCER WHO ARE H PYLORI POSITIVE SHOULD BE OFFERED ERADICAITON OF H.
PYLORI. (DAVIDSON 20TH P887)
OMEPRAZOLE FOR HEALING OF ULCER AND AMOXY+METRO FOR ERADICATION OF H.
PYLORI

A TWO YEARS OLD CHILD PRESENTS WITH FAILURE TO THRIVE FOR THE LAST 6 MONTHS
AND RIGHT BUTTOCK WASTING. HE PASSES BULKY STOOLS 2-3 TIMES A DAY. WHICH OF
THE FOLLOWING IS THE MOST APPROPRIATE INVESTIGATION?

A.C13 BREATH TEST

B.DUODENAL BIOPSY*****

C.STOOL CULTURE

D.STOOL MICROSCOPY

E.SWEAT TEST

CELIAC DSE DOES HAVE FAILURE TO THRIVE, MUSCLE WASTING AND FLAT BUTTOCKS. --
--NELSON'S PEDIATRICS

A YOUNG WOMAN WITH ANOVULATORY CYCLES FOR THE LONG TIME IS MOST LIKELY
TO DEVELOP WHICH OF THE FOLLOWING?

A.CERVICAL CANCER

B.ENDOMETRIAL CANCER

C.OVARIAN CANCER

D.VAGINAL CANCER

E.CANCER OF THE VULVA


DEVELOPMENT OF ENDOMETRIAL CARCINOMA IS RELATED TO UNOPPOSED ACTION OF
ESTROGEN. IT IS MORE LIKELY TO OCCUR IN WOMEN WHO ARE RELATIVELY INFERTILE.(LJ
281)

A YOUNG WOMAN WITH ANOVULATORY CYCLES FOR THE LONG TIME IS MOST LIKELY
TO DEVELOP WHICH OF THE FOLLOWING?

A.CERVICAL CANCER

B.ENDOMETRIAL CANCER

C.OVARIAN CANCER

D.VAGINAL CANCER

E.CANCER OF THE VULVA

I THINK I IGNORED THE AGE FACTOR HERE. SO, NOW I THINK OVARIAN CARCINOMA
SHOULD BE A BETTER OPTION HERE. SORRY!
I AM FOLLOWING YOUR QUESTIONS BULIMIA.

PLEASE NOTE THAT I HAVE ATTACHED REFERENCES TO A LOT OF MY ANSWERS.

REGARDING YOUR POST OP FLUID THERAPY QUESTION(BULIMIA) - I WENT THROUGH


THE AMC COMMENTARY YOU REFERRED TO. THAT WAS A DIFFERENT SCENERIO. A MAN
WHO WAS DEHYDRATED AND HAD PYLORIC STENOSIS. THIS IS NOT THE SAME AS POST OP
FLUID THERAPY. I REQUEST YOU TO GO THROUGH THE PERIOPERATIVE AND
POSTOPERATIVE CARE FROM SCOTT BEFORE YOU MAKE UP YOUR MIND.

A LADY CAME TO YOU AT 12 WEEKS PREGNANCY FOR ANTENATAL CHECK.HER 1ST


CHILD WAS BORN WITH SPINA BIFIDA.WHICH OF THE FOLLOWING INVESTIGATION
WOULD NOT BE USEFUL IN ASSESING THE SECOND BABY...

A-MATERNAL TRIPLE OR TETRA TEST AT 16-18 WEEK

B-AMNIOCENTESIS FOR ACETYLCHOLINE ESTRASE AT 11-13 WEEKS

C-USG FOR NUCHAL FOLD MEASUREMENT AT 11-13 WEEKS ??

D-USG FOR SPINAL ABNORMALITIES AT 16-18 WEEKS

E-DO CHORION VILLOUS SAMPLING AT 8-10 WEEKS ??

BACK TO TOP
GORANI

GUEST

POSTED: FRI SEP 29, 2006 1:02 PM POST SUBJECT: SPIGELIAN HERNIA

--------------------------------------------------------------------------------

SPIGELIAN HERNIA

A SPIGELIAN HERNIA IS AN ACQUIRED VENTRAL HERNIA THROUGH THE LINEA


SEMILUNARIS, THE LINE WHERE THE SHEATHS OF THE LATERAL ABDOMINAL MUSCLES
FUSE TO FORM THE LATERAL RECTUS SHEATH. SPIGELIAN HERNIAS ARE NEARLY ALWAYS
FOUND ABOVE THE LEVEL OF THE INFERIOR EPIGASTRIC VESSELS, AND MOST OFTEN
OCCUR WHERE THE SEMICIRCULAR LINE - FOLD OF DOUGLAS - CROSS THE LINEA
SEMILUNARIS. COMMONLY, THE PATIENT IS OVER THE AGE OF 50 YEARS. MEN AND
WOMEN ARE EQUALLY AFFECTED.

THE PATIENT PRESENTS WITH PAIN THAT IS LOCALISED TO THE HERNIAL SITE AND IS
AGGRAVATED BY ANY MOVEMENT THAT RAISES INTRA-ABDOMINAL PRESSURE. LATER,
THE PAIN BECOMES MORE DULL, CONSTANT, AND DIFFUSE.
A SOFT, REDUCIBLE MASS MAY BE PRESENT IN THE LOWER ABDOMINAL AREA WHICH
DISAPPEARS ON PRESSURE. WHEN THE MASS IS REDUCED, THE HERNIAL ORIFICE CAN
USUALLY BE PALPATED. DIAGNOSIS IS MORE DIFFICULT WHEN THE HERNIA DISSECTS
WITHIN THE LAYERS OF THE ABDOMINAL WALL - INTERNAL AND EXTERNAL OBLIQUES -
OR MAY BE LOCATED AT A DISTANCE FROM THE LINEA SEMILUNARIS. ULTRASOUND AND
CT SCAN MAY HELP TO CONFIRM THE DIAGNOSIS.

SPIGELIAN HERNIAS HAVE A HIGH INCIDENCE OF INCARCERATION AND SHOULD BE


REPAIRED. TREATMENT IS BY PRIMARY APONEUROTIC CLOSURE.

BACK TO TOP

GORANI

GUEST

POSTED: FRI SEP 29, 2006 1:03 PM POST SUBJECT:


--------------------------------------------------------------------------------

PATIENT PRESENTS WITH SUDDEN RUPTURE OF EXTENSOR POLLICIS LONGUS


TENDON,DX?

A) TENDONITIES?

B) TENOSYNOVITIS

C) RA???

A PATIENT WITH CANCER BREAST DEVELOP OSTEOPRPOSIS.WHAT IS THE TREATMENT

A) BISPHOSPHONATE

B) CYCLICAL ESTROGEN AND PROGESTERONE

C) CONTINUOUS ESTROGEN AND PROGESTERONE

INCREASE DIATERY CALCIUM 2000G / DAY

YOU ARE CALLED TO REVIEW A PATIENT WHO DEVELOPED RESTLESSNESS AND


DIFFICULTY BREATHING EIGHT HOURS

AFTER THYROIDECTOMY. WHAT WILL YOU DO FIRST?

A. ENDOTRACHEAL INTUBATION.

B.REMOVE SUPERFICIAL SUTURES

C.REMOVE SUPERFICIAL AND DEEP SUTURES

D.SALBUTAMOL INHALATION AS PATIENT PROBABLY HAS ASTHMA

A 43 YEARS OLD MAN WAS BROUGHT TO THE EMERGENCY DEPARTMENT WITH SEVERE
UPPER ABDOMINAL PAIN

AND SIGNS OF SHOCK. PHYSICAL EXAMINATION REVEALS ABDOMINAL WALL RIGIDITY


AND GUARDING. WHICH OF
THE FOLLOWING IS THE MOST LIKELY DIAGNOSIS?

A.PERFORATED GASTRIC ULCER

B.PENETRATING DUODENAL ULCER

C. MESENTERIC ISCHEMIA

D. ACUTE PANCREATITIS

A YOUNG MALE FELL FROM A HEIGHT BUT SUSTAINED MINOR INJURIES. AFTER TWO
HOURS HE LOST

CONSCIOUSNESS AND WAS BROUGHT TO YOU IN NORTHERN QUEENSLAND HOSPITAL


WITH LIMITED RADIOGRAPHIC

FACILITIES BY HIS FRIEND. ON EXAMINATION YOU FIND THAT HIS PUPILS ARE FIXED AND
DILATED. WHAT IS OUR

NEXT STEP?

A.ARRANGE FOR TRANSPORTATION TO ANOTHER HOSPITAL BY HELICOPTER.

B.GIVE IV MANNITOL

C GIVE IV CORTICOSTEROID D. D BURR HOLES

AN ELDERLY MAN CAME TO SEE YOU WITH COMPLAINTS OF SUDDEN PAIN IN THE
LOWER BACK WHILE LIFTING A HEAVY BOX. ON EXAMINATION YOU FIND THAT THE
LUMBAR SPINE HAS LIMITED MOVEMENT WITH PAIN IN HIS LOWER BACK ON
MOVEMENT. THERE ARE NO NEUROLOGICAL ABNORMALITIES. WHAT IS THE NEXT
INVESTIGATION THAT YOU'LL DO?

A.X RAY OF THE LUMBAR SPINE

B.CTSCAN

C.MRI OF THE SPINE

D.LUMBAR PUNCTURE
A 70 YEARS OLD WOMAN HAD UPPER GIB. SHE WAS ON TREATMENT WITH NSAID FOR
KNEE PAIN WHICH WAS

STOPPED A FEW MONTHS AGO. YOU DO AN ENDOSCOPY OF UPPER GL TRACT AND FIND
A DUODENAL ULCER WITH

RECENT BLEEDING. WHAT IS THE BEST MANAGEMENT?

A.OMEPRAZOL I.V. + AMPICILLIN + METRONIDAZOLE

B.OMEPRAZOL I.V.

C.HIGHLY SELECTIVE VAGOTOMY

D.VAGOTOMY

E.RANITIDINE I.V. + AMPICILLIN + METRONIDAZOLE

WHICH OF THE FOLLOWING IS MOST COMMON CAUSE OF ACUTE PANCREATITIS?

A.GALLSTONES,

B.CHOLECYSTITIS

C.ALCOHOL ABUSE

D.SPICY FOOD

E.GASTROENTERITIS

WHAT IS THE MOST COMMON CAUSE OF DEATH AFTER BURN INJURY IN AUSTRALIA ?

A.SHOCK???

B.RESPIRATORY FAILURE???

C.CARDIAC FAILURE

D.RENAL FAILURE
PAED

A 14 YEARS OLD GIRL HAS BREAST ENLARGEMENT, NORMAL GROWTH OF PUBIC AND
AXILLARY HAIR BUT SHE IS STILL

NOT MENSTRUATING. WHICH OF THE FOLLOWING IS CORRECT?

A.SHE IS LIKELY TO START MENSTRUATING TWO YEARS AFTER THE START OF PUBIC
HAIR GROWTH

B.SHE IS LIKELY TO START MENSTRUATING TWO YEARS AFTER THE START OF AXILLARY
HAIR GROWTH

C.SHE IS LIKELY TO START MENSTRUATING WITHIN TWO YEARS FROM THE TIME OF HER
BREAST BUDDING.

D..SHE IS LIKELY TO START MENSTRUATING WITHIN TWO YEARS FROM GROWTH SPURT.

E.THIS IS A CASE OF DELAYED PUBERTY

A 16 YEARS OLD GIRL PRESENTS WITH FEVER, HEADACHE AND RASH ON HER BODY. SHE
ALSO HAS PROFUSE BLEEDING FROM ALL VENIPUNCTURE SITES. WHAT IS THE MOST
PROBABLE DIAGNOSIS?

A.THROMBOCYTOPENIA

B.'DISSEMINATED INTRAVASCULAR COAGULATION

C.HENOCH SCHONLEIN PURPURA

D.ALLERGIC REACTION

A 12 MONTHS OLD BABY HAS FEVER. RECENTLY, TWO OTHER FAMILY MEMBERS HAD
AN UPPER RESPIRATORY TRACT INFECTION. ON EXAMINATION OF THE BABY YOU FIND
THAT HIS BODY TEMPERATURE IS 37.5 CTAND CHEST AUSCULTATION REVEALS BILATERAL
WHEEZING. WHAT IS THE MOST LIKELY CAUSE?

A.ACUTE BRONCHIOLITIS

B.VIRAL CROUP
C.BRONCHIAL ASTHMA

D.PNEUMONIA

E.FOREIGN BODY

A NEONATE WITH COUGH AND FEVER IS FOUND TO BE GRUNTING. CHEST X RAY


SHOWS ROUND LESIONS WITH LITTLE PLEURAL EFFUSION. WHICH OF THE FOLLOWING IS
THE BEST THE TREATMENT?

A.CRYSTALLINE PENICILLIN

B.GENTAMYCIN

D.AMOXICILLIN

E.FLUCLOXACILLIN

A TEN YEARS OLD GIRL HAS A HISTORY OF REPEATED EPISODES OF HEMIPARESIS FROM
WHICH SHE RECOVERS COMPLETELY. WHAT IS THE MOST LIKELY DIAGNOSIS?

A.MIDDLE CEREBRAL ARTERY OCCLUSION

B.PETIT MAL EPILEPSY

C.MIGRAINE

D.SUBARACHNOID HAEMORRHAGE

THE PARENTS WITH THEIR 10 YEARS OLD CHILD COME TO SEE YOU WITH COMPLAINTS
THAT THE CHILD HAS ENURESIS SINCE BIRTH. THE CHILD HAS NO PROBLEMS DURING THE
DAY TIME. PHYSICAL EXAMINATION DOES NOT REVEAL ANY OTHER ABNORMALITY.
WHICH OF THE FOLLOWING STATEMENTS IS TRUE IN MORE THAN 20 % OF SUCH CASES?

A.HE HAS CHRONIC PYELONEPHRITIS

B.HE HAS CONGENITAL ABNORMALITIES OF THE URINARY TRACT

C.THERE IS AN ASSOCIATED FAMILY HISTORY


D.HE HAS SMALL URINARY BLADDER.

E.IT IS ASSOCIATED WITH SPINA BIFIDA OCCULTA

A 5 YEARS OLD BOY PRESENTS WITH VOMITING EVERY MORNING AND MORNING
HEADACHE FOR THE LAST TWO WEEKS. WHAT IS THE MOST LIKELY CAUSE?

A.MENINGITIS

B.MIGRAINE

C.CRANIOPHARYNGIOMA

D.MEDUJLOBLASTOMA

E.WILM'S TUMOUR

AN ADOLESCENT BOY COMPLAINS OF SLIGHT PAIN IN THE RIGHT KNEE WHICH GETS
WORSE AFTER EXERCISE. ON

EXAMINATION YOU FIND A LUMP ON THE TIBIA TUBERCLE WITH SLIGHT TENDERNESS.
THE BOY CAN'T REMEMBER IF

HE HAS HAD ANY RECENT TRAUMA. WHICH OF THE FOLLOWING STATEMENTS IS


CORRECT?

A.REASSURE THAT THERE IS NOTHING WRONG

B.BIOPSY SHOULD BE DONE TO EXCLUDE MALIGNANCY

C.KNEE ASPIRATE SHOWS ORGANISM

D. OSTEOCHONDRITIS IS THE MOST LIKELY CAUSE

A 6 DAY OLD BABY WHO IS SUSPECTED TO HAVE DOWN'S SYNDROME PRESENTS WITH
NON PROJECTILE BILESTAINED VOMITING FOR THE LAST TWO DAYS. ON ABDOMINAL
EXAMINATION YOU FIND AN OLIVE SHAPEDPALPABLE MASS IN THE EPIGASTRIUM. WHICH
OF THE FOLLOWING IS THE MOST LIKELY DIAGNOSIS?

A.VOLVULUS
B.DUODENAL ATRESIA.

C.INTUSSUSCEPTION

D.MALABSORPTION

A TWO YEARS OLD CHILD PRESENTS WITH FAILURE TO THRIVE FOR THE LAST 6 MONTHS
AND RIGHT BUTTOCK WASTING. HE PASSES BULKY STOOLS 2-3 TIMES A DAY. WHICH OF
THE FOLLOWING IS THE MOST APPROPRIATE INVESTIGATION?

A.C13 BREATH TEST

B.DUODENAL BIOPSY

C.STOOL CULTURE

D.STOOL MICROSCOPY

E.SWEAT TEST

A DIABETIC MOTHER GIVES BIRTH TO A CHILD WEIGHT 4240 GM AND WITH AN APGAR
SCORE OF 7/10. ON EXAMINATION YOU FIND CYANOSIS OF HIS HANDS AND FEET AND THE
CHILD IS JITTERY. AXILLARY TEMPERATURE MEASURE SHOWS A READING OF 37.2 C. HIS
CONDITION IMPROVED AFTER MECONIUM ASPIRATION AND 100% O2. WHICH OF THE
FOLLOWING IS THE MOST LIKELY CAUSE?

A.HYPOGLYCAEMIA

B.HEART DISEASE

C.LUNG DISEASE.

D.HYPERGLYCAEMIA

E.HIS JAUNDICE IS CAUSED BY BREAST MILK.

A SIX YEARS OLD BOY PRESENTS WITH DELAYED DEVELOPMENT OF SPEECH AFTER A
PERIOD OF NORMAL DEVELOPMENT. ON EXAMINATION YOU FIND THAT HE AVOIDS EYE
CONTACT AND REVEALS A HISTORY OF UNUSUAL LIKING FOR DINOSAURS. WHICH OF THE
FOLLOWING IS THE MOST LIKELY DIAGNOSIS?
A.AUTISM

B.ADHD

C.DEAFNESS

OBS

A YOUNG WOMAN WITH ANOVULATORY CYCLES FOR THE LONG TIME IS MOST LIKELY
TO DEVELOP WHICH OF THE FOLLOWING?

A.CERVICAL CANCER

B.ENDOMETRIAL CANCER

C.OVARIAN CANCER

D.VAGINAL CANCER

E.CANCER OF THE VULVA

IN WHICH OF THE FOLLOWING CONDITIONS YOU WILL FIND DECREASED AMNIOTIC


FLUID?

A.FOETUS'S BLADDER NECK OBSTRUCTION OF THE FOETUS

B.T.O FISTULA

C.RHESUS INCOMPATIBITY

D.MULTIPLE PREGNANCY

A 52 YEARS OLD OBESE POST MENOPAUSAL WOMAN PRESENTS WITH NO SIGNS OF


MENOPAUSE AND AMENORRHOEA. WHAT COULD BE THE CAUSE?

A LOW OESTROGEN AND LOW FSH

B HIGH OESTROGEN AND HIGH FSH

C. NORMAL OESTROGEN AND HIGH FSH *


D.HIGH OESTROGEN AND LOW FSH €¢

E.LOW OESTROGEN AND HIGH FSH

A 16 YEARS OLD STUDENT ASKS FOR A PRESCRIPTION FOR CONTRACEPTIVE PILL. SHE
SAYS SHE DOESN'T HAVE

SEXUAL PARTNER. WHICH OF THE FOLLOWING IS THE MOST APPROPRIATE


MANAGEMENT?

A. ORAL CONTRACEPTIVE PILL

B.MINIPILL

C.DEPOT

D.ORAL CONTRACEPTIVE PILL + CONDOMS

E.CALL HER PARENTS.

A PREGNANT WOMAN IN 1ST TRIMESTER IS FOUND TO HAVE CIN III. WHAT IS YOUR
NEXT STEP?

A. TERMINATE THE PREGNANCY

B. CONE BIOPSY

C.HYSTERECTOMY

D.COLPOSCOPY

E.OBSERVE

14. A PREGNANT WOMAN IN 1ST TRIMESTER PRESENTS WITH SKIN DIMPLING OVER
THE RIGHT BREAST. EXAMINATION AND CLINICAL TESTS REVEALS CANCER OF THE RIGHT
BREAST. WHICH OF THE FOLLOWING IS YOUR MANAGEMENT?

A.TERMINATE THE PREGNANCY

B.START RADIOTHERAPY
C.WAIT TILL DELIVERY AND THEN START THE TREATMENT

D.SURGERY

A 19 YEARS OLD FEMALE COMES TO YOUR CLINIC FOR CONSULTATION. SHE HAS NEVER
HAD PERIODS. ON EXAMINATION HER BREAST DEVELOPMENT IS NORMAL. PUBIC AND
AXILLARY HAIR GROWTH IS ALSO NORMAL. THE

DEVELOPMENT OF HER GENITALS IS ALSO IN NORMAL RANGE. WHAT IS THE MOST


LIKELY CAUSE OF HER AMENORRHEA?

A. TURNER'S SYNDROME

B. PROLACTINOMA

C. ABSENT UTERUS D. PITUITARY LESION

E. LESION IN HYPOTHALAMUS

A 42 YEARS MAN WHO IS A PATIENT OF YOURS IS ARRESTED FOR ATTACKING HIS WIFE
IN A SUPERMARKET. THE POLICE COME TO YOU AND ASK YOU TO WRITE A REPORT
ABOUT THIS PATIENT. WHICH OF THE FOLLOWING IS TRUE FOR GOOD PRACTICE?

A.YOU WILL WRITE A REPORT ABOUT THE PATIENT'S BAD MENTAL STATE

B.YOU WILL TELL THE PATIENT NEVER TO COME TO YOUR SURGERY AGAIN

C.YOU WILL WRITE A GENERAL, UNBIASED AND BALANCED REPORT ABOUT HEALTH OF
THE PATIENT.

D.YOU WILL TELL THE POLICE THAT YOU DON'T WANT TO GIVE ANY REPORTS

E. ARRANGE FOR A MEETING WITH THE PATIENT

WHICH OF THE FOLLOWING WILL CAUSE SEROTONIN SYNDROME IF GIVEN TOGETHER


WITH SSRI LIKE FLUOXETINE EXCEPT?

A.L-TRYPTOPHAN

B.HALOPERIDOL
C.MOCLOBEMIDE

'D. CHLORPROMAZINE

E.CITALOPRAM

IN AUSTRALIA , ALMOST EVERY YEAR THERE ARE SEVERAL INCIDENCES OF BUSH FIRE.
USUALLY IT IS CAUSED BY

YOUNG PEOPLE AND TEENAGERS. WHICH OF THE FOLLOWING IS TRUE?

A. JUVENILE PYROMANIA IS THE MAIN CAUSE OF FIRES IN AUSTRALIA .

-B. IT IS DUE TO ACCIDENTAL FIRE CAUSED BY YOUNGSTER WHO GET SCARED AFTER
THE FIRE STARTS TO

SPREAD UNCONTROLLABLY AND THEY RUN AWAY.

C.IT HAS NO SERIOUS CONSEQUENCES

D.YOUNGSTERS LIKE TO IGNITE AND PLAY WITH FIRE.

E.IT IS DONE TO HIDE CRIMES

A SCHIZOPHRENIC WOMAN THINKS THAT SHE IS PERFECTLY WELL AND DENIES ANY
ILLNESS. SHE ALSO REFUSES TO

TAKE MEDICATION AND THINKS THAT SHE CAN TALK WITH ANGELS. WHAT IS THIS
CALLED IN PSYCHIATRY ?

A.LACK OF INSIGHT

B.OVERVALUED IDEA

C.REJECTION

D.DELUSION

E..FORMAL THOUGHT DISORDER


BACK TO TOP

ELLIEL

GUEST

POSTED: SAT SEP 30, 2006 12:51 PM POST SUBJECT:

--------------------------------------------------------------------------------

CLINICAL EXAM, MAY 20TH , 2006 SYDNEY

1.FEBRILE CONVULSION €“ TALK TO PT.


2.MID-CYCLE BLEEDING ON OCP €“ TALK TO PT.

3.CARPAL TUNNEL SYNDROME €“ EXAMINATION

4.PHOTO LUMP ON THE NECK €“ FIND PRIMARY SITE,/EXAMINATION/DDX

5.BENZODIAZEPINES DEPENDENCY €“ TALK TO PT / SE

6.DELIRIUM €“ EXAMINE PT / MS/ MX.

7.GASTROINTESTINAL SYSTEM EXAMINATION €“ ALCOHOLIC PT

.
8.HEADACHE/MENINGITIS €“ TAKE HISTORY/EXAMINE THE PT/ DDX

9.ENURESIS €“ TALK TO PARENTS

10.PREGANCY & PNEUMONIA PT ON DOXYCYCLINE €“ HISTORY/EXAMINATION/MX

CHANGE DOXYCYCLINE TO ERYTHROMYCIN (SAFE OR RELATIVELY SAFE DURING THE


PREGNANCY: AMOXYCILLIN/AMPICILLIN/CEPHALOSPORINE/ERYTHOMYCIN/RIFAMPICIN/

TRIMETHOPRIM/METHYLDOPA/ - BEISCHER , P 105)

11.PAIN RELIEF IN LABOUR €“ TALK TO PT.WHO IS 20/40 PREGNANT; P0G1.SHE IS


VERY CONCERNED ABOUT PAIN IN LABOUR. ( BEISCHER, P. 411)

ASK WHY SHE IS WORRIED ABOUT PAIN? ANY PARTICULAR REASON? WHAT IS HER
UNDERSTANDING ABOUT PAIN RELIEF DURING THE LABOUR ? CS / NORMAL LABOUR.

DISCUSS PHARMACOLOGICAL, SEDATIVE ( DIAZEPAM MAX 20 MG ) AND HYPNOTICS,


TRANQUILLISERS ( MAJOR: PHENOTHIAZINES - CHLORPROMAZINE, PROMETHAZINE, OR
MINOR: DIAZEPAM ), ( ANALGESICS (NON-NARCOTIC: ASPIRIN, PARACETAMOL, CODEINE;
NARCOTIC: PETHIDINE 50-100 MG IM; MORPHINE 10-15MG IM; SE: RESPIRATORY
DEPRESSION, NAUSEA & VOMITING, TACHYCARDIA, POSTURAL HYPOTENSION AND
DELAYED GASTRIC EMPTYING). PETHIDINE IS PREFERRED AND USUALLY GIVEN IM 100 MG;
IT IS GIVEN BY I>V ROUTE IN SOME CENTRES EITHER INTERMITTENTLY IN SMALL DOSES
25MG OVER 1-2 MINUTES EVERY 1-3 HOURS OR SELF ADMINISTERED BY THE WOMEN (
PCA €“ PATIENT CONTROLLED ANALGESIA)THE USUAL DURATION OF NARCOTICS DRUGS
IS 2-4 HOURS. PREFERABLY SUCH DRUGS SHOULD NOT BE ADMINISTERED WITHIN 1 HOUR
OF DELIVERY €“ AND IF IT IS THE BABY SHOULD BE GIVEN NALOXONE ( 0.02MG) IM
IMMEDIATELY AFTER BIRTH. DURING THIS PERIOD INHALATION OR REGIONAL BLOCK ARE
PREFERABLE; NARCOTICS ARE CI IN WOMEN RECEIVING MAO INHIBITORS.

THE USE OF SEDATIVES, HYPNOTICS AND MINOR TRANQUILLISERS VARIES


CONSIDERABLY IN OBSTETRIC PRACTICE IN DIFFERENT COMMUNITIES. MANY
PRACTITIONERS RELY ON SINGLE INJECTION OF PETHIDINE 100 MG IM WITH EITHER
PROCHLORPERAZINE (STEMETIL) 12,5 MG OR METOCLOPRAMIDE HCL (MAXOLON) 10 MG
± EPIDURAL ANALGESIA FOR PAIN RELIEF IN LABOUR.

INHALATIONAL ANALGESICS: NITROUS OXIDE (NO) IN CONCENTRATION OF 50% IS


EQUIVALENT TO 15 MG OF MORPHINE. NITROUS OXIDE/OXYGEN GIVEN BY MASK €ŒON
DEMAND€; IT IS USEFUL FOR ANALGESIA DURING THE LATE FIRST STAGE AND SECOND
STAGE (I.E. DURING DELIVERY) IN A 50-70% CONCENTRATION WITH THE OXYGEN. IN
THEIR LATE FIST STAGE OF LABOUR THE WOMEN SLEEPS BETWEEN CONTRACTIONS BUT
ROUSES AND BREATHES ON THE MASK WHEN THE CONTRACTION BEGINS. ADVANTAGES:
SAFETY, REASONABLE ANALGESIA, ON-TOXIC AND NON-IRRITATING TO RESPIRATORY
PASSAGES, QUICKLY ABSORBED AND ELIMINATED, UTERINE CONTRACTIONS UNAFFECTED
(NO PROLONGATION OF LABOUR OR POSTPARTUM ATONICITY), VOMITING RARE;
DISADVANTAGES: REQUIREMENT OF MACHINE TO DELIVER THE GAS AND
DISORIENTATION OF SOME WOMEN ;OCCASIONALLY, WOMEN WILL NOT USE THE MASK
BECAUSE OF FELLING OF SUFFOCATION OR BECAUSE OF DISORIENTATION.

MENTION : LOCAL ANALGESIA/EPIDURAL/PARACERVICAL BLOCK. DISCUSS IF YOU HAVE


TIME.

EPIDURAL NERVE BLOCK: THE LOCAL ANALGESIC IS INJECTED EXTRATHECALLY INTO


EPIDURAL SPACE; THE EXTENT OF ANALGESIA IS DETERMINED LARGELY BY THE VOLUME
AND CONCENTRATION OF DRUG INJECTED; TO CONTROL PAIN OF LATE LABOUR, A BLOCK
OF SEGMENTS T10 €“ L1 IS ADEQUATE; FOR CAESAREAN SECTION A HIGHER BLOCK €“
T8 OR EVEN T6 IS NEEDED; LOW DOSE EPIDURAL ANALGESIA (0.125 BUPIVACAINE)
INCREASES MOBILITY IN LABOUR AND ABOLISH THE URGE TO PUSH (PUSHING REFLEX)
BUT NOT ABILITY TO PUSH SO THAT NORMAL SPONTANEOUS DELIVERY IS STILL POSSIBLE.
ALTHOUGH EPIDURAL ANALGESIA IS ASSOCIATED WITH A HIGHER INCIDENCE OF FORCEPS
DELIVERY IT MUST BE RECOGNISED THAT WOMEN WHO REQUIRE EPIDURAL ANALGESIA
FOR PAIN RELIEF ARE MORE LIKELY TO REQUIRE FORCES DELIVERY, USUALLY BECAUSE OF
DYSTOCIA RESULTING FROM OCCIPITOPOSTERIOR POSITION.

INDICATION FOR EPIDURAL: PAIN RELIEF; SYMPTOMATIC HEART DISEASE €“ THE PAIN
AND DISTRESS OF LABOUR IS RELIEVED, HOWEVER EXTREME CARE IS NECESSARY IF
WOMEN HAS A FIXED CARDIAC OUTPUT (E.G. MITRAL OR AORTIC STENOSIS, PULMONARY
HYPERTENSION); HYPERTENSIVE DISORDERS (PREECLAMPSIA/ECLAMPSIA, CHRONIC
HYPERTENSION), CEREBROVASCULAR DISEASE (INTRACRANIAL ANEURISM AND ANGIOMA
ARE USUAL INDICATION); INCOORDINATE UTERINE ACTION (ELIMINATION OF PAIN AND
FEAR OFTEN HELP NORMALIZE THE ACTIVITY); BREECH AND TWIN DELIVERY( OPINION IS
DIVIDE ON THE VALUE OF EPIDURAL ANALGESIA IN THESE CONDITIONS: THE RELAXATION
OF PELVIC FLOOR IS ADVANTAGEOUS BUT THE LACK OF STIMULUS FOR THE WOMAN TO
PUSH CAN LEAD TO A HIGHER INTERFERENCE RATE; CONTRAINDICATIONS FOR EPIDURAL
ANALGESIA: OPPOSITION BY THE WOMAN; RECENT ANTEPARTUM HAEMORRHAGE(
BECAUSE COMPENSATORY VASCULAR REFLEXES ARE PARTLY ABOLISHED, SUDDEN
HAEMORRHAGE MAY PRODUCE MARKED HYPOTENSION; SUSPECTED CEPHALOPELVIC
DISPROPORTION; SEPSIS ( IN PROPOSED AREA OF OPERATION);SENSITIVITY TO LOCAL
ANALGESIC AGENTS; DIC ( SEVER PREECLAMPSIA - PREDISPOSE TO HAEMORRHAGE IN THE
EPIDURAL SPACE ).

COMPLICATION OF EPIDURAL ANALGESIA: TOXICITY (FROM OVERDOSAGE AND/OR


INTRAVASCULAR INJECTION; HYPOTENSION; UTERINE HYPOTONIA- OCCURS IN 5-10% OF
WOMEN IN EARLY LABOUR BUT USUALLY ACTIVITY RETURN IN 15-30 MINUTES; COLLAPSE
(WITH HYPOTENSION AND APNOEA MAY BE DUE TO TOXIC EFFECT OF THE DRUG);
HEADACHE (MAY RESULT FROM ACCIDENTAL PUNCTURE OF THE DURA AND LEAKAGE OF
CSF; THIS CAN BE TREATED BY INSERTION OF EPIDURAL AT HIGHER LEVEL; AFTER THE
DELIVERY HARTMANN SOLUTION IS INFUSED INTO EPIDURAL CATHETER; WOMAN
SHOULD BE NURSED FLAT, ADEQUATELY HYDRATED ± ASPIRIN, CODEINE, PARACETAMOL
; LOSS OF SENSATION MAY OCCUR IN BLADDER ( CAUSING OVER DISTENSION);LOSS OF
BEARING -DOWN REFLEX, RESULTING IN HIGHER INCIDENCE OF ASSISTED DELIVERY;
BACKACHE

MAJORITY OF THE ABOVE COMPLICATIONS ARE RARE, HOWEVER THEY NEED


EXPERIENCED PERSONNEL IN THIS TYPE OF ANALGESIA; ADVANTAGES OF EPIDURAL
ANALGESIA: WOMAN ALERT, COOPERATIVE REDUCED RISK OF INHALATION OF VOMITUS;
DISADVANTAGE OF EPIDURAL ANALGESIA: NEED FOR SKILLED PERSONNEL, THE
OCCASIONAL SERIOUS COMPLICATION, TENDENCY TO SLOW THE LABOUR (PARTICULARLY
SECOND STAGE) INCREASED INCIDENCE OF OPERATIVE DELIVERY; INTRAOPERATIVE
NAUSEA, VOMITING OR RESTLESSNESS IN SOME WOMEN.
IN CAESAREAN SECTION, REGIONAL ANALGESIA HAS A NUMBER OF ADVANTAGES OVER
GENERAL ANAESTHESIA €“ PARTICIPATION OF THE PARENTS IN THE BIRTH, BETTER
POSTOPERATIVE PAIN RELIEF, EARLY MOBILITY, LESS FEVER, BLOOD LOSS IS 50 % LESS
THAN WITH GENERAL ANAESTHESIA. DISADVANTAGES ARE THAT THE METHOD MAY FAIL,
THE WOMAN FEELS PAIN AND MAY REQUIRE A GENERAL ANAESTHETIC; IT IS TECHNICALLY
MORE DIFFICULT TO EXTRACT THE INFANT€™S HEAD FROM THE UTERUS, ESPECIALLY IF
A PFANNESTINIEL INCISION IS USED, BECAUSE ABDOMINAL MUSCLES ARE NOT
COMPLETELY RELAXED;

PARACERVICAL BLOCK: LA (6-10 MG OF 0.25 BUPIVACAINE) INJECTED BENEATH THE


MUCOSA OF VAGINA IN EACH LATERAL FORNIX. ACUPUNCTURE;

INCIDENCE OF SERIOUS NEUROLOGICAL COMPLICATIONS DUE TO EPIDURAL


ANALGESIA:

PARAPLEGIA, A SEVERE COMPLICATION TO EPIDURAL ANALGESIA €“ RARE.

BECAUSE OF THE RARITY OF PERMANENT NEUROLOGICAL DAMAGE RESULTING FROM


EPIDURAL ANALGESIA, IT IS DIFFICULT TO ESTIMATE ITS INCIDENCE. IN A COMBINED
SERIES OF MORE THAN 50 000 EPIDURAL ANAESTHETICS, ONLY THREE PATIENTS
SUFFERED PERMANENT LEG WEAKNESS (0.006% - AMERICAN JOURNAL OF ANAESTHESIA ,
KANE 1981)

12.OBESE PATENT / BMI 45 €“ COUNSELLING

13.DIZZINES/VERTIGO 60 YO €“ HISTORY/DDX


14. CHEST PAIN: €“ HISTORY/ (PERICARDITIS) DDX: MI/ANGINA/PE/AORTA DISSECTION

50-YEAR-OLD TRAIN DRIVER, C/O CHEST PAIN. FROM THE HISTORY: PAIN FOR THE LAST
6 HOURS, CONSTANT, 8/10, RADIATES TO THE BACK (ALLEVIATING/AGGRAVATING
FACTORS?). NON-SMOKER, HAD HISTORY OF CHEST INFECTION A 3/52 AGO. VITAL SIGNS
€“ PULSE 96 REGULAR, BP 140/97, TEMP. 37.4 º C. PHYSICAL EXAM €“ CONSTANT
NOISE DURING AUSCULTATION? NOT MURMUR; PROBABLY PLEURAL RUB

ACUTE PERICARDITIS: CHEST PAIN, WHICH MAY BE INTENSE, MIMICKING ACUTE MI,
BUT CHARACTERISTICALLY SHARP, PLEURITIC AND POSITIONAL (RELIVED BY LEANING
FORWARD) WORSE ON INSPIRATION; FEVER AND PALPITATIONS ARE COMMON

COMMON CAUSES: IDIOPATHIC; INFECTIONS (PARTICULARLY VIRAL €“ INFLUENZA,


COXSACKIE A/B) ACUTE MI, METASTATIC NEOPLASM, RADIATION THERAPY FOR THE
TUMOUR (UP TO 20 YEARS EARLIER), CONNECTIVE TISSUE DISEASE (SLE, RA) DRUG
REACTIONS (PROCAINAMIDE, HYDRALAZINE), €ŒAUTOIMMUNE €ŒFOLLOWING
HEART SURGERY OF MI €“ SEVERAL WEEKS/MONTHS LATER (DRESSLER€™S SY).

PHYSICAL EXAMINATION: RAPID OR IRREGULAR PULSE, COARSE PERICARDIAL FRICTION


RUB WHICH MAY VARY IN INTENSITY AND IS LOUDEST WITH PT. SITTING FORWARD.

LABORATORY/ECG: DIFFUSE ST ELEVATION (CONCAVE UPWARDS) USUALLY PRESENT IN


ALL LEADS EXCEPT AVR AND V1.CXR: INCREASED SIZE OF CARDIAC SILHOUETTE IF LARGE
(> 250 ML) PERICARDIAL EFFUSION IS PRESENT, WITH €ŒWATER BOTTLE€
CONFIGURATION ECHO: MOST SENSITIVE TEST FOR DETECTION OF PERICARDIAL
EFFUSION WHICH COMMONLY ACCOMPANIES ACUTE PERICARDITIS;

TREATMENT: ASPIRIN 650 €“975 MG QID OR NSAID (E.G. INDOMETHACIN 25 €“75


MG QID); FOR SEVERE REFRACTORY PAIN, PREDNISONE 40-60MG DAILY AND TAPERED
OVER SEVERAL WEEKS OR MONTHS. ANTICOAGULANTS ARE RELATIVELY
CONTRAINDICATED IN ACUTE PERICARDITIS BECAUSE OF RISK OF PERICARDIAL
HAEMORRHAGE. ( HARRISON, P583)
15.INTUSUSSEPCTION

16.INTERMITENT CLAUDICATIONS

BACK TO TOP

NADIAAL

GUEST

POSTED: SAT SEP 30, 2006 12:57 PM POST SUBJECT:


--------------------------------------------------------------------------------

AMC MCQ EXAM 2004 MELBOURNE

NADIA RECOLLECTION PAPER

(*) - DENOTES STAR QUESTIONS ( 60 QUESTIONS)

P - DENOTES PILOT QUESTIONS ( PROBABLY WILL APPEAR AS (*) IN NEXT EXAM ? )

1.ALL OF THE FOLLOWING DRUGS CAN CAUSE GYNECOMASTIA EXCEPT:

A/ SPIRONOLACTONE

B/ DIGOXIN

C/ METHYLDOPA

D/ FUROSEMIDE

E/

2. WHICH OF THE FOLLOWING JOINTS IS NOT INCLUDED IN RHEUMATIC ARTHRITIS AND


IS MORE COMMON IN OSTEOARTHRITIS?

A/ CARPOMETACARPAL JOINT OF THE THUMB P

B/ TEMPOROMANDIBULAR JOINT

C/ CERVICAL SPINE

D/
E/

3. ALL OF THE FOLLOWING STATEMENTS ABOUT INTRAVENOUS SOLUTIONS ARE TRUE


EXCEPT:

A/ NORMAL (0.9%) SODIUM CHLORIDE SOLUTION HAS AN APPROXIMATELY 150


MMOL/L NA + 150 MMOL/L CL

B/ HARTMAN SOLUTION IS SODIUM LACTATE WHICH CONSIST SODIUM 135 MMOL,


CALCIUM 2 MMOL AND SOME POTASSIUM

C/ IF 1 LITRE OF HARTMAN SOLUTION IS GIVEN IN 24 HOURS IT WILL BE SUFFICIENT FOR


DAILY POTASSIUM REQUIREMENT (*)

D/ N/5 CONTAINS 30 ML CL AND 30 ML SODIUM

E/ 5 % DEXTROSE €¦

4.HAND X €“ RAY, CARPAL BONES, FRACTURED SCAPHOID. WHICH OF THE FOLLOWING
IS CORRECT:

A/ IT WILL RESULT IN AVASCULAR NECROSIS OF DISTAL POLE

B/ INTERNAL FIXATION IS BEST TREATMENT BECAUSE OF MALUNION

C/CREPE BANDAGE FROM ELBOW TO THE WRIST

D/IMMOBILIZATION FROM ELBOW TO THE END OF THE THUMB FOR 6 WEEKS (*)

E/ PLASTER FROM ELBOW TO THE WRIST

5.PHOTO OF LEG FROM AMC BOOK ( SU-C130 ) PAGE 262


A/ CHRONIC INFECTION

B/ NECROTIZING FASCIITIS

C/CHRONIC VENOUS HYPERTENSION (*)

D/ARTERIAL INSUFFICIENCY

E/

6.PATIENT WITH A FOURTH ATTACH OF GOUT IN THE LAST 2 YEARS. KNEE IS SWOLLEN,
PAINFUL. WHAT IS MOST APPROPRIATE MANAGEMENT?

A/ ALLOPURINOL IN THE FIRST 24 HOURS

B/INDOMETHACIN 150 MG IN THE FIRST 24 HOURS (*)

C/HYDROCORTISONE INTRAARTICULAR

D/COLCHICINE

E/

7.URTICARIA CAN BE ASSOCIATED WITH ALL OF THE FOLLOWING EXCEPT:

A/ VASCULITIS P

B/ASPIRIN SENSITIVITY

C/

D/

E/
8. WHICH CAUSE SHOCK IN GALL-BLADDER SEPSIS?

A/STAPHYLOCOCCUS AUREUS

B/PSEUDOMONAS AERUGINOSA

C/ENTEROCOCCUS FAECALIS

D/ ESCHERICHIA COLI P

E/

9.PATIENT WITH SEVERE CHEST PAIN, RADIATING TO HIS BACK. CXR SHOW WIDENED
MEDIASTINUM, DIASTOLIC MURMUR ECG €“ ACUTE INFERIOR M.I. WHAT IS
APPROPRIATE MX?

A/GIVE STREPTOKINASE

B/GIVE ASPIRIN PLUS IV MORPHINE

C/GIVE MORPHINE, B-BLOCKER AND ARRANGE FOR T O E

D/

E/
10.WHAT IS MOST LIKELY TO BECOME CHRONIC?

A/HEPATITIS A

B/ HEPATITIS C (*)

C/ HEPATITIS B

D/ EBV

E/

11.SPINAL CORS LESION AT LEVEL L4 €“ L5 :

A/ ABSENT KNEE JERK

B/ ABSENT ANKLE JERK

C/ LOST OF DORSIFLEXION OF THE ANKLE

D/ LOST OF SENSATION SOLE OF THE FOOT

E/

12. PATIENT WITH CHRONIC CARDIAC FAILURE ON CAPTOPRIL FOR HYPERTENSION. ON


LONG-TERM CHLOROTHIAZIDE.

A/ SODIUM IS REDUCED

B/ K IS DECREASED

C/CHOLESTEROL IS INCREASED

D/

E/
13.PATIENT 3 DAYS AFTER CRANIAL SURGERY. NA 117, PLASMA OSMOLARITY 205 URINE
OSMOLARITY 825

A/ D . III

B/ S I A D H (*)

C/WATER INTOXICATION

D/

E/

14. ALL OF THE FOLLOWING ARE FEATURES OF HYPERCALCEMIA EXCEPT:

A/ DEPRESSION

B/ DIARRHOEA

C/POLYURIA

D/ POLYDIPSIA

E/

15.WHICH OF THE FOLLOWING IS THE MOST URGENT REQUIREMENT FOR THE PATIENT
IN D K A:

A/ INFUSION OF SODIUM BICARBONATE

B/ 5% DEXTROSE

C/ INFUSION OF NORMAL SALINE (*)


D/ GLUCAGON

16. BUILDER COME TO ER WITH 15 % OF BURNS, HIS WEIGHT IS 80 KG. WHICH OF THE
FOLLOWING FLUID REPLACEMENT WILL BE REQUIRED FOR THE FIRST 24 HOURS?

A/2 L HARTMAN + 2 L BLOOD

B/ 2 L NACL + 1 L BLOOD

C/ 3 L HARTMANN + 2 L 5 % DEXTROSE

D/ 3 L HARTMANN + 2 L BLOOD

E/

17. AFTER SPLENECTOMY FOR HEREDITARY SPHEROCYTOSIS ALL CAN BE EXPECTED


EXCEPT:

A/ PERSISTENCE OF ANAEMIA

B/ PERSISTENCE OF SPHEROCYTES

C/ NORMAL RBC LIFE SPAN SEE MERCK P 876

D/RETICULOCYTOSIS

E/

18. WHICH OF THE FOLLOWING IS NOT FEATURE OF MYCOPLASMA PNEUMONIAE:

A/ HEADACHE

B/PLEURITIC PAIN P

C/TEMPERATURE

D/ MYALGIA
E/COUGH

19.WHICH OF THE FOLLOWING DRUGS IF STOPPED DURING THE OPERATION WILL


CAUSE INTRA OPERATIVE HYPERTENSION?

A/CLONIDINE

B/METHYLDOPA

C/ENALAPRIL

D/PROPRANOLOL

E/FELODIPINE

20. PERIPHERAL PULSE WILL BE INCREASED IN ALL OF THE FOLLOWING EXCEPT:

A/ A €“ V FISTULAE

B/PAGET DISEASE OF THE BONE

C/V S D

D/PATENT DUCTUS ARTERIOSUS

E/ASD

21. Q ABOUT PSEUDOBULBAR PALSY. ALL IS ASSOCIATED EXCEPT:

A/ GAG €“ REFLEX ( VAGUS ) P

B/

C/
D/

22.PATIENT PLAYING GAME, SQUASH COURT SUDDENLY PAIN IN ANKLE, NO PLANAR


FLEXION, DORSAL FLEXION LIMITED AND PAINFUL:

A/ RUPTURE OF MEDIAL ANKLE LIGAMENT

B/ COMPLETE RUPTURE OF ACHILLES TENDON

C/ RUPTURE OF MEDIAL HEAD M. SOLEI

D/

E/

23. WELL ESTABLISHED SCIENTIST WAS ON OVERSEAS SCIENTIFIC MEETING WHERE HE


SHOULD GIVE LECTURE. SUDDENLY AFTER BOUT OF DIARRHOEA, AND PROFOUND
HUMILIATION HE LEFT MEETING. HE START TO CHANGE HIS DRESSES AT NIGHT,
WANDERING THROUGH THE STREETS, HIS BEHAVIOUR THAN CHANGE AS WELL. IN THE
MORNING HE IS AGAIN OK. HE REALISED SOMETHING IS WRONG WITH HIM AND AFTER A
FEW

VADIMK, OCT 15, 2006

#2

SYLVIA.

SYLVIA. GUEST

THE MOST COMMON CAUSE OF FALSE POSITIVE VDRL TERS IS:

A)SLE

B)RA
C)TB

D)YAWS

2. MOST COMMON CAUSE OF RECTAL BLEEDING IN AN INFANT IS :

A) COW MILK ALLERGY

B)FISSURE

C)FISTULA

D) INTUSSUSEPTION

3. A BOY PRESENTED WITH NAUSEA, VOMITING AND MORNING HEADACHES. HE


CANNOT SEE THE BLACKBOARD IN THE CLASS.

A) MEDULLOBLASTOMA

B) ASTROCYTOMA

C) CRANIOPHARYNGIOMA

4.A BOY WITH BITEMPORAL HEMIANOPIA AND NORMAL GROUTH:

A)CRANIOPHARYNGIOMA

B)SELLAR TUMOR

C)EISINOPHILIC CELL TUMOR OF PITUITARY GLAND

5. A BOY DEVELOPS SUDDEN INGUINAL PAIN, THE TESTIS IS HIGH UP. EPIDIDYMIS
CANNOT BE PALPATED. DIAGNOSIS?

A) TORSION OF TESTIS

B) TORSION OF TESTIC AND THE SPERMATIC CORD.

C) EPIDIDMOORCHITIS
6. A PT FELL TO THE GROUND WITH AN OUTSTRETCHED HAND, HE DEVELOPED PAIN IN
THE WRIST. X RAY NORMAL. RX?

A) REASSURE

B) SCAPHOID PLASTER AND REPEAT X RAY AFTER 2 WEEKS

C) X RAY AFTER 6 WEEKS.

SYLVIA., FEB 2, 2007

#3

ATOOSA-

ATOOSA- GUEST

WHICH ANTIPSYCHOTIC DRUG IS RESTRICTED IN AUSTRALIA BECAUSE

1. IN WHICH OF THE FOLLOWING DIABETIC KETOACIDOSIS MOST COMMONLY


PRESENTS WITH

1. UNDIAGNOSED IDDM

2. UNDIAGNOSED NIDDM

3. KNOWN IDDM WHEN STOPPED INSULIN

4. KNOWN IDDM WITH FOOT INFECTION

4. PICTURE €“ GIRL ACNE AND HIRSUTISM ON THE FACE. PRESENTED FOR THE FIRST
TIME

1. ANTIBIOTICS
2. ANTIBIOTICS AND RETINOIDS

3. STEROIDS CREAM

4. ANTIANTIGEN(DANAZOL)

5. CYPROPTERONE ACETATE

16. ERADICATION OF HELICOBACTERIA PYLORY

1. INCRESED HEALING ULCER

2. DECREASED RELAPSE RATE

20. 54 YEARS OLD MAN WITH SUDDEND ONSET OF LEFT SIDE WEEKNESS WITHOUT
DISPHASIA. POSSIBLE DIAGNOSIS.

A. RIGHT MID C ARTERY

B. CORATIC ARTERRY OCLUSIOON

C. EPILEPSY

D. INTERNAL CAPSULE

24. A HYPERTENSION PATIENT WITH SUDDEN ONSET PAIN BEHIND LEFT EYE, WITH
PTOSIS, PALSY SIGNS OF VII, V, VI, (PARALYSIS OF PALATE, EYE MOVEMENT PARALYSS),
DX?

A. ANEURYSM OF POSTERIOR COMMUNICATION WILLIAM RING

B. MIDBRAIN INFARCT
C. BRAIN STEM INFARCTMENT

27. A YOUNG MAN WITH COUGH, MANTOUX +, CXR SHOWED INACTICVE TB LESION,
MX?

A. TRIPLE TREATMENT FOR 6 MONTHS AND REPEAT CXR

B. ISONIAZID TX FRO 3 MONS AND REPEAT CXR

C. 6 SPUTUM AND AWAIT

D. REPEAT CXR IN 3 MONTHS

29. BLOOD GAS PH 7.45 (N 7.35-7.44), PCO2. 27, PO2 65, HCO3 23,

A. RESPIRATORY ALKALOSIS WITH GAS CHANGE IMPAIRMENT

B. RESPIRATORY ALKALOSIS WITHOUT GAS CHANGE IMPAIRMENT

30. COPD WHICH IS THE RISK OF RIGHT HEART FAILURE

A. FEV <1 L

B. DIFFUSING CAPACITY <50%

C. REDUCED PO2

D. INCREASED PCO2
35.THE ABATTOIR WORKER IS PRESENTED WITH JAUNDICE, FEVER, AND MALAISE.

WHAT IS THE POSSIBLE DS:

1.LEPTOSPEROSIS

2.BRUCELLOSIS

3.MALARIA

4.DENGUE FEVER

38 THE PATIENTS PRESENTS WITH OEDEMA OF THE FACE .MX: TREATMENT FOR
HYPERTENSION,GOUT AND ALOPURINOL ,INDOMETACIN.THE MOST POSSIBLE CAUSE OF
HIS OEDEMA:

1.INDOMETCIN INTOXICATION(STOP THE TREATMENT)

2.RENAL FAILURE

10. 15 YR OLD BOY GOT ABRASION IN HIS KNEE. HE HAS BEEN IMMUNISED AT 5 YEARS
OLD AND NO IMMUNIZATION AFTERWARDS. WHAT IS YOUR MX?

A. TOXOID

B. ADT

C. IMMUNOGLOBULIN AND TOXOID

D. DO NOTHING

E. DPT

13.A CHILD HAS 20 BROWN MACULOUS SPOTS ALL OVER THE BODY. HIS
GRANDMOTHER HAS SKIN NEUROMA. ALL RIGHT EXCEPT
A.

B. MORE HYPERTENSION

C. MORE DEAFNESS

D. HYPOTHYRODISM

E. INTELLECTUAL RETARDATION

17. A CHILD SWALLOWED WITH SODA. CHILD PRESENT WITH DROOLING, IMMEDIATE
ACTION,

A. EARLY ENDOSCOPY

B. DRINK MILK

18. A 3 YEAR BOY WITH ABDOMINAL MASS WHICH TAKES MOST SPACE OF RIGHT
ABDOMINAL SPACE, THE USEFUL DIAGNOSTIC METHODS ARE EXCEPT:

A. URINE VMA

B. ULTRASOUND

C. CXR

D. BARIUM SWALLOW

E. ENDOSCOPY

23. A CHILD PRESENT WITH NECK STIFFNESS, FEVER, PROTEIN 1.1G/L, GLUCOSE 2.5 (N2-
5), LYMPHOCYTE 100, NEUTROPHIL 50, DX?

A. TB
B. ECHOVIRUS

C. VIRUS

D. STAPHYLOCOCCUS AUREUS

3 WHICH ANTIPSYCHOTIC DRUG IS RESTRICTED IN AUSTRALIA BECAUSE IT HAS VERY


LONG Q-T INTERVAL

A. CHLORPROMAZINE

B. THIORIDAZINE

C. RISPERIDONE

13 HYPERPROLACTINAEMIA WILL NOT CAUSE ONE OF THE FOLLOWING,

A. SEXUAL DYSFUNCTION

B. ERECTILE PROBLEMS

C. GALLACTORHEA

D. LOSS OF LIBIDO

REGARDS

ATOOSA-, FEB 2, 2007

#4

CASES4PACES2009

CASES4PACES2009 GUEST
REFERENCE: AMCQ FRIDAY SEP 29 2006:GORANI

MY RESPONSE:

BLEEDING FROM VENEPUNCTURE SITE> TYPICAL OF DIC

RECURRENT NEUROLOGICAL DEFICITS > MIGRAINE

DENIAL OF ILLNESS>LACK OF INSIGHT

TALKS TO ANGELS > DELUSION

CASES4PACES2009, SEP 2, 2009

#5

CASES4PACES2009

CASES4PACES2009 GUEST

RESPONSE TO :ATOOSA ,POSTED: FRI FEB 02, 2007 10:20 AM

01_UNDIAGNOSED DM > DKA

20_RT MCA INFARCT > LEFT HEMIPARESIS WITHOUT SPEECH DEFICIT

24_ ANEURYSM > PAINFUL OPTHALMOPLEGIA

29_ HYPOXIA > REDUCED GAS EXCHANGE

35_ ABATTOIR > LEPTOSPIROSIS

38_ HTN, NSAID,GOUT > RENAL FAILURE

CASES4PACES2009, SEP 2, 2009


#6

GUEST

GUEST GUEST

GYNAE CANCER

RISK:

C_CIN,HPV,SEX

E_ POSTMENOPAUSE,UNOPPOSED ESTROGEN ( EG.OVARIAN DYSFUNCTION),INSULIN


RESISTANCE,HNPCC,OVARIAN/BREAST CANCER,TAMOXIFEN

O_NULLIPARITY,DELAYED CHILDBEARING,EARLY MENARCHE,DELAYED


MENOPAUSE,BRCA1 MUTATION

GUEST, SEP 4, 2009

#7

GUEST

GUEST GUEST

PRIMARY AMENORRHEA IN 17 YR GIRL

PUBERTAL GROWTH ( BREAST,PUBIC HAIR ) EXCLUDES HYPOTHALAMUS,PITUITARY AND


GONADAL PROBLEMS.

> ABSENT UTERUS


2007QUES PLZ ANS THESE QS

HI ALL,

CAN SOMEBODY PLZ ANS THESE QS

THANKS ALOT.

8.* A DIABETIC PT HAS VOMITED SINCE LAST EVENING. CAME TO YOU AT 10 AM AND
HAS MISSED HIS MORNING DOSE. HE MEASURED GLUCOSE AT HOME WHICH WAS
10MG/DL. WHAT TO ADVISE

A. SEE AGAIN IN THE MID DAY

B. RECHECK GLUCOSE LEVEL IN MID DAY

C. ASK TO DRINK FLUID TILL NAUSEA SETTLES

D. INCREASE THE DOSE

E. ASK TO TAKE MISSED DOSE NOW

65. A LADY WITH BRUISES AND MARKS OF VIOLENCE COMES TO YOU FOR TREATMENT
OF AND OLD DISEASE. ON QUESTIONING, SHE ADMITS THAT HER PARTNER BEATS HER.
WHAT WOULD YOU DO

A. CALL THE POLICE

B. TREAT HER OLD DISEASE

C. TALK TO HER PARTNER

D. TELL HER AND HER PARTNER TO HAVE MARRIAGE COUNSELLING


91. A MAN ENTERS YOUR CLINIC AND SALUTES YOU 3 TIMES BEFORE COMING IN. WHAT
IS THIS

A. OBSESSIONS

B. MANNERISM

C. PREVIOUS ARMY TRAINING

D. MANIA

12) THE PATHOLOGY REPORT FOLLOWING A THERAPEUTIC ABORTION SHOWS

ARIAS-STELLA PHENOMENON (DECIDUALIZATION). THE MOST APPROPRIATE

NEXT STEP IN THE MANAGEMENT OF THIS PATIENT WOULD BE?

A) REPEAT A PREGNANCY TEST

B) LAPOROSCOPY

C) REASSURE THE PATIENT *

D) PRESCRIBE ORAL CONTRACEPTION

E) REPEAT THE D&C

GUEST, MAY 14, 2007

#1

DR.S.ALESSAWY

DR.S.ALESSAWY GUEST

WISH YOU GOOD LUCK WITH YOUR EXAM


8.* A DIABETIC PT HAS VOMITED SINCE LAST EVENING. CAME TO YOU AT 10 AM AND
HAS MISSED HIS MORNING DOSE. HE MEASURED GLUCOSE AT HOME WHICH WAS
10MG/DL. WHAT TO ADVISE

A. SEE AGAIN IN THE MID DAY

B. RECHECK GLUCOSE LEVEL IN MID DAY

C. ASK TO DRINK FLUID TILL NAUSEA SETTLES**

D. INCREASE THE DOSE

E. ASK TO TAKE MISSED DOSE NOW

65. A LADY WITH BRUISES AND MARKS OF VIOLENCE COMES TO YOU FOR TREATMENT
OF AND OLD DISEASE. ON QUESTIONING, SHE ADMITS THAT HER PARTNER BEATS HER.
WHAT WOULD YOU DO

A. CALL THE POLICE

B. TREAT HER OLD DISEASE

C. TALK TO HER PARTNER

D. TELL HER AND HER PARTNER TO HAVE MARRIAGE COUNSELLING ***GIVE HER A
NUMBER TO CALL

91. A MAN ENTERS YOUR CLINIC AND SALUTES YOU 3 TIMES BEFORE COMING IN. WHAT
IS THIS

A. OBSESSIONS

B. MANNERISM**

C. PREVIOUS ARMY TRAINING

D. MANIA

12) THE PATHOLOGY REPORT FOLLOWING A THERAPEUTIC ABORTION SHOWS


ARIAS-STELLA PHENOMENON (DECIDUALIZATION). THE MOST APPROPRIATE

NEXT STEP IN THE MANAGEMENT OF THIS PATIENT WOULD BE?

A) REPEAT A PREGNANCY TEST

B) LAPOROSCOPY

C) REASSURE THE PATIENT *

D) PRESCRIBE ORAL CONTRACEPTION

E) REPEAT THE D&C

I AM NOT SURE ABOUT THE LAST ONE

DR.S.ALESSAWY, MAY 15, 2007

#2

GUEST

GUEST GUEST

THANKS ALOT FOR THE ANSWERS...I HAVE FEW MORE QUERIES.....IF ANYONE CAN
ANSWER THEM ASAP THEN IT WUD BE GR8...THANKS

1.72 YR LADY PRESENTS WITH CONFUSION,SHE IS AFEBRILE.SHE HAS AN ULCER ON


LOWER PART OF LEG ,WHICH HAS BEEN TREATED BY COMMUNITY NURSE FOR SEVERAL
MONTHS.WHAT WILL ASSIST YOU IN DX?

A.SWAB THE ULCER

B.URINE TEST

C)CT SCAN

D)LUMBER PUNCTURE
E)BLOOD CULTURE.

2)IN 6 HRS AFTER CYSTOSCOPY A PT PRESENTS WITH FEVER 40C AND CHILLS.WOF IS
MOST LIKELY?

A.ACUTE PYELONEPHRITIS

B.BLADDR RUPTURE

C.STAPH INFECTION

D.SUPPERATIVE URETHRITIS.

E.CYSTITIS

9. ONE OF YOUR COLLEAGUE IS TAKING ANTI PSYCHOTIC MEDICATION FOR HER OWN
PSYCHIATRIC ILLNESS. WHAT SHOULD BE YOUR ADVICE TO HER??

A. SHE SHOULD REFRAIN FROM SEEING PT. UNTIL SHE IS ASYMPTOMATIC

B. SHE SHOULD TAKE SPECIALIST REVIEW

C. YOU SHOULD CONTACT THE MEDICAL ADVISORY BOARD

10. WHAT IS THE MOST COMMON ASSOCIATION OF CHILDHOOD OBESITY IN


AUSTRALIA?

A. ABOVE AVERAGE HEIGHT

B. HYPERCHOLESTEROLAEMIA

C. DM

D. CATARACT
11. WHAT WILL BE THE FIRST S/S WHEN A PLASTER IS TOO TIGHT?

A. PAIN

B. CHANGE OF COLOUR

C. SWELLING

D. STIFFNESS

GUEST, MAY 16, 2007

#3

DR SONU

DR SONU GUEST

WHICH OF THE FOLLOWING IS TRUE ABOUT TUBERCULOSIS PLEURAL EFFUSION?

A- BLOOD-STAINED ALWAYS

B- ACID-FAST BACILLI ARE ALWAYS FOUND

C- MOST LIKELY CELLS ARE LYMPHOCYTES

D- IT IS TRANSUDATE

E- GLUCOSE IS NORMAL OR DECREASED

2. ACUTE ATTACKS OF ASTHMA CAN BEST BE TREATED WITH:

A- INHALED SALBUTAMOL

B- ORAL-STEROIDS

C- INHALED STEROIDS
D- ORAL THEOPHYLINE

E- PEFR MEASURED AT HOME

3. A YOUNG MAN PRESENTS WITH PLEURETIC CHEST PAIN AND COUGH. ON


PERCUSSION THERE IS DULLNESS OVER THE RIGHT BASE. THERE ARE NO BREATH SOUNDS
IN THE RIGHT LOWER ZONE, BUT BRONCHIAL BREATHING IS NOTED IN THE RIGHT MIDDLE
ZONE. WHICH IS THE MOST LIKELY DIAGNOSIS?

A- RIGHT LOWER LOBE CONSOLIDATION

B- RIGHT LOWER LOBE COLLAPSE

C- PLEURAL EFFUSION

D- RIGHT PNEUMOTHOREX

E- LEFT TENSION PNEUMOTHOREX

4 .ALL OF THE FOLLOWING ARE THE TRUE EMPHYSEMA, EXCEPT:

A- CLUBBING IS PRESENT

B- DISTANT HEART SOUNDS

C- DIFFUSING CAPACITY IS DECREASED

D- TOTAL LUNG CAPACITY IS DECREASED

E- HYPERINFLATION OF THE LUNGS

DR SONU, MAY 16, 2007

#4

DR SONU

DR SONU GUEST
PHOTOSENSITIVITY IS PRESENT IN:

A- AMIODARONE TREATMENT

B- SLE

C- NIACIN DEFICIENCY

D- THIAMINE DEFICIENCY

E-

2. IN A PATIENT WITH MASSIVE ASCITIES, ALL ARE POSSIBLE FINDING EXCEPT:

A- HEPATOMEGALY

B- SHIFTING DULLNESS

C- EVERSION OF UMBILICUS

D- DULLNESS TO PERCUSSION

3. WHICH ARE THE FOLLOWING DRUGS CAN GIVE NEPHROTOXICITY:

A- GENTAMYCIN

B- GOLD

C- DIGOXIN

D- SULPHNAMIDES

E- MEFENAMIC ACID

DR SONU, MAY 16, 2007

#5

DR SONU

DR SONU GUEST
. A 65 YRS OLD WOMAN BECAME CONFUSED. LABORATORY FINDINGS WERE: CALCIUM
4.8 INCREASED CAICIURIA, DECREASED PHOSPHATE. WHICH IS THE MOST LIKELY
DIAGNOSIS?

A- MALIGNANCY (OSTEOLYTIC METASTASES)

B- PRIMARY HYPERTHYROIDISM

C- IDIOPATHIC HYPERCALCAEMIA HYPERCALCIURIA

D- PAGET€™S DISEASE

E- MULTIPLE MYELOMA

2. PAPILLARY NECROSIS IS CAUSED MOSTLY BY:

A- DIABETES

B- ALCOHOL

C- ANALGASIC NEPHROPATHY

D- MEDULLARY SPONGE KIDNEY

E- SICKLE CELL CRISIS

3. IN A PATIENT WITH ACIDIC URINE, WHICH ARE THE MOST COMMONLY FOUND
STONES?

A- URIC ACID

B- CYSTINE

C- CALCIUM OXALATES

D- CALCIUM CARBONATE

E- XANTHINE
4. IN PATIENT WITH CHRONIC RENAL FAILURE:

A- MOST PATIENT DEVELOP OSTEOPOROSIS

B- BONE DYSTROPHY IS PRESENT

DR SONU, MAY 16, 2007

#6

DR SONU

DR SONU GUEST

A WOMAN PRESENTS WITH DARK COLOR URINE, PRURITUS AND JAUNDICE. BLOOD
EXAMINATION SHOWS A MILD INCREASE IN ALKALINE PHOSPHATES, HIGH AST AND ALT.
WHICH IS THE MOST LIKELY DIAGNOSIS?

A- VIRAL HEPATITIS

B- ACUTE FATTY LIVER

C- CHOLANGITIS

D- BILIARY OBSTRUCTION WITH CHOLESTASIS

E- CHOLECYSTITIS

2.WHICH OF THE FOLLOWING IS TRUE REGARDING PHENYTOIN:

A- SHOULD BE GIVEN THREE TIMES DAILY

B- IT WILL DECREASE THE EFFECT OF WARFARIN

C- IT WILL DECREASE THE EFFECT OF OCR

D- IF GIVEN IN COMBINATION WITH OTHER DRUGS, THE THERAPEUTIC DOES SHOULD


BE HALVED.

E- ?
DR SONU, MAY 16, 2007

#7

SIMPSON

SIMPSON GUEST

1.72 YR LADY PRESENTS WITH CONFUSION,SHE IS AFEBRILE.SHE HAS AN ULCER ON


LOWER PART OF LEG ,WHICH HAS BEEN TREATED BY COMMUNITY NURSE FOR SEVERAL
MONTHS.WHAT WILL ASSIST YOU IN DX?

A.SWAB THE ULCER

B.URINE TEST ++++++

C)CT SCAN

D)LUMBER PUNCTURE

E)BLOOD CULTURE.

2)IN 6 HRS AFTER CYSTOSCOPY A PT PRESENTS WITH FEVER 40C AND CHILLS.WOF IS
MOST LIKELY?

A.ACUTE PYELONEPHRITIS +++++++

B.BLADDR RUPTURE

C.STAPH INFECTION

D.SUPPERATIVE URETHRITIS.

E.CYSTITIS

9. ONE OF YOUR COLLEAGUE IS TAKING ANTI PSYCHOTIC MEDICATION FOR HER OWN
PSYCHIATRIC ILLNESS. WHAT SHOULD BE YOUR ADVICE TO HER??
A. SHE SHOULD REFRAIN FROM SEEING PT. UNTIL SHE IS ASYMPTOMATIC

B. SHE SHOULD TAKE SPECIALIST REVIEW

C. YOU SHOULD CONTACT THE MEDICAL ADVISORY BOARD ++++++++

10. WHAT IS THE MOST COMMON ASSOCIATION OF CHILDHOOD OBESITY IN


AUSTRALIA?

A. ABOVE AVERAGE HEIGHT +++++++++

B. HYPERCHOLESTEROLAEMIA

C. DM

D. CATARACT

11. WHAT WILL BE THE FIRST S/S WHEN A PLASTER IS TOO TIGHT?

A. PAIN ++++++++++

B. CHANGE OF COLOUR

C. SWELLING

D. STIFFNESS

WHICH OF THE FOLLOWING IS TRUE ABOUT TUBERCULOSIS PLEURAL EFFUSION?

A- BLOOD-STAINED ALWAYS

B- ACID-FAST BACILLI ARE ALWAYS FOUND

C- MOST LIKELY CELLS ARE LYMPHOCYTES +++++++

D- IT IS TRANSUDATE

E- GLUCOSE IS NORMAL OR DECREASED


2. ACUTE ATTACKS OF ASTHMA CAN BEST BE TREATED WITH:

A- INHALED SALBUTAMOL

B- ORAL-STEROIDS

C- INHALED STEROIDS ++++++

D- ORAL THEOPHYLINE

E- PEFR MEASURED AT HOME

3. A YOUNG MAN PRESENTS WITH PLEURETIC CHEST PAIN AND COUGH. ON


PERCUSSION THERE IS DULLNESS OVER THE RIGHT BASE. THERE ARE NO BREATH SOUNDS
IN THE RIGHT LOWER ZONE, BUT BRONCHIAL BREATHING IS NOTED IN THE RIGHT MIDDLE
ZONE. WHICH IS THE MOST LIKELY DIAGNOSIS?

A- RIGHT LOWER LOBE CONSOLIDATION

B- RIGHT LOWER LOBE COLLAPSE

C- PLEURAL EFFUSION +++++++++++++

D- RIGHT PNEUMOTHOREX

E- LEFT TENSION PNEUMOTHOREX

4 .ALL OF THE FOLLOWING ARE THE TRUE EMPHYSEMA, EXCEPT:

A- CLUBBING IS PRESENT +++++++

B- DISTANT HEART SOUNDS

C- DIFFUSING CAPACITY IS DECREASED

D- TOTAL LUNG CAPACITY IS DECREASED

E- HYPERINFLATION OF THE LUNGS


PHOTOSENSITIVITY IS PRESENT IN:

A- AMIODARONE TREATMENT

B- SLE

C- NIACIN DEFICIENCY +++++++++++++

D- THIAMINE DEFICIENCY

E-

2. IN A PATIENT WITH MASSIVE ASCITIES, ALL ARE POSSIBLE FINDING EXCEPT:

A- HEPATOMEGALY +++++++++++++

B- SHIFTING DULLNESS

C- EVERSION OF UMBILICUS

D- DULLNESS TO PERCUSSION

3. WHICH ARE THE FOLLOWING DRUGS CAN GIVE NEPHROTOXICITY:

A- GENTAMYCIN

B- GOLD +++++++++

C- DIGOXIN

D- SULPHNAMIDES

E- MEFENAMIC ACID

. A 65 YRS OLD WOMAN BECAME CONFUSED. LABORATORY FINDINGS WERE: CALCIUM


4.8 INCREASED CAICIURIA, DECREASED PHOSPHATE. WHICH IS THE MOST LIKELY
DIAGNOSIS?

A- MALIGNANCY (OSTEOLYTIC METASTASES)

B- PRIMARY HYPERTHYROIDISM

C- IDIOPATHIC HYPERCALCAEMIA HYPERCALCIURIA


D- PAGET€™S DISEASE

E- MULTIPLE MYELOMA ++++++++++

2. PAPILLARY NECROSIS IS CAUSED MOSTLY BY:

A- DIABETES

B- ALCOHOL

C- ANALGASIC NEPHROPATHY

D- MEDULLARY SPONGE KIDNEY ++++++++++++

E- SICKLE CELL CRISIS

3. IN A PATIENT WITH ACIDIC URINE, WHICH ARE THE MOST COMMONLY FOUND
STONES?

A- URIC ACID +++++++

B- CYSTINE

C- CALCIUM OXALATES

D- CALCIUM CARBONATE

E- XANTHINE

4. IN PATIENT WITH CHRONIC RENAL FAILURE:

A- MOST PATIENT DEVELOP OSTEOPOROSIS

B- BONE DYSTROPHY IS PRESENT+++++++

A WOMAN PRESENTS WITH DARK COLOR URINE, PRURITUS AND JAUNDICE. BLOOD
EXAMINATION SHOWS A MILD INCREASE IN ALKALINE PHOSPHATES, HIGH AST AND ALT.
WHICH IS THE MOST LIKELY DIAGNOSIS?
A- VIRAL HEPATITIS +++++

B- ACUTE FATTY LIVER

C- CHOLANGITIS

D- BILIARY OBSTRUCTION WITH CHOLESTASIS

E- CHOLECYSTITIS

2.WHICH OF THE FOLLOWING IS TRUE REGARDING PHENYTOIN:

A- SHOULD BE GIVEN THREE TIMES DAILY ++++++++

B- IT WILL DECREASE THE EFFECT OF WARFARIN

C- IT WILL DECREASE THE EFFECT OF OCR

D- IF GIVEN IN COMBINATION WITH OTHER DRUGS, THE THERAPEUTIC DOES SHOULD


BE HALVED.

E- ?

MCQ QUESTIONS JULY 2005

1. A YOUNG MEDICAL STUDENT, WHO HAS STARTED HIS CLINICAL POSTINGS, COMES TO
BECAUSE HIS COLLEAGUES NOTICED A YELLOWISH, TINGE TO HIS SCLERA. ON
EXAMINATION HE IS NORMAL EXCEPT FOR THE YELLOW SCLERA. INVESTIGATIONS
SHOWED:

TOTAL BILIRUBIN€”ELEVATED

DIRECT BILIRUBIN €“ ELEVATED

DIRECT BILIRUBIN > (TOTAL €“ DIRECT BILIRUBIN)

WHICH OF THE FOLLOWING IS MOST LIKELY?


A) GILBERT€™S SYNDROME

B) HAEMOLYTIC JAUNDICE

C) CA PANCREAS

D) HEPATITIS

E) CAROTENEMIA

2. A YOUNG MAN HAS A SYNCOPAL ATTACK WHILE WEIGHT LIFTING. HE HAS HAD
SIMILAR EPISODES TWICE BEFORE. HIS FATHER DIED OF CARDIAC DISEASE. WHAT IS THE
MOST APPROPRIATE MANAGEMENT?

A) HOLTER MONITOR

B) ECHOCARDIOGRAM

C) STESS TEST

D) BP IN SUPINE & LYING DOWN

E) CT SCAN

3. A 23-YEARS OLD YOUNG LADY COMPLAINTS OF LETHARGY AND WEAKNESS. SHE HAS
SORE THROAT AND 3 DAYS AGO SHE HAS RETURNED FROM JAVA WHERE SHE SUFFERED
FROM FEVER RECENTLY. HER WEAKNESS IS DUE TO --

A) TB

B) TONSILLITIS

C) POST VIRAL FEVER FATIGUE

D) INFECTIUOS MONONUCLEOSIS
4. A 29-YEAR OLD WOMAN, 6 MONTHS PREVIOUSLY HAD PLEURISY, IN THE LAST 2
YEARS MOUTH ULCERS, NOW DEVELOPS STIFFNESS AND OEDEMA OF THE WRIST AND
MCP JOINTS; IN THE MORNING WHICH DISAPPEARS AFTERWARDS. THE MOST LIKELY
CAUSE?

A) RHEUMATOID ARTHRITIS

B) CROHNS DISEASE

C) ULCERATIVE COLITIS

D) SLE

E) DEMATOMYOSITIS

5. WHICH OF THE FOLLOWING MOST COMMONLY CAUSES PULMONARY HYPERTENSION


& CORPULMONALE?

A) EMPHYSEMA

B) BRONCHIECTASIS

C) PULMONARY EMBOLISM

D) PNEUMOTHORAX

E) FOREIGN BODY
6. WHICH OF THE FOLLOWING IS A COMMON SIDE EFFECT OF CALCIUM CHANNEL
BLOCKERS?

A) PERIPHERAL OEDEMA

B) ANGIO-OEDEMA

C) HEADACHE

D) INSOMNIA

E) COUGH

7. IN DIABETIC NEUROPATHY YOU CAN FIND EACH OF THE FOLLOWING EXCEPT?

A) BRADYCARDIA

B) URINE RETENTION

C) IMPOTENCE

D) DIARRHOEA AT NIGHT

E) FOOT ULCER

8. ALL OF THE FOLLOWING ARE TRUE ABOUT DUODENAL ULCER ,EXCEPT?

A) HUNGER PAIN

B) LOSS OF APPETITE

C) WEIGHT GAIN
D) PELAPSES & REMISSIONS

E) RELIEF BY ANTACIDS

9. AN 18-YEAR-OLD STUDENT PRESENTS WITH JAUNDICE, SORE THROAT AND DARK


URINE. THERE WAS NO HEPATITIS CONTACT, NO IV DRUG USE, AND NO RECENT TRAVEL
HISTORY. BILIRUBIN 80, ELEVATED GAMMA-GT, AST 300, ALT 400 AND ALBUMIN
NORMAL. WHAT IS THE MOST LIKELY DIAGNOSIS?

A) HEPATITIS A

B) HEPATITIS B

C) INFECTIOUS MONONUCLEOSIS

D) CHOLANGITIS

E) ACUTE CHOLECYSTITS

10. HOW WOULD YOU DIFFERENTIATE BETWEEN HAEMOLYTIC ANAEMIA AND


ANAEMIA OF CHRONIC BLODD LOSS?

A) DECREASE HAPTOGLOBIN

B) INCREASED RETICULOCYTE COUNT

C) MEGALOBLASTOSIS

D) POLYCHROMASIA

E) RAISED FERRITIN
11. A 65-YEARS OLD PRESENTS WITH CONFUSION. THE BLOOD TEST SHOW RERUM NA
165, URINE OSMOLALITY 205. THE MOST LIKELY DIAGNOSIS IS?

A) SIADH

B) DIABETES MELLITUS

C) DIABETES INSIPIDUS

D) WATER INTOXICATION

E) RENAL FAILURE

12. A 70-YEARS OLD WOMAN PRESENTS WITH A 3-MONTH HISTORY OF FATIGUE,


GENERALIZED LYMPHADENOPATHY, AND HEPATOSPLENOMEGALY. WHAT IS THE MOST
LIKELY DIAGNOSIS?

A) HODGKIN€™S LYMPHOMA

B) NON HODGKIN€™S LYMPHOMA

C) ACUTE LYMPHOCYTIC LEUKAEMIA

D) CHRONIC LYMPHOCYTIC LEUKAEMIA

E) CHRONIC MYELOID LEUKAEMIA

13. REGARDING IMMUNOLOGY, WHICH OF THE FOLLOWING IS CORRECT?


A) NATURAL KILLER CELLS ARE A SUBSET OF T CELLS

B) B CELLS PRODUCE FIRST IG M AND LATER IG G

C) IG E IS INVOLVED IN MUCOSAL PROTECTION

D) OPSONIZATION€¦

E)

14. A 60-YEARS OLD OBESE MAN, ON ACE INHIBITORS FOR HYPERTENSION,


METFORMIN & GLIBENCLAMIDE FOR DIABETES, BENZODIAZEPINES & TCA€™S FOR
DEPRESSION & INSOMNIA, NOW COMPLAINTS OF IMPOTENCE. WHICH DRUG IS THE
MOST LIKELY RESPONSIBLE FOR THE IMPOTENCE?

A) GLIBENCLAMIDE

B) METFORMIN

C) TRICYCLIC ANTIDEPRESSANTS

D) BENZODIAZEPINE

E) ACE INHIBITORS

15. A 30-YEAR OLD BISEXUAL MALE, PRESENTS WITH CLEAR URETHRAL DISCHARGE,
MOUTH ULCERS, ARTHRALGIA, FATIGUE AND SAUSAGE LIKE FINGERS. CULTURE WAS
NEGATIVE. WHAT IS THE MOST LIKELY DIAGNOSIS?

A) REITERS DISEASE
B) BECHET€™S DISEASE

C) ANKYLOSING SPONDYLITIS

D) SLE

E)

16. A 56-YEAR OLD MAN PRESENTS CONFUSED. HE CANNOT ABDUCT HIS EYES, THERE IS
A SLIGHT PROBLEM WITH ELEVATING HIS EYE AND NYSTAGMUS IS PRESENT
THROUGHOUT. MOST LIKELY DIAGNOSIS IS?

A) WERNICKE€™S ENCEPHALOPATHY

B) .

C) .

D) .

E) .

17. A 23-YEARS OLD UNIVERSITY STUDENT HAS FEVER, ARTHRALGIA,


LYMPHADENOPATHY & MALAISE. ALT, AST & GGT ARE ELEVATED. PERIPHERAL SMEAR
SHOWS ATYPICAL LYMPHOCYTES. THE DIAGNOSIS IS?

A) HEPATITIS A

B) HEPATITIS C

C) INFECTIOUS MONONUCLEOSIS

D) CMV

E) ALL
18. HYPOKALEMIA OCCURS IN ALL EXCEPT?

A) THIAZIDE DIURETICS

B) PYLORIC STENOSIS

C) CHRONIC RENAL FAILURE

D) LAXATIVE ABUSE

E) HYPERALDOSTERONISM

19. A 22 YEAR OLD MAN DEVELOPED HAEMATURIA AND SLIGHT SWELLING OF THE
FACE. ON EXAMINATION BP-150/80 MMHG, RBC & HYALINE CASTS ARE PRESENT IN THE
URINE. MOST LIKELY DIAGNOSIS?

A) NEPHROTIC SYNDROME

B) GLOMERULONEPHRITIS

C) PYELONEPHRITIS

D) IG A NEPHROPATHY

E) RENAL TUBULAR NECROSIS

20. A 37-YEARS OLD WOMAN HAS UNILATERAL EXOPTHALMOS. WHICH OF THE


FOLLOWING IS THE MOST LIKELY CAUSE?

A) GLAUCOMA

B) NASOPHRYNGEAL TUMOUR INVADING THE ORBIT


C) A NEOPLASTIC LESION IN THE LAMINA CRIBROSA

D) THYROTOXICOSIS

E) INFECTION OF THE FRONTAL SINUS

F) MAXILLARY ANTRAL TUMOR

21. ALL OF THE FOLLOWING ARE SIDE EFFECTS OF PREDNISOLONE EXCEPT?

A) HIRSUTISM

B) LYMPHOPENIA

C) LEUCOCYTOSIS

D) OSTEOMALACIA

E) WEIGHT GAIN

22. A PATIENT YOU TREATED FOR DUODENAL ULCER WITH HELICOBACTER PYLORI FOR 1
WEEK WITH TREIPLE THERAPY. HE IS ASYMPTOMATIC NOW. WHICH OF THE FOLLOWING
IS THE BEST FOLLOW UP?

A) UREA BREATH TEST

B) ENDOSCOPY

C) SEROLOGY

D) HISTOLOGICAL EXAMINATION

E) CULTURE
23. A 20-YEAR OLD MAN IS FOUND TO HAVE HIV. WHAT DOES THIS INDICATE?

A) HE HAS AIDS NOW

B) HE HAS GOT LIFELONG INFECTION & INFECTIVITY

C) HE NEEDS TO START PNEUMOCYSTIS CARINII PROPHYLAXIS

D) HE REQUIRES TO START AZT THERAPY

E) HE CAN TRANSMIT THROUGH SALIVA NOW

24. A 70-YEAR OLD WOMAN COMPLAINTS OF FATIGUE. BLOOD PICTURE---

MCV 118, HB DECREASED, WBC DECREASED, PLATELETS DECREASED,


HYPERSEGMENTED POLYMORPHS PRESENT. THE MOST LIKELY DIAGNOSIS?

A) AUTOIMMUNE HAEMOLYTIC ANAEMIA

B) PERNICIOUS ANAEMIA

C) ANAEMIA OF CHRONIC DISEASE

D) ANAEMIA DUE TO SILENT GASTROINTESTINAL BLEEDING

E) LEUKAEMIA(CML)

25. A 68 YEARS OLD MAN WITH COPD WAS BROUGHT TO YOUR SURGERY FROM
NURSING HOME BY AMBULANCE. ON HIS WAY TO HOSPITAL HE RECEIVED O2, 10 L/M BY
MASK. HE IS STILL UNAROUSABLE AND HIS ABG MOST LIKELY--
A) PH 7.29 PACO2 65 PAO2 85

B) PH 7.15 PACO2 50 PAO2 68

C) PH 7.25 PACO2 25 PAO2 100

D) PH 7.35 PACO2 40 PAO2 40

E) PH 7.45 PACO2 85 PAO2 40

26. A 24-YEAR OLD FEMALE HAS ABDOMINAL PAIN EVERY WEEK. SOMETIMES SHE HAS
FACE & LIP SWELLING. HER MOTHER AND SISTER ALSO HAVE THIS. WHAT WOULD YOU DO
TO ESTABLISH A DIAGNOSIS?

A) MEASURE CL ESTERASE

B) CHECK FOR FOOD ALLERGIES

C) CHECK PB LEVELS

D) IG A DEFICIENCY

27. A PATIENT COMPLAINTS OF HEADACHE, VOMITING & PAPILLOEDEMA. WHICH OF


THE FOLLOWING IS THE MOST LIKELY CAUSE?**

A) GILOMA OF THE FRONTAL LOBE

B) TUMOUR OBSTRUCTING THE 4TH VENTRICLE

C) PITUITARY TUMOUR

D)
28. AN ELDERLY PATIENT HAS ACUTE ONSET UNILATERAL DEAFNESS, TINNITIS &
VERTIGO. WHAT IS THE DIAGNOSIS?

A) MENIERE€™S DISEASE

B) ACOUSTIC NEUROMA

C) VESTIBULA NEURONITIS

D)

29. A PATIENT HAS CALCIUM OXALATE RENAL STONES. WHAT IS THE MOST
APPROPRIATE MANAGEMANT?

A) ALLOPURINOL

B) ALKALINIZE THE URINE

C) STOP TAKING FOOD WITH CALCIUM

D) GIVE CALCIUM

30. HEPARIN THERAPY IS MONITORED BY WHICH OF THE FOLLOWING TESTS?

A) APTT
B) INR

C) BLEEDING TIME

D) SERUM FIBRINOGEN

31. A MIDDLE-AGED MAN IS BROUGHT TO YOUR SURGERY WITH A HISTORY OF BEING


BITTEN BY A BLACK SNAKE. ON EXAMINATION THERE ARE SCRATCH MARKS ON THE
LOWER LEG AND NO OTHER SYMPTOMS. WHAT IS YOUR MANAGEMENT?

A) REASSURE AND SEND HIM HOME

B) EXCISE WOUND

C) ADMINISTER ANTIVENIN IMMEDIATELY

D) APPLY TOURNIQUET

E) OBSERVATION AND ANTIVENOUM IF NECESSARY

32. OPENING SNAP INDICATES:

A) MITRAL VALVE MOBILITY

B) ATRIAL FIBRILLATION CAUSES DISAPPEARANCE OF THE OPENING SNAP

C) REPLACES S3

D) BEST HEARD AT 2ND RIGHT INTERCOSTALS SPACE

E) REMAINS UNALTERED DESPITE PROGRESSION OF THE DISEASE


33. A MAN PRESENTS WITH A HISTORY OF TREATMENT WITH SULPHASALAZINE. THE
LEAST LIKELY COMPLICATION FOR THE CONDITION HE WAS TREATED FOR WOULD BE?**

A) PERICHOLANGITIS

B) IRITIS

C) ERYTHEMA NODOSUM

D) PERIPHERAL NEUROPATHY

E) ANKYLOSING SPONDYLITIS

34. IN CONTACT DERMATITIS, WHICH IS COMMON?

A) CONTACT WITH PLANTS

B) CAN BE TREATED BY TOPICAL STEROIDS

C) CAN BE TREATED BY ORAL STEROIDS

35. ALL OF THE FOLLOWING ARE PRESENT IN INTERSTITIAL FIBROSING ALVEOLITIS


EXCEPT?

A) CLUBBING IS COMMON

B) NORMAL FEV1/FVC DOES NOT EXCLUDE THE DIAGNOSIS

C) DIFFUSE EXPIRATORY CRACKLES

D) DECREASED VC
36. A 72-YEAR OLD MAN WITH A HISTORY OF NON-SMALL CELL LUNG CANCER NOW
COMPLAINS OF LETHARGY, CONSTIPATION AND THIRST FOR THE PAST FEW WEEKS. WHAT
IS THE MOST LIKELY PROBLEM?

A) SIADH

B) ECTOPIC PARATHORMONE PRODUCTION FROM THE CANCER ITSELF

C) METASTASIS TO THE BRAIN

D) .

E)

37. A 64-YEAR-OLD MALE COMPLAINTS OF PAIN BEHIND THE RIGHT EYE, ASSOCIATED
WITH LACRIMATION, WHICH APPEARS EVERY MORNING & DISAPPEARS IN THE
AFTERNOON, FOR THE PAST 2 WEEKS. HAD SIMILAR EPISODES PREVIOUSLY. WHICH IS THE
PROPHYLACTIC TREATMENT?**

A) METHYSERGIDE

B) ERGOTAMINE

C) ACETOZOLAMIDE

D) NSAID

E) SUMATRIPTIN

38. A PATIENT HAS DIARRHOEA, FATIGUE AND PALLOR. BLOOD PICTURE SHOWS MCV
110. WHAT IS THE DIAGNOSIS? (DAV €“ 642)
A) PERNICIOUS ANAEMIA

B) CROHN€™S DISEASE

C) GLUTEN-SENSITIVE ENTEROPATHY

D) TROPICAL SPRUE

39. A 60-YEAR-OLD MAN, WHO SMOKES 60 PACK PER YEAR, PRESENTED WITH
SHORTNESS OF BREATH, WAS GIVEN 28% OXYGEN BY MASK. 30 MINUTES LATER, HIS ABG
WAS PAO2 68, PACO2 60. WHAT WOULD YOU DO NEXT?

A) STOP OXYGEN AND CHECK ABG AFTER 30 MINUTES

B) CONTINUE OXYGEN UNTIL NORMAL PAO2 IS EACHED

C) IMMEDIATE INTUBATION & VENTILATION

D) GIVE IMMEDIATE ANTIBIOTIC COVER AGAINST GRAM NEGATIVE BACTERIA

E) START AMINOTRYPTILLIN IV INFUSION

40. WHICH OF THE FOLLOWING CONDITIONS IS MOST LIKELYTO BECOME CHRONIC?

A) HAV

B) HBV

C) HEPATITIS C VIRUS

D) EBV
E) CHOLECYSTITIS

41. A YOUNG MAN PRESENTS WITH PLEURITIC CHEST PAIN AND COUGH. ON
EXAMINATION THERE IS DULLNESS ON PERCUSSION AND BROCHIAL BREATHING OVER
THE RIGHT LOWER ZONE, POSTERIORLY. WHICH OF THE FOLLOWING IS THE MOST LIKELY
DIAGNOSIS?

A) RIGHT LOWER LOBE CONSOLIDATION

B) RIGHT LOWER LOBE COLLAPSE

C) PLEURAL EFFUSION

D) ASTHMA

E) TB CAVITY

42. ERYTHEMA NODOSUM OCCURS IN ALL EXCEPT?

A) STEPTOCOCCUS INFECTION

B) RHEUMETIC FEVER

C) TB

D) LEPROSY

E) SLE
43. A YOUNG MAN, WHO HAD TAKEN SOME INTRAVENOUS DRUG, WAS FOUND
UNCONSCIOUS, WITH 3 BREATHS PER MINUTE AND CONSTRICTED PUPILS. WHICH OF THE
FOLLOWING WOULD BE THE MOST APPROPRIATE TREATMENT?

A) NALOXONE

B) ATROPINE

C) SCOPALAMINE

D) METHADONE

E) NALTRAXONE

44. AMYLOID CAN OCCUR IN EACH OF THE FOLLOWING EXCEPT?

A) RHEUMATOID ARTHRITIS

B) TB

C) LEPROSY

D) CHRONIC ACTIVE HEPATITIS

E) MULTIPLE MYELOMA

45. A 68-YEARS OLD FEMALE PRESENTS WITH A 2-WEEK HISTORY OF UNILATERAL


HEADACHE AND AN ESR 80 MM/HR. THE MOST APPROPRIATE TREATMENT IS?

A) NSAID€™S
B) ASPIRIN

C) CARBEMAZEPINE

D) PREDNISOLONE

E) ERGOTAMINE

46. A FEMALE AGED 23 YEARS, PRESENTED WITH PROTEINURIA. THERE IS A PREVIOUS


HISTORY OF ENURESIS TILL 13 YEARS OF AGE. HER SISTER ALSO HAD ENURESIS AS A CHILD.
THE HISTORY IS SUGGESTIVE OF WHICH OF THE FOLLOWING?

A) CHRONIC GLOMERULONEPHRITIS

B) REFLUX NEPHROPATHY

C) CONGENITAL NEPHRITIS

D) POLYCYSTIC KIDNEY DISEASE

E) ANALGESIC NEPHROPATHY

47. A PATIENT, WHO HAD BEEN TAKING A PARTICULAR DRUG, NOW PRESENTS WITH
HAEMATURIA. THE DRUG IS MOST LIKELY?

A) CALCIUM CHANNEL BLOCKER

B) DIGOXIN

C) NAPROXEN-NSAID

D) .

E)
48. A PERSON PRESENTS WITH ALPHA ANTITRYPSINOGEN DEFICIENCY. WHAT IS THE
MOST LIKELY ASSOCIATION?**

A) ONE PARENT HAS DIED OF EMPHYSEMA

B) LIMITED TO BRONCHIOLES

C) UNINHIBITED PROTEASE ACTION

D) .

E)

49. A 65-YEAR-OLD MAN PRESENTED WITH CONFUSION. LABORATORY FINDINGS


SHOWED HYPERCALCAEMIA, CALCIURIA, AND PHOSPHATES WITHIN NORMAL LIMITS.
WHICH IS THE MOST LIKELY DIAGNOSIS?

A) MALIGNANCY(OSTEOLYTIC METASTASIS)

B) PRIMARY HYPERPARATHYROIDISM

C) IDIOPATHIC HYPERCALCAEMIA/HYPERCALCIURIA

D) PAGETS DISEASE MYELOMA

50. A 28-YEAR OLD MALE PRESENTS WITH DISTAL WEAKNESS AND ATROPHY OF THE
SMALL MUSCLES OF

BOTH HANDS(INTEROSSEI, LUMBRICALS, THENAR & HYPOTHENAR). WHAT IS THE MOST


LIKELY DIAGNOSIS?
A) MULTIPLE SCLEROSIS

B) BILATERAL MEDIAN NERVE PALSY

C) SYRINGOMYELIA

D) BILATERAL ULNAR NERVE PALSY

E) BRAINSTEM INFARCTION

51. A PERSON WITH SEVERE AORTIC STENOSIS HAS A SYSTOLIC GRADIENT OF 55


MMHG. WHAT OTHER FEATURE REQUIRED SUGGESTING THE SEVERITY?

A) LV EJECTION FRACTION

B) LV END DIAGNOSIS PRESSURE

C) CARDIAC OUTPUT

D) LV HYPERTROPHY

E) LV END SYSTOLIC PRESSURE

52. AN ELDERLY MAN PRESENTS TO THE EMERGENCY DEPT WITH CHEST PAIN. NO ECG
CHANGES AND CARDIAC ENZYMES ARE NORMAL. WHAT IS TRUE?

A) IF ECG IS NORMAL HE DOES NOT HAVE A CARDIAC PROBLEM

B) HE SHOULD HAVE AN IMMEDIATE STRESS TEST DONE

C) INTRAVENOUS NITRATES & HEPARIN ARE NOT INDICATED

D) NO FOLLOW UP ECG IS REQUIRED


E) STREPTOKINASE IS INDICATED IF THERE ARE ECG CHANGES

53. THE MOST COMMON CAUSE OF TRAVELLER€™S DIARRHOEA IS?

A) SALMONELLA TYPHI

B) SHIGELLA

C) ENTEROTOXIC E. COLI

D) STAPHYLOCOCCUS

E) GIARDIA LAMBLIA

54. TASTE SENSATION OF THE ANTERIOR 2/3 OF THE TONGUE IS CARRIED BY WHICH
NERVE?

A) V

B) FACIAL NERVE(VII)

C) IX

D) X

E) XII
55. WHICH OF THE FOLLOWING IS LEAST LIKELY TO CAUSE FACIAL NERVE PALSY?

A) SKULL FRACTURE

B) MASTOIDITIS

C) CHRONIC PAROTITIS

D) PAROTID TUMOUR

E) ACOUSTIC NEUROMA

56. A PATIENT HAS MASSIVE ASCITIS AND IN THE FLUID MALIGNANT CELLS ARE FOUND.
WHICH OF THE FOLLOWING IS A POSSIBLE FINDING?

A) SPLENOMEGALY

B) HEPATOMEGALY

C) SUPRACLAVICULAR SWELLING

D) ON DIGITAL RECTAL EXAMINATION, NODULES CAN BE PALPATED

57. THE SINGLE MOST RELIABLE TEST FOR HAEMOCHROMATOSIS IS?

A) RED CELL MASS

B) SERUM IRON

C) SERUM FERRITIN

D) SERUM TRANSFERRIN

E) TRANSFERRIN SATURATION
58. IN WHICH OF THE FOLLOWING, IS ADMINISTRATION OF IMMUNOGLOBIN AS A
PROPHYLAXIS NOT USEFUL?

A) HEPATITIS A

B) HEPATITIS B

C) RUBELLA

D) MUMPS

E) VARICELLA

59. IN VON WILLEBRANDT€™S DISEASE YOU WILL FIND:

A) INCREASED PLATELET COUNT

B) INCREASE BLEEDING TIME ( BLEEDING TIME IS PROLONGED, PLATELETS SHOW


REDUCED ADHESION AND LEVELS OF FACTOR VIII ARE LOW )

C) NORMAL APTT

D) .

E)
60. 12 YEARS OLD BOY CAME WITH SEVERE DYSPNOEA AND AFIBRILE CONDITION. O/E
YOU FOUND R/R- 40/MIN, PULSE- 130/MIN. THERE IS O2 SATURATION AT ROOM AIR OF
85%. WOF WILL BE APPROPRIATE--

A) GIVE SALBUTAMOL $ HOURLY AT HOME

B) GIVE SALBUTAMOL AND AMINOPHYLLIN 4 HOURLY AT HOME

C) GIVE HIM O2 AND SEND HIM TO HOSPITAL BY AN AMBULANCE.

61. ALL THE FOLLOWING DRUGS ARE PROVEN TO DECREASE THE MARALITY IN
MYOCARDIAL INFECTION, EXCEPT?

A) ASPIRIN

B) ACE INHIBITORS

C) BETA BLOCKERS

D) NIFEDIPINE

E) STREPTOKINASE

62. A PATIENT WITH COPD CAME IN THE EMERGENCY DEPARTMENT WITH SEVERE
DYSPONEA, 28% O2, 8 LITER WAS GIVEN BY MASK. AFTER ONE HOUR YOU DID ARTERIAL
BLOOD GAS & YOU FOUND THAT PAO2 LEVEL WAS 42 MM HG, AND PA CO2 LEVEL WAS
68 MM HG. WOF YOU WILL DO NOW?
A) INCREASE O2 INHALATION BY MASK

B) INCREASE I/V FLUID

C) REPEAT ARTERIAL BLOOD GAS LEVEL

D) ASK THE PATIENT FOR INTUBATION AND VENTILATION

63. A 50 YRS OLD LADY ADMITTED WITH ATRIAL FIBRILLATION.BLOOD TESTS SHOW :

A) TSH : DECREASE (NOTE:DX€”THYROTOXICOSIS

B) T3: INCREASE IF ALL DECREASE, DX PITUITARY DISEASE)

C) T4: INCREASE

YOUR MANAGEMENT WILL BE€”

A) TOTAL THYROIDECTOMY

B) SUBTOTAL THYROIDECTOMY

C) RADIOACTIVE IODINE

D) NEOMERCAZOLE(CARBIMAZOLE)

E) ECHO

64. AN OFFICE CLERK WITH HIGH BLOOD PRESSURE. HE IS OBESE AND HE TAKES 3-4
CUPS OF COFFEE EVERYDAY. WOF IS NOT CORRECT---

A) CONSUMPTION OF COFFEE CONTRIBUTES GREATLY TO HIS BP

B) REDUCING BODY WEIGHT WILL HELP TO REDUCE HIS BP


C) IN 90-95% CASES NO CAUSE IS FOUND FOR BP.

D) HE SHOULD BE ADVISED ABOUT SAFE LEVEL OF DRINKING ALCOHOL.

65. A MALE PRESENTS WITH A SUDDEN ONSET OF VERTIGO AND NAUSEA. DURING THE
ATTACK HE HAS DEAFNESS. WOF IS YOUR DIAGNOSIS IS --

A) POSITIONAL VERTIGO

B) ACOUSTIC NEUROMA

C) LABYRINTHITIS

D) BASILAR ARTERY INFECTION

66. PHOTOGRAPH 1: SHOWS A SKIN LESION ON THE FOREARM. SLIGHTLY RAISED, RED
PLAQUE, SLIGHTLY SCALY. WHAT IS THE DIAGNOSIS?

A) LICHEN PLANUS

B) DISCOID ECZEMA

C) SQUAMOUS CELL CARCINOMA

D) PSORIASIS

E) CONTACT DERMATITS
67. PHOTOGRAPH 2: SHOWS A SKIN LESION, PINKISH BROWN COLORED LUMP. THE
LESION IN THE PICTURE WAS FOUND ON THE FOREARM OF A YOUNG MALE. 2 WEEKS
LATER HE DEVELOPS PNEUMONIA ASSOCIATED WITH CREPITATION THROUGH OUT BOTH
LUNGS.WOF ORGANISM IS CAUSING THIS LESION-

A) STAPHYLOCOCCUS PNEUMONIA

B) STAPHYLOCOCCUS AUREUS

C) MYCOPLASMA PNEUMONIA

D) PNEUMOCYSTIS CARNII

E) MYCOBACTERIUM AVIUM

68. PHOTOGRAPH 3 : X-RAY SHOWING AN OPEN FRACTURE OF THE FIBULA & TIBIA. A
YOUNG MAN, AFTER AN ACCIDENT ON HIS MOTORCYCLE, PRESENTED WITH 3 CM OF
BONE PROTRUDING THROUGH THE SKIN OF HIS LEG. WHAT IS YOUR NEXT LINE OF
MANAGEMENT?

A) INTERNAL FIXATION OF THE FRACTURE IMMEDIATELY

B) SKIN CLOSURE OF THE WOUND IMMEDIATELY TO PREVENT INFECTION

C) REDUCE THE FRACTURE AND APPLY PLASTER OF PARIS

D) GIVE TETANUS TOXOID IM & ANTIBIOTICS IV

E) DEBRIBEMENT IS ONLY INDICATED WHEN THE WOUND IS SEVERELY CONTAMINATED

69. PHOTOGRAPH 4 : A DIABETIC MAN COMES TO SEE YOU. HIS RIGHT LEG BELOW HIS
KNEE IS RED & SHINY. WHAT IS THE BEST INITIAL MANAGEMENT?
A) PENICILLIN & FLUCLOXALILLIN

B) METRONIDAZOLE

C) AMPICILLIN

D) AMPICILLIN & GENTAMICIN

E) AMPICILLIN, GENTAMICIN & METRONIDAZOLE

70. PHOTOGRAPH 5: SHOWS THE HEAD & NECK OF A FEMALE. THERE IS A LARGE
SWELLING AT THE CENTER OF THE NECK AND NO EXOPHTHALMOS OF THE EYES. WHAT IS
THE MOST LIKELY DIAGNOSIS?

A) MULTINODULAR GOITER

B) CA THYROID

C) SUB ACUTE THYROIDITIS

D) THYROID CYST

E) GRAVES DISEASE

71. PHOTOGRAPH 6: SHOWS A RED HEMISPHERICAL LESION ON THE BACK OF A HAND.


SOME BROWNISH/NECROTIC TISSUE PRESENT AT THE BORDER OF THE LESION, NO VISIBLE
PUNCTUM. WHAT IS THE DIAGNOSIS?

A) MALIGNANT SKIN LESION

B) INFLAMED KERATOACANTHOMA

C) INFECTED LIPOMA

D) LICHEN PLANUS

E) SOLAR KERATOSIS

72. PHOTOGRAPH 7: X-RAY SHOWING COLLES FRACTURE.


AN ELDERLY PERSON SUSTAINED A FALL ON AN OUTSTRETCHED HAND. WHAT IS TRUE?

A) MEDIAN NERVE FUNCTION SHOULD BE ASSESSED

B) THE PLASTER CAST IS NOT SUITABLE TO CORRECT THE FRACTURE IN THE ELDERLY

C) IMMEDIATE INTERNAL FIXATION

D) PLASTER CAST FROM BELOW ELBOW TO MCP JOINT, FOR AT LEAST 10 WEEKS

73. PHOTOGRAPH 8 : SHOWING A DEFORMED HAND(SWAN NECK DEFORMITY).

WHICH OF THE FOLLOWING IS TRUE?

A) IT IS DUE TO EROSION & RUPTURE OF THE FLEXOR TENDONS

B)IT IS MORE COMMON IN RHEUMATOID ARTHRITIS THAN OSEOARTHRITIS

C) IT IS DUE TO ULNER NERVE COMPRESSION

74. PHOTOGRAPH 9 : A YOUNG CHILD WITH A PATCH OF HAIR LOSS ON THE SCALP.
SOME SMALL FLAKES OF SKIN ARE PRESENT. WHAT IS YOUR DIAGNOSIS?

A) TINEA

B) PSORIASIS

C) ALOPECIA AREATA

D) TRICHOTILLOMANIA

E) SLE
75. PHOTOGRAPH 10 : A MAN CAME WITH HIS WIFE. A SLOW GROWING SOFT LARGE
LUMP ON HIS BACK IN BETWEEN THE SCAPULAE REMAINS IN THAT POSITION FOR LAST 10
YEARS. WOF IS YOUR MX ----

A) OBSERVATION AND REASSURANCE

B) OPINION FROM SPECIALIST

C) RADIOTHERAPY

*********

1. A MAN IS SUFFERING FROM BACK PAIN, SO HE IS SLEEPING ON A HARD WOODEN


BED FOR LAST THREE WEEKS. NOW HE COMPLAINS OF PAIN IN HIS RIGHT ELBOW. WOF IS
THE CAUSE OF THE PAIN IN ELBOW--

A) OLECRANON BURSITIS

B) INJURY TO THE INSERTION OF FLEXOR MUSCLES

C) INJURY TO THE INSERTION OF EXTENSOR MUSCLES

2. DURING AN OPERATION SPLEEN WAS ACCIDENTALLY INJURED AND SPLEENECTOMY


WAS DONE TO MINIMIZE THE COMPLICATIONS. NEXT MORNING THE PATIENT ASKED YOU
ABOUT THE SUCCESS & PROGRESS OF THAT OPERATION. AS AN INTERN YOU WILL TELL
HIM----

A) EVERY THING HAS GONE SMOOTHLY

B) YOUR SPLEEN HAS BEEN REMOVED


C) DURING THE OPERATION PROCEDURE COMPLICATION ARISED AND TO MINIMIZE
FURTHER COMPLICATION IT WAS NECESSARY TO REMOVED YOUR SPLEEN.

D) ONLY THE SURGEON COULD TELL YOU ABOUT YOUR OPERATION.

3. A YOUNG MAN INJURED IN A ROAD TRAFFIC ACCIDENT (RTA) HAS FRACTURES OF


THREE RIBS ANTERIORLY ON THE LEFT AND A MINIMAL LEFT PNEUMOTHORAX.
PERITONEAL LAVAGE IS POSITIVE FOR BLOOD, AND LAPAROTOMY IS PLANNED FOR
INTRAPERITOEA HEMORRHAGE. THE MOST IMPORTANT STEP PRIOR TO LAPAROTOMY IS -
---

A) INSERTION OF A NASOGASTRIC TUBE.

B) INSERTION OF INTERCOSTALS DRAINAGE TUBE

C) DETERMINATION OF PAO2 AND PACO2

D) INSERTION OF A CENTRAL VENOUS PRESSURE LINE ON THE RIGHT SIDE.

E) DELAY SURGERY UNTIL BP COMES TO NORMAL

4. AFTER AN OPERATION FOR AORTIC ANEURYSM/AORTOGRAPHY, THERE IS


UNCONTROLLABLE BLEEDING FROM THE SITE OF THE GRAFTING. WOF IS BEST TO
TRANSFUSE----

A) FRESH BLOOD

B) DESMOPRESSIN

C) FRESH FROZEN PLASMA

D) CRYOSIPITATE

E) FACTOR VIII
5. A MAN CAME TO YOU COMPLAINING OF PAIN IN THE LEG. O/E YOU COULD NOT FIND
PULSE IN HIS LEG. YOU CLINICALLY BECOME SURE THAT THERE IS ATHEROSCLERASIS OR
ATHEROMA IN THE PERIPHERAL ARTERY. THIS IS DUE TO ----

A) STENOSIS OF FEMORAL ARTERY

B) THROMBOSIS OF FEMORAL ARTERY

C) POLYARTERITIS NODOSA(PAN)

D) RUPTURE ANEURYSM

6. A 42-YEARS OLD MAN COMPLAINTS OF PAIN IN HIS RIGHT HAND. HE TELLS YOU THAT
THE PREVIOUS DAY HE USED A CHAINSAW FOR LONG HOURS TO CUT TREES. O/E YOU
FOUND HIS RIGHT ARM IS SLIGHTLY SWOLLEN THAN OTHER ARM, AND THERE IS PAIN
TOO. WOF IS MOST APPROPRIATE INVESTIGATION--

A) VENOGRAM

B) LYMPHANOGRAM (DX: RAYNAUD€™S PHENOMENON; DUE TO USE OF VIBRATORY


TOOLS)

C) CT SCAN

D) MRI

E) ARTERIOGRAM

7. REGARDING UNILATERAL UNDESCENDED TESTIS, WHICH OF THE FOLLOWING IS


MOST COMMONLY ASSOCIATED?
A) MALIGNANCY

B) VARICOCELE

C) INGIUNAL HERNIA(INDIRECT)

D) HYDROCELE

E) TORSION OF THE TESTIS

F) DIRECT INGUNAL HERNIA

8. A 64-YEARS OLD MAN PRESENTS WITH PROFUSE BRIGHT RED BLEEDING PER
RECTUM. WHAT IS THE MOST LIKE CAUSE?

A) CA COLON

B) POLYP

C) DIVERTICULOSIS

D) DUODENAL ULCER

E) HAEMORRHOIDS

9. A 50-YEAR OLD MAN PRESENTS WITH SUDDEN ONSET OF SEVERE ABDOMINAL PAIN.
WHICH OF THE FOLLOWING IS LEAST LIKELY TO BE THE CAUSE?

A) ACUTE APPENDICITIS

B) PERFORATED PEPTIC ULCER

C) ACUTE PANCREATITIS
D) RUPTURED ABDOMINAL AORTIC AORTIC ANEURYSM

E) RENAL COLIC

10. WHICH OF THE FOLLOWING DISSEMINATED MALIGNANCIES HAS THE WORST


PROGNOSIS?

A) METASTATIC BEAST CANCER

B) METASTATIC CHORIOCARCINOMA

C) METASTATIC SEMINOMA

D) HODGKIN€™S LYMPHOMA

E) NON-HODGKIN€™S LYMPHOMA

11. WHICH OF THE FOLLOWING IS THE MOST COMMON CAUSE OF BLOODY DISCHARGE
FROM THE NIPPLE?

A) INTRADUCTAL CARCINOMA

B) INTRADUCTAL PAPILLOMA

C) PAGET€™S DISEASE OF THE BRAEST

D) FIBROADENOMA
E) DUCT ECTASIA

12. HUTCHINSON€™S MELANOTIC FRECKLE. WHICH OF THE FOLLOWING IS NOT


CORRECT?

A) MOSTLY OCCURS IN THE ELDERLY

B) MAINLY OCCURS IN COVERED PARTS OF THE BODY

C) MALIGNANT

D) IRREGULAR BORDER

E) IRREGULAR COLOUR (FROM BLACK TO BROWN)

13. A DIABETIC PATIENT WITH AN ULCER AT THE HEAD OF THE 2ND METATARSAL ON
THE SOLE OF THE FOOT. WHICH OF THE FOLLOWING WOULD BE THE MOST LIKELY CAUSE?

A) MACROVASCLAR DISEASE

B) DIABETIC NEUROPATHY

C) INFECTION

D) VARICOSE VENIS

E) HYPERGLYCAEMIA

14. A 60-YEAR OLD MAN PRESENTS WITH 3 DAYS VOMITING AND LOSS OF WEIGHT (3
KG.). SIX YEARS AGO HE WAS TREATED WITH CIMETIDINE. OVER THE LAST 3 MONTHS HE
HAS HAD EPIGASTRIC PAIN INTERMITTENTLY, FOR WHICH HE TAKES ASPIRIN. VOMITUS IS
CLEAR IN COLOUR WITH IDENTIFIABLE FOOD PARTICLES. THE LIKELY DIAGNOSIS IS?

A) DRUG INDUCED GASTRITIS

B) CHRONIC DUODENAL ULCER DISEASE

C) CA PANCREAS

D) CANCER OF THE CARDIA OF THE STOMACH

E) PYLORIC STENOSIS

15. THE MOST COMMON CAUSE OF DIARRHOEA IN BEDRIDDEN ELDERLY PATIENTS IS?

A) CA COLON

B) CROHN€™S DISEASE

C) ULSERATIVE COLITIS

D) DIVERTICULOSIS

E) FAECAL IMPACTION

16. A FEMALE HAD MASTALGIA FOR THE LAST 1 YEAR. CONSERVATIVE TREATMENT HAS
NOT WORKED. HOW WOULD YOU MANAGE THIS PATIENT?
A) BROMOCRIPTINE

B) DANAZOLE

C) CLOMIPHENE

D) OCP

E) NSAID€™S

17. WITH FAT EMBOLISM, PETECHIAE ARE MOST TO BE SEEN WHERE?

A) ALL OVER THE BODY

B) PRESSURE DEPENDENT AREAS

C) SUBUNGUAL

D) UPPER PART OF THE BODY

E) LOWER BODY

18. A 65-YEAR OLD WOMAN DEVELOPS ACUTE ABDOMINAL PAIN WITH DISTENSION.
THERE IS A HISTORY OF 12 HOURS OF VOMITING AND CRAMPING PAIN. ON
EXAMINATION, IRREGULAR PULSE 120/MIN, BP-100/60 MMHG, ABDOMEN IS TENDER
WITH GUARDING AND BOWEL SOUNDS ARE ABSENT. RECTAL EXAMINATION REVEALS A
DARK BLOODSTONE ON THE FINGER. WHICH OF THE FOLLOWING IS MOST LIKELY?

A) PERFORATED PEPTIC ULCER

B) MESSENTERIC ARTERIAL OCCLUSION

C) DIVERTICULOSIS
D) PERFORATED APPENDICITIS

E) CA COLON

19. A 21-YEAR OLD MAN PRESENTS WITH INCREASING DYSPONEA, TACHYPNOEA A


HYPERINFLATED LEFT HEMITHORAX WITH EVIDENCE OF A MEDIASTINAL SHIFT. YOUR
IMMEDIATE MANAGEMENT WOULD BE?

A) INSERT A WATER SEALED CHEST DRAIN

B) INTUBATE

C) INSERT A VENFLON INTO 4TH INTERCOSTALS SPACE MIDAXILLARY LINE

D) INSERT A WIDE BORE NEEDLE INTO THE 2ND INTERCOSTALS SPACE, MIDCLAVICULAR
LINE

20. WHICH OF THE FOLLOWING IS MOST LIKELY TO BE ASSOCIATED WITH


OTOSCLEROSIS?

A) NORMAL TYMPANIC MEMBRANE

B) RED & INFLAMED TYMPANIC MEMBRANE

C) TENSE & TRANSPARENT TYMPANIC MEMBRANE WITH FLUID LEVEL BEHIND


D) BLUE GRAY SCLERA

E) OBSTRUCTION OF THE EUSTACHIAN TUBE

21. WHICH OF THE FOLLOWING IS TRUE REGARDING CARCINOMA OF THE


NASOPHARYNX?

A) ARISES FROM THE LYMPHATICS

B) NOT SENSITIVE TO RADIOTHERAPY

C) RESPONDS WELL TO CHEMOTHERAPY

D) CAUSED BY EBV

E) LOCALLY DESTRUCTIVE LESION

22. ALL ARE SEEN IN ISCHAEMIA OF THE LEG, EXCEPT?

A) CLAUDICATION

B) PALLOR

C) PAIN ON WALKING

D) SHOOTING PAIN DOWN THE POSTERIOR ASPECT OF THE THIGH

23. WHICH OF THE FOLLOWING IS LEAST LIKELY TO CAUSE METASTASIS TO THE BRAIN?

A) PROSTATE CANCER
B) MELANOMA

C) SMALL CELL CARCINOMA OF THE LUNG

D) BREAST CANCER

E) RENAL CARCINOMA

24. WHICH OF THE FOLLOWING IS NOT A FEATURE OF UNCOMPLICATED


HAEMORRHOIDS?

A) BRIGHT RED BLEEDING

B) PROLAPSE OF MUCOSA

C) PURITIES

D) PAIN

E) MUCOUS

25.WHICH OF THE FOLLOWING NERVES WOULD BE DAMAGED IN A MID-HUMERUS


FRACTURE?

A) AXILLARY NERVE

B) RADIAL NERVE

C) MUSCULOCUTANEOUS NERVE

D) ULNER NERVE

E) MEDIAN NERVE
26. A FEMALE COMPLAINTS OF BLOATING & BELCHING. SHE IS FOUND TO HAVE
GALLSTONES, WHICH IS SUBSEQUENTLY TREATED BY CHOLECYSTECTOMY.
CHOLANGIOGRAM WAS DONE DURING THE OPERATION AND WAS CLEAR (NOTHING WAS
FOUND). SHE WAS SYMPTOM FREE FOR 1 WEEK, BUT THEN THE SAME SYMPTOMS
RETURNED. WHAT IS THE MOST LIKELY?

A) PANCREATITIES

B) CYSTIC DUCT STUMP SYNDROME

C) STONE LEFT IN THE COMMON BILE DUCT

D) SYMPTOMS ARE NOT RELATED TO THE GALLSTONES

E) POST CHOLECYSTECTOMY SYNDROME

27. ALL ARE TRUE ABOUT GALLSTONES, EXCEPT?

A) USUALLY ASYMPTOMATIC

B) STONE IN THE BILE DUCT PRESENTS WITH CLINICAL SEPSIS

C) USUSALLY RADIOLUCENT

D)

28. WHICH OF THE FOLLOWING IS TRUE REGARDING MAMMOGRAPHY?

A) MORE SENSITIVE IN PREMENOPOSAL THAN POSTMANUPOSAL WOMEN

B) PAINLESS
C) MOST USEFUL IN WOMAN WITH A PALPABLE BREAST LUMP

D) IT HAS A BETTER PICK UP RATE THAN SELF EXAMINATION

E) CAN DIFFERENTIATE BETWEEN A SOLID & CYSTIC TUMOUR

29. ALL OF THE FOLLOWING WOULD BE EXPECTED AFTER SPLENECTOMY FOR


SPHEROCYTOSIS, EXCEPT?

A) TRANSIENT LEUCOCYTIS

B) PERSISTENCE OF ANAEMIA

C) SAME OSMOTIC GRAGILITY

D) PERSISTENCE OF SPHEROCYTOSIS

E) NORMAL LIFE SPAN OF THE ERYTHROCYTES

30. WHAT IS THE MOST IMPORTANT ASPECT IN MANAGEMANT OF AN OPEN


FRACTURE?

A) ANTIBIOTICS

B) INTENAL FIXATION

C) DEBRIDEMENT

D) TETANUS TOXOID

E) IMMOBILISATION
31. A 35-YEARS-OLD WOMAN HAVING SEVERE SUDDEN ABDOMINAL PAIN
THROUGHOUT THE NIGHT WAKES UP AND PASSES DARK URINE IN THE BATHROOM. SHE
IMMEDIATELY GOES TO SEE THE DOCTOR. INVESTIGATIONS SHOWED:

- BILIRUBIN €“ 5 TIMES NORMAL

- ALKALINE PHOSPHATASE €“ 3 TIMES NORMAL

- AST & ALT €“ 4 TIMES NORMAL

- S.AMYLASE €“ 2 TIMES NORMAL

WHAT IS THE MOST LIKELY DIAGNOSIS?

A) ACUTE CHOLELITHIASIS

B) ACUTE CHOLECYSTITIS

C) CA GALLBLADDER

D) ACUTE CHOLEDOCHOLITHIASIS

E) ?

32. WHAT IS TRUE ABOUT FISTULAS?

A) IT IS A COMMUNICATING TRACK BETWEEN 2 EPITHELIAL SURFACES


B) IT IS LINED BY GRANULATION TISSUE

C) IT IS A BLIND TRACK LEADING FROM THE SURFACE DOWN TO THE TISSUE

D) A KIND OF ULCER

E) ?

33. WHAT IS TRUE REGARDING PAIN MANAGEMENT?

A) PETHIDINE IS BEST GIVEN BY CALCULATING MG/KG BODY WEIGHT

B) PETHDINE IS BETTER THAN MORPHINE

C) PETHIDINE SHOULD NOT BE REPEATED WITHIN 4 HOURS

D) OLDER PATIENTS REQUIRE LESS, BECAUSE THEY TOLERATE PAIN BETTER

E) ?

34. YOU ARE PERFORMING EXTERNAL CARDIAC MASSAGE ON A PATIENT WHO HAS JUST
SUFFERED A CARDIAC. WHICH OF THE FOLLOWING PROVIDES BEST INDICATION THAT
RESUSCITATION IS EFFECTIVE?

A) ECG

B) PALPATION OF THE RADIAL PULSE

C) COLOR OF SKIN & MUCOUS MEMBRANES

D) SIZE & REACTION OF THE PUPILS

E) RESPONSE TO STIMULI
35. ALL OF THE FOLLOWING ARE RISK FACTORS FOR BREAST CANCER, EXCEPT?

A) EARLY ARTIFICIAL MENOPAUSE

B) FIRST PREGNANCY AFTER 35 YEARS OF AGE

C) EARLY MENARCHE

D) NO BREAST FEEDING

E) CANCER OF THE OTHER BREAST

F) AGE - PEAK INCIDENCE 45-75 YEARS BUT ANY AGE POSTMENARCHE >> 4X

G) COUNTRY OF RESIDENCE - HIGH IN WEST > 4X E.G. UK, LOW IN EAST E.G. JAPAN

H) PREVIOUS BREAST CANCER > 4X

I) IRRADIATION OF CHEST - SHOWS A LINEAR DOSE-RESPONSE RELATIONSHIP 2-4X

J) SOCIAL CLASS (I VS. V) 2-4X

K) RACE - MORE COMMON IN CAUCASIANS < 2X

L) PREVIOUS OVARIAN OR ENDOMETRIAL CANCER < 2X

M) EARLY MENARCHE OR LATE MENOPAUSE < 2X

N) NULLIPARITY OR OLDER THAN 30 YEARS BEFORE FIRST CHILD < 2X

O) HORMONAL SUPPLEMENTATION < 2X

P) OBESITY - OESTROGEN SYNTHESIS IN ADIPOSE TISSUE

Q) ALCOHOL CONSUMPTION

IN THE MALE, KLINEFELTER'S SYNDROME IS A RISK FACTOR FOR BREAST CANCER.


36. A 32-YEARS OLD MAN FELT PAIN IN THE MAXILLARY AREA AT NIGHT, WHICH
RADIATES TO THE SIDE OF HIS HEAD. THERE WAS WATER INSIDE HIS NOSTRILS DURING
THE ATTACK. WOF IS YOUR DIAGNOSIS---

A) MIGRAINE

B) CLUSTER HEADACHE

C) MAXILLARY SINUSITIS

37. A 35-YEARS OLD FEMALE CAME WITH COMPLAINTS OF WAKE UP DURING THE
NIGHT WITH PAIN IN HER RIGHT HAND & A SHOOTING PAIN UP THE ARM. SHE HAS
DIFFICULTY USING HER RIGHT HAND WHEN CARRYING A SHOPPING BAG. EXAMINATION
SHOWS SLIGHT THENAR WASTING. WHAT IS YOUR DIAGNOSIS?

A) BRACHIAL NEURITIS

B) THORACIC OUTLET OBSTRUCTION

C) CERVICAL SPONDYLOSIS

D) CARPAL TUNNEL SYNDROME

E) CERVICAL RIB

38. A MAN PRESENTS WITH HISTORY OF AN INGUINO-SCROTAL SWELLING, WHICH


DISAPPEARS ON LYING DOWN. WHAT IS THE MOST PROBABLE DIAGNOSIS?

A) HYDROCELE

B) SAPHENA VARIX
C) FEMORAL HERNIA

D) VARICOCELE

E) DIRECT INGUINAL HERNIA

39. A 25 YEARS OLD MAN IS AWAKENED DURING NIGHT VERY SEVERE PAIN IN HIS
RIGHT LOIN. HE NOTICES BRIGHT BLOOD IN HIS URINE. NEXT MORNING HE COMES TO
YOU- YOU FOUND HIM A FEBRILE AND PLAIN X RAY SHOWS CALCIUM OXALATE STONE IN
HIS BLADDER. WOF YOU WILL ADVICE --

A) NOT TO TAKE CALCIUM CONTAINING FOOD

B) AVOID RED MEAT

C) AVOID ALL KIND OF DRINKS, TEA, COFFEE

40. MOST COMMON CAUSE OF DEATH IN SEVERE BURNS, IN AUSTRALIA IS?

A) IRREVERSIBLE SHOCK

B) SEPTICAEMIA

C) RESPIRATORY FAILURE DUE TO RESPIRATORY TRACT BURNS

D) UNABLE TO MAINTAIN POSITIVE NITROGEN BALANCE

41.ALL OF THE FOLLOWING CAUSE ACUTE PANCREATITIS,EXCEPT--

A) ALCOHOL ( NOTE: DR. SUSAN€”GET SMASHED)

B) MUMPS VIRUS INFECTION 1. GALL STONES


C) DIABETES MELLITUS 2.ETHANOL

D) GALL STONE 3.TRAUMA

E) PANCREATIC ISLET CELL TUMOUR 4.STEROIDS 5.MUMPS

7.AUTOIMMUNE 8.SCORPION VENOM

9.HYPERLIPIDAEMIA 10.ERCP 11.DM/DRUGS: AZATHIOPRINE,PENTAMDINE

42. WHEN IS THE BEST TIME TO ADMINISTER PROPHYLACTIC ANTIBIOTICS?

A) 1 DAY BEFORE OPERATION

B) 1 HOUR BEFORE OPERATION

C) AT THE TIME OF INCISION

D) AFTER THE INFECTION

43. A FEMALE PT. COMPLAINTS OF FULLNESS FAT INDIGESTION AFTER FATLY MEALS. IN
AN ULTRASOUND STONES SEEN IN GALLBLADDER, BUT THERE IS NO SEPSIS. AFTER
SURGERY NO STONE LEFT IN THE GALL BLADDER AND FREE OF SYMPTOMS FOR FEW
WEEKS, THEN SYMPTOMS RECUR. WHAT IS THE EXPLANATION--

A) PANCREATITIS AFTER CHOLECYSTITIS

B) STONE WAS LEFT IN CBD

C) IT IS A RESULT OF REMOVAL OF GALL BLADDER-FAT INDIGESTION

D) GALL STONES WERE NOT HER SYMPTOMS

E) CYSTIC DUCT STUMP SYNDROME

F) RECURRENCE OF STONE FORMATION AFTER CHOLECYCTECTOMY


44. A MIDDLE AGED FEMALE COMPLAINTS OF MORNING DIARRHOEA AND CRAMPS. ON
EXAMINATION THERE ARE NO REMARKABLE FINDINGS. WHICH IS THE MOST LIKELY
DIAGNOSIS?

A) IRRITABLE BOWEL DISEASE

B) CA COLON

C) CROHNS DISEASE

D) ULCERATIVE COLITIS

45.A PATIENT CAME WITH WEAKNESS IN EXTENSION OF HAND AND PRONATION.ON


EXAMINATION THERE IS NO WASTING OF HAND MUSCLES.FLEXION NORMAL, BICEPS &
TRICEPS REFLEXES ALSO NORMAL.NO BRACHIORADIALIS JERK.WHERE IS THE LESION?

A) MEDIAN NERVE

B) RADIAL NERVE

C) ULNER NERVE

D) NERVE TO POSTERIOR INTEROSEUS

E) NERVE TO ANTERIOR INTEROSEUS

1. A COUPLE CAME TO YOUR GP SURGERY TO KNOW ABOUT THE MODE OF


INHERITANCE/NUMBER OF INCIDENCE OF CYSTIC FIBROSIS IN THE POPULATION. YOUR
ANSWER WILL BE €“
A) 1 IN 2

B) 1 IN 4- MODE OF INHERITANCE

C) 1 IN 25

D) 1 IN 100

E) 1 IN 2500- NUMBER OF INCIDENCE

2. A 15 MONTHS OLD CHILD WAS FOUND WITH AMBIGUOUS GENITALIA, WHILE IN


CEROTYPE ANALYSIS 46XY WAS DIAGNOSED. WOF IS THE NEXT APPROPRIATE IX ?

A) X- RAY ( KUMAR & C-1024/1054 )

B) 17 HYDROXYPROGESTERONE---INCREASED IN CONGENITAL ADRENAL HYPERPLASIA

C) CT SCAN

D) US OF PELVIC REGION

E) TESTOSTERONE LEVEL

3. AN 8-YEAR-OLD BOY PRESENTS WITH PHOTOPHOBIA, IRRITABILITY & NECK


STIFFNESS. LUMBAR PUNCTURE FINDINGS SHOW 50 NEUTROPHILS, 100 LYMPHOCYTES,
PROTEINS NORMAL, GLUCOSE NORMAL. WHICH IS THE MOST LIKELY DIAGNOSIS?

A) TUBERCULOSIS MENINGITIS

B) H. INFLUENZA

C) ECHO VIRUS

D) E. COLI
E) HSV ENCEPHALITIS

4. A 5 MONTH OLD BABY UNWELL FOR A WEEK IS NOTICED BY HIS PARENTS TO HAVE
EPISODES OF LEANING FORWARD AND SHAKING HIS ARMS. HIS PARENTS ARE
CONCERNED, BECAUSE HE IS NOT RESPONDING AS HE USED TO. WHAT IS THE LIKELY
CAUSE?

A) INFANTILE SPASM

B) FEBRILE SEIZURES

C) BREATH HOLDING SPELLS

D) INFANTILE MYOCLONIC SEIZURES

E) PETIT MAL EPILEPSY

5. A 2 MONTH OLD CHILD HAS STRAWBERRY NAEVUS ON ITS BACK BETWEEN THE
SCAPULAE, 2 CM IN SIZE. WHAT IS THE MOST APPROPRIATE MANAGEMENT?

A) CRYOTHERAPY

B) LASE THERAPY

C) OBSERVATION

D) INJECTION OF A SCLEROSING AGENT

E) SIMPLE EXCISION
6. A CHILD WITH PERORBITAL OEDEMA AND ANARARCA, BP IS NORMAL. ALL OF THE
FOLLOWING ARE TRUE, EXCEPT?

A) STEROIDS ARE USEFUL IN TREATMENT

B) FOCAL GLOMERULONEPHRITS IS THE COMMONEST CAUSE

C) PROTEINURIA MORE THAN 3 GRAMS/DAY IS USUAL

D) THIS IS COMMONEST BETWEEN 1 AND 4 YEARS OF AGE

E) ?

7. A FULLY BREAST FEED BABY, FROM A MOTHER ON A HEALTHY, BALANCED DIET, CAN
HAVE DEFICIENCY OF WHICH VITAMIN?

A) VITAMIN A

B) VITAMIN B COMPLEX

C) VITAMIN C

D) VITAMIN D

E) VITAMIN K

8. YOU CAN SEE AN INFANT WITH SUBDURAL HAEMATOMA, BRUISING AND MULTIPLE
SUBPERIOSTEAL SCLEROSING ZONES ON X-RAY OF THE LIMBS. WHAT DO YOU FIRST THINK
OF?

A) RICKETS
B) VITAMIN C DEFICIENCY

C) PAGETS DISEASE

D) NON ACCIDENTAL INJURY

E) HAEMOPHILIA

9. WHAT IS TRUE ABOUT PHENYLKETONURIA INHERITANCE?

A) ONLY BOYS ARE AFFECTED

B) ONLY GIRLS ARE AFFECTED

C) 1 IN 2 CHANCE OF ANY SEX BEING AFFECTED

D) 1 IN 4 CHANCE OF ANY SEX BEING AFFECTED

E) SPORADIC

10. AN 8-WEEK-OLD BABY PRESENTS WITH UNILATERAL STICKY EYE. SIMILAR EPISODE
BEFORE WAS TREATED WITH ANTIBIOTICS FOR 3 DAYS AND RESOLVED. WHAT IS THE
MOST LIKELY DIAGNOSIS?
A) GONOCOCCAL CONJUNCTIVITIS

B) ALLERGIC CONJUNCTIVITIS

C) INADEQUATE ANTIBIOTIC TREATMENT

D) DECREASED IG A IN THE TEARS

E) BLOCKED NASO-LACRIMAL DUCT

11. A NEONATE WAS BORN AT FULL TERM WITH A BIRTH WEIGHT OF 1500 GRAMS,
WAS JITTERY ON HANDLING NOTICED TO HAVE CYANOSIS ON THE HANDS & FEET. THE
INFANT HAD NORMAL TEMPARATURE; RESPIRATORY RATE 40/MIN AND LUNG & HEART
WERE NORMAL ON EXAMINATION. THE NEONATE HAS BEEN GIVEN OXYGEN. WHICH OF
FOLLOWING IS YOUR STEP IN MANAGEMENT?

A) REASSURE THE INFANTS MOTHER AND CHECK THE INFANT LATER

B) CHECK BLOOD CALCIUM

C) GIVE HIGH CONCENTRATION OXYGEN VIA MASK

D) TAKE BLOOD SAMPLE FOR BLOOD SUGAR TEST

E) TAKE BLOOD FOR FULL BLOOD EXAMINATION

12. A CHILD WAS RECENTLY VACCINATED FOR POLIO, NOW PRESENTS WITH 1-WEEK
HISTORY OF FEVER, FLACCID PARALYSIS OF BOTH LOWER LIMBS WITH NO REFLEXES. THE
MOST LIKELY DIAGNOSIS IS?

A) INFECTION WITH WILD POLIOMYELITIS

B) COMPLICATION OF THE POLIO VACCINE


C) GULLIAN BARRE SYNDROME

D) UMN LESION

E) ACUTE POLYNEUROPATHY

13. WHICH OF THE FOLLOWING IS THE LEAST LIKELY CAUSE OF IRON DEFICIENCY
ANAEMIA IN CHILDREN?

A) COWS MILK

B) THALASSEMIA

C) PREMATURITY

D) COELIAC DISEASE

E) MULTIPLE PREGNANCY (E.G. TWINS)

14. A BABY DELIVERED BY NORMAL VAGINAL DELIVERY, IS WELL AFTER BIRTH. ON THE
4TH DAY, THE BABY IS FOUND COLLAPSED IN THE COT, BREATHLESS AND FLOPPY. ON
EXAMINATION THERE ARE NO MURMURS. POSSIBLE CAUSE COULD BE--

A) TOF

B) PDA

C) TGA WITH VSD

D) PULMONARY STENOSIS

E) L. VENTRICULAR HYPERTROPHY

15.A 9-DAY-OLD BABY WITH PROJECTIVE VOMITING AND DEHYDRATION WITH K 7.1, NA
125, CL 80, & HCO3 20. WHAT IS YOUR DIAGNOSIS:
A) PYLORIC STENOSIS

B) DLACTOSEMIA

C) SEPSIS

D) HYPOGLYCAEMIA

E) CONGENITAL ADRENAL HYPERTROPHY

16. A CHILD WITH PROFUSE DIARRHOEA FOR 5 DAYS DEVELOPS CONVULSIONS. WHICH
OF THE FOLLOWING BLOOD TEST RESULTS WOULD MOST LIKELY CAUSE THIS
CONVULSION?

A) K 2.2

B) K 6.5

C) NA 132

D) NA 156

E) CL 100

17. A 37-YEAR-OLD FEMALE GIVES BIRTH TO HER 2ND CHILD. AT THE 10TH DAY THE
CHILD BECOMES JAUNDICED. HIS MOTHER TELLS THE DOCTOR THAT HER FIRST CHILD HAD
JAUNDICE AFTER BIRTH AND THEN DEVELOPED BILATERAL CATARACT AFTER THE
JAUNDICE. WHAT IS THE DIAGNOSIS?

A) CONGENITAL RUBELLA

B) NEONATAL HEPATITIS

C) DIABETES MELLITUS
D) SYPHILIS

E) GALACTOSAEMIA

18. A CHILD HAS TROUBLE SEEING THE BLACK BOARD AT SCHOOL, BUT NO PROBLEMS
WHEN USING THE COMPUTER. HIS VISION IS IMPROVED WITH PINHOLE TEST. WHAT IS
YOUR DIAGNOSIS?

A) MYOPIA

B) HYPERMETROPIA

C) CATARACT

D) GLAUCOMA

E) ?

19. A CHILD WITH HEADACHE AND MORNING VOMITING, ATAXIA AND NYSTAGMUS
FOR 6 WEEKS. HIS SCHOOL HAD A CHICKEN POX 2 WEEKS AGO. HE HAS A FAMILY HISTORY
MIGRAINE. WHAT IS THE MOST PROBABLE DIAGNOSIS?

A) MIGRAINE

B) VARICELLA CEREBELLITIS

C) INFRATENTORIAL TUMOUR

D) FRIEDRICH€™S ATAXIA

E) ?
20.IN DOWN€™S SYNDROME, ASIDE FROM THE CARDIAC COMPLICATIONS, ALL OF THE
FOLLOWING ARE ASSOCIATED WITH DOWN€™S SYNDROME, EXCEPT-

A) ACUTE LEUKAEMIA

B) MYOPIA

C) HYPOTHYROIDISM

D) VESICOURETERIC REFLUX

E) DEAFNESS

F) ALZHEIRMER€™S DISEASE

21. A 15-MONTH-OLD CHILD IS BROUGHT TO YOU BY HIS PARENTS. HE WAS BORN AT 36


WEEKS BY NORMAL VAGINAL DELIVERY. AT 8 MONTHS A MOTHER CHILD HEALTH CENTER
TEST FOR HEARING WAS DONE, WHICH WAS NORMAL. THE PARENTS SAY THAT HE
BABBLES, BUT DOES NOT SPEAK 2-3 WORD PHRASES YET. ON PHYSICAL EXAMINATION,
THE CHILD APPEARS NORMAL. WHAT WILL YOU DO?

A) REASSURE THE PARENTS THAT THIS IS A NORMAL VARIANT

B) REASSESS AT 18 MONTHS

C) REPEAT MOTHER CHILD TEST FOR HEARING

D) ARRANGE FOR AUDIOMETRY TESTING

E) RT. ORCHIDOPEXY
22. A MOTHER NOTICES A LUMP IN THE RIGHT GROIN OF HER 2-YEAR-OLD SON, WHICH
DISAPPEARED AFTER A FEW HOURS. DESPITE A THOROUGH EXAMINATION YOU ARE
UNABLE TO DISCOVER ANYTHING. THE MOST APPROPRIATE MANAGEMENT WOULD BE?

A) REASSURANCE & SEND HOME

B) ADMISSION INTO HOSPITAL FOR SURGERY

C) REQUEST THE MOTHER TO BRING THE CHILD TO YOU IMMEDIATELY WHEN THE
LUMP REAPPEARS AGAIN

D) REVIEW AFTER 1 WEEK

E) ?

23. AN 8-YEAR-OLD BOY WAS STUNG BY A BEE. HE HAD DIFFICULTY BREATHING AND
FACIAL OEDEMA. WHAT IS THE BEST TREATMENT?

A) ADRENALIN 1:1000 IM/SC

B) ADRENALINE I/V

C) ANTIHISTAMINE IV

D) HYDROCORTISONE IV

E) INTRAVENOUS FLUIDS

24. A CHILD HAS TENDERNESS & PAIN AT UPPER TIBIA OF THE LEFT LEG AND SWELLING
OF THE KNEE WHICH IS WARM. HE IS FEBRILE(39 DEGREES CELSIUS) AND GETS PAIN AT 30
DEGREES FLEXION. WHAT IS YOUR DIAGNOSIS?

A) OSTEOMYELITIS

B) OSTEOSARCOMA

C) SEPTIC ARTHRITIS
D) FRACTURE TIBIA

E) IRRITABLE HIP (TRANSIENT SYNOVITIS)

F) THROMBOPHLEBITIS/CELLULITIS

25. A 2-YEAR-OLD CHILD PRESENTS WITH A CLEAN SUPERFICIAL LACERATION ON THE


FOREHEAD. THE CHILD HAS NEVER BEEN IMMUNIZED. WHAT IS THE MOST APPROPRIATE
MANAGEMENT?

A) GIVE ANTIBIOTICS

B) TETANUS TOXOID

C) TETANUS IMMUNOGLOBIN

D) DTP VACCINATION

E) REASSURE & SEND HOME

26. THE COMMONEST CAUSE OF HIP PAIN IN A 3-YEAROLD CHILD?

(DH-288)

A) TRANSIENT SYNOVITIS
B) PERTHES DISEASE

C) SLIPPED UPPER FEMORAL EPIPHYSIS

D) ARTHRITIS

E) TUBERCULOSIS

27. A 2-WEEK-OLD BABY GAINED 200 GRAMS/WEEK SINCE BIRTH. THE MOTHER
COMPLAINTS THAT THE CHILD VOMITS MILK SOON AFTER FEEDING, OTHERWISE OKEY.
WHAT WILL YOU DO?

A) REDUCE THE TIME BETWEEN FEEDS

B) SHORTEN THE DURATION OF FEEDING

C) GIVE WATER BEFORE FEEDING WITH MILK

D) ENDOSCOPY

E) URINE MICROSCOPY

28. A CHILD WAS BORN PREMATURE(30 WEEKS GESTATION), VIA VAGINAL DELIVERY.
APGAR 5 AT 1 MIN. & 8 AT 5 MIN. AT 18 MONTHS, THE CHILD IS BROUGHT BY THE
MOTHER WITH MODERATE MENTAL RETARDATION. WHICH FAMILY HISTORY WOULD BE
LIKELY CAUSE OF THE MENTAL RETARDATION?

A) UNCLE HAS MENTAL RETARDATION

B) SISTER HAS FEBRILE CONVULSIONS

C) MOTHER HAS 2 CAFé AU LAIT SPOTS

D) FATHER IS AN ALCOHOLIC
E) PATERNAL GRANDMOTHER HAS HYPOTHYROIDISM

29. A 4-YEAR-OLD CHILD PRESENTS WITH SUDDEN ONSET OF COUGH, UNILATERAL


WHEEZE AND DECREASED RESPIRATORY MOVEMENT ON ONE SIDE. THERE IS NO FAMILY
HISTORY OF ATOPY. WHAT IS THE MOST APPROPRIATE MANAGEMENT?

A) CHEST X RAY

B) FULL BLOOD EXAMINATION

C) CT OF CHEST

D) ULTRASOUND

E) INSPIRATORY & EXPIRATORY CHEST X RAY

30. AN OBESE 8 YRS. OLD BOY CAME TO YOU. O/E HIS WEIGHT WAS 48 KG, WHICH IS
MORE THAN 98TH PERCENTILE AND THE HEIGHT WAS 140 CM, WHICH IS MORE THAN
90TH PERCENTILE, OTHERWISE NORMAL. WOF IS CORRECT IN RELATION TO THIS BOY---

A) ADVANCED BONE AGE

B) INCREASED BLOOD SUGAR LEVEL

C) THYROTOXICOSIS

D) HYPOTHYROIDISM

E) PRIMARY HYPERALDESTERONISM
31.A BABY IS BORN A NORMAL FULL TERM DELIVERY. ON EXAMINATION OF THE
NEWBORN, WHICH OF THE FOLLOWING MAY NORMALLY BE SEEN?

A) BREAST ENLARGEMENT IN MALE INFANTS

B) ENLARGED CLITORIS IN FEMALE INFANTS

C) FUSION OF LABIA

D) ENLARGE LEVER MORE THAN 5 CM AND PALPABLE

E) CLEFT PLATE

32. A MOTHER BRINGS HER BABY WHO IS BABBLING AND SQUEAKING, IS ABLE TO HOLD
HIS HEAD, MOVE FROM PRONE TO SUPINE POSITION, IS ABLE TO SIT UP UNSUPPORTED,
CAN REACH FOR OBJECTS AND WHEN SUPPORTED, CAN STAND AND BOUNCE. WHAT IS
THE POSSIBLE AGE OF THIS CHILD?

A) 5 WEEKS

B) 2 WEEKS

C) 7 MONTHS

D) 10 MONTHS

E) 12 MONTHS

33. DURING 12 WEEKS OF PREGNANCY ULTRASOUND SHOWS NUCHAL THICKNESS OF


THE FETUS. KERIOTYPE ALSO DONE. NOW YOUR ADVICE SHOULD BE---
A)TERMINATE THE PREGNANCY AT 12 WEEK

B)REPEAT KERIOTYPE TEST

C)SERUM FETOPROTEIN TEST AT SECOND TRIMESTER

D)FULL BLOOD TEST

34.AFTER A DIFFICULT FORCEPS DELIVERY, IT IS NOTICED THAT THE BABY HANGS HIS
ARM TO THE SIDE, AND CANNOT MOVE IT. WHAT IS THE LIKELY CAUSE?

A) FRACTURE HUMERUS

B) ERBS PALSY

35. SIX WEEK OLD CHILD INITIALLY WAS NORMAL BUT AFTER 3 WKS SHE WAS NOT
SUCKING MILK. SHE APPEARED AS A FLOPPY CHILD, WHICH WAS PROGRESSIVELY GETTING
WORSE. WOF IS YOUR DIAGNOSIS--

A) BOTULISM

B) WERDNIG-HOFFMAN DISEASE(SPINAL MUSCULAR ATROPHY)

C) CEREBRAL PULSY

D) MYASTHENIA GRAVIS

E)
36. A 6 WEEK OLD INFANT VOMITING INTERMITTENTLY SINCE BIRTH. SHE LOOKS THIN,
NOT GAINING WEIGHT. NO ABNORMALITY ON PHYSICAL EXAMINATION. WHAT IS YOUR
DX :

A) CYSTIC FIBROSIS

B) PYLORIC STENOSIS

C) SUBDURAL HAEMATOMA

D) PHENYL KETONURIA

E) REGURGITATION

F) UTI

1. WHILE DOING HYSTERECTOMY IN LITHOTOMY POSITION WOF NERVE IS PRONE TO


INJURY---

A) PUDENDAL NERVE

B) SCIATIC NERVE

C) COMMON PERONEAL NERVE

2. A WOMAN IS CHALLENGED WITH COMBINED OESTROGEN & PROGESTERONE, BUT


DOES NOT GET WITHDRAWAL BLEEDING. WHICH ORGAN IS MOST LIKELY TO BE
AFFECTED?

A) OVERIES

B) UTERUS
C) PITUITARY

D) HYPOTHALAMUS

E) FALLOPIAN TUBES

3. THE MOST APPROPRIATE OCP FOR A 24-YEAR-OLD WOMAN TAKING PHENYTOIN IS?

A) MICROGYNON 30

B) MICROGYNON 50

C) DIANE 35

D) CLOMIPHENE

E) OESTROGEN 85 MICROGRAMS

4. A 20-YEAR-OLD FEMALE PRESENTS TO YOU WITH AMENORRHOEA. ON ULTRASOUND,


A CYST(1.8 X 1.3 X 1.0 CM) IS FOUND ON THE RIGHT OVERY. WHAT IS THE MOST
APPROPRIATE MANAGEMENT?

A) LAPROSCOPY TO FIND CYTOLOGY

B) REPEAT ULTRASOUND IN 4 WEEKS TIME

C) DO NOTHING

D) OCP

E) OOPHORECTOMY

5. REGARDING OXYTOCIN, WHICH OF THE FOLLOWING IS TRUE?

A) HAS AN ANITDIURECTIC EFFECT


B) IT IS A STEROID

C) IT IS PRODUCED BY THE POSTERIOR PITUITARY

D) IT HAS AN ACTION ON MOST OF THE INVOLUNTARY MUSCLES

E) IT CAUSES MILK PRODUCTION

6. A FEMALE, 17 WEEKS PREGNANT, HAS ACUTE RIGHT ILIAC FOSSA PAIN. WHICH OF
THE FOLLOWING IS THE LEAST LIKELY CAUSE OF THE PAIN?

A) PYELONEPHRITIS

B) RED DEGENERATION OF A FIBROID

C) ECTOPIC PREGNANCY

D) ACUTE APPENDICITIS

E) TORSION OF AN OVARIAN CYST

7. DEPO PROVERA CAN CAUSE ALL THE FOLLOWING, EXCEPT?

A) MILD HYPERTENSION

B) AMENORRHOEA AFTER THE 3RD INJECTION

C) FERTILITY PROBLEMS FOR 6-12 MONTHS AFTER STOPPING THE TREATMENT

D) MENSTRUAL IRREGULARITIES

E) WEIGHT GAIN
8. AN 8-WEEK GESTATION PRIMIGRAVIDA, PRESENTED WITH SLIGHT VAGINAL
BLEEDING. YOU WILL DO AN ULTRASOUND EXAMINATION FOR ALL OF THE FOLLOWING,
EXCEPT?

A) TO SEE IF THE SAC IS WITHIN THE UTERUS OR NOT

B) TO SEE IF THE FOETUS IS ALIVE OR DEAD

C) TO CHECK FOR ANY CONGENITAL ANOMALIES

D) TO LOOK FOR A CLOT INSIDE THE UTERUS

E) TO LOOK FOR THE POSITION OF THE PLACENTA

9. A 25-YEAR-OLD FEMALE PRESENTS WITH GALACTORRHOEA AND AMENORRHOEA, CT


SCAN SHOWS THE PRESENCE OF PITUITARY ADENOMA, 3 MM IN DIAMETER, 1.6 MM
FROM THE OPTIC CHIASMA, WITH NO VISUAL DISTURBANCES. PROLACTIN LEVEL IS
HIGH(4250). WHAT IS THE MOST APPROPRIATE MANAGEMENT?

A) TRANS-SPHENOIDAL SURGERY

B) PHENOTHIAZINE

C) BROMOCRIPTINE

D) CLOMIPHENE

E) OCP

10. A PRIMIGRAVIDA, 14 WEEKS GESTATION, IS FOUND TO HAVE PROTEINURIA(+++), BP


150/95 MMHG AND SWELLING. WHAT IS TRUE?
A) SHE HAS PRE-ECLAMPSIA(24 WKS)

B) SHE HAS UTI

C) SHE WILL NOT BE CARRY ON WITH THE PREGNANCY

D) RENAL HYPERTENSION

E) PRE-EXISTING RENAL DISEASE

11. IN A POSTMENOPAUSAL WOMAN, WHAT IS THE LEAST LIKELY CAUSE OF GREENISH


FOUL SMELLING VAGINAL DISCHARGE?

A) TRICHOMONAS VAGINALIS- LONG STANDING CASES DISCHARGE IS GREENISH AND


FROTHY

B) GARDNERELLA VAGINALIS €“ STRONG FISHY SMELL, GRAYISH DISCHARGE

C) CHLAMYDIA €“ MAY BE MUCOPURULENT

D) FOREIGN BODY

E) CERVICAL CARCINOMA

12. WHAT IS TRUE REGARDING CARPAL TUNNEL SYNDROME IN PREGNANCY?

A) RELIEVED BY USE OF SPLINT WITH HAND IN EXTENDED POSITION

B) CAUSES SEVERE PAIN IN THE THENAR EMINENCE POSITION

C) RARELY REQUIRES SURGERY

D) DIURETICS SHOULD BE GIVEN


E) BECOMES BETTER AS GESTATION PROGRESSES

13. A WOMAN COMES TO YOU 2 YEARS AFTER HER MENOPAUSE ASKING FOR
SOMETHING TO RELIEVE HER POSTMENOPAUSAL SYNDROMES. YOU DECIDE TO GIVE HER
COMBINED HRT. YOU EXPLAIN TO HER THAT ALL OF THE FOLLOWING ARE THE EFFECTS OF
PROGESTERONE, EXCEPT?

A) SHE WILL HAVE PRE-MENSTRUAL LIKE SYMPTOMS

B) SHE WILL HAVE WITHDRAWAL BLEEDING EVERY MONTH

C) PROGESTERONE WILL OPPOSE THE ACTION OF OESTROGEN ON THE ENDOMETRIUM

D) PROGESTERONE WILL POTENTIATE THE ACTION OF OESTROGEN IN INCREASING HDL

E) PROGESTERONE WILL AUGMENT THE ACTION OF OESTROGEN FOR PREVENTION OF


OSTEOPOROSIS

14. WHICH OF THE FOLLOWING IS NOT TRUE, FOR A WOMAN HAVING ANTENATAL
CARE?

A) PRESENCE OF ANTI-RHO MEANS THAT THE BABY CAN HAVE PERMANENT HEART
BLOCK

B) A PLATELET COUNT OF 100000 IS A RISK FOR DEVELOPING INTRACRANIAL BLEED IN


THE INFANT

C) THE PRESENCE OF ANA COULD CAUSE GROWTH RETARDATION OF THE FOETUS

D) ?
15. WHICH OF THE FOLLOWING OCCURS IN PREGNANCY BY PROGESTERONE?

A) NO RENAL CHANGES

B) DILATATION OF THE URETERS & CALYCES

C) DILATATION OF THE CALYCES ONLY

D) DILATATION OF THE URETER ONLY

E) REDUCE INCIDENCE OF INFECTION

16. WHICH OF THE FOLLOWING CTG PATTERNS INDICATES FOETAL DISTRESS?

A) ACCELERATION WITH FOETAL MOVEMENTS

B) EARLY DECELERATIONS WITH UTERINE CONTRACTIONS

C) DECELERATIONS WHEN THE FOETUS IS SLEEPING

D) ACCELERATIONS ON SOUND STIMULATION

E) A BEAT TO BEAT VARIABILITY< 5 BEATS/MIN

17. REGARDING ADMINISTRATION OF ANTI-D GAMMA GLOBULIN IN RH -VE WOMEN,


WHICH OF THE FOLLOWING IS TRUE?

A) IT IS A FORM OF ACTIVE IMMUNIZATION

B) ADMINISTRATION IN THE 2ND TRIMESTER OF PREGNANCY HAS BEEN SHOWN TO


DECREASE AUTOIMMUNIZATION IN THE MOTHER

C) IT INCREASE THE RISK OF HIV TRANSMISSION


D) IT SHOULD BE GIVEN WITHIN 24 HOURS OF THE DELIVERY-2ND

E) IT IS NOT NECESSARY TO GIVE IT IN CASES OF ABO INCOMPATIBILITY-1ST

18. A YOUNG WOMAN, PREGNANT FOR 14 WEEKS, UTERINE SIZE CORRESPONDING TO


16 WEEKS, NOW PRESENTS WITH BLEEDING AND PASSING OUT GRAPE LIKE MATERIAL
THROUGH A DILATED CERVIX. WHICH OF THE FOLLOWING IS THE BEST FOLLOW UP?

A) WEEKLY MONITORING OF URINE HCG, UNTIL IT COMES TO NORMAL LEVEL

B) ULTRASOUND

C) CHEST X RAY FOR METASTASIS

D) CHECK PLACENTA FOR SIGNS OF MALIGNANCY

E) OBSERVATION & REASSURANCE

19. A 32-YEARS WOMAN BECAME PREGNANT. SHE HAD 5 PREGNANCIES BEFORE, OF


WHICH SHE LOST 3 SPONTANEOUSLY BETWEEN 8-10 WEEKS & 2 WERE STOPPED BY
INTERVENTION AT 12 WEEKS. WHAT IS THE MOST LIKELY CAUSE?

A) CERVICAL INCOMPETENCE

B) CHROMOSOMAL ABNORMALITIES

C) HORMONAL IMBALANCE

D) OVARIAN TUMOR

E) VALVO VAGINITIS
20. ALL OF THE FOLLOWING ARE TRUE ABOUT NEURAL TUBE DEFECT, EXCEPT?

A) DECREASE IN ALPHA FETOPROTEIN

B) POLYHYDRAMNIOS

C) FOLIC ACID PROPHYLAXIS CAN DECREASE THE RISK OF NEURAL TUBE DEFECTS

D) THERE IS A 4% CHANCE OF NEURAL TUBE DEFECT IN THE 2ND PREGNANCY

E) CAN BE DETECTED BY AMNIO CENTESIS

21. A 19-YEAR OLD PRIMIGRAVIDA, 38 WEEKS GESTATION, WITH HYPER-REFLEXIA,


HYPERTENTION AND OEDEMA, MIGHT BE EXPECTED TO HAVE ELEVATED BLOOD LEVELS
OF ALL OF THE FOLLOWING, EXCEPT?

A) PLATELETS

B) URIC ACID

C) CREATININE

D) SGOT

E) BUN

22. WHICH OF THE FOLLOWING IS AN ABSOLUTE CONTRAINDICATION FOR OCP USE?

A) MIGRAINE WITH HEMIPLEGIC AURA

B) SMOKING

C) VARICOSE VEINS
D) DIABETES MELLITUS

E) GALL BLADDER DISEASE

23. ALL OF THE FOLLOWING ARE USED IN DYSMENNORHOEA, EXCEPT?

A) D & C

B) PROGESTERONE

C) MEFNAMIC ACID

D) OCP

E) BROMOCRIPTINE

24. A 62-YEARS-OLD FEMALE COMPLAINTS OF BLEEDING PER VAGINA. AFTER


EXAMINATION AND INVESTIGATION, ENDOMETRIAL HYPERPLASIA WAS FOUND AND A
SOLID MASS (5 CM IN DIAMETER) WAS FOUND IN ONE OF THE OVARIES. POSSIBLE
DIAGNOSIS IS?

A) TERATOMA

B) OVARIAN CA

C) FIBROMA

D) GRANULOSA CELL TUMOR


25. A PREGNANT WOMAN, 34 WEEKS-GESTATIONAL AGE, PRESENTS WITH
ANTEPARTUM HAEMORRHAGE. YOU DIAGNOSE ABRUPTIO PLACENTA. ALL OF THE
FOLLOWING ARE POSSIBLY SEEN, EXCEPT?

A) TENSE & TENDER UTERUS

B) BP OF 180/110 MMHG

C) SHOCK OUT OF PROPORTION WITH THE BLEEDING

D) FOETAL HEAD MOBILE ABOVE THE PELVIC BRIM

E) DEAD FOETUS

26. OCP€™S MAINLY AFFECT WHICH SITE?

A) HYPOTHALAMUS

B) PITUITARY

C) OVARIES

D) ENDOMETRIUM

E) CERVICAL MUCOUS

27. ON CLINICAL EXAMINATION, HOW DO YOU DIFFERENTIATE AN OVARIAN TUMOR


FROM ASCITIS?
A) DULLNESS IN THE PERIPHERY & CENTRAL TYMPANETIC NOTE, ON PERCUSSION

B) TYMPANETIC NOTE IN THE PERIPHERY & CENTRAL DULLNESS, ON PERCUSSION

C) SHIFTING DULLNESS

D) DECREASED BOWEL SOUNDS

E) FLUID THRILL

28. A 26-YEARS OLD PRIMI, 152 CM TALL, WITH VERTEX PRESENTATION, HEAD AT 0
STATION, CERVIX EFFACED COMPLETELY AND DILATED TO 4 CM, WITH INTACT
MEMBRANES AT 37 WEEKS. WHAT IS THE MOST APPROPRATE MANAGEMENT?

A) LOWER SEGMENT CAESAREAN SECTION

B) OXYTOCIN TO INDUCE LABOUR

C) ULTRASOUND TO DETECT FOETAL ABNORMALITIES

D) X-RAY PELVIMETRY

E) TRIAL LABOUR

29. A 24-YEAR OLD WOMAN CAME WITH SECONDARY AMENORRHOEA & WAS
DIAGNOSED TO HAVE POLY CYSTIC OVARIAN SYNDROME. WOF IS TRUE ABOUT HER
CONDITIONS--

( NOTE: IN PCO

A) IMMEDIATE LAPAROSCOPIC REMOVAL OF CYSTS TESTOSTERONNE-INCREASE LH:FSH-


INCREASE

B) FSH:LH RATIO WILL BE 3:1 LH-INCREASE INSULINE€”INCREASE)


C) INCREASE ANDROSTENEDIONE €“ ALSO TESTOSTERONE LJ-223

D) 17-HYDROXY PROGESTERONE WILL BE NORMAL

E) USG WILL SHOW ECHO DENSITY OVER THE AFFECTED PART OF THE OVARIES--
ECHOFREE

30. IN A TWIN PREGNANCY, AFTER DELIVERY OF THE FIRST TWIN, YOU DO ALL OF THE
FOLLOWING, EXCEPT?

A) VAGINAL EXAMINATION

B) EXTERNAL VERSION IF THE SECOND TWIN IS BREECH

C) OXYTOCIN INFUSION IF CONTRACTIONS DON€™T START

D) CHECK FOETAL HEART RATE

E) CHECK LIE OF THE SECOND TWIN

31.REGARDING UTERINE RUPTURE AFTER PREVIOUS LOWER SEGMENT CEASARIAN


SECTION WOF IS MOST LIKELY INDICATION?

A) INCIDENSE IS INCREASED BY 5%

B) INCREASE WITH PROSTAGLANDIN INDUCTION

C) INCREASE WITH OXYTOCIN AUGMENTATION

32. WHICH OF THE FOLLOWING CANCER IS MOST COMMON TO METASTASIZE TO BONE


?

A) PROSTATE CANCER [ MURTAGH-GP:247 B-P-L: BREAST-PROSTATE-LUNG]


B) STOMACH CANCER

C) BREAST CANCER

D) LUNG CANCER

E) CEREBRAL CANCER

33. REGARDING CONTRACEPTION, ALL OF THE FOLLOWING ARE CORRECT EXCEPT-

A) IT IS ABSOLUTELY CONTRAINDICATED IN WOMEN WITH MIGRAINOUS AURA [ GP-


961]

B) IT SHOULD NOT BE PRESCRIBED FOR A PATIENT WITH DVT

C) IF PATIENT IS ON 20 MICROGRAM THERE IS NO NEED TO STOP PRE-OPERATIVELY

D) 50 MICROGRAM SHOULD ONLY BE STARTED IF 30 MICROGRAM CAUSES IRREGULAR


BLEEDING

34. A 26 YRS OLD YOUNG LADY WITH 10 WEEKS OF AMENORRHOEA. WOF TEST WILL
AFFECT HER MANAGEMENT AFTER THE DIAGNOSIS OF HER PREGNANCY?

A) VARICELLA ANTIBODY

B) HIV

C) GLUCOSE CHALLENGE TEST

35. WOF IS NOT TRUE REGARDING NEURAL TUBE DEFECTS-

A) DECREASE ALPHA FETO PROTEIN

B) FOLATE INTAKE CAN DECREASE THE INCIDENCE

C) CAN BE DETECTED BY AMNIOCENTESIS


D) SOCIOECONOMIC FACTOR IS AN IMPORTANT FACTOR

1. WHILE SHOPPING IN A SHOPPING CENTER A LADY FOUND THAT EVERYTHING IS


MOVING AROUND HER. AT THAT TIME SHE FELT HERSELF SEPARATED FROM THIS WORLD.
THESE HAPPENED SEVERAL OCCASIONS IN THE PAST. IN ONE OCCASION THIS HAPPENED
WHEN SHE WAS TALKING WITH HER RELATIVES AND UTTERED SOME UN-RECOGNIZABLE
WORDS, WHICH COULD NOT UNDERSTOOD BY THEM. WOF IS NEXT APPROPRIATE STEP---

A) TALK TO HER RELATIVES

B) CT SCAN OF HEAD

C) MRI OF BRAIN

D) MEASURE BP IN LYING AND STANDING POSITION

E) EEG

2. A YOUNGMAN WAS FOUND BY HIS PARENTS AGITATED AT NIGHT IN HIS ROOM. THEY
TOLD YOU THAT THEIR SON DID NOT SLEEP FOR LAST 3 NIGHTS AND ALSO HAS NOT
TAKEN FOOD FOR 3 DAYS. HE USED CANNABIS AND HE IS ON €ŒLITHIUM€. THE
YOUNG MAN TOLD YOU THAT HE WAS COMPLYING WITH LITHIUM PRESCRIBE BY THE
DOCTOR. O/E YOU FOUND TREMOR, AGITATION, ARRYTHMIAS AND ATAXIA. WOF IS YOUR
NEXT STEP----

A) ELECTROLYTE ESTIMATION

B) URINE TEST

C) SERUM LITHIUM LEVEL ESTIMATION

3. ABOUT €ŒCORE GROUP€ TO HELP ALCOHOLICS AND ADDICTS. WOF IS CORRECT---


A) ASK OPINION AND HELP FROM NON-PROFESSIONALS

B) CO-OPERATION AND SKILL SHOULD BE INCREASED AMONG MEMBERS OF THE


GROUP.

4. SCHIZOPHRENIC PATIENT HAS GOOD PROGNOSIS IF THEY HAD---

A) PROLONG ONSET

B) AFFECTED SYMPTOM

C) FAMILY HISTORY

D) IF ALL THE RELATIVE HELP IN PSYCHOTHERAPY

5. A 60-YEAR-OLD WITH WERNICKE€™S ENCEPHALOPATHY PRESENT TO CASUALTY. ON


EXAMINATION HE IS FOUND TO BE HYPOGLYCAEMIC, HAS NYSTAGMUS, DRINKS 6GM
ALCOHOL PER DAY. WHAT IS YOUR INITIAL MANAGEMENT?

A) NORMAL SALINE (0.9%) & DEXTROSE SOLUTION

B) GLUCAGON IV

C) 50% DEXTROSE IV

D) INTRAVENOUS THIAMINE FOLLOWED BY DEXTROSE SOLUTION

E) INSULINE & DEXTROSE IV


6. A MAN IS BROUGHT TO THE HOSPITAL BY THE POLICE. HOW WILL YOU DIFFERENT
WHETHER HE HAS SCHIZOPHRENIA OR DELIRIUM?

A) FLUCTUATING LEVEL OF CONSCIOUSNESS

B) HALLUCINATIONS

C) CONFUSION

D) IRRITABILITY

E) AGITATION

7. A 33-YEAR-OLD LADY COMES TO SEE YOU AT YOUR PRACTICE WITH COMPLAINTS OF


INSOMNIA. IN THE LAST 2 MONTHS SHE HAS LOST 2 KGS. SHE HAS 3 CHILDREN AGED
10,7& 4 YRS. IN THE PAST FEW MONTHS SHE DOESN€™T FEEL AS CLOSE SHE USED TO,
TO HER HUSBAND. SHE FEELS MORE DISTANT FROM HIM. THEY HAVE BEEN LIVING IN A
RENTED HOUSE FOR THE LAST 10 YRS OR SO AND HAVE NOT BEEN ABLE TO GO FOR A
HOLIDAY DUE TO FINANCIAL DIFFICULTIES. SHE ALSO COMPLAINS OF HEAVY PERIODS.
YOU DID A FBC (FULL BLOOD COUNT), THYROID HORMONE LEVELS, ETC AND ALL CAME
OUT WITHIN NORMAL RANGE. PHYSICAL EXAMINATION WAS UNREMARKABLE. WHAT IS
YOUR NEXT STEP IN MANAGEMENT FOR THIS WOMAN?

A) REFER TO GYNECOLOGIST

B) REFER TO PSYCHIATRIST

C) REFER TO MARRIAGE COUNSELOR

D) INTERVIEW HUSBAND

E) SUGGEST THAT SHE TAKES A HOLIDAY

8. WHICH OF THE FOLLOWING IS REVERSIBLE INHIBITOR OF MONOAMINE OXIDASE?


A) PHENELZINE

B) FLOUXITINE

C) MOCLOBEMIDE

D) MIANSERIN

E) TRANYLCYCLOMINE

9. ANOREXIA NERVOSA WILL HAVE ALL EXCEPT?

A) FLIGHT OF IDEAS

B) HYPOKALEMIA

C) LANUGO HAIR

D) MAINLY SEEN IN FEMALES

E) DEPRESSION

10. THE MENTAL HEALTH ACT WAS INTRODUCED APPROXIMATELY 20 YEARS AGO. OF
THE FOLLOWING, WHICH IS TRUE?

A) IT IS THE SAME IN ALL STATES & TERRITORIES IN AUSTRALIA

B) IT CAN SAVE LIVES

C) IT TAKES AWAY THE INDIVIDUALS FREEDOM

D) IT CAN BE INVOLUNTARILY IMPOSED ON EVERYONE

E) IT NEEDS SIX PEOPLE TO IMPOSE IT


11. THE PARENTS OF AN ADOPTED GIRL, WHO HAS BEEN DIAGNOSED AS
SCHIZOPHRENIC, HAVE COME TO CONSULT YOU ABOUT THEIR OWN BIOLOGICAL CHILD.
THEY INQUIRE ABOUT THE POSSIBILITY OF SCHIZOPHRENIA DEVELOPING IN THEIR CHILD.
YOU SHOULD TELL THEM?

A) NIL

B) LESS THAN 2%

C) 10%

D) 50%

E) 75%

12. IN WHICH OF THE FOLLOWING CONDITIONS IS THERE DE€™JA€™VU?

A) OCCIPITAL LOBE TUMOUR

B) TEMPORAL LOBE TUMOUR

C) FRONTAL LOBE TUMOUR

D) RAISED INTRACRANIAL PRESSURE

E) KORSAKOFF€™S SYNDROME

13. A YOUNG WOMAN WHO IS AN ACCOUNTANT IN A BANK, AFTER EXPERIENCING


BANK ROBBERY IN WHICH SHE WAS TAKEN AS A HOSTAGE FOR SEVERAL HOURS. AFTER 3-
4 WEEKS SHE RETURNED TO WORK. WHAT SHOULD YOU EXPECT ON HER?

A) EROTIC MEMORIES OF THE EVENT

B) A BRIEF PSYCHOTIC REACTION


C) VISUAL HALLUCINATION OF THE EVENT

D) IRRITABILITY AND OUTBURSTS OF ANGER

E) HYPERSOMNIA

14. A 36 YEARS OLD ENGINEER IS DUE TO GIVE AN SPEECH SOON AND IS RELUCTANT TO
BE EXPOSED TO PEOPLE. AT THE SAME TIME, HE DOES NOT WANT TO SIGN CHEQUES OR
WRITE ANYTHING IN PUBLIC. HE ALSO THINKS HIS BOSS IS CONTROLLING WHAT HE IS
DOING ALTHOUGH HE KNOWS THAT THERE IS NO MOTIVE FOR THAT BECAUSE HE IS A
VERY GOOD WORKER. HE REALIZES THERE IS NO REASON TO ACT THIS WAY. LATELY HE
HAS BEEN AVOIDING TO GO TO THE CANTEEN WITH HIS MATES AND TAKES ALCOHOL TO
COPE WITH THE SITUATION. WHAT HE IS SUFFERING FROM---

A) SOCIAL PHOBIA

B) AGORAPHOBIA

C) ANTISOCIAL PERSONALITY DISORDER

D) PANIC DISORDER

E) PARANOID DISORDER

15. WHAT IS INCORRECT CONCERNING GILLE DE LA TOURETTE SYNDROME?

A) ASSOCIATED WITH TICS

B) PATIENT IS NOT DISTRESSED BY THE TICS

C) PATIENT UTTERS OBSCENE WORDS IN LESS THAN 10% OF CASES


16. WOF IS MOST LIKELY ASSOCIATED WITH BULIMIA?

A) CACHEXIA

B) INDUCED VOMITING

C) HALLUCINATION

D) SEXUAL DISORDER

E) DISTURBED BODY IMAGE

17. A PERSON WITH HYPNOLOGIC HALLUCINATIONS:

A) A FEELING OF INSECTS CRAWLING OVER THE SKIN

B) USUALLY OCCURS AT THE BEGINNING OF SLEEP

C) SEEN IN SCHIZOPHRENIC PATIENT

D) SEEN IN AMPHETAMINE ENTOXICATION

E) SEEN WITH CANNABIS USE

18. ALL THE FOLLOWING CAN CAUSE SEROTONIN SYNDROME EXCEPT?

A) HALOPERIDOL

B) L TRYPTOPHAN

C) CLONAZEPAM (BENZODIAZEPINE)

D) MOCLOBEMIDE
E) DEXTROMETHORPHAN

19. DEFENSE MECHANISM HAVE BEEN CLASSIFIED AS MATURE ANDIMMATURE. WHICH


OF THE FOLLOWING IS AN IMMATURE DEFENSE MECHANISM?

A) PROJECTION

B) SUBLIMATION

C) IDENTIFICATION

D) DISSOCIATION

20. CONCRETE INTERPRETATION OF A PROVERB IS CHARACTERISTIC OF?

A) DYSTHYMIA

B) DEPERSONALISATION

C) DEMENTIA

D) DELUSIONAL DISORDER

E) DEPRESSION

21. BULIMIA NERVOSA

A) HYPERKALEMIA

B) NEVER OCCUR IN MALE

C) ASSOCIATED WITH DEPRESSION


22. CONTRAINDICATION TO TRICYCLIC THERAPY:

A) TCA

B) ALCOHOL

C) LITHIUM

D) MAOI

E) ANTIPSYCHOTIC DRUG

23. HALUCINATION OF MINIATURE OBJECT OR ANIMAL CRAWLING OCCURS IN

A) SCHIZOPHRENIA

B) GENERALIZED EPILEPSY

C) DELIRIUM TREMENS

24. WHICH OF THE FOLLOWING WOULD GIVE A BETTER PROGNOSIS IN


SCHIZOPHRENIA?

A) NO PRECIPITATING FEATURES

B) ABRUPT ONSET

C) SCHIZOID PERSONALITY

D) DRUG ABUSE

E) WITH AFFECTIVE SYMPTOMS


25. A 21 YRS OLD YOUNG GIRL WAS BROUGHT TO YOU BY HER PARENTS. SHE TELLS YOU
THAT HER FRIENDS ARE PUTTING MESSAGE IN HER BRAIN TRYING TO DISTURB HER.
TODAY WHEN SHE HAS APPEARED IN THE TUTORIAL CLASS THEY TRIED TO LOCK HER
THOUGHT AND TRIED TO INFLUENCE HER. WOF IS YOUR NEXT STEP?

A) CT SCAN

B) X-RAY

C) OLANZAPINE

D) OCP

E) FULL BLOOD TEST

26. WOF IS FORMAL THOUGHT DISORDER?

A) THOUGHT ECHOING

B) OVER INCLUSIVENESS

C) DELUSIONS

D) HALLUCINATIONS

E) ALL OF THE ABOVE

27. WHICH OF THE FOLLOWING IS CORRECT REGARDING €ŒBABY BLUES€?

A) IS PRESENT IN MORE THAN 50% OF WOMEN


B) POSTPARTUM BLUES LASTS FOR AT LEAST 3 MONTHS

C) PREDISPOSES TO POSTPARTUM PSYCHOSIS

D) SHE REQUIRES ANTIPSYCHOTIC MEDICATION

E) SHE REQUIRES ECT

28. A LONELY MAN WAS FOUND TO BE SEMICONSCIOUS AND HAD BILATERAL SIX
NERVE PALSY. LIKELY DIAGNOSIS IS---

A) ALCOHOLIC TOXICITY

B) DELIRIUM TREMEN

C) WEMICKE€™S ENCEPHALOPATHY

29. PANIC DISORDER: EXEPT---

A) IS EQUALLY FREQUENT IN MEN & WOMEN

B) PATIENT SHOULD AVOID TRIGGER FACTORS

C) STARTS IN THE 20€™S

30. A 28-YEARS OLD MALE PRESENTS WITH GRANDIOSE BELIEFS OF HIS OWN
IMPORTANCE, OVER CONCERN FOR HIS APPEARANCE AND EXTREME SENSITIVITY TO
OTHERS CRITICISM. HE LIKEWISE HAS UNREALISTIC FANTASIES OF UNLIMITED SUCCESS,
CAPABILITIES AND POWER. HE ALSO LACKS EMPATHY AND USES OTHER PEOPLE FOR HIS
OWN BENEFIT. THIS HISTORY IS SUGGESTIVE OF WHAT TYPE OF PERSONALITY?
A) SCHIZOTYPAL PERSONALITY

B) ANTISOCIAL PERSONALITY

C) NARCISSISTIC PERSONALITY

D) BORDERLINE PERSONALITY

AMC MCQS 2001 MAY RECALLS

MCQ 2001 MAY RECALL QUESTIONS

MEDICINE

1. IN WHICH OF THE FOLLOWING DIABETIC KETOACIDOSIS MOST COMMONLY


PRESENTS WITH

1. UNDIAGNOSED IDDM

2. UNDIAGNOSED NIDDM

3. KNOWN IDDM WHEN STOPPED INSULIN

4. KNOWN IDDM WITH FOOT INFECTION

2. WHICH IS CORRECT FOR NIDDM

1. GENETIC FACTOR IS MORE IMPORTANT IN IDDM THEN NIDDM

2. THEY WILL NEVER REQUIRE INSULIN

3. 10-20% OF IDDM NEED HYPOGLYCAEMIC AGENTS

4. ABDOMINAL FAT IS THE RISK FACTOR FOR NIDDM


3. PAPILLARY NECROSIS EXCEPT

1. ANALGESIC NEPHROPATHY

2. ALCOHOL NEPHROPATHY

3. MEDULLARY SPONGE KIDNEY

4. DIABETIS

4. PICTURE – GIRL ACNE AND HIRSUTISM ON THE FACE. PRESENTED FOR THE FIRST TIME

1. ANTIBIOTICS

2. ANTIBIOTICS AND RETINOIDS

3. STEROIDS CREAM

4. ANTIANTIGEN(DANAZOL)

5. CYPROPTERONE ACETATE

5A. PICTURE WITH 3 LESIONS- WITH RAISED BORDER

1. GRANULOMA ANNULARE

2. ERYTHEMA MULTIFORME

3. PICTURE –AMC BOOK – LEISION ON THE FACE

4. MYCROSPORUM CANIS

5. WHAT IS NOT TRUE ABOUT PARKINSONS DISEASE

1. CAN STOP WHILE WALKING INVOLUNTARY

2. TREMOR AT SLEEPING

3. REST TREMOR
4. AFFECT ONE SIDE MORE THAN THE OTHER

6. MYCOPLASMA PNEUMONIA NOT TRUE

1. SEVERE COUGH

2. HIGH FEVER

3. PLEURATIC PAIN

7. WHICH OF THE FOLLOWING DRUGS CAUSES HYPERTENSION WHEN STOPPED


ABRUPTLY.

1. CLONIDINE

2. METOPROLOL

3. METHYLDOPA

4. ACE INHIBITORS

8. REGARDING SERUM LIPID PROFILE AND DIET WHAT IS TRUE

1. OLIVE OIL WILL INCREASE LDL/HDL RATO

2. POSITIVE ENERGY BANCES INCREASE LDH

3. INCRESE IN DIETRY STURATED FATTY ACIDS WILL INCREASE VLDL

4. FISH 100 300GM PER WEEK WILL HAVE NO EFFECT ON PLASMA LIPID

5. LARGE ALCLHOL INTAKE CAN REDUCE LDL

9. POLYCYTHEMIA RUBRA VERA

1. CLINICAL CYANOSIS MAY BE DIFFICULT TO DETECT

2. ERYTHROPOETIN LEVELS SUPRESSED

3. ESR
4. SLEAP APNOEA MAY CAUSE THE PROBLEM

12. WHICH OF THE MOST COMMON ANTIBIOTIC- WHICH CAUSE PSEUDOMEMBRANOUS


COLITIS

1. METRONIDAZOLE

2. AMPICILLIN

13. MYASTHENIA GRAVIS WHAT IS CORRECT?

1. THYMECTOMY IS ONLY INDICATED FOR THYMOMA

2. EMG CAN ALWAYS POSITIVE

3. SOME BODY DOES NOT HAVE ANTIBODY

4. ANTICHOLINEGIC IS THE TREATMENT

14. HEPATITIS CMOST COMMONLY - IN AUSTRALIA

1. DRUG USER

2. HOMOSEXUAL

3. HETEROSEXUAL

4. BLOOD PRODUCTS

15. WHAT ARE THE SIDE EFFECTS CORTICOSTEROIDS EXCEPT

1. INCREASE WBC

2. NEUTROPENIA

3. OSTEOMALASIA
16. ERADICATION OF HELICOBACTERIA PYLORY

1. INCRESED HEALING ULCER

2. DECREASED RELAPSE RATE

17. AS A PUBLIC HEALTH OFFICER WHAT IS THE MOST SENSITIVE SCREENING TEST –FOR
LEAD POISONING

1. DO AN ENVIRONMENTAL HISTRY STUDY

2. BLOOD LEAD LEVEL

3. PHYSICAL EXAMINATION OF EVERY CHILD

18. A PREGNANT WOMEN FIXED AND WIDE SPLITING SECOND HEART SOUND

1. ASD

2. VSD

3. PULMONARY STENOSIS

4. COARCTATION OF AORTA

19. A 55 Y.O. MAN PRESENTS WITH LOW MOTOR NEURON SYMPTOMS AND DEPRESSED
BRACHIAL REXLEXES . ALSO HE HAS A SYMPROMES OF UPPER MOTOR NEURON AT HIS
LEGS.WHAT IS THE MOST LIKELY DIAGNOSIS? AMC BOOK

1.MOTOR NEURON DISEASE

2.CERVICAL DISC PROLAPS

20. 54 YEARS OLD MAN WITH SUDDEND ON SET LEFT SIDE WEEKNESS WITHOUT
DISPHASIA. POSSIBLE DIAGNOSIS.

A. RIGHT MID C ARTERY


B. CORATIC ARTERRY OCLUSIOON

C. EPILEPSY

D. INTERNAL CAPSULE

21. 30 YEARS OLD LADY PRESENTING WITH CHEEK PAIN AND SENSORY LOSS, DX?

A. TRIGERMINAL NEURALGIA

B. MULTIPLE SCLEROSIS

22. G6PD DEFICIENCY, CAUSES EXCEPT

A. MOTH BALL

B. AMPICILLIN

C. BROAD BEAN

D. SULFERNAMIDE

E. FIORANTURIN

23. KEROTOAKATHOSIS, CORRECT

IT USUALLY GROWTH QUICKLY AND SPONTANEUS RESOLVE

24. A HYPERTENSION PATIENT WITH SUDDEN ONSET PAIN BEHIND LEFT EYE, WITH
PTOSIS, PALSY SIGNS OF VII, V, VI, (PARALYSIS OF PALATE, EYE MOVEMENT PARALYSS),
DX?

A. ANEURYSM OF POSTERIOR COMMUNICATION WILLIAM RING

B. MIDBRAIN INFARCT

C. BRAIN STEM INFARCTMENT


25. INDICATION FOR THROMBOLYTIC THERAPY ?

A. NEW LBBB

B. NEW RBBB

C. Q WAVES

D. T INVERSION

E. ST DEPRESSION

26. A MEN PRESENTING WITH CHEST PAIN MIMIC ISCHIMIC CHEST PAIN WITH
PROLONGED PERIOD, WHICH IS CORRECT?/

A. HEPARIN AND IV NITRATE ARE NOT INDICATED

B. THROMBOLYTIC TX CAN NOT GIVEN IF ECG IS NORMAL

27. A YOUNG MAN WITH COUGH, MANTOUX +, CXR SHOWED INACTICVE TB LESION,
MX?

A. TRIPLE TREATMENT FOR 6 MONTHS AND REPEAT CXR

B. ISONIAZID TX FRO 3 MONS AND REPEAT CXR

C. 6 SPUTUM AND AWAIT

D. REPEAT CXR IN 3 MONTHS

28. A LADY WITH DIARRHOEA, ANAEMIA, FATIGUE, MCV 110

A. PERNICIOUS ANAEMIA

B. GLUTEN-SENSITIVE ENTEROTOMY

C. CROHN’S DISEASE

29. BLOOD GAS PH 7.45 (N 7.35-7.44), PCO2 27, PO2 65, HCO3 23,
A. RESPIRATORY ALKALOSIS WITH GAS CHANGE IMPAIRMENT

B. RESPIRATORY ALKALOSIS WITHOUT GAS CHANGE IMPAIRMENT

30. COPD WHICH IS THE RISK OF RIGHT HEART FAILURE

A. FEV <1 L

B. DIFFUSING CAPACITY <50%

C. REDUCED PO2

D. INCREASED PCO2

31. COPD PATIENT WAS BROUGHT IN BY AMBULANCE. WAS GIVEN 10 L O2 ON THE


WAY. ABG PCO2 65 PO2 60 AND HCO3 23.

A. REDUCE O2 AND REDO THE ABG 30 MINS LATER

32.INFECTIVE ENDOCARDITIS ASSSOCIATED WITH

A. VASCULITIS IS THE PRESENTATION OF ENDOCARDITIS

B. STAPHAL IS THE COMMON CAUSE

C. IT IS THE DIRECT FROM RHEUMATIC FEVER

33.THE PATIENT PRESENTS IN THE EMERGENCY DEPARTMENT WITH BP-90/50

PULSE RATE 98 MIN. CVP IS 0.5 WATER. WHAT IS THE POSSIBLE CAUSE?

1.CARDIAC ARREST

2.HYPOVOLUEMIC SHOCK

3.BOWEL OBSTRUCTION

4.PANCREATITIS
34.THE PATIENT PRESENTS WITH THE SEVERE PAIN WHICH IS RADIATING TO THE BACK,
BP-180/95. WHAT IS THE FEATURE YOU ARE GOING TO FIND ON THE CXR:

1.WIDENED MEDIASTINUM

35.THE ABATTOIR WORKER IS PRESENTED WITH JAUNDICE, FEVER, AND MALAISE.

WHAT IS THE POSSIBLE DS:

1.LEPTOSPEROSIS

2.BRUCELLOSIS

3.MALARIA

4.DENGUE FEVER

36.CHRONIC DIARRHOEA FOR 3 MONTHS COULD BE CAUSED BY EXCEPT:

1.CAMPYLOBACTER JEJUNI

2.CROHN’S DISEASE

3.ULCERATIVE COLITIS

4.GIARDIASIS

5. LAXATIVE ABUSE

37.MOST COMMON CAUSE PERFUSE BLEEDING IN ELDERLY:

1.DIVERTICULOSIS

2.CANCER OF THE COLON

3.CROHN DISEASE

4.ULCERATIVE COLITIS

5. ISCHEMIC COLITIS
38 THE PATIENTS PRESENTS WITH OEDEMA OF THE FACE .MX: TREATMENT FOR
HYPERTANSION,GAAUT AND ALOPURINOL ,INDOMETACIN.THE MOST POSSIBLE CAUSE OF
HIS OEDEMA:

1.INDOMETCIN INTOXICATION(STOP THE TREATMENT)

2.RENAL FAILURE

39.IGA NEPHROPATHY

40. A LADY WITH OLIGIO AND CREPTITAION, POOR PROGNOSIS IS?

A. HAEMATURIA IS THE POOR PROGNOSIS

41. CAUTE AND CHRONIC RENAL FAILURE?

A. ULTRASOUND

42. INCREASE TRANSFERRIN SEEN IN:

A. HAEMOCHROMATOSIS

43. A PATIENT WITH COPD , GIVEN O2 10L/MIN, BECOME UNRESPONSIVE, WHICH OF F


IS POSSIBLE BLOOD GAS

A. PH7.15, PO2 65, PCO2 100

44. PATIENT WITH HEART FAILURE AND GOUT, TREATED WITH THIAZIDE DIUETICS, ACE
–OR, ALLOPERIDOL AND INDOMETHOCINE. TEST : K7, NA 1.25

A. NA LOW INDICATE NA DEPLETION


45. NA OF 122 INDICATES?

A. LOW PLASMA OSMOLORITY.

46. A LONG LIST OF BLOOD FILM

A. MULTIPLE MYOLOMA

47. INDICATION OF SPELETOMY

A. CHRONIC ITP

48. ASTHMA, ALL EXCEPT

A. TOTAL LUNG CAPACITY REDUCED.

49. INDICATION FOR POSTERIOR COLUMN LESION?

A. ROMBERG’S TEST POSITIVE

50. HYPOTHYRODISM, EXCEPT?

A. SMALL HAND MUSCLE ATROPHY

51. OPENING SNAP INDICATE?

A. MOBILITY

52. WHAT IS NOT CHARACTERISTIC OF AUTOSOME RECESSIVE


A. 50/50 CHANCE OF INHERIANCE

53. DM WITH AUTONEUROPATHY ALL EXCEPT

A. URINE INCONTINENCE

B. DIARRHOEA

C. BRADYCARDIA

54. MOST RELIABLE DX FOR GIARDIA

A. STOOL EXAM OF CYST

B. DUODENAL FLUID

55. COELEIC DIS DX TO CONFIRM?

A. DUODENAL BIOPSY

B. ANTIBODY-GLIADIN

C. ANTIBODY-ENDOMYIAL

56. INFECTIOUS MONONUCLEOSIS, ALL CORRECT EXCEPT,

A. IS USUALLY ASSOCIATED WITH HEPATITIS

B. DIAGNOSED BY ANTIGEN DETECTION

57. ANGIOPLASTY AND STENT COMPARING, WHICH IS CORRECT?

A. REDUCE RESTENOSIS BUT SAME COMPLICATIONS AS ANGIOPLASTY

B. MORE COMPLICATIONS THAN ANGIOPLASTY ALONE

C. NO DIFFERENCE
PAEDIATRICS

1. 18TH MONTH OLD BABY BOY WITH EXCESSIVE BREAST DEVELOPMENT AND NO
OTHER SECONDARY SEXUAL CHARACTERISTIC. CHILD HAS SAME CHRONOLOGICAL AGE
WITH HER BONY AGE. WHICH OF THE FOLLOWING WILL BE CORRECT:

A. PRECOCIOUS PUBERTY

B. CAH

C. NORMAL DEVELOPMENT

D. IDIOPATHIC PRECOCIOUS THELARCHE

2. 8 MONTH OLD CHILD WITH PAROXYSMAL COLICKY ABDOMINAL PAIN WITH LEGS
FLEXED FOR SEVERAL WEEKS. BABY WAS VOMITING GREENISH STUFF OCCASIONALLY. IN-
BETWEEN HE WAS OK. ON EXAMINATION NAD. WHAT WOULD YOU DO?

1.ULTRASOUND

2.

3. REASSURANCE

4. BARIUM MEAL WITH FOLLOW THROUGH.

5. CT WITH CONTRAST

3. 24 MONTH OLD BABY CAN DO

1.BUILD 2-3 BLOCKS

2. CAN BUTTON

4. COUNT 1-5
5. POINT TO A COLOUR

6. DRAW A PICTURE WITH 6 PARTS

4. A CHILD CAN SIT WITH HIS HAND SUPPORTED IN THE FRONT. HE CAN GRAB THINGS
WITH HIS PALM. HE CAN TURN PRONE TO SUPINE AND SMILE TO MIRROR. WHAT HIS
AGE?

1.10 WEEKS MONTHS

2. 3MONTHS

3. 7 MONTHS

4. 9 MONTHS

5. 11 MONTHS

5. A CHILD 7 YEARS OLD HAS A HEADACHE AND VOMITING FOR 6 WEEKS IN THE
MORNING. CONTACT H/O CHICKEN POX IN SCHOOL. HE HAD ATAXIA,

A. MIGRAINE

B. POSTERIOR FOSSA TUMOUR

C. VARICELLA CEREBELLITIS

6.CHILD PRESENTED WITH DYSPNOEA, ON EXAMIN. THE APEX BEAT ON MID-AXILLARY


LINE, MURMUR HEART BEST ON LEFT STERNAL BORDER AND CAN BE HEARD ALL OVER
THE PRECORDIUM..

A. VSD

B. ASD

C. COARCTATION OF AORTA
7. MOTHER WITH A CHILD SUFFERING FROM PHENYLKETONURIA, WHAT ARE THE
CHANCES THE HER 2ND CHILD WILL SUFFER FROM THE DISEASE

A HALF OF THE CHILDREN

B ONE IN FOUR CHILDREN

C ALL OF THEM

D. ONE FEMALE CHILD AND ONE MALE CHILD

8 A CHILD HAS FEVER AND URINE COLLECTED IN BAG FOR CULTURE SHOWED MIXED
GROWTH COUNT > 100000. WHAT DO YOU DO NEXT?

A. GIVE ANTIBIOTICS

B. MICTURATING URETHROGRAM

C. SUPRA PUBIC PUNCTURE

9 RECURRENT STICKY EYES IN A CHILD WHICH IMPROVES WITH ANTIBIOTIC FOR 2 DAYS
BUT RECURS AGAIN. WHAT IS THE CORRECT……………

A. RECURRENT NASAL INFECTION

B. LACRIMAL DUCT BLOCK

C. GIVE ANTIBIOTIC FOR 10 DAYS

10. 15 YR OLD BOY GOT ABRASION IN HIS KNEE. HE HAS BEEN IMMUNISED AT 5 YEARS
OLD AND NO IMMUNIZATION AFTERWARDS. WHAT IS YOUR MX?

A. TOXOID

B. ADT

C. IMMUNOGLOBULIN AND TOXOID

D. DO NOTHING

E. DPT
11. A CHILD WITH DEVELOPMENTAL DELAY. THE BABY WAS BORN AT 38 WEEKS WITH
3800G. WHAT IS THE LIKELY EXPLANATION?

A. FATHER ALCOHOLIC

B. MATERNAL UNCLE INTELLECTUAL DISABILITY

C. GRANDFATHER WITH

D. SISTER WITH FIBRIL CONVULSION

12. A CHILD WITH ECZEMA, WHICH IS CORRECT?

A. IT IS USUALLY ASSOCIATED WITH DRY SKIN

B. USUALLY IN THE EXTENSION SIDE OF JOINS.

13.A CHILD HAS 20 BROWN MACULOUS SPOTS ALL OVER THE BODY. HIS
GRANDMOTHER HAS SKIN NEUROMA. ALL RIGHT EXCEPT

A.

B. MORE HYPERTENSION

C. MORE DEAFNESS

D. HYPOTHYRODISM

E. INTELLECTUAL RETARDATION

14. A 3 WEEKS BABY BOY WITH INTERMITTENT PROJECTING VOMITING, ELECTROCYTE


FINDING IS: K7.7, NA 135, CL 89, HCO3 18. CHILD IS DEHYDRATED. WHAT IS THE DX?

A. ADRENAL DEFICIENCY HYPERTROPHY

B. PYLORIC STENOSIS
15. A BABY FROM GREEK FAMILY WITH JAUNDICE AT DAY 5. BLOOD BILIRUBIN IS 220
UM/L, UNCONJUGATED IS 200 UM, DX

A. BREAST FEEDING JAUNDICE

B. PHYSIOLOGICAL JAUNDICE

C. THALASSAEMIA MAJOR

16. A BABY BOY 13 YEARS OLD WITH HEIGHT PERCENTILE 3%(EQUVALENT TO 50%
PEARCENTILE AT 9 YEARS), WOF GIVE BETTER FUTURE GROWTH PATTERN

A. BONE AGE 9

B. BONE AGE 13

17. A CHILD SWALLOWED WITH SODA. CHILD PRESENT WITH DROOLING, IMMEDIATE
ACTION,

A. EARLY ENDOSCOPY

B. DRINK MILK

18. A 3 YEAR BOY WITH ABDOMINAL MASS WHICH TAKES MOST SPACE OF RIGHT
ABDOMINAL SPACE, THE USEFUL DIAGNOSTIC METHODS ARE EXCEPT:

A. URINE VMA

B. ULTRASOUND

C. CXR

D. BARIUM SWALLOW

E. ENDOSCOPY

19. A 4 YEAR OLD WITH ASTHMA, WHAT IS THE MOST CRITICAL INDICATOR OF BEDSIDE
ASSESSMENT OF SEVERITY OF ASTHMA?
A. PEAK FLOW

B. SURRICIC SULCI

C. INTRACITON OF SUBSTERNAL

20. A BABY 12 MONTHS PALLOR WITH BREAST FEEDING AND INDUCE SOLID FOOD
LATER, THE MOST LIKELY CAUSE OF ANAEMIA

A. IRON-DEFICIENCY ANAEMIA

21. A CHILD PRESENDT WITH A LUMP IN THE POSTERIOR LOWER NECK WHCIS IS
PUNCTUATE AND SOFT, DX?

A. CYSTIC HYGROMA

22. A CHILD WITH WHEEZE AND COUGH, PE REVEALED UNILATERAL WHEEZE, MX?

A. INSPIRATION AND EXPIRATION OF CXR

23. A CHILD PRESENT WITH NECK STIFFNESS, FEVER, PROTEIN 1.1G/L, GLUCOSE 2.5 (N2-
5), LYMPHOCYTE 100, NEUTROPHIL 50, DX?

A. TB

B. ECHOVIRUS

C. VIRUS

D. STAPHYLOCOCCUS AUREUS

24.FEVER SUBSIDES RASH AFTER 3 DAYS

A.ROSEOLA

B.MUMPS
C.SCARLET FEVER.

D.MEASLES.

PSYCHIATRY

1. A MAN THREE SALIVATING EVERYDAY TIME BEFORE HIS MEDICATION

A. MANNERISM

B. OBSESSIVE COMPULSIVE DISEASE

2. A YOUNG MAN DRESSES AS FEMALE CAME WITH ANOTHER MAN, WI WANTS TO BE A


FEMALE, TAKING FEMALE HORMONE FOR ONE YEAR

A. TRANSVESTISM

B. TRANSSEXUALISM

C. SEXUAL DYSFUNCTION

3 WHICH ANTIPSYCHOTIC DRUG IS RESTRICTED IN AUSTRALIA BECAUSE IT HAS VERY


LONG Q-T INTERVAL

A. CHLORPROMAZINE

B. THIORIDAZINE

C. RISPERIDONE

4. ONE OF THE FOLLOWING IS A IMMATURE DEFENSE MECHANISM

A. RESOLUTION
B. PROJECTION

C. HUMOUR

5. A CHILD STEALING PENCIL FOR THE FIRST TIME FROM HIS FRIEND

A. REASSURE MOTHER BECAUSE NOTHING WRONG WAIT AND WATCH POLICY

B. TAKE HIM TO POLICE

C. REFER TO A PSYCHIATRIST

6 THE PROVERB ‘COGNITIVE TESTING AND MENTAL TEST FOR:

A. DELUSIONAL

B. DEMENTIA

7 A LADY CAME WITH LOWER LIMB PARALYSIS IN A WHEEL CHAIR

A. REFER HER TO PSYCHIATRIST

B. REFER HER TO PHYSIOTHERAPIST

C. DO PSYCHOANALYSIS TO UNDERSTAND THE INNER CONFLICT WITH


PSYCHOMECHANICS

8. A 32 YEAR SCHOOL TEACHER WITH OCD, DRUG OF CHOICE

A. SSRI

B. ANTIPSYCHOTICS

9. IN GP PRACTICE WHAT IS NOT TRUE;

A. APPROX 30% PATIENT HAVE SOME KIND OF PSYCHIATRY PROBLEM

B. A MINOR NUMBER ARE REFERRED TO A PSYCHIATRIST


C. PSYCHOSIS IS THE COMMONEST TYPE OF PSYCHIATRY PROBLEM

D. DRUG AND ALCOHOL ABUSE ARE NOT TAKEN INTO CONSIDERATION

10 A LADY WITH MARRIAGE PROBLEM AND HUSBAND PROBLEM, NEVER GONE FOR
HOLIDAYS

A. SUFFERING FROM DEPRESSION

B. ADVISE THEM TO GO FOR HOLIDAY

C. REFER THEM TO MARRIAGE COUNCILLOR

11. A SCHIZOPHRENIA THE PRE-MORBID CONDITION WILL BE

A. IDEA OF REFERENCE

B. UNDER LYING FAMILY HISTORY PROBLEM

12. WHO IS ABUSE MOST TRUE

A. AN ELDERLY FEMALE

B. AN ELDERLY MALE

C. ABUSER USUALLY IS NOT A CLOSE RELATIVE

13 HYPERPROLACTINAEMIA WILL NOT CAUSE ONE OF THE FOLLOWING,

A. SEXUAL DYSFUNCTION

B. ERECTILE PROBLEMS

C. GALLACTORHEA

D. LOSS OF LIBIDO

14. ABOUT ANTIDEPRESSANT WHAT IS TRUE


A. WITHDRAWAL SLOWLY

B. GIVE ALTERNATE DAY

C. HAS TO BE ON ANTIDEPRESSANT IN LOWER DOSE OR SOME PERIOD THROUGHOUT


HIS LIFETIME

15. TOURETTE SYNDROME

A. TICS BEGINS

B. STARTS ALWAYS BELOW 18 YEARS

16. THE BORDERLINE PERSONALITY WILL HAVE

A. MOOD CHANGES

17. CYTOCHROME P450

A BENEFITS TO MEASURE ACTION OF BENZODIAZEPAM

18. ALCOHOL WITHDRAWAL HALLUCINATION, DRUG OF CHOICE

A. DIAZEPAM

B. HALOPERIDOL

C. ANTIDEPRESSANT AND ANTIPSYCHOTIC

19. A MAN WITH ACCIDENT HIT ON THE CHEST AND LOWER LIMB, NO LOSS OF
CONCIOUS NESS, ADMITTED TO HOSPITAL, AFTER THREE DAYS DEVELOP ACUTE
CONFUSIONAL STATE, DUE TO:

A. SUBDURAL HAEMATOMA

B. ALCOHOL WITHDRAWAL
C. NARCOTIC REACTIONS

20. NALTREXONE IS USED FOR OPIOID TOXICITY IT HAS/IS

A SHORT ACTION

B EFFECTIVE ORALLY

OBSTETRICS & GYNAECOLOGY

1. BARTHOLIN’S CYST

1. MOST COMMONEST CAUSE IS STAPHYLOCOCCUS AURUS

2. MAY BE DUE TO GONOCOCCUS

3. USUALLY TREATED BY ANTIBIOTICS

4. USUALLY SPONTANEOUS RECOVERY

2. G3P0 WITH 2 PREVIOUS 2ND TRIMESTER MISCARRIAGE WITHOUT ANY


CONTRACTION AND SIGN OF UTERUS PRODUCT. CURRENTLY 16 WEEKS PREGNANCY AND
VERY CONCERNED ABOUT THIS PREGNANCY. EXCETP REASSURANCE WHAT IS YOUR
MANAGEMENT?

1.SULBUTAMAL TREATMENT

2.CERVICAL STITCH

3. DURING UTERUS CONTRACTION, PLACENTAL BLOOD SUPPLY.

1.REDUCED

2. CEASED
3.INCREASED

4. UNCHANGED

4. 54YEARS OLD LADY WITH 3 CHILDREN AND 3 YEAR POSTMENAPAUL WITHOUT


HOTFLUSH AND OSTEOPOROSIS. SHE IS MODERATELY OBESITY. PE ALL NORMAL. WHY SHE
HAS NOT POST MENOPAUSAL SYMPTOMS

1. CONVERSION OF ANDROSTEOTERONE TO ESTRONE

2. CONVERSION OF ADHD TO ESTRODIAL

5. RISK OF A 38 YEARS LADY HAVING A BABY WITH DOWN SYNDROME.

1. 1:50

2. 1:100

3. 1:200

4. 1:500

5. 1:1000

6. G1P0 39WEEKS NORMAL ANTENATAL PROCEDURE. COME TO WARD COTRACTION AT


1AM START CONTRACTION EVERY 4-5 MINS. CTG NORMAL. CERVIX 2 CM DILATED AND
COMPLETED EFFACEMENT. LOA PRESENTATION. IN WHICH SITUATION OBSTRIC
CONSULTATION OPINION NEEDED?

1. 5AM CERVIS FULLY DILATED, MEMBRANE BORKEN, CONTACTION 5MIN STILL 2CM
ABOVE IPISINE SPINE

2. 5AM CERVIX 3 CM,MEMBRANE NOT RUPTURNED, CONTRACTION BECOME WEAK


AND IRREGULAR.

7. G1P0 14 HOUR AFTER STARTING LABOR, CTG NORMAL INITIALLY. PETHIDINE WAS
GIVEN(3 DOSED TOTAL300 MG). CONTRACTION OK AND AFTER 30 MINS, CTG SHOWED
BASE LINE HR IS 90 MIN WITH REDUCED VARIABILITY AND MODERATELY VARIABLE
DECELERATION. WHAT THIS CTG INDICATE?

1. PETHIDINE DEPRESSED FETUS

2. UTERUS TONE INCREASED BETWEEN CONTRACTION

3. IT INDICATES THAT PROLONGED LABOUR OCCUR

8. MULLIRENE DEGENERATION EXCEPT

1. X-LINKED RECESSIVE

2. USUALLY WITH VIGINA, THOUGH SHORT.

3. NORMAL BREAST DEVELOPMENT

9. A 32 YS OLD LADY WITH MASTALGIA, CONSERVATIVE TREATMENT FAILED, WHAT IS


YOUR NEXT MANAGEMENT.

1. DANAZOL

2. PROMECRIPTINE

3. CLIMIPROTIN

4. PROGESTRERONE

10. A LADY WITH DEPRESSION, BREAST TENDERNESS AND LOW MOOD AND SHE
BELIVED SHE HAS PREMENSTRUL SYNDROM. WHAT THIS THE CONFIRMING DIAGNOSIS
METHOD?

1. CHART RECORDING FOR 3 MONTHS

2. LOW ESTRODIOL

3. SLIGHTLY INCREASED PROLACTION

4. INCREASED PROGESTERONE
11. OCP TAKING OVER 10 YEARS, ALL ARE BENEFICIAL EXCEPT

1. REDUCED BREASED CANCER

2. REDUCED BENIGNE BREASET DISEASES

3. RESUCED PID

4. REDUCED IRON-DEFICIENCY ANEMIA

12. A 38 WEEKS PREGANCY LADY WITH ABDOMINAL PAIN. BLOOD LOSS ASSESSED
AROUND 1000ML. SUSPECTED PLACETA ABRUPTION. ALL COLLECT EXPECT

1. PATIENT WITH PAIN AND BLEEDING

2. BP 180/110

3. FETUS HEAD IS HIGH ABOVE THA IPSITUS SAPINE.

13. A 28 YEAR OLD WITH HIRSUTISM AND IRREGULAR PERIOD. 17-HYDROXYESTROL


NORMAL AND ADHD SLIGHTLY INCREASED. RATION OF LH/FSH INCREASED. WHAT IS THE
DX

1. PCO

14. A LADY IS GOING TO DO LAPAROSCOPY . DURING DISCUSSION ABOUT THE RISK OF


ENDOSCOPY, WHAT ADVICE WILL YOU GIVE EXCEPT?

1. BECAUSE YOU CAN SEE THE SURFACE AND DO BIOPSY , THE LAPAROSCOPY IS THE
MOST EFFICIENT DIAGNOSTIC METHOD FOR POC

2. BECAUSE CO2 CAN NOT BE FULLY REMOVED, SO THE ABD PAIN AND SHOUDER TIP
PAIN IS COMMON.

3. ALTHOUGH IT DOES NOT DEMAGE CERASAL, IT MAY CAUSE RUPTURE OF OTHER


ORGANS

4. IT IS THE DX TOOL FOR ENDOMETRIOSIS


15. HYPERPROLACTINEMIA, EXCEPT(PSY)

1. INCREASED LIBIDO

2. PREMATURE EJACULATION

3. AMENORRHEA

16. VAGINAL DISCHARGE, EXCEPT

1. IN EARLY TEENAGER GIRL, THE MOST COMMON CAUSE OF VAGINITIS IS DUE TO


CANDIDA INFECTION.

2. POSTMENOPAUSAL WOMEN WITH PROGESTERONE ONLY HRT HAVE MORE CANDIDA


INFECTION THAN THOSE WITH COMBINED HRT.

3. 4 YEARS OLD, THE MOST COMMON CAUSE IS ATROPHY VAGINITIS.

17. A LADY WITH NORMAL PAP SMEAR FOR LAST 2 YEARS. CURRENT PAP SMEAR
SHOWED SQUAMOUS CELL HYPERPLASIA. THIS IS RELEVANT BECAUSE IT IS

1. IT IS NORMAL WITH NO SIGNIFICANCE

2. IT IS DUE TO WART INFECTION AND NEED OBSERVED CAREFULLY

3. IT IS INVASIVE CERVICAL CANCER

4. THE CELLS IS SEEN IN PRE-INVASIVE CERVICAL CANCER.

18. ASTHMA EXCEPT

1. IT USUALLY CAUSES MORE BRONCHSPAMS

2. CAN NOT BE TREATED WITH SALBUTAMOL

3. CAN NOT BE TREATED WITH CORTICOSTERIODS

19. A LADY WITH 12 WEEKS AMENORRHOEA, VAGINAL EXAM FOUND THAT UTERUS IS
EQUIVALENT TO 15 WEEKS. THE MOST LIKELY CAUSE IS
1. SHE REMEMER WRONG DATE

2. UTERUS FIBROID

3. HYDATIFORM MOLE

20. SPERM 1000,000, 20% MOTILITY, 90% ABNORMAL MORPHALOGYINFERTILITY,


WHAT IS CORRECT?

1. SPONTANEOUS PREGNANCY IS UNLIKELY

2. SPONTANEOUS PREGNANCY IS IMPOSSIBLE

21. WHICH OF FOLLOWING INDICATE THE OVARY TUMOUR RATHER THAN ASCITES

1. DULL ANTERIORLY AND TYMPANY LATERALLY

2. DULL LATERALLY AND TYMPANY ANTERIORLY

3. SHIFTING DULLNESS

22. TUMOUR MAKERS FOR GYNAECOLOGICAL TUMOURS, EXCEPT

1. BCA1

2. CA125

3. AFP

4. B-HCG

5. INHIBIN

23. A LADY 54 YEARS OLD WITH 3 CHILDREN URGENCY INCONTINENCE

1. SHE CAN PASS URINE IMMEDIATELY AFTER MICTURITION

2.BONINE (?) TEST (VAGINAL DIGITAL BILATERAL UTERAL ) NEGATIVE


3.SHE CAN LOSS LARGE AMOUNT OF URINE

4. SHE LOSS URINE WHILE RUNNING OR JUMPING

24. ASYMPTOMATIC URINE INFECTION IN PREGNANCY, WHICH IS THE CAUSE OF


PREGNANT WOMEN PRONE TO UTI

1. DILATION OF UTERAL AND CALYCES DUE TO PROGESTERONE

2. PRESSURE OF ENLARGED UTERUS

25. WOF CORRECT EXCEPT?

A. FULL BREAST FEEDING WITH PREGNANCY RATE LESS THAN 2% IF AMENORRHEA FOR
6 MONTHS

B. THE INITIAL MENORRHEA USUALLY IS ANOVULATION

C. DEPOPROVERA MAY INCREASE MILK PRODUCTION

D. BROMOCRIPTINE MAY DELAY OVULATION

SURGERY

1. MAMMOGRAPHY

A. BETTER FOR PICKING UP EARLIER CANCER

B. PAINLESS

2. PICTURE OF A NETBALL PLAYER ANUS WITH BLUISH SWOLLEN. MX

A. HAMORECTOMY UNDER ANTHESIS

B. INCISION UNDER LOCAL ANAESTHETIC


C. HOT PATH

3. A OLD MAN WITH TWO LUMPS IN GROIN, DX?

A. METASTASIS OF LYMPH NODE

B. FALSE ANEURYSM

C. SAPHRIX VARIX

4 PICTURE OF A MAN’S TONGUE, RIGHT SIDE SMALLER THAN RIGHT SIDE, WHICH
CORRECT?

A. LESION IN RIGHT SIDE AND POINT TO R IF PROTRUDE TONGUE

B. LESION IN LIGHT SIDE AND POINT TO L IF PROTRUDE TONGUE

5 PICTURE OF A EJECTED PENIS

A. PERSISTENT PRIAPISM

B. PARAPHIMOSIS

6 PICTURE OF A BOBY’S CXY WITH RESPERITORY STRESS SYMPTOMS, DX?

A. DIAPHRAGM HERNIA

B. STAPHAL AUREUS INFECTION

7 PALM PENETRATED BY A SCREW NAIL. DAY 4 WITH DEEP ABSCESS AND DORSUM
SWOLLEN, WOF CORRECT?

A. IT IS A SIGN OF EXTERNAL TENDON INVOLVED

B. INCISION AND DRAINAGE OF DORSUM SIDE


8 A PATIENT AFTER TRAUMA WITH DYSPENEA, PE, L BREATH SOUND REDUCED,
TRACHEA DIVATED TO R.SUBCUTANEOUS EMPHSEMA BP 100/70, P120, DX?

A. TENSION PNEUMOTHORAX

B. HAEMOPNEUMOTHORAX

C. BRONCHAL RUPTURE

9 ICU PATIENT WITH LOW BP, HR 110, CVP 0.5, DX?

A. HYPOVOLUMIC SHOCK

10 NECK VEIN DESTAIN AND SHOCK

A. CARDIA TAMPONADE

11 MOST COMMEN SITE OF OCCLUSION FOR INTERMITENT CLAUDICATION?

A. SUPERFACIAL FEMORAL A

B. ABD AORTA BIFURCATION

12. MASSIVE BLOOD T/F, MOST SIGNIFICANT OUTCOME?

A. COAGULATION DISORDERS

13.WOF MORE LIKELY CAUSE ARTERIAL THROBOSIS?

A. HIP JOINT DISLOCATION

B. FEMOR SHIFT FRACTURE

C. KNEE FRACTION AND DISLOCATION

14. STYE?
A. INFECTION OF A GREAT MEBOBIAN GLAND

B. EYELISH FOLLICAL INFECTION

C. PUNCTITIS

15. SPLEENECTOMY, CORRECT EXCEPT?

A. PERSISTENT ANAEMIA

B. FRAGILIGH REMAINS ABNORMAL

16. A LADY WITH CONFUSION 3 DAYS AFTER RIGHT HEMICOLONECTOMY SUEGERY,


WOF CORRECT?

A. ELECTROLYTE DISTURBANCE

B. ATELECTASIS

17. HEPARIN REVERSED BY?

A. PROTAMINE SULFATE

18.UNILATER NASAL DISCHARGE, DX?

A. FOREIGN BODY

19.DUPUYTREN’S CONTRACTION, WOF CORRECT?

A. A PALM APPONEUSON

20.A YOUNG MAN FALLED WITH OUTSTRETCHED HAND, X-RAY NIL, WOF CORRECT?

A. SCAPHOID FRACTURE
21.ANKLE JERK ABSENT, LESION IS IN WHICH ROOT?

A. L2

B. L3

C. L4

D. L5

E. S1

22.CALF PAIN 2 DAYS AFTER SURGERY, IX?

A. DOPPLER

B. ANGIOGRAPHY

C. VENOGRAPHY

23. WOF IS MOST SIGNIFICANT TO PAY ATTENTION TO FOLLOWING INJURY?

A. PALLER OF DISTAL EXTREMITIES.

24.TO DETECT DX COMPARTMENT SYNDROME?

A. DORSUM REFLEXION OF TOE

25.A MAN COLLAPSED PLAYING SQUASH CLUTCHING HIS ANKEL. HE CAN NOT
DOPLANTAR REFLEXION BUT CAN DO DORSUM FLEXION, DX?

A. ACHILLES TENSION FRACTURE COMPLETELY.

26.A LUMP IN SUBCLAVICULAR FOSSA, ALL COMMON POSSIBLE EXCEPT?

A. BREAST CANCER METASTASIS

B. LUNG
C. SUBCLAVICULAR ANEURSM

D. SUBCLAVICULAR VEIN THROBOSIS

27.COMPOUND FRACTURE, MOST EFFICIENT TO PREVENT INFECTION?

A. A PROLONGED BROAD SPECTRUM ANTIBIOTICS

B. FULLY SURGICAL DEBRIDMENT AND DRAINAGE

28.WOF MOST LIKELY CAUSE CANCER?

A. JEVENILE POLYPS

B. JESOPHUS-SYNDROM

C. GARDNER SYNDROM

D. COLONIC METAPLASIA

29.A 75 LADY WITH SUDDEN ONSET DIFFUSE ABD PAIN, PE, ABD DISTENSION, PULSE
IRREGULAR. PASSED DARK BLOOD, DX?

A. MENSENTERIC VASCULAR THROMBOSI

30.ALL CAUSE CANCER EXCEPT

A. DIVERTICULOSIS

31.UNDESENDED TESTES MOST COMMENLY ASSOCIATED WITH?

A. INGUINAL HERNIA

32.MOST COMMON PRESENTATION OF TESTES CANCER?

A. SWOLLING
33.SEMINOMA, WOF CORRECT?

A. YOUNG MAN, DISTANT SPREAD AND RADIOSENSITIVE

34.WOF NOT FOUND TO CAUSE SCC CANCER?

A. APHROUS ORAL ULCER

35WOF INDICATES CANCER?

A. RECURRENT LARYNGEAL NERVE PALSY

B. RETROSTERNAL COMPRESSION

C. PERBONTON SIGN +

36. MOST COMMONEST CAUSE OF BLINDNESS WORLDWIDE?

A. CATARACT

B. TRACHOMA

37. WOF NEED TO STOP ANTICOAGULATION FOR PATIENT WITH BEFORE OPERATION
TEMPORATELY,

A. ABD NEEDLE ASPIRATION

B. EMBOLECTOMY

38.NO ANTIBIOTICS NEEDED ?

FUNDALPLICATION OF OESPHAGEAL

39. HYPOPARATHAYRADIAM FOLLOWING THROIDECTOMY, ALL EXCEPT?


A. IT IS DIFFICAUTY TO TREAT BY VIT D BECAUSE THE THERAPEUTIC WINDOW IS
NARROW.

C. IT DECREASE CA AND INCREASE PTH

40. WOF CAN HELP TO INDICATE THE POSSIBILITY OF CONSERVATIVE SURGERY FOR
BREAST CANCER?

A. MAMMOGRAPHY

B. AXILLARY LYMPH NODES

41. WOF ABOUT HAEMORRHOID ALL CORRECT EXCEPT

A. SEVERE PAIN

B. PRURITIS

C. BLEEDING

42. 15 YEAR GIRL WITH RIGHT HAND ERYTHEMA (HEPATITIS)

A. PERIDUCT TRACT LYMPHOCYTE AND PLASMA CELL INFILTRATION.

43. ACUTE CHOLECYSTITIS CAUSES?

A. STONE IN THE GALL BLADDER NECK

44. WOF CAUSE HYPERCALCINEMIA EXCEPT?

A. PANCREATIS

45. VARICOCELE

A. DILATION OF VEINS OF PSERMATIC CORD


B. LYING DOWN DISAPPEAR

46. WALKING INTO A WARD AND SEE A PATIENT CHOKING A LUMP OF MEAT, MX?

A. UPPER ABD PRESSURE

B. INTUBATION

47. CYSTIC ADENOMA

A. USUALLY SEEN IN 20-30 YEARS

B. BIOPSY SEEN IN THE SAME LOBE OF GLAND

C. OPEN BIOPSY IS THE MANAGEMENT

48. THE 1ST STEP IN MANAGEMENT OF BREAST LUMP

A. NEEDLE ASPIRATION

49. HEPATIC TRIAD

A. ANTERIOR OF BILE TRACT AND POSTERIOR OF PORT VEIN

GUEST, MAR 28, 2006

#1

GUEST

GUEST GUEST

I WISH SOMEONE WOULD PUT THIS UP AS A STICKY.. I NEARLY MISSED ALL THESE
QUESTIONS
GUEST, APR 7, 2006

#2

BRONZE

BRONZE GUEST

PLEASE HELP ME WITH THESE MCQS

HI WOULD U PLEASE CORRECT MY ANSWERS

THESE ARE PEDEATRIC QUESTIONS

1. 18TH MONTH OLD BABY BOY WITH EXCESSIVE BREAST DEVELOPMENT AND NO
OTHER SECONDARY SEXUAL CHARACTERISTIC. CHILD HAS SAME CHRONOLOGICAL AGE
WITH HER BONY AGE. WHICH OF THE FOLLOWING WILL BE CORRECT:

A. PRECOCIOUS PUBERTY

B. CAH

C. NORMAL DEVELOPMENT

D. IDIOPATHIC PRECOCIOUS THELARCHE **

IS HE A BOY OR A GIRL CAUSE IN THE 3RD LINE THE QUESTION SAYS HER BONE AGE

2. 8 MONTH OLD CHILD WITH PAROXYSMAL COLICKY ABDOMINAL PAIN WITH LEGS
FLEXED FOR SEVERAL WEEKS. BABY WAS VOMITING GREENISH STUFF OCCASIONALLY. IN-
BETWEEN HE WAS OK. ON EXAMINATION NAD. WHAT WOULD YOU DO?

1.ULTRASOUND

2.

3. REASSURANCE **?
4. BARIUM MEAL WITH FOLLOW THROUGH.

5. CT WITH CONTRAST

3. 24 MONTH OLD BABY CAN DO

1.BUILD 2-3 BLOCKS **

2. CAN BUTTON

4. COUNT 1-5

5. POINT TO A COLOUR **

6. DRAW A PICTURE WITH 6 PARTS

AT 24 MONTH BABIES CAN BUILD 4-5 BLOCKS TOWER

4. A CHILD CAN SIT WITH HIS HAND SUPPORTED IN THE FRONT. HE CAN GRAB THINGS
WITH HIS PALM. HE CAN TURN PRONE TO SUPINE AND SMILE TO MIRROR. WHAT HIS
AGE?

1.10 WEEKS MONTHS

2. 3MONTHS

3. 7 MONTHS **

4. 9 MONTHS

5. 11 MONTHS

5. A CHILD 7 YEARS OLD HAS A HEADACHE AND VOMITING FOR 6 WEEKS IN THE
MORNING. CONTACT H/O CHICKEN POX IN SCHOOL. HE HAD ATAXIA,

A. MIGRAINE

B. POSTERIOR FOSSA TUMOUR

C. VARICELLA CEREBELLITIS **
6.CHILD PRESENTED WITH DYSPNOEA, ON EXAMIN. THE APEX BEAT ON MID-AXILLARY
LINE, MURMUR HEART BEST ON LEFT STERNAL BORDER AND CAN BE HEARD ALL OVER
THE PRECORDIUM..

A. VSD **

B. ASD

C. COARCTATION OF AORTA

7. MOTHER WITH A CHILD SUFFERING FROM PHENYLKETONURIA, WHAT ARE THE


CHANCES THE HER 2ND CHILD WILL SUFFER FROM THE DISEASE

A HALF OF THE CHILDREN

B ONE IN FOUR CHILDREN **

C ALL OF THEM

D. ONE FEMALE CHILD AND ONE MALE CHILD

8 A CHILD HAS FEVER AND URINE COLLECTED IN BAG FOR CULTURE SHOWED MIXED
GROWTH COUNT > 100000. WHAT DO YOU DO NEXT?

A. GIVE ANTIBIOTICS

B. MICTURATING URETHROGRAM

C. SUPRA PUBIC PUNCTURE **?

9 RECURRENT STICKY EYES IN A CHILD WHICH IMPROVES WITH ANTIBIOTIC FOR 2 DAYS
BUT RECURS AGAIN. WHAT IS THE CORRECT……………

A. RECURRENT NASAL INFECTION

B. LACRIMAL DUCT BLOCK **

C. GIVE ANTIBIOTIC FOR 10 DAYS


10. 15 YR OLD BOY GOT ABRASION IN HIS KNEE. HE HAS BEEN IMMUNISED AT 5 YEARS
OLD AND NO IMMUNIZATION AFTERWARDS. WHAT IS YOUR MX?

A. TOXOID

B. ADT**

C. IMMUNOGLOBULIN AND TOXOID

D. DO NOTHING

E. DPT

11. A CHILD WITH DEVELOPMENTAL DELAY. THE BABY WAS BORN AT 38 WEEKS WITH
3800G. WHAT IS THE LIKELY EXPLANATION?

A. FATHER ALCOHOLIC

B. MATERNAL UNCLE INTELLECTUAL DISABILITY

C. GRANDFATHER WITH

D. SISTER WITH FIBRIL CONVULSION

I DID NOT UNDERSTAND THE QUESTION

12. A CHILD WITH ECZEMA, WHICH IS CORRECT?

A. IT IS USUALLY ASSOCIATED WITH DRY SKIN

B. USUALLY IN THE EXTENSION SIDE OF JOINS.

13.A CHILD HAS 20 BROWN MACULOUS SPOTS ALL OVER THE BODY. HIS
GRANDMOTHER HAS SKIN NEUROMA. ALL RIGHT EXCEPT

A.

B. MORE HYPERTENSION
C. MORE DEAFNESS

D. HYPOTHYRODISM

E. INTELLECTUAL RETARDATION

14. A 3 WEEKS BABY BOY WITH INTERMITTENT PROJECTING VOMITING, ELECTROCYTE


FINDING IS: K7.7, NA 135, CL 89, HCO3 18. CHILD IS DEHYDRATED. WHAT IS THE DX?

A. ADRENAL DEFICIENCY HYPERTROPHY **

B. PYLORIC STENOSIS

15. A BABY FROM GREEK FAMILY WITH JAUNDICE AT DAY 5. BLOOD BILIRUBIN IS 220
UM/L, UNCONJUGATED IS 200 UM, DX

A. BREAST FEEDING JAUNDICE

B. PHYSIOLOGICAL JAUNDICE

C. THALASSAEMIA MAJOR

16. A BABY BOY 13 YEARS OLD WITH HEIGHT PERCENTILE 3%(EQUVALENT TO 50%
PEARCENTILE AT 9 YEARS), WOF GIVE BETTER FUTURE GROWTH PATTERN

A. BONE AGE 9 **

B. BONE AGE 13

17. A CHILD SWALLOWED WITH SODA. CHILD PRESENT WITH DROOLING, IMMEDIATE
ACTION,

A. EARLY ENDOSCOPY **

B. DRINK MILK
18. A 3 YEAR BOY WITH ABDOMINAL MASS WHICH TAKES MOST SPACE OF RIGHT
ABDOMINAL SPACE, THE USEFUL DIAGNOSTIC METHODS ARE EXCEPT:

A. URINE VMA

B. ULTRASOUND

C. CXR **?

D. BARIUM SWALLOW

E. ENDOSCOPY

19. A 4 YEAR OLD WITH ASTHMA, WHAT IS THE MOST CRITICAL INDICATOR OF BEDSIDE
ASSESSMENT OF SEVERITY OF ASTHMA?

A. PEAK FLOW

B. SURRICIC SULCI

C. INTRACITON OF SUBSTERNAL

20. A BABY 12 MONTHS PALLOR WITH BREAST FEEDING AND INDUCE SOLID FOOD
LATER, THE MOST LIKELY CAUSE OF ANAEMIA

A. IRON-DEFICIENCY ANAEMIA

21. A CHILD PRESENDT WITH A LUMP IN THE POSTERIOR LOWER NECK WHCIS IS
PUNCTUATE AND SOFT, DX?

A. CYSTIC HYGROMA

22. A CHILD WITH WHEEZE AND COUGH, PE REVEALED UNILATERAL WHEEZE, MX?

A. INSPIRATION AND EXPIRATION OF CXR


23. A CHILD PRESENT WITH NECK STIFFNESS, FEVER, PROTEIN 1.1G/L, GLUCOSE 2.5 (N2-
5), LYMPHOCYTE 100, NEUTROPHIL 50, DX?

A. TB **?

B. ECHOVIRUS

C. VIRUS

D. STAPHYLOCOCCUS AUREUS

24.FEVER SUBSIDES RASH AFTER 3 DAYS

A.ROSEOLA **

B.MUMPS

C.SCARLET FEVER.

D.MEASLES.

You might also like